Printable Version of Topic
Click here to view this topic in its original format
თბილისის ფორუმი > ჯანმრთელობა და მედიცინა > რა არის ეს?


Posted by: Guardian 17 Nov 2008, 02:17
წინა თემა იხილეთ აქ -
http://forum.ge/?f=43&showtopic=33720157&st=0

Posted by: Guardian 17 Nov 2008, 02:22
In the urgent care clinic, you are evаluating a 47-
year-old woman with poorly controlled diabetes who has
a chief complaint of “sinusitis.” She does not have a history
of atopy. She first noticed a headache 2 days ago and
now feels very congested in her upper nasal passages. She
has hyperesthesia over her nasal bridge as well and is inquiring
about antibiotics to treat her infection. She has a
bloody nasal discharge with occasional black specks. On
examination, the sinuses are full and tender. She has a
temperature of 38.3°C. Oral examination shows a black
eschar on the roof of her mouth surrounded by discolored
hyperemic areas on the palate. What is the most appropriate
intervention at this time?

A. Ciprofloxacin and quarantine for possible anthrax
B. ENT consultation if no improvement with oral antibiotics
C. Immediate biopsy of the involved areas and amphotericin
D. Immediate biopsy of the lesion and voriconazole
E. Intranasal decongestants and close follow-up

http://imageshack.us

Posted by: basa-ttt 17 Nov 2008, 02:26
Guardian
საერთოდ რატომ დებთ ამ ფოტოებს?
თუ გგონიათ, რომ პრაქტიკოსს ექიმს
ეს რამეში გამოადგება?
პათოლოგანატომს კი ბატონო -
მაგრამ არც ერთი კლინიცისტი ამ პრეპარატებს ვერ გამოიყენებს თავის ცხოვრების მანძილზე.
ზრდილობისათვის თემას სათაური მაინც შეუცვალეთ -
და კლინიკურ შეთხვევას მაინც ნუ დაარქმევთ.
ან მართლა დაავადების კლინიკა მაინც აღწერეთ

Posted by: vano_t 17 Nov 2008, 04:52
Guardian
QUOTE
In the urgent care clinic, you are evаluating a 47-
year-old woman with poorly controlled diabetes who has
a chief complaint of “sinusitis.” She does not have a history
of atopy. She first noticed a headache 2 days ago and
now feels very congested in her upper nasal passages. She
has hyperesthesia over her nasal bridge as well and is inquiring
about antibiotics to treat her infection. She has a
bloody nasal discharge with occasional black specks. On
examination, the sinuses are full and tender. She has a
temperature of 38.3°C. Oral examination shows a black
eschar on the roof of her mouth surrounded by discolored
hyperemic areas on the palate. What is the most appropriate
intervention at this time?

A. Ciprofloxacin and quarantine for possible anthrax
B. ENT consultation if no improvement with oral antibiotics
C. Immediate biopsy of the involved areas and amphotericin
D. Immediate biopsy of the lesion and voriconazole
E. Intranasal decongestants and close follow-up

C. დიაბეტიკში (და იმუნოსუპრესიული მდგომერეობების დროს) გვხვდები მუკორმიკოზი-ძაან მაღალი სიკვდილიანობის სოკოვანი ინფექცია. თუ ეჭვი შეგაქვს ამ დაავადებაში, მაშინ სწრაფი მკურნალობის დაწყება აუცილებელია ამფოტერიცინ B-თ.

ნეკროზული ესქარები კი ჩნდება ანთრაქსის დროს, მაგრამ კანზე. აქ გაქვს ლორწოვანი გარსების და სინუსების ინფექცია, თანაც როგროც ჩანს საკმაოდ მძიმე. ორალური ანტიბიოტიკები სერიოზული ინფექციის დროს არ შეიძლება: ინტრავენური ყოველთვის (ან იტრათეკალური მენინგიტის დროს მაგალითად). ვერიკონაზოლს ასპერგილიოზის დროს, მარა ეს ასპერგილიოზი არ არის არც კლინიკურად და არც ჰიფები არ არის ასპერგილუსის (ბევრი სეპტა აქვს მაგას და 45 გრადუსით იტოტება, არა 90-ით, როგორც სურათზე).

ამ ინფექციას კიდევ ქირურგი ჭირდება და ყველანაირი მკვდარი ქსოვილი ქირურგიულად უნდა მოიცილონ.

ისე ამ ინფექციაში იჭვის შეტანა ნებისმიერი სინუსიტის დროს უნდა გქონდეს, განსაკუთრებით იმუნოსუპრესირებულ ავადმყოფებში დიაბეტიკების ჩათვლით.

P.S. ასპერგილუსის ჰიფა
user posted image

Posted by: vano_t 17 Nov 2008, 20:48
QUOTE
ერთი შეკითხვა იმათთვს, ვინც  ვირტუალურად ამერიკაშია-
საქართველოში  ასპერგილოზის დიაგნოსტირება სად ხდება?
რომელ ლაბორატორიაში?

ხდება არ ხდება, მაგი არ ვიცი. თუ არ ხდება, ძალიან ცუდი. ისე ასპერგილუსების სახეობები ფართოდ არიან გავრცელებული ბუნებაში და ენდემურ პათოგენებს არ წარმოადგენენ.

განსაკუთრებით ფუმიგატუს სახეობა არის გავრცელებული, რომელიც ყველანაირ დაავადებას იწვევს იმუნოსუპრესირებულებში-ჩვენ კიდევ გვყავს ბევრი შიდსით დაავადებული და ბევრი ავადმყოფი ქმეოთერაპიით. ამას თუ მიამატებ იმას, რომ ასეთი ავადმყოფები ხშირად ინფექციებით იღუპებიან და ამ ინფექციების დიაგნოზი არ ხდება, ამათ შორის აღმოჩნდება საკმაო რაოდენობით ასეთი ავადმყოფები. დღეს არ ხდება დიაგნოზი, მაგრამ ხვალ მოხდება, განვითარდება მაქაც მედიცინა იმედია.

Posted by: LUKA-BRAZI 17 Nov 2008, 21:21
vano_t
QUOTE
განვითარდება მაქაც მედიცინა იმედია.

user.gif ეეჰჰჰჰ, იმდია, ოღონდაც რამე ეშველოს !!! ..... აუ რატო ვერ ვიცლი რა, რო ქეისებს ჩავუჯდე biggrin.gif მარა რა ვქნა, ხუთშაბათამდე ნევროლოგიას ვეჭიდავები და ვერ ვიცლი სამწუხაროდ user.gif აი, რო დავბრუნდები ისეთ ქეისს დავდებ, სულ დაგებნეთ თავგზა შენ, გარდიანს, თორჩა_ქილს და ყველას ვინც აქ პოსტავთ biggrin.gif
P.S. თორჩა_ქილ, რჩევა, გაუფრთხილდი ჰეპატოციტებს და ნეირონებს biggrin.gif

Posted by: Tornike Alashvili 17 Nov 2008, 21:21
QUOTE (vano_t @ 17 Nov 2008, 20:48 )
QUOTE
ერთი შეკითხვა იმათთვს, ვინც   ვირტუალურად ამერიკაშია-
საქართველოში  ასპერგილოზის დიაგნოსტირება სად ხდება?
რომელ ლაბორატორიაში?

ხდება არ ხდება, მაგი არ ვიცი. თუ არ ხდება, ძალიან ცუდი. ისე ასპერგილუსების სახეობები ფართოდ არიან გავრცელებული ბუნებაში და ენდემურ პათოგენებს არ წარმოადგენენ.

განსაკუთრებით ფუმიგატუს სახეობა არის გავრცელებული, რომელიც ყველანაირ დაავადებას იწვევს იმუნოსუპრესირებულებში-ჩვენ კიდევ გვყავს ბევრი შიდსით დაავადებული და ბევრი ავადმყოფი ქმეოთერაპიით. ამას თუ მიამატებ იმას, რომ ასეთი ავადმყოფები ხშირად ინფექციებით იღუპებიან და ამ ინფექციების დიაგნოზი არ ხდება, ამათ შორის აღმოჩნდება საკმაო რაოდენობით ასეთი ავადმყოფები. დღეს არ ხდება დიაგნოზი, მაგრამ ხვალ მოხდება, განვითარდება მაქაც მედიცინა იმედია.

დიაგნოსტირება ასპერგილუსების როგორ არ ხდება-რამდენიც გინდა
მახსოვს ერთი ასეთი პაციენტი –ფეხი ქონდა დაზიანებული ამ სოკოთინ და ჭყიპავდნენ უძლიერესი ანტიბიოტიკებით.
აბსოლიტურად უშედეგოდ.პრაკტიკულად ინტოკსიკაციიდან იღუპებოდა.ვერც მე ვერაფერი ვერ გაუკეთე.სიმსივნეა თუ სოკო–ვეღარც გაარჩევდი –ისეთი საზიზღრობაა

Posted by: Guardian 17 Nov 2008, 23:27
vano_t
QUOTE
C. დიაბეტიკში (და იმუნოსუპრესიული მდგომერეობების დროს) გვხვდები მუკორმიკოზი-ძაან მაღალი სიკვდილიანობის სოკოვანი ინფექცია. თუ ეჭვი შეგაქვს ამ დაავადებაში, მაშინ სწრაფი მკურნალობის დაწყება აუცილებელია ამფოტერიცინ B-თ.

ამომწურავი პასუხია. yes.gif

Posted by: Blind_Torture_Kill 18 Nov 2008, 01:39
50 year old man presents with sudden weakness in his left leg.He has felt well lately and has no past medical history of coronary artery disease,hyperlipidemia or hypertension and no family history of MI or stroke.
PE - motor weakness in the left leg with no other neurologic deficits and no cardiac murmur.MRI of brain demonstrates small ischemic infarct in the arterial distribution of the correlating with motor control of left leg.
Angiography and echocardiography reveal normal coronaries,normal valves and no vegetations + small right-to-left shunt

user.gif

Whatdafuck is wrong with this person ?

Posted by: Cousteau 18 Nov 2008, 02:43
QUOTE (Blind_Torture_Kill @ 18 Nov 2008, 01:39 )


user.gif

Whatdafuck is wrong with this person ?

ატრაკებს? gigi.gif


პარადოქსული ებოლიაა?

Posted by: vano_t 18 Nov 2008, 03:00
Blind_Torture_Kill
QUOTE
50 year old man presents with sudden weakness in his left leg.He has felt well lately and has no past medical history of coronary artery disease,hyperlipidemia or hypertension and no family history of MI or stroke.
PE - motor weakness in the left leg with no other neurologic deficits and no cardiac murmur.MRI of brain demonstrates small ischemic infarct in the arterial distribution of the correlating with motor control of left leg.
Angiography and echocardiography reveal normal coronaries,normal valves and no vegetations + small right-to-left shunt.

ტიპიური შემთხვევაა. ისქემიური ინსულტის დროს ეძებ ემბოლიის წყაროს, იმიტომ რომ ხშირად ესენი ემბოლიის გამოა გამოწვეული. ტვინში მიმავალი ემბოლიის წყარო გაქვს ძირითადა მარცენა ცირკულაციაში: 1) ან კაროტიდების დონეზე (და იმიტომ აკეთებ კაროტიდების დოპლერს ან ბირთვულ-მაგნიტურ ანგიოგრაფიას) ან 2) გულში, კერძოდ წინაგულში და განსაკუთრებით წინაგულების ფიბრილაციის დროს. ამის დასადგენად გჭირდება ექოკარდიოგრამა. ჩვეულებრივი ექო არ არის ძალიან მგრძნობიარე ტესტი წინაგულში თრომბების დასადგენად. ყველაზე კარგია ამ დროს ტრანსეზოფაგიალური ექო, რადგანაც მარცხენა წინაგულის ვიზუალიზაცია ძაან კარგია ამ დროს. ამ კოკრეტულ შემთხვევაში ვერ იტყვი სინამდვილეში, რომ წინაგულში არ არის თრომბი, მიუხედავად იმისა რომ ექოკარდიოგრამა ნეგატიური იყო. მარა, აქ ცოდნას ამოწმებენ იმაზე, რომ გაითვალისწინო მარჯვენა ცირკულაციის (ქვემო ღრუ ვენის) თრომბმა შეიძლება მოქცეს ემბოლი ტვინში, თუ გულში გაქვს მარჯვნიდან-მარცხნივ შუნტი. ანუ, ქვემო ღრუ ვენების თრომბოზი თუ გაქვს, აქედან თრომბი შევა მარჯვენა წინაგულში/პარკუჭში და ამ შუნტით შეიძლება მოხდეს მარცხენა მხარეს და შემდეგ ტვინში. ამიტომ ქვემო ღრუ ვენის დოპლერიც უნდა გააკეთო, რომ თრომბი იქაც გამორიცხო. შუნტის გარეშე ეს არ გჭირდება.

იმედია გასაგებია.

Posted by: Blind_Torture_Kill 18 Nov 2008, 03:11
Cousteau
QUOTE
პარადოქსული ებოლიაა?

კი
ტაკ დერჟაც

vano_t
საღოლ


Posted by: basa-ttt 18 Nov 2008, 08:37
QUOTE
ხდება არ ხდება, მაგი არ ვიცი. თუ არ ხდება, ძალიან ცუდი

არ ხდება-
ყოველ ემთხვევაში, როცა მე დამჭირდა რამდენიმე წლის წინ ერთ ასთმიან პაციეტთან -
მაშინ არ კეთდებოდა.

ის კი არა,
მგონი მიკოპლაზმური ინფექციის დიაგნოსტირებაც არ ხდება საქართველოში - მაგას ვინღა ჩივის.


Posted by: mtvareuli 18 Nov 2008, 13:15
ეს ძალიან კარგი თემაა და მკაცრად გავაკონტროლებ

წაიშლება წყველანაირი ოFტოპიკი, თემიდან გადახვევა, ჩეთი და ა.შ.

არსებობს შესაბამისი თემები და სხვა საკითხები იქ განიხილეთ

აქ დაიდება ქეისები და მხოლოდ პასუხები ან გარჩევა უნდა იყოს, სხვა ყველაფერი ჩაითვლება ოFტოპიკად.

გარდა ამისა, სანამ ერთზე პასუხს არ გასცემენ, ახალ კითხვას არავინ დასვამს!!!

თუ ვერავინ გასცა პასუხი, კითხვის ავტორმა კითხვის დასმიდან 24 საათის განმავლობაში უნდა თქვას თუ რა იყო


პრეტენზიებისთვისაც არსებობს შესაბამისი თემა, თუ რამეა http://forum.ge/?f=43&showtopic=33636183

Posted by: Cousteau 18 Nov 2008, 18:48
ქეისი არ არის, მაგრამ ეგებ დაგაინტერესოთ (მეთვითონ არ ვიცოდი და დღეს წავიკითხე):

არსებობს TPN (Total parenteral nutrition) და PPN (Partial Parenteral nutrition), TPN-ის ჩადგმისას კათეტერი უნდა იყოს ცენტრალურ ვენაში (SVC), ხოლო PPN-ის ჩადგმა შეიძლება პერიფ.ვენაშიც, კითხვა: რატომ არ შეიძლება TPN-ის 'ჩადგმა' პერიფერულ ვენაში?

და გასართობათ ბარემ მეორე კითხვა კიდევ (ძალიან ბევრმა ექიმმა არ იცის ეს smile.gif ) რატომ ვერ გრძნობს ადამიანი სურდოს დროს საჭმელის (ვთქვათ კატლეტის smile.gif ) გემოს?

Posted by: Blind_Torture_Kill 18 Nov 2008, 19:47
QUOTE
არსებობს TPN (Total parenteral nutrition) და PPN (Partial Parenteral nutrition), TPN-ის ჩადგმისას კათეტერი უნდა იყოს ცენტრალურ ვენაში (SVC), ხოლო PPN-ის ჩადგმა შეიძლება პერიფ.ვენაშიც, კითხვა: რატომ არ შეიძლება TPN-ის 'ჩადგმა' პერიფერულ ვენაში?


ამაზე რაც ვნახე ის იყო რომ TPN-ი ჰიპერტონული ხსნარია და მისი პერიფერიიდან შეყვანა ფლებიტს ან სხვა გართულებას გამოიწვევს
ცენტრალური - უფრო დიდი კალიბრისაა და შესაბამისად სითხესაც უფრო მეტი რაოდენობით შეიცავს რაც ანზავებს უცებ

ახალი ქეისი

Four days after a previously healthy 4-year-old child
appears to be recovering from a bout of the "flu,'" he
develops severe vomiting and irritability. He is admitted to
a hospital.
PE - he is increasingly icteric and lethargic.
Laboratory testing reveals:
total bilirubin level of 7.8 mg/dL, direct bilirubin of 6.1 mg/dL,
alkaline phosphatase of 125 U/L, AST of 622 U/L, and ALT of 705 U/L in
serum. The blood ammonia concentration is 119 /xmol/L.
The child dies of cerebral edema.

რა არის და როდის გვხვდება

Posted by: Guardian 18 Nov 2008, 21:12
Blind_Torture_Kill

რეის სინდრომი.

დარწმუნებული ვარ რა საკუთარი პასუხის სისწორეში, ვდებ ახალ ქეისს -

A 29-year-old woman is brought to the emergency room
with altered mental status, fever, and leg pain. Her husband reports
that she first complained of pain in her leg yesterday,
and there was some slight redness in this area. Over the
night, she developed a fever to as high as 39.8°C and became
obtunded this morning. At that point, her family brought
her to the emergency room. Upon arrival, she is unresponsive
to voice and withdraws to pain. The vital signs are:
blood pressure 88/40 mmHg, heart rate 126 beats/min, respiratory
rate 28 breaths/min, temperature 39.3°C, and SaO2
95% on room air. Examination of the left leg shows diffuse
swelling with brawny edema. The patient grimaces in
pain when the area is touched. There are several bullae filled
with dark blue to purple fluid. Laboratory studies show: pH
7.22, PaCO2 28 mmHg, PaO2 93 mmHg. The creatinine is 3.2
mg/dL.White blood cell count is elevated at 22,660/μL with
a differential of 70% polymorphonuclear cells, 28% band
forms, and 2% lymphocytes. A bulla is aspirated and the
Gram stain shows gram-positive cocci in chains.
What is the most appropriate therapy for this patient?

A. Ampicillin, clindamycin, and gentamicin
B. Clindamycin and penicillin
C. Clindamycin, penicillin, and surgical debridement
D. Penicillin and surgical debridement
E. Vancomycin, penicillin, and surgical debridement

Posted by: vano_t 18 Nov 2008, 21:14
Cousteau
QUOTE
ქეისი არ არის, მაგრამ ეგებ დაგაინტერესოთ (მეთვითონ არ ვიცოდი და დღეს წავიკითხე):

არსებობს TPN (Total parenteral nutrition) და PPN (Partial Parenteral nutrition), TPN-ის ჩადგმისას კათეტერი უნდა იყოს ცენტრალურ ვენაში (SVC), ხოლო PPN-ის ჩადგმა შეიძლება პერიფ.ვენაშიც, კითხვა: რატომ არ შეიძლება TPN-ის 'ჩადგმა' პერიფერულ ვენაში?

პარენტერალური კვება ყოველთვის დიდ ვენებში ადმინისტრაციით ხდება. ყოველ შემთხვევაში მე არ მინახია სხვანაირად. განსხვავება ტოტალურსა და ნაწილობრივს შორის მხოლოდ ის არის, რომ ტოტალურის დროს მხოლოდ პარენტერალური კვება ხდება და ნაწილობრივის დროს შეგიძლია მისცე ორალური საკვებიც (ან პაციენტი დაღეჭავს თუ შეუძლია, ან ნაზოგასტრული ან ანზოდუოდენალური კათეტერით მისცემ). ბლაინდ-ტორჩა-კილი მართალია იმ ამბავში, რომ პარენტერალურ კვებას ახასიათებს თრომბოფლებიტები (ცენტრალური ვენებით ადმინისტრაციის დროსაც კი). ოღონდ, ცენტრალური ვენებით ადმინისტრაციის დროს ამის შანსი ნაკლებია, ვიდრე პერიფერული ვენებით ადმინისტრაციის დროს.

ქვემოთ დავდებ სურათს და აღწერეთ რა ხდება უჯრედში. დიაგნოზი არ არის საინტერესო, არამედ ის რაც ხდება. კარგი სასწავლო სლაიდია. გარდიანი ეჭვგარეშე გამოიცნობს smile.gif მარა, აღწერაც მიაყოლოს სხვებმა რომ გაიგონ.
user posted image

Posted by: Guardian 18 Nov 2008, 21:19
პაპაჩემმა იცოდა ხოლმე თქმა - რა ყველას ერთდროულად აგიდგებათ ხოლმეო? biggrin.gif

Posted by: Blind_Torture_Kill 18 Nov 2008, 21:25
Guardian

surgical debridement biggrin.gif

სწორი ხარ და ისიც დაამატე რატო დაემართა იმ ბავშვს ?


კარიორექსი და კარიოლიზი ხომ არაა ?

Posted by: Guardian 18 Nov 2008, 21:27
Blind_Torture_Kill
QUOTE
surgical debridement

ეგ სამ პასუხშია და რომელია მაგათგან სწორი?

QUOTE
სწორი ხარ და ისიც დაამატე რატო დაემართა იმ ბავშვს ?

ასპირინი მისცეს.

vano_t
QUOTE
აღწერეთ რა ხდება უჯრედში

კიბოა?

Posted by: mtvareuli 18 Nov 2008, 21:34
vano_t

ჯერ გარდიანის შეკითხვაზე უნდა გაგეცათ პასუხი და მერე დაგედოთ mo.gif

პ.ს. ქართულად აღარ ვლაპარაკობთ?

Posted by: Blind_Torture_Kill 18 Nov 2008, 21:34
Guardian
C. Clindamycin, penicillin, and surgical debridement

Posted by: Guardian 18 Nov 2008, 21:39
Blind_Torture_Kill
QUOTE
C. Clindamycin, penicillin, and surgical debridement

დიახ.

This patient is presenting with septic shock secondary to
necrotizing fasciitis with group A streptococcus. Necrotizing fasciitis presents with fever
and pain of the affected area that progresses rapidly to severe systemic symptoms. Swelling
and brawny edema may be present early in the disease, progressing rapidly to darkred
induration with bullae filled with bluish to purple fluid. Pathologically, the underlying
dermis shows extensive thrombosis of vessels in the dermis. Necrotizing fasciitis is
commonly caused by group A streptococcus, specifically S. pyogenes, or mixed aerobicanaerobic
infections. In this patient, the presence of gram-positive cocci in chains suggests
S. pyogenes as the underlying cause. The initial treatment of patients with necrotizing
fasciitis is surgical debridement of the affected area. The area of debridement is
frequently very large. During surgery, all necrotic tissue should be removed and any increased
compartment pressure should be relieved. In addition, appropriate antibiotics
should be initiated. For group A streptococcus, the combination of clindamycin and penicillin
should be used. Penicillin is bacteriocidal for streptococcus as is clindamycin. Clindamycin
also neutralizes the toxins produced by group A streptococcus. Antibiotic
therapy alone should not be used as necrotizing fasciitis is rapidly fatal without surgical
intervention. Vancomycin is not a first-line antibiotic in necrotizing fasciitis and should
be considered only for those with penicillin allergy.

Posted by: Blind_Torture_Kill 18 Nov 2008, 21:42
mtvareuli

ინგლისურად თავის მუღამი აქ

Posted by: Guardian 18 Nov 2008, 21:47
Blind_Torture_Kill
QUOTE
ინგლისურად თავის მუღამი აქ

Invia est in medicina via sine english language. biggrin.gif

Posted by: Blind_Torture_Kill 18 Nov 2008, 21:50
QUOTE
Invia est in medicina via sine english language


გეთანხმები
yes.gif

Posted by: Guardian 18 Nov 2008, 22:33
რა არის ეს?
------------------

Posted by: Guardian 18 Nov 2008, 22:39
---------------------------------------------------------------------

Posted by: vano_t 18 Nov 2008, 22:41
mtvareuli
QUOTE
ჯერ გარდიანის შეკითხვაზე უნდა გაგეცათ პასუხი და მერე დაგედოთ mo.gif

სანამ მე ვწერდი პასუხს, იმ დროში 3 კაცმა იმოქმედა. არადა, პასუხის წერა რო დავიწყე, არაფერი იყო ნაკითხი კუსტოს კითხვის გარდა.

Blind_Torture_Kill
QUOTE
კარიორექსი და კარიოლიზი ხომ არაა ?
კი, ეგენიც გაქვს და პიკნოზები გაქვს კიდევ. მიუთითე რომელია პიკქნოზი, რომელი კარიორექსი და რომელი კარიოლიზი.
* * *
Guardian
QUOTE
რა არის ეს?
------------------

ფილტვის ემბოლიაა კატეზე?

Posted by: Guardian 18 Nov 2008, 22:47
vano_t
QUOTE
ფილტვის ემბოლიაა კატეზე?

არა.
CT და მაკრო პათოლოგია ერთი და იგივე შემთხვევაა.

Posted by: Cousteau 18 Nov 2008, 23:04
QUOTE (Guardian @ 18 Nov 2008, 22:47 )
vano_t
QUOTE
ფილტვის ემბოლიაა კატეზე?

არა.
CT და მაკრო პათოლოგია ერთი და იგივე შემთხვევაა.

ARDS ?

Posted by: vano_t 18 Nov 2008, 23:12
QUOTE (Cousteau @ 18 Nov 2008, 23:04 )
QUOTE (Guardian @ 18 Nov 2008, 22:47 )
vano_t
QUOTE
ფილტვის ემბოლიაა კატეზე?

არა.
CT და მაკრო პათოლოგია ერთი და იგივე შემთხვევაა.

ARDS ?

ARDS დიფუზური პროცესია და კტ-ზე სხვანაირად გამჩნდება. ეგ არ არის ARDS. მაქ მარცხენა ფილტვში გაქვს რადიოკაონტრასტული მასა. პლევრის გამონადენმაც შეიძლება მოგცეს ეგ, მარა როცა წევს ავადმყოფი, პლევრის ღრუში სითხე ასე თუ ისე სწორდება. შეიძლება ფილტვისპროცესია ან პლევრიდან მომავალი. ისე პლევრასთან შეხება დიდი აქვს და შეიძლება იქიდან მოდის. მაკროპათოლოგიით მაინც ვერაფერს მივხვდები. მარა კტ-ზე შეიძლება მეზოთელიომაა.

იტოგში გარდიან ცოტა კლინიკა უნდა მიაყოლო მასეთ რაღაცეებს.

Posted by: Cousteau 18 Nov 2008, 23:14
QUOTE (vano_t @ 18 Nov 2008, 23:12 )
მაკროპათოლოგიით მაინც ვერაფერს მივხვდები.

მაგიტომ ვთქვი ARDS, იდეაში ფიბრინულ ჩანაცვლებას მივამსგავს (რავი ეგეთი წარმომიდგენია smile.gif არასდორს არ მინახავს) რომელიც არდსს დროს ლაგდება + კტზც ეგეთი მახსოვს და აი დიფუზური როა მაგაში მართალიხარ, ცალ მხარეს არ იქნება ''ჩალაგებული''

Posted by: basa-ttt 18 Nov 2008, 23:33
QUOTE
ისე პლევრასთან შეხება დიდი აქვს და შეიძლება იქიდან მოდის.

ალვეოლები ჩანს ფოტოზე -
ფილტვშია..
თეთრი ნადებია -
რაც ფიბროზს გვაგონებს,
კტ -ზეც გაძლიერებულია დიფუზურად სურათი.
თან დიდი ზომის ზონაა - და პლევრასთან თითქოს არის ხაზივით საზღვარი -
მაგრამ სასაზღვრო ხაზი დაკლაკნილია-
ალბათ ჩაზრდილია?
სიმსივნეა.?

Posted by: texasuri jleta benzoxerxit 18 Nov 2008, 23:39
ბრონქოექტაზური დაავადება გართულებული პლევრის ემპიემით? boli.gif baby.gif
პნევმოფიბროზი?

Posted by: vano_t 18 Nov 2008, 23:39
QUOTE (basa-ttt @ 18 Nov 2008, 23:33 )
QUOTE
ისე პლევრასთან შეხება დიდი აქვს და შეიძლება იქიდან მოდის.

ალვეოლები ჩანს ფოტოზე -
ფილტვშია..
თეთრი ნადებია -
რაც ფიბროზს გვაგონებს,
კტ -ზეც გაძლიერებულია დიფუზურად სურათი.
თან დიდი ზომის ზონაა - და პლევრასთან თითქოს არის ხაზივით საზღვარი -
მაგრამ სასაზღვრო ხაზი დაკლაკნილია-
ალბათ ჩაზრდილია?
სიმსივნეა.?

შეიძლება ფილტვშია, მაგრამ არა მაგის გამო. მეზოთელიომა ინვაზიური სიმსივნეა. ფიტლვში ღრმად შეიძლება შევიდეს.

Posted by: basa-ttt 18 Nov 2008, 23:41
QUOTE
შეიძლება ფილტვშია, მაგრამ არა მაგის გამო. მეზოთელიომა ინვაზიური სიმსივნეა. ფიტლვში ღრმად შეიძლება შევიდეს.

აი ნახე კარგად -

user posted image

პლევრასთან არის საზღვარი

კლინიკა გვჭირდება
თუ ტკივილით დაიწყო გვერდში -
პლევრისაა.
თუ სიცხე იყო?
ან სისხლიანი ნახველი?
ამას ნიშვნელობა აქვს

Posted by: Guardian 18 Nov 2008, 23:44
vano_t
QUOTE
შეიძლება მეზოთელიომაა.

არა.

Posted by: basa-ttt 18 Nov 2008, 23:47
სიმსივნე სავარაუდოდ ბრონქებიდან გამოდის
ერთ ადგილას მიზრდილივითაა ბრონქთან.
აქ ბრონქოსკოპია და ბიოფსია გვინდა
მერე გავაგზავნით მასალას
Guardian-თან
და დაგვიწერს პათანატომიურ პასუხს.

Posted by: Cousteau 18 Nov 2008, 23:49
Guardian
შეგიძლია რენტგენი დადო ამის ?

Posted by: Guardian 18 Nov 2008, 23:50
აჰა, მესამე სურათიც -----------

Posted by: Cousteau 18 Nov 2008, 23:55
QUOTE (Guardian @ 18 Nov 2008, 23:50 )
აჰა, მესამე სურათიც -----------

შეიძლება რამე პნეუმოკონიოზია.... მე არვიცი მოკლედ user.gif

Posted by: Guardian 18 Nov 2008, 23:59
Cousteau
QUOTE
შეგიძლია რენტგენი დადო ამის ?

რენტგენზე მხოლოდ კერლის B ხაზები ჩანს.

Posted by: texasuri jleta benzoxerxit 19 Nov 2008, 00:00
რაღაც გენეზის პნევმოფიროზს გავს...მაგრამ პლევრაში რაღა ხდება?ბრონქოექტაზია და პლევრის ემპიემა არ იყო ხო სწორი?

Posted by: Guardian 19 Nov 2008, 00:03
texasuri jleta benzoxerxit
QUOTE
რაღაც გენეზის პნევმოფიროზს

არა.
QUOTE
ბრონქოექტაზია და პლევრის ემპიემა არ იყო ხო სწორი?

არა.

Posted by: Cousteau 19 Nov 2008, 00:03
QUOTE (Guardian @ 18 Nov 2008, 23:59 )
კერლის B ხაზები ჩანს.

ეგ რაღაა? gigi.gif user.gif
ბლინ.... ინგლისურად დამიწერე მაგათი სახელი მაგაში მაინც გავნათლდე

Posted by: Guardian 19 Nov 2008, 00:12
Cousteau
QUOTE
ინგლისურად დამიწერე მაგათი სახელი მაგაში მაინც გავნათლდე

Kerley's B lines.

Posted by: basa-ttt 19 Nov 2008, 00:14
QUOTE
ბლინ.... ინგლისურად დამიწერე მაგათი სახელი მაგაში მაინც გავნათლდე

არ გრცხვენიათ?
ამერიკაში ცხოვრობთ?
კარტული სულ დამავიცკდა?
gigi.gif

Posted by: Cousteau 19 Nov 2008, 00:16
QUOTE (basa-ttt @ 19 Nov 2008, 00:14 )
QUOTE
ბლინ.... ინგლისურად დამიწერე მაგათი სახელი მაგაში მაინც გავნათლდე

არ გრცხვენიათ?
ამერიკაში ცხოვრობთ?
კარტული სულ დამავიცკდა?
gigi.gif

პოსტიც არ შემჩნევაც უკულტურობაა და 'რეზკიი' პასუხის გაცემაც უკულტურობაა და რომელი უკულტურობა ჩავიდინო ამ ორი უკულტურობიდან? smile.gif)

ქართული ლეღვი წიგნებიდან მირჩევნია ინგლისური წიგნიდან წავკითხო user.gif

Posted by: Blind_Torture_Kill 19 Nov 2008, 00:17
idiopathic pulmonary fibrosis

cause unknown biggrin.gif

1.კარიოლიზი
2 პიკნოზი
3 კარიორექსი

Posted by: Guardian 19 Nov 2008, 00:23
Blind_Torture_Kill
QUOTE
idiopathic pulmonary fibrosis

არა.

აი, კერლის B ხაზები (ისარი) -

Posted by: basa-ttt 19 Nov 2008, 00:27
QUOTE
პოსტიც არ შემჩნევაც უკულტურობაა და 'რეზკიი' პასუხის გაცემაც უკულტურობაა და რომელი უკულტურობა ჩავიდინო ამ ორი უკულტურობიდან? )

ქართული ლეღვი წიგნებიდან მირჩევნია ინგლისური წიგნიდან წავკითხო

დედა ენა არ დაივიწკო...
ყველა ტერმინს აქვს ქართული შესატყვისი-
რენტგენის მიხედვით თითქოს კიბოა.
ბრონქოგენული

Posted by: Blind_Torture_Kill 19 Nov 2008, 00:27
Guardian

რამე პატარა ინფო დაამატე რა

Posted by: Cousteau 19 Nov 2008, 00:28
QUOTE (Guardian @ 19 Nov 2008, 00:23 )

აი, კერლის B ხაზები (ისარი) -

ვნახე მერკში ორ ადგილას წერია მარტო ორივე ადგილას ნახსენებია მარცხენა წინაგულის წნევის მომატების და heart failure-ს მაჩვენებელიარიო, ერთი ხაზი წერია მარტო.

Kerley's B lines-ო indicates-ო ინტერსტიციალურ ედემასო LA-ში წნევის მომატებისასო

QUOTE
დედა ენა არ დაივიწკო...

no.gif

Posted by: basa-ttt 19 Nov 2008, 00:31
Линии Керли B - яркие горизонтальные линии в нижних и средних сегментах легких, они вызваны отеком междолевых щелей и переполнением лимфатических сосудов (рентгенография грудной клетки).

Posted by: vano_t 19 Nov 2008, 00:32
კერლის ხაზები ზოგადად მიუთითებს ინტერსტიციალურ შეშუპებაზე და ბევრი რამის დროს შეიძლება გაჩნდეს, რაც გამოიწვევს ინტერსიტციუმში სითხის დაგროვებას.

ინტერსიტციალური პნევმონიებიც შევა ამაში და ფილტვების შესუპებაც. კტ არასპეციფიურია. მაკრომორფოლოგიურად არ ვიცი რა ხდება.

იტოგში, მოაყოლე კლინიკა ხოლმე. უფრო საიტერესოა ასე.

Posted by: Cousteau 19 Nov 2008, 00:34
ესეგი, ახველებდა ახველებდა, მერე გაატარეს კტზე და ამდროს ჰოპაა და აორტის ანევრიზმა გაუსკდა user.gif

არ დაგვინდო yes.gif user posted image

ან არა, ესეიგი ამ ანევრიზმის ფისტულა ქონდა ბრონქთან და საერთოდ იგ 'ჩასკდა'

Posted by: Guardian 19 Nov 2008, 00:36
basa-ttt
QUOTE
რენტგენის მიხედვით თითქოს კიბოა.
ბრონქოგენული

არა.

Blind_Torture_Kill
QUOTE
რამე პატარა ინფო დაამატე რა

აჰა, მიკრო სურათი - ეხლაც თუ ვერ გამოიცანით, ე.ი. არა ხართ თქვენ მაგის გამომცნობები biggrin.gif -

http://imageshack.us

Posted by: basa-ttt 19 Nov 2008, 00:44
Рак легкого развивается из эпителия слизистой бронхов. Опухоль примерно с одинаковой частотой возникает в правом и левом легком. Рак, поражающий главные, долевые или сегментарные бронхи, называют центральным. Опухоль, возникающую в бронхах меньшего калибра, чем сегментарные, называют периферическим раком.
Формирование периферической опухоли из бронхиального эпителия дистальных, наиболее периферических отделов воздухоносных путей - субсегментарных и более мелких бронхов, чаще всего обуславливает ее равномерное развитие в паренхиме легкого с формированием характерного округлого, "шаровидного" образования. По мере дальнейшего роста такие опухоли часто переходят на близлежащие внелегочные анатомические структуры: париетальную плевру, грудную стенку, диафрагму и другие. Вариантом периферической опухоли в легком является так называемый рак "типа Пенкоста", характеризующийся опухолью шаровидной формы, располагающейся в верхней доле легкого и переходящей на нервы плечевого сплетения, подключичные сосуды, ствол симпатического нерва с симптомокомплексом Горнера (птоз, миоз и энофтальм на стороне поражения).

Posted by: Blind_Torture_Kill 19 Nov 2008, 00:46
Guardian

სარკოიდოზი ხომ არაა ?

Posted by: Guardian 19 Nov 2008, 00:51
basa-ttt
QUOTE
Рак легкого

არ არის ეგ ფილტვის კიბო.

Blind_Torture_Kill
QUOTE
სარკოიდოზი ხომ არაა ?

არა.
მიკრო სურათზე რას ხედავ?

Posted by: Blind_Torture_Kill 19 Nov 2008, 00:56
Guardian


QUOTE
არა. მიკრო სურათზე რას ხედავ?


ჯირკვლოვანი სტრუქტურაა მგონი

Posted by: Guardian 19 Nov 2008, 00:59
Blind_Torture_Kill
QUOTE
ჯირკვლოვანი სტრუქტურაა მგონი

მეტს ვერაფერს ხედავ?

Posted by: Blind_Torture_Kill 19 Nov 2008, 01:02
Guardian


სხვა რაკურსით სლაიდი დადე biggrin.gif
ვერა

Posted by: Guardian 19 Nov 2008, 01:05
Blind_Torture_Kill
QUOTE
სხვა რაკურსით სლაიდი დადე

არა, მეტს აღარაფერს დავდებ - თუკი ხვალ საღამომდე ვერავინ გამოიცნობს, მერე ვიტყვი სწორ პასუხს.

QUOTE
ვერა

ანუ, შენი აზრით, ერთგვაროვანი შენება აქვს?

Posted by: Blind_Torture_Kill 19 Nov 2008, 01:09
Guardian

რამე პატარა მინიშნება გააკეთე თორე ესე ვერაფერს გავიგებთ
ცოტა დიდ გადიდებაზე მანახე ის ქსოვილი

Posted by: Guardian 19 Nov 2008, 01:11
Blind_Torture_Kill
QUOTE
ცოტა დიდ გადიდებაზე მანახე ის ქსოვილი

ძალიან კარგ გადიდებაზეა. biggrin.gif
წავედი ეხლა მე დასაძინებლად. smile.gif

Posted by: vano_t 19 Nov 2008, 01:18
QUOTE (Guardian @ 19 Nov 2008, 00:36 )
basa-ttt
QUOTE
რენტგენის მიხედვით თითქოს კიბოა.
ბრონქოგენული

არა.

Blind_Torture_Kill
QUOTE
რამე პატარა ინფო დაამატე რა

აჰა, მიკრო სურათი - ეხლაც თუ ვერ გამოიცანით, ე.ი. არა ხართ თქვენ მაგის გამომცნობები biggrin.gif -

http://imageshack.us

ჰამარტომაა? რაცხა ნორმალური არანორმალური ქსოვილია: ეპითელიუმიც, ჰიალინური შემაერთებელიც. გამიხარე გული, მითხარი რომ მართალი ვარ biggrin.gif

Posted by: Guardian 19 Nov 2008, 01:19
vano_t
QUOTE
ჰამარტომაა? რაცხა ნორმალური არანორმალური ქსოვილია: ეპითელიუმიც, ჰიალინური შემაერთებელიც. გამიხარე გული, მითხარი რომ მართალი ვარ

არა. biggrin.gif

Posted by: Blind_Torture_Kill 19 Nov 2008, 01:33
იმენა გარტყმაში არ ვარ რა შეიძლება იყოს

რამე ექტოპიური ქსოვილი ხომ არაა ?

ცოტა დაგვაკვალიანე ბიჯო ჩვენი მტერი ხომ არ ხარ

Posted by: vano_t 19 Nov 2008, 01:34
QUOTE (Guardian @ 19 Nov 2008, 01:19 )
vano_t
QUOTE
ჰამარტომაა? რაცხა ნორმალური არანორმალური ქსოვილია: ეპითელიუმიც, ჰიალინური შემაერთებელიც. გამიხარე გული, მითხარი რომ მართალი ვარ

არა. biggrin.gif

ახლა რა მითხარი იცი, აი ის ქსოვილი (თეთრი სივრცეები რომ არის) ალვეოლარული სივრცეა თუ არა?

Posted by: Blind_Torture_Kill 19 Nov 2008, 01:47
Guardian

Pleuropulmonary blastoma type 3

Posted by: Guardian 19 Nov 2008, 01:50
Blind_Torture_Kill
QUOTE
Pleuropulmonary blastoma type 3

ეუჰ.
არა. biggrin.gif

Posted by: Blind_Torture_Kill 19 Nov 2008, 01:53
Guardian
QUOTE
ეუჰ. არა.

საიდან მოვთხარე biggrin.gif ამის დედა ვატირე არ კაიფობ


მოგვეხმარე თორე გვენძრევა smile.gif


არც ადენოკარსინომაა ?

Posted by: vano_t 19 Nov 2008, 02:11
ლიმფანგიტური კარცინომატოზი (თუ ფიტვების ჩვეულებრივი შეშუპება არ არის-რაზეც ადრე პასუხი არ გაგიცია და ჩავთვალეთ რომ არ არის).

Posted by: Guardian 19 Nov 2008, 02:17
vano_t
QUOTE
ლიმფანგიტური კარცინომატოზი

დიახ.

ფაქტიურად, ეს არის რომელიმე კიბოს მეტასტაზირება ფილტვში, სიმსივნის გავრცელებით წილაკთაშორისი ლიმფური სადინარებით.

ბოლო სურათზე ლიმფური კვანძი ჩანდა მასში ადენოკარცინომის მეტასტაზით.

Posted by: Blind_Torture_Kill 19 Nov 2008, 02:18
vano_t

up.gif

იმენა საღოლ

Posted by: vano_t 19 Nov 2008, 02:21
QUOTE (Guardian @ 19 Nov 2008, 02:17 )
vano_t
QUOTE
ლიმფანგიტური კარცინომატოზი

დიახ.

ფაქტიურად, ეს არის რომელიმე კიბოს მეტასტაზირება ფილტვში, სიმსივნის გავრცელებით წილაკთაშორისი ლიმფური სადინარებით.

ბოლო სურათზე ლიმფური კვანძი ჩანდა მასში ადენოკარცინომის მეტასტაზით.

რა ეს ტვინის საჭლეტ კეისებს რომ დებ. ისეთი რამე დადე პირველივეზე რო გამოიცნოს კაცმა biggrin.gif

ე.ი. იმ მიკროსლაიდზე, ლიმფური სადინრებია და შიგ სიმსივნური უჯრედებია კახველი, ჰა?

Posted by: Blind_Torture_Kill 19 Nov 2008, 02:24
იმენა სახელს ძლივს ვკითხულომ ლიმფანგიტური კარცინომატოზი biggrin.gif ენის გასატეხი უფროა

ჰა დაიდო აბა ახალი ქეისი

Posted by: Guardian 19 Nov 2008, 02:32
vano_t
QUOTE
ე.ი. იმ მიკროსლაიდზე, ლიმფური სადინრებია და შიგ სიმსივნური უჯრედებია კახველი, ჰა?

მიკროზე - არა, მაკროზე.
მიკრო მარტო ბოლო სურათი იყო.

Blind_Torture_Kill
QUOTE
დაიდო აბა ახალი ქეისი


A 36-year-old male is referred to your clinic for evаluation
of hypercalcemia noted during a health insurance
medical screening. He has noted some fatigue, malaise,
and a 4-lb weight loss over the last 2 months. He also has
noted constipation and “heartburn.” He is occasionally
nauseated after large meals and has water brash and a sour
taste in his mouth. The patient denies vomiting, dysphagia,
or odynophagia. He also notes decreased libido and a
depressed mood. Vital signs are unremarkable. Physical
examination is notable for a clear oropharynx, no evidence
of a thyroid mass, and no lymphadenopathy. Jugular
venous pressure is normal. Heart sounds are regular with
no murmurs or gallops. The chest is clear. The abdomen is
soft with some mild epigastric tenderness. There is no rebound
or organomegaly. Stool is guaiac-positive. Neurologic
examination is nonfocal. Laboratory values are
notable for a normal complete blood count. Calcium is
11.2 mg/dL, phosphate is 2.1 mg/dL, and magnesium is 1.8
meq/dL. Albumin is 3.7 g/dL, and total protein is 7.0 g/dL.
TSH is 3 μIU/mL, prolactin is 250 μg/L, testosterone is 620
ng/dL, and serum insulin-like growth factor 1 (IGF-1) is
normal. Serum intact parathyroid hormone level is 135
pg/dL. In light of the patient’s abdominal discomfort and
heme-positive stool, you perform an abdominal computed
tomography (CT) scan that shows a lesion measuring 2 cm
by 2 cm in the head of the pancreas. What is the diagnosis?

Posted by: vano_t 19 Nov 2008, 02:43
QUOTE (Guardian @ 19 Nov 2008, 02:32 )
vano_t
QUOTE
ე.ი. იმ მიკროსლაიდზე, ლიმფური სადინრებია და შიგ სიმსივნური უჯრედებია კახველი, ჰა?

მიკროზე - არა, მაკროზე.
მიკრო მარტო ბოლო სურათი იყო.

Blind_Torture_Kill
QUOTE
დაიდო აბა ახალი ქეისი


A 36-year-old male is referred to your clinic for evаluation
of hypercalcemia noted during a health insurance
medical screening. He has noted some fatigue, malaise,
and a 4-lb weight loss over the last 2 months. He also has
noted constipation and “heartburn.” He is occasionally
nauseated after large meals and has water brash and a sour
taste in his mouth. The patient denies vomiting, dysphagia,
or odynophagia. He also notes decreased libido and a
depressed mood. Vital signs are unremarkable. Physical
examination is notable for a clear oropharynx, no evidence
of a thyroid mass, and no lymphadenopathy. Jugular
venous pressure is normal. Heart sounds are regular with
no murmurs or gallops. The chest is clear. The abdomen is
soft with some mild epigastric tenderness. There is no rebound
or organomegaly. Stool is guaiac-positive. Neurologic
examination is nonfocal. Laboratory values are
notable for a normal complete blood count. Calcium is
11.2 mg/dL, phosphate is 2.1 mg/dL, and magnesium is 1.8
meq/dL. Albumin is 3.7 g/dL, and total protein is 7.0 g/dL.
TSH is 3 μIU/mL, prolactin is 250 μg/L, testosterone is 620
ng/dL, and serum insulin-like growth factor 1 (IGF-1) is
normal. Serum intact parathyroid hormone level is 135
pg/dL. In light of the patient’s abdominal discomfort and
heme-positive stool, you perform an abdominal computed
tomography (CT) scan that shows a lesion measuring 2 cm
by 2 cm in the head of the pancreas. What is the diagnosis?

მიკორპრეპარატზე აბა რა არის თხელკედლიანი სივრცეები შიგნით ბევრი ბირთვებით?

ამ კეისში ნორმალური დონეები დაასახელე ჰორმონების თუ შეიძლება. განსაკუთრებით პარათჰორმონი. მე როგორც მახსოვს ყველაზე მაღალი არის 160 (ჩვენი ლაბის მიხედვით). მაშინ სიპერპარათიროიდიზმი ექნება, მარა მარტო ეგ არა. მე მგონი მაგას ჭირს multiple endcocrine neoplasia-ს რომელიღაც ფორმა.

აჰა, ფორმაც დავადგინე: 1-ლი. მაგისთვის არის დამახასიათებელი პანკრეასის სიმსივნეებიც (ან ინსულინომა ან გასტრინომა ან ვიპომა). რეფლუქსები თუ აქ, მაშინ გასტრინომა აქვს პახოდუ ალბათ.

Posted by: Guardian 19 Nov 2008, 02:48
vano_t
QUOTE
მიკორპრეპარატზე აბა რა არის თხელკედლიანი სივრცეები შიგნით ბევრი ბირთვებით?

სად?

QUOTE
multiple endcocrine neoplasia

QUOTE
1-ლი

QUOTE
გასტრინომა აქვს

კი.

Posted by: Blind_Torture_Kill 19 Nov 2008, 02:50
Guardian

გასტრინომის დროს დიარეა არააქ ადამიანს ?

Posted by: vano_t 19 Nov 2008, 02:50
პროლაქტინიც მაღალი ყოფილა ეგ დონე. ამას მიემატება pituitary ადენომაც. ეგეც მენ 1-სთვის არის დამახასიათებელი.
* * *
QUOTE (Blind_Torture_Kill @ 19 Nov 2008, 02:50 )
Guardian

გასტრინომის დროს დიარეა არააქ ადამიანს ?

არაა სავალდებულო. თანაც, ჰიპერკალცემია შეკრულობას იწვევს და შეიძლება ან ერთი ქონდეს ან მეორე.

Posted by: Blind_Torture_Kill 19 Nov 2008, 02:57
vano_t

როცა დიდი რაოდენობით HCL გამოიყოფა ეგ დიარეას იწვევს

ოკ

Posted by: vano_t 19 Nov 2008, 07:41
Guardian
QUOTE
QUOTE
მიკორპრეპარატზე აბა რა არის თხელკედლიანი სივრცეები შიგნით ბევრი ბირთვებით?

სად?

სადა და, აი ქვემო მარცხენა კვადრატში სულ ეგ სტრუქტურებია. ქვემო მარჯვენა კვადრატში რამოდენიმეა. მარა არა, ეგენი ლიმფური სადინრები არ მგონია: ახლა დავაკვირდი, მრავალშრიანი ეპითელიუმია, რომელიც ქმნის მილებს. ამ ღრუიან სტრუქტურებში კიდევ ბირთვები ჩანს თავმოყრილი. ხოდა, ესენი მთლიანობაში რას წამოადგენენ?
user posted image

Posted by: Guardian 19 Nov 2008, 10:35
vano_t
QUOTE
აი ქვემო მარცხენა კვადრატში სულ ეგ სტრუქტურებია. ქვემო მარჯვენა კვადრატში რამოდენიმეა. მარა არა, ეგენი ლიმფური სადინრები არ მგონია: ახლა დავაკვირდი, მრავალშრიანი ეპითელიუმია, რომელიც ქმნის მილებს. ამ ღრუიან სტრუქტურებში კიდევ ბირთვები ჩანს თავმოყრილი. ხოდა, ესენი მთლიანობაში რას წამოადგენენ?

ეგეთ პატარა გადიდებაზე ბირთვი მხოლოდ პატარა მუქი წერტილის სახით ჩანს.
ეგ ბირთვები არ არის - ეგ არის ჯირკვლოვანი ეპითელიუმის ერთი შრის შიგნით ჩახვეული ეპითელიუმის მეორე შრე.

Posted by: Blind_Torture_Kill 19 Nov 2008, 12:17
A forensic pathologist is asked to evаluate a fatal gunshot involving the left thorax of 27 year old drug dealer.even though a gun and suicide note were found next to the body, the pathologist has concluded that the wound was probably not self-inflicted.which of the following finding is most supportive of that conclusion ?

A.the entrance wound is smaller than the exit wound
B.the entrance wound is stellate shaped
C.the exit wound is irregularly shaped
D.there is no stippling of skin, and the wound is oval with clean margins
E.there is stippling of skin from unburned gunpowder

Posted by: Solveig 19 Nov 2008, 15:39
Blind_Torture_Kill
ეს?
QUOTE
there is no stippling of skin, and the wound is oval with clean margins


Posted by: Cousteau 19 Nov 2008, 15:52
Blind_Torture_Kill : )

არც გამივლია ეგ არსად ალბათ

E.there is stippling of skin from unburned gunpowder ?




Posted by: Blind_Torture_Kill 19 Nov 2008, 15:52
Solveig

QUOTE
Blind_Torture_Kill ეს?QUOTEthere is no stippling of skin, and the wound is oval with clean margins



ყოჩაღ
* * *
Cousteau


თვითონ რომ ესროლა-ახლოდან გაირტყავდა ტყვიას ხოლო წამალი დაწვას ვერ მოასწრებდა და კანზე ლაქებს დატოვებდა

ახალი ქეისი

a 30 y/o man has 1 week history of increased thirst, and increased urine volumes. the result of water deprivation test reveal increased >300mOsm/kg-plasma osmolality and elevated urine osmolality (>120mOsm/kg)
he is currently receiving treatment for non-insulin-dependent diabetes mellitus (type II) as well as for bipolar disorder

what is most likely diagnosis baby.gif

normal value-12 to 14 hour fluid restriction: Greater than 850 mOsm/kg (urine)
* * *
ჯერ კიდე არ გაგიციათ პასუხი

ვინმემ რამე ახალი მაინც დადეთ

Posted by: vano_t 20 Nov 2008, 06:01
Blind_Torture_Kill
QUOTE
a 30 y/o man has 1 week history of increased thirst, and increased urine volumes. the result of water deprivation test reveal increased >300mOsm/kg-plasma osmolality and elevated urine osmolality (>120mOsm/kg)
he is currently receiving treatment for non-insulin-dependent diabetes mellitus (type II) as well as for bipolar disorder

what is most likely diagnosis
აქ გაქვს არასაკმარისი მონაცემი და დიაგნოზს ვერ დასვამ. აგიხსნი რატომ. მოკლედ, ტესტის ავტორს სურს გაარკვიოს რა არის პოლიურია/პოლიდიპსიის მიზეზი. ამის დიფერენციალი ძირითადად მოიცავს 3 რამეს: 1) ნორმალური ადამიანი პირველადი პოლიდიპსიით-ანუ პირველადად არის წყურვილის გაძლიერება უფრო ხშირად ფსიქოლოგიური მიზეზების გამო და არა წყლის დაკარგვის გამო; 2) ცენტრალური უშაქრო დიაბეტი,როცა ანტიდიურეზული ჰორმონის (ADH) გამოყოფა დაქვეითებულია ან შეწყვეტილია; 3) ნეფროგენული უშაქრო დიაბეტი, როცა ADH-ის გამოყოფა ნორმალურია (ან ნორმაზე მეტია) მაგრამ თირკმლის შემკრები მილაკები რეზისტენტულია ჰორმონისადმი. წყლის დეპრივაციის ტესტსაც მაგიტომ აკეთებ, რომ ამათ შორის გააკეთო დიფერენციაცია. როცა წყლის დეპროვაციას ახლავს სისხლის ოსმოლარობის აწევა და შარდის ოსმოლარობის აწევა, ეს ყველაფერი მიუთითებს იმაზე, რომ ორგანიზმში ADH-ც ნორმალური რაოდენობით გამოიყოფა და თირკმლის მიერ შარდის კონცენტრირების უნარიც ნორმალურია. ეს გამორიცხავს 2-ს და 3-ს (ანუ ცენტრალური და ნეფროგენულ უშაქრო დიაბეტებს), რომელთათვისაც წყლის დეპრივაციის შემდეგ დამახასიათებელი იქნება სისხლის მაღალი და შარდის დაბალი ოსმოლალობა, რამეთუ წყლის შეწოვა არ მოხდება თირკმელში და შარდი იქნება ყოველთვის განზავებული, ხოლო სისხლი კონცენტრირებული. 2-სა და 3-ს შორის რომ გააკეთო დიფერენციაცია, ამისათვის ავდმყოფს უსინჯავ ADH დონეს და თან აძლევ დამატებით ADH-ს. ცენტრალურის შემთხვევაში ADH მიცემის შემდეგ შარდის ოსლმოლარობა მაღლა აიწევს, რადგანაც ამ დროს უბრალოდ დეფიციტი გაქვს ჰორმონის. შესაბამისად ასეთ ადამიანებს ADH ძაან დაბალი აქვთ ან სულ არ აქვთ პლაზმაში. ნეფროგენულის დროს ADH-ს მიცემა არაფერს იზამს, რადგანაც ამათ ADH უკვე ნორმაში აქვთ (და უფრო ხშირად ნორმაზე მეტიც) და თირკმელი არ "გრძნობს" ჰორმონს.

ამრიგად, შენს კეისში 2 და 3 გამოირიცხა, მაგრამ ეს არ ნიშნავს რომ ამ ავადმყოფში პოლიურია/პოლიდიპსიის მიზეზი წყურვილის პირველადი გაძლიერებაა. ამ ავადმყოფს შაქრიანი დიაბეტიც აქვს და არაკონტროლირებული შაქრიანი დიაბეტი, როგორც პოლიურია პოლიდიპსიის მიზეზი, უნდა გამოირიცხოს. ამისათვის უნდა გამორიცხო გლუკოზურია და კეტონურია/კეტოაციდოზი. თუ ამ დიაბეტიკში დიაბეტი ნორმალურად არის ნაკონტროლები და შარდში არ გაქვს გლუკოზა და კეტომჟავები, მაშინ შეგიძლია თქვა, რომ მიზეზი არის პირველადი პოლიდიპსია. ფსიქიატრიულ ავადმყოფებში ხშირია პირველადი პოლიდიპსია. ეგენი ხშირად ეტანებიან დიდი რაოდნეობით წყალს. ზოგადად, არავინ არ გააკეთებს წყლის დეპრივაციის ტესტს უკონტროლო დიაბეტიკში: შემდგომმა დეჰიდრატაციამ შეიძლება კეტოაციდოზამდეც მიიყვანოს ავადმყოფი. ამიტომ შეიძლება თქვა რომ დიაბეტი არ არის ალბათ აქ მიზეზი და პირველად პოლიდიპსიაზეა საუბარი, მაგრამ ეს მაინც უნდა აღინიშნოს.

ფსიქიატრიულ ავადმყოფში უნდა გაითვალისწინო ლითიუმიც, რომელიც ნეფროგენულ უშაქრო დიაბეტს გაძლევს. ამ შემთხვევაში ნოფროგენულიც და ცენტრალურიც გამოირიცხა წყლის დეპრივაციის ტესტით.

Posted by: vano_t 20 Nov 2008, 06:43
ახლა დავაკვირდი შენს ამოცანაში მონაცემებს კიდევ და მთლად უფრო გაურკვევლად არის ყველაფერი. ჩემი პასუხი ეხებოდა შარდის მაღალ ოსმოლარობას (როცა შარდის ოსმოლარობა შრატის ოსმოლარობაზე 2-3 ჯერ მაღალია). მარ შენ ამაოცანაში იმენნა გაუგებრად წერია, რომ შარდის ოსმოლარობა მაღალიაო და >120-ზეო. მე მეგონა 1200-ზე მეტი გეწერა მაქ. თუ 120-ზე მეტია, მაშინ რამდენია? 120-ზე მეტია 700-ც, 800-ც, 150-ც, 200-ც. მოკლედ, თუ სისხლის ოსმოლარობაზე 2-3 ჯერ მეტია შარდის ოსმოლარობაზე წყლის დეპრივაციის შემდეგ, მაშინ პირველადი პოლიდიპსია გაქვს, ხოლო თუ შარდის ოსმოლარობა დაბალია (დავუშვათ 200-300 -ის ფარგლებში), მაშინ უშაქრო დიაბეტი გაქვს, ოღონდ ვერ დაადგენ ნეფროგენულია თუ ცენტრალური სანამ არ გაზომავ პლაზმის ADH და შარდის ოსმოლარობას ADH-ით სტიმულაციის შემდეგ. თუმცა, სავარაუდოდ, თუ ეს მონაცემები არ გაქვს და თავის ქალას ტრამვას არ ახსენებენ და ახსენებენ ბიპოლარულ დაავადებას, მაშინ სავარაუდოა ნეფროგენული უშაქრო დიაბეტი ლითიუმის გამო (ლითიუმი შემკრებ მილაკებს ხდის რეზისტენტულს ADH-ის მიმართ).

Posted by: vano_t 20 Nov 2008, 07:15
აგერ რეალური კეისი. 57 წლის მამრი ავამდყოფი მოდის წლიურ ფიზიკურ გამოკვლევაზე. ჩივილები არ აქვს. ოღონდ გამოკითხვის დროს გეუბნება, რომ ხანდახან აქვს ეპიზოდები, როცა გულის ფრიალი ეწყება უცებ და მთავრდება უცებ. ეს ეპიზოდები აქვს ბოლო 2 წლის განმავლობაში. ამ ეპიზოდებს არ ახლავს გულისტკილი და ქოშინი (არც მოსვენებისას და არც დატვირთვისას). ხანდახან, თუმცა, ცოტა თავრბუსხვევა აქვს ამ ეპიზოდებთან ერთად. ეპიზოდები ემართება თვეში 1-ხელ. რაც ავადმყოფმა შეამჩნია არის ის, რომ ხველის შემდეგ ეს ეპიზოდები ჩერდება და შესაბამისად, როცა ფრიალი ეწყება, ჩაახველებს და ეპიზოდების ტერმინაციას აკეთებს. ეს ყველაფერი თვითონ დაამუღამა.

ავადმყოფს არ აქვს რაიმე დატვირთული ოჯახური ისტორია.
თვითონ არ ეწევა და არ სვამს.
იღებს ლიპიტორს (ერთიერთი სტატინია) და ლევოთიროქსინს.
წარსულში ქონდა საზარდულის თიაქრის ქირურგია.

ფიზიკური გამოკვლევა ნორმალურია. პულსი ნორმალურია და რეგულარული. ტემპერატურა ნორმალური. წნევა ორივე მხარზე ნორმალურია და თითქმის ერთიდაიგივე. ფილტვების სუფთა აქვს; ნორმალური S1 და S2 ტონები აქვს. არ ისმინება S3/S4/შუილები/ხახუნი. საუღლე ვენები ნორმალურია. კაროტიდებზე შულიები არ ისმინება. რადიალური პულსები სიმეტრიულია და ნორმალური. ქვემო კიდურებზეც პერიფერული პულსები ნორმალურია და სიმეტრიული. კაპილარული შევსება (capillary refill) სწრაფია. ქვემო კიდურის შეშუპებები არ აქვს.

და დანარჩენი კითხვები თვითონ დასვით. ინფორმაციას კითხვის მიხედვით მოგაწოდებთ.

მოკლედ, რას უზამთ ამ ავდმყოფს და რატომ უზამთ? (ავადმყოფის საწოლში იმისქნა და ყოველგვარი დესპოტური რამეები არ იყო)

Posted by: basa-ttt 20 Nov 2008, 08:26
QUOTE
აგერ რეალური კეისი. 57 წლის მამრი ავამდყოფი მოდის წლიურ ფიზიკურ გამოკვლევაზე. ჩივილები არ აქვს. ოღონდ გამოკითხვის დროს გეუბნება, რომ ხანდახან აქვს ეპიზოდები, როცა გულის ფრიალი ეწყება უცებ და მთავრდება უცებ. ეს ეპიზოდები აქვს ბოლო 2 წლის განმავლობაში. ამ ეპიზოდებს არ ახლავს გულისტკილი და ქოშინი (არც მოსვენებისას და არც დატვირთვისას). ხანდახან, თუმცა, ცოტა თავრბუსხვევა აქვს ამ ეპიზოდებთან ერთად. ეპიზოდები ემართება თვეში 1-ხელ. რაც ავადმყოფმა შეამჩნია არის ის, რომ ხველის შემდეგ ეს ეპიზოდები ჩერდება და შესაბამისად, როცა ფრიალი ეწყება, ჩაახველებს და ეპიზოდების ტერმინაციას აკეთებს. ეს ყველაფერი თვითონ დაამუღამა.

ექსტრასისტოლების ეპიზოდებს ჰგავს ეგა -
და ჰოლტერი უნდა წესით
ან ველოერგომეტრია.
+ პროთრომბინის ინდექსი .

QUOTE
ლევოთიროქსინს
.
ამას რატომ სვამს? ვინ დაუნიშნა?
ფარისებრზე რა აქვს?

Posted by: vano_t 20 Nov 2008, 08:46
ბასა-ტტტ
QUOTE
QUOTE
ლევოთიროქსინს
.
ამას რატომ სვამს? ვინ დაუნიშნა?
ფარისებრზე რა აქვს?
ჰიპოთირეოზის გამო სვამს. რა არსებითი მნიშვნელობა აქვს იმას, რომ პაციენტი ლევოთიროქსინს სვამს?

QUOTE
ექსტრასისტოლების  ეპიზოდებს  ჰგავს ეგა -
და ჰოლტერი უნდა წესით
ან ველოერგომეტრია.
+ პროთრომბინის ინდექსი .
რა არის პროთრომბინის ინდექსი (პროთრომბინის დრო არ არის ნაღდად-პროცენტებში იზომება ეგა და იქიდან ვიცი) და კოაგულაციის ტესტი საერთოდ რა დახმარებას გაგიწევს? დავუშვათ გაუკეთე. მერე?

ჰოლტერი რას მოგცემს? ჰოლტერი 24 საათიანი ტესტია და ამ ავადმყოფს აქვს იშვიათი შეტევები. კი შეგიძლია ჰოლტერი გაატანო, მარა ძაან დაბალი შანსია, რომ ჰოლტერმა დაიჭიროს რაიმე. ხოდა, უარყოფითი ჰოლტერის შემთხვევაში რას აკეთებ? (მე არ დამინიშნია ჰოლტერი და ჯერ არ ქონია მაგ ავადმყოფს ეგ შეტევა უკვე 2 კვირაა-ანუ რომ დამენიშნა, სავარაუდოდ უარყოფითი პასუხი მოვიდოდა). ჰოლტერის მაგივრად ჯერ რა უნდა გაუკეთო ოფისში მარტივი და საკმაოდ ინფორმაციული? და მერე, ჰოლტერის მსგავსი რამ?

ველოერგომეტირა რა არის? ველოერგომეტრით დატვირთვის ტესტი? თუ ასეა, მაშინ რატომ გაუკეთებ ამას?

და რაც მთავარია, რატო ექსტრასისტოლები? ექსტრასისტოლები მოგცემს უეცრად დაწყებულ და უეცრად ჩამთავრებულ შეტევებს, რომელიც შეიძლება წუთობით გაგრძელდეს? დავუშვათ და ნახე ექსტრასისტოლა? აქ გაჩერდები და აღარ გააგრძელებ კვლევას?

Posted by: basa-ttt 20 Nov 2008, 09:06
vano_t
შეკითხვებს შენ აქეთ რატომ მისვამ?
1) რამდენია პროთრომბინის ინდექსი
2) ეკგ -ზე რაა + ჩასუნთქვის ტესტი
3) ჰიპოთირეოზის დიაგნოზი ვინ დაუსვა და რაა ფარისებრზე.
4) შინაგანი ორგანოები - განსაკუთრებით ნაღვლის ბუშტი გამოკვლეულია?

Posted by: vano_t 20 Nov 2008, 10:57
basa-ttt
QUOTE
შეკითხვებს შენ აქეთ რატომ მისვამ?
იმიტომ რომ ვერ ვხვდები რა კავშირი აქვს შენს კითხვებს ამ შემთხვევასთან. თუ აქვს კავშირი, მაშინ ამიხსენი. რა არის ამაში ცუდი? თუ არ მოგწონს შეკითხვები, მაშინ მე და შენ დავამთავროთ შეკითხვობანა. თუ მოგწონს, მაშინ გავაგრძელოთ.

QUOTE
1) რამდენია პროთრომბინის ინდექსი
რა არის მეთქი პროთრომბინის ინდექსი? ჩვენ არ ვზომავთ მასეთ რამეს. თუ გაინტერესებს პროთრომბინის დრო, მაშინ იგი ნორმალურია. ხოდა ისევ გეკითხები: დავუშვათ და მომატებულია პროთრომბინის დრო, ეგ რა დახმარებას გაგიწევს?

QUOTE
2)  ეკგ -ზე რაა  + ჩასუნთქვის ტესტი
ეკგ-ზე სინუსური რითმია და ნორმალურია. რაც მთავარია, 5 წლის წინ გაკეთებული ეკგ იგივეა და არ შეცვლილა. ჩასუნთქვის ტესტი რა არის?

QUOTE
3)  ჰიპოთირეოზის დიაგნოზი ვინ დაუსვა და რაა ფარისებრზე.
ჰიპოთირეოზი მე დავუსვი. ფარისებრზე არაფერია. აქვს სტანდარტული ჰიპოთირეოზი, აუტოიმუნური დაავადება (ალბათ ჰაშიმოტო).

QUOTE
4) შინაგანი ორგანოები - განსაკუთრებით ნაღვლის ბუშტი გამოკვლეულია?
ნაღვლის ბუშტი გამოკველეული არ არის და არ არის საჭირო. არ აქვს ავადმყოფს არავითარი გასტროინტესტინალური სიმპტომები. შენ თუ მეტვყი რატომ არის ნაღვლის ბუშტის გამოკვლევა საჭირო და დამისაბუთებ, პირობას გაძლევ მაგ ტესტსაც შევუკვეთავ.

Posted by: Blind_Torture_Kill 20 Nov 2008, 11:06
vano_t

QUOTE
a 30 y/o man has 1 week history of increased thirst, and increased urine volumes. the result of water deprivation test reveal increased >300mOsm/kg-plasma osmolality and elevated urine osmolality (>120mOsm/kg) he is currently receiving treatment for non-insulin-dependent diabetes mellitus (type II) as well as for bipolar disorder


გაზრდილი ოსმოლალობა შარდში ანუ მანამდე 120 მდე ქონდა და მერე მოიმატა (წყლის დეპრივაციის ტესტის შემდეგ)

იტოკში გამოიცანი
ლითიუმის ბრალი იყო ეგ ყველაფერი

შენ ექიმი ხარ ?
* * *
vano_t
ეკგ როგორია მაგ ტიპის

Posted by: vano_t 20 Nov 2008, 11:20
Guardian
QUOTE
ეგეთ პატარა გადიდებაზე ბირთვი მხოლოდ პატარა მუქი წერტილის სახით ჩანს.
ეგ ბირთვები არ არის - ეგ არის ჯირკვლოვანი ეპითელიუმის ერთი შრის შიგნით ჩახვეული ეპითელიუმის მეორე შრე.

რაც თქვი, საკმაოდ აზრიანი. ახლა კი წარმოვიდგინე. მაგრამ, მე რომ ვთქვი ბირთვები, ცარიელი ბირთვები კი არ მიგულისხმია შე კაცო. ბირთვს ციტოპლაზმა მოყვება ნატურში. ციტოპლაზმას ვერ გაარჩევ ასეთ გადიდებას. ბირთვს კიდევ დაინახავ. ანუ, ის ჩახვეული ეპითელიუმის ბირთვები ყოფილა ეგ იტოგში.

მასეთი ეპითელიუმი რომელ ჯირკვალ(ებ)ს აქვს? და საერთოდ რა არის მაშინ მაგ მიკროპრეპარატზე გამოსახული?

Blind_Torture_Kill
QUOTE
გაზრდილი ოსმოლალობა შარდში ანუ მანამდე 120 მდე ქონდა და მერე მოიმატა (წყლის დეპრივაციის ტესტის შემდეგ)
არა, მაგ კეისში ეგრე არ წერია-ძაან ორაზროვნად წერია ყველაფერი. მასეც რომ იყოს, სადამდე მოიმატა? თუ მოიმატა 800-მდე (ან 900-მდე და ა.შ.), მაშინ ვერ იქნება ლითიუმი. თუ მცირედ მოიმატა (დავუშვათ 150 ზე ავიდა), მაშინ შეიძლება ლითიუმია. ოღონდ, როგორვ ვთქვი, მარტო ამით ვერ დაადგენ ამას. ADH-ის სტიმულაციაც უნდა გააკეთო და პლაზმის ADH-იც გაზომო.

მე ექიმი ვარ.

QUOTE
ეკგ როგორია მაგ ტიპის
სინუსური რითმი: ბასა-ტტტ-ს დავუწერე ეკგ-ზე უკვე.

Posted by: Blind_Torture_Kill 20 Nov 2008, 11:31
QUOTE
არა, მაგ კეისში ეგრე არ წერია-ძაან ორაზროვნად წერია ყველაფერი. მასეც რომ იყოს, სადამდე მოიმატა? თუ მოიმატა 800-მდე (ან 900-მდე და ა.შ.), მაშინ ვერ იქნება ლითიუმი. თუ მცირედ მოიმატა (დავუშვათ 150 ზე ავიდა), მაშინ შეიძლება ლითიუმია. ოღონდ, როგორვ ვთქვი, მარტო ამით ვერ დაადგენ ამას. ADH-ის სტიმულაციაც უნდა გააკეთო და პლაზმის ADH-იც გაზომო.


ორაზროვნად იმიტო წერია რომ დაგაბნიოს ტესტი მაგისაა
მაშინ დაწერდნენ არც დიაბეტი აქვს
ხოდა რაც ლითიუმის ხმარება დაიწყო იმის მერე დაემართაო
800-900მდე რომ მომატებულიყო მაშინ დავწერდი მარა უმნიშვნელო ზრდა თან ესეთ განზავებულ შარდში 120 მილიოსმოლზე მეტი-12 საათიანი დეპრივაციის მერე პასუხს გაძლევს რომელიც გამოიცანი

ექოკარდიოგრაფიით თუ ჩანს რამე ?

Posted by: vano_t 20 Nov 2008, 11:52
Blind_Torture_Kill
QUOTE
ორაზროვნად იმიტო წერია რომ დაგაბნიოს ტესტი მაგისაა
მაშინ დაწერდნენ არც დიაბეტი აქვს
ხოდა რაც ლითიუმის ხმარება დაიწყო იმის მერე დაემართაო
800-900მდე რომ მომატებულიყო მაშინ დავწერდი მარა უმნიშვნელო ზრდა თან ესეთ განზავებულ შარდში 120 მილიოსმოლზე მეტი-12 საათიანი დეპრივაციის მერე პასუხს გაძლევს რომელიც გამოიცანი
ლითიუმის ხმარება კი არ უნდა იყოს ნახსენები. არამედ, დეპრივაციამდე და დეპრივაციის შემდეგ შარდის ოსმოლარობები უნდა იყოს მკვეთრად მოცემული-ისე მკვეთრად, როგორც ეს ხდება კლინიკაში როცა რაიმე ტესტს უკვეთავ. და ამასთანავე, უნდა გქონდეს ADH-ის დონეები პლაზმაში მოცემული, პლიუს ADH-ით სტიმულაციის შემდეგ შარდის ოსმოლარობები. სხვანაირად ცენტრალურ დიაბეტს ნეფროგენულისაგან ვერ განასხვავებ. სად ნახე ეგ ტესტი? თუ USMLE I-ის ან II-ის ტესტია, მაშინ მასეთ ტესტებს კლინიკასთან არაფერი აქვს საერთო, დამიჯერე. სულ სხვანაირია კლინიკური ტესტები (მაგალითად board-ის ან in-service ტესტები). კლინიკურ ტესტში მსგავსი ორაზროვნება და გაურკვევლობა არ იქნება.

და ვაფშე, რატო უნდა დაგვაბნიოს, რა დავაშავეთ? biggrin.gif

QUOTE
ექოკარდიოგრაფიით თუ ჩანს რამე ?
ექო არ გაკეთებულა ჯერ. ისე პროგრამაშია. ჯერ სხვა ტესტი შევუკვეთე. მე მაინტერესებს, უკვე არსებული მონაცემებიდან რისი ვარაუდი შეიძლება.

Posted by: basa-ttt 20 Nov 2008, 11:55
QUOTE
რა არის მეთქი პროთრომბინის ინდექსი? ჩვენ არ ვზომავთ მასეთ რამეს. თუ გაინტერესებს პროთრომბინის დრო, მაშინ იგი ნორმალურია. ხოდა ისევ გეკითხები: დავუშვათ და მომატებულია პროთრომბინის დრო, ეგ რა დახმარებას გაგიწევს?

შედედებას გვიჩვენებს, მარტივ იტესტია და მთელი კოაგულოგრამა საჭირო არაა-
Протромбиновый индекс (или индекс протромбина)
Определение индекса протромбина выражает отношение протромбинового времени здорового человека к протромбиновому времени больного. Выражается в процентах.
http://www.policlinica.ru/analiz4_7.html
მაი თქვენ ამერიკულ კეისებში არ ვიცი მაგას რა შეესაბამება.
იმიტომ გეკითხები, რომ უამრავი შემთხვევა მქონდა მე პირადად
ანალოგიური ჩივილებით (ფრიალი გულის არეში)
როცა მარტო ეს ინდექსი იყო მომატებული და სხვა ყველაფერი - ნორმა.



QUOTE
ჰიპოთირეოზი მე დავუსვი. ფარისებრზე არაფერია. აქვს სტანდარტული ჰიპოთირეოზი, აუტოიმუნური დაავადება (ალბათ ჰაშიმოტო).

QUOTE
ნაღვლის ბუშტი გამოკველეული არ არის და არ არის საჭირო

თუ ექსტრასისტოლას დაიჭერ -
და პარკუჭოვანი იქნება
(სავარაუდოდ პარკუჭოვანია -
ვინაიდან ავადმყოფი შეიგრძნობს მას-)
უნდა ეძებო ექსტრაკარდიალური მიზეზი.
ყველზე ხშირად ქართველებში ნაღვლის ბუშტია-
გვიყვარს გემრიელი საჭმელები.
დედაჩემს ჰქონდა კლასიკური ბიგემინია.
და როგორც კი ნაღვლის ბუშტის გამწმენდი დავუნიშნე (ჰომეოპათიური)
2 საათში მოეხსნა ბიგემინია და ფეხზე ადგა ქალი./

Posted by: Blind_Torture_Kill 20 Nov 2008, 12:02
QUOTE
ექო არ გაკეთებულა ჯერ. ისე პროგრამაშია. ჯერ სხვა ტესტი შევუკვეთე. მე მაინტერესებს, უკვე არსებული მონაცემებიდან რისი ვარაუდი შეიძლება.


მიქსომა
* * *
vano_t

რისი ექიმი ხარ ?

Posted by: vano_t 20 Nov 2008, 12:56
basa-ttt
QUOTE
შედედებას გვიჩვენებს, მარტივ იტესტია და მთელი კოაგულოგრამა საჭირო არაა-
Протромбиновый индекс (или индекс протромбина)
Определение индекса протромбина выражает отношение протромбинового времени здорового человека к протромбиновому времени больного. Выражается в процентах.
http://www.policlinica.ru/analiz4_7.html
მაი თქვენ ამერიკულ კეისებში არ ვიცი  მაგას  რა შეესაბამება.
იმიტომ  გეკითხები, რომ უამრავი შემთხვევა მქონდა მე პირადად
ანალოგიური ჩივილებით (ფრიალი გულის არეში)
როცა მარტო ეს ინდექსი იყო მომატებული და სხვა ყველაფერი  - ნორმა.
არავითარი კავშირი არ არსებობს. ეგ ყოფილა უბრალოდ გამონათვალი ისევე როგორც არის INR (Международное нормализованное отношение (МНО)), რომელიც ცდილობს პროთრომბინის დროს სტანდარტიზაციას. არ არის კარგი გამონათვალი და იმიტომაც არ ხმარობენ "მეჟდუნაროდები". ასეც და ისეც, პროთრომბინის დრო (და მისგან წარმოებული პროთრომბინის ინდექსი ან საერთაშორისო ნორმალიზებული შეფარდება) არ მოგცემს არავითარ ინფორმაციას. ეგ ტესტი იხმარება მხოლოდ არანორმალური სისლხდენების დიაგნოზისათვის და კუმადინის თერაპიული დოზის განსაზღვისათვის. საქართველოში, შემიძლია გითხრა, რომ მაგ ტესტს უმეტესად ყოველგვარი დანიშნულების გარეშე აკეთებენ-რაც არასწრია (მაგალითად, ღრმა ვენების თრომბოზების დიაგნოსტიკისათვის, რაც სავსებით არარაციონალურია).

QUOTE
QUOTE
ნაღვლის ბუშტი გამოკველეული არ არის და არ არის საჭირო

თუ ექსტრასისტოლას დაიჭერ -
და პარკუჭოვანი იქნება
(სავარაუდოდ პარკუჭოვანია -
ვინაიდან ავადმყოფი შეიგრძნობს მას-)
უნდა ეძებო ექსტრაკარდიალური მიზეზი.
ყველზე ხშირად ქართველებში ნაღვლის ბუშტია-
გვიყვარს გემრიელი საჭმელები.
დედაჩემს ჰქონდა კლასიკური ბიგემინია.
და როგორც კი ნაღვლის ბუშტის გამწმენდი დავუნიშნე (ჰომეოპათიური)
2 საათში მოეხსნა ბიგემინია და ფეხზე ადგა ქალი./
მოდი, ძალიან გთხოვ, არ გვინდა ეს ჰომეოპათია აქ. აქ ვისაუბროთ ჩვეულებრივ მედიცინაზე. მასე სიმელოტეც შეიძლება იწვევდეს არითმიებს. ან კიდევ კეხიანი ცხვირი. მარა, ასეთ რამეები ჯერ არ დაუდგენია მედიცინას. ხოდა, რაც იციან იმის ფარგლებში ვისაუბროთ. ხოდა, ამიტომ მსგავსი პასუხი არ მიიღება. თუ ნაღვლის ბუშტის დაავადებებსა და არითმიებს შორის რაიმე კავშირი არსებობს, მაშინ უშუალოდ მიმითითე ასეთ კავშირზე. მანამდე ავადმყოფს არ დაენიშნება ნაღვლის ბუშტის გამოკვლევა.

რატოა საერთოდ პარკუჭოვანი? ან რატოა ექსტრასისტოლა? ხომ გკითხე, როგორ ხსნის იმ ფაქტს, რომ ავადმყოფს შეტევები ეწყება უცებ და უმთავრდება უცებ. თანაც შეტევა რამოდენიმე ხანი გრძელდება. ექსტრასისტოლები ამას გაძლევენ? მეორეც, არ შეიძლება წინაგულოვანმა რამეებმა მოგცეს თავბრუსხვევა?

Blind_Torture_Kill
QUOTE
მიქსომა
მიქსომას როგორ აკავშირებ ამ ყველაფერთან? (ჯერ ექო არ გამიკეთებია ისე)

მე კიდევ ინტერნისტი ვარ.

Posted by: Blind_Torture_Kill 20 Nov 2008, 13:29
vano_t
სავარაუდო დიაგნოზი ვთქვი

წნევები ნორმალური,სარქვლოვანი დაავადებები არაა,ეკგ ნორმალური

გულის ფრიალი + თავბრუსხვევა ისიც ხანდახან (თვეში ერთხელ)

ექოს პასუხის მერე გეტყვი biggrin.gif დიაგნოზს

Posted by: basa-ttt 20 Nov 2008, 13:31
QUOTE
თუ ნაღვლის ბუშტის დაავადებებსა და არითმიებს შორის რაიმე კავშირი არსებობს, მაშინ უშუალოდ მიმითითე ასეთ კავშირზე

ექსტრაკარდიალური მიზეზი ექსტრასისტოლების აბა რაა?
ის რომ პაციენტი ჩახველებით ასწორებს ფრიალს-
სწორედ ექსტრასისტოლაზე მიუთითებს ირიბად.

QUOTE
რატოა საერთოდ პარკუჭოვანი?

ხომ გითხარი -
შეგრძნებას იძლევა უფრო ხშირად პარკუჭოვანი, რომელიც არის უფრო ხშირად ექსტრაკარდიალური.
QUOTE
არ შეიძლება წინაგულოვანმა რამეებმა მოგცეს თავბრუსხვევა?

თაბრუსხვევა კი -
გულის ფრიალი ნაკლებად.
მაშინ იშემიური დაავადება უნდა გქონდეს და ეს კი ველოზე გამოჩნდება თუ არ ჩანს ეკგ- ზე.


QUOTE
ასეც და ისეც, პროთრომბინის დრო (და მისგან წარმოებული პროთრომბინის ინდექსი ან საერთაშორისო ნორმალიზებული შეფარდება) არ მოგცემს არავითარ ინფორმაციას.

მე მაძლევს, და როცა გამომისწორებია ეს მაჩვენებელი -
პაციენტი კარგადაც გამხდარა-
ალბათ გასაგებია, რომ ამისათვის ჩემს წამლებს ვხმარობ და არა ასპირინს.
* * *
Из нарушений ритма сердца для гипотиреоза наиболее характерна брадикардия: она встречается у 30-60% больных. Однако развившийся на фоне ИБС и кардиосклероза гипотиреоз может сопровождаться тахикардией (10% больных), суправентрикулярной или вентрикулярной экстрасистолией (24% больных) и даже мерцательной аритмией. Такие нетипичные для гипотиреоза нарушения ритма сердца и являются причиной несвоевременной диагностики этого состояния.

http://www.medlinks.ru/article.php?sid=33491

აი კიდევ ერთი მიზეზი ექსტრასისტოლიის (თუ კი ის არსებობს ამ შემთხვევაში)

Posted by: vano_t 20 Nov 2008, 14:24
basa-ttt
QUOTE
ექსტრაკარდიალური  მიზეზი  ექსტრასისტოლების აბა  რაა?
ის რომ პაციენტი ჩახველებით ასწორებს ფრიალს-
სწორედ ექსტრასისტოლაზე მიუთითებს ირიბად.
რაც არ უნდა იყოს მიზეზი, ის ბოლოს გულის გამტარი სისტემის ცვლილებას იწვევს. თანაც, ის რომ ჩახველებით შეტევის დამთავრება ხდება სულ სხვა რამეს მიუთითებს. ეგ არის რეფლექსი და ქვია თავისი სახელი. ხოდა, ამ რეფლექსს თუ იცნობ, მაშინ აღარ იტვყი რომ პარკუჭოვანია. შესაძლებელი არის რომ პარკუჭოვანი იყოს, მაგრამ ნაკლებსავარაუდო. შენ ექსტრაკარდიალურ მიზეზს რას ეძახი არ ვიცი, მარა, შენ მითხარი რა ტესტები გაინტერესებს და თუ ჩატარებული აქვს, ტესტების პასუხს გეტყვი.

QUOTE
QUOTE
არ შეიძლება წინაგულოვანმა რამეებმა მოგცეს თავბრუსხვევა?

თაბრუსხვევა კი -
გულის ფრიალი ნაკლებად.
მაშინ იშემიური დაავადება უნდა გქონდეს და ეს კი ველოზე გამოჩნდება თუ არ ჩანს ეკგ- ზე.
რას ამბობ? გულს ფრიალს მოგცემს ნებისმიერი პაროქსიზმული ტაქიკარდია. და თავბრუსხვევა პირიქით, უფრო მიუთითებს ვენტრიკულურზე, თუ გულის დაავადება არ გაქვს თან (იშემიური არ არის აუცილებელი). თავბრუსხვევა გამოწვეულია წუთმოცულობის დაქვეითებით, რაც უფრო მოხდება ვენტრიკულური ტაქიკარდიების დროს ან დაავადებულ გულში წინაგულოვანი ტაქიკარდიების დროსაც. მარა, ჯანმრთელ ადამიანსაც შეიძლება დაემართოს, განსაკუთრებით როცა ღელავს ამ დროს და ჰიპერვენტილაცია აქვს.

QUOTE
QUOTE
რატოა საერთოდ პარკუჭოვანი?

ხომ გითხარი -
შეგრძნებას იძლევა უფრო ხშირად პარკუჭოვანი, რომელიც არის უფრო ხშირად ექსტრაკარდიალური.
მეც გითხარი, რომ არ არის მართალი.

QUOTE
QUOTE
ასეც და ისეც, პროთრომბინის დრო (და მისგან წარმოებული პროთრომბინის ინდექსი ან საერთაშორისო ნორმალიზებული შეფარდება) არ მოგცემს არავითარ ინფორმაციას.
მე მაძლევს, და როცა გამომისწორებია ეს მაჩვენებელი -
პაციენტი კარგადაც გამხდარა-
ალბათ გასაგებია, რომ ამისათვის ჩემს წამლებს ვხმარობ და არა ასპირინს.
შენ განსაკუთრებულ მედიცინას თუ მისდევ არ ვიცი. ამიხსენი, რატომ გაძლევს შენ და რატომ არ მაძლევს მე? ან საერთოდ როგორ შეიძლება გაასწორო პროთრომბინის დრო? აბა მითხარი, რას წარმოადგენს პროთრომბინის დრო და რას ზომავს? დანარჩენი მერე ვილაპარაკოთ.

QUOTE
Из нарушений ритма сердца для гипотиреоза наиболее характерна брадикардия: она встречается у 30-60% больных. Однако развившийся на фоне ИБС и кардиосклероза гипотиреоз может сопровождаться тахикардией (10% больных), суправентрикулярной или вентрикулярной экстрасистолией (24% больных) и даже мерцательной аритмией. Такие нетипичные для гипотиреоза нарушения ритма сердца и являются причиной несвоевременной диагностики этого состояния.

http://www.medlinks.ru/article.php?sid=33491

აი კიდევ ერთი მიზეზი ექსტრასისტოლიის  (თუ კი ის არსებობს ამ შემთხვევაში)
რატომ მოგყავს არატიპიური მაგალითი, როცა ამ დროს ტიპიური შეიძლება გქონდეს. რას მოგცემს ჰიპერთირეოზი და შეიძლება თუ არა მოცემულ ავადმყოფს ქონდეს ჰიპერთირეოზი? ასეც და ისეც (გინდა ტიპიური იყოს და გინდა არატიპიური) ხომ უნდა დაადგინო, რომ ფარისებური ჯირკვალია მიზეზი?

Blind_Torture_Kill
QUOTE
სავარაუდო დიაგნოზი ვთქვი

წნევები ნორმალური,სარქვლოვანი დაავადებები არაა,ეკგ ნორმალური

გულის ფრიალი + თავბრუსხვევა ისიც ხანდახან (თვეში ერთხელ)

ექოს პასუხის მერე გეტყვი biggrin.gif დიაგნოზს
კაი მაშვინ, დაველოდოთ ექოს biggrin.gif

მე კიდე წავედი ახლა დასაძინებლად და აბა თქვენ იცით.

Posted by: basa-ttt 20 Nov 2008, 14:30
QUOTE
რაც არ უნდა იყოს მიზეზი, ის ბოლოს გულის გამტარი სისტემის ცვლილებას იწვევს. თანაც, ის რომ ჩახველებით შეტევის დამთავრება ხდება სულ სხვა რამეს მიუთითებს. ეგ არის რეფლექსი და ქვია თავისი სახელი. ხოდა, ამ რეფლექსს თუ იცნობ, მაშინ აღარ იტვყი რომ პარკუჭოვანია. შესაძლებელი არის რომ პარკუჭოვანი იყოს, მაგრამ ნაკლებსავარაუდო.

მე ჩემი გითხარით -
დანარჩენი თქვენი საქმისა თქვენ იცით.

უბრალოდ უთხარით, რომ როცა ასეთი შეტევები ექნება-
მაჯაზე დაიდოს ხელი
და ნახოს როგორი პულსი აქვს.

Posted by: Blind_Torture_Kill 20 Nov 2008, 14:39
ჰო კიდე ჰიპერთირეოზი ვერ ექნება პაციენტს ლევოთიროქსინზე ზის (თუ თვეში ერთხელ დოზას არ ამეტებს biggrin.gif)

basa-ttt

შენი სავარაუდო დიაგნოზი რა არის ?

Posted by: basa-ttt 20 Nov 2008, 14:44
QUOTE
შენი სავარაუდო დიაგნოზი რა არის ?

წინასწარ დიაგნოზს არასოდეს ვამბობ-
მაგრამ
თუ ასეთი შეტევის დროს პაციენტს პულსი ეცვლება - ექსტრასისტოლაა.
თუ რა ტიპის - ეს ბევრ რამეზეა დამოკიდებული.

Posted by: Blind_Torture_Kill 20 Nov 2008, 14:57
მეც წავედი რამეს გავაკეთებ მანამდე


Posted by: vano_t 20 Nov 2008, 21:01
Blind_Torture_Kill
QUOTE
ჰო კიდე ჰიპერთირეოზი ვერ ექნება პაციენტს ლევოთიროქსინზე ზის (თუ თვეში ერთხელ დოზას არ ამეტებს biggrin.gif)
ო, მაქ ცდები. ნებისმიერ დროს შეიძლება განუვითარდეს ჰიპერთირეოზი პაციენტს, როცა თიროიდულ ჰორმონს იღებს, მიუხედავად იმისა აჭარბებს თუ არა დანიშნულ დოზას. ამიტომ, TSH შემოწმება აუცილებელია. TSH ისეც ამოწმებენ არითმიების დროს ყოველთვის. ამ კონკრეტული პაციენტის შემთხვევაში ნორმაში იყო TSH, რაც თითქმის გამორიცხავს წამლისმიერ ჰიპერთირეოზს.

basa-ttt
QUOTE
წინასწარ დიაგნოზს არასოდეს ვამბობ-
მაგრამ
თუ ასეთი შეტევის დროს პაციენტს პულსი ეცვლება - ექსტრასისტოლაა.
როგორ ადგენ პულსის მიხედვით ექსტრასისტოლას?

მოცემული ავადმყოფის შემთხვევაში, ავადმყოფს აქვს შეგრძნება, რომ გული სწრაფად ცემს შეტევის დროს.

კიდევ ერთხელ დავსვამ ამ კითხვას: ხველის დროს რომ გულის აჩქარების დამთავრება ხდება, რა ქვია მაგას და რაზე მიუთითებს?

Posted by: basa-ttt 20 Nov 2008, 21:20
QUOTE
მოცემული ავადმყოფის შემთხვევაში, ავადმყოფს აქვს შეგრძნება, რომ გული სწრაფად ცემს შეტევის დროს.

ვაიმე დედა
gigi.gif
ხომ მიხვდება პაციენტი პულსი რითმულია თუ არა?
მაჯა უნდა ნახოს.
თუ რითმულია პაროქსიზმია
თუ არითმული ექსტრასისტოლაა.

QUOTE
კიდევ ერთხელ დავსვამ ამ კითხვას: ხველის დროს რომ გულის აჩქარების დამთავრება ხდება, რა ქვია მაგას და რაზე მიუთითებს

gigi.gif
არითმიაზე.
მე პირადად როცა ექსტრასისტოლები მქონია
ჩახველებას ითხოვს.
თუ პაროქსიზმია
ღრმა ჩასუთქვა უნდა გააკეთოს და სუნთქვა შეიკავოს -
ანუ ვაგუსი გააღიზიანოს
და მოიხსნება


Posted by: vano_t 20 Nov 2008, 22:18
ბასა-ტტტ
QUOTE
ხომ მიხვდება პაციენტი პულსი რითმულია თუ არა?
მაჯა უნდა ნახოს.
თუ რითმულია პაროქსიზმია
თუ არითმული  ექსტრასისტოლაა.
ავამდყოფი კი არადა, გამოცდილი ექიმიც ვერ მიხვდება მაგას თუ ელექტროკარდიოგრამა არ ნახა მაგ დროს. პაროქსიზმი ნიშნავს სწრაფად დაწყებულ და სწრაფად დამთავრებულ შეტევას. ავადმყოფს შეიძლება დაეწყოს წინაგულების ფიბრილაცია სწრაფი ვენტრიკულური გამტარებლობით, რაც შეიძლება პაროქსიზმული იყოს. ამ დროს გექნება არითმული პულსი, რაც შენს წინადადებას უკვე უარყოფს. ასევე, ვენტრიკულრი ტაქიკარდიების დროს ზოგი კოპმლექსი საერთოდ არ მოგცემს პულსს (პულსის დეფიციტი) და შესაბამისად მოგცემს არითმულ პულსს.

ასევე, შეიძლება გქონდეს პარკუჭოვანი ბიგემინია (რაც ექსტრასისტოლებია) ისე რომ დავუშვათ ნორმალურიR-ექსტრასისტოლისR ინტერვალსა და ექსტრასისტოლისR-ნორმალურიR ინტერვალებს შორის მცირე განსხვავება იყოს, ისე რომ პულსით ვერაფერი დაადგინო. შესაბამისად, ექსტრასისტოლები გექნება პულსით ყოველგვარი რითმის დარღვევის გარეშე. ან შეიძლება ყოველ ექსტრასისტოლაზე პულსის დეფიციტი გქონდეს და ისევ რითმულ პულსს მოგცემს. ანუ, რითმული პულსი არ ნიშნავს ექსტრასისტოლას.

ე.ი. რითმული პულსი არ ნიშნავს ექსტრასისტოლას და არითმული პულსი არ ნიშნავს რომ სუპრავენტრიკულური ტაქიკარდია არ გექნება.

QUOTE
QUOTE
კიდევ ერთხელ დავსვამ ამ კითხვას: ხველის დროს რომ გულის აჩქარების დამთავრება ხდება, რა ქვია მაგას და რაზე მიუთითებს

gigi.gif
არითმიაზე.
მე პირადად როცა ექსტრასისტოლები მქონია
ჩახველებას ითხოვს.
თუ პაროქსიზმია
ღრმა ჩასუთქვა უნდა გააკეთოს და სუნთქვა შეიკავოს -
ანუ ვაგუსი გააღიზიანოს
და მოიხსნება
სადღაც ახლო ხარ პასუხთან, მაგრამ მაინც ვერ პასუხობ დალაგებულად. მოკლედ, ჩახველება და ამოსუნთქვის მცდელობა დახურული ეპიგლოტის დროს (ანუ გაჭინთვა) და არა სუნთქვის შეკავება, წარმოადგენს ე.წ. ვალსალვას პროცედურის ვარიაციებს. ორივეს დროს ხდება გულმკერდის ღრუში წნევის მომატება და რეფლექსური ბრადიკარდია.

ვალსალვას პროცედურას იყენებენ წინაგულოვანი ტაქიკარდიების დროს ტაქიკარდიის მოსახსნელად.

მოკლედ, ამ ავადმყოფს დანიშნული აქვს event monitor, რაც ჰოლტერის მსგავსი რამ არის. მარა, ჰოლტერი გამუდმებულ ჩაწერას აკეთებს და დიდი ხნით ამის პაციენტზე დადება ძალიან არახელსაყრელია და თანაც ძვირი. 48 საათზე მეტი იშვიათად კეთდება. event monitor აქტივირდება მხოლოდ პაციენტის მიერ, როცა სიმპტომები ეწყება მაშინ. ამ დროს პაციენტი დააწვება ღილაკს და გააქტიურებს ხელსაწყოს. ამიტომ, როცა იშვიათი სიმპტომები აქვთ, ეს გაცილებით ხელსაყრელი სადიაგნოსტიკო მეთოდია.

ისე დიაგნოზის შემდეგ საქმის ყურში ჩაგაყენებთ.

P.S. ავადმყოფი შემწმებულია ყველა ელექტროლიტზე (კალიუმი, მაგნიუმი, კალციუმი, ნატრიუმი და ა.შ.) რამაც შეიძლება მოგცეს გულის ელექტროფიზიოლოგიის შეცვლა. ეს ყველაფერი ნორმაშია. ექოკარდიოგრამ ნამდვილად უნდა დატვირთვის ტესტთან ერთად და ესენიც გაუკეთდება.

Posted by: basa-ttt 20 Nov 2008, 22:21
QUOTE
ავამდყოფი კი არადა, გამოცდილი ექიმიც ვერ მიხვდება მაგას თუ ელექტროკარდიოგრამა არ ნახა მაგ დროს. პაროქსიზმი ნიშნავს სწრაფად დაწყებულ და სწრაფად დამთავრებულ შეტევას. ავადმყოფს შეიძლება დაეწყოს წინაგულების ფიბრილაცია სწრაფი ვენტრიკულური გამტარებლობით, რაც შეიძლება პაროქსიზმული იყოს. ამ დროს გექნება არითმული პულსი, რაც შენს წინადადებას უკვე უარყოფს. ასევე, ვენტრიკულრი ტაქიკარდიების დროს ზოგი კოპმლექსი საერთოდ არ მოგცემს პულსს (პულსის დეფიციტი) და შესაბამისად მოგცემს არითმულ პულსს.

არ უნდა ამდენი ფილოსოფია
ადვილი გასარკვევია მაგი მაჯით.

QUOTE
ავადმყოფი შემწმებულია ყველა ელექტროლიტზე (კალიუმი, მაგნიუმი, კალციუმი, ნატრიუმი და ა.შ.)

არ გადარიო პაციენტი
gigi.gif

Posted by: zviadcardio 20 Nov 2008, 22:31
რომ არ დავწვრილმანდეთ ყველაზე მაღალი ალბათობით მას ემართება SVT (AVNRT ან AVRT)

Posted by: tribal 20 Nov 2008, 22:33
რად უნდა მაგას ბევრი ბაზარი. სავარაუდოდ სუპრავენტრიკულური პაროქსიზმები აქვს მაგ ტიპს. ნუ პარკუჭოვანიც შეიძლება, მაგრამ უფრო ნაკლები შანსია. ივენტ მონიტორი დაიჭერს მაგას. ნუ თუ არ დაიჭერს, მერე ელექტროფიზიოლოგიური გამოკვლევა უნდა. რა პროთრომბინის ინდექსი, რის პროთრომბინის ინდექსი. ეგ ქართული სინამდვილის ტრაგედიაა, ყველაფერზე "პროთრომბინის" განსაზღვრა და მერე ამით რაღაც მკურნალობების დანიშვნა.

Posted by: vano_t 20 Nov 2008, 22:56
tribal
QUOTE
რად უნდა მაგას ბევრი ბაზარი. სავარაუდოდ სუპრავენტრიკულური პაროქსიზმები აქვს მაგ ტიპს. ნუ პარკუჭოვანიც შეიძლება, მაგრამ უფრო ნაკლები შანსია. ივენტ მონიტორი დაიჭერს მაგას. ნუ თუ არ დაიჭერს, მერე ელექტროფიზიოლოგიური გამოკვლევა უნდა. რა პროთრომბინის ინდექსი, რის პროთრომბინის ინდექსი. ეგ ქართული სინამდვილის ტრაგედიაა, ყველაფერზე "პროთრომბინის" განსაზღვრა და მერე ამით რაღაც მკურნალობების დანიშვნა.
ასპროცენტიანი ხარ (შენ და ზვიადკარდიომ ეტყობა ძაან კარგად იცით კარდიოლოგია), მარა აბა ბასა-ტტტ მედავება და რა ვქნა?

აგერ თურმე ნაღვლის ბუშტის ამბებს ექსტრასისტოლები და რაღაცეები ცოდნია.

პროთრომბინს ყველას უკვეთავენ და არ იციან ტესტი რას წარმოადგენს და რას ზომავს. მართლა პრობლემაა ეგ საქართველოში. მარა, ახალი სტუდენტები რომ მოდიან, ბევრმა ძაან კარგად იცის უკვე ეგ ყველაფერი. ამ ფორუმზეც არიან საკმაო რაოდნებოით ახალგაზრდები, რომელთაც კარგად ესმით მედიცინა. ასეთი ტოპიკები კიდევ წაეხმარებათ ბევრს. გარდიანმა კარგი საქმე გააკეთა იტოგში.

Posted by: zviadcardio 21 Nov 2008, 01:10
QUOTE
პროთრომბინს ყველას უკვეთავენ და არ იციან ტესტი რას წარმოადგენს და რას ზომავს.

"პროთრომბინის ანალიზის მითი", ეს Most Popular ანალიზია.
ყველაზე საინტერესო ისაა, რომ ხშირად პაციენტები თვითონ იზომავენ (კაცმა არ იცის რისთვის)

Posted by: basa-ttt 21 Nov 2008, 01:24
QUOTE
"პროთრომბინის ანალიზის მითი", 

რატომ მითი?
როცა რეალურად ყოველთვის
ამ მაჩვენებლის მატებას თან ახლავს რეალური ჩივილები,
რომლებსაც ექიმები აღარ აქცევენ ყურადღებას -
და რომლებიც პაციენტს გარკვეულ დისკომფორტს უქმნის.
ერთ ერთი ირიბი მაჩვენებელია შედედების მომატების.
მატულობს ძირითადად იმათში,
ვისაც მიდრეკილება აქვს ვარიკოზული გაგანიერებისაკენ.
მე მინახავს 12 წლის ბავშვი -
გულის არეში ფრიალით -
რომელსაც მხოლოდ ეს ინდექსი ჰქონდა მომატებული -
და რომელიც მოხვდა ბავშვთა კარდიოლოგთან ბ\რესპუბლკურში,
მადლობა ღმერთს, ჭკვიანი კარდიოლოგი შეხვდა და ეკგ - ს რომ დახედა
ეგრევე დაუნიშნა ეს გამოკვლევა.
თუ მიხვდებით რა ცვლილებები ჰქონდა ეკგ -ზე.
(ამ ბავშვის მამას ვენების ვარიკოზული გაგანიერება ჰქონდა.)

Posted by: vano_t 21 Nov 2008, 01:41
zviadcardio
QUOTE
QUOTE
პროთრომბინს ყველას უკვეთავენ და არ იციან ტესტი რას წარმოადგენს და რას ზომავს.

"პროთრომბინის ანალიზის მითი", ეს Most Popular ანალიზია.
ყველაზე საინტერესო ისაა, რომ ხშირად პაციენტები თვითონ იზომავენ (კაცმა არ იცის რისთვის)

"ექიმო, პროთრომბინი გამიზომე-რაღაც ვერ ვარ ხასიათზე", ეგრე ხო smile.gif

ბასა-ტტტ
QUOTE
ერთ ერთი ირიბი მაჩვენებელია შედედების მომატების.
აი ეს არის ერთ-ერთი ყველაზე ხშირი მითი. პროთრომბინის დრო მხოლოდ და მხოლოდ მოგცემს ინფორმაციას სისხლის შედედების დაკლებაზე. როცა პროთრომბინის დროს მომატებულია, ეს ნიშნავს, რომ სისხლის შედედება ვერ ხდება კარგად და ავადმყოფებს აქვთ სისხლდენებისაკენ მიდრეკილება! პროთრომბინის დროის დაკლება (თუ არსებობს ასეთი რამ) არაფერს ინფორმაციას არ გაძლევს. არანორმალური თრობმბოზი თუ აქვს ავადმყოფს (მაგალითად ღრმა ვენების თრომბოზი) არავითარი ცვლა არ ხდება ამ მონაცემის! და თრომბოზების მიზეზის გამოსაკვლევად უნდა შეუკვეთო სპეციალური ტესტები: პროტენი s, პროტეინ c, ანტითრომბინი III და ლეიდენის ტიპის მე-5 ფაქტორი. თუ რა თქმა უნდა ავადმყოფს აშკარა მიზეზი აქვს თრომბოზისა (მაგალითად ქვემო კიდურების ქირურგია), მაშინ თავიდან ასეთი ტესტები შეიძლება არ დაჭირდეს. ეს ტესტები მე მგონი საერთოდ არ კეთდება საქართველოში.

სიმართლე გითხრა, იმისთვის არ ვწერ, რომ შენ დაიჯერებ ამას და გაიგებ. არამედ იმისთვის, რომ ახალგაზრდა სტიდენტებიც კითხულობენ აქ და მასეთი არარაციონალური მიდგომა იმათაც რომ არ გადაედოთ.

QUOTE
მადლობა ღმერთს, ჭკვიანი კარდიოლოგი შეხვდა და ეკგ - ს რომ დახედა
ეგრევე დაუნიშნა ეს გამოკვლევა.
ბევრჯერ სიჩუმე ჯობს თქმას.

Posted by: basa-ttt 21 Nov 2008, 01:51
Протромбин – сложный белок, один из важнейших показателей коагулограммы, характеризующий состояние свертывающей системы крови. Протромбин или фактор II свертывания крови – это предшественник тромбина (белка, стимулирующего образование тромба).

Протромбин синтезируется в печени при участии витамина К. На основании анализа протромбинов врач может оценить работу и выявить заболевания печени и желудочно-кишечного тракта.

Для характеристики системы гемостаза (свертывающей системы крови) анализ протромбинов крови является наиболее важным тестом, входящим в гемостазиограмму.

Нормы протромбина: 78 - 142 % (анализ протромбина крови по Квику).

Повышенный протромбин – симптом следующих заболеваний:

тромбоэмболия
инфаркт миокарда, предынфарктное состояние
полицитемия
злокачественные опухоли.
Низкий протромбин в крови дает врачу основание поставить следующий диагноз:

врожденный или приобретенный дефицит протромбина
нехватка витамина К.
QUOTE
არავითარი ცვლა არ ხდება ამ მონაცემის!

რანაირად არ ხდება, როცა უამრავი ანალიზი მინახავს სწორედ ამ შეცვლილი მონაცემებით -
საიდან მოიტანე რომ არ ხდება?


QUOTE
ბევრჯერ სიჩუმე ჯობს თქმას.

როცა ვერ ვამბობთ ხომ?
გინდა გასწავლი იმ კარდიოლოგის მისამართს? -
ბავშვთა კარდიოლოგიაში მე მაგაზე უკეთესი სპეციალისტი საქართველოში არ მეგულება.

QUOTE
სიმართლე გითხრა, იმისთვის არ ვწერ, რომ შენ დაიჯერებ ამას და გაიგებ. არამედ იმისთვის, რომ ახალგაზრდა სტიდენტებიც კითხულობენ აქ და მასეთი არარაციონალური მიდგომა იმათაც რომ არ გადაედოთ.

შენ უკვე 40 წლის სამედიცინო გამოცდილება გაქვს ხომ მხრებზე აკიდებული?
როდის მერე გახდი ძველგაზრდა?
* * *
QUOTE
პროთრომბინის დრო მხოლოდ და მხოლოდ მოგცემს ინფორმაციას სისხლის შედედების დაკლებაზე. როცა პროთრომბინის დროს მომატებულია, ეს ნიშნავს, რომ სისხლის შედედება ვერ ხდება კარგად და ავადმყოფებს აქვთ სისხლდენებისაკენ მიდრეკილება

მართალია-
ამ დროს ინდ. მაჩვენებელი ნორმაზე ნაკლებია.


QUOTE
პროთრომბინის დროის დაკლება (თუ არსებობს ასეთი რამ) არაფერს ინფორმაციას არ გაძლევს.

თუ სწრაფად ხდება კოლტის წარმოქნა?
Протромбиновое время
Отражает время свертывания плазмы крови, выражается в секундах.




* * *
დავუშვათ რომ არაა ჯერ ჩამოყალიბებული ვენების ვარიკოზული გაგანიერება,
და ჯერ არაა ჩამოყალიბებული თრომბოზი-
რა ცვლილებები უსწრებს წინ პათოლოგიას, როცა
ადამიანის მდგომარეობა არის ჯანმრთელობის და ავადმყოფობის საზღვარზე?
ანუ არც ავადმყოფად ჩაითვლება, მაგრამ
უკვე არის ცვლილებები ბიოქიმიის დონეზე?
ანუ პირველად რა მაჩვენებელები იცვლება ორგანიზმში -
სანამ ჩამოყალიბდება ვარიკოზული თრომბოზი?

Posted by: vano_t 21 Nov 2008, 02:25
basa-ttt
QUOTE
QUOTE
სიმართლე გითხრა, იმისთვის არ ვწერ, რომ შენ დაიჯერებ ამას და გაიგებ. არამედ იმისთვის, რომ ახალგაზრდა სტიდენტებიც კითხულობენ აქ და მასეთი არარაციონალური მიდგომა იმათაც რომ არ გადაედოთ.

შენ უკვე 40 წლის სამედიცინო გამოცდილება გაქვს ხომ მხრებზე აკიდებული?
როდის მერე გახდი ძველგაზრდა?
ძველგაზრდა არ ვარ, მაგრამ 9 წელი პრაქტიკაში ვარ. მე მგონი საკმაო გამოცდილებაა ექიმისათვის.

QUOTE
Нормы протромбина: 78 - 142 % (анализ протромбина крови по Квику).
შენ ეტყობა შენსვე დადებულ ლინკებს არ კითხულობ. http://www.policlinica.ru/analiz4_7.html
QUOTE
Протромбин по Квику

А. Quik в 1935г предложил этот тест с целью анализа свертываемости крови по определению уровня протромбина. Анализ отображает активность протромбина в процентах, выявленную по калибровочному графику, построенному в результате измерения протромбинового времени в разведенных растворах нормальной плазмы. На сегодняшний день этот тест самый популярный метод анализа протромбина.
როგორ ფიქრობ, 1935 წელს შექმნილი ტესტი დღესაც აქტუალურია? მაგ ტესტს და თიმოლის სინჯებს (ასევე ოდინდელი ტესტები) ყოფილი საბჭოთა კავშირის ქვეყნების გარდა არსად ხმარობენ მსოფლიოში. ეგ ტესტები არიან არასენსიტიური, არასპეციფიური, არაზუსტი ტესტები. დროა შევუერდეთ მოწინავე მსოფლიოს მედიცინას.

Posted by: basa-ttt 21 Nov 2008, 02:33
vano_t
არც შენ კითხულობ
მე ვლაპარაკობ არა პროთრომბინის დროზე
არამედ შეფარდებით ინდექსზე,
რომელიც სწორედ რომ კლებულობს როცა პროთრომბინის დრო მატულობს.
ანუ პირიქით.
მაგრამ მე სხვა რამეს ვამბობ.


QUOTE
როგორ ფიქრობ, 1935 წელს შექმნილი ტესტი დღესაც აქტუალურია?

საქართველოში - სანამ უკეთესს არ შემოიტანენ
ჯერ ჯერობით ამ ტესტებით ხელმძღვანელობენ კოაგულაციის შესაფასებლად
და 75 წელია ამ ტესტებით ნიშნავენ ანტიკოაგულანტებს.
75 წელიც რაღაცას ნიშნავს -
და ვინაიდან ჯერ ჯერობით ამართლებს
სხვა გზა არაა.
მართალია პროთ ინდ. ხალხში პოპულარულია,
მაგრამ ემთხვევა გესმის?
ემთხვევა კლინიკას.




QUOTE
არავითარი ცვლა არ ხდება ამ მონაცემის!

აი ეს საიდან მოიტანე? სად წერია?

QUOTE
თიმოლის სინჯებს

მეც აღარ ვხმარობ,
იმის მერე რაც შემოვიდა ალტ და ასტ.
სანამ შემოვიდოდა
ვხმარობდით.

Posted by: vano_t 21 Nov 2008, 07:31
ბასა-ტტტ
QUOTE
დავუშვათ რომ არაა  ჯერ ჩამოყალიბებული ვენების  ვარიკოზული გაგანიერება,
და  ჯერ არაა ჩამოყალიბებული თრომბოზი-
რა ცვლილებები უსწრებს წინ პათოლოგიას, როცა
ადამიანის  მდგომარეობა არის ჯანმრთელობის და ავადმყოფობის  საზღვარზე?
ანუ არც ავადმყოფად ჩაითვლება, მაგრამ
უკვე არის ცვლილებები ბიოქიმიის დონეზე?
ანუ პირველად რა მაჩვენებელები იცვლება  ორგანიზმში -
სანამ ჩამოყალიბდება ვარიკოზული თრომბოზი?
თრომბოზები და ვარიკოზული ვენები არის 2 სრულიად განსხვავებული რამ. თუ გაინტერესებს რა ცვლება უძღვის წინ თრომბოზებს, წაიკითხე ვირხოვის ტრიადაზე და ჰიპერკოაგულაციურ მდგომარეობებზე. ის ბიოქიუმიური ცვლილებები გენეტიკური დეფექტებია, რომელიც ადამიანს დაბადებიდან დაყვება და შემდეგ შეიძლება გამოვლინდეს როგორც თრომბოზები ნორმალურ ადგილას. არის შეძენილი მდგომარეობებიც: მაგალითად ლუპუს ანტიკოაგულანტი, ან ანტიკარდიოლიპინური ანტისხეულები, ან სისხლძარღვის ენდოთელიუმის დაზიანება, რომლესაც შეიძლება მოყვეს დისემინირებული ინტრავასკულარული კოაგულაცია მაგალითად (და რომლელთა დროსაც პროთრომბინის დრო მაღლა მიდის, ისევე როგორც მაღლა მიდის "გათხელებული" სისხლის დროს). მოკლედ, არავინ ეძებს მასეთ მდგომარეობებს, გარდა ისეთი შემთხვევებისა, როცა ადამიანს აღმოაჩნდება რაიმე გენეტიკური დეფექტი და მისი ნათესავების სკრინინგი ხდება.

QUOTE
Повышенный протромбин – симптом следующих заболеваний:

тромбоэмболия
инфаркт миокарда, предынфарктное состояние
полицитемия
злокачественные опухоли.
Низкий протромбин в крови дает врачу основание поставить следующий диагноз:

врожденный или приобретенный дефицит протромбина
нехватка витамина К.
რუსული სკოლაა. მოკლედ, სანამ რუსები არ განვითარდებიან, მანამდე ძნელია იმის წარმოდგენა, რომ რუსული მედიცინა განვითარდება. რუსული ციტატების მოყვანა საერთოდ უნდა აიკრძალოს smile.gif უკვე გითხარი, რომ მომატებულ პროთრომბინს არავითარი დიაგნოსტიკური ღირებულება არ გააჩნია! პროთრომბინი არის ერთ-ერთი ცილა ე.წ. მწვავე ფაზის პროტეინებს შორის (რაშიც შედის ასევე კარგად ცნობილი ც რეაქტიული პროტეინი). ეს იმას ნიშნავს, რომ პროთრომბინი შეიძლება მაღლა წავიდეს ნებისმიერი ანთებითი პროცესის დროს.

პროთრომბინის რაოდენობა და პროთრომბინის დრო სხვადასხვა ტესტებია. პროთრომბინის დროს პროთრომბინის (ფაქტორი II) გარდა განაპირობებს I, V, VII და X ფაქტორები. ამათგან უმეტესობა ვიტამინ K დამოკიდებული ფაქტორებია და ღვიძლში ხდება მათი სინთეზი. თუ რომელიმე ფაქტორის ნაკლებობაა ან ფუნქციონალური დეფექტია, მაშინ ეს გამოიწვევს პროთრომბინის დროს გაგრძელებას და შესაბამისად INR-ის და პროთრომბინის ინდექსის ცვლილებას. ეს იმას ნიშნავს, რომ გაზრდილი პროთრომბინის დრო (ანუ აწეული INR) შეიძლება მოგცეს I, II, V, VII და X ფაქტორების ნაკლებობამ (ნაკლები სინთეზი ხდება ამათი თუ დეფექტური პროტეინის სინთეზი ხდება, ამას მნიშვნელობა არ აქვს). კლინიკაში ყველაზე ხშირია ამათი გაზრდა რამოდენიმე მიზეზის გამო: 1) ღვიძლის უკმარისობის დროს. როგორც ვთქვი, ღვიძლში ხდება ამათი სინთეზი და როცა ღვიძლის ფუნქცია არასაკმარისია, შესაბამისად ამათი სინთეზიც დაქვეითდება და პროთრომბინის დროს მაღლა აგიწევს. ამ ფაქტორების უკმარისობის გამოა სისხლდენები რომ ემართებათ ჰეპატიტიანებს ცხვირიდან. ამათ რომ გაუზომო პროთრომბინის დრო, აწეული უქნებათ. 2) ამ ფაქტორების სინთეზი კლებულობს ვიტამინ K-ს დეფიციტის დროს და იგივე პრობლემას წააწდები. 3) კუმადინის მიღების დროს. კუმადინი ფაქტიურად ვიტამინ K-ს ანტაგონისტია და 2-ის მსგავსად იწვევს ყველაფერს.

ახლა აგიხსნი რატომ არ აქვს აზრი პროთრომბინის დროის (და შესაბამისად პროთრომბინის ინდექსის ან INR-ის) შემოკლებას აზრი.

მასეთ რამეს კლინიკური მნიშვნელობა არ აქვს. აიღე მაგალითად ღრმა ვენების თრომბოზი. ასეთ ავადმყოფებს ენიშნებათ კუმადინი, მაგრამ კუმადინის დანიშვნამდე უმოწმებ საწყისს INR-ს. ეს არის სტანდარტიზირებული ტესტია და ეს უნდა გამოიყენო (და არა პროთრომბინის დრო), რადგანაც პროთრომბინის დრო ძაან ცვალებადია და დამოკიდებულია იმაზე თუ რა ფირმის ქსოვილურ ფაქტორს იყენებს ლაბორატორია. ყველა ამ ქსოვილურ ფაქტორს მოყვება ე.წ. ISI, რაც რიცხვითი სიდიდეა და ამ ქსოვილურ ფაქტორს ახასითებს. შემდეგ ამ ISI-ს ჩასვამ ფორმულაში და მიიღებ INR-ის მნიშვნელობას. ხოდა ეს INR არის ნორმალური თრომბოზული მდგომარებების დროს, ან მომატებულია და არა დაკლებული. ღრმა ვენის თრომბოზიან ადამიანს დასაწყისში აქვს ნორმალური INR და არა დაკლებული! INR შეიძლება გაიზარდოს კიდეც ბევრი თრომბოზული მდგომარეობების დროს. მაგალითი ადრეც ვახსენე: მაგალითად დისემინირებული ინტრავასკულარული კოაგულაცია. ამ დროს გაქანებული თრომბოზები გაქვს ყველგან, მაგრამ INR მაღლა ადის, იმიტომ რომ პლაზმის ფაქტორების მოხმარება ხდება და შესაბამისად მათი დაკლება პლაზმაში. ამას მოყვება ასევე სისხლდენები.

მე პირადად არცერთ წიგნში არ მინახია, რომ დაბალ INR-ს სადიაგნოსტიკო ღირებულება ქონდეს. შეიძლება ქონდეს და ვინმემ რაღაც იპოვოს (ან იპოვა), მარა ჯერ მასეთი წყარო არ მინახია. შენ თუ იცი რაიმე ასეთი, მაშინ მიუთითე და გადავხედოთ.

Posted by: Cousteau 21 Nov 2008, 10:43
თუ არ ვცდები INR მხოლოდ ვარფარინის საკონტროლოდ გამოიყენება, სხვა შემთხვევებში INR-ით არ წერენ პროთრომბინის ინდექსს

Posted by: vano_t 21 Nov 2008, 12:20
Cousteau
QUOTE
თუ არ ვცდები INR მხოლოდ ვარფარინის საკონტროლოდ გამოიყენება, სხვა შემთხვევებში INR-ით არ წერენ პროთრომბინის ინდექსს

პროთრომბინის ინდექსს საერთოდ არ ხმარობენ ბევრგან. პროთომბინის დროს რომ ზომავ, პრობლემა მდგომარეობს ტესტის სტანდარტიზაციაში. A ლაბორატორია რომ უჩვენებს 17 წამს, B-მ შეიძლება იგივე სისხლზე 25 მოგცეს და ორივე ნორმა იყოს თავიანთ ფარგლებში. თანაც, A-სათვის 25 შეიძლება იყოს ნორმის გარეთ და პირიქით. ამიტომ არის ტესტის სტანდარტიზაცია საჭირო და INR ზუსტად ამას ემსახურება. ანუ, INR არის პროთრომბინის დრო "დასტანდარტებული". ეს არ ეხება მარტო კუმადინით მკურნალობას. ეს ეხება საერთოდ პროთრომბინის დროს. მასე არც INR იქნება საჭირო თუ ერთიდაიმავე ლაბორატორიის მონაცემებს იყენებ (როცა ლაბორატორია ტესტირებისას იყენებს ქსოვილური თრომბოპლასტინის ერთიდაიმავე წარმოებას ერთიდაიმავე ISI რიცხვით). მე ჩემს ლაბორატორიაში სრულებით არ დამჭირდება რომ INR ვიხმარო. ჩემს ლაბორატორიას შევეკითხები უბრალოდ პროთრომბინის რა დრო შეესაბამება INR-ის მნიშვნელობა 2-ს და 3-ს. თუ ლაბორანტი მეტყვის, რომ 34-45 წამიო, მაშინ მთელი მკურნალობაც ისე შემიძლია წავმართო, რომ პროთრომბინის დრო ამ რიცხვებში მოვაქციო. სამაგიეროდ, თუ ჩემი პაცინტი სხვაგან წავიდა და ჩვენი ლაბორატორიის გაზომილი პროთრომბინის ინდექსი მიუტანა სხვა ექიმს, ის ვერარფერს იტყვის, რამეთუ მისი ლაბორატორიის შესაბამისი რიცხვები სხვანაირია. ეს არის INR-ის გამოყენების არსი და მეტი არაფერი. ანუ, INR არის იგივე პროთრომბინის დრო, ოღონდ სტანდარტიზირებული. შესაბამისად, INR შეგიძლია გამოიყენოს ყველგან, სადაც უკვეთავ პროტრომბინის დროს საერთოდ (მაგალითად კოაგულაციის პრობლემების დროს).

აი INR-ის გამოთვლის ფორმულაც (INR გამოთვლილი სიდიდეა და არა გაზომილი): INR=[(Patients PT)/(Control PT)]^ISI, სადაც PT ნიშნავს პროთრომბინის დროს და ISI არის სპეციალური რიცხვი, რომელსაც მწარმოებელი ანიჭებს თავის ქსოვილური ფაქტორს (ქსოვილური ფაქტორი საჭიროა, რომ პლაზმის კოლტირება მოახდინო და პროთრომბინის დრო გაზომო)

Posted by: Cousteau 21 Nov 2008, 12:22
ვიცი INR რაც არის, უბრალოდ მაგაში მგონი მარტო ''ვარფარინოთერაპიას ზომავენ'' ეგრე მახსოვს სადღაც ეწერა : S

Posted by: vano_t 21 Nov 2008, 13:13
QUOTE (Cousteau @ 21 Nov 2008, 12:22 )
ვიცი INR რაც არის, უბრალოდ მაგაში მგონი მარტო ''ვარფარინოთერაპიას ზომავენ'' ეგრე მახსოვს სადღაც ეწერა : S

არასწორად გახსოვს. არამგონია მასე ეწეროს სადმე. ჯერ ერთი, როცა უკვეთავ პროთრომბინის დროს, ორი მნიშვნელობას (გაზომილ დროს და გამოთვლილ INR-ს) გაძლევს ლაბორატორია. შენ რომელი მნიშვნელობაც გინდა გამოიყენე. თუ შეუკვეთე პროთრომბინის დრო ავადმყოფში, რათა დაადგინო სისხლდენის მიზეზი (თან ავადმყოფი არ არის კუმადინზე) და უყურებ INR-ს, რომელიც არის 1.6, აღარ უნდა აქ ფილოსოფია. ამ ადამიანს გახანგრძლივებული აქვს პროთრომბინის დრო. სხვანაირად 1.6 ვერ მიიღებდი INR-ს. ნორმალური INR ითვლება <= 1.2 (ან შეიძლება თქვა 1.3-ზე).

მეორეც, INR ყოვლედღე ვუკვეთავ და ასევე ვხედავ ჰემატოლოგების შეკვეთილებს.

დაუფიქრდი, დავუშვათ ლაბორატორიას აქვს პროთრომბინის დროის ზემოთა ზღვარი 22. ამის შესაბამის INR უკვე იქნება ისე გამოყვანილი, რომ ნორმის ზედა ზღვარი იყოს, თანაც გასატანდარტებული.

Posted by: Cousteau 21 Nov 2008, 13:29
QUOTE (vano_t @ 21 Nov 2008, 13:13 )

არასწორად გახსოვს. არამგონია მასე ეწეროს სადმე.


maybe, მაგრამ მახსოვს smile.gif მოკლედ თუ ვნახე სად ეწერა დავდებ, 100% არ ვარ დარწმუნებული smile.gif


error smile.gif მოკლედ ის მახსოვდა ''ესე'' რო მარტო Vit K mediated anticoagulation-ს აკონტროლებენ და არა ჰეპარინს... რეამინაციის წიგნში ეწერა yes.gif

my fault

Posted by: texasuri jleta benzoxerxit 21 Nov 2008, 14:42
QUOTE (vano_t @ 20 Nov 2008, 12:56 )
basa-ttt
QUOTE
შედედებას გვიჩვენებს, მარტივ იტესტია და მთელი კოაგულოგრამა საჭირო არაა-
Протромбиновый индекс (или индекс протромбина)
Определение индекса протромбина выражает отношение протромбинового времени здорового человека к протромбиновому времени больного. Выражается в процентах.
http://www.policlinica.ru/analiz4_7.html
მაი თქვენ ამერიკულ კეისებში არ ვიცი  მაგას  რა შეესაბამება.
იმიტომ  გეკითხები, რომ უამრავი შემთხვევა მქონდა მე პირადად
ანალოგიური ჩივილებით (ფრიალი გულის არეში)
როცა მარტო ეს ინდექსი იყო მომატებული და სხვა ყველაფერი  - ნორმა.
არავითარი კავშირი არ არსებობს. ეგ ყოფილა უბრალოდ გამონათვალი ისევე როგორც არის INR (Международное нормализованное отношение (МНО)), რომელიც ცდილობს პროთრომბინის დროს სტანდარტიზაციას. არ არის კარგი გამონათვალი და იმიტომაც არ ხმარობენ "მეჟდუნაროდები". ასეც და ისეც, პროთრომბინის დრო (და მისგან წარმოებული პროთრომბინის ინდექსი ან საერთაშორისო ნორმალიზებული შეფარდება) არ მოგცემს არავითარ ინფორმაციას. ეგ ტესტი იხმარება მხოლოდ არანორმალური სისლხდენების დიაგნოზისათვის და კუმადინის თერაპიული დოზის განსაზღვისათვის. საქართველოში, შემიძლია გითხრა, რომ მაგ ტესტს უმეტესად ყოველგვარი დანიშნულების გარეშე აკეთებენ-რაც არასწრია (მაგალითად, ღრმა ვენების თრომბოზების დიაგნოსტიკისათვის, რაც სავსებით არარაციონალურია).

QUOTE
QUOTE
ნაღვლის ბუშტი გამოკველეული არ არის და არ არის საჭირო

თუ ექსტრასისტოლას დაიჭერ -
და პარკუჭოვანი იქნება
(სავარაუდოდ პარკუჭოვანია -
ვინაიდან ავადმყოფი შეიგრძნობს მას-)
უნდა ეძებო ექსტრაკარდიალური მიზეზი.
ყველზე ხშირად ქართველებში ნაღვლის ბუშტია-
გვიყვარს გემრიელი საჭმელები.
დედაჩემს ჰქონდა კლასიკური ბიგემინია.
და როგორც კი ნაღვლის ბუშტის გამწმენდი დავუნიშნე (ჰომეოპათიური)
2 საათში მოეხსნა ბიგემინია და ფეხზე ადგა ქალი./
მოდი, ძალიან გთხოვ, არ გვინდა ეს ჰომეოპათია აქ. აქ ვისაუბროთ ჩვეულებრივ მედიცინაზე. მასე სიმელოტეც შეიძლება იწვევდეს არითმიებს. ან კიდევ კეხიანი ცხვირი. მარა, ასეთ რამეები ჯერ არ დაუდგენია მედიცინას. ხოდა, რაც იციან იმის ფარგლებში ვისაუბროთ. ხოდა, ამიტომ მსგავსი პასუხი არ მიიღება. თუ ნაღვლის ბუშტის დაავადებებსა და არითმიებს შორის რაიმე კავშირი არსებობს, მაშინ უშუალოდ მიმითითე ასეთ კავშირზე. მანამდე ავადმყოფს არ დაენიშნება ნაღვლის ბუშტის გამოკვლევა.

რატოა საერთოდ პარკუჭოვანი? ან რატოა ექსტრასისტოლა? ხომ გკითხე, როგორ ხსნის იმ ფაქტს, რომ ავადმყოფს შეტევები ეწყება უცებ და უმთავრდება უცებ. თანაც შეტევა რამოდენიმე ხანი გრძელდება. ექსტრასისტოლები ამას გაძლევენ? მეორეც, არ შეიძლება წინაგულოვანმა რამეებმა მოგცეს თავბრუსხვევა?

Blind_Torture_Kill
QUOTE
მიქსომა
მიქსომას როგორ აკავშირებ ამ ყველაფერთან? (ჯერ ექო არ გამიკეთებია ისე)

მე კიდევ ინტერნისტი ვარ.

ჩემო ვანო ქოლეცისტო-კარგდიული,გასტროკარდიული და ზოგადად კი ვისცერო-კარდიული რეფლექსი არ გსმენია????გასტრიტმა,ქოლეცისტიტმა,საყლაპავის ხვრელის თიაქარმა,სანაღვლე გზების დისკინეზიამ,რომ შეიძლება როგორც წინაგუოვანი ასევე პარკუჭოვანი ექსტრასისტოლია მოგცეს და იშვიათად წინაგულოვანი პაროქსიზმული ტაქიკარდიაც კი,ეს რამე ახალია?ამაზე სტუდენტმა რომ არ მიპასუხოს ,არ ჩავუთვლი.
ჰომეპათია არ ვიცი,ვერაფერს ვიტყვი,მაგრამ იმის მტკიცება,რომ ნაღვლის ბუშტის დაავადებები კარდიალურ სიმპტომებს არ მოგცემს... wow.gif

შენ ქეისში სავარაუდოდ პაროქსიზმული ტაქიკარდიის ეპიზოდებია.უეცარი დასაწყისი,უეცარი შეწყვეტა...საჭიროა TEE და ექტოპიური აქტივობის კერის მოძებნა,თუ ასეთი იქნა...აბლაცია.
ასევე საძიებელის ექსტრაკარდიალური მიზეზები,ძირითადად გასტრო-ინტესტინალური ტრაქტის მხრივ boli.gif
რაღა თქმა უნდა მედიკამენტოზური (ლევოთიქროქსინის)გადაჭარბებაც უნდა იქნას გათვალისწინებული და გაკეთდეს შესაბამისი ანალიზები

Posted by: mika9 21 Nov 2008, 16:18
vano_t
რა პროფესიის ექიმი ხარ?
...................................................................

პროტრომბინის ტესტი რომელია PT????

Posted by: texasuri jleta benzoxerxit 21 Nov 2008, 16:52
mika9
ჯარა-ექიმია givi.gif ვხუმრობ,ძალიან კარგი თერაპევტია.ინტერნისტი.აშშ-ში მოღვაწე თანაც boli.gif

Posted by: mika9 21 Nov 2008, 19:22
vano_t

ჰოსპიტალისტი ხარ?

რომელ ჰოსპში მუშაობ??

სასიამოვნოა აქ ამერიკელი ექიმების ნახვა

Posted by: zviadcardio 21 Nov 2008, 19:36
texasuri jleta benzoxerxit
QUOTE
საჭიროა TEE და ექტოპიური აქტივობის კერის მოძებნა

რისთვისაა საჭირო TEE?
როგორ უნდა მოხდეს "ექტოპიური აქტივობის კერის მოძებნა"?
და თუ "ექტოპიური აქტივობის კერა" არ აქვს პაციენტს და ნოდალური რიენტი აქვს, მაშინ?


QUOTE
ამაზე სტუდენტმა რომ არ მიპასუხოს ,არ ჩავუთვლი.

მოკლედ მეგობარო მე თქვენი საწინააღმდეგო არაფერი მაქვს მაგრამ კარდიოლოგიის ამ ნაწილს (არითმიებს ვგულისხმობ) "ვერ ჩაგითვლით".
იმედია არ მიწყენთ.
* * *
vano_t
1-2 წლის წინ პროთრომბინზე რასაც წერთ, დაახლოებით ის რომ დავწერე ფორუმზე თითქმის ქვა მესროლეს.
ასე რომ, ვინც იცის-იცის. ვინც არა და... მაგ თემაზე აღარ ვკამათობ.

ISI-ს არგათვალისწინება მთავარი მიზეზია პროთრომბინის როგორც ანალიზის უზუსტობისა.

Posted by: Cousteau 21 Nov 2008, 20:19
QUOTE
ვინც იცის-იცის. ვინც არა და...

იმას უნდა ასწავლო yes.gif




Posted by: basa-ttt 21 Nov 2008, 20:31
QUOTE
ძალიან კარგი თერაპევტია.ინტერნისტი.აშშ-ში მოღვაწე თანაც

სულ მქონდა გრძნობა
რომ სადღაც მთვარედან გველაპარაკებოდა...
რეალობას მოწყვეტილი

QUOTE
რუსული სკოლაა. მოკლედ, სანამ რუსები არ განვითარდებიან, მანამდე ძნელია იმის წარმოდგენა, რომ რუსული მედიცინა განვითარდება. რუსული ციტატების მოყვანა საერთოდ უნდა აიკრძალოს  უკვე გითხარი, რომ მომატებულ პროთრომბინს არავითარი დიაგნოსტიკური ღირებულება არ გააჩნია! პროთრომბინი არის ერთ-ერთი ცილა ე.წ. მწვავე ფაზის პროტეინებს შორის (რაშიც შედის ასევე კარგად ცნობილი ც რეაქტიული პროტეინი). ეს იმას ნიშნავს, რომ პროთრომბინი შეიძლება მაღლა წავიდეს ნებისმიერი ანთებითი პროცესის დროს.

გამარჯობა შენი -
თერაპევტი თუ ვერ არჩევს კლინიკურად ანთებასთან აქვს საქმე, თუ არა -
კარგად ყოფილა მისი საქმე.
რუსული სკოლა ბევრ რამეში ჯობია ამერიკულს,
აკრძალო არა კვახი.
მაგრამ ეს სხვა საკითხია.

QUOTE
ჩემო ვანო ქოლეცისტო-კარგდიული,გასტროკარდიული და ზოგადად კი ვისცერო-კარდიული რეფლექსი არ გსმენია????

ამერიკაში ეტყობა არ სმენიათ -
იქ ფხალი და საცივი არ იციან რაა...

დავუბრუნდეთ კოაგულაციის მაჩვენებლებს...
QUOTE
პროთრომბინის რაოდენობა და პროთრომბინის დრო სხვადასხვა ტესტებია

კაი ერთი -

QUOTE
პროთრომბინის დროს პროთრომბინის (ფაქტორი II) გარდა განაპირობებს I, V, VII და X ფაქტორები. ამათგან უმეტესობა ვიტამინ K დამოკიდებული ფაქტორებია და ღვიძლში ხდება მათი სინთეზი. თუ რომელიმე ფაქტორის ნაკლებობაა ან ფუნქციონალური დეფექტია, მაშინ ეს გამოიწვევს პროთრომბინის დროს გაგრძელებას და შესაბამისად INR-ის და პროთრომბინის ინდექსის ცვლილებას. ეს იმას ნიშნავს, რომ გაზრდილი პროთრომბინის დრო (ანუ აწეული INR) შეიძლება მოგცეს I, II, V, VII და X ფაქტორების ნაკლებობამ (ნაკლები სინთეზი ხდება ამათი თუ დეფექტური პროტეინის სინთეზი ხდება, ამას მნიშვნელობა არ აქვს). კლინიკაში ყველაზე ხშირია ამათი გაზრდა რამოდენიმე მიზეზის გამო: 1) ღვიძლის უკმარისობის დროს. როგორც ვთქვი, ღვიძლში ხდება ამათი სინთეზი და როცა ღვიძლის ფუნქცია არასაკმარისია, შესაბამისად ამათი სინთეზიც დაქვეითდება და პროთრომბინის დროს მაღლა აგიწევს. ამ ფაქტორების უკმარისობის გამოა სისხლდენები რომ ემართებათ ჰეპატიტიანებს ცხვირიდან. ამათ რომ გაუზომო პროთრომბინის დრო, აწეული უქნებათ. 2) ამ ფაქტორების სინთეზი კლებულობს ვიტამინ K-ს დეფიციტის დროს და იგივე პრობლემას წააწდები. 3) კუმადინის მიღების დროს. კუმადინი ფაქტიურად ვიტამინ K-ს ანტაგონისტია და 2-ის მსგავსად იწვევს ყველაფერს.

ეს ჩვენ არ გვაინტერესებს. აქ გასაგებია ყველაფერი

ჩვენ გვაინტერესებს სიტუაცია, როცა მატულობს სისხლის შედედება..

QUOTE
ახლა აგიხსნი რატომ არ აქვს აზრი პროთრომბინის დროის (და შესაბამისად პროთრომბინის ინდექსის ან INR-ის) შემოკლებას აზრი.

მასეთ რამეს კლინიკური მნიშვნელობა არ აქვს. აიღე მაგალითად ღრმა ვენების თრომბოზი

ისევ არასწორი მიმართულებით მიდიხარ.
მოდი შეეშვი ამ კუმადინს..
და მიპასუხე მარტივ კითხვებზე -
1) მე რომ მოვიყვანე შემთხვევა 12 წლის ბიჭის -
რომელსაც ჰქონდა პროთრომბინის ინდექსი მომატებული -
და ჰქონდა გენეტიკური განწყობა (მამას ვარიკოზი)
ასეთ სიტუაციაში როგორ იცვლება კოაგულოგრამის მაჩვენებლები?
ანუ შეიძლება ცალკე პროთრომბინის დრო არ იყოს შეცვლილი იზოლირებულად და თან ახლავდეს სხვა
მაჩვენებლების ცვლილებებს.
ანუ ასეთ სიტუაციაში კოლტის წარმოქმნა სისხლში ხდება უფრო სწრაფად და სისხლი უფრო ბლანტია-
რაც აისახა კიდეც ეკგ - ზე.

2) როგორ იცვლება კოაგულოგრამის მაჩვენებლები -
როცა გვაქვს ვენების ვარიკოზული გაგანიერება და ჯერ არ გვაქვს თრომბი ჩამოყალიბებული,
ანუ არის სტაზი ვენოზურ სისტემაში..
იცვლება ვენების ენდოთელიუმის ზედაპირის ელპოტენციალი,
ხდება მის ზედაპირზე ადგეზიური ცვლილებები და ა შ და სხვ.....

3) უკვე ჩამოყალიბებული თრომბის შემთხვევაში - კი ბატონო გასაგებია....

Posted by: Guardian 21 Nov 2008, 20:42
Cousteau
QUOTE
ვინც იცის-იცის. ვინც არა და...
იმას უნდა ასწავლო

A person who knows not and knows not that he knows not is a fool,
shun him.
A person who knows not and knows that he knows not is a child,
teach him.
A person who knows and knows not that he knows is asleep,
wake him.
A person who knows and knows that he knows is wise,
follow him.
—Ancient Babylonian Aphorism


zviadcardio
QUOTE
მოკლედ მეგობარო მე თქვენი საწინააღმდეგო არაფერი მაქვს მაგრამ კარდიოლოგიის ამ ნაწილს (არითმიებს ვგულისხმობ) "ვერ ჩაგითვლით". იმედია არ მიწყენთ.

Don't let school interfere with your education.
—Mark Twain

Posted by: basa-ttt 21 Nov 2008, 21:14
Ничто так не унижает человека, не делает его таким жалким, как тщеславие; оно ярчайшая примета посредственности.
Люк де Клапье Вовенарг

Posted by: Cousteau 21 Nov 2008, 22:07
QUOTE (Guardian @ 21 Nov 2008, 20:42 )
Cousteau
QUOTE
ვინც იცის-იცის. ვინც არა და...
იმას უნდა ასწავლო

A person who knows not and knows not that he knows not is a fool,
shun him.
A person who knows not and knows that he knows not is a child,
teach him.
A person who knows and knows not that he knows is asleep,
wake him.
A person who knows and knows that he knows is wise,
follow him.
—Ancient Babylonian Aphorism


ვიცი ეგ smile.gif

ენდოკრინოლოგიაში მასწავლებელი მყავდა, მთხოვა რამე დამიწერეო და ეგ დავუწერე : )


''Each One Teach One''
-Old Hebrew Aphorism




QUOTE
რომ სადღაც მთვარედან გველაპარაკებოდა...
რეალობას მოწყვეტილი


ჯერესერთი ნეტავ ეგეთი 'რეალობას მოწყვეტილი' მე ვიყო, მეორესერთი 'მთვარიდან' არ ლაპარაკობს, ლაპარაკობს იქედან საიდანაც დღევანდელი (და ხვალინდელი) მედიცინა ისწავლება
მესამე: ადამიანმა რომელმაც ეს შეძლო როგორიც არ უნდა იყოს ლაპარაკით მაინც პატივისცემას უნდა იმსახურებდეს
wink.gif

Posted by: basa-ttt 21 Nov 2008, 22:45
QUOTE
ადამიანმა რომელმაც ეს შეძლო როგორიც არ უნდა იყოს ლაპარაკით მაინც პატივისცემას უნდა იმსახურებდეს

და ვინ თქვა რომ არ იმსახურებს?
სხვაგვარად არც კი დაველაპარაკებოდი.
2kiss.gif


QUOTE
საიდანაც დღევანდელი (და ხვალინდელი) მედიცინა ისწავლება

დღევანდელი კი -
მარა ხვალინდელი მედიცინა - ჰომეოპათიაა.
gigi.gif

Posted by: Blind_Torture_Kill 21 Nov 2008, 22:49
კაი
რაღაც ძაან ძაბავთ პროთრომბინზე user.gif

აი ქეისი

a 4 y/o boy presents with recurrent joint pain involving knees and hips.ha has always bruised easily and recently the parents
had seen blood in his urine.
a presumptive diagnosis of classic hemophilia is made and coagulations blood tests are perforemed.which of the following is the most likely set of findings of coagulation screening test ? (PT,APTT,bleeding time,platelet count)

Posted by: Cousteau 21 Nov 2008, 23:07
QUOTE (Blind_Torture_Kill @ 21 Nov 2008, 22:49 )
კაი
რაღაც ძაან ძაბავთ პროთრომბინზე  user.gif

აი ქეისი

a 4 y/o boy presents with recurrent joint pain involving knees and hips.ha has always bruised easily and recently the parents
had seen blood in his urine.
a presumptive diagnosis of classic hemophilia is made and coagulations blood tests are perforemed.which of the following is the most likely set of findings of coagulation screening test ? (PT,APTT,bleeding time,platelet count)

bleeding time ?

VIII და IX ფაქტორები უნდა მაგას

Posted by: texasuri jleta benzoxerxit 22 Nov 2008, 00:16
zviadcardio
არ გიწყენ,პრი მოიხ ზაპროსახ მე თვითონ არ ჩავუთვლიდი ჩემ თავს არითმოლოგიას,მის ღრმა ელექტროფიზიოლოგიურ მონაცემებში ლავირების ასპექტში.

მაგრამ იმედია შენ მაინც არ აპროტესტებ ვისცერო-კარდიალურ რეფლექსს,თუმცა ის,რომ ამ დროს ექსტრასისტოლიის გარდა შეიძლება სუპრავენტრიკულური პაროქსიზმული ტაქიკარდიაც განვითარდეს,ამისთაობაზე ერთ-ერთ ტოპიკში დავხოცეთ უკვე ერთმანეთი...ბენზოხერხით biggrin.gif

Posted by: Blind_Torture_Kill 22 Nov 2008, 00:43
Cousteau

QUOTE
PT,APTT,bleeding time,platelet count


ამეებში რა გექნება ეგ მაინტერესებს

QUOTE
VIII და IX ფაქტორები უნდა მაგას


VIII

Posted by: Cousteau 22 Nov 2008, 00:51
QUOTE (Blind_Torture_Kill @ 22 Nov 2008, 00:43 )


QUOTE
VIII და IX ფაქტორები უნდა მაგას


VIII

და IX (ჰემოფილია ბსთვის)

QUOTE
Cousteau

QUOTE
PT,APTT,bleeding time,platelet count


ამეებში რა გექნება ეგ მაინტერესებს

bleeding time მგონი

Posted by: vano_t 22 Nov 2008, 00:59
Blind_Torture_Kill
QUOTE
QUOTE
VIII და IX ფაქტორები უნდა მაგას


VIII

სწორს ამბობს. ჰემოფილია 2 ტიპის არის. ერთისათვის არის დამახასითებელი მე-8 ფაქტორის დეფიციტი და მეორესათვის დამახასიათებელია მე-9 ფაქტორის დეფიციტი.

ისე ზოგადად, თუ თრომბოციტების ნაკლებობა გაქვს ან თვისებრივი დეფექტი, მაშინ ეს გამოიწვევს სისხლდენის დროის გაზრდას.

თუ გარეგანი ფაქტორების (extrinsic pathway-ს ფაქტორები) ნაკლებობა გაქვს, მაშინ გექნება PT-ს მომატება. მე-7 ფაქტორი შედის აქ.

თუ შინაგანი ფაქტორების (intrinsic pathway-ს ფაქტორები) დეფიციტი გაქვს, მაშინ გექნება aPTT-ს მომატება. აქ შედის მე-8 და მე-9 ფაქტორები. ასევე მე-12 და მე-11 ფაქტორები.

თუ common pathways-ს (ანუ გარეგანი და შინაგანი გზის საერთო კომპონენტები, რომელიც სისხლის შედედების კასკადის ბოლო ფაქტორებია), მაშინ aPTT-ც და PT-ც მოიმატებს.

Posted by: Cousteau 22 Nov 2008, 01:06
QUOTE (vano_t @ 22 Nov 2008, 00:59 )
Blind_Torture_Kill
QUOTE
QUOTE
VIII და IX ფაქტორები უნდა მაგას


VIII

სწორს ამბობს. ჰემოფილია 2 ტიპის არის. ერთისათვის არის დამახასითებელი მე-8 ფაქტორის დეფიციტი და მეორესათვის დამახასიათებელია მე-9 ფაქტორის დეფიციტი.

ისე ზოგადად, თუ თრომბოციტების ნაკლებობა გაქვს ან თვისებრივი დეფექტი, მაშინ ეს გამოიწვევს სისხლდენის დროის გაზრდას.

თუ გარეგანი ფაქტორების (extrinsic pathway-ს ფაქტორები) ნაკლებობა გაქვს, მაშინ გექნება PT-ს მომატება. მე-7 ფაქტორი შედის აქ.

თუ შინაგანი ფაქტორების (intrinsic pathway-ს ფაქტორები) დეფიციტი გაქვს, მაშინ გექნება aPTT-ს მომატება. აქ შედის მე-8 და მე-9 ფაქტორები. ასევე მე-12 და მე-11 ფაქტორები.

თუ common pathways-ს (ანუ გარეგანი და შინაგანი გზის საერთო კომპონენტები, რომელიც სისხლის შედედების კასკადის ბოლო ფაქტორებია), მაშინ aPTT-ც და PT-ც მოიმატებს.

მარტო PT და apttთი იმჰო ჰემოფილიის დიაგნოზს ვერ დასვავ
მარა ბიდინგ თაიმი გახანგრძლივებული ექნება მგონი მაინც

Posted by: vano_t 22 Nov 2008, 01:18
QUOTE (Cousteau @ 22 Nov 2008, 01:06 )
მარტო PT და apttთი იმჰო ჰემოფილიის დიაგნოზს ვერ დასვავ
მარა ბიდინგ თაიმი გახანგრძლივებული ექნება მგონი მაინც

PT არ უნდა შეიცვალოს ჰემოფილიის დროს. არც სისხლდენის დრო არ იცვლება მაგ დროს. სისხლსდენის დრო არ არის დამოკიდებული ფაქტორებზე. მარტო aPTT უნდა შეიცვალოს.

ისე მართალი ხარ იმაში, რომ მარტო ამით ვერ დასვამ დიაგნოზს. ეს უბრალოდ არის სკრინინგის ტესტი. ანუ, თუ ასეთ ბავშვს გახანგრძლივებული აქვს მაგალითად სისხლდენის დრო და სხვა რამეები ნორმაშია, ან PT აქვს გახანგრძლივებული და სხვა რამეები ნორმაშია, მაშინ ჰემოფილიაზე აღარ ფიქრობ. როცა ნახავ, რომ aPTT მომატებულია, ფაქტორებზე უნდა შეამოწმო. მე-8 და მე-9 ფაქტორების დეფიციტი ამ დროს მე მგონი ყველაზე ხშირია. ისე შესაძლებელია მე-12 ან მე-11 ფაქტორების დეფიციტიც ქონდეს. თუმცა მე-12 ფაქტორის (ჰაგემანის ფაქტორი) დეფიციტი კი გაძლევს aPTT-ს გახანგრძლივებას, მაგრამ სისხლდენები იშვიათია ამისათვის.

Posted by: Cousteau 22 Nov 2008, 01:21
QUOTE
სისხლსდენის დრო არ არის დამოკიდებული ფაქტორებზე


platelette aggregation-ზე და ეგეთ რამეებზეა დამოკიდებული?
აღარ მახსოვს : S

Posted by: Blind_Torture_Kill 22 Nov 2008, 01:24
QUOTE
a presumptive diagnosis of classic hemophilia


მე-8 ფაქტორის დეფიციტი გაძლევს
ხოდა დიაგნოზზე კი ნუ მსჯელობთ (უკვე დასვეს)

რა ცვლილებები გექნება ამ დროს ეგ მაინტერესებს და მოკლედ დაწერეთ

Posted by: Cousteau 22 Nov 2008, 01:25
QUOTE
Blind_Torture_Kill

ცუდ ხასაითზე ხარ? gigi.gif

მე დავწერე ბლიდინგთაიმი და მერე ვანომ არა აპტტ-ო

Posted by: vano_t 22 Nov 2008, 01:26
QUOTE (Cousteau @ 22 Nov 2008, 01:21 )
QUOTE
სისხლსდენის დრო არ არის დამოკიდებული ფაქტორებზე


platelette aggregation-ზე და ეგეთ რამეებზეა დამოკიდებული?
აღარ მახსოვს : S

კი. ანუ ზოგადად სისხლდენის დრო დამოკიდებულია ნორმალური რაოდენობის და ნორმალური ფუნქციის თრომბოციტების არსებობაზე. ან რაოდენობა რომ დაეცეს, ან ფუნქცია არ იყოს ნორმალური, მაშინ მოხდება სისხლდენის დროის გახანგრძლივება. შეძენილების მაგალითია: ITP (როცა თრომბოციტები დაბალია) და ასპირინით მკურნალობა (როცა ასპირინი ცვლის თრომბოციტების აგრეგაციას).

ახლა ვნახე, მე-11 ფაქტორის დეფიციტს ქვია როზენთალის სინდრომი და კიდევ ჰემოფილია C-საც ეძახიან.

Posted by: Blind_Torture_Kill 22 Nov 2008, 01:28
Cousteau

QUOTE
ცუდ ხასაითზე ხარ?


არატო საიდან მოიტანე biggrin.gif


Posted by: Cousteau 22 Nov 2008, 01:32
QUOTE (Blind_Torture_Kill @ 22 Nov 2008, 01:28 )
Cousteau

QUOTE
ცუდ ხასაითზე ხარ?


არატო საიდან მოიტანე biggrin.gif

gigi.gif
რავი user.gif

Posted by: Blind_Torture_Kill 22 Nov 2008, 01:34
Cousteau

აბა პასუხი უცებ რა შედეგები გვექნება (PT,APTT,bleeding time,და platelet count)-ში

ეჰჰ ისევ სურათებით ჯობია smile.gif
ამის მერე სურათებით დავალაგებ ქეისებს

Posted by: vano_t 22 Nov 2008, 08:14
აგერ ახალი შემთხვევა.

65 წლის მდედრი მოდის ეპიგასტრიუმის და გულმკერდი ქვემო ნაწილის ტკივილით. ტკივილი დაიწყო ერთი დღის წინ. თან აქვს გულსრევა და ღებინება. ასევე აღენიშნება თავბრუსხვევა. ავადმყოფს აქვს წარსული ისტორია კორონარების დაავადების, ჰიპერტენზიის და ჰიპერლიპიდემიის. კლინიკაში ნახულობს ლოკალურ ექიმს. გულმკერდის ქვემო ნაწილში ტკივილის გამო და რისკ ფაქტორების გამო, ეს ექიმი ავადმყოფს იღებს დაკვირვებისათვის საავადმყოფოში ინფარქტის გამოსარიცხად. უკვეთავს ენზიმებს, ეკგ-ს, გულმკერდის რენტგენს, ლეიკოციტებს, ერითროციტებს, ჰემოგლობინს, ელექტროლიტებს. ყველაფერი მოდის ნორმაში. ავადმყოფს აკვირდება 24 საათის განმავლობაში და ინფარქტის გამორიცხვის შემდეგ უშვებს სახლში.

ავადმყოფი განაგრძობს ტკივილებს და გულისრევა/ღებინება/თავბრუსხვევას. ამის შემდეგ ავადმყოფს ეს ექიმი უშვებს დიდ ჰოსპიტალში შემდგომი გამოკვლევებისათვის და მკურნალობისათვის. იქაც უკეთებენ დამატებით ტესტებს. ამ ტესტებიდან ერითროციტების დალექვის სიჩქარე მომატებულია 82-მდე (ამ ქალისათვის ნორმა შეიძლება ჩაითვალოს 50-მდე ალბათ).

ჰემოგლობინი ცოტათი დაქვეითებულია 10 მგ/დლ (ნორმის ქვედა ზღვარი 11.5 ქალისათვის ამ ლაბორატორიის მიხედვით); ამინოტრანსფერაზები ნორმაშია; ბილირუბინი, მჟავე ფოსფატაზა, ალბუმინი, გლობულინი ნორმაშია; მუცლის ღრუს პათოლოგიების გამოსარიცხად (განსაკუთრებით ქოლეცისტიტის, რაშიც კონსულტატმა ქირურგმა შეიტანა ეჭვი) ავამდყოს უკეთდება მუცლის ღრუს კომპიუტერული ტომოგრაფია, რაც ნორმაშია. ასევე გაუკეთდა ნაღვლის ბუშტის/ღვიძლის ექო, რაც ასევე ნორმა აღმოჩნდა (ნაღვლის ბუშტის კედელი არ არის გასქელებული, პერიქოლეცისტური სითხე არ არის, არ არის გაფართოებული ნაღვლის სადინარები).

ამასობაში ავადმყოფი განაგრძობს სიმპტომებს. ასევე უვითარდება მცირე ჰიპოტენზია და სინუსური ბრადიკარდია (დაახლოებით 50-ის ფარგლებში).

მოკლედ, რა უნდა გაუკეთდეს ავადმყოფს პირველი, მეორე, მესამე და ა.შ.

დიაგნოზის დასმას არ ვითხოვ, არამედ მართვის სტრატეგიას. თუ რაიმე ტესტი გჭირდებათ, იკითხეთ და თუ გაკეთებული ქონდა მოგახსენებთ.

ბლაინდ-ტორჩა-კილ
შენს კითხვაზე პასუხი ერია უკვე კაი ხანია, მარა არ დააკვირდი ალბათ.

Posted by: basa-ttt 22 Nov 2008, 09:36
QUOTE
65 წლის მდედრი მოდის ეპიგასტრიუმის და გულმკერდი ქვემო ნაწილის ტკივილით. ტკივილი დაიწყო ერთი დღის წინ. თან აქვს გულსრევა და ღებინება. ასევე აღენიშნება თავბრუსხვევა

გასტროსკოპიაზე არაფერი გიწერია-
იქნებ წყლულია ან გასტრიტი ან ეროზია-
თავბრუსხვევა რომ აქვს - სისხლმდენი წყლული არ იყოს...

კტ - ზე ყოველთვის არ ჩანს მცირე ზომის დეფექტები ლოწროვანზე.

Posted by: vano_t 22 Nov 2008, 10:43
basa-ttt
QUOTE
გასტროსკოპიაზე არაფერი გიწერია-
იქნებ წყლულია ან გასტრიტი ან  ეროზია-
თავბრუსხვევა რომ აქვს  -  სისხლმდენი წყლული არ იყოს...

კტ - ზე ყოველთვის არ ჩანს მცირე ზომის  დეფექტები ლოწროვანზე.

1) გასტრიტით ასე მძიმედ არ მოდის ავადმყოფი.
2) კუჭის წყლულის პერფორაცია შესაძლებელია და დიფერენციალურ დიაგნოზში დგას. მასეთი სისხლდენა რომ ქონოდა ან აღებინებდა სისხლს, ან მელენა ექნებოდა ან ხულული სისხლდენა სწორი ნაწლავიდა. თანაც ჰემოგლობინი მნიშვნელოვნად დაქვეითდება სისხლდენიდან 2-3 დღეში. ამიტომ, ეს არ არის აქ საალბათო დიაგნოზი საერთოდ.
3) რა გასტროსკოპიაზეა ლაპარაკი, როცა ავადმყოფი ჰიპოტენზიურია და ბრადიკარდიული?

Posted by: Blind_Torture_Kill 22 Nov 2008, 11:18
vano_t

ვერ ვნახე პასუხი

CBC-ში დაბალი გემოგლობინის და მაღალი ESR გარდა რამე ააქვს ?
ეკგ გამეორდეს და თუ ნორმაა

ენდოსკოპია გაუკეთდეს
ან ბარიუმ ეზოფაგოგრამა

Posted by: Cousteau 22 Nov 2008, 11:39
vano_t
intersting

გულმკერდის კტ არის?
ან განავალში/შარდში სისხლის ანალიზი?

საყლაპავი გამოკვლეულიაქვს? (MI-ს მიმიკას საყლაპავის დაავადებები იწვევენ + ედსის ცვლა შეიძლება შტაცკის რგოლმა ან საყლაპავის დივერტიკულმა მოგცეს)

Posted by: vano_t 22 Nov 2008, 12:30
Cousteau
QUOTE
გულმკერდის კტ არის?
ან განავალში/შარდში სისხლის ანალიზი?

საყლაპავი გამოკვლეულიაქვს? (MI-ს მიმიკას საყლაპავის დაავადებები იწვევენ + ედსის ცვლა შეიძლება შტაცკის რგოლმა ან საყლაპავის დივერტიკულმა მოგცეს)
გულმკერდის კტ არ გაკეთებულა. შარდის ანალიზი ნორმალური იყო. განავალი ნორმალური ქონდა. ამიტომ ანალიზი არ გაკეთებულა. არც საყლაპავი გამოკვლეულა. შემდეგი გამოკვლევა რაც გაკეთდა, სავარაუდო დიაგნოზი მიიღო.

Blind_Torture_Kill
QUOTE
CBC-ში დაბალი გემოგლობინის და მაღალი ESR გარდა რამე ააქვს ?
ეკგ გამეორდეს და თუ ნორმაა

ენდოსკოპია გაუკეთდეს
ან ბარიუმ ეზოფაგოგრამა
სხვა ლაბორატორიული დარღვევები არ ქონდა. კარდიოლოგმა ნახა და კარდიოლოგმა გული გამორიცხა. ენდოსკოპიას არასტაბილურ პაციენტს ვერ გაუკეთებ.

Posted by: mika9 22 Nov 2008, 13:24
vano_t

ინფექცია უნდა ეჭვი ალბათ.....

სალმონელაა????


Posted by: vano_t 22 Nov 2008, 14:12
mika9
QUOTE
ინფექცია უნდა ეჭვი ალბათ.....

სალმონელაა????

ინფექცია ნამდვილად პირველია დიფ-დიაგნოზში. ავადმყოფს აქვს სეფსისი, სანამ არ დაამტკიცებ სხვანაირად. სპეციფიურ ინფექციაზე ეჭვი ჯერ ადრეა ოღონდ. პირველ რიგში არასტაბილური ავადმყოფი უნდა დაასტაბილურო.

ასეთი ავადმყოფი უნდა იყოს ინტენსიურში. დიდი რაოდენობით კრისტალოიდი უნდა გადაუსხა და ფართო სპექტრის ანტიბიოტიკები დაიწყო. ოღონდ, ანტიბიოტიკების დაწყებამდე სისხლის დათესვა აუცილებელია.

ამ შემთხვევაში კონსულტანტ ქირურგს უნდოდა რაღაცის გამორიცხვა და ეს ტესტი გაუკეთა:user posted image

Posted by: Blind_Torture_Kill 22 Nov 2008, 17:12
QUOTE
სხვა ლაბორატორიული დარღვევები არ ქონდა. კარდიოლოგმა ნახა და კარდიოლოგმა გული გამორიცხა. ენდოსკოპიას არასტაბილურ პაციენტს ვერ გაუკეთებ.


სეფსისი თუა სისხლში სხვა ცვლილებები არ გაქ ?

ნაღვლის ბუშტია სურათზე ?

Posted by: basa-ttt 22 Nov 2008, 17:36
QUOTE
65 წლის მდედრი მოდის ეპიგასტრიუმის და გულმკერდი ქვემო ნაწილის ტკივილით. ტკივილი დაიწყო ერთი დღის წინ. თან აქვს გულსრევა და ღებინება. ასევე აღენიშნება თავბრუსხვევა. ავადმყოფს აქვს წარსული ისტორია კორონარების დაავადების, ჰიპერტენზიის და ჰიპერლიპიდემიის. კლინიკაში ნახულობს ლოკალურ ექიმს. გულმკერდის ქვემო ნაწილში ტკივილის გამო და რისკ ფაქტორების გამო, ეს ექიმი ავადმყოფს იღებს დაკვირვებისათვის საავადმყოფოში ინფარქტის გამოსარიცხად. უკვეთავს ენზიმებს, ეკგ-ს, გულმკერდის რენტგენს, ლეიკოციტებს, ერითროციტებს, ჰემოგლობინს, ელექტროლიტებს. ყველაფერი მოდის ნორმაში. ავადმყოფს აკვირდება 24 საათის განმავლობაში და ინფარქტის გამორიცხვის შემდეგ უშვებს სახლში.

რაის სეფსისი როცა ლეიკოციტებს ნორმას მიწერ?


QUOTE
2) კუჭის წყლულის პერფორაცია შესაძლებელია და დიფერენციალურ დიაგნოზში დგას. მასეთი სისხლდენა რომ ქონოდა ან აღებინებდა სისხლს, ან მელენა ექნებოდა ან ხულული სისხლდენა სწორი ნაწლავიდა. თანაც ჰემოგლობინი მნიშვნელოვნად დაქვეითდება სისხლდენიდან 2-3 დღეში.

ჯერ ერთი თუ წყლული თორმეტგოჯაშია - სისხლით ღებინება შეიძლება სულ არ იყოს
თუ სისხლდენა მცირეა - მელენაც შეიძლება არ იყოს. და ჰემოგლიბონი უცებ არ დაეცეს.
შეიძლება სისხლდენა წვრილი ნაწლავიდანაც იყოს - თორმეტგოჯასთან ახლოს..

QUOTE
3) რა გასტროსკოპიაზეა ლაპარაკი, როცა ავადმყოფი ჰიპოტენზიურია და ბრადიკარდიული?

ჯერ ერთი მცირე ჰიპოტენზი და ბრადიკარდია არაა უკუჩვებნება ენდოსკოპიის.
და საერთოდ სანამ დამძიმდებოდა
მაგ ექიმს პირველი ეგ უნდა გაეკეთებინა - უფრო სწორედ მეორე...
ქალი გულისრევას და ეპიგასტრიუმის არეში ტკივილს უჩივის
და გულის ინფარქტის გამორიცხვის მერე კუჭი არ უნდა გამოეკვლიათ?
ბრადიკარდია სწორედ კუჭის პათოლოგიას ახასიათებს.

QUOTE
ამ ტესტებიდან ერითროციტების დალექვის სიჩქარე მომატებულია 82-მდე (ამ ქალისათვის ნორმა შეიძლება ჩაითვალოს 50-მდე ალბათ).

ეს ვერ გავიგე.
ნორმა რამდენია ?-
ჩვენთან ედსი ნორმა - 12 - მდეა.

Posted by: Blind_Torture_Kill 22 Nov 2008, 17:42
QUOTE
ეს ვერ გავიგე. ნორმა რამდენია ?-ჩვენთან ედსი ნორმა - 12 - მდეა.


ასაკოვან ხალხში უფრო გაზრდილია
თან ანემიაც რომ აქვს ხელს უწყობს კიდევ ზრდას smile.gif

Posted by: mika9 22 Nov 2008, 17:44
basa-ttt

რა აუცილებელია ლეიკოციტოზი იყოს..............................

რატომგაც მგონია რომ სალმონელაა

ბრადიკარდია თან ტეკოციტებიც ნორმააა ..................................


Posted by: basa-ttt 22 Nov 2008, 17:44
QUOTE
თან ანემიაც რომ აქვს ხელს უწყობს კიდევ ზრდას

ანემიაზე ტოვარიშჩ ვანო გვიწერს რომ აქვს უმნიშვნელოოო

QUOTE
ჰემოგლობინი ცოტათი დაქვეითებულია 10 მგ/დლ (ნორმის ქვედა ზღვარი 11.5 ქალისათვის ამ ლაბორატორიის მიხედვით);


QUOTE
რატომგაც მგონია რომ სალმონელაა

სალმონელამ ისეთი ტრაწი იცის -
არ შეგეშლება
gigi.gif

Posted by: Blind_Torture_Kill 22 Nov 2008, 17:50
QUOTE
ბრადიკარდია თან ტეკოციტებიც ნორმააა


მაგან იცის ლეიკოპენია smile.gif თუ ტიფს გულისხმობ

Posted by: basa-ttt 22 Nov 2008, 17:50
QUOTE
მაგან იცის ლეიკოპენია  თუ ტიფს გულისხმობ

მუცლის ტიფმა იცის ბრადიკარდია
და ლეიკოპენია
მაგრამ სიცხე უნდა ჰქონდეს.

ლეიკოციტარულ ფორმულას ნორმას გვიწერს ამხ ვანო.

Posted by: vano_t 23 Nov 2008, 01:50
Blind_Torture_Kill
QUOTE
სეფსისი თუა სისხლში სხვა ცვლილებები არ გაქ ?

ნაღვლის ბუშტია სურათზე ?
სეფსისი განსაზღვულია როგორც SIRS (systemic inflammatory response syndrome) პლიუს ინფექციის საეჭვო კერა. SIRS-ის დროს უნდა გქონდეს 4-იდან 2 რამ. ამ პაციენტს ქონდა მარტო ტაქიპნეა. ლეიკოციტოზი (ან ლეიკოპენია) და ტემპერატურა 36-ზე ნაკლები ან 38-ზე მეტი ასევე ერთ-ერთი განმსაზღვრელი ნიშნებია, მაგრამ არ არის აუცილებელი მათი არსებობა. ამ პაციენტს რომ ქონოდა ტაქიკარდია, მაშინ უკვე სეფსისის განსაზღვრებას დააკმაყოფილებდა. თუცა, სეფსის განსაზღვრებას თუ არ აკმაყოფილებს პაციენტი, არ ნიშნავს, რომ სეფსისი არ გაქვს. ჰიპოტენზია მნიშვნელოვანი ნიშანია, რომ სეპსისი დადგეს #1 დიფ-დიაგნოზში. რა თქმა უნდა დეჰიდრატაციას შეუძლია მნიშვნელოვანი ჰიპოტენზია გამოიწვიოს, მაგრამ დეჰიდრატაციულ ავადმყოფს წყალს გადაუსხამ და მოკვიტინდება. არადა, თუ ჩათვალე რომ ავადმყოფს მარტო დეჰიდრატაცია აქვს და სეფსისზე არ იფიქრე, მაშინ სეფსისი თუ აქვს ავადმყოფს და შენ ანტიბიოტიკები არ დაგიწყია, სიკვდილიანობის შანსი გაიზრდება. დაქტვეითებული წუთმოცულობა (გულის და ფილტვების სხვადასხვა დაავადებების დროს) რა თქმა უნდა მოგცემს ჰიპოტენზიას, მაგრამ კარდიოლოგმა გამორიცხა ეს მიზეზი და ეგ უკვე ვახსენე.

basa-ttt
QUOTE
ეს ვერ გავიგე.
ნორმა რამდენია ?-
ჩვენთან ედსი ნორმა - 12 - მდეა.
ტიპოგრაფიული შეცდომაა. 40-მდე უნდა იყოს. ედს-ის ნორმის ზედა ზღვარი დამოკიდებულია ასაკზე და სქესზე. შეიძლება იხმარო დაახლოებით ფორმულა ამისათვის:
მამაკაცის მქს. ედს=ასაკი/2
დედაკაცი მაქს. ედს=(ასაკი+10)/2, რაც ამ პაციენტში გაძლევს 37.5.

QUOTE
რაის სეფსისი როცა ლეიკოციტებს ნორმას მიწერ?

იხილე სრული პასუხი ზემოთ. მოკლედ, სეფსისისათვის არ არის აუცილებელი ლეიკოციტოზი (ან ლეიკოპენია)

QUOTE
ჯერ ერთი თუ წყლული თორმეტგოჯაშია - სისხლით ღებინება შეიძლება სულ არ იყოს
თუ სისხლდენა მცირეა - მელენაც შეიძლება არ იყოს. და ჰემოგლიბონი უცებ არ დაეცეს.
შეიძლება სისხლდენა წვრილი ნაწლავიდანაც იყოს - თორმეტგოჯასთან ახლოს..
ჯერ ერთი წყლული შეიძლება იყოს კუჭშიც და თორმეტგოჯაშიც. მეორეც, თუ სისხლდენა მასე მცირეა, რომ არც მელენას გაძლევს, არც ჰემატემეზს და არც სისხლდენას სწორი ნაწლავიდან, მაშინ არათუ შეიძლება, არამედ ძაან ნელა დაეცემა ჰემოგლობინი (თუ ავადმყოფს თან არ აქვს ანემიების კიდევ სხვა მიზეზი). და რაც ყველაზე მთავარია, ჩემი პასუხი ეხებოდა იმას, რომ შენ სისხლდენა დაასახელა მიზეზად იმიტომ, რომ ავადმყოფს ქონდა თავბრუსხვევა. არცერთ ავადმყოფს არ განუვითარდება თავბრუსხვევა უცებ მასეთი მცირე ქრონიკული სისხლის დაკარგვით. ამაშია საქმე, თორემ, სისხლდენა რომ მცირე შეიძლება იყოს, ჩვენც გაგვიგია.

QUOTE
ჯერ ერთი მცირე ჰიპოტენზი და ბრადიკარდია არაა უკუჩვებნება ენდოსკოპიის.
და საერთოდ სანამ დამძიმდებოდა
აქეთ უკუჩვენებაა და რა გითხრა აბა. ჯერ ავადმყოფს ასტაბილურებენ ამისათვის. სითხეებს გადაუსხამენ, მოაკვიტინებენ და მერე შეიძლება გაუკეთონ. თანაც, ანესთეზიას. რომელიც გასტროსკოპიას ჭირდება, თავისთავად შეუძლია გამოიწვიოს ჰიპოტენზია და არ არის ლამაზი როცა წნევადაქვეითებულ ავადმყოფს კიდევ უფრო უქვეითებ წნევას. იტოგში, არ გაუკეთებიათ გასტროსკოპია და არც აღმოჩნდა საჭირო.

იმ სურათზე ქოლესცინტიგრაფიაა გაკეთებული, რასაც სხვანაირად HIDA scan-ს ეძახიან. ამ დროს ავამყოფს რადიოაქტიულ ნაერთს აძლევენ, რომელიც ღვიძლში გროვდება და შემდეგ ღვიძლის სადინარებით გამოიყოფა. მერე ნაწლავში იცლება ეს ყველაფერი. ნაღვლის ბუშტშიც შედის ეს ნაერთი, მაგრამ ქოლეცისტოკინინის მიცემის შემდეგ უცებ იწყება ნაღვლის ბუშტის დაცლა. ნორმაში ნაღვლის ბუშტის გადასროლის ფრაქცია (EF) არის 35 პროცენტზე მეტი. ამ სურათზე 13 % და ჩვენ ავადმყოფს ქონდა 11 პროცენტი. ანუ, ნაღვლის ბუშტი თითქმის არ იცლებოდა კონტრასტისაგან ქოლეცისტოკინინის მიცემის შემდეგ. დანარჩენი უნდა მიხვდეთ რატომ.

Posted by: basa-ttt 23 Nov 2008, 02:05
vano_t
ეს ყველაფერი რაც დაწერე -
გადასარევია-
მაგრამ გავიწყდებათ ის რის გამოც შემოვიდა კლინიკაში -

ტკივილი ეპიგასტრიუმის არეში

პირველად იყო ეს!
ტკივილი+გულისრევა+ღებინება+თავბრუსხვევა.



ისე კი ავადმყოფობის ისტორიაში არ წერ ქრონოლოგიას.
რამდენი დღე იყო კუჭის არეში ტკივილი,?
ჯერ იყო ტკივილი და მერე ღებინება თუ პირიქით ?-
მერამდენე დღეს გადაიყვანეს დიდი კლინკაში -?
სახლში რამდენი ხანი დაჰყო-?
წინა დღეებში რა ჭამა.?
ხომ არ სვამდა რამე პრეპარატებს, ვინაიდან ნამნეზშია არტ ჰიპრტენზია, გიდ.
მაგ ასპირინს ხომ არ ღებულობდა.

* * *
QUOTE
SIRS-ის დროს უნდა გქონდეს 4-იდან 2 რამ. ამ პაციენტს ქონდა მარტო ტაქიპნეა. ლეიკოციტოზი (ან ლეიკოპენია) და ტემპერატურა 36-ზე ნაკლები ან 38-ზე მეტი ასევე ერთ-ერთი განმსაზღვრელი ნიშნებია, მაგრამ არ არის აუცილებელი მათი არსებობა. ამ პაციენტს რომ ქონოდა ტაქიკარდია, მაშინ უკვე სეფსისის განსაზღვრებას დააკმაყოფილებდა. თუცა, სეფსის განსაზღვრებას თუ არ აკმაყოფილებს პაციენტი, არ ნიშნავს, რომ სეფსისი არ გაქვს. ჰიპოტენზია მნიშვნელოვანი ნიშანია, რომ სეპსისი დადგეს #1 დიფ-დიაგნოზში

ცოტა უცნაური მიდგომა გაქვთ -
რაღაც ტესტურ გამოცდას მაგონებს..
ჰიპოტენზია ამ შემთხვევაში უკვე გართულებაა -
ანუ მერე განვითარებული სიმპტომია.
ანუ გამოდის, რომ თავიდანვე სეფსისი იყო - როცა ღებინებით და კუჭის ტკივილით შემოვიდა?
თუ მუცელში მოხდა რაღაც და მერე წავიდა სეფსისი?
მუცლის კუნთების რიგიდობა გაქვთ?
პალპაციით რაა?
მუცლის არეს პალპაცია ჩაუტარეთ?
რისი საპროექციო არე სტკივა ხელის დაჭერით?

Posted by: Blind_Torture_Kill 23 Nov 2008, 02:30
vano_t


ბილიარული დისკინეზია + ქოლანგიტი
აქ მაგ პაციენტს

სეფსისი რომ გაქ უნდა მიანიშნო რამით აბა ეგრე ნოსტრადამუსი ხომ არ ვარ
რითი გავიგოთ
CRPს მონაცემები გეთქვა
სისხლში ცვლილებები რა პონტში არ გაქ ვერ გავიგე

Posted by: vano_t 23 Nov 2008, 06:47
Blind_Torture_Kill
QUOTE
სეფსისი რომ გაქ უნდა მიანიშნო რამით აბა ეგრე ნოსტრადამუსი ხომ არ ვარ
რითი გავიგოთ
CRPს მონაცემები გეთქვა
სისხლში ცვლილებები რა პონტში არ გაქ ვერ გავიგე
მოკლედ არ კითხულობთ, რასაც ვამბობ. მე არ მითქვამს ავადმყოფს სეფსისი აქვს თქო. აი რა ვთვქვი, როცა მიკამ თქვა, რომ ინფექცია უნდა იეჭვიანოო: ინფექცია ნამდვილად პირველია დიფ-დიაგნოზში. ავადმყოფს აქვს სეფსისი, სანამ არ დაამტკიცებ სხვანაირად. სპეციფიურ ინფექციაზე ეჭვი ჯერ ადრეა ოღონდ. პირველ რიგში არასტაბილური ავადმყოფი უნდა დაასტაბილურო. ეს იმას ნიშნავს, რომ ავადმყოფს მკურნალობ ისე, როგორც სეფსისიანს, სანამ არ გამორიცხავ სეფსის და რატომ, უკვე ავხსენი. სისხლში ცვლილებები იმ პონტში არ გაქვს, რომ სეფსისი დროს არ არის აუცილებელი გქონდეს სისხლში ცვლილებები. CRP გაუკეთდა თუ არა, ვერ გეტვყი, თუმცა, ამას სადიაგნოსტიკო ღირებულება არ აქვს სეფსისი დროს. ნომრალური CRP არ გამორიცხავს სეფსის, მაგრამ, პროგნოზს განსაზღვრავს ზოგიერთი კვლევის მიხედვით. ალაბთ გაუკეთდა ავადმყოფს ეგ. მე არ მაქვს ეგ მონაცემი.

ნოსტრადამუსობა არ უნდა იმას, რომ ჰიპოტენზიურ ავადმყოფში, მუცლის ტკივილით და ღებინებით, სეფსისი და სეპტიური შოკი დასვა #1 ადგილეზე, მიუხედავად იმისა, თუ რას გაჩვენებს ლაბორატორია. აი ეს, კი არ უნდა დაიმახსოვრო, არამედ თუ პრაქტიკულ ექიმათ მოგიწევს მუშუაობა, ყოველთვის უნდა განიხილო ასეთი ავადმყოფის შემთხვევაში. არ უნდა ამას ლაპარაკი: ამ ავადმყოფს აქვს სეფსისი (მიუხედავად იმისა რა აღმოჩნდება საბოლოოდ კვლევებით) სანამ არ დამტკიცდება სხვანაირად.

შემდგომში, მიუხედავად იმისა, რომ ავადმყოფს ლაბორატორიული ცვლილებები არ ქონია და ტემპერატურაც ნორმალური იყო, სისხლის დათესვით გაიზარდა E. coli.

მოდი ასე ვილაპარაკოთ. კეისს ისე კი არ შევხედოთ როგორც ტესტს, არამედ როგორც პრაქტიკული ღირებულების რაღაცას. თუ გინახია სეფსისები (მე შემიძლია გითხრა, რომ უამრავი რაოდენობით სეპსისები მინახია), მაშინ არ იტყვი მაგას. პრაქტიკაში იმისთანა შემთხვევებს ნახავ, რომ ეს შემთხვევა გასწავლის თუ რატომ არ უნდა იყო ჯიუტი და რატომ უნდა გქონდეს ფართო დიაგნოზები და დიფ-დიაგნოზები როცა ავადმყოფთან გაქვს საქმე.

QUOTE
ბილიარული დისკინეზია + ქოლანგიტი
აქ მაგ პაციენტს
აღმავალი ქოლანგიტი ნამდვილად უნდა იყოს ყველა ასეთი ავადმყოფის დიფ დიაგნოზში, მიუხედავად იმისა, რომ ბილირუბინიც (ასეთი პაციენტების უმრავლესობას ბილირუბინი მაღალი აქვთ), ღვიძლის ენზიმებიც და მუცლის კომპიტერული ტომოგრაფია ნორმაშია. თუმცა ეს ტესტი (HIDA სკანირება) ვერ დასვამს მაგის დიაგნოზს. თუ კლინიკა და კტ არ გაძლევს ამის დიაგნოზს და დიაგნოზში მაინც ეჭვი გაქვს, მაშინ ERCP უნდა ავადმყოფს. შეიძლება ჩატარებოდა კიდეც ამ ავადმყოფს ეგ ტესტი, თუ ქოლესცინტიგრაფია ნორმალური იქნებოდა.

ბილიარული დისკინეზია ნამდვილად მოგცემს არანორმალურ ქოლესცინტიგრაფიას. მაგრამ ამ შემთხვევაში ამის დიაგნოზს აზრი არ აქვს, რომც ქონდეს ავადმყოფს ეს დაავადება. ბილიარული დისკინეზია (ანუ აკალკულოზური ქოლეცისტოპათია) გაძლევს ქრონიკულ განმეორებად ტკივილებს და არა მწვავე ღებინებას და ჰიპოტენზიას. ამ შემთხვევაში (თანაც ადრე სპეციალურად ვახსენე, რომ ქირურგს დიდი ეჭვი ქონდა ქოლეცისტიტზე) ქირურგმა გადაწყვიტა, რომ აკალკოლოზური ქოლეცისტიტი გამოერიცხა. ამ შემთხვევაში კი არა, საერთოდ ყოველთვის როცა ფიქრობ ქოლეცისტიტზე, აკალკულოზური იქვეა.

თავიდან კონსერვატორული ჩარევა დაიწყო. სითხეები და ანტიბიოტიკები. მაგრამ ანტიბიოტიკებმა არაფერი ქნა მარტო. მერე ქირურგმა გააკეთა ქოლეცისტექტომია და ავამდყოფიც მალე გამოკეთდა.

უქვო ქოლეცისტიტი ქოლესცინტიგრაფიაზე გამოსახულებას ბილიარული დისკინეზიის მსგავსად იძლევა, იმიტომ რომ ამ დაავადების პათოგენეზში ადანაშაულებენ ნაღვლის ბუშტის შეკუმშვის მწვავე დარღვევას და ატონიას, რაც ერთგვარი მწვავე "დისკინეზიაა".

Posted by: vano_t 23 Nov 2008, 07:39
ბასა-ტტტ
QUOTE
მუცლის  არეს  პალპაცია ჩაუტარეთ?
რისი საპროექციო არე სტკივა  ხელის დაჭერით?
ერთადერთი ჭეშმარიტი პრეტენზია შენგან ამ დღეებშ smile.gif რა თქმა უნდა მუცლის ფიზიკური გამოკვლევა ჩაუტარდა, ოღონდ გამოკლვევის შედეგი გამომრჩა. პაციენტს ქონდა ეპიგასტრიუმის და მარჯვენა ზემო კვადრატის მტკივნეულობა პალპაციის დროს. ასევე ძალიან დაქვეითებული ნაწლავის ხმა. რიგიდობა არ ქონდა. არც rebound (ქართულად არ ვიცი რა ქვია ამას, მარა ეს არის ტკივილის გაძლიერება, როცა მუცელს დააწვები და მერე უცებ მოაცილებ ხელს).

QUOTE
ისე კი ავადმყოფობის  ისტორიაში არ წერ ქრონოლოგიას.
რამდენი დღე იყო   კუჭის არეში ტკივილი,?
ჯერ იყო ტკივილი  და მერე ღებინება თუ პირიქით ?-
მერამდენე დღეს გადაიყვანეს დიდი კლინკაში -?
სახლში რამდენი ხანი დაჰყო-?
წინა დღეებში რა ჭამა.?
ხომ არ სვამდა რამე პრეპარატებს,  ვინაიდან ნამნეზშია არტ ჰიპრტენზია, გიდ.
მაგ ასპირინს ხომ არ ღებულობდა.
მე რაც ჩავთვალე საჭიროდ, ის დავწერე. რა თქმა უნდა ფიზიკური გამოკველვა მუცლის უნდა დამეწერა. თუ შენ დამატებითი ინფორმაცია გჭირდება იკითხე და თუ იქნა გაკეთებული არ დაგიმალავ.

QUOTE
QUOTE
SIRS-ის დროს უნდა გქონდეს 4-იდან 2 რამ. ამ პაციენტს ქონდა მარტო ტაქიპნეა. ლეიკოციტოზი (ან ლეიკოპენია) და ტემპერატურა 36-ზე ნაკლები ან 38-ზე მეტი ასევე ერთ-ერთი განმსაზღვრელი ნიშნებია, მაგრამ არ არის აუცილებელი მათი არსებობა. ამ პაციენტს რომ ქონოდა ტაქიკარდია, მაშინ უკვე სეფსისის განსაზღვრებას დააკმაყოფილებდა. თუცა, სეფსის განსაზღვრებას თუ არ აკმაყოფილებს პაციენტი, არ ნიშნავს, რომ სეფსისი არ გაქვს. ჰიპოტენზია მნიშვნელოვანი ნიშანია, რომ სეპსისი დადგეს #1 დიფ-დიაგნოზში

ცოტა უცნაური მიდგომა გაქვთ -
რაღაც ტესტურ გამოცდას მაგონებს..
ჰიპოტენზია ამ შემთხვევაში უკვე გართულებაა -
ანუ მერე განვითარებული სიმპტომია.
ანუ გამოდის, რომ თავიდანვე სეფსისი იყო - როცა ღებინებით და კუჭის ტკივილით შემოვიდა?
არ გამოდის მასე. შეიძლება თავიდანვე სეფსისი იყო, შეიძლება არა. ადამიანს შეიძლება ქონდეს ინფექციის წყარო არაბაქტერიემიით და რამოდენიმე დღე არ იყოს ავად, არამედ ქონდეს მხოლოდ ბანალური ინფექციის ნიშნები. სეფსისი არ ყალიბდება დროის ერთ კონკრეტულ მომენტში. სეფსისი ვითარდება ან ნელა ან სწრაფად. ამ ავადმყოფს შეიძლება ქონდა თავდაპირველი ჰოსპიტალიზაციის დროს ქონდა სეფსისი, შეიძლება არა. შეიძლება სხვა საავადმყოფოში გადაყვანის შემდეგაც არ ქონია სეფსისი. მთავრია ის, რომ ეს დიაგნოზი არის ერთ-ერთი უპირველესი. შენ რომ გეთქვა, რომ ავადმყოფი უნდა გამოკვლეული იქნას ჰიპოტენზიის მიზეზების ყველა მიმართულებით (სეპტიური შოკი, დეჰიდრატაცია, კარდგიოგენული შოკი და ა.შ.) და გარკვეული დიაგნოზები გამოგერიცხა (მაგალითად სისხლდენა, რაც შენ კი არ გამორიცხე, არამედ ჩართე დიაგნოზში) და რაც მთავარია პაციენტის სტაბილიზაცია მოგეხდინა უპირველესად, მაშინ აღარ შემოგედავებოდი.

ასეა თუ ისეა, დამთავრდა ეს შემთხვევა და გადავიდეთ ახალ შემთხვევაზე.

Posted by: vano_t 23 Nov 2008, 09:57
92 წლის დედაკაცი მიიღეს სავადმყოფოში არასპეციფიური ტკივილების გამოსაკვლევად. პაციენტი უჩივის ზურგის, გულმკერდის და მენჯ-ბარძაყის ტკივილს, ასევე გენერალიზებულ სისუსტეს. უარყოფს გულისრევას, ღებინებას და დიარეას. ავადმყოფი არის მოხუცთა სახლის წევრი. წარსული სამედიცინო ისტორიაში აღინიშნება მსუბუქი ფორმის ალცჰეიმერის დაავადება.
ფიზიკური გამოკვლევისას გარდა ალცჰეიმერის ნიშნებისა ავადმყოს პალპაციისას აღინიშნება ტკივილი ხერხემლის ლუმბო-საკრალურ არეში. მოტორული ძალა ქვემო და ზემო კიდურებში თანაბრად არის დაქვეითებული 3.5/5 -ის ტოლია.
ლაბორატორიული მონაცემები: Ca-8.0 მგ/დლ (მცირე ჰიპოკალცემია), ფოსფორი-2.3 მგ/დლ (მცირე ჰიპოფოსფორემია), ალბუმინი 3.7 მგ/დლ, გლობულინი 2.9 გ/დლ, ტუტე ფოსფატაზა 138 ერთეული/ლ, გულმკერდის რადიოგრაფია აჩვენებს მალების ძველ და მრავლობით მოტეხილობებს. ასევე ხერხემლის მალების ოსტეოპენიას. ბარძაყის რეტგენი უჩვენებს ძვლის სიმკრივის დაქვეითებას, კორტიკალური ძვლის გათხელებას და ტრაბეკულების განლევას. ასევე აღინიშნება ბარძაყის ყელის მიდამოში რადიოგამჭვირვალე ხაზები.
ყველაზე სავარაუდო დიაგნოზია:
1) პირველადი ჰიპერპარათიროიდიზმი
2) მრავლობითი მიელომა
3) D ვიტამინის ნაკლებობა
4) პეჯეტის ძვლის დაავადება

დაუსაბუთებელი პასუხები არ მიიღება. 25 %-ია შემთხვევით გარტყმის. ამიტომ პასუხი უნდა დასაბუთდეს.

Posted by: Blind_Torture_Kill 23 Nov 2008, 13:03
QUOTE
3) D ვიტამინის ნაკლებობა



QUOTE
Ca-8.0 მგ/დლ (მცირე ჰიპოკალცემია), ფოსფორი-2.3 მგ/დლ (მცირე ჰიპოფოსფორემია)


92 წლის დედაკაცი-ამას თან ოსტეოპოროზიც ექნება

Posted by: vano_t 23 Nov 2008, 15:15
QUOTE (Blind_Torture_Kill @ 23 Nov 2008, 13:03 )
QUOTE
3) D ვიტამინის ნაკლებობა



QUOTE
Ca-8.0 მგ/დლ (მცირე ჰიპოკალცემია), ფოსფორი-2.3 მგ/დლ (მცირე ჰიპოფოსფორემია)

რათა ვითომ? პეჯეტი, MM და პირველადი ჰიპერპარათირეოიდიზმი არ იწვევს ჰიპოკალცემიას ან ჰიპოფოსფატემიას?

Posted by: Blind_Torture_Kill 23 Nov 2008, 15:42
vano_t

პეჯეტი-აქ კალციუმი ნორმა გაქ ან შეიძლება მცირედ მომატებული იყოს (Roგრამაზე მარტო ოსტეოპენია არაა) + მაღალი ტუტე ფოსფატაზა
MM-ჰიპერკალცემია გაქ + მაღალი ტუტე ფოსფატაზა
პირველადი ჰიპერპარათირეოიდიზმი-ჰიპერკალცემია + ჰიპოფოსფატემია + მაღალი ტუტე ფოსფატაზა

Posted by: basa-ttt 23 Nov 2008, 20:07
QUOTE
პაციენტს ქონდა ეპიგასტრიუმის და მარჯვენა ზემო კვადრატის მტკივნეულობა პალპაციის დროს. ასევე ძალიან დაქვეითებული ნაწლავის ხმა. რიგიდობა არ ქონდა. არც rebound (ქართულად არ ვიცი რა ქვია ამას, მარა ეს არის ტკივილის გაძლიერება, როცა მუცელს დააწვები და მერე უცებ მოაცილებ ხელს).

ნაღვლის ბუშტის ექოზე ნორმას მიწერდი -
ქოლანგიტი შეიძლება იყოს, მაგრა ეს უნდა გამოჩენილიყო წესით ექოზე.
პალპაციით ემთხვევა -
მაგრამ თუ სანაღვლე გზების რაიმე პათოლოგიაა -ეს ექოზე ჩანს ხოლმე -
როგორც წესი.
rebound - შოტკინ ბლუმბერგი-
პერიტონიტის გამოსარიცხად.


QUOTE
მე რაც ჩავთვალე საჭიროდ, ის დავწერე. რა თქმა უნდა ფიზიკური გამოკველვა მუცლის უნდა დამეწერა. თუ შენ დამატებითი ინფორმაცია გჭირდება იკითხე და თუ იქნა გაკეთებული არ დაგიმალავ.

რამდენი დღის ანამნეზია?
დღეების რაოდენობა

QUOTE
შენ რომ გეთქვა, რომ ავადმყოფი უნდა გამოკვლეული იქნას ჰიპოტენზიის მიზეზების ყველა მიმართულებით (სეპტიური შოკი, დეჰიდრატაცია, კარდგიოგენული შოკი და ა.შ.) და გარკვეული დიაგნოზები გამოგერიცხა (მაგალითად სისხლდენა, რაც შენ კი არ გამორიცხე, არამედ ჩართე დიაგნოზში) და რაც მთავარია პაციენტის სტაბილიზაცია მოგეხდინა უპირველესად, მაშინ აღარ შემოგედავებოდი.

ეს ყველაფერი შენ თვითონ გამორიცხე-
და თანაც მცირედი ჰიპოტენზიაო-
მაინც რას ნიშნავს ეს მცირედი?
80 /50 ზე?
სისხლდენას არ გამოვრიცხავ - არც ახლა.

QUOTE
ასეა თუ ისეა, დამთავრდა ეს შემთხვევა და გადავიდეთ ახალ შემთხვევაზე

რაის დამთავრდა?
ახლა პაციენტი სადაა?





Posted by: vano_t 24 Nov 2008, 03:21
QUOTE
ახლა პაციენტი სადაა?
სახლში. დაუბრუნდა ნორმალურ ცხოვრებას.

ახალი ქეისი:

28 წლის გოგოს უტარებენ წინასაოპერაციო გამოკვლევას. გოგო დაკავებული იყო აქტიური სპორტით, რამაც გამოიწვიო მუხლის სახსრების დაზიანება და შესაბამისად ამზადებენ ართროსკოპიული ქირურგიისათვის. ლაბორატორიები აჩვენებს ნორმალურ PT-ს (პროთრომბინის დროს; PTT (ანუ partial thrombolplastin time) არის 44 წამი (ნორმა 25-34 წამი); თრომბოციტების რაოდენობა ნორმალურია; სისხლდენის დრო (bleeding time) ნორმალურია; სხვა ლაბორატორიული მონაცემებიც (CBC-ის ჩათვლით) ნორმალურია. ავადმყოფს არ აქვს წარსული ისტორია გაძლიერებული სისხლდენების და გენეტიკური ისტორიაც უარყოფითია. დეტალური გამოკითხვა ავლენს, რომ ავადმყოფს წარსულში ქონდა სპონტანური აბორტები 3-ჯერ. რა არის სავარაუდო დიაგნოზი და რა ტესტები ჭირდება ადამიანს დიაგნოზის დასასმელად?

მეორე ქეისი მარტივი, მაგრამ სასწავლად კარგი სურათია. სურთზე გამოსახულია შარდის ბუშტი. რა არის დიაგნოზი?

Posted by: basa-ttt 24 Nov 2008, 11:24
QUOTE
ახლა პაციენტი სადაა?

სახლში. დაუბრუნდა ნორმალურ ცხოვრებას.

ვერა ხართ რა-
ხან რეანიმაციაში აწვენთ -
ხან სახლში უშვებთ -
კუჭი დატოვეთ ალბათ გამოუკვლეველი სეფსისის მაძიებლებო.....
gigi.gif
* * *
QUOTE
28 წლის გოგოს უტარებენ წინასაოპერაციო გამოკვლევას. გოგო დაკავებული იყო აქტიური სპორტით, რამაც გამოიწვიო მუხლის სახსრების დაზიანება და შესაბამისად ამზადებენ ართროსკოპიული ქირურგიისათვის. ლაბორატორიები აჩვენებს ნორმალურ PT-ს (პროთრომბინის დროს; PTT (ანუ partial thrombolplastin time) არის 44 წამი (ნორმა 25-34 წამი); თრომბოციტების რაოდენობა ნორმალურია; სისხლდენის დრო (bleeding time) ნორმალურია; სხვა ლაბორატორიული მონაცემებიც (CBC-ის ჩათვლით) ნორმალურია. ავადმყოფს არ აქვს წარსული ისტორია გაძლიერებული სისხლდენების და გენეტიკური ისტორიაც უარყოფითია. დეტალური გამოკითხვა ავლენს, რომ ავადმყოფს წარსულში ქონდა სპონტანური აბორტები 3-ჯერ. რა არის სავარაუდო დიაგნოზი და რა ტესტები ჭირდება ადამიანს დიაგნოზის დასასმელად?

врожденный или приобретенный дефицит протромбина
нехватка витамина К.
ღვიძლსაც გამოიკვლევდა კაცი

QUOTE
მეორე ქეისი მარტივი, მაგრამ სასწავლად კარგი სურათია. სურთზე გამოსახულია შარდის ბუშტი. რა არის დიაგნოზი?

შარდის ბუშტის დივერტიკული.

Posted by: vano_t 24 Nov 2008, 11:57
basa-ttt
QUOTE
врожденный или приобретенный дефицит протромбина
нехватка витамина К.
ღვიძლსაც გამოიკვლევდა კაცი
ღვიძლს, კუჭს, ნაღვლის ბუშტს, ტვინს და ა.შ. არ იკვლევენ ასეთ შემთხვევებში (და არ დამიწყო ახლა გასტროკოაგულოგრამული და ქოლეცისტოკოაგულოგრამული რეფლექსები არ გაგიგიაო?)

არ არის სწორი. რაც გამორიცხა ლაბორატორიამ და ანამნეზმა, მაინცდამიანიც ის დიაგნოზები დასვი ახლა შენ.

QUOTE
შარდის ბუშტის დივერტიკული.
ამას ჩაგითვლი.

Posted by: basa-ttt 24 Nov 2008, 11:59
QUOTE
არ არის სწორი. რაც გამორიცხა ლაბორატორიამ და ანამნეზმა

ჰოდა დაწერე ნორმალურად ანამნეზი
მერე იტყვი უი ამის თქმა დამავიწყდაო.
პრეპარატებიდან რას იყენებდა ადრე?

Posted by: vano_t 24 Nov 2008, 12:06
QUOTE (basa-ttt @ 24 Nov 2008, 11:59 )
QUOTE
არ არის სწორი. რაც გამორიცხა ლაბორატორიამ და ანამნეზმა

ჰოდა დაწერე ნორმალურად ანამნეზი
მერე იტყვი უი ამის თქმა დამავიწყდაო.
პრეპარატებიდან რას იყენებდა ადრე?

წერია ყველაფერი. 28 წლის ჯანმრთელი კალათბურთელი ქალი. ლაბორატორიაში ყველაფერია ნათქვამი. ამიტომ, შეეცადა იქ ნუ იტყვი რამეს, რაშიც გიჭირს.

Posted by: mika9 24 Nov 2008, 12:07
vano_t

ანუკლეარული ანტისხეულები, ალბათ ლუპუსია

Posted by: vano_t 24 Nov 2008, 12:14
QUOTE (mika9 @ 24 Nov 2008, 12:07 )
ანუკლეარული ანტისხეულები, ალბათ ლუპუსია

ავადმყოფი ჯანმრთელია და რატომ ფიქრობ ლუპუსზე? ანამნეზი უარყოფითია და გამოკვლევითაც არაფერია. ლაბორატორიებიც ნორმალური აქვს, გარდა PTT-სა.

ისე, რაღაც ახლოს ხარ wink.gif

Posted by: mika9 24 Nov 2008, 12:19
vano_t

ანტიფოსპოლიპიდური სინდრომი, იმიტომ რომ იზოლირებულად Pთ მომატებული და მრავლობიტი აბორტები.

ხოდა ანტიპოსპოლიპიდური ანტისხეულებს ნახავ ან ვდრლ ინქება + ფალს პოზიტიური

Posted by: basa-ttt 24 Nov 2008, 12:26
QUOTE
(პროთრომბინის დროს; PTT (ანუ partial thrombolplastin time არის 44 წამი (ნორმა 25-34 წამი)

აჰა,
წესიერად ვერ დაწერ ?
შთაბეჭდილება შეიქმნა რომ პროთრომბინის დრო იყო გაზრდილი.
თუ ერთ მაჩვენებელს წერ ქართულად-
მაშინ დაწერე
PTT - ც ქართულად.

PTT მე მეგონა პროთრომბინის დროს ინგლისური ვარიანტი
წერეთ ფორუმზე ქართულად.

Posted by: vano_t 24 Nov 2008, 12:36
mika9
QUOTE
ანტიფოსპოლიპიდური სინდრომი, იმიტომ რომ  იზოლირებულად Pთ მომატებული და მრავლობიტი აბორტები.

ხოდა ანტიპოსპოლიპიდური ანტისხეულებს ნახავ  ან ვდრლ ინქება + ფალს პოზიტიური

დიაგნოზი სწორია, მაგრამ, VDRL არ გჭირდრება დიაგნოზში. ანტიკარდიოლიპინის ანტისხეულები უნდა ნახო შრატში და აუცილებლად უნდა ნახო ასევე ლუპუს ანტიკოაგულანტი. ლუპუს ანტიოკოაგულანტს როგორ დაადგენ?

კიდევ რამედნიმე კითხვა: ჰაგემანის ფაქტორის დეფიციტი არ შეიძლება იყოს?

ან, იქნება სპონტანური აბორტების მიზეზი სხა რამეა და ავადმყოფის PTT კიდევ იმიტომ არის გახანგრძლივებული, რომ ჰეპარინი იყო გარეული საკვლევ პლაზმაში? (მაგალითად, სისხლი ექთანმა აიღო heplock-იდან)

ამ რამეებს როგორ გამორიცხავ? ადვილი კითხვები არ არის, მარა ცოტას თუ მიდგები, აუცილებლად გაცემ პასუხს. ამიტომ ველოდები პასუხს ამ კითხვებზე wink.gif

ბასა-ტტტ
კაი, ყველაფერი მე დამაბრალე biggrin.gif

Posted by: basa-ttt 24 Nov 2008, 12:38
QUOTE
კაი, ყველაფერი მე დამაბრალე

აბა ამას რატომ დავწერდი?

QUOTE
врожденный или приобретенный дефицит протромбина
нехватка витамина К.

სახის ფერიც გვითხარი იმ ქალბატონის.

Posted by: vano_t 24 Nov 2008, 12:50
QUOTE (basa-ttt @ 24 Nov 2008, 12:38 )
სახის ფერიც გვითხარი იმ ქალბატონის.

ელექტრომაგნიტური გამოსხივების ხილულ სპექტრშია იმ ქალბატონის სახის ფერი.

დამთავრდა ეს კეისი. მიკამ გასცა პასუხი უკვე.

აბა დროებით. მე დავისვენე ახლა და ძაან არ ინერვიულო, არ უხდება ჯანმრთელობას wink.gif

Posted by: mika9 24 Nov 2008, 13:12
vano_t
ვდრლ ისე ტუფტაზე ვტქვი,,,, ტუ მაინც გაუკეტეს ფალს პოსიტივე იქნებოდატქო....

ჰაგემანი და სხვა დანარჩენები ეესე ზეპირად არ ვიცი გადავხედავ წიგნებში, ჰაგემანის დეფ ზაან იშვიატია, ისე ნებისმიერი ფაქტორის დეფ არსებობს მაგრამ ზააან დაბალი ინსიდენცი აქვს

Posted by: vano_t 24 Nov 2008, 20:29
mika9
QUOTE
ვდრლ ისე ტუფტაზე ვტქვი,,,, ტუ მაინც გაუკეტეს ფალს პოსიტივე იქნებოდატქო....

ჰაგემანი და სხვა დანარჩენები ეესე ზეპირად არ ვიცი გადავხედავ წიგნებში, ჰაგემანის დეფ ზაან იშვიატია, ისე ნებისმიერი ფაქტორის დეფ არსებობს მაგრამ ზააან დაბალი ინსიდენცი აქვს

ჰაგემანის ამბავში მართალი ხარ, ძაან იშვიათია. მარტო PTT-ს გახანგრძლივებას გაძლევს ისე და სხვანაირად ჯანმრთელია ადამიანი. არ არის დამახასიათებელი თრომბოზები და მისი გართულებები (სპონტანური აბორტების ჩათვლით).

ჰეპარინის არსებობას საკვლევ პლაზმაში ხშირად ამოწმენებენ, განსაკუთრებით როცა მარტო PTT ია გახანგრძლივებული და მიზეზის დადგენა გინდა.

ზოგადად, ფაქტორის დეფიციტის განსხვავება ფაქტორების ინჰიბიტორების არსებობისაგან ადვილია ტესტებში. და თუ ინჰიბიტორზე გაქვს ეჭვი, მაშინ უნდა დაადგინო ჰეპარინია პლაზმაში თუ სხვა რამ (მაგალითად ლუპუს ანტიკოაგულანტი). აი როგორ ხდება ამის დადგენა შენთვის მოგვინდია.

ახალი ქეისი. რა არის ქვემო ეკგ-ზე მოცემული?
user posted image

Posted by: basa-ttt 24 Nov 2008, 20:37
QUOTE
ახალი ქეისი. რა არის ქვემო ეკგ-ზე მოცემული?

იბრიდება პაციენტი-
ფიბრილაციაა.

Posted by: Blind_Torture_Kill 24 Nov 2008, 20:49
QUOTE
იბრიდება პაციენტი- ფიბრილაციაა.


ვეთანხმები

დეფიბრილატორი უნდა

Posted by: vano_t 24 Nov 2008, 21:00
QUOTE (Blind_Torture_Kill @ 24 Nov 2008, 20:49 )
QUOTE
იბრიდება პაციენტი- ფიბრილაციაა.


ვეთანხმები

დეფიბრილატორი უნდა

QRS კომპლექსები გაქვს და საიდან არის ფიბრილაცია? ფიბრილაცია არ არის.

დეფიბრილატორი დაჭირდება თუ არა (დიაგნოზს რომ დაადგენ მერე იფიქრე ამაზე) დამოკიდებულია იმაზე სიმპტომატურია თუ არა პაციენტი.

Posted by: basa-ttt 24 Nov 2008, 21:05
QUOTE
QRS კომპლექსები გაქვს და საიდან არის ფიბრილაცია?

აი სად ხედავ QRS კომპლექსებს?

QUOTE
დეფიბრილატორი დაჭირდება თუ არა (დიაგნოზს რომ დაადგენ მერე იფიქრე ამაზე) დამოკიდებულია იმაზე სიმპტომატურია თუ არა პაციენტი

ამას მიხვდები მაშინ, როცა დაინახავ პაციენტს

Posted by: Blind_Torture_Kill 24 Nov 2008, 21:06
მაშინ ვენტრიკულური ტაქიკარდიაა რომელიც გადავა ფიბრილაციაში
V1 ში გაქ ცოტა გამოკვეთილად
სხვაგან არ ჩანს

Posted by: texasuri jleta benzoxerxit 24 Nov 2008, 21:08
არა ფიბრილაცია არ არის,არც თრთოლვა.პარკუჭოვანი ტაქიკარდია...

Posted by: basa-ttt 24 Nov 2008, 21:17
QUOTE
მაშინ ვენტრიკულური ტაქიკარდიაა გადასული ფიბრილაციაში
V1 ში გაქ ცოტა გამოკვეთილად
სხვაგან არ ჩანს

მოჟეტ -
QRS (?) განიერია.
თუმცა ზოგან აბრადაკაბრაა და არა კომპლექსები
ასაკი პაციენტის -
რა წამლებს სვამდა. და ა.შ...

Posted by: Blind_Torture_Kill 24 Nov 2008, 21:25
basa-ttt
QUOTE
მოჟეტ -QRS (?) განიერია. თუმცა ზოგან აბრადაკაბრაა და არა კომპლექსები ასაკი პაციენტის -რა წამლებს სვამდა. და ა.შ...


ხო ჯერ ფიბრილაცია არაა მარა დაემართება wink.gif

აქ მგონი EKG ს მაგრა დავამუღამებ smile.gif

Posted by: zviadcardio 24 Nov 2008, 21:42
torsades de pointes გნებავთ TdP

Posted by: vano_t 24 Nov 2008, 21:59
zviadcardio
QUOTE
torsades de pointes                                      გნებავთ TdP

გავს, მაგრამ არ არის ტორსადები. დააკვირდი RR ინტერვალებს და დაინახავ, რომ ვარიაცია გაქვს მნიშვნელოვანი. ეს უნდა დაგეხმაროს დიაგნოზში.

ვენტრიკულურ ტაქიკარდიას (პოლიმორფულს) კი წააგავს ეს ეკგ, მარა მსგავს რამეს გაძლევს სხვა რაღაცაც smile.gif
* * *
basa-ttt
QUOTE
აი სად ხედავ  QRS კომპლექსებს?

ჭიაყელასავით რო იკლაკნებიან აღმა დაღმა, ეგენი ალფები არ არიან. მაგათ QRS -ები ქვიათ.

Posted by: Blind_Torture_Kill 24 Nov 2008, 22:20
დაწერე პასუხი რა
ან ისტორია მოაყოლე

Posted by: vano_t 24 Nov 2008, 22:23
Blind_Torture_Kill
QUOTE
დაწერე პასუხი რა
ან ისტორია მოაყოლე

პასუხი ეკგ-ზეა smile.gif

აგერ დამატებითი ეკგ იგივე პაციენტის (უფრო სწორად მსგავსი დიაგნოზის მაგრამ სხვა პაციენტის) ნორმალური მდგომარეობის დროს.
user posted image

Posted by: zviadcardio 24 Nov 2008, 22:27
მართალია კარგად არ დავაკვირდი
ეი ფიბი + პრეეგზიტაცია

Posted by: Blind_Torture_Kill 24 Nov 2008, 22:38
vano_t

ჰიპერკალემიის გამოა ?

ამ ინტერნეტმა დამტანჯა მე ამის პროვაიდერი ...

Posted by: vano_t 24 Nov 2008, 22:46
zviadcardio
QUOTE
მართალია კარგად არ დავაკვირდი
ეი ფიბი + პრეეგზიტაცია

up.gif შენ მაგი არ შეგეშლებოდა smile.gif

ანუ, ხალხს რომ ავუხსნათ, ავადმყოფს აქვს წინაგულების ფიბრილაცია და ამასთან ერთად დამატებითი გზა. ხოდა, ფიბრილაციის შედეგად წარმოშობილი იმპულსები გადიან არა AV კვანძით (რომელსაც შეუძლია იმპულსების გატარების ფიზიოლოგიური შემცირება) არამედ ამ დამატებითი კვანძით (რომლის რეფრაქტერული პერიოდი მოკლეა და არ გააჩნია იმპულსის შენელების უნარი). ხოდა ავადმყოფს უვითარდება ძალიან სწრაფი ტაქიკარდია, ფართო და სხვადასხვა ფორმის QRS კომპლექსებით. RR ინტერვალები არიან განსხვავებულები, რაც ნამიოკია ასეთი ტაქიკარდიისა (განსსხვავებული ფორმის ქრს კომპლექსებთან ერთად).

Posted by: Blind_Torture_Kill 24 Nov 2008, 22:52
vano_t

არა ძმაო კარდიოლოგიის მამები ხართ
EKGს ბრდღვნით

WPWითია ხო


http://imageshack.us
http://g.imageshack.us/img127/ecgresource54qf2.jpg/1/


Posted by: vano_t 24 Nov 2008, 23:58
Blind_Torture_Kill
QUOTE
არა ძმაო კარდიოლოგიის მამები ხართ
EKGს ბრდღვნით

WPWითია ხო


http://imageshack.us
http://g.imageshack.us/img127/ecgresource54qf2.jpg/1/

არა, მე არ ვარ კარდიოლოგიის მამა. დრ.ჯოჰანსენი და ზვიადკარდიო ნაღდად არიან.

რაც შეეხები ისრებს, არასწორ ადგილას გაქვს დასმული. ეგ ისრები უნდა იყოს P კბილების მერე ეგრევე, ე.წ. დელტა კბილზე. თუ დააკვირდები PR ინტერვალი არის ძალიან მოკლე (დაახლოებით 1 პატარა უჯრა, არადა უნდა იყოს მინიმუმ 3). ხოდა ქრს იწყება შედარებით ნაკლები დახრით და მერე უცებ მიდის მაღლა (დააკვირდი გუმკერდის განხრებს განსაკუთრებით). აი ის, ნელა რო იწყება, არის დელტა კბილი (ან ტალღა) და ეგ მიუთითებს იმას, რომ იმპუსლის ეგ ნაწილი წარმოიქმნება იმიტომ, რომ პირველად იმპულსი გადის დამატებით გზაში. მარა, აქ (ამ ნორმლურ ეკგ-ზე) საბოლოოდ იმპულსი ნორმალურად ვრცელდება AV კვანძის გავლით.

იმ ეკგ-ზე კი, რომ შევა დამატებიტ გზაში იმპულსი, მთლიანად ვრცელდება მერე პარკუჭებში მაქედან. იმიტომ არის ძაან ფართო ქრს კომპლექსები.

ერთი რამ უნდა დაიმახსოვრო, რომ ასეთ ავადმყოფს არავითარ შემთხვევაში არ უნდა მიცე AV კვანძის მაბლოკირებელი წამლები (მაგალითად ბეტა ბოლკატორი ან კალციუმის არხების ბლოკატორი ან ადენოზინი). AV კვანძი თუ დათრგუნე, მთლიანად დააკარგვინებ იმის შანსს, რომ არითმია ჩაცხრეს. ჩვეულებრივ სუპრავენტრიკულური ტაქიკარდიების დროს აძლევ AV კვანძის დამთრგუნველებს, მაგრამ აქ არავითარ შემთხვევაში. აქ შეგიძლია მიცე პროკაინამიდი ან ამიოდარონი (ისიც თუ ჰემოდინამიკურად სტაბილურია ავადმყოდი). სხვა შემთხვევაში უნდა dc კარდიოვერსია.

Posted by: Blind_Torture_Kill 25 Nov 2008, 00:41
vano_t

smile.gif

QUOTE
რაც შეეხები ისრებს, არასწორ ადგილას გაქვს დასმული. ეგ ისრები უნდა იყოს P კბილების მერე ეგრევე, ე.წ. დელტა კბილზე. თუ დააკვირდები PR ინტერვალი არის ძალიან მოკლე (დაახლოებით 1 პატარა უჯრა, არადა უნდა იყოს მინიმუმ 3). ხოდა ქრს იწყება შედარებით ნაკლები დახრით და მერე უცებ მიდის მაღლა (დააკვირდი გუმკერდის განხრებს განსაკუთრებით). აი ის, ნელა რო იწყება, არის დელტა კბილი (ან ტალღა) და ეგ მიუთითებს იმას, რომ იმპუსლის ეგ ნაწილი წარმოიქმნება იმიტომ, რომ პირველად იმპულსი გადის დამატებით გზაში. მარა, აქ (ამ ნორმლურ ეკგ-ზე) საბოლოოდ იმპულსი ნორმალურად ვრცელდება AV კვანძის გავლით.


აქ მაღალი T კბილი ვიგულისხმე ისრებით (დელტა არა)
ხოდა ჰიპერკალემია იმიტო დავწერე თავიდან

Posted by: vano_t 25 Nov 2008, 01:05
ეს რა არის და დიფ დიაგნოზი ამისა:

user posted image

Posted by: Blind_Torture_Kill 25 Nov 2008, 03:00
QUOTE
ეს რა არის და დიფ დიაგნოზი ამისა


SVT

Posted by: vano_t 25 Nov 2008, 03:36
Blind_Torture_Kill
QUOTE
QUOTE
ეს რა არის და დიფ დიაგნოზი ამისა


SVT

SVT კია, მარა რამდენნაირი SVT არსებობს და ამ კონკრეტულ შემთხვეაში კიდევ რომლებია შესაძლებელი და ეს კონკრეტული შემთხვევაში რომელია უფრო სავარაუდოდ?

Posted by: Blind_Torture_Kill 25 Nov 2008, 04:07
vano_t

P კბილი ან არ ჩანს ან მე არ მინდა დანახვა smile.gif AVNRT

HR-214
left axis deviation

დავწექი თორე თვალები მომიკვდა

ხვალე მე დავდებ ქეისებს

Posted by: vano_t 25 Nov 2008, 05:50
Blind_Torture_Kill
QUOTE
P კბილი ან არ ჩანს ან მე არ მინდა დანახვა smile.gif AVNRT

HR-214
left axis deviation

HR არის დაახლოებით 187 (300/1.6=187). მარტივად სიხშირის დათვლა ასე შეგიძლია. 300 უნდა გაყო დიდი კვადრატების (RR ინტერვალებს შორის რომ არის მოთავსებული) რიცხვზე. აქ RR-ს შორის მანძილი არის 1 დიდი და 3 პატარა კვადრატი. თითოეული პატარა კვადრატი არის 0.2 დიდი კვადრატი. ანუ, ჩვენს შემთხვევაში 300 ყოფ 1.6.

მარცხნივ გადახრა ვერ გექნება ამ შემთხვევაში. ამის მარტივი შემოწმება ასე ხდება: თუ aVF-ში (და მითუმეტეს III-ში) QRS-ის საერთო მიმართულება დადებითია, მაშინ მარცხნივ ვერ გექნება გადახრა. ვერც მარჯვნივ გექნება გადახრა. მარჯვნინ გადახრა რომ გქონდეს, მაშინ I-ში (და მითუმეტეს aVL-ში) უნდა გქონდეს უარყოფითი საერთო გადახრა. მოკლედ, ქრს-ის ღერძი სადღაც 0-სა და 90-ს შორის არის.

ისე კი სავარაუდოდ AVNRT უფროა ეგ. მაგრამ შენ არ აღნიშნე მიზეზები მაგისა. ის რომ P კბილი არ ჩანს (ისე ჩანს და ამ ეკგ-ზე არის მაგის ნიშანი) არ ნიშნავს რომ ტაქიკარდია AVNRT არის. P კბილი შეიძლება არ გქონდეს წინაგულოვანი ტაქიკარდიის დროსაც, როცა ტაქიკარდიის კერა ძალიან დისტალურად არის და ახლოა AV კვანძთან. ასევე შეიძლება არ გამოჩნდეს P კბილი (ანუ ჩაფლული იყოს ქრს-ში) junctional ტაქიკარდიის დროს.

წინაგულოვანი და junctional ტაქიკარდიები უბრალოდ იშვიათია (და შესაბამისად ნაკლებსავარაუდო) და თანაც junctional ტაქიკარდია არ არის ისე სწრაფი, როგორც AVNRT არის.

ნამიოკი თუ როგორ უნდა მოძებნო P კბილი: ამ ავდმყოფს რომ გადაუღო ეკგ ნორმის დროს, V1-ში ნახავ rQ კომპლექსს, მაგრამ ტაქიკარდიის დროს აქვს rQr' კომპლექსი მაგ განხრაში. ახლა ქე უნდა მიხვდე ა ხდება smile.gif

Posted by: vano_t 25 Nov 2008, 06:48
აბა ეს რაა? ადვილია წესით.
user posted image

Posted by: Blind_Torture_Kill 25 Nov 2008, 11:01
QUOTE
აბა ეს რაა? ადვილია წესით.


ებშტეინის გული

QUOTE
HR არის დაახლოებით 187 (300/1.6=187). მარტივად სიხშირის დათვლა ასე შეგიძლია. 300 უნდა გაყო დიდი კვადრატების (RR ინტერვალებს შორის რომ არის მოთავსებული) რიცხვზე. აქ RR-ს შორის მანძილი არის 1 დიდი და 3 პატარა კვადრატი. თითოეული პატარა კვადრატი არის 0.2 დიდი კვადრატი. ანუ, ჩვენს შემთხვევაში 300 ყოფ 1.6.


ეს შენი გამოთვლის მექანიზმი ვერ გავიგე sad.gif
ძაან პატარა კვადრატი ხომ არის 0.04 მაშინ 1500 = 1 წუთს ხო
ხოდა თუ RR ინტერვალი არ იცვლება მაშინ 1500 გავყოფთ მათ შორის მოთავსებულ პატარა უჯრების რაოდენობაზე და მივიღებთ HR

რავიცი ესე მაქვს ნასწავლი რომ როცა მაგალი R კბილი გაქ 1 განხრაში მაშინ მარცხნივ გადახრაა როცა მე 3 განხრაში მარჯვნივ

Posted by: vano_t 25 Nov 2008, 11:04
QUOTE (Blind_Torture_Kill @ 25 Nov 2008, 11:01 )
ებშტეინის გული

კი, ებშტეინის ანომალიაა.

Posted by: Blind_Torture_Kill 25 Nov 2008, 11:10
QUOTE
HR არის დაახლოებით 187 (300/1.6=187). მარტივად სიხშირის დათვლა ასე შეგიძლია. 300 უნდა გაყო დიდი კვადრატების (RR ინტერვალებს შორის რომ არის მოთავსებული) რიცხვზე. აქ RR-ს შორის მანძილი არის 1 დიდი და 3 პატარა კვადრატი. თითოეული პატარა კვადრატი არის 0.2 დიდი კვადრატი. ანუ, ჩვენს შემთხვევაში 300 ყოფ 1.6.


ეს შენი გამოთვლის მექანიზმი ვერ გავიგე sad.gif
ძაან პატარა კვადრატი ხომ არის 0.04 მაშინ 1500 = 1 წუთს ხო
ხოდა თუ RR ინტერვალი არ იცვლება მაშინ 1500 გავყოფთ მათ შორის მოთავსებულ პატარა უჯრების რაოდენობაზე და მივიღებთ HR

რავიცი აქ ესე მაქვს ნასწავლი რომ როცა მაღალი R კბილი გაქ 1 განხრაში + უარყოფითი q ან s კბილი მესამეში მაშინ მარცხნივ გადახრაა როცა პირიქით მარჯვნივ

Posted by: vano_t 25 Nov 2008, 11:40
QUOTE
ეს შენი გამოთვლის მექანიზმი ვერ გავიგე sad.gif
ძაან პატარა კვადრატი ხომ არის 0.04 მაშინ 1500 = 1 წუთს ხო
ხოდა თუ RR ინტერვალი არ იცვლება მაშინ 1500 გავყოფთ მათ შორის მოთავსებულ პატარა უჯრების რაოდენობაზე და მივიღებთ HR
ჩემი გამონათვალი იგივეა რაც შენი გამონათვალი. შენ 1500 ყოფ პატარა უჯრების რაოდენობაზე. მე 300 ვყოფ დიდი უჯრედების რაოდენობაზე. დიდი უჯრედი შეიცავს 5 პატარა უჯრედს. შესაბამისად, თუ გადახვალ პატარა უჯრედების გაყოფაზე, 300 ჯერ უნდა გაამრავლო 5-ზე. იტოგში ერთიდაიგივეა ეგ. მთავარია გარდაქმნა არ დაგავიწყდეს. თუ 300 ხმარობ, მაშინ პატარა უჯრედების რაოდენობა უნდა გადაიყვანო დიდ უჯრედების რაოდენობაში: ყოველი პატარა უჯრედი უდრის 0,2 დიდ უჯრედს. ჩვენ აქ გვაქვს 8 პატარა უჯრედი, რომელიც ქმნის 1,6 დიდ უჯრედს. ახლა გინდა 1500:8 გინდა 300:1,6 მიიღებ ერთიდაიგივე პასუხს.

QUOTE
რავიცი აქ ესე მაქვს ნასწავლი რომ როცა მაღალი R კბილი გაქ 1 განხრაში + უარყოფითი q ან s კბილი მესამეში მაშინ მარცხნივ გადახრაა როცა პირიქით მარჯვნივ
არავითარ შემთხვევაში არ გააკეთო მასე. თუ გინდა დაახლოებით გამოთვალო ქრს ღერძი, მაშინ ნახე როგორია საერთო გადახრის ნიშანი aVF-ში და I-ში. საერთო გადახრას თვლი შემდეგნაირად: აიღე ქრს კომპლექსი და ყველა დადებითი გადახრის უჯრედები შეკრიბე, მერე ყველა უარყოფითი გადახრის უჯრედები შეკრიბე და დადებითი გადახრის უჯრედებს გამოაკელი უარყოფითი გადახრის უჯრედები. თუ მიიღე + რიცხვი, მაშინ საერთო გადახრა ამ განხრაში დადებითია და თუ მიიღე - ნიშანი, მაშინ უარყოფითი. რომ დაადგენ საერთო გადახრებს, მერე გამოიყენე წესი: თუ aVF-ში საერთო გადახრა - და I-ში +, მაშინ გაქვს მარცხნივ გადახრა. თუ I-ში გაქვს - ნიშანი და aVF-ში + ნიშანი, მაშინ გაქვს მარჯვნივ გადახრა (ანუ ქრს ღერძი 90 გრადუსზე მეტია). თუ ორივე დადებითია, მაშინ ღერძი არის 0-სა და 90-ს შორის.

ჩვენს შემთხვევაში AVF-ში საერთო გადახრა იქნება +4-0=+4. I-ში საერთო გადახრა იქნება +8-2=+6. ანუ ორივე დადებითია და შესაბამისად ქრს ღერძი არის მოთავსებული 0-სა და 90-ს შორის.

შეგიძლია შედარებით ზუსტადაც გამოთვალო ღერძი თუ იცი ვექტორების შეკრების პარალელოგრამის წესი. მაგრამ ამისათვის საკოორდინატო სისტემა უნდა ააგო და აირჩიო ერთეული და იტოგში დრო ჭირდება. ამდენი არც გინდა რომ გამოთვალო პრაქტიკაში.

Posted by: Blind_Torture_Kill 25 Nov 2008, 11:51
QUOTE
ახლა გინდა 1500:8 გინდა 300:1,6 მიიღებ ერთიდაიგივე პასუხს.


ხო ეგ გავარკვიე
მე 7 უჯრა დავითვალე RRს შორის და მაგიტო ამერია

QUOTE
არავითარ შემთხვევაში არ გააკეთო მასე. თუ გინდა დაახლოებით გამოთვალო ქრს ღერძი, მაშინ ნახე როგორია საერთო გადახრის ნიშანი aVF-ში და I-ში. საერთო გადახრას თვლი შემდეგნაირად: აიღე ქრს კომპლექსი და ყველა დადებითი გადახრის უჯრედები შეკრიბე, მერე ყველა უარყოფითი გადახრის უჯრედები შეკრიბე და დადებითი გადახრის უჯრედებს გამოაკელი უარყოფითი გადახრის უჯრედები. თუ მიიღე + რიცხვი, მაშინ საერთო გადახრა ამ განხრაში დადებითია და თუ მიიღე - ნიშანი, მაშინ უარყოფითი. რომ დაადგენ საერთო გადახრებს, მერე გამოიყენე წესი: თუ aVF-ში საერთო გადახრა - და I-ში +, მაშინ გაქვს მარცხნივ გადახრა. თუ I-ში გაქვს - ნიშანი და aVF-ში + ნიშანი, მაშინ გაქვს მარჯვნივ გადახრა (ანუ ქრს ღერძი 90 გრადუსზე მეტია). თუ ორივე დადებითია, მაშინ ღერძი არის 0-სა და 90-ს შორის.

ჩვენს შემთხვევაში AVF-ში საერთო გადახრა იქნება +4-0=+4. I-ში საერთო გადახრა იქნება +8-2=+6. ანუ ორივე დადებითია და შესაბამისად ქრს ღერძი არის მოთავსებული 0-სა და 90-ს შორის.

შეგიძლია შედარებით ზუსტადაც გამოთვალო ღერძი თუ იცი ვექტორების შეკრების პარალელოგრამის წესი. მაგრამ ამისათვის საკოორდინატო სისტემა უნდა ააგო და აირჩიო ერთეული და იტოგში დრო ჭირდება. ამდენი არც გინდა რომ გამოთვალო პრაქტიკაში.


აი ამას ვერ ჩავწვდი smile.gif

Posted by: vano_t 25 Nov 2008, 11:58
QUOTE
აი ამას ვერ ჩავწვდი smile.gif
შენ მითხარი რას ვერ ჩაწვდი, და მე აგიხსნი. ოღონდ ახლა არა; ახლა ჩემი ძილის დრო მოვიდა biggrin.gif მანამდე ან P.M.-ში შემეკითხე და ან აქ და გიპასუხებ. ხოდა, ხვალე რომ მოვიცლი გავარკვიოთ ეგ საკითხი.

Posted by: Blind_Torture_Kill 25 Nov 2008, 19:52
QUOTE
შენ მითხარი რას ვერ ჩაწვდი, და მე აგიხსნი. ოღონდ ახლა არა; ახლა ჩემი ძილის დრო მოვიდამანამდე ან P.M.-ში შემეკითხე და ან აქ და გიპასუხებ. ხოდა, ხვალე რომ მოვიცლი გავარკვიოთ ეგ საკითხი.


გაიხარე

ახალი ქეისი

a farm worker is brought to the emergency department because of wheezing and signs of severe bronchospasm. he is confused and lethargic and has several episodes of vomiting while in hospital. soon he develops respiratory failure and is intubated.
PE - miosis , bradycardia + hypotension and increased bowel sounds

რა ჭირს ?

Posted by: vano_t 26 Nov 2008, 08:24
მოდი ელექტრული ღერძის გამოცნობის პრინციპებს დავდებ და ამის შემდეგ უნდა მიხვდე ადვილად.

1) გულის კუნთის შეკუმშვისას, როგორც იცი, მემბრანის გასწვრივ არსებული ელექტრული პოტენციალი იცვლება. მოსვენებული გულის კუნთის ბოჭკოს გარეთ არის უფრო დადებითი მუხტი და აგზნებული გულის კუნთის ბოჭკოს გარეთ არის უფრო უარყოფითი მუხტი. ეს იმიტომ, რომ აგზნებისას ნატრიუმის იონის მოძრაობა ხდება ქსოვილური სითხიდან უჯრედის შიგნით და შესაბამისად მიაქვს დადებითი მუხტი შიგნით, რაც იმის ტოლასია, რომ უარყოფით მუხტს ზრდის გარეთ. ახლა, როცა გული შეკუმშვას იწყებს და აგრძელებს, ზოგი ნაწილი აგზნებულ მდგომარეობაში და ზოგიც მოსვენებულში (მაგალითად როცა წინაგულების იწყებენ შეკუმშვას, წინაგულების ქვემო ნაწილი და პარკუჭი ისევ მოსვენებულ მდგომარეობაშია). შესაბამისად, იქ სადაც კუნთი იკუმშება ქსოვილური სითხე უარყოფით მუხტიანი თუ შეადარებ მოსვენებული კუნთის ქსოვილურ სითხეს. ეს ქმნის ე.წ. დიპოლს, ანუ 2 პოლუსიან სისტემას. ახლა რა არის მნიშვნელოვანი: ტრანსმემბრანული პოტენციალი სხვა რამეა და ეკგ-ს მიერ დაჭერილი პოტენციალები სხვა რამე. ეკგ იჭერს ზუსტად ამ დიპოლის მიერ გამოწვეულ ცვლილებებს.

2) დიპოლი არის ვექტორული სიდიდე. ეს იმას ნიშნავს, რომ მას გააჩნია მიმართულება და სიდიდე. დიპოლის მიმართულებას განსაზღვრავს ის, თუ სად არის მოთავსებული დიპოლის დადებით მუხტი და მისი უარყოფითი მუხტი. შეთანხმების თანახმად, დიპოლის მიმართულებად ითვლება დადებითი მუხტიდან უარყოფითი მუხტისაკე მიმართულება. ეს რომ კარგად გაიგო, ახლა მიმოიხედე შენს ოთახში. მაგ ოთახის ერთ-ერთ კუთხეში წარმოიდგინე რომ მოთავსებულია დადებითი მუხტი. მეორე კუთხეში წარმოსახვით მოათავსე უარყოფით მუხტი. ახლა ამ ორ წერტილს შორის გაავლე შემაერთებელი მონაკვეთი და დადებით მუხტთან დაუსვი ისრის ნიშანი. ხოდა ეგ მონაკვეთი არის შენი დიპოლი, რომელიც იწყება დადებითი მუხტიდან და მიდის უარყოფითი მუხტისაკენ. ეს უფრო სწორად არის დიპოლის მიმართულება. მაგრამ დიპოლს მიმართულების გარდა გააჩნია სიდიდეც. ამ სიდიდეს განსაზღვრავს დადებით და უარყოფით მუხტს შორის სხვაობა. რაც უფრო დიდია ეს სხვაობა, მით უფრო დიდია დიპოლის სიდიდე. ისევ გადავიდეთ ჩვენს წარმოსახვით შემთხვევაზე. პირველ მაგალითში ჩვენ მივიღეთ დიპოლის მიმართულება. ახლა რომ ცვალო კუთხეებში მოთავსებული მუხტების სიდიდეები და შესაბამისად მათ შორის განსხვავება, მიიღებ ან უფრო გრძელ მონაკვეთს, ან უფრო მოკლეს, მაგრამ ამ მონაკვეთების მიმართულება ყველა ვარიანტში ერთიდაიგივე იქნება. მხოლოდ მათი სიგრძე იქნება განსხვავებული. ეს სიგრძე ამოკიდებულია დიპოლის სიდიდეზე. რაც უფრო დიდია დიპოლის სიდიდე, მით უფრო გრძელ ისარს ხატავ.

3) ეკგ მთლიანად წარმოადგენს ამ დიპოლის ცვლილების დაფიქსირებას გარკვეულ ღერძზე. როცა გული იკუმშება, ყოველ მომენტში იცვლება იმ დიპოლის სიდიდე და მიმართულება, რომელიც გამოწვეულია შეკუმშული ნაწილის გარეთ არსებული უარყოფითი მუხტით და შეუკუმშავი ნაწილის გარეთ არსებული დადებითი მუხტით. კარგად რომ წარმოიდგინო ეს, წარმოიდგინე ასე. აიღე რაიმე ღერძი. მიაბი მას რეზინა და ეს რეზინა გაწელე რაღაც მიმართულებით და დაიწყე მოძრაობა ამ ღერძის ირგვლივ ხან მარცხნივ და ხან მარჯვნივ. თან ხან ახლოს მიდი ღერძთან (ანუ რეზინის გაჭიმვა შეამცირე) და ხან კიდევ დაცილდი ამ ღერძს (ანუ გაჭიმვა გაზარდე). აი როგროც ამ შემთხვევაში ყოველ მყისიერ მომენტში შეიცვლება რეზინის სიგრძე და მიმართულება, ზუსტად ისე იცვლება დიპოლის სიგრძეც და მიმართულებაც.

4) ახლა შემოგვაქვს ხელსაწყო, რომელსაც შეუძლია ამ დიპოლის სიდიდის დადგენა, ოღონდ მხოლოდ გარკვეული მიმართულებით. დავაუშვათ ავიღე ეს ხელსაწყო და გავაკეთე ასე: ამ ხელსაწყოს აქვს 2 ელექტროდი: ერთი + და ერთი - ელექტროდი. ამ ელექტრული დიპოლის ურთიერთქმედება ხელსაწყოს + და - ელექტროდებთან გადამყავს ვიზუალურ მდგომარეობაში. ხელსაწყოს როგორ გავაკეთებ მთლიანად დამოკიდებულია ჩემზე. ხოდა, ასე ვირჩევ ამ ყველაფერს. თუ + და - ელექტორის შემაერთებელი ხაზის გასწვრივ დიპოლის სიდიდეა მაღალია, მაშინ ეკრანზე მოხდება გარკვეული ნეიტრალური მდგომარეობიდან (რასაც ვუწოდებ იზოხაზს) წერილის მოძრაობა ზევით ან ქვევით (საით, ჯერ არ ვამბობ). ამ ხელსაწყოს ვაკეთებ ისე, რომ რაც უფრო ძლიერია დიპოლი, მით უფრო დიდი გადახრა ხდება წერტილისა იზოხაზიდან. ახლა მეორე დეტალს ვამატებ ხელსაწყოს. თუ დიპოლის მიმართულება ჩვენი ხელსაწყოს ელექტროდების + მიმართულებას ემთხვევა, მაშინ ამ წერტილს ვამოძრავებ იზოხაზის ზევით და თუ დიპოლის მიმართულება - ელექტროდის მიმართულებას ემთხვევა, მაშინ ამ წერტილს ვამოძრავებ იზოხაზის დაბლა. ეს ყველაფერი ჩემზეა ისე დამოკიდებული. მე ხელსაწყოს როგორც მინდა ისე გავაკეთებ. შემიძლია, მაგალითად, ხელსაწყო ისე გავაკეთო, რომ უფრო ძლიერმა დიპოლმა უფრო ნაკლები გადახრა მოგცეს და - ელექტროდისკენ მიმართულმა დიპოლმა მოგცეს გადახრა იზოხაზის ზევით. ანუ, ყველაფერი პირიქით გავაკეთო. მაგრამ ეკგ-ს მანქანაში დღეს ისე შეთანხმებაა, როგორიც პირველად ავღწერე.

5) ახლა რაზეა დამოკიდებული წერტილის იზოხაზიდან გადახრის სიდიდე? რა თქმა უნდა დიპოლის აბსოლიტურ სიდიდეზე (ანუ სიგრძეზე). მაგრამ, უყურე რა ხდება. შენ შენი ხელსაწყოს ელექტორები ისე შეიძლება მოათავსო, რომ + და - ელექტროდების შემაერთებელი ხაზის მიმართულება A დიპოლის მიმართულების პარალელურია. ამ შემთხვევაში დაფიქსირდება მაქსიმალური გადახრა ხელსაწყოს მიერ. მაგრამ, ელექტროდების შემაერთებელი ხაზი შეიძლება მოატრიალო და ამ A დიპოლის მიმართულებასთან რაღაც ღერძი შექმნა. რაც უფრო მოატრიალებ, მით უფრო ნაკლებ სიდიდეს დაიჭერს შენი ხელსაწყო ერთიდაიმავე დიპოლისათვის. ისევ ანალოგია. დავუშვათ უყუებ ისარს, რომელიც შენს წინ დევს ისე, რომ ისრის მაქსიმალურ სიგრძეს ხედავ. ისარი არის დიპოლი. შენ ხარ ხელსაწყო, რომელიც აღიქვავს დიპოლს და შენს მიერ დანახული ისრის სიგრძე დამოკიდებულია იმაზე, თუ რა გრადუსით მოატრიალებ მაგ ისარს შენი ფრონტალური სიბრტყის მიმართ. თუ ისრის მიმართულება პარალელურია ფრონტალური სიბრტყისა, მაშინ მაქსიმალური სიგრძის ისარს ხედავ. თუ ისრის მიმართულება პერპენდიკულარულია ამ სიბრტყის, მაშინ წერტილს ხედავ მარტო და მეტს არაფერს. ზუსტად ესეა აქაც. თუ ელექტროდების მიმართულება "უყურებს" დიპოლის მიმართულებას მაშინ როცა ორივე პარალელურია, მაშინ გაქვს მაქსიმალური გადახრა იზოხაზიდან. თუ დიპოლის მიმართულება პერპენდიკულარულია ელექტროდების შემაერთებელი ხაზის მიმართულების, მაშინ ეს ხელსაწყო ვეღარ "ხედავს" ამ დიპოლს და იზოხაზზეა ჩვენი მოძრავი წერტილი. იმედია აქამდე გაიგე.

6) ახლა პრინციპში ბოლო ნაწილი დარჩა. იმისათვის რომ ამ დიპოლის მოძრაობაზე საკმაო წარმოდგენა შეგექმნას, საჭიროა რამოდენიმე მიმართულებით დადებული ელექტროდები და ყველა ამ ელექტროდის მიერ ჩაწერილი გადახრების ანალიზი. იგივე ადამიანის შემთხვევა ავიღოთ. მაგალითად მე ვაკვირდები ისარს, რომელიც არამარტო მიმართულება იცვლის, არამედ სიგრძესაც. როცა ეს ისარი სიგრძესაც იცვლის, მაშინ საერთოდ ძნელია სიტუაციის შეფფასება მარტო მე თუ დავაკვირდები ისარს. წარმოიდგინე, რომ ისარს თავიდან მაქსიმალური სიგრძისას ვხედავ და შემდეგ ეს ისარი იწყებს ტრიალს. ტრიალიტ ისრის ხილული სიგრძე შემცირდება, მაგრამ ჩვენი ისარი ისეთი ჭკვიანია, რომ ტრიალის სიჩქარის შესაბამისად ისე იზრდის თავის სიგრძეს, რომ ხილული სიგრძე უცვლელი რჩება. ამას მე ვერ დავადგენ. მაგრამ, თუ მყავს მეორე დამკვირვებელი, რომელიც სხვა კუთხით უყურებს ამ ყველაფერს, მაშინ ის დაინახავს სიგრძის ცვლილებას. იტოგში, ეკგ-ს შემთხვევაშიც, რაც უფრო სხვადასხვა მიმართულებიდან უყურებენ დიპოლს ელექტროდები, მით უფრო მეტი ინფორმაციის მიღება შეგიძლია. ზუსტად ეს არის სხვადასხვა განხრა და მეტი არაფერი.

7) გადავედით ქრს-ის ღერძის განსაზღვრაზე. ამისათვის, ჯერ უნდა დავადგინოთ რა არის დიპოლის საშუალო სიდიდე, რომელსაც ხედავს რომელიმე განხრა. საშუალო სიდიდეს ასე განვსაზღავ. დავთვლი იზოხაზის მაღლა არსებულ გადახრების სიდიდეს და ამ სიდიდეებს შევკრებ. მერე დავთვლის იზოხაზის დაბლა არსებული გადახრების სიდიდეებს და ამათაც შევკრებ. შემდეგ იზოხაზის ზევით არსებულ გადახრებს გამოვაკლებ იზოხაზის ქვევით არსებულ გადაზრებს და რასაც მივიღებ ის იქნება საშუალო სიდიდე, რასაც მოცემული განხრა ხედავს. დავუშვათ AVF განხრაში გაქვს qRSr' კომპლექსი. q კბილი ჩადის 3 კვადრატით იზოხაზის დაბლა; R კბილი ადის 7 კვადრატით იზოხაზის მაღლა; S კბილი ჩადის 5 კვადრატით იზოხაზის დაბლა და r' ადის 4 კვადრატით იზოხაზის მაღლა. განსაზღვრების თანახმად, ჩვენი დიპოლის საშუალო სიდიდე AVF-ში არის (7+4)-(3+5)=+3 მილივოლტი. ანუ ეს იმას ნიშნავს, რომ ჩვენი ქრს დიპოლი AVF-ს მიერ საშუალოდ იქნა დანახული როგორც 3 მილივოლტის სიდიდის დიპოლი, რომელიც მიმართულია AVF-ს დადებითი მიმართულებისაკენ. ეს სიდიდე რომ მიგეღო -7 მილივოლტი, მაშინ მაგ დიპოლი საშუალოდ დანახული იქნებოდა AVF-ს მიერ როგორც 7 მილივოლტის სიდიდის დიპოლი, რომელიც მიმართული იქნებოდა AVF-ს უარყოფითი ელექტროდისაკენ.

8) ახლა ერთი ნაბიჯიღა დარჩა. უბრალოდ ასეთი საშუალო სიდიდეები უნდა გამოითვალო 2 სხვადასხვა განხრისათვის. ოღონდ, ეს ორივე განხრა აიღე ან I, II და III-დან ან AVF, AVL, AVR-დან. A-გაძლიერებული განხრებია. ეს იმას ნიშნავს, რომ ამ განხრების მიერ დანახული სიგნალის გაძლიერება ხდება, სანამ მისი ჩაწერა მოხდება. ანუ, თუ დავუშვათ განხრამ სინამდვილეში დაინახა 2 მილივოლტი სიგნალი, მაშინ ამ სიგნალს გააორმაგებს, რომ ადვილი დასანახი იყოს. ამიტომ A-განხრების შედარება ჩვეულებრივ განხრებთან არ შეიძლება. შენიშვნა: ეს არ შეიძლება როცა გინდა ზუსტი ღერძის განსაზღვრა. მაგრამ, დაახლოებით თუ საზღვრავ ღერძს (როგროც წინა პოსტში გაჩვენე), მაშინ არ გჭირდება ერთნაირი განხრების არჩევა.

9) ახლა უშუალოდ ვაჩვენოთ როგორ ხდება ღერძის გამოთვლა. დავუშვათ III განხრაში საშუალო გადახრა იყო -3 მილივოლტი და I განხრაში იყო +6 მილივოლტი. ახლა რაც გევალება არის ის, რომ მოძებნო ვექტორი, რომლის პროექციაც ამ განხრებში მოგცემს მასეთ გადახრებს. ჯერ ავირჩე მასშტაბს. შემდეგ III განხრის უარყოფითი ღერძისაკენ გადავთვლი 3 მასშტაბის შესაბამის უჯრას. მერე I განხრის დადებითი ღერძისაკენ გადავთვლი 6 მასშტაბის შესაბამის უჯრას. შემდეგ ამ წერტილებიდან ავღმართავ პერპენდიკულარებს და მათი გადაკვეთის წერტილი იქნება საძიებელი ვექტროის ბოლო წერტილი. საწყისი წერტილი არის III და I ღერძის გადაკვეთის წერტილი. ის წითელი ვექტორი (რომელიც შეესაბამება ქრს-ის საშუალო დიპოლს) მოგცემს მხოლოდ ისეთ საშუალო სიდიდეებს, რაც ავღწერეთ ამ მაგალითში. შემდეგ გრადუსმთვლელით ნახავ რა გრადუსს ქმნის I ღერძთან და ეგ იქნება შენი ქრს-სი ღერძიც.

Posted by: LUKA-BRAZI 27 Nov 2008, 18:13
გაუმარჯოს ყველას... დიდი ხანია აქ აღარ ვყოფილვარ და უკვე ამდენი გვერდი შეგივსიათ... ყოჩაღ ბიჭებო! wink.gif smile.gif

Blind_Torture_Kill
აღნიშნული სიმპტომები გავს მწვავე პანკრეატიტს.... რაც მაგ კაცს ჭირს სავსებით შესაძლებელია გამოწვეული იყოს პანკრეასის ფერმენტების გადასვლით სისხლში და სუნთქვის ცენტრის გაღიზიანებით... ასევე მწვავე პანკრეატიტის დროს აღინიშნება ვაგუსის ირიტაცია, რაც სუნთქვის გაძნელებითა და ბრადიკარდიით ვლინდება, რამაც თავის მხრივ შეიძლება გამოიწვიოს ჰიპოტენზია, კოლაფსი –> შოკი... + ამას, ქეისში წერია რომ პაციენტს ქონდა ღებინება... ესეც მწვავე პანკრეატიტის ნიშანია... ასევე სხვა ჩამოთვლილი სიმპტომებიც ამას მიუთითებს.... შესაძლოა მწვავე პანკრეატიტი შენი ქეისისთვის სჭორი პასუხი არ არის, მაგრამ სიმპტომებით კი ძალიან გავს.

vano_t
კარგად გაქვს ყველაფერი ახსნილი...
მომეწონა ძალიან!

Posted by: Cousteau 27 Nov 2008, 19:06
QUOTE (Blind_Torture_Kill @ 25 Nov 2008, 19:52 )

a farm worker is brought to the emergency department because of wheezing and signs of severe bronchospasm. he is confused and lethargic and has several episodes of vomiting while in hospital. soon he develops respiratory failure and is intubated.
PE - miosis , bradycardia + hypotension and increased bowel sounds

რა ჭირს ?

ბარემ ვარიანტებიც დადე smile.gif
ის ჭირს რაც ესეთ კეისებში farm worker-ებს ემართებათ ან რამე ინფექციური არ რამე პნეუმოკონიოზი (მაგრამ უფრო ინფექცია)
მეზარების ქექვა user.gif

Posted by: vano_t 27 Nov 2008, 22:00
LUKA-BRAZI
გაუმარჯოს. სად დაიკარგე?

QUOTE
აღნიშნული სიმპტომები გავს მწვავე პანკრეატიტს.... რაც მაგ კაცს ჭირს სავსებით შესაძლებელია გამოწვეული იყოს პანკრეასის ფერმენტების გადასვლით სისხლში და სუნთქვის ცენტრის გაღიზიანებით... ასევე მწვავე პანკრეატიტის დროს აღინიშნება ვაგუსის ირიტაცია, რაც სუნთქვის გაძნელებითა და ბრადიკარდიით ვლინდება, რამაც თავის მხრივ შეიძლება გამოიწვიოს ჰიპოტენზია, კოლაფსი –> შოკი... + ამას, ქეისში წერია რომ პაციენტს ქონდა ღებინება... ესეც მწვავე პანკრეატიტის ნიშანია... ასევე სხვა ჩამოთვლილი სიმპტომებიც ამას მიუთითებს.... შესაძლოა მწვავე პანკრეატიტი შენი ქეისისთვის სჭორი პასუხი არ არის, მაგრამ სიმპტომებით კი ძალიან გავს.
შესაძლებელი არის, მაგრამ ნაკლებ სავარაუდოა ეგ. პანკრეატიტმა არ უნდა მოგცეს wheezing თუ რა თქმა უდნა არ გართულდა ასპირაციით და პნევმონიით. ეს კაცი კიდევ ჯერ მოდის wheezing-ით და გონების დაბინდვით (არც ამას მოგცემს თავიდან პანკრეატიტი). გარდა ამისა, ამ კაცს აქვს მიოზი, რასაც პანკრეატიტი მარტო ვერ ახსნის. და რაც მთავარია, პანკრეატიტისათვის დამახასიათებელია საკმაოდ ძლიერი მუცლის ტკივილი, უმტესად ეპიგასტრიუმში. ამ პიროვნებას კი ეგ არ აქვს.

Cousteau
QUOTE
ის ჭირს რაც ესეთ კეისებში farm worker-ებს ემართებათ ან რამე ინფექციური არ რამე პნეუმოკონიოზი (მაგრამ უფრო ინფექცია)
პეუნომოკიოზი ქრონიკულია. შენ ალბათ hypersensitivity pneumonitis გაქვს მხედველობაში. ან რაიმე inhalation injury. ესენი, როგორც წესი, ბრადიკარდიას, მიოზს და ჰიპოტენზიას არ მოგცემენ.

კიდევ რა არის დამახასითებელი ასეთი ჯგუფისათვის? რასთან შეიძლება შეხება ქონდეთ და რამ შეიძლება ასეთი სურათი მოგცეს? (ცხოველთან იქნება ეს შეხება, ფრინველთან თუ შხამ-ქიმიკატთან, ყველაფერი უნდა გაითვალისწინო და ნახო რამ შეიძლება მოგცეს ეს სიმპტომები).

მიოზი მნიშვნელოვანი სიმპტომია აქ.

Posted by: LUKA-BRAZI 27 Nov 2008, 22:29
vano_t
QUOTE
გაუმარჯოს. სად დაიკარგე?

გაგიმარჯოს ვანო, როგორ ხარ? სად დავიკარგე და ნევროლოგიის კურაცია მქონდა + გამოცდა და რეფერატები, თან ISP–ის გამოცვლაც მინდა თორემ სულ აღარ მაქვს ინტერნეტი, ეხლაც ათასი ხრიკებით ვარ აქ შემოპარული biggrin.gif
პანკრეატიტზე გეთანხმები, ტკივილის არარსებობა გამომრჩა smile.gif ეხლა ვფქრობ Q–ცხელებასთან ხომ არ გვაქვს საქმე? აქაც ემთხვევა სიმპტომები, ოღონდ ცხელება აკლია... ეტიოლოგიურადაც მისაღებია ფერმერებისთვის ეს დაავადება. რაც შეეხება მიოზს, ეგ ოკულომოტორიუსის ან მისი ბირთვების დაზიანების ბრალია.... მარა პათოგენეზური კავშირი?... და შემთხვევით პესტიციდებით მოწამვლა ხომ არ არის? ტოქსიკოლოგია ჩემი სუსტი მხარეა, სამედიცინოზე ეს საგანი მხოლოდ 5 დღიანი კურაციით არის წარმოდგენილი (!) ...

Posted by: Guardian 27 Nov 2008, 22:40
აცეტილქოლინესთერაზას ფოსფორორგანული შეუქცევადი ინჰიბიტორით არის ეგ მოწამლული.
ინსექტიციდებად იყენებენ მაგათ - ქლოროფოსი, დიქლოფოსი, და ა.შ.

Posted by: vano_t 27 Nov 2008, 22:41
LUKA-BRAZI
ე.ი. მორჩა ნევროლოგიის კურაციას. ხოდა ძალიან კარგი smile.gif

ახლა ნევროლოგიის კურაცია დაუკავშირე შენს ნათქვამს: და შემთხვევით პესტიციდებით მოწამვლა ხომ არ არის? და პასუხიც უნდა მიიღო smile.gif

აქ ერთი ძაან "მაგარი" "ტოქსიკოლოგი" შემოდიოდა ხოლმე (გვარს და სახელს არ დავასახელებ, მარა ის გოგირდით მოწამლვებს რო 5-ანში ურტყამს და ტოქსიკოლოგიას რო გვასწავლის) და მეთქი ის გაცემდა პასუხს ამ კითხვას, მარა ვერ ქნა საქმე sad.gif

Posted by: LUKA-BRAZI 27 Nov 2008, 22:55
vano_t
ჰო, გარდიანს დავეთანხმები... ფოსფორორგანული ნივთიერებების ბრალია... ორმხრივი მიოზი ფოტოკორნეალური რეფლექსის დათრგუნვით ან მოსპობით ახასიათებს ოპიათებით, მუსკარინით, ორგანულ ფოსფატებით ანდა სხვა ქოლინერგული ან ქოლინესთერაზის მაბლოკირებელი ნივთიერებებით მოწამვლას. ქეისიდან ჩანს პარასიმპატიკური სისtემის ირიტაციის ნიშნები: ბრადიკარდია, ჰიპოტენზია და ა.შ.
ჰო, ფერმერები კი საკმაოდ აქტიურად იყენებენ ფოსფატებს biggrin.gif

Posted by: basa-ttt 27 Nov 2008, 23:52
პაციენტი მამაკაცი 56 წლის,

საოჯახო ანამნეზიდან
5 წლის ასაკში ჰქონდა ასთმური ბრონქიტის ეპიზოდები, იყო ხშირად მოავადე რესპირატორული ინფექციებით.
მამა გარდაიცვალა ალკოჰოლიზმის გამო.
დედა - ტუბერკულოზით
ძმა გარდაიცვალა გულის ათეროსკლეროზული დაზიანების გამო.

22 წლის ასაკში დაეწყო აბდომინალური ჩივილები ხშირი დიარეით, კოლიკებით მუცლის არეში, ხან ყაბზობა - ხან დიარეა.
პერიოდულად ტემტერატურული რეაქციით.
20 წლის მერე ამას დაემატა ანორექსია, დეჰიდრატაცია, უძლიერესი აბდომინალური კოლიკები,
პაციენტი ამ პერიოდში იწყებს ალკოჰოლის სმას.

1-2 წლის მერე ამას დაემატა კანის და ლორწოვანი გარსების სიყვითლე, ტკივილი და ღებინება, ეს სიმპტომები მეორდებოდა პერიოდულად რამდენიმე თვეში ერთხელ.
28 წლის ასაკიდან დაეწყო სმენის დაქვეითება, უმეტესად მარცხნივ, ყურებში შუილით, 44 წლის ასაკში სმენის დაქვეთებამ საგრძნობლად მოიმატა. 52 წლის ასაკში დაემატა ამ ყველაფერს თავის ტკივილები და სახსრების ტკივილი.
აღინიშნება ანამნეზში თითის დაჩირქება, ყბის ალვეოლარული აბსცესი,
55 წლის ასაკში აღენიშნებოდა სისხლდენები ცხვირიდან, ნახველი პერიოდულად მცირედ სისხლიანი. (ტუბერკულოზი გამოირიცხა)
სვამდა ბევრს - ალკოჰოლს. იყო აღგზნებული,

საბოლოოდ 56 წლის ასაკში აღენიშნებოდა ტკივილი გულმკერდის არეში, ხველა (დაუსვეს პნევმონიის დიაგნოზი) შეშუპება ქვემო კიდურებზე, ასციტი. ჰეპატომეგალია მოგვიანებით განუვითარდა დიარეა, ანორექსია, ღებინება ანურია, და 1 კვირაში

გარდაიცვალა 56 წლის ასაკში -
გარდაცვალების წინ ასციტის გამო ამოუღეს დიდი რაოდენობით სითხე .
აუტოფსიაზე აღენიშნებოდა მსხვილკვანძოვანი ციროზი, макроузелковый цирроз,
სფლენომეგალია, ქრ პანკრეატიტის ნიშნები და ნაღველკენჭოვანი დაავადება.
სასმენი მილის არტერიები სკლეროზირებული, სასმენი ნერვები истончены ატროფირებული.

რა სჭირდა პაციენტს?

Posted by: Cousteau 28 Nov 2008, 00:00
Cystic Fibrosis-ს აქვს ყველაფერი რაც დაწერე მაგრამ მაგ ასაკამდე ვერ ცოცხლობენ. .

შენი პაციენტი

user posted image
ამას ხო არ გავს? gigi.gif

Отец композитора страдал от алкоголизма, отчего и умер в 1791 году. Мать умерла пятью годами раньше от туберкулеза, от которого скончался и один из братьев композитора. Другой брат умер от атеросклеротического заболевания сердца.



С 1792 года у композитора начались абдоминальные расстройства в виде повторяющейся диареи и болей в животе, что позднее начало сопровождаться допотифоидной лихорадкой, перемежающимися приступами диареи и сильнейших запоров.

В изолированном виде все это можно было трактовать как воспалительное заболевание кишечника.

В 1812-1820 годах к описанной клинической картине добавились анорексия, дегидратация и сильнейшие абдоминальные колики, спасаясь от которых Бетховен стал злоупотреблять алкоголем. В 1821 году у композитора были отмечены желтуха, боли и рвота, которые повторялись через несколько месяцев.

В 28-летнем возрасте у Бетховена начинает развиваться глухота, сопровождавшаяся звоном в ушах, которую ему тщательно удавалось скрывать в течение трех лет.

Однако в возрасте 44 лет композитор полностью оглох. Травмированный душевно, он продолжал пить. В 1822 году у него появляются сильные головные боли, усиливаются проблемы с желудочно-кишечным трактом и рекуррентные ревматические атаки.
Для объяснения глухоты авторы дифференциально-диагностически разбирают: повторные воспаления среднего уха, повреждения VIII пары черепных нервов (возможно, инфекционного, в том числе сифилитического, генеза), отосклероз, болезнь Педжета и болезнь Меньера.
Известно, что глухота наложила отпечаток на творчество композитора. Если до 1800 года в его музыке чувствуется влияние Моцарта и Гайдна, то с 1800 по 1815 год, когда глухота прогрессировала, Бетховен пишет знаменитую Лунную сонату и Третью, Четвертую, Пятую симфонии. Будучи уже полностью глухим, тяжело больным соматически, с расстроенной психикой, он создает потрясающую Девятую симфонию и Messa Solemnis.
В 1804 году у композитора отмечались сильная лихорадка и абсцесс, едва не повлекший за собой потерю пальца. В 1822 году диагностирована торакальная подагра, спустя год он жалуется на боль в глазах (по-видимому, начавшийся увеит). В 1825 году у него вновь диагностируют желтуху, которая сопровождается психическими расстройствами, пиодермией, носовыми кровотечениями, повторным кровохарканьем. В 1826 году у Бетховена отмечаются боли в груди, кашель (врачи ставят диагноз пневмонии), появляются припухлости голени и лодыжек, анорексия, гепатомегалия и асцит. В связи с этим врачи вынуждены давать композитору небольшие дозы алкоголя, чтобы предотвратить белую горячку и облегчить боли. В марте 1827 года появляется сообщение о поверхностном дыхании, кровохарканьи, правосторонних болях в груди.
Неделей позже у композитора развиваются диарея, желтуха, рвота, сильный отек лодыжек и, очевидно, анурия. Далее симптомы усилились. У него откачали большое количество асцитной жидкости. 24 марта Бетховен впал в коматозное состояние и через два дня скончался.
По меньшей мере трижды (две эксгумации) его тело подвергалось аутопсии. При первом вскрытии сообщалось о макроузелковом циррозе печени (что говорит о неалкогольной его природе), портальной гипертензии и спленомегалии (размер селезенки превышал норму в 3 раза), хроническом панкреатите и желчнокаменной болезни. Свод черепа был равномерно и существенно уплотнен. Слуховые нервы истончены (больше слева), слуховые артерии склерозированы. Однако признаков облитерирующего эндартериита, как это бывает, например, при сифилисе, не было обнаружено.

Posted by: basa-ttt 28 Nov 2008, 00:10
Cousteau ფიბროზის და სკლეროზის ნიშნები ნამდვილად არის.
მაგრამ მართალი ხარ -
ის 56 წლის ასაკში გარდაიცვალა.

ჰგავს.
მერე?

Posted by: Cousteau 28 Nov 2008, 00:12
QUOTE (basa-ttt @ 28 Nov 2008, 00:10 )
Cousteau ფიბროზის და სკლეროზის ნიშნები ნამდვილად არის.
მაგრამ მართალი ხარ -
ის 56 წლის ასაკში გარდაიცვალა.

ჰგავს.
მერე?

: )
ლუდვიგ ვან ბეთხოვენს ვუსვავთ დიაგნოზს? gigi.gif

Posted by: basa-ttt 28 Nov 2008, 00:13
Cousteau
რატომაც არა?

კლინიკა მშვენივრადაა აღწერილი.
yes.gif

Posted by: Cousteau 28 Nov 2008, 00:16
QUOTE (basa-ttt @ 28 Nov 2008, 00:13 )
Cousteau
რატომაც არა?

კლინიკა მშვენივრადაა აღწერილი.
yes.gif

: )
კიბატონო მაგრამ მაგას რა ჭირდა კაციშვილმა არ იცის, შეიძლება მოწამლეს 25 წლის რო მომკვდარიყო მაშინ უფრო რო CF-ის დიაგნოზი ექნებოდა. თუმცა უფრო ჭკვიანებს დაველოდოთ ეგებ რამეს იტყვიან yes.gif

Posted by: basa-ttt 28 Nov 2008, 00:20
პირადად ჩემთვის რა სჭირდა მაშინვე გასაგები იყო, როცა გავეცანი მის კლინიკას.

ქართულად თუ დაწერთ ფორუმზე და არა ინგლიურად-
უკეთესი იქნება

Posted by: Cousteau 28 Nov 2008, 00:26
QUOTE (basa-ttt @ 28 Nov 2008, 00:20 )
პირადად ჩემთვის რა სჭირდა მაშინვე გასაგები იყო, როცა გავეცანი მის კლინიკას.

გაამხელ საიდუმლოს?

QUOTE

ქართულად თუ  დაწერთ ფორუმზე  და არა ინგლიურად-
უკეთესი იქნება

ეს რაზე CF-ზე? spy.gif

P.S.
QUOTE
საოჯახო ანამნეზიდან
5 წლის ასაკში ჰქონდა ასთმური ბრონქიტის ეპიზოდები


ასთუმური ბრონქიტი - ასეთი ტერმინი არარსებობს!

Posted by: basa-ttt 28 Nov 2008, 00:34
QUOTE
ასთუმური ბრონქიტი - ასეთი ტერმინი არარსებობს!

ტექსტი თარგმნილია
ასთუმური არ არსებობს.

QUOTE
ეს რაზე CF-ზე?

ინგლისურენოვან ტექსტებზე-
ეს გასაგებია
* * *
QUOTE
გაამხელ საიდუმლოს?

მოჟნო

Posted by: LUKA-BRAZI 28 Nov 2008, 01:22
basa-ttt
QUOTE
22 წლის ასაკში დაეწყო აბდომინალური ჩივილები ხშირი დიარეით, კოლიკებით მუცლის არეში, ხან ყაბზობა - ხან დიარეა.
პერიოდულად ტემტერატურული რეაქციით.
20 წლის მერე ამას დაემატა ანორექსია, დეჰიდრატაცია, უძლიერესი აბდომინალური კოლიკები,
პაციენტი ამ პერიოდში იწყებს ალკოჰოლის სმას.

არასპეციფიური წყლულოვანი კოლიტი ?

Posted by: Cousteau 28 Nov 2008, 01:30
QUOTE (LUKA-BRAZI @ 28 Nov 2008, 01:22 )

არასპეციფიური წყლულოვანი კოლიტი ?

ერთ-ერთი ვარიანტია გარჩევებში ეგ
მარა რაღაც-რაღაცეებს ვერ ხსნის ეგ დიაგნოზი მგონი

Posted by: vano_t 28 Nov 2008, 01:38
Cousteau
QUOTE
: )
ლუდვიგ ვან ბეთხოვენს ვუსვავთ დიაგნოზს?  gigi.gif

lol.gif
სად წაიჭირე, ჰა? biggrin.gif თან ტუბერკულოზი რო გამოირიცხა მე-18 საუკუნეში ნახველის დათესვის გარეშე, არ გაინტერესებს?

ეს შეიძლება იყოს რამოდენიმე რამ ერთად, არ არის გამორიცხული, მარა შეუმოწმებელი ქეისის დადება რას მოგცემს? ალბათ სიფილისურ-მუნური მიაზმით იყო დაავადებული, ჰა? გოგირდით მოწამვლა გამორიცხულია, გერმანიაში ეგ არ ქონდათ. ალუმინის ქვაბში შეიძლება აკეთებდენე რაღაც რაღაცეებს მაშინ. ეგ კანის სიმშრალეს ახსნის. მეტს ვერაფერს დავამატებ.

Posted by: basa-ttt 28 Nov 2008, 01:39
QUOTE
სად წაიჭირე, ჰა?  თან ტუბერკულოზი რო გამოირიცხა მე-18 საუკუნეში ნახველის დათესვის გარეშე, არ გაინტერესებს?

ეს შეიძლება იყოს რამოდენიმე რამ ერთად, არ არის გამორიცხული, მარა შეუმოწმებელი ქეისის დადება რას მოგცემს? ალბათ სიფილისურ-მუნური მიაზმით იყო დაავადებული, ჰა? გოგირდით მოწამვლა გამორიცხულია, გერმანიაში ეგ არ ქონდათ. ალუმინის ქვაბში შეიძლება აკეთებდენე რაღაც რაღაცეებს მაშინ. ეგ კანის სიმშრალეს ახსნის. მეტს ვერაფერს დავამატებ.

ისევ ისტერიკა.
რა გჭირს?

Posted by: LUKA-BRAZI 28 Nov 2008, 01:42
vano_t
წინა ქეისზე პასუხი რა არის ვანო? ორგანული ფოსფატები?

Posted by: vano_t 28 Nov 2008, 01:44
QUOTE (basa-ttt @ 28 Nov 2008, 01:39 )
ისევ ისტერიკა.
რა გჭირს?

მადლობთ, კარგად. ტქვენ როგორ გიკითხოთ?

რაის ისტერიკა? ძაან ხალისიანად ვიცინე შენს კეისზე. ეტყობა ისტორიული დაავადებები რომ გიყვარს. ისე მაინც რა დიაგნოზი დასვა?

Posted by: LUKA-BRAZI 28 Nov 2008, 01:49
basa-ttt
ისე მართლა რეალური შემთხვევაა ეგ თუ მოგონილი? ანუ ააქვს მაგ ქეისს პასუხი? ისე კი აუტოიმუნურ პათოლოგიას გავს ეგ ყველაფერი, ასე მთლიანად რომ მოეშალა ორგანიზმი.... +დიაბეტი, +კრონის დაავადება, +სმენის ნერვის ნევრინომა და პათანატომიური მონაცემები, ციროზი ქონია..... და რა, შენ გინდა დავასახლოთ ერთი კონკრეტული მიზეზი? რაიმე ტრიგერული აგენტი ამ ყველაფრის?

Posted by: texasuri jleta benzoxerxit 28 Nov 2008, 01:52
vano_t
basa-ttt
ცოტა კორექტული დისკუსია რომ გქონდეთ,არა?....უკვე დამღლელია ამ კინკლაობის თვალის დევნება boli.gif

Posted by: LUKA-BRAZI 28 Nov 2008, 01:52
უბრალოდ ვერ გავიგე კითხვაში "რა ჭირდა პაციენტს" რას გულისხმობ.... user.gif

Posted by: basa-ttt 28 Nov 2008, 01:54
QUOTE
რაის ისტერიკა? ძაან ხალისიანად ვიცინე შენს კეისზე. ეტყობა ისტორიული დაავადებები რომ გიყვარს. ისე მაინც რა დიაგნოზი დასვა?

პირადად ჩემთან ეს პაციენტი რომ მოსულიყო
მე მივცემდი მას პლუმბუმს...
და სხვათაშორის როცა ბეთჰოვენის დაავდბაზე ვმსჯელობდით რამდენიმე წლსი წინ
ჰომეოპათები -
ვამბობდით, რომ ტყვიის პათოგენეზია...

ამის გამო -

QUOTE
კოლიკებით მუცლის არეში

QUOTE
20 წლის მერე ამას დაემატა ანორექსია, დეჰიდრატაცია, უძლიერესი აბდომინალური კოლიკები

QUOTE
28 წლის ასაკიდან დაეწყო სმენის დაქვეითება, უმეტესად მარცხნივ, ყურებში შუილით

QUOTE
დაემატა ამ ყველაფერს თავის ტკივილები და სახსრების ტკივილი

QUOTE
სასმენი მილის არტერიები სკლეროზირებული, სასმენი ნერვები истончены ატროფირებული.

და როგორც სწორედ აღნიშნა Cousteau
სკლეროზი და ფიბროზი.

ახლა ნახეთ პლუმბუმის ანუ ტყვიის პათოგენეზი
http://lechebnik.info/445/61.htm


და ნახეთ რა ვიპოვე დღეს ინტერნეტის ქსელში.

http://www.trud.ru/issue/article.php?id=200011022062502

Будучи в боннском Доме Бетховена, где в центре одного зала установлен его бюст, я, помимо всего прочего, узнал, что там хранятся несколько локонов его волос, остриженных за несколько лет до смерти в 1827 году. Ему тогда было 56.

Помню, как меня поразил ответ сотрудницы музея на мой вопрос, где находится усыпальница Бетховена: "Ее нет". Как нет? Оказывается, он был похоронен там, где и жил - в Вене, но затем его тело эксгумировали и поместили в музей. Оттуда оно непонятным образом исчезло, и виновники этого варварства до сих пор не установлены, как и место захоронения останков гения, если оно вообще имело место.

И вот теперь я узнал, что именно те самые локоны, возможно, могут рассказать не только о причинах тяжких недомоганий, которые терзали Бетховена с 20 лет, но и о самой его смерти, вызванной, по заключению тогдашних медиков, "пневмонией и осложнениями желудочной болезни". Чтобы стало ясно, о чем идет речь, начать нужно тоже с бетховенских локонов, но других.

Четыре года они исследовали волоски (их в общей сложности было 582, и каждый из них пронумерован) - теми почти фантастическими методами, которые доступны современной науке. И оба научных центра пришли к идентичным выводам. Бетховен не болел сифилисом, как считают многие историки. Биологический анализ не нашел ни малейшего следа меркурия, которым тогда лечили это заболевание. Но зато обнаружено, что его волосы содержат в 100 раз больше свинца, чем обычно бывает у людей. То есть он наверняка большую часть жизни страдал от плюмбизма - отравления свинцом. Чем и объяснялись желудочные боли, резкие смены настроения и общая непредсказуемость его поведения, особенно в последние годы жизни, а возможно, и глухота, начавшая развиваться у него в 31 год и ставшая почти абсолютной к 42 годам.

სხვათაშორის ის იურისტი
ასევე იყო პლუმბუმის პათოგენეზი.
მე როცა ვკითხე
ტყვიასთან რამე შეხება გაქვს მეთქი? -
მითხრა ნადირობა მიყვარსო.
ეგ არა საკმარისი მეთქი
და უცებ შემოირტყა თავში ხელი
პატარა რომ ვიყავი მამაჩემს ტყვიის საწარმო ჰქონდა
და ეზოში გორა იდგა ტყვიის ნაერთისო-
და იქ ვთამაშობდი მთელი ჩემი ბავშვობაო..

მე მას ტყვია - პლუმბუმი მივეცი
და წნევები დარეგულირდა.
არ ვიცი დაყრუვდებოდა თუ არა ის ბეთჰოვენივით,
მარა თირკმელში რომ სკლეროზული პროცესები წავიდოდა წლების მერე
ეს ნამდვილად ცხადი იყო.

Posted by: vano_t 28 Nov 2008, 01:56
LUKA-BRAZI
QUOTE
წინა ქეისზე პასუხი რა არის ვანო? ორგანული ფოსფატები?

მაგ კეისის ავტორს ეგ დიაგნოზი უნდა ალბათ. თუმცა, ყველაზე სწორი იქნება რომ თქვა ქოლინერგული სინდრომი, ან ქოლინერგული ნაერთით მოწამლვა. რადგანაც ასეთი მოწამლვა შეიძლება მოგცეს ზოგიერთმა სოკომაც, რომელიც აცეტილქოლინის მიმეტიკებს შეიცავს და ზოგიერთმა წამალმა (მაგალითად კარბაქოლი, ბეთანექოლი და ა.შ.-აცეტილქოლინის მიმეტიკები და ფიზოსტიგმინი, ნეოსტიგმინი-აცეტილქოლინესთერაზას მიმეტიკები). ამიტომ, სინამდვილეში ასეთი ავადმყოფი რომ გყავდეს უნდა იკითხო რაიმე წამალს ხომ არ სხვამს ან თვითონ ან ოჯახის წევრი, რაიმე სოკო ხომ არ უჭამია, ან ინსექტიციდებთან/პესტიციდებთან კონტაქტი თუ ქონდა. ფერმერი რომ არის, არ ნიშნავს, რომ მაინცდამაინც ორგანოფოსფატით არის მოწამლული.

ტეხასური ჟლეტა ბენზოხერხით
QUOTE
ცოტა კორექტული დისკუსია რომ გქონდეთ,არა?....უკვე დამღლელია ამ კინკლაობის თვალის დევნება
მედიცინის განყოფილებაში მედიცინაზე უნდა იყოს საუბარი და არა ფოკუზნიკობაზე.

Posted by: texasuri jleta benzoxerxit 28 Nov 2008, 01:59
და რა არ მესმის იცი...ტუ ტყვიით მოწამლვაა,ტყვიასვე რატომ აძლევთ?იგივე კითხვა გამოჩნდა ალუმინთან დაკავშირებით წინა ქეისზე...


"ფოკუსნიკობა" არ ვიცი მე1ცოტა მეტი პაივიხსცემა ერთმანეთის მიმართ რა.ნადოიელო უჟე...

Posted by: basa-ttt 28 Nov 2008, 02:01
texasuri jleta benzoxerxit
იმიტომ რომ მსგავსი იკურნება მსგავსით

მთავარია დოზა შეარჩიო.

QUOTE
"ფოკუსნიკობა" არ ვიცი მე1ცოტა მეტი პაივიხსცემა ერთმანეთის მიმართ რა.ნადოიელო უჟე...

vano_t
ავადაა საკუთარი პატივმოყვარეობით

Posted by: LUKA-BRAZI 28 Nov 2008, 02:05
გეყოთ ეხლა ! უკვე ეგ კამათი აღარ ეხება მედიცინას ხომ ხედავთ?!

Posted by: basa-ttt 28 Nov 2008, 02:08
LUKA-BRAZI
სხვათაშორის ჩვენთან მოსულები იყვნენ 9 აპრილს მოწამლულები
გამოკითხვით ვფიქრობთ, რომ გამოყენებული იყო დარიშხანის რაღაც ნაერთი.
ვნახოთ - თუ ოდესმე 9 აპრილის საქმე გაიხსნა-
და თუ დაემთხვევა ჩვენი პროგნოზი.

Posted by: LUKA-BRAZI 28 Nov 2008, 02:14
ხვალ ერთ კარგ ქეისს გამოვაცხობ, მაგრამ ყველაზე მაგარი იქითა კვირისთვის მაქვს შემონახული. Blind_Torture_Kill-ი რომ დაბრუნდება (რამდენიმე დღით აქ არ ვიქნებიო), Guardian–იც რომ იქნება და შენც vano_t და დანარჩენებიც (basa-ttt იმედია მაგ დროისთვის მოგვარდებით შენ და ვანო და შენც მიიღებ მონაწილეობას განხილვაში), ისეთი ქეისი უნდა დავდო (სხვათაშორის მაგ თემაზე რეფერატიც მაქვს დაწერილი და პროფესორმა რომ ნახა – ეს საიდან გაგახსენდაო smile.gif ) რომ.... უიშვიათესი, მაგრამ უსაინტერესოესი biggrin.gif იმდენად იშვიათი რომ ინტერნეტში ძლივს ვნახე მასალა smile.gif ასე რომ.... თუმცა მანამდე განვიხილოთ სხვები! ოღონდ ცივილიზებულ ფარგლებში რა....

Posted by: texasuri jleta benzoxerxit 28 Nov 2008, 02:15
basa-ttt
რა საინტერესოა...იგივე პრინციპით ვთქვათ მენინგოკოქცემიის დროს მენინგოკოკები ან მათი ენდოტოქსინი უნდა შევუყვანოთ ავადმყოფს? ან უილსონის დავადების დროს სპილენძი?ირონიის გარეშე,მართლა მაინტერესებს...


P.S.ისე მოწოდებაში კორექტულობისკენ თქვენც გიცულისხმეთ boli.gif

Posted by: Cousteau 28 Nov 2008, 02:23
და მაინც არ მითხარით რატომ ვერ გრძნობს სურდოს დროს ადამიანი კატლეტის გემოს user.gif

Posted by: vano_t 28 Nov 2008, 02:27
ტყვიით მოწამვლ ქონდა ბეთჰოვენს, რაც ექსპერტიზით დამტკიცდა. დღევანდელ დღეს ტყიით მოწამლვას ფიქრობ მხოლოდ ბავშვებში, რომელიც ძველი საღებავით შეღებილ სახლში ცხოვრობენ და საღებავის ჩამონფცქვენს ჭამენ. ან ძველ სათამაშოებს ხმარობენ, რომლის შეღებვაც ადრე ხდებოდა ტყვიით.

მონადირე მოიწამლება ტყვიით, თუ საფანტს ჭამს რეგულარულად: დილით, შუადღით და საღამოთი! სხვანაირად როგორ უნდა მოიწამლოს ტყვიით?

ამიტომაც ვამბობ, ცოტა სერიოზულად თუ შეიძლება.


ახალი შემთხვევა:
60 წლის პაციენტი ნახულობს ექიმს ოფისში. ავადმყოფი მოდის დაქვეითებული მადით და წონაში კლებით (2.5 კგ), ლეთარგიით, თმის ცვენით, შეკრულობით და სიცივისადმი ტოლერანტობის დაქვეითებით. წნევა როცა წამოწოლილია არის 100/80 და როცა ფეხზე დგას 80/60. თირეოიდული ჯირკვალი გამოკვლევისას არ ისინჯება. განავალი უარყოფითია ფარული სისხლისაგან. ღრმა რეფლექსებში აღინიშნება დაგვიანებული რელაქსაციის ფაზა. ლაბორატორიები:

ჰემატოკრიტი 36%
შრატის T4 1 მკგ/დლ
თავისუფალი T4-ის ინდექსი 1
TSH 6.5

რა არის სავარაუდო დიაგნოზი?
1) ჰაშიმოტოს თირეოიდიტი
2) ადისონის დაავადება
3) აუტონომიური უკმარისობა (ანუ ავტონომიური ნერვული სისტემის უკმარისობა)
4) კრანიოფარინგიომა
5) ეუთირეოიდული ავადმყოფობის სინდრომი (eusthyroid sick syndrome)

Posted by: vano_t 28 Nov 2008, 04:09
ნორმები ქალებისათვის:

ჰემატოკრიტი 35-45 %
შრატის T4 არის 5-12
თავისუფალი T4-ის ინდექსი 2-5
TSH 0.5-4.5

Posted by: vano_t 28 Nov 2008, 04:28
ბასა-ტტტ
QUOTE
მე როცა ვკითხე
ტყვიასთან რამე შეხება გაქვს მეთქი? -
მითხრა ნადირობა მიყვარსო.
ეგ არა საკმარისი მეთქი
და უცებ შემოირტყა თავში ხელი
პატარა რომ ვიყავი მამაჩემს ტყვიის საწარმო ჰქონდა
და ეზოში გორა იდგა ტყვიის ნაერთისო-
და იქ ვთამაშობდი მთელი ჩემი ბავშვობაო..
ბოლომდე არ წამიკითხია შენი ნაწერი ეტყობა. აქ ბოდიში უნდა მოგიხადო. ყოფილა შენს მონაყოლში (და არა შენი ავადმყოფის მონაყოლში-ეგ შენი ავადმყოფი არ არის!) რაღაც ტყვიით მოწამვლის წყარო.

ისე აჯობებს სხვისი შემთხვევები (განსაკუთრებით 200 წლის შემთხვევები) შენს ნანახ შემთხვევებად არ გაასაღო. შენი პაციენტი რომ არ არის ეს იქიდან ჩანს, რომ ერთი ერთზე კოპიოა ეგ შემთხვევა ბეთჰოვენის შემთხვევის.

Posted by: basa-ttt 28 Nov 2008, 09:18
QUOTE
შენი პაციენტი რომ არ არის ეს იქიდან ჩანს, რომ ერთი ერთზე კოპიოა ეგ შემთხვევა ბეთჰოვენის შემთხვევის.

უიმე დედა!!!
spy.gif
გაგაცნობ პირადად იმ იურისტს -
რახან ეჭვიანობითაც ვართ შეპყრობილები...
gigi.gif

QUOTE
რა საინტერესოა...იგივე პრინციპით ვთქვათ მენინგოკოქცემიის დროს მენინგოკოკები ან მათი ენდოტოქსინი უნდა შევუყვანოთ ავადმყოფს?

აცრა აბა რაა?
არსებობს ე.წ. ნოზოდებიც...
მაგალითად არსებობს ტუბერკულოზის ჩხირიდან მიღებული ჰომ. წამალი.

Posted by: vano_t 28 Nov 2008, 09:42
QUOTE (basa-ttt @ 28 Nov 2008, 09:18 )
QUOTE
რა საინტერესოა...იგივე პრინციპით ვთქვათ მენინგოკოქცემიის დროს მენინგოკოკები ან მათი ენდოტოქსინი უნდა შევუყვანოთ ავადმყოფს?

აცრა აბა რაა?
არსებობს ე.წ. ნოზოდებიც...
მაგალითად არსებობს ტუბერკულოზის ჩხირიდან მიღებული ჰომ. წამალი.

აცრა სხვაა და მკურნალობა სხვა რამ. აცრის დროს ორგანიზმი დაავადებული არ არის. ამ დროს შეგყვას "დაავადება" რათა მომავალში აიცილო დაავადება. როცა დაავადებულია ორგანიზმი, მაგ დროს აღარ შეგყავს "დაავადება", არამედ იცილებ არსებულ დაავადებას-ასეა ეს კლსიკურ მედიცინაში. ასე რომ, აცრა და მკურნალობა სხვადასხვა რამ არის.

Posted by: LUKA-BRAZI 29 Nov 2008, 13:51
vano_t
ვაა, ძალიან საინტერესო ქეისია smile.gif ყველა ჩამოთვლილი შეიძლება მსგავსი სიმპტომებით გამოვლინდეს, მაგრამ უფრო კრანიოფარინგიომა მგონია.... ყველაზე უფრო აღწერილი სიმპტომები ამ დაავადებას შეესაბამება.... გამოვიცანი? smile.gif smile.gif

Cousteau
QUOTE
და მაინც არ მითხარით რატომ ვერ გრძნობს სურდოს დროს ადამიანი კატლეტის გემოს

იმიტომ რომ გემოს სრულყოფილად შესაგრძნობად აუცილებელია ყნოსვის ფუნქციის არსებობაც. ამიტომაც ენოლოგები ღვინის დეგუსტაციას როგორც გემოთი, ისე მისი სუნით ახდენენ, ანუ ე.წ. "ღვინის ბუკეტი". როდესაც ცხვირის ლორწოვანი ჰიერემიულია, ანუ ანთებაა, საყნოსავი ნერვის უწვრილესი ბოჭკოები ვერ იკავშირებენ სუნის მოლეკულებს, ამიტომაც უგემრიელესი კატლეტი შეიძლება საერთოდ უგემოვნოდ შეჭამო biggrin.gif

Posted by: vano_t 29 Nov 2008, 14:22
LUKA-BRAZI
QUOTE
ვაა, ძალიან საინტერესო ქეისია smile.gif ყველა ჩამოთვლილი შეიძლება მსგავსი სიმპტომებით გამოვლინდეს, მაგრამ უფრო კრანიოფარინგიომა მგონია.... ყველაზე უფრო აღწერილი სიმპტომები ამ დაავადებას შეესაბამება.... გამოვიცანი? smile.gif  smile.gif

ვერ გამოიცანი. ანუ, სწორიც რომ იყოს პასუხი, შენ ახსნა არ მოგიცია. როგორც ადრე ავღნიშნე, შემთხვევით რომ აირჩიო პასუხი, 1/5 შანსი გაქვს. ამიტომ, არცერთი პასუხი არ მიიღება დასაბუთების გარეშე wink.gif

თუ კრანიოფარინგიომაა, მაშინ რატომ არ არის ჰაშიმოტო, eusthyroid sick syndrome, ადისონი ან ავტონომიური უკმარისობა?

თუ სხვა რამ არის, მაშინ რატომ არ არის დარჩენილი 4 რამ?

და რა თქმა უნდა, სავარაუდო დიაგნოზის შედეგად მიღებული წინააღმდეგობებიც უნდა აიხსნას.

ბოდიში, მარა ვერ ჩაგითვლი biggrin.gif

ახლა დავიძინე მე, და თქვენ იფიქრეთ ამ კეისზე. ისე ძაან ძნელი კეისია ეს: ძალიან სპეციფიური რამეების ცოდნაა ჭირდება.

Posted by: LUKA-BRAZI 29 Nov 2008, 14:49
vano_t
QUOTE
ვერ გამოიცანი. ანუ, სწორიც რომ იყოს პასუხი, შენ ახსნა არ მოგიცია. როგორც ადრე ავღნიშნე, შემთხვევით რომ აირჩიო პასუხი, 1/5 შანსი გაქვს. ამიტომ, არცერთი პასუხი არ მიიღება დასაბუთების გარეშე wink.gif

თუ კრანიოფარინგიომაა, მაშინ რატომ არ არის ჰაშიმოტო, eusthyroid sick syndrome, ადისონი ან ავტონომიური უკმარისობა?

თუ სხვა რამ არის, მაშინ რატომ არ არის დარჩენილი 4 რამ?

და რა თქმა უნდა, სავარაუდო დიაგნოზის შედეგად მიღებული წინააღმდეგობებიც უნდა აიხსნას.

ბოდიში, მარა ვერ ჩაგითვლი biggrin.gif

ახლა დავიძინე მე, და თქვენ იფიქრეთ ამ კეისზე. ისე ძაან ძნელი კეისია ეს: ძალიან სპეციფიური რამეების ცოდნაა ჭირდება.


მოიცა, მოიცა, ჯერ არ დაიძინო biggrin.gif ჯერ ერთი გარტყმას და პროსტა რაღაცის თქმას რა აზრი აქვს? ასე რომ ვაკეთებდე, საერთოდ დაეკარგება აზრი ქეისების განხილვას, ამიტომ ამას არც ვაკეთებ! მეორე კი ვერ გავიგე, ვერ გამოვიცანი თუ გამოვიცანი მაგრამ არ მეთვლება რახან ახსნის გარეშე დავწერე? smile.gif
მოკლედ:
ჰაშიმოტო იმიტომ არაა, რომ მაგას ობესითი, ტოისც სიმსუქნე ახასიათებს, შენს ქეისში კი პირიქით, ავადმყოფი წონაში იკლებს. + ამას ჯირკვალი გადიდებული არააო.
ადისონი რომ იყოს, მაშინ ისეთი მნიშვნელოვანი რამ უნდა გეხშენებინა ქესში, როგორიცაა კანის ჰიპერპიგმენტაცია.
აუტონომიური ნერვ. სისტემის დისფუნქცია კი ფართო მცნებაა და იმდენ სიმპტომს მოიცავს, რომ ყველა სპეციფიური დაავადება უნდა გამორიცხო, რომ მაგაზე შეჩერდე.
ხოლო ეუთირეოიდიზმი იგივე გრეივსის დაავადება არაა? ჰოდა გრეივსი მაგ პაციენტს ყველაზე ნაკლებად რომ ქონდეს...
როგორც ვთქვი ძალიან ბევრი სიმპტომით გვანან აღნიშნული დაავადებები, თითქმის ყველას ახასიათებს თირეოიდული ჰორმონების ცვლილებეი სისხლში, ასევე წნევის ლაბილობა და ორთოსტატიკური ჰიპოტენზიაც... მაგალითად სიცივისადმი ტოლერანტობის დავეითება ჰაშიმოტოს ახასიათებს, ხოლო თმის ცვენა კრანიოფარინგიომაზე მაქვს წაკითხული.... მოკლედ მართალი ხარ, სპეციფიური რამეების ცოდნაა საჭირო, ეტყობა რაღც ისეთს, რაიმე სუუულ პატარას, მაგრამ პათოგნომურს, არ ვაქცევ ყურადღებას, რაც მიმანიშნებადა.... Loser user.gif

Posted by: basa-ttt 29 Nov 2008, 17:11
კანის ფერიც გვითხარი პაციენტის.


Posted by: mika9 29 Nov 2008, 17:41

ჰაშიმოტო................................................


ვინც მეტანხმება იმან დაამტკიცოს რატომ,,,,,,,,,,,,,,,,,,,,,,,,,,,,,

Posted by: vano_t 29 Nov 2008, 23:29
LUKA-BRAZI
QUOTE
ჰაშიმოტო იმიტომ არაა, რომ მაგას ობესითი, ტოისც სიმსუქნე ახასიათებს, შენს ქეისში კი პირიქით, ავადმყოფი წონაში იკლებს. + ამას ჯირკვალი გადიდებული არააო.
ადისონი რომ იყოს, მაშინ ისეთი მნიშვნელოვანი რამ უნდა გეხშენებინა ქესში, როგორიცაა კანის ჰიპერპიგმენტაცია.
აუტონომიური ნერვ. სისტემის დისფუნქცია კი ფართო მცნებაა და იმდენ სიმპტომს მოიცავს, რომ ყველა სპეციფიური დაავადება უნდა გამორიცხო, რომ მაგაზე შეჩერდე.
ხოლო ეუთირეოიდიზმი იგივე გრეივსის დაავადება არაა? ჰოდა გრეივსი მაგ პაციენტს ყველაზე ნაკლებად რომ ქონდეს...
როგორც ვთქვი ძალიან ბევრი სიმპტომით გვანან აღნიშნული დაავადებები, თითქმის ყველას ახასიათებს თირეოიდული ჰორმონების ცვლილებეი სისხლში, ასევე წნევის ლაბილობა და ორთოსტატიკური ჰიპოტენზიაც... მაგალითად სიცივისადმი ტოლერანტობის დავეითება ჰაშიმოტოს ახასიათებს, ხოლო თმის ცვენა კრანიოფარინგიომაზე მაქვს წაკითხული.... მოკლედ მართალი ხარ, სპეციფიური რამეების ცოდნაა საჭირო, ეტყობა რაღც ისეთს, რაიმე სუუულ პატარას, მაგრამ პათოგნომურს, არ ვაქცევ ყურადღებას, რაც მიმანიშნებადა.... Loser  user.gif

არა, ჰაშიმოტო არასწორედ გამორიცხე. ჰაშიმოტოსთვის არ არის დამახასიათებელი სიმსუქნე. სიმსუქნე შეიძლება იყოს გვიანი სტადიის (როცა ჰიპოთითირეოზი ძალიან გამოხატულია) დამახასიათებელი. გარდა ამისა, ჰაშიმოტოს დროს არ არის აუცილებელი ფარისებური ჯირკვალი გადიდებული იყოს. კიდევ ისა, რომ თმის ცვენა და საერთოდ კანისა და თმის ცვლილება ჰიპოთირეოზის ნიშანია საერთოდ, და არა კრანიოფარინგიომის. კრანიოფარინგიომა სწორი პასუხია, მაგრამ შენი ახსნა არასწორია. და არასწორად გამორიცხე, როგორც უკვე ვთქვი, ჰაშიმოტო. რაც შეეხება წნევის ორთოსტატულ ვარდნას, ეგ შეიძლება იყოს გამოწვეული დაქვეითებული მადით და დაქვეითებული oral intake-ით. ეს უფრო ხშირი მიზეზია ორთოსტატიკის, ვიდრე სხვა რამე. რატომ არის ამ კეისში დაბალი წნევა ნახსენები და ერთ-ერთი პასუხი ადისონი წერია?

არასწორად მიდიხარ საერთოდ. ჯერ უნდა დაადგინო სინდრომი: ჰიპერთირეოზი vs. ჰიპოთირეოზი; მერე უნდა დაადგინო პირველადი vs. მეორადი და მერე განაგრძო საქმე.

ადისონის ამბავში მართალი ხარ. მარა, ადისონს არ ახასიათებს ასევე თირეოიდულ დარღვევები.

mika9
QUOTE

ჰაშიმოტო................................................
ვინც მეტანხმება იმან დაამტკიცოს რატომ,,,,,,,,,,,,,,,,,,,,,,,,,,,,,
ასე ადვილი რო ყოფილიყო ეგ ქეისი, არ დავდებდი smile.gif ჰაშიმოტოს დროს ასეთი რამ ხდება: T4-ის განახევრებას სისხლში (რაც მნიშვნელოვანი ჰიპოთირეოზის ტოლფასია) TSH-ის დაახლებით 80-ჯერ გაზრდა მოყვება. ეს არც მე ვიცოდი და ეს ქეისი რომ გავმაზე, მერე ვისწავლე (შეცდომებზე ადვილია სწავლა smile.gif). ანუ, ჩვენთან T4 დაახლოებით 5-ჯერ არის დაქვეითებული ნორმის ქვედა ზღვარიდან და T4-ის ინდექსი 2-ჯერ. ნუ, რომც ჩავთვალო თავისუფალი ჰორმონის მინიმუმ 2-ჯერ დაქვეითება, TSH უნდა იყოს მინუმ 40-50 მაინც. მაგრამ აქ გაქვს ძაან მცირედ მომატებული თირეოტროპული ჰორმონი.

ეს პრაქტიკულად გამორიცხავს ჰაშიმოტოს.

ახლა დარჩა კრანიოფარინგიომა (დანარჩენები არ მოგცემენ ჰიპოთირეოზს). კრანიოფარინგიომა ჰიპოთირეოზსაც ახსნის და შესაძლო ადრენალური უკმარისობას (ორთოსტატიკური წნევის ვარდნა). მარა, კრანიოფარინგიომის დროს ჰიპოთერეოზი მეორადია. ჯერ გაქვს ჰიპოთალამუს-ჰიპოფიზის ღერძის დაზიანება და მერე გაქვს ჰიპოთირეოზი. ანუ, ჯერ გაქვს TRH-ის ან TSH-ის ან ოირვეს დაქვეითება და მეორადი ჰიპოთირეოზი. ამ შემთხვევაში გაქვს მცირედ მომატებული TSH (რის გამოც შენ დასვი ჰაშიმოტო და მეც დავსვი ჰაშიმოტო). რაშია საქმე?

ხოდა, რაც მთავარია და მნიშვნელოვანი: რა მკურნალობას დაიწყებთ და რა თანმიმდევრობით?

basa-ttt
QUOTE
კანის ფერიც გვითხარი პაციენტის.
ნორმალური კანის ფერია და იმიტომ არ წერია კანის ფერი.

Posted by: mika9 30 Nov 2008, 01:41
vano_t

მეც ვიცოდი რომ ძაან მატულობს ტსჰ სხვა ვერაფერი ჩავსვი,

მაგრამ კრანიოპარინგიომა ასე ჩახლართულად ნამდვილად ვერ გავიაზრე (ასაკი, ჰიპოპიტუიტარიზმი არ ჩანდა) თან ტსჰ თუ მომიმატა თ4 რატო დარჩა უცვლელი?

რაგაც ძაან ჩახლართულია

ტსჰ მერე ოდნავ მომატებას რამე კონკტერული მიზეზი ექნება და მე არ ვიცი......

Posted by: vano_t 30 Nov 2008, 02:50
mika9
QUOTE
მაგრამ კრანიოპარინგიომა  ასე ჩახლართულად ნამდვილად ვერ გავიაზრე (ასაკი, ჰიპოპიტუიტარიზმი არ ჩანდა) თან ტსჰ თუ მომიმატა თ4 რატო დარჩა უცვლელი?

რაგაც ძაან ჩახლართულია

ტსჰ მერე ოდნავ მომატებას რამე კონკტერული მიზეზი ექნება და მე არ ვიცი......
ჰიპოპიტუიტრიზმი არ ჩანს აშკარად. თუმცა, ორთოსტატული ვარდნა წნევის მიუთითებს შესაძლო თირკმელზედა ჯირკვლის უკმარისობას. TSH ძაან მაღალი რომ ყოფილიყო, მაშინ იტყოდი რომ წნევის ვარდნა დეჰიდრატაციის გამოა (დაქვეითებული ორალური კვების გამო). მარა ამ კეისში ძაან ოხრად არის საქმე biggrin.gif

TSH-ის მცირედი მომატება მთლად დამაბნეველია. თურმე აი რა ხდება: მეორეული ჰიპოთირეოზის დროს, განსაკუთრებით როცა ჰიპოთალამუსი ზიანდება და TRH-ის (თირეო-რილიზინგ ჰორმონი) წარმოქმნა/გამოყოფა დაქვეითებულია, ხდება ბიოლოგიურად არააქტიური მაგრამ იმუნოლოგიურად აქტიური TSH-ის წარმოქმნა. ბიოლოგიურად აქტიური TSH დაქვეითებულია, რაც გაძლევს მნიშვნელოვან ჰიპოთირეოზს. იმუნოლოგიურად აქტიური TSH გაძლევს მომატებულ ტესტს (TSH-ს იმუნოლოგიური ტესტებით ადგენენ). ერთგვარი ცრუ-დადებითი (false-positive) რეზულტატია თვითონ ბიოლოგიურად აქტიური TSH-ის მიმართ.

რა მკურნალობა ჭირდება ავადმყოფს და რა თანმიმდევრობით?

Posted by: LUKA-BRAZI 30 Nov 2008, 12:34
vano_t
მაინც ვერ გავიგე რატომ არ მითვლი ამ ქეისს გამოცნობილად user.gif ანუ არასწორი ახსნით მივედი სწორ პასუხამდე? ანუ საქმე ჩამოთვლილი პათოლოგიების დიფერენცირებული დიაგნოსტირებისას ძირითადად მათ შორის ჰორმონული დარღვევების განსხვავებაშია? ანუ ამ კუთხით უნდა ამეხსნა? ok, ok smile.gif

მკურნალობა ასეთია: თუ სიმსივნე დიდია, ანუ შორსწასული შემთხვევაა, მაშინ ჯერ დასხივება და მერე ქირურგიული ჩარევა, თუ პატარაა, შესაძლებელია რადიაციით შემოვიფარგლოთ, ისე კი კრანიოფარინგიომის მკურნალობა სტანდარტულად ქირურგიულია, თუმცა დასხივებაც საჭიროა მეტასტაზების ელიმინირებისათვის.
smile.gif

Posted by: vano_t 30 Nov 2008, 12:58
LUKA-BRAZI
QUOTE
მაინც ვერ გავიგე რატომ არ მითვლი ამ ქეისს გამოცნობილად user.gif ანუ არასწორი ახსნით მივედი სწორ პასუხამდე?
თუ გინდა ჩაგითვლი biggrin.gif ისე არასწორად მიხვედი დაკსვნამდე და იმიტომ. არასწორად მიხვალ სწორ დაკსვნამდე თუ შემთხვევით აირჩევ სწორ დაკსვნას, არ აქვს დიდ მნიშვნელობა. ჰაშიმოტო და euthyroid sick syndrome არასწორად გამორიცხე. euthyroid sick syndrome არ იცოდი რა იყო და შეიძლება იმიტომ. ეს არის მძიმე ავადმყოფობის დროს თირეოიდული ჯირკვლის ფუნქციის დროებითი დაღვრევები. მარა, ჰაშიმოტო ყველა ვარიანტში არასწორად გამორიცხე.

QUOTE
მკურნალობა ასეთია: თუ სიმსივნე დიდია, ანუ შორსწასული შემთხვევაა, მაშინ ჯერ დასხივება და მერე ქირურგიული ჩარევა, თუ პატარაა, შესაძლებელია რადიაციით შემოვიფარგლოთ, ისე კი კრანიოფარინგიომის მკურნალობა სტანდარტულად ქირურგიულია, თუმცა დასხივებაც საჭიროა მეტასტაზების ელიმინირებისათვის.
smile.gif
არა მგონია ამ სიმსივნეს მეტასტაზები ქონდეს. მარა ეს მაინც არ არის პირველი მკურნალობა. ავადმყოფი ოპერაციამდე უნდა მოამზადო. ფარისებურის ამხელა ნაკლოვანებით და თირკმელზედა ჯირკვლის ამხელა დეფიციტით ავადმყოფი რომ საოპერაციოდ გაიყვანო, ჩათვალე რომ დარჩება მაგიდაზე.

ჯერ ჯირკვლების ფუნქციები უნდა დაარეგულირო და მერე იფიქრო ეგ. რომელი უკმარისობა უნდა მოხსნა ჯერ და რატომ? (ანუ, ჯერ თირკმელზედას უკმარისობას უმკურნალებ თუ ჯერ ფარისებურის უკმარისობას?)

Posted by: LUKA-BRAZI 30 Nov 2008, 13:38
vano_t
ასეც ვიცოდი რომ არც ამას ჩამითვლიდი biggrin.gif შეკაცო ავადმყოფს რომ წინასოპერაციოდ მომზადება უნდა, ამაზე ორი აზრი არ არსებობს. მე დავწერე მკურნალობის ძირითადი პრინციპები. რაც შეეხება შეკითხვას, ჯერ ამის, თუ ჯერ იმის მკურნალობა ცოტა მიუღებელია, მაქსიმალურად უნდა ეცადო ორივე ჯირკვლის ფუნქცია დაარეგულირო, ისე კი მაინც ავღნიშნავ, რომ ალბათ წინასაოპერაციოდ ჯობია თირკმელზედა ჯირკვალი უფრო "ყოჩაღად" იყოს, ოპერაცია მაინც ინვაზიურია და ორგანიზმისათვის დიდი სტრესი, ამიტომ ჯობია ადრენერგული სისტემა მწყობრში იყოს, ანუ მოწესრიგდრეს წნევა, გლუკოზის ცვლა, გლუკოკორტიკოიდების ცვლა და ა.შ.
მაგრამ შენი ამბავი რომ ვიცი, არც ეს პასუხი მოგეწონება biggrin.gif

Posted by: mika9 30 Nov 2008, 14:24
LUKA-BRAZI

ესე ზოგადად მსჯელობა არასწორია,

კაცმა დასვა კითხვა რა არის შემდეგი ნაბიჯიო ისიც გითხრა რომ შემდეგი ნაბიჯი მკურნალობააა,

ის კი არ უკითხია რა არის დეფინიტივ ტრიტმენტიო, დეფინიტივ ტრიტმენტი არ არის მარტო ოპერაცია ყველა კრანიოპარინგიომისთვის,

vano_t

ესეთი რთული ჩახლართული ქეისი ჯერ არ შემხვედრია,

Posted by: LUKA-BRAZI 30 Nov 2008, 14:35
mika9
ზოგადად დასვა კითხვა, ჯერ ფარისებრიო თუ ჯერ თირკმელზედაო, რა, მე წამლები და დოზები ჩამოვთვალო?

Posted by: vano_t 30 Nov 2008, 14:39
LUKA-BRAZI
QUOTE
ისე კი მაინც ავღნიშნავ, რომ ალბათ წინასაოპერაციოდ ჯობია თირკმელზედა ჯირკვალი უფრო "ყოჩაღად" იყოს, ოპერაცია მაინც ინვაზიურია და ორგანიზმისათვის დიდი სტრესი, ამიტომ ჯობია ადრენერგული სისტემა მწყობრში იყოს, ანუ მოწესრიგდრეს წნევა, გლუკოზის ცვლა, გლუკოკორტიკოიდების ცვლა და ა.შ.
მაგრამ შენი ამბავი რომ ვიცი, არც ეს პასუხი მოგეწონება biggrin.gif
biggrin.gif ახლა გამაცინე. ე.ი. წინასაოპერაციოდ ჯობია ჯერ თირკმელზედა ჯირკვალი მოაკვიტინო ხო? wink.gif არ ხარ მართალი. წინასაოპერაციოდ ორივეს დარეგულირებაა საჭირო, დროებით მაინც. საქმე იმაშია, რას მიცემ პირველი ავადმყოფს: თიროქსინს თუ სტეროიდებს?

mika9
QUOTE
ესეთი რთული ჩახლართული ქეისი ჯერ არ შემხვედრია,
internal medicine-ის ბორდის კეისია და იმიტომ არის ეგეთი. უმეტესობა კეისები ასეთი არაა ოღონდ.

Posted by: LUKA-BRAZI 30 Nov 2008, 15:14
vano_t
QUOTE
რაც შეეხება შეკითხვას, ჯერ ამის, თუ ჯერ იმის მკურნალობა ცოტა მიუღებელია, მაქსიმალურად უნდა ეცადო ორივე ჯირკვლის ფუნქცია დაარეგულირო

აი თიროქსინი თუ სტეროიდები, ეს უკვე აღარ ვიცი, ამდენი ჯერ არ უსწავლებიათ biggrin.gif ალბათ ორივე უნდა მივცეთ... ან უფრო სტეროიდები რათა მოკლე ხანში დროულად მივიღოთ ჰორმონული დონის ბალანსი.... მოკლედ არ ვიცი...
* * *
ჰოდა თუ არასწორი ვა და ვცდები, თუ არ დაგეზარება ორი სიტყვით ამიხსენი smile.gif

Posted by: vano_t 1 Dec 2008, 01:34
LUKA-BRAZI
QUOTE
აი თიროქსინი თუ სტეროიდები, ეს უკვე აღარ ვიცი, ამდენი ჯერ არ უსწავლებიათ biggrin.gif ალბათ ორივე უნდა მივცეთ... ან უფრო სტეროიდები რათა მოკლე ხანში დროულად მივიღოთ ჰორმონული დონის ბალანსი.... მოკლედ არ ვიცი...
* * *
ჰოდა თუ არასწორი ვა და ვცდები, თუ არ დაგეზარება ორი სიტყვით ამიხსენი smile.gif

როცა ორივეს უკმარისობა გაქვს, ჯერ უნდა მიცე კორტიზოლი და მერე თიროქსინი. მიზეზი არის ის, რომ როცა თიროქსინს აძლევ პირველად, ყველაფრის მეტაბოლიზმს აძლიერებს (სტეროიდების გაძლიერებული დაშლის ჩათვლით) და თირკმელზედას უკმარისობა კიდევ უფრო გაგიძლიერდება, რასაც შეიძლება მოყვეს ადრენალური კრიზი-ეს კი სასიკვდილო შეიძლება იყოს.

Posted by: vano_t 1 Dec 2008, 02:59
43 წლის ქალი მოდის გაძლიერებული ქავილის ჩივილით. ქავილი ძლიერია და ღამეც აღვიძებს ამ ადამიანს. სხვა ჩივილები არ აქვს. ფიზიკური გამოკვლევაც ნორმალურია, გარდა გადიდებული ღვიძლისა. ღვიძლი 7 სმ-ით გამოდის ნეკნთა რკალიდან.

CBC (ანუ ლეიკო-, ერითრო-, თრომბოციტები, ჰემოგლობინი და ლეიკოციტების დიფერენციალი) ნორმალურია. სისხლის ქიმიური ანალიზი უჩვენებს შემდეგს:

კრეატინი 0.8 მგ/დლ (ნ)
ბილირუბინი 0.6 მგ/დლ (ნ)
ალანინ-ამინო-ტრანსფერაზა 78 ერთ/ლ (მცირედ მომატებული)
ალბუმინი 4.2 გ/დლ (ნ)
ტუტე ფოსფატაზა 450 (საკმაოდ, 3-4ჯერ, მომატებული)

დიაგნოზის დასასმელად რომელი ტესტი უნდა გამოიყენო?

1) შრატის პროტეინების ელექტროფორეზი
2) ანტი-გლუვი კუნთების ანტისხეულები (anti-smooth muscle antibody)
3) ანტიმიტოქონდრიული ანტისხეულები
4) ტექნეციუმ 99-m ღვიძლ-ელენთის სკანირება
5) ენდოსკოპიური რეტროგრადული ქოლანგიოპანკრეატოგრაფია (ERCP)


შენიშვნა: 'ნ' ნიშნავს ნორმალურს.

ისევ დაი ისევ: პასუხი უნდა იყოს დასაბუთებული. რა თქმა უნდა, ძებნა და ჩალიჩი მოსულია. რამეთუ ზუსტად ეს არის ტესტის მიზანი, რომ ვეძებოთ და ვიჩალიჩოთ, ამდაგვარად ახალი ცოდნა შევიძინოთ.

თუ რაიმე გაუგებარია, კითხვებიც მოსულა smile.gif

Posted by: mika9 2 Dec 2008, 00:37

ERCP...... obstruqciaas gavs Tumca albat jer Seizleba antimitoxondrialuri gaaketo

Posted by: Blind_Torture_Kill 2 Dec 2008, 02:10
QUOTE
3) ანტიმიტოქონდრიული ანტისხეულები


მაგ ქალს აქვს პირველადი ბილიარული ციროზი

ჰოუმ სვით ჰოუმ

გამარჯობათ ხალხო

Posted by: vano_t 2 Dec 2008, 10:53
Blind_Torture_Kill
გაგიმარჯოს და მართალი ხარ smile.gif მარა რატო არ შეიძლება იყოს დავუშვათ პირველადი მასკლეროზებელი ქოლანგიტი და რატომ არ გაუკეთებ ERCP-ს ჯერ?

Posted by: Blind_Torture_Kill 2 Dec 2008, 11:56
QUOTE
43 წლის ქალი მოდის გაძლიერებული ქავილის ჩივილით. ქავილი ძლიერია და ღამეც აღვიძებს ამ ადამიანს. სხვა ჩივილები არ აქვს. ფიზიკური გამოკვლევაც ნორმალურია, გარდა გადიდებული ღვიძლისა. ღვიძლი 7 სმ-ით გამოდის ნეკნთა რკალიდან

CBC (ანუ ლეიკო-, ერითრო-, თრომბოციტები, ჰემოგლობინი და ლეიკოციტების დიფერენციალი) ნორმალურია. სისხლის ქიმიური ანალიზი უჩვენებს შემდეგს:

კრეატინი 0.8 მგ/დლ (ნ)
ბილირუბინი 0.6 მგ/დლ (ნ)
ალანინ-ამინო-ტრანსფერაზა 78 ერთ/ლ (მცირედ მომატებული)
ალბუმინი 4.2 გ/დლ (ნ)
ტუტე ფოსფატაზა 450 (საკმაოდ, 3-4ჯერ, მომატებული)


ჯერ ისტორიით არის სავარაუდო ეგ დიაგნოზი მეორე მასკლეროზირებელი ქოლანგიტი რომ ყოფილიყო ბილირუბინის დონე მაღალი იქნებოდა თან ეს დაავადება გაქვს წყლულოვან კოლიტის დროს ძირითადად და ამ ქალს ყველაფერი დანარჩენი ნორმაში აქვს

თან ERCPი ჯიგრულად ინვაზიურია და მომდევნო ეტაპზე შემოვინახავდი თუ ანტისხეულების მაჩვენებელი უარყოფითი გამოდგა

Posted by: basa-ttt 2 Dec 2008, 12:29
ექსოკოპია უნდა მაგას დოპლერით.
და გაირკვევა ყველაფერი.
მათ შორის გამოირიცხება სანაღველე გზების და ღვიძლის ონკოლოგიაც.
QUOTE

3) ანტიმიტოქონდრიული ანტისხეულები

საქართველოში არ კეთდება -
მე ასე ვიცი...

Posted by: mika9 2 Dec 2008, 14:14
basa-ttt

ექოსკოპიით ძაან ცოტა რამე გაირკვევა....

Posted by: basa-ttt 2 Dec 2008, 17:42
QUOTE
ექოსკოპიით ძაან ცოტა რამე გაირკვევა

ექოსკოპიაზე ღვიძლის ციროზი ჩანს-
სტრუქტურაა შეცვლილი ღვიძლის პარენქიმის
+ პორტალური ჰიპერტენზიაც ,
ღვიძლის კიბო ან მეტასტაზები ღვიძლში,
სანაღვლე გზების სიმსივნე ,
(ფატერის დვრილის სიმსივნის გარდა)
ამ დაავადებებს გამორიცხავდა.



Posted by: Cousteau 2 Dec 2008, 17:56
QUOTE (basa-ttt @ 2 Dec 2008, 12:29 )
ექსოკოპია უნდა მაგას დოპლერით.

ეგ ვარიანტებში არ არის

QUOTE
და გაირკვევა ყველაფერი.

არც ერთი ტესტი არარსებობს ყველაფერს რო არკვევდეს

blind_Torture_Kill
up.gif up.gif up.gif



Posted by: basa-ttt 2 Dec 2008, 18:08
Cousteau
თქვენ ეტყობა თითოეულ სიტყვაზე ხართ ჩასაფრებული -
რაც ღვიძლს ეხება - და მის სტრუქტურას -
იმ "ყველაფერს" გაარკვევს ეს მეთოდი.

QUOTE
ეგ ვარიანტებში არ არის

პაციენტი რომ მოვა ამ ჩივილებით - ვუთხრათ რომ ეს ვარიანტებში არაა.

Posted by: Cousteau 2 Dec 2008, 18:10
QUOTE (basa-ttt @ 2 Dec 2008, 18:08 )

პაციენტი რომ მოვა ამ ჩივილებით - ვუთხრათ რომ ეს ვარიანტებში არაა.

yes.gif yes.gif yes.gif

Posted by: mika9 2 Dec 2008, 20:19
basa-ttt

მაგ დაავადებებისთვის, ექოს დაბალი 50% (არც კი ცილდება ალბათ) სენსიტიურობა და სპეციპიურობა აქვს.
წყლის ნახვაა დაზუსტებით მაინც ვერფერს იტყვი. . . ექო კაია კენჭებისთვის უფრო მაგალ სენსიტიურია და დაბალ სპეციპიური,

Posted by: basa-ttt 2 Dec 2008, 20:28
mika9
თუ იცი,
საქართველოში კეთდება ანტიმიტოქონდრიული ანტისხეულები?
მე რაღაც არ გამიგია...

Posted by: vano_t 2 Dec 2008, 21:12
Blind_Torture_Kill
QUOTE
ჯერ ისტორიით არის სავარაუდო ეგ დიაგნოზი მეორე მასკლეროზირებელი ქოლანგიტი რომ ყოფილიყო ბილირუბინის დონე მაღალი იქნებოდა თან ეს დაავადება გაქვს წყლულოვან კოლიტის დროს ძირითადად და ამ ქალს ყველაფერი დანარჩენი ნორმაში აქვს

თან ERCPი ჯიგრულად ინვაზიურია და მომდევნო ეტაპზე შემოვინახავდი თუ ანტისხეულების მაჩვენებელი უარყოფითი გამოდგა

up.gif
ჯიგრულად უპასუხე biggrin.gif

P.S. ექო ამ დაავადების დროს არაფერს მოგცემს. როცა ციროზი ყალიბდება, უკვე ძაან შორს წასულია ეს დაავდება. თანაც ექო ციროზისათვის არ არის კარგი ტესტი. ასეთ ავადმყოფს დიაგნოზის დასმის შემდეგ ჭირდება ღვიძლის ბიოფსია. არსებობს კიდევ ახალ ტესტი, რასაც ქვია მაგნიტურ-რეზონანსული ელასტოგრაფია (ქსოვილების ელასტიურობას ზომავს არაინვაზიურად). ამ ტესტს დიდი გამოყენება ექნება მომავალში, როგორც ჩანს. თავიდან შეიძლება აცილებული იქნას ღვიძლის ბიოფსია ბევრ შემთხვევაში.

Posted by: Guardian 2 Dec 2008, 21:30
A 54-year-old man is admitted for persistent lower abdominal
and groin pain that began 7 months previously.
Two months before his present admission, he required exploratory
laparoscopy for acute abdominal pain and presumed
cholecystitis. This revealed necrotic omental tissue
and pericholecystitis necessitating omentectomy and cholecystectomy.
However, the pain continued unchanged. He
currently describes it as periumbilical and radiating into his
groin and legs. It becomes worse with eating. The patient has
also had episodic severe testicular pain, bowel urgency, nausea,
vomiting, and diuresis. He has lost ~22.7 kg over the
preceding 6 months. His past medical history is significant
of hypertension that has recently become difficult to control.
Medications on admission include aspirin, hydrochlorothiazide,
hydromorphone, lansoprazole, metoprolol,
and quinapril. On physical examination, the patient appears
comfortable. His blood pressure is 170/100 mmHg,
his heart rate is 88 beats/min, and he is afebrile. He has
normal first and second heart sounds without murmurs,
and an S4 is present. There are no carotid, renal, abdominal,
or femoral bruits.
His lungs are clear to auscultation. Bowel sounds are normal.
Abdominal palpation demonstrates minimal diffuse
tenderness without rebound or guarding. No masses are
present, and the stool is negative for occult blood. During
the examination, the patient develops Raynaud's phenomenon
in his right hand that persists for several minutes. His
neurologic examination is intact. Admission laboratory
studies reveal an erythrocyte sedimentation rate of 72 mm/h,
a BUN of 17 mg/dL, and a creatinine of 0.8 mg/dL. The patient
has no proteinuria or hematuria. Tests for antinuclear
antibodies, anti-double-stranded-DNA antibodies, and antineutrophil
cytoplasmic antibodies are negative. Liver
function tests are abnormal with an AST of 89 IU/L and an
ALT of 112 IU/L. Hepatitis B surface antigen and e antigen
are positive. Mesenteric angiography demonstrates small
beaded aneurysms of the superior and inferior mesenteric
veins. What is the most likely diagnosis?

Posted by: mika9 2 Dec 2008, 22:05
პოლიYYYYYYYYY XXXXXX.....


გამოიცანით Y ები და X ები



Posted by: vano_t 2 Dec 2008, 22:12
QUOTE (Guardian @ 2 Dec 2008, 21:30 )
A 54-year-old man is admitted for persistent lower abdominal
and groin pain that began 7 months previously.
Two months before his present admission, he required exploratory
laparoscopy for acute abdominal pain and presumed
cholecystitis. This revealed necrotic omental tissue
and pericholecystitis necessitating omentectomy and cholecystectomy.
However, the pain continued unchanged. He
currently describes it as periumbilical and radiating into his
groin and legs. It becomes worse with eating. The patient has
also had episodic severe testicular pain, bowel urgency, nausea,
vomiting, and diuresis. He has lost ~22.7 kg over the
preceding 6 months. His past medical history is significant
of hypertension that has recently become difficult to control.
Medications on admission include aspirin, hydrochlorothiazide,
hydromorphone, lansoprazole, metoprolol,
and quinapril. On physical examination, the patient appears
comfortable. His blood pressure is 170/100 mmHg,
his heart rate is 88 beats/min, and he is afebrile. He has
normal first and second heart sounds without murmurs,
and an S4 is present. There are no carotid, renal, abdominal,
or femoral bruits.
His lungs are clear to auscultation. Bowel sounds are normal.
Abdominal palpation demonstrates minimal diffuse
tenderness without rebound or guarding. No masses are
present, and the stool is negative for occult blood. During
the examination, the patient develops Raynaud's phenomenon
in his right hand that persists for several minutes. His
neurologic examination is intact. Admission laboratory
studies reveal an erythrocyte sedimentation rate of 72 mm/h,
a BUN of 17 mg/dL, and a creatinine of 0.8 mg/dL. The patient
has no proteinuria or hematuria. Tests for antinuclear
antibodies, anti-double-stranded-DNA antibodies, and antineutrophil
cytoplasmic antibodies are negative. Liver
function tests are abnormal with an AST of 89 IU/L and an
ALT of 112 IU/L. Hepatitis B surface antigen and e antigen
are positive. Mesenteric angiography demonstrates small
beaded aneurysms of the superior and inferior mesenteric
veins. What is the most likely diagnosis?

მეზენტერული ისქემია აქვს პაციენტს, მაგრამ რისგან ეგ არის საკითხავი.

მომატებული ედს, რეინო, წონაში დაკლება და ჰეპატიტის არსებობა გაფიქრებინებს იმუნურ პროცესზე. თუ მეზენტერული ისქემია აქვს და რეინო, სავარაუდოა თრომბოზული გართულებები. ჰეპატიტის დროს შეიძლება გქონდეს კრიოგლობულინემია (უფრო ხშირად C-ს დროს, მაგრამ B-ს დროსაც შეიძლება).

Posted by: mika9 2 Dec 2008, 22:22
ფოკალური ვასკულიტი>> ანევრიზმებს

ედს მომატებულია

პილტვები სუფთააა

კაცი საშუალო ასაკის

HB ანტიგენ პოზიტიურია,

თირკმელს დაერხა გულს.....



Posted by: Blind_Torture_Kill 2 Dec 2008, 22:37
პოლიარტერიტის ნოდოზა
მაგას აქვს ხშირად კავშირი B ჰეპატიტთან

Posted by: Cousteau 2 Dec 2008, 22:41
QUOTE (Blind_Torture_Kill @ 2 Dec 2008, 22:37 )

მაგას აქვს ხშირად კავშირი B ჰეპატიტთან

: სახეშიხელისშემორტყმისსმაილიკი : (საკუთარ სახეში smile.gif )

up.gif up.gif

მაგარი ყ ვარ ეგ რო ვერ გავიხსენე user.gif
მცხვენია yes.gif

P.S
საღოლ, საღოლ ცვეტში ეგაა

PPS
რესპეკტ yes.gif


ძალიან გამიტყდა ამდენი რო ვიფიქრე gigi.gif user.gif user.gif

Posted by: vano_t 2 Dec 2008, 22:57
პოლიარტერიტი და საერთოდ სხვა ტიპის ვასკლუტიც შესაძლებელია. ANCA და ANA უარყოფითია და ეგ ამცირებს დიაგნოზს (არ გამორიცხავს ოღონდ).

ისე მიკამ პირველად ახსნა ეგ დიაგნოზი (X და Y-ებით).

Posted by: Blind_Torture_Kill 2 Dec 2008, 22:58
Cousteau

QUOTE
მაგარი ყ ვარ ეგ რო ვერ გავიხსენე 
მცხვენია


Xუი სნიმ მერე რა

ეხლა კაი ქეისს დავაგდებ და მაგაზე იფიქრე
* * *
A 33 y/o female presented with 9 months history of persistant watery diarrhea.prior to delivery of her first child she had been in excellent health apart from occasional urinary tract infection.diarrhea had commenced quite suddenly 1.5 weeks after delivery.normal bowel habit was thereafter never re-established.stools were liquid often watery of normal colour and inoffensive and numbering up to 15 daily.there was occasional mucus but never blood.pain fever constitutional disturbances were strikingly absent.

PE – serum pH 7.35 , hypercalcemia , hypokalemia , achlorhydria

რა ჭირს აბა

Posted by: mika9 3 Dec 2008, 00:06

ატროპიული გასტრიტი ჰყპერ ჰყპერ გასტრინემია>>>>>>დიარეა....

მაგრამ ეხლა ვეძებდი ორსულობასთან კავშირს მაგრამ რავი ვერ ვიპოვე და თუ ჩემი პასუხი სწორია ორსულობა არის რამე შუაში????


ისე შენიშვნა

სასურVეია ყველა ქეისი დაიდოს ქართულად.................................ჯერ ერთი Yველამ არ იცის ინგლისური და მერე ის რომ ადრე როცა ეს თემა გაიხსნა მაშინ ინგლისურად იდებოდა და როგორც აღმოჩნდა გოოგლში რამოდენიმე სიტყვის კომბინაციის და სერQვის შემდეგ გიგდებდა ქეისს......


Posted by: Blind_Torture_Kill 3 Dec 2008, 00:15
mika9

კაი ამის მერე ვთარგმნი და ისე დავდებ smile.gif

მგონი ამის ძებნა ცოტა გაძნელდება smile.gif

Posted by: mika9 3 Dec 2008, 00:19
Blind_Torture_Kill
ჩემმი პასუხი არასწორია?

Posted by: vano_t 3 Dec 2008, 00:40
QUOTE (Blind_Torture_Kill @ 2 Dec 2008, 22:58 )
A 33 y/o female presented with 9 months history of persistant watery diarrhea.prior to delivery of her first child she had been in excellent health apart from occasional urinary tract infection.diarrhea had commenced quite suddenly 1.5 weeks after delivery.normal bowel habit was thereafter never re-established.stools were liquid often watery of normal colour and inoffensive and numbering up to 15 daily.there was occasional mucus but never blood.pain fever constitutional disturbances were strikingly absent.

PE – serum pH 7.35 , hypercalcemia , hypokalemia , achlorhydria

რა ჭირს აბა

PE რა შუაშია ლაბორატორიასთან? მერე აქლორჰიდრიაში რას გულისხმობ: შრატის ტესტები გაქვს და მერე უცებ ახსენებ აქლორჰიდრიას.

იტოგში ცოტა უცნაური პრეზენტაციაა. არ მგონია ასე ადვილად მაქ დიაგნოზი დაისვას. მარა სავარაუდო დიაგნოზები ბევრი შეიძლება იყოს.

ასეთი დიარეის დროს რომ არ ახსენებ განავლის ანალიზს საოცარია. ქრონიკული დიარეის დროს პირველ რიგში უნდა დაადგინო რა ტიპის დიარეასთან გაქვს საქმე (სეკრეტორული, მალაბსორბცია, ანთება და ა.შ.). განავლის ოსმოლარობა და ანიონური სხვაობა უნდა დაიდოს აუცილებლად. მერე, შიმშილი ხსნის თუ არა დიარეას ისიც უნდა ახსენო.

1) ლაქსატივების abuse მოგცემს ყველაფერს მაგას, გარდა აციდოზისა. ჰიპოკალემია და ჰიპერკალცემია იქნება კალიუმის დაკარგვის და დეჰიდრატაციის შედეგი. შიეძლება წონაში დაკლება უნდა და ლაქსატივებს იღებს.

2) multiple endocrine neoplasia-მაც შეიძლება მოგცეს ეგ. ზოგს შეიძლება ზოლინგერ ელისონი ქონდეს; ზოგს კიდე medullary thyroid carcinoma.

Posted by: Blind_Torture_Kill 3 Dec 2008, 00:53
QUOTE
PE რა შუაშია ლაბორატორიასთან? მერე აქლორჰიდრიაში რას გულისხმობ: შრატის ტესტები გაქვს და მერე უცებ ახსენებ აქლორჰიდრიას


ეს მაპატიეთ
ინსტინქტურად დავწერე
* * *
QUOTE
stools were liquid often watery of normal colour and inoffensive and numbering up to 15 daily



Posted by: vano_t 3 Dec 2008, 01:01
ხო კიდე, VIP-ომებიც შევა დიაგნოზში.

Posted by: Blind_Torture_Kill 3 Dec 2008, 01:02
vano_t
QUOTE
VIP-ომებიც შევა დიაგნოზში


ვერნერ მორისონის სინდრომი
საღოლ
* * *
vano_t

ამ ქეისებს ლაივ სიტუაციებიდან კი არ ვიღებ ესე სტროგად რომ მთხოვთ მთელ ლაბორატორიულ მონაცემებს smile.gif

Posted by: vano_t 3 Dec 2008, 01:10
QUOTE (Blind_Torture_Kill @ 3 Dec 2008, 00:53 )
QUOTE
stools were liquid often watery of normal colour and inoffensive and numbering up to 15 daily

ეს თავისთავად არაფერს ნიშნავს. უფრო მეტად მიუთითებს სეკრეტორულ ან ოსმოსურ დიარეაზე და ყველა ზემოჩამოთვლილმა რამეებმა შეიძლება მოგცეს. შეიძლება 15-ჯერ არ გავიდნენ დღეში, მარა ზოლინგერ-ელისონმა და ფარისებრის მედულარულმა კიბომ, ასევე დიდი რაოდენობით ლაქსატივების გამოყენებამ შეიძლება მოგცეს ძლიერი დიარეა.

QUOTE
ამ ქეისებს ლაივ სიტუაციებიდან კი არ ვიღებ ესე სტროგად რომ მთხოვთ მთელ ლაბორატორიულ მონაცემებს
დაისჯები, ო, დაისჯები biggrin.gif

ისე კაი ხანდახან ლაბორატორიაც, საქმეს უხდება ხოლმე და დაიგნოზში გეხმარება wink.gif

Posted by: Blind_Torture_Kill 3 Dec 2008, 01:14
vano_t

აქლორჰიდრია იმიტო დავწერე რომ მაგი გევარაუდათ

ზოლინგერ ელისონის დროს დიარეა + მრავალი წყლულები წვრილ ნაწლავში და ტკივილიც ექნება პაციენტს და სისხლიც განავალში

ლაქსატივებზე დავწერდი მაშინ რომ სქელი ქალი იყოთქო

მედულარულ კიბოზე კიდე ფარისებრს ერთხელ მაინც ვახსენებდი ქეისში biggrin.gif

Posted by: vano_t 3 Dec 2008, 01:26
QUOTE (Blind_Torture_Kill @ 3 Dec 2008, 01:14 )
აქლორჰიდრია იმიტო დავწერე რომ მაგი გევარაუდათ

ზოლინგერ ელისონის დროს დიარეა + მრავალი წყლულები წვრილ ნაწლავში და ტკივილიც ექნება პაციენტს და სისხლიც განავალში

ტკივილი ხშირია, ოღონდ არ არის აუცილებელი. სისხლდენები კი ნაკლებად ხშირია (25 % პაციენტების, emedicine-ის მიხედვით).

სავსებით შესაძლებელია, რომ მასეთ პაციენტს ქონდეს ზოლინგერ ელისონი, ან რომელიმე მრავლობითი ენდოკრინული ნეოპლაზია, ან ლაქსატივების მიღება. და მასეთი დიარეის დროს ალბათ უნდა გამოიკვლიო კიდეც მაგ ყველაფერზე. უფროს სავარაუდო დიაგნოზი შეიძლება არის VIP-ომა, მაგრამ სავარაუდო და არა აუცილებელი.

დიფ-დიაგნოზ უპირველეს ყოვლისა. აკაკი წერეთელი biggrin.gif

Posted by: Blind_Torture_Kill 3 Dec 2008, 01:36
vano_t

ამის მერე რამეს უფრო ჩავხლართავ smile.gif




რა არის და როგორია პროგნოზი

Posted by: Guardian 3 Dec 2008, 11:11
mika9
QUOTE
Yველამ არ იცის ინგლისური

ჰოდა, ისწავლონ მერე. smile.gif

QUOTE
როგორც აღმოჩნდა გოოგლში რამოდენიმე სიტყვის კომბინაციის და სერQვის შემდეგ გიგდებდა ქეისს......

ჩემი ქეისები არ იგუგლება. biggrin.gif

Blind_Torture_Kill

ეგ საერთოდ რა ორგანოა? spy.gif

Posted by: Blind_Torture_Kill 3 Dec 2008, 11:38
Guardian

biggrin.gif

QUOTE
Blind_Torture_Kill

ეგ საერთოდ რა ორგანოა? 



გამოიცანი

Posted by: Tornike Alashvili 3 Dec 2008, 17:56
ქალბატონს 55 წლისა დაეწყო ღრძილებიდან ჩირქისდენა–უკავშირებდა სიგარეტზე თავის დანებებას
2–3 თვე მკურნალობდა სტომატოლოგებთან უშედეგოდ.დაერთო საშინელი სისუსტე და კიდურების დაბრუჟებები.შენელდა მეტყველებაც .გააგრძელა კონსულტაციები ნევროპათოლოგებთან და სხვა სპეციალისტებთან 6 თვე მდგომარეობა პროგრესირებდა ...პროფესორ–აკადეიკოსების აზრი –არ ვიცით რისი ბრალია ვარაუდობდნენ ინტოქსიკაციას იკვლევდნენ ინფექციებზე .... ვიზიტის დროს ყურადღებას იბყრობდა ის გარემოებაც რომ ქალბატონი თითქოს დაჭკნა და დაბერდა ამ 6 თვეში(მანამდეც ვიცნობდი –ქალიშვილი და სიძე დადიოდნენ ჩემთან და მახსოვდა–ცოცხალი ენერგიული ქალბატონი იყო)

დიაგნოზი მარტივია– მადრამ ცოტა არატიპიური.

Posted by: donvaso 3 Dec 2008, 18:06
Blind_Torture_Kill
გაგლეჯილი ღვიძლი?????????
eek.gif
Tornike Alashvili
არ თქვათ კლიმაქსის რომელიმე უჩვეულო ფორმაათქო და თქვენ მერე ალბათ გაახალგაზრდავეთ კიდეც....
war.gif

Posted by: Blind_Torture_Kill 3 Dec 2008, 18:17
QUOTE
Blind_Torture_Kill გაგლეჯილი ღვიძლი?????????


donvaso
არა
wink.gif

Posted by: Tornike Alashvili 3 Dec 2008, 19:00
donvaso
არავითარი კლიმაქსი

Posted by: Solveig 3 Dec 2008, 23:26
სურავანდი მაშინ biggrin.gif

30 ტაბლეტი ვიტამინი

Posted by: vano_t 3 Dec 2008, 23:26
Tornike Alashvili
QUOTE
ქალბატონს 55 წლისა დაეწყო ღრძილებიდან ჩირქისდენა
რას ქვია ღრძილებიდან ჩირქდენა? ადგილობრივი ჩრიქდენა იყოს? (ანუ ლოკალური აბსცესი) თუ დიფუზური?

QUOTE
პროფესორ–აკადეიკოსების აზრი –არ ვიცით რისი ბრალია ვარაუდობდნენ ინტოქსიკაციას იკვლევდნენ ინფექციებზე
მაინც როგორ მოხდა ინფეზციებზე კვლევა და რა ინფექციები გამოირიცხა?-ეს არის ყველაზე მნიშვნელოვანი კითხვა ამჟამად, რამეთუ ავადმყოფს აშკარა ქრონიკული ინფექციის ნიშნები აქვს (ქრონიკული ჩირქდენა) და საჭიროა იმის ცოდნა, თუ რა ინფექციები გამოირიცხა და როგორ? თუ არ არის ყველა სავარაუდო ინფექცია გამორიცხული, მაშინ არ ყოფილა სრული გამოკვლევა. რას ნიშნავს ვარაუდობდნენ ინტოკსიკაციას და იკვლევდენ სხვა რამეს (ინფექციებს)? რითი ინტოქსიკაციას ვარაუდობდნენე და თუ იქნა ტოქსინების შემცველობა დადგენილი სისხლში?

QUOTE
ვიზიტის დროს ყურადღებას იბყრობდა ის გარემოებაც რომ ქალბატონი თითქოს დაჭკნა და დაბერდა ამ 6 თვეში(მანამდეც ვიცნობდი –ქალიშვილი და სიძე დადიოდნენ ჩემთან და მახსოვდა–ცოცხალი ენერგიული ქალბატონი იყო)
ეს თითქოსდა დაჭკნობა რაიმე პათოგმონური ნიშანია?

P.S. არ მიიღება ისეთი პასუხები, როგორიც არის სიძისადმი ალერგია (და შესაბამისად სიძის უმცირეს-არარსებული დოზებით მკურნალობა) ან კიდევ ნიკოტინის შეწყვეტა, როგორც ამ ყველაფრის გამომწვევი (მთვარის ბნელ მხარეს ყველაფერია მოსალოდნელი, მაგრამ ბნელი მხარე არ ვიცითი და ნათელი მხარის თვისებებზე ვისაუბროთ თუ შეიძლება). შესაბამისად, იმედია ავადმყოფს არ ურჩია ვინმემ ნიკოტინის განახლება (ნუ ძაან განზავებული ნიკოტინის დოზები წავა ალბათ).

Posted by: basa-ttt 4 Dec 2008, 08:26
QUOTE
P.S. არ მიიღება ისეთი პასუხები, როგორიც არის სიძისადმი ალერგია

ძალიან საინტერესო სიმპტომია.
და ხშირად ნევროზის მიზეზი -
მარა თქვენ ეს გაინტერესებთ?
არა ,
არ გაინტერესებთ...

QUOTE
ნუ ძაან განზავებული ნიკოტინის დოზები წავა ალბათ

100 გვერდიან თემას ცოტა შედეგი დაეტყო.
სწორია.

Posted by: vano_t 4 Dec 2008, 09:36
basa-ttt
QUOTE
მარა თქვენ ეს გაინტერესებთ?
არა ,
არ გაინტერესებთ...
დიახ: არა, არ მაინტერესებს.

QUOTE
QUOTE
ნუ ძაან განზავებული ნიკოტინის დოზები წავა ალბათ

100 გვერდიან თემას ცოტა შედეგი დაეტყო.
სწორია.
თფუი, დალახვროს ეშმაკმა. ამ განყოფილებაში ხუმრობაც აღარ შეიძლება biggrin.gif

Posted by: basa-ttt 4 Dec 2008, 09:43
QUOTE
დიახ: არა, არ მაინტერესებს.

ვიცი ვიცი,
დაიტანჯოს ადამიანი ნევროზით -
რა მოხდა მერე -
gigi.gif


QUOTE
თფუი, დალახვროს ეშმაკმა. ამ განყოფილებაში ხუმრობაც აღარ შეიძლება

ქვეცნობიერად ღაღადებ სიმართლესა....
gigi.gif

Posted by: Blind_Torture_Kill 4 Dec 2008, 14:46

http://imageshack.us


ამის დიაგნოზი მინდა

Posted by: mika9 4 Dec 2008, 15:03
basa-ttt

ნერვოზი რა არის???? დიაგნოზს როგორ დასვამ???? და როგორ უმკურნალებ?????







Posted by: Tornike Alashvili 4 Dec 2008, 16:16
vano_t
QUOTE
რას ქვია ღრძილებიდან ჩირქდენა? ადგილობრივი ჩრიქდენა იყოს? (ანუ ლოკალური აბსცესი) თუ დიფუზური?

დიფუზური–სტომატოლოგებმა უარი თქვეს–ჩვენი პრობლემა არ არისო.

QUOTE
მაინც როგორ მოხდა ინფეზციებზე კვლევა და რა ინფექციები გამოირიცხა?-ეს არის ყველაზე მნიშვნელოვანი კითხვა ამჟამად, რამეთუ ავადმყოფს აშკარა ქრონიკული ინფექციის ნიშნები აქვს

ბორნელია ქლამიდია და რიგი სხვა –მაგრამ არასწორი მიმართულებაა არც დადასტურდა რაიმე ინფექცია და საერთოდ არ იყო ეს პრობლემა
QUOTE
ეს თითქოსდა დაჭკნობა რაიმე პათოგმონური ნიშანია?

არა მაგრამ ერთერთი ნიშანია
QUOTE
იმედია ავადმყოფს არ ურჩია ვინმემ ნიკოტინის განახლება (ნუ ძაან განზავებული ნიკოტინის დოზები წავა ალბათ).

ნიკოტინს განზავებულსაც არ ვაძლევდით და ეს არ არის წამყვანი საერთოდ –აქაც არ გირჩევთ ძებნას
კლინიკური დიაგნოზი გვინდა –არავითარი ჰომეოპათიური ან რაიმე ჩინური –რა შეიძლება იყვეს ან კიდევ რა გამოკვლევებს ჩაუტარებდით?

Posted by: Cousteau 4 Dec 2008, 16:49
Blind_Torture_Kill

ერთიდაიგივე დაავდებაა?

Posted by: LUKA-BRAZI 4 Dec 2008, 16:50
ბიჭებს გაუმარჯოს, როგორც იქნა დავბრუნდი ფორუმზე (თუ კიდე რამე არ მოხდა biggrin.gif), კარგია რომ თემას სიმწვავე არ აკლდება! ჰო, იმის თქმა მინდა რომ დღეს, თუ არადა ხვალ აუცილებლად, ისეთ ქეისს დავდებ, რომლის გამოცნობაც ღირსების და პრესტიჟის საქმეა! biggrin.gif biggrin.gif მოკლედ თავის გაცხელება მოგიწევთ და ვნახოთ! ვინც ჩემს ქეისს პირველივე ჯერზე გამოიცნობს, არ ვიცი, ალბათ ძალიან მაგარი ვინმე იქნება wink.gif

Posted by: Blind_Torture_Kill 4 Dec 2008, 16:51
Cousteau

QUOTE
ერთიდაიგივე დაავდებაა?



კი

* * *
LUKA-BRAZI

გაგიმარჯოს smile.gif
მოუთმენლად ველით შენ ქეისს

Posted by: LUKA-BRAZI 4 Dec 2008, 16:56
Blind_Torture_Kill
QUOTE
გაგიმარჯოს smile.gif მოუთმენლად ველით შენ ქეისს

შენც გაგიმარჯოს! smile.gif კი, კი უბრალოდ ამოიწუროს ეს ქეისები, ანუ გაეცეს პასუხები და ეგრევე დავდებ! შენს დაბრუნებასაც ველოდებოდი პრინციპში და ამასობაში მე ნეტი არ მქონდა ასე რომ ჩემი დაბრუნება ხმაურიანი ქეიცით უნდა ავღნიშნო biggrin.gif

Posted by: Blind_Torture_Kill 4 Dec 2008, 16:59
LUKA-BRAZI

QUOTE
შენც გაგიმარჯოს!კი, კი უბრალოდ ამოიწუროს ეს ქეისები, ანუ გაეცეს პასუხები და ეგრევე დავდებ! შენს დაბრუნებასაც ველოდებოდი პრინციპში და ამასობაში მე ნეტი არ მქონდა ასე რომ ჩემი დაბრუნება ხმაურიანი ქეიცით უნდა ავღნიშნო


ვიცი შენი წინა პოსტი წავიკითხე wink.gif
* * *
Tornike Alashvili

მეტი მახასიათებელი არაფერი არაა ?

leprosy

Posted by: Cousteau 4 Dec 2008, 17:08
QUOTE (Blind_Torture_Kill @ 4 Dec 2008, 16:51 )
Cousteau

QUOTE
ერთიდაიგივე დაავდებაა?



კი


ხ.ე.ზ სინდრომია ალბათ : /
tiger stripes-ს გავს მარა ეგ ცვეტში არაა : /
მოკლედ არვიცი ნამდვილად

Posted by: Blind_Torture_Kill 4 Dec 2008, 17:15
Cousteau

QUOTE
ხ.ე.ზ სინდრომია ალბათ


ეს რა არის ?

Posted by: Tornike Alashvili 4 Dec 2008, 17:26
QUOTE
მეტი მახასიათებელი არაფერი არაა ?

არა ცოტა იყო სიმპტომები მაგრამ გაუსაძლისი –დაბრუჟებები კიდურების ,სისტუსტე ,მეტყველების შეფერხება(შენელება)
ამ ეტაპზე უკვე დაისვა დიაგნოზი რომელიც შემდგომი კვლევებით დადასტურდა და დიაგნოზი იყო მარტივი ძაან და უბრალო –გამოკვლევების შემდეგ პაციენტმა დაამატა –როგორ ვერ მივხვდით –სხვათაშორის თმებიც რომ შევიჭერი ნახევარი წლის წინ –როგორც მაშინ იყო ისე დარჩაო ( არ გაზრდილა)

Posted by: Cousteau 4 Dec 2008, 17:33
QUOTE (Blind_Torture_Kill @ 4 Dec 2008, 17:15 )
Cousteau

QUOTE
ხ.ე.ზ სინდრომია ალბათ


ეს რა არის ?

gigi.gif

ხ*უ ეგო ზნაეტ (ხ.ე.ზ.) სინდრომი user.gif

Posted by: Tornike Alashvili 4 Dec 2008, 17:47
Cousteau
ХЕЗ-სინდრომი–მაგარია lol.gif

Posted by: Blind_Torture_Kill 4 Dec 2008, 17:52
Cousteau


lol.gif lol.gif lol.gif lol.gif

რა ორგანოა სურათზე

Posted by: Cousteau 4 Dec 2008, 17:57
QUOTE (Blind_Torture_Kill @ 4 Dec 2008, 17:52 )
Cousteau


lol.gif lol.gif lol.gif lol.gif

რა ორგანოა სურათზე

ჩორტივო ზნაეტ
ნაღვლის ბუშტი? user.gif

Posted by: donvaso 4 Dec 2008, 17:59
+ უზარმაზარი სიმსივნე........ ?


Posted by: Blind_Torture_Kill 4 Dec 2008, 18:04
QUOTE
ნაღვლის ბუშტი?


yes.gif

Posted by: Cousteau 4 Dec 2008, 18:17
QUOTE (Blind_Torture_Kill @ 4 Dec 2008, 18:04 )
QUOTE
ნაღვლის ბუშტი?


yes.gif

კედლის ფიბროზი/კალციფიკაცია ხო არაა ქრონიკული ქოლეცისტის შემდეგ? : / ტიპა კარცინომა დ.ა.შ.?

*****************************************

ვნახე Cholesterolosis of gallbladder - strawberry gallbladder

It is not necessarily tied to cholelithiasis or cholecystitis.
The condition has no clinical significance and is not associated with any symptoms.
Etiology
•Unknown
Pathogenesis
•Unknown,
Epidemiology
•Incidental finding


ეგაა?
რაარის მართალიროგითხრა საერთოდ არვიცი


ეგაა smile.gif
http://radiology.uchc.edu/eAtlas/GI/955.htm


Posted by: Blind_Torture_Kill 4 Dec 2008, 18:39
QUOTE
ვნახე Cholesterolosis of gallbladder - strawberry gallbladder

It is not necessarily tied to cholelithiasis or cholecystitis. The condition has no clinical significance and is not associated with any symptoms. Etiology •UnknownPathogenesis •Unknown,Epidemiology •Incidental finding

ეგაა? რაარის მართალიროგითხრა საერთოდ არვიცი


ეგაა

დალექილი ქოლესტეროლია კედელზე up.gif

Posted by: basa-ttt 4 Dec 2008, 18:39
QUOTE
Cholesterolosis of gallbladder - strawberry gallbladder

ქართულად რაა?
დიაგნოზი ქართულად დაწერეთ..

Posted by: Cousteau 4 Dec 2008, 18:39
QUOTE
დიაგნოზი ქართულად დაწერეთ..

ნაღვლის ბუშტის ქოლესტეროლოზი (ჟოლოსებრი (???) ნაღვლის ბუშტი) მგონი ასე იქნება


სანამ რამე სერიოზულს დადებენ

user posted image

?

Posted by: Blind_Torture_Kill 4 Dec 2008, 18:43
Cousteau

ასოში ძვლებია
biggrin.gif

Posted by: Cousteau 4 Dec 2008, 18:45
QUOTE (Blind_Torture_Kill @ 4 Dec 2008, 18:43 )
Cousteau

ასოში ძვლებია
biggrin.gif

gigi.gif
კი, რატო? gigi.gif

Posted by: Blind_Torture_Kill 4 Dec 2008, 18:49
Cousteau

QUOTE
Cousteau

ასოში ძვლებია

კი, რატო?


თითია და მაგიტო smile.gif

Posted by: Cousteau 4 Dec 2008, 18:50
QUOTE (Blind_Torture_Kill @ 4 Dec 2008, 18:49 )
Cousteau

QUOTE
Cousteau

ასოში ძვლებია

კი, რატო?


თითია და მაგიტო smile.gif

gigi.gif
yes.gif

და რა უნდა თითს ასოში? gigi.gif

Posted by: Blind_Torture_Kill 4 Dec 2008, 18:54
Cousteau

QUOTE
და რა უნდა თითს ასოში?

ალბათ მიაბეს ან ხ.ი.ზ სინდრომი biggrin.gif

Posted by: basa-ttt 4 Dec 2008, 19:03
იმპლანტანტია და მიაბეს?

ანუ ქალი კაცად გადააქციეს?
gigi.gif

Posted by: Cousteau 4 Dec 2008, 19:05
QUOTE (Blind_Torture_Kill @ 4 Dec 2008, 18:54 )
Cousteau

QUOTE
და რა უნდა თითს ასოში?

ალბათ მიაბეს ან ხ.ი.ზ სინდრომი biggrin.gif

QUOTE
იმპლანტანტია და მიაბეს?

ანუ ქალი კაცად გადააქციეს?


yes.gif

ამ ჭკვიანებმა თითი მოაჭრეს იმისთვის რო ეგ გაეკეთებინათ და გამოიყურება კიდე ძააააალიან ცუდად. (საზღვარგერეთ მგონი ასეთი რამე თავშიც არ მოსვლია არავის, თუმცა ზუსტად არ ვიცი)

Posted by: basa-ttt 4 Dec 2008, 19:11
მარჯვენა ბარძაყის ძვალთან შიდა ზედაპირზე - რაღაც ჩრდილია კიდოვ -
ბარძაყის ძვლის თავსა და მორჩს შორის.
ეგ რაღაა?

Posted by: Blind_Torture_Kill 4 Dec 2008, 19:44
http://imageshack.us
http://g.imageshack.us/img115/18549437pl1.jpg/1/


რა არის და როდის გვხვდება

Posted by: basa-ttt 4 Dec 2008, 19:57
სპიროქეტებია?
ანუ სიფილისი?

თხოვნაა - ფოტოსთან ერთად დაიდოს კლინიკაც...

Posted by: BadbadGirl 4 Dec 2008, 20:35
QUOTE (Cousteau @ 4 Dec 2008, 19:05 )
QUOTE (Blind_Torture_Kill @ 4 Dec 2008, 18:54 )
Cousteau

QUOTE
და რა უნდა თითს ასოში?

ალბათ მიაბეს ან ხ.ი.ზ სინდრომი biggrin.gif

QUOTE
იმპლანტანტია და მიაბეს?

ანუ ქალი კაცად გადააქციეს?


yes.gif

ამ ჭკვიანებმა თითი მოაჭრეს იმისთვის რო ეგ გაეკეთებინათ და გამოიყურება კიდე ძააააალიან ცუდად. (საზღვარგერეთ მგონი ასეთი რამე თავშიც არ მოსვლია არავის, თუმცა ზუსტად არ ვიცი)

ხალხო, ვიცი უადგილოა ჩემი შეკითხვა მაგრამ სულ მაინტერესებდა და იქნებ თქვენ იცოდეთ.
ასეთ შმეთხვევებში ერექცია როგორ ხდება?თუ არ ხდება საერთოდ ?

არ დამცინოთ იდოტ შეკითხვაზე პლიზ.


Posted by: basa-ttt 4 Dec 2008, 20:38
QUOTE
თუ არ ხდება საერთოდ ?

რა თქმა უნდა არ ხდება -
gigi.gif
მარა ევასებათ რომ აქვთ....
gigi.gif

Posted by: Tornike Alashvili 4 Dec 2008, 20:40
ერექცია არ ხდება –მოხრა და გასწურება–კი
ოღონდ სულ იმაზე უნდა იფიქრონ რომ თითი გამართული იყოს

user posted image

Posted by: donvaso 4 Dec 2008, 20:48
სულ ერეგირებულები არიან ხო???

ისადა, პლაჟზე მაგარ ცვეტში იქნებიან ეგენი.... biggrin.gif biggrin.gif

Posted by: Blind_Torture_Kill 4 Dec 2008, 20:59
basa-ttt

სპიროხეტებს ეგეთი მიკროსკოპით ვერ დაინახავ

Posted by: Solveig 4 Dec 2008, 21:05
Blind_Torture_Kill
ტრიპანოსომოზი.

Posted by: Blind_Torture_Kill 4 Dec 2008, 21:09
Solveig

up.gif


ყოჩაღ

ექიმი ხარ ?

Posted by: donvaso 4 Dec 2008, 21:11
Blind_Torture_Kill
Solveig - მისი სპეციალობაც ეგა რომ მასეთები ნახოს და იცნოს... biggrin.gif biggrin.gif wink.gif

Posted by: Blind_Torture_Kill 4 Dec 2008, 21:14
QUOTE
Blind_Torture_Kill Solveig - მისი სპეციალობაც ეგა რომ მასეთები ნახოს და იცნოს...

პარაზიტოლოგია ?

Posted by: Solveig 4 Dec 2008, 22:24
donvaso
QUOTE
მისი სპეციალობაც ეგა რომ მასეთები ნახოს და იცნოს

არა, არა smile.gif არც ექიმი ვარ, ბიოლოგიური განათლება მაქვს. უბრალოდ, ჰემატოლოგიას ვსწავლობდი რამდენიმე წლის წინ დიდი მონდომებით (ჩემი ვიწრო სპეციალობა მჭიდრო კავშირშია და იმიტომ).

ეს თემა ყველაზე მეტად იმიტომ მომწონს, რომ ჰისტოპრეპარატების სურათებსაც დებთ ხოლმე...ჰისტოლოგია ძალიან მიყვარდა და მაინტერესებდა, სამწუხაროდ, არ მომეცა საშუალება, კარგად შემესწავლა.

Posted by: vano_t 4 Dec 2008, 22:25
Tornike Alashvili
QUOTE
QUOTE
რას ქვია ღრძილებიდან ჩირქდენა? ადგილობრივი ჩრიქდენა იყოს? (ანუ ლოკალური აბსცესი) თუ დიფუზური?

დიფუზური–სტომატოლოგებმა უარი თქვეს–ჩვენი პრობლემა არ არისო.

QUOTE
მაინც როგორ მოხდა ინფეზციებზე კვლევა და რა ინფექციები გამოირიცხა?-ეს არის ყველაზე მნიშვნელოვანი კითხვა ამჟამად, რამეთუ ავადმყოფს აშკარა ქრონიკული ინფექციის ნიშნები აქვს

ბორნელია ქლამიდია და რიგი სხვა –მაგრამ არასწორი მიმართულებაა არც დადასტურდა რაიმე ინფექცია და საერთოდ არ იყო ეს პრობლემა
QUOTE
ეს თითქოსდა დაჭკნობა რაიმე პათოგმონური ნიშანია?

არა მაგრამ ერთერთი ნიშანია
QUOTE
იმედია ავადმყოფს არ ურჩია ვინმემ ნიკოტინის განახლება (ნუ ძაან განზავებული ნიკოტინის დოზები წავა ალბათ).

ნიკოტინს განზავებულსაც არ ვაძლევდით და ეს არ არის წამყვანი საერთოდ –აქაც არ გირჩევთ ძებნას
კლინიკური დიაგნოზი გვინდა –არავითარი ჰომეოპათიური ან რაიმე ჩინური –რა შეიძლება იყვეს ან კიდევ რა გამოკვლევებს ჩაუტარებდით?

მაგ ავადმყოფს არაფერი აქვს გაკეთებული და არაფერია გამორიცხული.

1) ჯერ ერთი, მეტად არასპეციფური სიმპტომები აქვს. ამ დროს დიაგნოზი კი არ უნდა დასხვა, არამედ სწორი სტრატეგია უნდა გქონდეს, რომელიც სავარაუდო დიაგნოზთან მიგიყვანს რაც შეიძლება მალე და ეფექტურად. 4000 დაავადების ცალკე განხილვა არაფერს მოგცემს.

2) ინფექციაზე საერთოდ არ არის გაცემული პასუხი შენს მიერ. რას ქვია ყველაფერი გაუკეთდა? ქრონიკული ჩირქდენა აქვს ავადმყოფს და არ მეუბნები რა გაუკეთდა, გარდა იმისა, რომ ყველაფერი გაუკეთდაო? იმიტომ გკითხე რა გაუკეთდა, რომ მიპასუხო რა გაუკეთდა. თუ არ იცი რა გაუკეთდა, მაშინ არაფერი გაკეთებულა ან არ იცი რა უნდა გაკეთებულიყო. ჩირქი დაითესა საერთოდ ანაერობებზე და აერობებზე? თუ დაითესა რა ბაქტერიები ინახა? სისხლი დაითესა საერთოდ? თუ დაითესა, რა ბაქტერიები ინახა? ინფექციის კერის ძებნა ჩატარდა საერთოდ? თუ ჩატარდა რა მიმართულებებით? მაგალითად, ნაწლავების დაავადებები გამოირიცხა? ენდოკარდიტი გამოირიცხა? ღრმა აბსცესები გამოირიცხა? თავიდან თუ ჩაუტარდა (დათესვების შემდეგ) ემპირიული მკურნალობა ანტიბიოტიკებით (და შემდეგ მგრძნობელობის მიხედვით) და რომელი ანტიბიოტიკებით? თუ ჩაუტარდა, ქონდა თუ არა მკურნალობას შედეგი?

3) სისუსტე, სიდამჭკნე, დაბუჯება და ა.შ. არის არასპეციფიური სიმპტომები, რაც ბევრ ხშირ დაავადებას შეიძლება ახლდეს თან. ამის გამო, ასეთი ხშირი დაავადებები თავიდანვე უნდა გამოირიცხოს და თუ ასეთი დაავადება აღმოჩნდა ხელშემწყობი მიზეზი ინფექციების და ეს დაავადება გვიანამდე არ იქნა ნაპოვნი, ეს კიდევ უფრო მიუთითებს გაპარტახებულ სამედიცინო განათლებას.

4) გამოირიცხა თუ არა დიაბეტი? გამოირიცხა თუ არა ფარისებური ჯირკვლის ფუნქციის დარღვევები? შემოწმდა თუ არა ღვიძლის ენზიმები და სხვა ტესტები ღვიძლისათვის (ბილირუბინი, ალბუმინი, გლობულინი, ტუტე ფოსფატაზა)? თუ შემოწმდა, როგორი იყო რეზულტატი? როგორია თირკმლის ფუნქცია (კრეატინინი, შარდოვანა) და როგორია ელექტროლიტები (კალციუმი, მაგნიუმი, ქლორი, ბიკარბონატი, კალიაუმი და ნატრიუმი)?

გაუკეთდა ეს მარტივი და ინფორმაციული ტესტები? თუ გაუკეთდა რა არის შედეგები.

ჯერ ესენი უნდა გაუკეთდეს და მერე ვილაპარაკოთ დანარჩენზე: კიდევ არის ბევრი რამ, რაც შეიძლება მსგავსი პრეზენტაციის მიზეზი იყოს, ოღონდ ჯერ დავიწყოთ ხშირი დაავადებებით.

Posted by: basa-ttt 4 Dec 2008, 22:42
vano_t
"ნახევარი წლის წინ შევიჭერიო თმები და იმის მერე არ გაზრდილაო."
ჰოდა მაგას კითხულობს ავტორი-
კიდევ რას გამოიკვლევდითო...
მე მგონი ყველაფერი ნათქვამია.

Posted by: vano_t 4 Dec 2008, 23:06
QUOTE (basa-ttt @ 4 Dec 2008, 22:42 )
"ნახევარი წლის წინ შევიჭერიო თმები და იმის მერე არ გაზრდილაო."
ჰოდა მაგას კითხულობს ავტორი-
კიდევ რას გამოიკვლევდითო...
მე მგონი ყველაფერი ნათქვამია.

1) თმა ან ნელა იზრდება, ან თუ შეწყდა მისი ზრდა, მაშინ თმის ცვენა გადააჭარბებს და გამელოტება დაიწყება.

2) თმის ზრდის შენელება შეიძლება დამახასიათებელი იყოს სხვადასხვა ჰორმონული პრობლემებისათვის და არა 1 დაავადებისათვის.

3) აქ თმის ზრდის შეჩერება რა კავშირშია ქრონიკულ ჩირქოვან დიფუზურ გინგივიტთან?

4) რა დაავადებები გაძლევს სისუსტეს, დაბუჯებას და მეტყველების დარღვევას?

5) შეიძლება თუ არა ავადმყოფს ქონდეს რამოდენიმე პათოლოგია?

Posted by: donvaso 4 Dec 2008, 23:10
vano_t
ვერ მიხვდით?????
აქ რაღაც მისტიკაა ბატონო ვანო....... biggrin.gif
დაჭკნობა, თმის არზრდა - ასეთ კლინიკურ სურათს ენიჭება ძირითადი მნიშვნელობა... wink.gif biggrin.gif

Posted by: LUKA-BRAZI 4 Dec 2008, 23:12
ბოლო პოსტებს ვეღარ მივადევნე თვალი, ამოიხსნა ქეისი? შეიძლება უკვე ახლის დადება?

Posted by: basa-ttt 4 Dec 2008, 23:14
QUOTE
4) რა დაავადებები გაძლევს სისუსტეს, დაბუჯებას და მეტყველების დარღვევას?

QUOTE
ნელა იზრდება

http://mega.km.ru/HEALTH/encyclop.asp?topic=STAT_ENDOCRIN_6671&rubr=STAT_ENDOCRIN_6671

Заболевание обычно развивается постепенно. Больные жалуются на вялость, сонливость, безразличие к окружающему. Обращает на себя внимание замедленность речи и движений. Лицо заплывшее, желтовато-бледное, с узкими щелками глаз и очень бедной мимикой.

Нередко развиваются пародонтоз и кариес зубов

QUOTE
ვერ მიხვდით?????
აქ რაღაც მისტიკაა ბატონო ვანო.......

გიპასუხებდი "რაღაცას" -
მარა მარხვაა ახლა...
givi.gif

Posted by: Blind_Torture_Kill 4 Dec 2008, 23:37
LUKA-BRAZI

დადე ახალი გელოდები

basa-ttt

ვერ გავიგე ჰიპოთირეოზია ?

Posted by: basa-ttt 4 Dec 2008, 23:40
QUOTE
ვერ გავიგე ჰიპოთირეოზია ?

კითხეთ კეისის ავტორს - მისი პაციენტია
მე რათ მეკითხებით?
gigi.gif

ეს ჩემი ვერსიააა.

Posted by: Blind_Torture_Kill 4 Dec 2008, 23:46
QUOTE
კითხეთ კეისის ავტორს - მისი პაციენტია მე რათ მეკითხებით?

ეს ჩემი ვერსიააა.


ჰიპოირეოზის დროს თმა მსხვრევადი ხდება თორე ზრდით კი იზრდება

კიდე მრავალი სხვა სიმპომია რომელიც არ ხსენებულა

Posted by: basa-ttt 4 Dec 2008, 23:52
QUOTE
ჰიპოირეოზის დროს თმა მსხვრევადი ხდება თორე ზრდით კი იზრდება

კიდე მრავალი სხვა სიმპომია რომელიც არ ხსენებულა

ნელა იზრდება თმებიც და ფრჩხილებიც.

QUOTE
კიდე მრავალი სხვა სიმპომია რომელიც არ ხსენებულა

ეს კეისი არაა -
კონკრეტული პაციენტია -
აუცილებელი არაა ყველა სიმპტომი ჰქონდეს
იშვიათია ცხოვრებაში კლასიკური შემთხვევები

Posted by: LUKA-BRAZI 5 Dec 2008, 00:17
Blind_Torture_Kill
Fu ra! Isev micydeba neti, dges agar gamova sad.gif mobilit var exla shemosuli. Xval sagamos davdeb am uishviatess qeiss. Ogond imedia yvelani iqnebit, vano_t, Guardian -c da a.sh. Imitom rom martlac uishviatesi patologiis gamocnoba mogicevt da martla mainteresebs ra taqtikit mixvalt diagnozamde da vin ufro axlos miva scor pasuxtan. Dgesve mindoda amis gaketeba magram provaiders es sulac ar adardebs biggrin.gif

Posted by: Blind_Torture_Kill 5 Dec 2008, 00:18
QUOTE
QUOTEჰიპოირეოზის დროს თმა მსხვრევადი ხდება თორე ზრდით კი იზრდება

კიდე მრავალი სხვა სიმპომია რომელიც არ ხსენებულა

ნელა იზრდება თმებიც და ფრჩხილებიც. QUOTEკიდე მრავალი სხვა სიმპომია რომელიც არ ხსენებულა

ეს კეისი არაა -კონკრეტული პაციენტია - აუცილებელი არაა ყველა სიმპტომი ჰქონდეს იშვიათია ცხოვრებაში კლასიკური შემთხვევები


ფრჩხილებიც მსხვრევადია მარა იზრდება

ეგ გასაგებია ყველაფერი ერთად რომ ვერ ექნება მარა საკვანძო რამე მაინც უნდა მიანიშნო

LUKA-BRAZI

ველით baby.gif

Posted by: basa-ttt 5 Dec 2008, 00:22
QUOTE
ეგ გასაგებია ყველაფერი ერთად რომ ვერ ექნება მარა საკვანძო რამე მაინც უნდა მიანიშნო

რა ქნას იმ საცოდავმა პაციენტმა -
მეტი ჩივილები არ აქვს და
კეისის ხათრით მოიგონოს?
gigi.gif


LUKA-BRAZI -
მეც ინტრესში ჩავვარდი-
რაა ამისთანა?
givi.gif


Posted by: LUKA-BRAZI 5 Dec 2008, 00:28
basa-ttt
xval naxavt biggrin.gif ogond mogrovdit yvela smile.gif

Posted by: vano_t 5 Dec 2008, 00:46
basa-ttt
QUOTE
QUOTE
4) რა დაავადებები გაძლევს სისუსტეს, დაბუჯებას და მეტყველების დარღვევას?

QUOTE
ნელა იზრდება

http://mega.km.ru/HEALTH/encyclop.asp?topic=STAT_ENDOCRIN_6671&rubr=STAT_ENDOCRIN_6671

Заболевание обычно развивается постепенно. Больные жалуются на вялость, сонливость, безразличие к окружающему. Обращает на себя внимание замедленность речи и движений. Лицо заплывшее, желтовато-бледное, с узкими щелками глаз и очень бедной мимикой.

Нередко развиваются пародонтоз и кариес зубов

1) სისუსტეს, დაბუჟებას და მეტყველების დარღვევას გაძლევს უამრავი რამ. მათ შორის დიაბეტიც. გარდა ამისა, დიაბეტიანი ავადმყოფები გაცილებით უფრო მაღალი რისკის პაციენტები არიან ქრონიკული ინფექციებისათვის.

2) როცა ასეთი არასპეციფიური სიმპტომებით მოდის ავამდყოფი, არ უნდა აქ ფილოსოფია: ყოველთვის ამოწმებ მარტივ ტესტებს (ის რაც ჩამოვთვალე და რისვთვისაც ყურადღება არ მიგიქცევია), მათ შორის TSH და გლუკოზას. იმდენად მარტივია და კარგი სადიაგნოსტიკო ეს ტესტები და იმდენად ხშირია ეს დაავადებები (თანაც იმდენად არასპეციფიური და მრავალფეროვანი პრეზენტაცია აქვთ), რომ ამ ტესტს ყველას უკეთებ. არანორმალურია ტესტი? მაშინ დიაგნოზი გაქვს. ნორმალურია ტესტი? მაშინ ეგ დიაგნოზები გამორიცხულია. ასე მარტივად არის აქ საქმე.

3) პეროდონტოზი და კარიესი რა შუაშია ქრონიკულ ჩირქდენასთან? პაროდონზტოზი ანთებითი პროცესიც კი არ არის. კარიესი თავისთავად არაფერს ჩირქს არ მოგცემს თუ დენტალური აბსცესი არ აქვს პაციენტს.

და კიდევ, არ დამიწყო ახლა კამათი. რაც არ იცი, იმაზე ჯობია გაჩერდე.

Posted by: basa-ttt 5 Dec 2008, 01:03
QUOTE
1) სისუსტეს, დაბუჟებას და მეტყველების დარღვევას გაძლევს უამრავი რამ. მათ შორის დიაბეტიც. გარდა ამისა, დიაბეტიანი ავადმყოფები გაცილებით უფრო მაღალი რისკის პაციენტები არიან ქრონიკული ინფექციებისათვის.

გარკვევით დაგიწერეს გამოკვეულია პაციენტი ლაბორატორიულადაო, თუ მარტო შენ ხარ ჭკვიანი და ვერ მიხვდნენ შაქარიც ენახათ?

QUOTE
2) როცა ასეთი არასპეციფიური სიმპტომებით მოდის ავამდყოფი, არ უნდა აქ ფილოსოფია: ყოველთვის ამოწმებ მარტივ ტესტებს (ის რაც ჩამოვთვალე და რისვთვისაც ყურადღება არ მიგიქცევია),

ეს შენ არ მიგიქცევია ყურადღება-
ავტორ იკითხულობს -
რას გაუკეთებთო
და სწორედ ეგ უნდა გეთქვა ამდენ ფილოსოფიას.

QUOTE
3) პეროდონტოზი და კარიესი რა შუაშია ქრონიკულ ჩირქდენასთან? პაროდონზტოზი ანთებითი პროცესიც კი არ არის. კარიესი თავისთავად არაფერს ჩირქს არ მოგცემს თუ დენტალური აბსცესი არ აქვს პაციენტს.

პეროდონტოზი რაა?
ისე კი თუ მოხდა კბილის გარშემო ღრძილის ანთება და წარმოქმნილი ჯიბის დაჩირქება - არ შეიძლება ჩირდენა იყოს?
აკრძალულია აქვს?

QUOTE
და კიდევ, არ დამიწყო ახლა კამათი. რაც არ იცი, იმაზე ჯობია გაჩერდე

givi.gif

ვანოს აქვს თვითმიზანი ცხოვრებაში -
დაამტკიცოს რომ ის არის ჭკვიანი
და ჰომეოპათები არა.


QUOTE
იმდენად მარტივია და კარგი სადიაგნოსტიკო ეს ტესტები და იმდენად ხშირია ეს დაავადებები (თანაც იმდენად არასპეციფიური და მრავალფეროვანი პრეზენტაცია აქვთ), რომ ამ ტესტს ყველას უკეთებ. არანორმალურია ტესტი? მაშინ დიაგნოზი გაქვს. ნორმალურია ტესტი? მაშინ ეგ დიაგნოზები გამორიცხულია. ასე მარტივად არის აქ საქმე.

მანდ ჯიბის საქმეც არის მარტივად, ვიდრე აქ
და იცი აქ რა ღირს ეს მარტივი გამოკვლევები?
50 ლარზე ადის-
ზოგს კი პენსია 70 ლარი აქვს.
და ასე მარტივად არ წყდება ეგ ამბავი-
ასეა აქ.
იქ ყველაფერს მზამზარეულს გართმევენ
და აქ მეტი უნდა იფიქრო ნაკლებ კომფორტულ სიტუაციაში

ეს კეისები ვირტუალური სამყაროა.
და მათი პრაქტიკული ღირებულება 20 -30 % თუა..

Posted by: Tornike Alashvili 5 Dec 2008, 02:31
vano_t
QUOTE
4) გამოირიცხა თუ არა დიაბეტი? გამოირიცხა თუ არა ფარისებური ჯირკვლის ფუნქციის დარღვევები? შემოწმდა თუ არა ღვიძლის ენზიმები და სხვა ტესტები ღვიძლისათვის (ბილირუბინი, ალბუმინი, გლობულინი, ტუტე ფოსფატაზა)? თუ შემოწმდა, როგორი იყო რეზულტატი? როგორია თირკმლის ფუნქცია (კრეატინინი, შარდოვანა) და როგორია ელექტროლიტები (კალციუმი, მაგნიუმი, ქლორი, ბიკარბონატი, კალიაუმი და ნატრიუმი)?

დიაბეტი გამოირიცხა ფარისებული ჯირყვლის ფუნქციები სანამ ჩეამდე მოვიდოდა არ იყო გამოკვლეული ,ღვიძლის ფუნქციებიც –ნორმაში იყო ,თირკლის ფუნქციაც და ბიოქიმიაც –ნორმის ფარგლებში ,კალი ,კალცი და მინერალები –არ გაკეთებულა

მოკლედ აწი სირთულეს არ უნდა წარმოადგენდეს
რადგან ვანო ფარისებულ ჯირყვალს გამოიკვლევდა –აღმოაჩენდა რომ აუტოიმუნური თირეოიდიტია ფარისებული ჯირყვლის ჰიპოპლაზიით (საერთო მოცულობა –4.8 სმკუბში) თირეოტროპული ჰორმონი 50 იყო (15–20ჯერ აწეული)
სხვა პათოლოგია პაციენტს არ ქონია ვინაიდან დავიწყეთ თუ არა ჩიყვის მკურნალობა –გამონადენიც შეწყდა ღრძილებიდან და დაბრუჟებებმაც გაიარა, სისუსტე და მეტყველების შეფერხებასთან ერთად.ერთ თვეში ფარ.ჯირყვლის მოცულობა 10 იყო.2–3 თვეში –12.გასულია 3–4 წელი.სხვა პათოლოგია გამოვლენილი არ არის პაციენტის მდგომარეობა დამაყმაყოფილებელია

რა კავშირია თამბაქოსთან –ზუსტად არ ვიცი მაგრამ ისეთი შთაბეჩტილება შენმექმნა რამოდენიმე მსგავსი შემთხვევიდან –თამბაქოს წევა სტიმულირებს ფარისებული ჯირყვლის ფუნქციას და მისი შეწყვეტისას –ფუნქცია ეცემა –და ამ დროს ადამიანი წონაში იმატებს ან აქტივირდება ფარულად მიმდინარე თირეოიდიტი

Posted by: vano_t 5 Dec 2008, 05:28
Cousteau
QUOTE
ხ*უ ეგო ზნაეტ (ხ.ე.ზ.) სინდრომი  user.gif

lol.gif lol.gif
უმძიმესი სინდრომია biggrin.gif პ.ს.ო-ს (პიზ**ც, სკაზალ ოტეც) შკალაზე ყველაზე მძიმე ფორმაა.

Posted by: basa-ttt 5 Dec 2008, 09:28
vano_t
დედა,
რა გაცინებს?
გამოვქლიავდით?
აბა ერთი ამას გაეცანით
და მერე იცინეთ...

http://www.dr-md.ru/news/?id=12

Но что именно оказывает влияние на щитовидную железу? Дым? Продукты горения или особые вещества самого табака? Научные исследования показали, что существует несколько механизмов, влияющих на уровень гормонов щитовидной железы. Табачный дым содержит несколько токсинов. К ним относится тиоцианат и 2,3-гидроксипиридин. Тиоцианат, продукт с полураспадом в 6 дней (то есть выводится из организма за это время), изменяет поступление и усвоение йода в щитовидной железе, а также способствует утечке йода из железы. Дефицит йода ведёт к увеличению щитовидной железы. Развивается зоб. 2,3-гидроксипиридин, наоборот, предотвращает дейодирование, и временно повышает уровень тироксина (Т4) в крови.

В одном из исследований определяли различие течения гипотиреоза у курящих и некурящих женщин. У женщин с субклиническим течением гипотиреоза (начальной, малопроявленной формой заболевания щитовидной железы, с низкой функциональной способностью) на фоне курения определялась более высокая концентрация гормона ТТГ (тиреотропного гормона гипофиза) и более выраженное нарушение гормонального обмена Т4 и Т3, по сравнению с некурящими. Вместе с тем, при клинически проявленном гипотиреозе, независимо от курения, определялись близкие уровни концентраций гормона щитовидной железы, но у курящих проявления заболевания щитовидной железы были более выражены. Это свидетельствует о том, что курение значимо уменьшает продукцию гормонов щитовидной железой при субклиническом течении гипотиреоза, и увеличивает проявления заболевания железы при выраженном гипотиреозе.
Источник: European Journal of Endocrinology

თუ მაიმუნივით ვიცინით ყველაფერზე -
რასაც ჰომეოპათი იტყვის?

მართლა ცუდად იქცევით უკვე რამდენიმე იუზერი.

Posted by: vano_t 5 Dec 2008, 10:14
basa-ttt
QUOTE
vano_t
დედა,
რა გაცინებს?
გამოვქლიავდით?
აბა ერთი ამას გაეცანით
და მერე იცინეთ...

........

თუ მაიმუნივით ვიცინით ყველაფერზე -
რასაც ჰომეოპათი იტყვის?

მართლა ცუდად იქცევით უკვე რამდენიმე იუზერი.

შენ ხარ უზრდელი. უკვე მერამდენედ გამოხატე უტაქტო შეურაცყოფა. რა შუაშია აქ ჰომეოპათია? რა შუაში ხარ შენ? წაიკითხე საერთოდ რაზე გავიცინეთ? რაიმე შენზე გავიცინე? დასაცინი როცა ხარ (და ხარ) არ მერიდება და პირდაპირ გეუბნები.

ძაან ნუ გაგივა თავზე!

Posted by: roo 5 Dec 2008, 10:23
QUOTE (basa-ttt @ 5 Dec 2008, 09:28 )
vano_t
დედა,
რა გაცინებს?
გამოვქლიავდით?
აბა ერთი ამას გაეცანით
და მერე იცინეთ...


თუ მაიმუნივით ვიცინით ყველაფერზე -
რასაც ჰომეოპათი იტყვის?

მართლა ცუდად იქცევით უკვე რამდენიმე იუზერი.

შემდეგჯერზე ვორნი არ აგცდება ეგეთი პოსტისთვის yes.gif მოდერი არ ვარ მარა დაჟინებით ვიზრუნებ მაგისთვის gigi.gif

Posted by: basa-ttt 5 Dec 2008, 10:32
roo
სხვა რამეზე რატომ არაა ვორნები?
პირველად დაინახეთ ასეთი პოსტი?


vano_t
არაერთხელ მოიქეცი შენც უტაქტოდ.
და ჩავთვალე რომ მორიგი გამოხტომა იყო ზოგადად ჰომეოპათების წინააღმდეგ -
თუ ასე არ იყო -
ბოდიშს გიხდით .
და საერთოდ -
შევეშვათ პირადულს.

Posted by: Blind_Torture_Kill 5 Dec 2008, 13:26
ეს თემაც ფრონტის წინა ხაზად ნუ გადააქციეთ აქ მხოლოდ ქეისები

Posted by: LUKA-BRAZI 5 Dec 2008, 17:07
Blind_Torture_Kill
QUOTE
ეს თემაც ფრონტის წინა ხაზად ნუ გადააქციეთ აქ მხოლოდ ქეისები

მართალია yes.gif რა მოგივიდათ კაცო თქვენ? მაგრამ ახლა ამისთვის აღარ გეცლებათ biggrin.gif ისეთ ქეისს ვდებ, რომ არ ვიცი.... ვინც გამოიცნობს, საღოლ..... უიშვიათესი შემთხვევაა.... ამის გამოცნობა პრესტიჟის საქმეა smile.gif კლინიკას სპეციალურად არ ვდებ. აბა ვინ ხართ მაგრაები? smile.gif

Posted by: basa-ttt 5 Dec 2008, 17:10
QUOTE
კლინიკას სპეციალურად არ ვდებ

ეეჰჰჰ.....
მე კი ღრმად ჩავისუნთქე და მოვემზადე
gigi.gif
"პრეპარატები" 20 წლის წინ ჩაბარდა წარსულს...
cry.gif

Posted by: LUKA-BRAZI 5 Dec 2008, 17:12
basa-ttt
QUOTE
"პრეპარატები" 20 წლის წინ ჩაბარდა წარსულს...

საქმე იმაშია რომ სხვა თითქმის ვერაფერი ვერ ავლენს ამ დაავადებას, კტ-ც კი ძალიან არაინფორმაციულია ხოლმე ამ დროს!
* * *
basa-ttt
ხოლო რაც შეეხება კლინიკას, აქ ზოგიერთი ისეთი პათოგნომური დეტალია, რომ თუ ამ დაავადების შესახებ რაიმე იცი, მაშინვე გამოიცნობ. ვნახოთ, შემოვიდნენ დანარჩენებიც.... და თუ მაინც ვერავინ გამოიცნო მერე დავამატებ ნამიოკებს smile.gif

Posted by: Cousteau 5 Dec 2008, 18:25
LUKA-BRAZI
ის მაინც თქვი რა პრეპარატია თორე ნაღვლის ბუშტი ძლივს გამოვიცანი smile.gif

QUOTE
სეთ ქეისს ვდებ, რომ არ ვიცი.... ვინც გამოიცნობს, საღოლ..... უიშვიათესი შემთხვევაა.... ამის გამოცნობა პრესტიჟის საქმეა

დავიძაბეთ gigi.gif

Posted by: LUKA-BRAZI 5 Dec 2008, 18:43
Cousteau
givi.gif
არ ვხუმრობ, მართლაც მასეა! 1000000-ზე 0.რაღაცა სიხშირითაა, თუმცა ისეთი გამოსავალი აქვს ამ დაავადებას რომ.... !!!! ....
smile.gif

Posted by: Cousteau 5 Dec 2008, 18:45
QUOTE (LUKA-BRAZI @ 5 Dec 2008, 18:43 )
1000000-ზე 0.რაღაცა სიხშირითაა,

არამგონია რო ვიცოდე smile.gif

რა პრეპარატია თქვი რა

Posted by: Tornike Alashvili 5 Dec 2008, 18:48
LUKA-BRAZI

ნიგერის სხეულებია ძაღლის ტვინის პრეპარატში?

Posted by: LUKA-BRAZI 5 Dec 2008, 19:03
Tornike Alashvili
ახლოს ხართ ბატონო თორნიკე! იმ მხრივ რომ პრეპარატზე მართლაც თავის ტვინის ანათალია.... ოღონდ ადამიანის smile.gif თუმცა ბაბეშ-ნეგრის სხეულაკები არაა. სტატისტიკას მიაქციეთ ყურადღება.... მე 10-15 ცუთით გავალა და მოვალ ისევ smile.gif

Posted by: Cousteau 5 Dec 2008, 19:17
არვიცი შეიძლება რამე ნავაროჩენი სიმსივნეა...

QUOTE
საქმე იმაშია რომ სხვა თითქმის ვერაფერი ვერ ავლენს ამ დაავადებას, კტ-ც კი ძალიან არაინფორმაციულია ხოლმე ამ დროს!

ხოლო რაც შეეხება კლინიკას, აქ ზოგიერთი ისეთი პათოგნომური დეტალია


ამ ლოგიკით შეიძლება CNS-ის ენდომეტრიოზი იყოს : /
ნეზნაუ

Posted by: mtvareuli 5 Dec 2008, 19:19
რა ხალხი ხართ რა... ამ თემაში შემოხედვა აღარ მინდა ხოლმე

Posted by: LUKA-BRAZI 5 Dec 2008, 19:31
Cousteau
პრეპარატი არის თავის ტვინის....
mtvareuli
QUOTE
რა ხალხი ხართ რა... ამ თემაში შემოხედვა აღარ მინდა ხოლმე

რეიზა ნთვარე? biggrin.gif ცუდ ქეისებს ვდებთ, თუ ვანოსა და ბასას კამათზე ამბობ?
* * *
Cousteau
ტაც შეეხება CT, რათქმაუნდა რაღაცას ყველა შემთხვევაში ავლენს, მაგრამ სპეციფიური არ არის. ის კი არა ამ დაავადების კტ სურათის ნახვა რომ მინდოდა, google-მაც კი ვერაფერი ღირებული ვერ მონახა, ამიტომ არც დამიდვია biggrin.gif

Posted by: mtvareuli 5 Dec 2008, 19:39
LUKA-BRAZI
QUOTE
ვანოსა და ბასას კამათზე ამბობ?

yes.gif



Posted by: LUKA-BRAZI 5 Dec 2008, 19:40
mtvareuli
ნუ ეგ ჰოოოო.... ცოტა ზედმეტი მოსდით ხოლმე yes.gif

Posted by: mtvareuli 5 Dec 2008, 19:43
basa-ttt
vano_t
Warning:
გაფრთხილება ორივეს. კიდევ  ერთხელ გადაუხვევთ თემას და გადახვალთ შეურაცხყოფებზე და დაივორნებით


სხვებმაც გაითვალისწინეთ

მხოლოდ კითხვები და პასუხები თემის ირგვლივ! და თანმიმდევრულად!!!

Posted by: LUKA-BRAZI 5 Dec 2008, 20:07
ვეთანხმები ნთვარეს! რას ჰგავს ამხანაგებო თქვენი საქციელი?! ვერ ხედათ გეგმა შესასრულებელი, ქეისია გამოსაცნობი, თქვენ კიდევ.....
ესეც copy-paste-ი gigi.gif
QUOTE
მართალია yes.gif რა მოგივიდათ კაცო თქვენ? მაგრამ ახლა ამისთვის აღარ გეცლებათ biggrin.gif ისეთ ქეისს ვდებ, რომ არ ვიცი.... ვინც გამოიცნობს, საღოლ..... უიშვიათესი შემთხვევაა.... ამის გამოცნობა პრესტიჟის საქმეა smile.gif კლინიკას სპეციალურად არ ვდებ. აბა ვინ ხართ მაგრაები? smile.gif

ჰააა, აბა, აღარ არის იდეები? smile.gif

Posted by: BadbadGirl 5 Dec 2008, 20:20
LUKA-BRAZI
QUOTE
ჰააა, აბა, აღარ არის იდეები? smile.gif

ლამაზი სურათია.ჩემ კედელს მოუხდებოდა ეგეთი აბსტაქტული ნაშრომი Ö=

Posted by: basa-ttt 5 Dec 2008, 20:22
LUKA-BRAZI
ეს ვნახე -
და მივამსგავსე...
http://www.binderlab.northwestern.edu/pickbodies.html
პიკის დაავადება -
ტვინის ატროფიით და
დემენციით.

Posted by: mika9 5 Dec 2008, 20:22
LUKA-BRAZI

ჰემოსიდეროზი?

Posted by: LUKA-BRAZI 5 Dec 2008, 20:27
basa-ttt
შენც ახლოს ხარ, მაგრამ პიკის დაავადება არაა, თუმცა კი ჩემი ქეისიც თვინის ატროფიითA და დემენციით მიდის, ამიტომაც ვთქვი კტ და სხვა ინსტრუმენტული გამოკვლევები სპეციფიურს ბევრს არაფერს გვაძლევს მეთქი smile.gif
mika9
QUOTE
ჰემოსიდეროზი?

no, no, dude biggrin.gif ჰემოსიდეროზიც არაა smile.gif

Posted by: Cousteau 5 Dec 2008, 20:47
Lewy Body Disease ?
Hurst Leukoencephalitis ?


დამჭიპე!!! gigi.gif

Posted by: basa-ttt 5 Dec 2008, 20:47
არის კიდევ ლევის სხეულაკები -
ასევე დემენციის დროს...

http://missinglink.ucsf.edu/lm/ids_104_neurodegenerative/Case2/Case2Micro.htm
რავი აბა -
ჰისტოლოგი არ ვარ და....

Posted by: LUKA-BRAZI 5 Dec 2008, 20:52
Cousteau
nischt ! biggrin.gif
basa-ttt
კაცო შენ ხო ვერ გაგაგებინე რა biggrin.gif ერთადერთი საშუალება რაც 100% დიაგნოზსს დაგასმევინებს არის ბიოფსია და მასალის მიკროსკოპირება. გითხარი CT არასპეციფიურია მეთქი, დავდო მაინცდამაინც? smile.gif

Posted by: Cousteau 5 Dec 2008, 20:55
QUOTE (LUKA-BRAZI @ 5 Dec 2008, 20:52 )
Cousteau
nischt ! biggrin.gif

ვიცი რო nicht ეს ყველაზე პიზდეცინტელექტუალური რაც ამოვქაჩე ჩემ თავს ეგ იყო : ) შეიძლება ADEM-ი იყოს ან რავიცი...

ის თქვი თვითონ დაავადებაა თუ რამე (მაგ სისტემური) დაავადების CNS-ის ფორმაა და თავს გაგანებებ
გარდიანი და BTK გვინდა იმათაც თუ არ იციან აგვიხილე თვალები


Posted by: LUKA-BRAZI 5 Dec 2008, 21:03
Cousteau
yes.gif მეც მაგათ ველოდები biggrin.gif ანუ მინდა ყველამ მიიღოს მონაწილეობა.... არადა ეს პრეპარატი ისეთი სპეციფიურია (განსაკუთრებით B), რომ ..... smile.gif ჯერ კიდევ ვიფიქროთ ცოტა smile.gif

Posted by: Tornike Alashvili 5 Dec 2008, 21:04
ვილსონის დაავადებაა და ღვიძლშიც გროვდება არა მარტო ტვინში სპილენძი

user posted image

Posted by: basa-ttt 5 Dec 2008, 21:05
Creutzfeldt-Jakob disease???
იშვიათი დაავადებაა
უი
კურუ?

Kuru - disease -
კანიბალების დაავადება... gigi.gif

Posted by: LUKA-BRAZI 5 Dec 2008, 21:06
Tornike Alashvili
არა ბატონო თორნიკე, ამჯერად არ ხართ მართალი. მიაქციეთ ყურადღება სტატისტიკას? სტატისტიკა და ეს პრეპარატია სპეციფიური.

Posted by: Cousteau 5 Dec 2008, 21:10
. .

Posted by: LUKA-BRAZI 5 Dec 2008, 21:12
basa-ttt
QUOTE
Creutzfeldt-Jakob disease??? იშვიათი დაავადებაა უი კურუ?

Kuru - disease - კანიბალების დაავადება...

რამ გაფიქრებინ ეს დაავადებები? რამემ მიგანიშნა თუ ისე თქვი "ზაპროსტა" ? ცოტა ახსნაც მოაყოლეთ, თორემ იმდენი ჩამოთვალეთ უკვე რომ რომელიღაცა იქნება ბოლოს! biggrin.gif

Posted by: Cousteau 5 Dec 2008, 21:15
QUOTE (LUKA-BRAZI @ 5 Dec 2008, 21:12 )
basa-ttt
QUOTE
Creutzfeldt-Jakob disease??? იშვიათი დაავადებაა უი კურუ?

Kuru - disease - კანიბალების დაავადება...

რამ გაფიქრებინ ეს დაავადებები? რამემ მიგანიშნა თუ ისე თქვი "ზაპროსტა" ? ცოტა ახსნაც მოაყოლეთ, თორემ იმდენი ჩამოთვალეთ უკვე რომ რომელიღაცა იქნება ბოლოს! biggrin.gif

????

ესენია?
ამათ პათოგნომური სიმპტომი რა აქვთ რო? : /
და დიაგნოზიც მარტო მიკროსკოპით არ ისმება

Posted by: LUKA-BRAZI 5 Dec 2008, 21:18
Cousteau
სანამ არ ახსნით, მართალიც რომ იყოს არ ჩაითვლება, თორემ იშვიათი დაავადებების სია არც ისე გრძელია..... ყველას ჩამოთვლა შეიძლება........
QUOTE
ამათ პათოგნომური სიმპტომი რა აქვთ რო? : / და დიაგნოზიც მარტო მიკროსკოპით არ ისმება

ნიშნები აქვთ, და დიაგნოზიც მარტო მიკროსკოპით არ ისმევა. უბრალოდ სხვა მეთოდებს მეორეხარისხოვანი ადგილი უჭირავთ ჰისტოლოგიური გამოკვლევის მერე

Posted by: Cousteau 5 Dec 2008, 21:22
QUOTE (LUKA-BRAZI @ 5 Dec 2008, 21:18 )

QUOTE
ამათ პათოგნომური სიმპტომი რა აქვთ რო? : / და დიაგნოზიც მარტო მიკროსკოპით არ ისმება

ნიშნები აქვთ, და დიაგნოზიც მარტო მიკროსკოპით არ ისმევა. უბრალოდ სხვა მეთოდებს მეორეხარისხოვანი ადგილი უჭირავთ ჰისტოლოგიური გამოკვლევის მერე

QUOTE
Cousteau
სანამ არ ახსნით, მართალიც რომ იყოს არ ჩაითვლება, თორემ იშვიათი დაავადებების სია არც ისე გრძელია..... ყველას ჩამოთვლა შეიძლება........

CJD და Kuru-ს ჰისტოლოგიაზე ვერ ავხსნი 10.000 წელი კიდევ gigi.gif
მარა მაგათი დიაგნოზი ისმევა კლინიკით
და

Electroencephalography — often has characteristic triphasic spikes

Cerebrospinal fluid analysis for 14-3-3 protein

MRI of the brain — often shows high signal intensity in the caudate nucleus and putamen bilaterally on T2-weighted images.

Diffusion Weighted Imaging (DWI) images are the most sensitive

+ ნაღდად არ მეგონა CJD ესეთი იშვიათი თუ იყო user.gif
1 შემთხვევა 1 000 000 ადამიანზე : /



ამით შეიძლებოდა დაახლოებით მიხვედრა

Microscopic "holes" are characteristic in prion-affected tissue sections, causing the tissue to develop a "spongy" architecture

მეკიდე ის მეგონა რაქვია... ''ამას ყურადღებას ნუ მიაქცევთ ''პრეპარატია გამორეცხილი'' '' © ლამარა დეიდა - ლაბორანტი

Posted by: basa-ttt 5 Dec 2008, 21:24
QUOTE
რამ გაფიქრებინ ეს დაავადებები?

დემენციის გამო -
და იშვიათობაო -
თან ჩანართები ტვინშიო -
gigi.gif
ჰოდა მეტი ჰისტოლოგია აღარ მახსოვს, ბალღებო...
თან მაგარი საინტერესო დავადებაა -
ნობელიც კი მისცეს მის აღმომჩენს

Posted by: LUKA-BRAZI 5 Dec 2008, 21:28
Cousteau
მართალი ხარ, აქვე რაღაცას მივამატებდი შენს ნაწერს უფრო სრულყოფილი სურათი რომ შეიქმნას CJD და მერე კი მიხვდებოდი რატომ ჯერ ჰისტოლოგია და მერე კლინიკა, მაგრამ ჯერ ბასა_ტტტ- ს ახსნა მაინტერესებს, არტომ კრუცფელდი და არა სხავ, რამ მიანიშნა, თუ უბრალოდ გაარტყა, მოკლედ როგორც ვაკეთებთ ხოლმე ქეისის განხილვის დროს, პასუხი + ახსნა, ასე რომ ველოდები მის "ახსნა-განმარტებებს" smile.gif

Posted by: Cousteau 5 Dec 2008, 21:33
user posted image

Smart Cannibals dont eat brains!!!


ეეჰ წავედი რამეს წავუმეცადინებ თორე ''შეჭამეს ტვინი'' gigi.gif

Posted by: Blind_Torture_Kill 5 Dec 2008, 21:33
თავს არ დავდებ ნაღდად არ ვიცოდი smile.gif

Posted by: LUKA-BRAZI 5 Dec 2008, 21:34
basa-ttt
მაშინ up.gif ყოჩაღ, მარა ახსნა სრულებითაც არ მომეწონა! დაახლოებით ეს უნდა გეთქვა:
Cousteau
QUOTE
Microscopic "holes" are characteristic in prion-affected tissue sections, causing the tissue to develop a "spongy" architecture

ამიტომაც შედის კრუიცფელდის და კურუც "ღრუბლისებრ ენცეფალოპათიების" ჯგუფში. ამიტომაც გითხარით B სურათს დააკვირდით მეთქი. ესაა სპეციფიური, ხოლო დემენცია და მსგავსი კლინიკა ალცჰაიმერს, ატროფიულ ენცეფალოპათიებს და ა.შ. ახასიათებთ.
Blind_Torture_Kill
უხ! სად ხარ აქამდე შენ კაცო!

Posted by: Blind_Torture_Kill 5 Dec 2008, 22:50
LUKA-BRAZI

რამე კაი ქეისს მოვნახავ და დავბრუნდები sa.gif

Posted by: LUKA-BRAZI 5 Dec 2008, 22:54
Blind_Torture_Kill
ჰო მიდი აბა.... ვანხოთ რას გამოძებნი ამჯერად biggrin.gif

Posted by: Blind_Torture_Kill 5 Dec 2008, 23:57
70 კგ-ანი ჯანმრთელი მამაკაცი კარგავს 2 ლიტრა სიხთეს ოფლის სახით ფიზიკური მუშაობის პროცესში.ამის შემდეგ იგი სვავს 2 ლიტრა წყალს რათა აანაზღაუროს დანაკარგი.რა ცვლილებებია მოსალოდნელი ?

ა. გაიზრდება უჯრედსგარე სითხის ოსმოლალობა
ბ. გაიზრდება უჯრედსგარე სითხის მოცულობა
გ. გაიზრდება უჯრედშიდა სითხის ოსმოლალობა
დ. გაიზრდება უჯრედშიდა სითხის მოცულობა
ე. გაიზრდება Na-ის კონცენტრაცია პლაზმაში

მეორე კურსის მასალაა biggrin.gif

ხო დამავიწყდა - ამას ახსნის გარეშე ფასი არააქვს


მიკვირს ამდენი ხანი აცოცხლეთ ეს კითხვა war.gif

Posted by: LUKA-BRAZI 6 Dec 2008, 12:39
Blind_Torture_Kill
QUOTE
მიკვირს ამდენი ხანი აცოცხლეთ ეს კითხვა

biggrin.gif ეხლავე მოვუღებთ ბოლს biggrin.gif
მოკლედ პასუხია 'დ' და რატომ: ოფლი წაყალთან ერთად ხომ შეიცავ იონებს? ე.ი. ოფლის დაკარგვასთან ერთად იკარგება იონები და რაღაც მოცულობითი სითხეც (2 ლ) ინტერსტიციალური სივრციდან, ხოლო წყლის მიღების შემდეგ (რომელსაც ოსმოლარობა არ გააჩნია იდეაში smile.gif ) მოხდება მხოლოდ დაკარგულის სითხის მოცულობის შევსება იონების გარეშე, რაც გამოიწვევს უჯრედშორისი სითხის ოსმოლარობის დაქვეითებას და სითხე დაგავა უჯრედში, ანუ დაბალი ოსმოლარული სივრციდან მაღალში, გრადიენტის მიმართულებით.
ჰა ასე თუ არა? biggrin.gif

Posted by: Blind_Torture_Kill 6 Dec 2008, 12:53
LUKA-BRAZI

up.gif
ფიზიოლოგია უმაგრესი საგანია და ტაკ დერჟაც
საღოლ

Posted by: LUKA-BRAZI 6 Dec 2008, 13:40
Blind_Torture_Kill
დანქე wink.gif მარა რამე ჯიგრული ქეისი ხიე რა, ერთი 2-3 გვერდი რომ შევავსოთ გამოცნობაში biggrin.gif
P.S. ეხლა რომ გადავიკითხე ჩემი წინა პოსტი, ცოტა "ნიტო" გამომსვლია, გრადიენტში ვიგულისხმე ის რომ სითხე შევა უჯრედში რათა განაზავოს უჯრედშიდა იონების მაღალი კონცენტრაცია.... შენ კი მიხვდი რაც ვიგულისხმე, მარა სხვა შეიძლება ვერ მიხვდეს, ცოტა ბუნდოვნად გადმოვეცი ნათქვამი ...

Posted by: Blind_Torture_Kill 6 Dec 2008, 19:43
QUOTE
დანქე  მარა რამე ჯიგრული ქეისი ხიე რა, ერთი 2-3 გვერდი რომ შევავსოთ გამოცნობაში 
P.S. ეხლა რომ გადავიკითხე ჩემი წინა პოსტი, ცოტა "ნიტო" გამომსვლია, გრადიენტში ვიგულისხმე ის რომ სითხე შევა უჯრედში რათა განაზავოს უჯრედშიდა იონების მაღალი კონცენტრაცია.... შენ კი მიხვდი რაც ვიგულისხმე, მარა სხვა შეიძლება ვერ მიხვდეს, ცოტა ბუნდოვნად გადმოვეცი ნათქვამი ...


კაი პასუხი იყო

რა არის ეს

Posted by: basa-ttt 6 Dec 2008, 19:56
სტიმულატორი უდგას?...............

რაღაც შუასაყარიც გაგანიერებული ჩანს და მარცხენა ფილტვი ზომებში შემცირებული.. კლინიკა რაა?

Posted by: Blind_Torture_Kill 6 Dec 2008, 20:16
QUOTE
სტიმულატორი უდგას?...............

რაღაც შუასაყარიც გაგანიერებული ჩანს და მარცხენა ფილტვი ზომებში შემცირებული.. კლინიკა რაა?


ამაზე მეტი რაღა გინდა smile.gif

რა არის და რა ჭირს ამ ადამიანს

Posted by: basa-ttt 6 Dec 2008, 20:21
QUOTE
ამაზე მეტი რაღა გინდა

პნევმოთორაქსია?

Posted by: mtvareuli 6 Dec 2008, 20:30
basa-ttt
QUOTE
პნევმოთორაქსია?

პნევმოთორაქსის დროს პათოლოგიური განათებაა და არა ასეთი სასტიკი დაჩრდილვა

Posted by: basa-ttt 6 Dec 2008, 20:37
QUOTE
პნევმოთორაქსის დროს პათოლოგიური განათებაა და არა ასეთი სასტიკი დაჩრდილვა

სინუსები ჩანს მარცხნივ, პლევრის ემპიემაზე ან სველ პლევრიტზე რომ იფიქრო..

მარჯვენა ფილტვშიც შვარტაა მგონი...

არა რა,
კლინიკის გარეშე - მე პას....

gigi.gif

Posted by: Blind_Torture_Kill 6 Dec 2008, 20:41
basa-ttt

კლინიკას რომ არ ვდებ იმას ნიშნავს რომ მაგაზე კარგ სურათს ვდებ smile.gif

პაციენტს აქვს დისპნეა biggrin.gif

Posted by: LUKA-BRAZI 6 Dec 2008, 22:28
Blind_Torture_Kill
გულ-ფილტვის უკმარისობა? გულისმიერი ფილტვი?

Posted by: Blind_Torture_Kill 6 Dec 2008, 22:51
LUKA-BRAZI
QUOTE
გულისმიერი ფილტვი?


მეორე სურათზე რას ხედავ მიპასუხე და 5 იანი

Posted by: LUKA-BRAZI 6 Dec 2008, 22:55
Blind_Torture_Kill
QUOTE
მეორე სურათზე რას ხედავ მიპასუხე და 5 იანი

biggrin.gif
დარწმუნებული არ ვარ მაგრამ ალვეოლებში უნდა იყოს ტრანსუდანტი + ერითროციტები, ანუ პლაზმე + ცოტა სისხლი, ის ყავისფერები მგონი ჰემოგლობინია ხო?
smile.gif
მგონი გავმაზე? biggrin.gif

Posted by: Blind_Torture_Kill 6 Dec 2008, 23:00
LUKA-BRAZI

QUOTE
დარწმუნებული არ ვარ მაგრამ ალვეოლებში უნდა იყოს ტრანსუდანტი + ერითროციტები, ანუ პლაზმე + ცოტა სისხლი, ის ყავისფერები მგონი ჰემოგლობინია ხო? მგონი გავმაზე? 


გამოიცანი რომ ფილტვებში შეგუბება გაქ მარა მეორე სურათი სწორედ ვერ განმიმარტე

Posted by: LUKA-BRAZI 6 Dec 2008, 23:08
Blind_Torture_Kill
ჰო ვერ ვხვდები.... ფაქტია შიგ რაღაც არის.... უჯრედები + ალბათ შემაერთებელი ქსოვილიც, მოკლედ ინფილტრაციაა, მარა რა არის ის ყავისფერები? მე დაჟანგული ჰემოგლობინი მეგონა..... ან ჰემოსიდერინი.... მარა არა ხო? smile.gif

Posted by: Blind_Torture_Kill 6 Dec 2008, 23:12

QUOTE
მე დაჟანგული ჰემოგლობინი მეგონა..... ან ჰემოსიდერინი.... მარა არა ხო?


ეს სხვა საკითხია smile.gif
yes.gif

მოკლედ რა ჭირს

Posted by: LUKA-BRAZI 6 Dec 2008, 23:37
Blind_Torture_Kill
QUOTE
მოკლედ რა ჭირს

თუ ორივე სურათს მივიღებთ მხედველობაში, ალბათ გულის ქრონიკული უკმარისობის გამწვავებაა სახეზე, ანუ კარდიული ასთმა, რაც რენტგენზეც ჩანს და ალვეოლებში ტრანსუდაციითაც..... ალბათ ამას მეკითხები ხომ?
ჰო რენტგენზე მარცხენა პარკუჭი რაღაც დილატირებულს გავს + ამას ეტყობა პაცენტი მონიტორზეა შეერთებული, ე.ი. რეანიმაციაში წევს + ამას თუ დისპნოეც აქვს და ტრანსუდატიც, გამოდის რომ კარდიული ასთმა ქონია.
ვოტ ტაკ! smile.gif

Posted by: Blind_Torture_Kill 6 Dec 2008, 23:41
LUKA-BRAZI

QUOTE
გულის ქრონიკული უკმარისობის გამწვავებაა სახეზე

კარგია smile.gif

მეორე სურაში ჰემოსიდერინით გაძეგილი მაკროფაგები (heart failure cells) არიან

Posted by: LUKA-BRAZI 6 Dec 2008, 23:48
Blind_Torture_Kill
QUOTE
კარგია smile.gif

მეორე სურაში ჰემოსიდერინით გაძეგილი მაკროფაგები (heart failure cells) არიან 

ჰო, ჰო მაკროფაგები, რომლებიც ჰემოსიდერინს მიირთმევენ biggrin.gif forgot biggrin.gif დავდო ქეისი თუ გვეყო დღეს? თუმცა ბევრი ქეისი არც განგვიხილiა biggrin.gif

Posted by: Blind_Torture_Kill 6 Dec 2008, 23:51
LUKA-BRAZI

დადე მიდი მე ერთს ვამზადებ მანამდე


რა დაავადებაა ?

Posted by: LUKA-BRAZI 6 Dec 2008, 23:58
Blind_Torture_Kill
ეს მგონი იყო ადრე, თუ რაღაც ამის მსგავსი spy.gif მარა პასუხი არ მახსოვს biggrin.gif მგონი ნორმალური სისხლის ნაცხია.... რას მაბნევ შენა კაცო, ა? gigi.gif

Posted by: Blind_Torture_Kill 7 Dec 2008, 00:04
LUKA-BRAZI

ნორმა არაა
და აბა რატო არაა შენთვის მომინდია მაგის გამოცნობა

Posted by: mtvareuli 7 Dec 2008, 00:13
Blind_Torture_Kill

http://forum.ge/?showtopic=33720157&view=findpost&p=11465367

Posted by: LUKA-BRAZI 7 Dec 2008, 00:15
ხო ვთქვი იყო უკვე მეთქი biggrin.gif
mtvareuli
up.gif

Posted by: Blind_Torture_Kill 7 Dec 2008, 00:25
მე მაპატიეთ
არ ვიცოდი

მაშინ რამე ახალს ვნახავ

Posted by: vano_t 7 Dec 2008, 00:30
Blind_Torture_Kill
QUOTE
ნორმა არაა
და აბა რატო არაა შენთვის მომინდია მაგის გამოცნობა

ჰიპოსეგმენტირებული ნეიტროფილებია. ნეიტროფილს 3-4 სეგმენტი უნდა ქონდეს. მაგ უჯრედს ქვია Pseudo Pelger Huet cells. შეიძლება გაჩნდეს CML-ის, რკინადეფიციტური ანემიის, მიელოდისპლაზიური სინდრომის დროს. შეიძლება სხვა დაავადებების დროსაც გაჩნდეს.


********

შენს რენტგენზე გულის უკმარისობის გამწვავება არ არის სწორი პასუხი. სწორი პასუხია ორმხრივი დიფუზური ინფილტრატები. ეს კიდევ შეიძლება გაჩნდეს ფილტვების ორმხრივი შეშუპების დროს ნებისმიერი მიზეზით (ამაში შევა ფილტვების ედემა გულის უკმარისობის დროს, მწვავე რესპირატორული დისტრესს სინდრომის დროს).

*********

მაგ ავადმყოფს სტიმულატორი არ აქვს. ეგ მრგვალი რაღაც არის ეკგ მონიტორის ერთ-ერთი განხრის ელექტროდი. ეგ ავამდყოფი ინტენსიური თერაპიის ავადმყოფია (ინტუბირებულია და ნაზოგასტრული ზონდიც აქვს). სტუმულაოტირ უფრო მაღლა ისმება. სხვანაირი ფორმა აქვს და მისგან გამოსული მავთულები გულში მიდიან.

Posted by: Blind_Torture_Kill 7 Dec 2008, 00:36
QUOTE
შენს რენტგენზე გულის უკმარისობის გამწვავება არ არის სწორი პასუხი. სწორი პასუხია ორმხრივი დიფუზური ინფილტრატები. ეს კიდევ შეიძლება გაჩნდეს ფილტვების ორმხრივი შეშუპების დროს ნებისმიერი მიზეზით (ამაში შევა ფილტვების ედემა გულის უკმარისობის დროს, მწვავე რესპირატორული დისტრესს სინდრომის დროს).


baby.gif

გულის უკმარისობა აქვს და ჰემოსიდერინით მაკროფაგები
ეხლა გაუმწვავდა თუ გუშინწინისწინ ეგ არ მაინტერესებდა და არც ის თერაპიული პაციენტია თუ ინტენსიურის wink.gif

Posted by: vano_t 7 Dec 2008, 00:41
QUOTE (Blind_Torture_Kill @ 7 Dec 2008, 00:36 )
გულის უკმარისობა აქვს და ჰემოსიდერინით მაკროფაგები
ეხლა გაუმწვავდა თუ გუშინწინისწინ ეგ არ მაინტერესებდა და არც ის თერაპიული პაციენტია თუ ინტენსიურის wink.gif

რენტეგენი არ უჩვენებს გულის უკმარისობას! რენტგენით მაგ დიაგნოზი არ ხდება!

მიკროპრეპარატზე რა არის, მაგას არ აქვს მნიშვნელობა. შენ რენტგენით მოითხოვე დიაგნოზი. ასეა ეს და შენ გინდ დაიჯერე და გინდა არა. ეგ შენზეა დამოკიდებული.

Posted by: LUKA-BRAZI 7 Dec 2008, 00:47
vano_t
QUOTE
რენტეგენი არ უჩვენებს გულის უკმარისობას! რენტგენით მაგ დიაგნოზი არ ხდება!

რეიზა? დილატირებული მარცხენა პარკუჭი არაპირდაპირ მიუთითებს შესაძლო უკმარისობაზე

Posted by: Blind_Torture_Kill 7 Dec 2008, 00:47
vano_t

QUOTE
რენტეგენი არ უჩვენებს გულის უკმარისობას! რენტგენით მაგ დიაგნოზი არ ხდება!

მიკროპრეპარატზე რა არის, მაგას არ აქვს მნიშვნელობა. შენ რენტგენით მოითხოვე დიაგნოზი. ასეა ეს და შენ გინდ დაიჯერე და გინდა არა. ეგ შენზეა დამოკიდებული.


Ro გრამით არ მომითხოვია დიაგნოზი
მაშინ მიკროპრეპარატს არ დავდებდი
ხოდა მთელი მუღამი იმაში იყო რომ მაგ მაკროფაგებით ეპასუხა ხალხს
უბრალოდ ორივე სურათი ერთად ვერ დავდე

Posted by: vano_t 7 Dec 2008, 00:49
რა განსხვავვებაა შენს სურათსა და ამ სურათს შორის?
user posted image

ან რა განსხვავებაა შენს სურათსა და ამ სურათს შორის?
user posted image

არაფერი განსხვავება არ არის. ყველა ეს აჩვენებს დიფუზურ ორმხრივ ინფილტრატებს. პირველ სურათზე გაქვს ფილტვების ედემიანი ავადმყოფი (მიზეზი არ არის დასახელებული) და მეორე სურათზე გაქვს ავადმყოფი ARDS-ით.

ფილტვების ედემის კი გამოიწვეს ბევრი რამ, ამათი ჩათვლით:
Ethiology

The most likely causes of Acute Pulmonary Edema are cardiacs ones. The identification of primary cause can be lifesaving. Bellow are listed the main causes for pulmonary edema.

I. Altered capillary permeability

A. Infectious pulmonary edema (viral or bacterial)

B. Inhaled toxins

C. Circulating toxins

D. Vasoactive substances (histamine, kinins)

E. Disseminated intravascular coagulation

F. Immunologic reactions

G. Radiation pneumonia

H. Uremia

I. Near-drowning

J. Aspiration pneumonia

K. Smoke inhalation

L. Adult respiratory distress syndrome

II. Increased pulmonary capillary pressure

A. Cardiac causes

1. Left ventricular failure from any cause

2. Mitral stenosis

3. Subacute bacterial endocarditis

B. Noncardiac causes

1. Pulmonary venous fibrosis

2. Congenital stenosis of the origin of the pulmonary veins

3. Pulmonary venoocclusive disease

C. Overinfusion of fluids

III. Decreased oncotic pressure

A. Hypoalbuminemia from any cause (renal, hepatic, nutritional, or protein-losing enteropathy)

IV. Lymphatic insufficiency

V. Mixed or unknown mechanisms

A. High-altitude pulmonary edema

B. Neurogenic pulmonary edema (CNS trauma, subarachnoid bleeding)

C. Heroin overdose (also other narcotics)

D. Pulmonary embolism (very rare)

E. Pulmonary parenchymal disease

F. Eclampsia

G. Cardioversion

H. Postanesthetic

I. Cardiopulmonary bypass

თითოეული ეს დიაგნოზი შეიძლება იყოს მასეთი სურათის მიზეზი!

Posted by: Blind_Torture_Kill 7 Dec 2008, 00:56
QUOTE
The most likely causes of Acute Pulmonary Edema are cardiacs ones. The identification of primary cause can be lifesaving. Bellow are listed the main causes for pulmonary edema.

I. Altered capillary permeability

A. Infectious pulmonary edema (viral or bacterial)

B. Inhaled toxins

C. Circulating toxins

D. Vasoactive substances (histamine, kinins)

E. Disseminated intravascular coagulation

F. Immunologic reactions

G. Radiation pneumonia

H. Uremia

I. Near-drowning

J. Aspiration pneumonia

K. Smoke inhalation

L. Adult respiratory distress syndrome

II. Increased pulmonary capillary pressure

A. Cardiac causes

1. Left ventricular failure from any cause

2. Mitral stenosis

3. Subacute bacterial endocarditis

B. Noncardiac causes

1. Pulmonary venous fibrosis

2. Congenital stenosis of the origin of the pulmonary veins

3. Pulmonary venoocclusive disease

C. Overinfusion of fluids

III. Decreased oncotic pressure

A. Hypoalbuminemia from any cause (renal, hepatic, nutritional, or protein-losing enteropathy)

IV. Lymphatic insufficiency

V. Mixed or unknown mechanisms

A. High-altitude pulmonary edema

B. Neurogenic pulmonary edema (CNS trauma, subarachnoid bleeding)

C. Heroin overdose (also other narcotics)

D. Pulmonary embolism (very rare)

E. Pulmonary parenchymal disease

F. Eclampsia

G. Cardioversion

H. Postanesthetic

I. Cardiopulmonary bypass

თითოეული ეს დიაგნოზი შეიძლება იყოს მასეთი სურათის მიზეზი!


რამე სპეციფიური რომ ყოფილიყო მივანიშნებდი რომ სეფსისი ქონდა ან ხ.ი.ზ კიდე რამე




Posted by: vano_t 7 Dec 2008, 01:01
LUKA-BRAZI
QUOTE
რეიზა? დილატირებული მარცხენა პარკუჭი არაპირდაპირ მიუთითებს შესაძლო უკმარისობაზე

რაც იმას ნიშნავს, რომ დილატირებული პარკუჭი შეიძლება იყოს (უფრო სწორად გულის ზომა იყოს გადიდებული-მარცხენა პარკუჭის გამოა თუ არა, რენტგენზე ზუსტად ვერ გაარკვევ) მასეთ სურათთან ერთად და მაგ სურათიზ მიზეზი იყოს სხვა. ან პირიქით, გულის ზომა არ იყოს გადიდებული და და მასეთი სურათი გქონდეს. მოკლედ, მაგ ნიშანი არაფერში დაგეხმარება. კლინიკა დაგეხმარება ბევრად.

რაც შეეხება ამ სურათს, აქ გულის ზომა არ არის გადიდებული. cardiac enlargement-ის რენტგენოლოგიური კრიტერიუმი არის ის, რომ გულის ყველაზე დიდი ზომა გულმკერდის ყველაზე დიდ ზომის ნახევარს უნდა აღემატებოდეს. ამ სურათზე ასე არ არის. გულის ზომა ნორმალური ჩანს.

Blind_Torture_Kill
QUOTE
რამე სპეციფიური რომ ყოფილიყო მივანიშნებდი რომ სეფსისი ქონდა ან ხ.ი.ზ კიდე რამე
არაფერით არ მიგინიშნებია ან ერთი, ან მეორე. მარტო მიუთითე, რომ ავადმყოფს დისპნეა აქვს biggrin.gif რასაც მითითება არ ჭირდება ასეთი რენტგენის დროს. მიკროპრეპარტზე ვერ გეტვყი რა ხდება. მაგასაც გადავხედავ და ვნახავ მასეთი უჯრედი პათოგმონურია მწვავე უკმარისობის თუ ქრონიკულის.

Posted by: Blind_Torture_Kill 7 Dec 2008, 01:05
QUOTE
არაფერით არ მიგინიშნებია ან ერთი, ან მეორე. მარტო მიუთითე, რომ ავადმყოფს დისპნეა აქვსრასაც მითითება არ ჭირდება ასეთი რენტგენის დროს. მიკროპრეპარტზე ვერ გეტვყი რა ხდება. მაგასაც გადავხედავ და ვნახავ მასეთი უჯრედი პათოგმონურია მწვავე უკმარისობის თუ ქრონიკულის.


მიკროპრეპარატი დავდე მაგისთვის

Posted by: LUKA-BRAZI 7 Dec 2008, 01:18
არაპრინციპულ რამეზე ვკამათობთ... საქმე იმაშია რომ რახან მარტო რენტგენით ეგ დაიგნოზი არ ისმევა, ამიტომაც დადო პრეპარატის სურათი....

Posted by: basa-ttt 7 Dec 2008, 01:22
QUOTE
არაპრინციპულ რამეზე ვკამათობთ... საქმე იმაშია რომ რახან მარტო რენტგენით ეგ დაიგნოზი არ ისმევა, ამიტომაც დადო პრეპარატის სურათი....

QUOTE
რა არის ეს?, კლინიკური შემთხვევა, N 2.

კლინიკა დავდოთ ხოლმე.
ასე ჯობია.

Posted by: vano_t 7 Dec 2008, 01:24
ჰემოსიდერინით დატვირთული მაკროფაგები სულაც არ ყოფილა გულის უკმარისობით გამოწვეული ფილტვების მწვავე შეშუპების პათოგნომური. ასეთ მაკროფაგების წარმოქმნა ხდება ნებისმიერ დროს, როცა ალვეოლარურ სივრცეში ხდება ერითროციტის მოხვერდა. ამას გამოიწვევს კიდევ სხვადასხვა რამ. მათ შორის გულის ქრონიკული უკმარისობის სხვადასხვა მიზეზი. ამ დროს მარცხენა პარკუჭის ფუნქცია შეიძლება საერთოდ ნორმალური გქონდეს. მაგალითად მიტრალური სტენოზის დროს. ასეთი მაკროფაგები გვხდება ფილტვების ჰემოსიდეროზის დროს, რომელსაც თავის მხრივ თავისი დიფერენციალური დიაგნოზი აქვს. აქ შედის იდიოპათური ფორმა და მეორადი ფორმები (მაგალითად მიტრალური სარქვლის უკმარისობის დროს ან გუდპასჩერის დროს). სხვა მიზიზები emedicine.com ვებსიატის მიხედვით:
QUOTE
Causes

    * Primary PH
    *
          o IPH - The most common cause of pulmonary hemosiderosis in childhood
          o Heiner syndrome - Hypersensitivity to proteins from cow's milk
          o Goodpasture syndrome - Anti-GBM antibody–mediated hemosiderosis
    * Secondary PH
    *
          o Congenital or acquired cardiopulmonary abnormalities - Bronchogenic cyst, pulmonary sequestration, congenital arteriovenous fistula, tetralogy of Fallot, Eisenmenger complex, mitral valve stenosis, pulmonic valve stenosis, congenital pulmonary vein stenosis, pulmonary arterial stenosis, pulmonary embolism, left ventricular failure
          o Infections and their complications - Bacterial pneumonia, sepsis (disseminated intravascular coagulation [DIC]), pulmonary abscess, bronchiectasis, and bronchiolitis obliterans
          o Immunologically mediated diseases - SLE, periarteritis nodosa, Wegener granulomatosis, Henoch-Schönlein purpura, immune complex–mediated glomerulonephritis, allergic bronchopulmonary aspergillosis
          o Neoplasms - Primary bronchial tumors (adenoma, carcinoid, sarcoma, hemangioma, angioma) or metastatic lesions (sarcoma, Wilms tumor, osteogenic sarcoma)
          o Drugs - Penicillamine, cocaine
          o Toxins - Pesticide substances (synthetic peritroids)
          o Environmental molds - S atra, Memnoniella echinata
          o Miscellaneous causes - Retained foreign body, pulmonary trauma, pulmonary alveolar proteinosis, congenital hyperammonemia


სხვა ლინკები:
http://radiographics.rsnajnls.org/cgi/content-nw/full/19/4/965/F6
http://linkinghub.elsevier.com/retrieve/pii/S0002914904008057
http://arpa.allenpress.com/arpaonline/?request=get-document&doi=10.1043%2F1543-2165(2006)130%5B1684:SASOHM%5D2.0.CO%3B2

Posted by: LUKA-BRAZI 7 Dec 2008, 01:29
vano_t
ანუ ქეისის დადების დროს უეჭველი პათოგნომური მასალა უნდა დაიდოს? ჰო მარა, კასიკური კლინიკით ხშირად არ მიმდინარეობს დაავადება....

Posted by: Blind_Torture_Kill 7 Dec 2008, 01:33
vano_t

გასაგებია რომ სხვებსაც ახასიათებს

რა ქვია მაგ უჯრედებს ?
ხშირად გვხვდება მარცხენა პარკუჭის უკმარისობისას და მაგიტომ დავდე

რამე სხვა რომ ყოფილიყო მაგ.მიტრალური სტენოზი დავწერდი მარცხენა წინაგული დილატირებული აქვს და ეკგსაც დავდებდი წინაგულების ფიბრილაციით

იტოკში ყველა ქეისზე რამე მახასიათებელს ვდებ და ნუ მიჩივით

Posted by: vano_t 7 Dec 2008, 01:41
LUKA-BRAZI
QUOTE
არაპრინციპულ რამეზე ვკამათობთ... საქმე იმაშია რომ რახან მარტო რენტგენით ეგ დაიგნოზი არ ისმევა, ამიტომაც დადო პრეპარატის სურათი....

არაპრინციპულზე არ ვკმათობ. პრინციპულ საკითხზე ვკამათობთ. ჯერ ერთი, რად უნდა გულის მწვავე უკმარისობით გამოწვეული ფიტლვების შეშუპების დიაგნოზს ბიოფსია? კლნიკური სურათი რენტგენთან ერთად საკმარისია. რენტგენი, თავისთავად, ვერ მოგცემს დიაგნოზს, გარდა იმისა, რომ მიუთითებს დიფუზურ ორმხრივ ინფილტრატზე. რენტგენოლოგს ასეთ სურათს რომ უგზავნი (და საერთოდ ნებიმსიერ სურათს) უნდა მიაყოლომ მცირე კლინიკა. ყველა რენტგენოლოგი კითხულობს კლინიკას!

მეორეც, როგორც ჩანს ჰემოსიდერინით დატვირთული მაკროფაგები გხვდება ბევრი რამის დროს და არა ფილტვების მწვავე ედემის დროს მარტო. უფრო მეტიც, რომ დავგუგლე ორივე ერთად (heart failure and hemosiderosis), ბევრი ლინკი არ გამოვიდა რაც მაგას მიუთითებს. ერთი რაც გამოვიდე დავდე.

http://arpa.allenpress.com/arpaonline/?request=get-document&doi=10.1043%2F1543-2165(2006)130%5B1684:SASOHM%5D2.0.CO%3B2 ჰემოსიდერინით დატვირთული ალვეოლარური მაკროფაგების სენსიტიურობა და სპეციფიურობა პედიატრიულ პოპულაციაში. აქ ძირითადად გვხდება პირველადი ჰემოსიდეროზი და ეს ტესტი საკმაოდ სპეციფიურიაო. რაც იმას ნიშნავს, რომ საკმაოდ არასპეციფიურია გულის უკმარისობის დროს.

მოკლედ მაგ რენტგენით ხომ არ შეიძლება და არ შეიძლება დიაგნოზის დასმა, და არც ეგ მიკროპრეპარატი არის დიაგნოზის მიმთითებელი.

აგერ სურათი:user posted image
კითხე ნებისმიერ რენტგენოლოგს ამის დიაგნოზი. თუ ჭკვიანი რენტგენოლოგია, მაშინ ჩამოგითვლის რამოდენიმე ვარიანტს. ამ სურათზე ავადმყოფს დაჟე მწვავე პრეზენტაციაც არ აქვს. ქრონიკული ავადმყოფია მიტრალური სარქველის სტენოზით. შენი აზრით, ამას რომ გაუკეთო ბიოფსია რას ნახავ? უამრავ მაკროფაგს, რომელიც ჰემოსიდერინით არის დატვირთული. არ უნდა ამას კამათი.

http://radiographics.rsnajnls.org/cgi/content-nw/full/19/4/965/F6 თამხლები გვერდი.

Posted by: LUKA-BRAZI 7 Dec 2008, 01:50
vano_t
სად ნახე ამხელა ფორმატის სურათი lol.gif
არა, რასაც შენ ამბობ ყველაფერი გასაგებია, მარა მე ვერ გავიგე ის, რომ რა თორჩა_ქილმა არასწორად დადო ქეისი?
ანუ გულის უკმარისობის ქეისისთვის შეუფერებელი მასალა დადო?

Posted by: Blind_Torture_Kill 7 Dec 2008, 01:53
vano_t

heart failure cells ესე ეძახიან მაგას

უნდა დამეწერა
ავადმყოფს აწუხებდა გულითქო (მოკვდა პაციენტი)
3 კიბეზე რომ ავიდოდა დასვენება ჭირდებოდა
ერთ დღესაც ძლიერმა ხველამ გამოაღვიძა და საავადმოფოში მიყვანამდე გარდაიცვალა
აუტოფსიით ინახა დილატირებული მარცხენა პარკუჭი ხოლო ფილტვის მიკროპრეპარატი წარმოდგენილიამეთქი ქვემოთ

Ro გაკეთებულია წინა ვიზიტზე


Posted by: basa-ttt 7 Dec 2008, 01:57
QUOTE
Blind_Torture_Kill

ეს რას ნიშნავს?

Posted by: LUKA-BRAZI 7 Dec 2008, 01:57
Blind_Torture_Kill
ბარემ დიაგნოზიც მიგეწერა lol.gif

Posted by: vano_t 7 Dec 2008, 01:59
LUKA-BRAZI
QUOTE
არა, რასაც შენ ამბობ ყველაფერი გასაგებია, მარა მე ვერ გავიგე ის, რომ რა თორჩა_ქილმა არასწორად დადო ქეისი?
ანუ გულის უკმარისობის ქეისისთვის შეუფერებელი მასალა დადო?

გულის უკმარისობისათვის შეუფერებელი მასალა არ დადო. არამედ უკმარისი მასალა დადო biggrin.gif მაგის მიხედვით დიაგნოზი არ დაისმება. ამიტომ, ასეთ რამეებს რომ დებ, ან კლინიკა უნდა მიაყოლო, ან უბრალოდ რენტგენის (და/ან მიკროფოტოგრაფიის) აღწერა და დიფერენციალური დიაგნოზი მოითხოვო.

მის მიერ დადებული კეისი თუ გულის უკმარისობაა, ეს იმიტომ კი არ არის რომ პათოგნომური და გასაგები კეისია, არამედ გულის უკმარისობის კეისი ნახა, სადაც სხვა მიზეზებით იცოდნენ რომ გულის უკმარისობა იყო. აი ამაშია საქმე.

ამ შემთხვევაში თუ არ გინდა კლინიკის მითითება (და კლინიკაც შეიძლება ხანდახან მსგავსი ქონდეს სხვადასხვა რამეს), მაშინ შეიძლება მიუთითო, რომ დავუშვათ ყველაზე ხშირი მიზეზი რა არის ამისაო და რაჩაც მსგავსი.

Posted by: Blind_Torture_Kill 7 Dec 2008, 01:59
LUKA-BRAZI

lol.gif lol.gif lol.gif

QUOTE
ბარემ დიაგნოზიც მიგეწერა


ამის მერე გავითვალისწინებ

QUOTE
ეს რას ნიშნავს?


დააბრმავე აწამე მოკალი

Posted by: basa-ttt 7 Dec 2008, 02:00
QUOTE
დააბრმავე აწამე მოკალი

და ამას წერს ექიმი?

gigi.gif

სად ხარ წიე ჰიპოკრატე?
gigi.gif

Posted by: LUKA-BRAZI 7 Dec 2008, 02:02
QUOTE
Blind_Torture_Kill

ჰო ეს მეც მაინტერესებს, მართლაც რას ნიშნავს? ექსპერტის უხშირეს შემთხვევებს?
smile.gif

Posted by: vano_t 7 Dec 2008, 02:03
QUOTE (Blind_Torture_Kill @ 7 Dec 2008, 01:53 )
vano_t

heart failure cells ესე ეძახიან მაგას

უნდა დამეწერა
ავადმყოფს აწუხებდა გულითქო (მოკვდა პაციენტი)
3 კიბეზე რომ ავიდოდა დასვენება ჭირდებოდა
ერთ დღესაც ძლიერმა ხველამ გამოაღვიძა და საავადმოფოში მიყვანამდე გარდაიცვალა
აუტოფსიით ინახა დილატირებული მარცხენა პარკუჭი ხოლო ფილტვის მიკროპრეპარატი წარმოდგენილიამეთქი ქვემოთ

Ro გაკეთებულია წინა ვიზიტზე

user posted image
აბა გამოიცანი ვისი მშობელია.

Posted by: Blind_Torture_Kill 7 Dec 2008, 02:04
basa-ttt

ექიმი არ ვარ user.gif

ეს ადრე გავაკეთე გაჭედილი მქონდა მაშინ
ეხლა დავწყნარდი მარა ნიკის გამოცვლა არ ვიცი თორე გამოვცვლიდი

Posted by: LUKA-BRAZI 7 Dec 2008, 02:07
Blind_Torture_Kill
მიმართე ტიმს smile.gif მე მეხმარება ხომე რაღაც რაღაცეებში, შენც დაგეხმარება smile.gif

Posted by: Blind_Torture_Kill 7 Dec 2008, 02:12
QUOTE
აბა გამოიცანი ვისი მშობელია.

gigi.gif gigi.gif

აბა ეხლა დავაგდებ ერთ ქეისს და გამოიცანით
კიდევ სლაიდები იქნება ოღონდ

http://imageshack.us


დიაგნოზი

Posted by: LUKA-BRAZI 7 Dec 2008, 02:13
მიდი ოღონდ მალე, თორემ თავს ვეღარ ვიჭერ ისე მეძინება biggrin.gif
* * *
Blind_Torture_Kill
ღვიძლის ციროზი პორტული ჰიპერტენზიით?

Posted by: Blind_Torture_Kill 7 Dec 2008, 02:20
LUKA-BRAZI

lol.gif lol.gif lol.gif lol.gif როგორ ფიქრობ ამხელა შესავალი მაგისთვის გავაკეთე ?

Posted by: vano_t 7 Dec 2008, 02:21
QUOTE (Blind_Torture_Kill @ 7 Dec 2008, 02:12 )
QUOTE
აბა გამოიცანი ვისი მშობელია.

gigi.gif gigi.gif

აბა ეხლა დავაგდებ ერთ ქეისს და გამოიცანით
კიდევ სლაიდები იქნება ოღონდ

http://imageshack.us


დიაგნოზი

ციროზი კი არის, მარა რითიაო იმას გეკითხება და სურათზე ერითროციტები არის უამრავი რაოდენობით. იტოგში პოლიციტემია ვერა რუბრას გამოა.

Posted by: LUKA-BRAZI 7 Dec 2008, 02:22
Blind_Torture_Kill
ჰო მეც გამიკვირდა gigi.gif
აზრზე არ ვარ.... პოლიციტემია?
* * *
ვაკეზის დაავადება? baby.gif
smile.gif

Posted by: Blind_Torture_Kill 7 Dec 2008, 02:32
QUOTE
ვაკეზის დაავადება


ეგ რომელია

Posted by: LUKA-BRAZI 7 Dec 2008, 02:35
ვაკეზის დაავდება არის პოლიციტემია, როდესაც ძალიან ბევრი ერითროციტია სისხლში... პათფიზიოლოგიაში გვასწავლეს ოდესღაც, დეტალებს ვერ ვიხსენებ, რამისაა კლავიატურაზე დავიძინო biggrin.gif მოკლედ ჰიპერსპლენიზმი ხო აქვს ამ პაციენტს გამოხატული?

Posted by: Blind_Torture_Kill 7 Dec 2008, 02:36
QUOTE
ვაკეზის დაავდება


არ გამიგია

მოკლედ დიაგნოზს ველი რა ჭირს ამ კაცს
რატო აქ ასციტი

Posted by: vano_t 7 Dec 2008, 02:42
ვა, ახლა არ თქვა პოლიციტემიაზე მეტ დიაგნოზს (რაც უკვე ლუკამაც თქვა და მეც) აკეთებ მაქედან biggrin.gif არ უნდა ამას საუბარი. აქვს პოლიციტემია (პირველადი უფრო მეტი შანსით) და გვიანი გართულება: ციროზი.

აი ზუსტად მასეთი სურათი
user posted image

აიუ გვერდიც: http://www.microscopyu.com/galleries/pathology/polycythemialarge.html

Posted by: LUKA-BRAZI 7 Dec 2008, 02:43
oooo! Shuqi cavida chemtan, mobilit var exla... Xval cavikitxav postebs... Daceqit exla, daidzinet biggrin.gif

Posted by: Blind_Torture_Kill 7 Dec 2008, 02:44
vano_t

ციროზი არააქ

ასციტი რის გამოა ?

Posted by: vano_t 7 Dec 2008, 02:47
QUOTE (Blind_Torture_Kill @ 7 Dec 2008, 02:44 )
vano_t

ციროზი არააქ

ასციტი რის გამოა ?

იმიტომ გეუბენბი რომ შენი კეისები არამკაფიოა. შენ თუ პორტული ვენების თრომბოზი გაქვს მხედველობაში არ ვიცი. მარა, ასეთ ავადმყოფს ციროზიც უყალობდება რომ იცოდე.

ისე პორტული ჰიპერტენზიის მიზეზები ბევრია. ხოდა ეგრე შენს ჭკუაში ვერ იქნება სხვა. შენი კეისი არის, "გამოიცანი მე რას ვფრიქრობ" ტიპის კეისები.

P.S. ღვიძლის ვენების თრომბოზიც შესაძლებელია.

Posted by: Blind_Torture_Kill 7 Dec 2008, 02:55
QUOTE
იმიტომ გეუბენბი რომ შენი კეისები არამკაფიოა. შენ თუ პორტული ვენების თრომბოზი გაქვს მხედველობაში არ ვიცი. მარა, ასეთ ავადმყოფს ციროზიც უყალობდება რომ იცოდე.

ისე პორტული ჰიპერტენზიის მიზეზები ბევრია. ხოდა ეგრე შენს ჭკუაში ვერ იქნება სხვა. შენი კეისი არის, "გამოიცანი მე რას ვფრიქრობ" ტიპის კეისები.

P.S. ღვიძლის ვენების თრომბოზიც შესაძლებელია.


ეს დედანაქაჩი ასციტის სურათი დავდე იმისთვის რომ მიხვიდე დიაგნოზამდე
პოლიციტემია კაი გამოიცანი მარა რა კავშირშია ეგ ასციტთან ?
ხოდა ციროზამდე ჯერ სხვა გართულება უნდა გქონდეს

ბადჩიარის სინდრომი არი პასუხი
რაც გამოიცანი მარა მიჩივი პახოდუ


Posted by: vano_t 7 Dec 2008, 03:05
QUOTE (Blind_Torture_Kill @ 7 Dec 2008, 02:55 )
ეს დედანაქაჩი ასციტის სურათი დავდე იმისთვის რომ მიხვიდე დიაგნოზამდე
პოლიციტემია კაი გამოიცანი მარა რა კავშირშია ეგ ასციტთან ?
ხოდა ციროზამდე ჯერ სხვა გართულება უნდა გქონდეს

ბადჩიარის სინდრომი არი პასუხი
რაც გამოიცანი მარა მიჩივი პახოდუ

დედანაქაჩი ასციტის (ასციტის ახალი კლასიფიკაციაა ალბათ biggrin.gif) მიზეზი ლუკამაც გითხრა-პორტული ჰიპერტენზია. ა, გამახსენდა, შენ მიზეზის მიზეზი გინდა ხო biggrin.gif

ჩივილით კიდევ არავისთან მიჩივლია და არ გიჩილვებ, ნუ გეშინია. არ ვარ მომჩივანი. მიგითითე რომ მკაფიოდ მიუთითე თქო, მეტი არაფერი.

იტოგში, მართალი ხარ. არ შეიძლება მითითება. ეგ ჯამრთელობას ვნებს.

P.S. წავედი ახლა მე და შენც დაიძინე მშვიდად. ამდენი ნერვიულობა ერთ დღეში არ შეიძლება. აბა ძილი ნებისა smile.gif
* * *
Blind_Torture_Kill
QUOTE
ხოდა ციროზამდე ჯერ სხვა გართულება უნდა გქონდეს

მოდი რამოდენიმე რამეზე შევთანხმდეთ. თვითონ კეისებს მშვენიერს დებ. სასწავლებლად კარგია. ყველასათვის არის ინფორმაცია, მაგრამ, არ გეწყინოს და მიდგომა გაქვს სტუდენტური.

1) არ არსებობს მედიცინაში ოქროს სტანდარტი. მედიცინაში ტესტების უმრავლესობას (რაშიც შედის ასევე კლინიკური სურათი და ყევლანაირი ტესტების ერთობლიობა) გააჩნია საშუალო ან ცოტა მაღალი სპეციფიურობა დაავადებების დასადგენად. ყველაზე მაღალი სპეციფიურობით გამოირჩევა უმეტეს შემთხვევაში მიკროსკოპული პათოლოგიები, მაგრამ აქაც უმრალვეს შემთხვევაში არსებულ სურათს მოგცემს ორი პათოლოგია მაინც.

2) მედიცინაში დიაგნოზი არის ინდუქციური და არა დედუქციური. ინდუქციური დიაგნოზი არასდროს იქნება 100 %-იანი. 100 %-იანი შეიძლება იყოს დედუქციური მსჯელობა მხოლოდ. ამის მიზეზი ნაწილობრივ არის #1.

სწორედ ამის გამო, როცა ქეისს იძლევი იმას აქვს რამოდენიმე შესაძლო პასუხი. იმიტომ გითხარი, რომ შენს პირველ ქეისში რენტგენის მიხედვით შეიძლება გეთქვა მხოლოდ რომ არის ორმხრივი დიფუზური ინფილტრატი და მიზეზები არის ესა და ეს. მიკროფოტო არ ვიცოდი მე რა იყო. მაგრამ როცა ახსენე შენ ჰემოსიდერინით დატვირთული მაკროფაგები, დავგიგლე და ვნახე, რომ ამასაც რამოდენიმე პათოლოგია იძლევა. heart failure cells არის, როგორც უცხოელები იტყვიან, misnomer. მსგავსი უჯრედები შეიძლება შეგხვდეს ნებისმიერი ქრონიკული ჰემორაგიის დროს.

შენს მეორე კეისზე, მართალია, პირველი დიაგნოზი უნდა იყოს ღვიძლის ვენების თრომბოზი. ეგ უნდა გვეთქვა პირველ რიგში. მაგრამ მასეთ ავადმყოფებს შეიძლება ჩამოუყალიბდეთ ციროზიც ღვიძლის ვენების თრომბოზის გარეშეც! ისე რომ, ციროზამდე სხვა გართულება არ არის საჭირო. ასეთ ავადმყოფებს შეიძლება ქონდეთ მიკროთრომბოზები.

http://www3.interscience.wiley.com/journal/112180069/abstract?CRETRY=1&SRETRY=0 საინტერესო აბსტრაქტი. უბრალოდ დამაინტერესა ციროზის რა მიზეზებია ხშირია ასეთ პაციენტებში და ამ აბსტრაქტში არის გადმოცემული ერთი კარგი კვლევა. შეხედე რას ამბობს ეს კვლევა:
QUOTE
The pathogenesis of portal hypertension arising in patients with myeloproliferative disorders has been difficult to understand because liver biopsy findings often show minimal changes. It has been suggested that increased splenic blood flow, hepatic infiltration with hematopoietic cells or sinusoidal fibrosis may be important. We have reviewed the autopsy findings and clinical histories of 97 patients with polycythemia vera and 48 patients with agnogenic myeloid metaplasia collected from three institutions and from the Polycythemia Vera Study Group. Cirrhosis was present in seven patients, one of whom had bleeding varices. Esophageal varices were present clinically in 10 patients without cirrhosis (seven polycythemia and three agnogenic myeloid metaplasia). All of these patients had lesions in small or medium-sized portal veins and four had stenosis of the extrahepatic portal vein with histology compatible with organized thrombi. Nodular regenerative hyperplasia occurred in 14.6% and correlated closely with the presence of portal vein lesions. Thirty patients had > 500 ml of ascites, seven of these patients also had varices and six of them had hepatic vein thrombosis. Ascites also correlated with hepatic vein disease confined to small intrahepatic branches. No correlation was seen between hepatic hematopoietic infiltration and signs of portal hypertension.
საინტერესო ის არის, რომ ასციტიან პაციენტებში უფრო ხშირად კორელაცია იყო ისეთ თრომბოზებთან, რომელიც მხოლოდ პატარა ინტრაჰეპატურ სისხლძარღვებში იყო განვითარებული. კიდევ ის ფაქტია საინტერესო, რომ ეს აბსტრაქტი არ იჩემებს, რომ ასეთ ავადმყოფებში პორტული ჰიპერტენზიის მიზეზი არის ღვიძლის ვენების (ან სხვა მცირე სისხლძარღვების) თრომბოზი. არამედ, აქ აშკარად ამბობენ რომ ამ დაავადების დროს პორტული ჰიპერტენზიის მიზეზი უცნობია და რამოდენიმე შესაძლო ფაქტორს ასახელებენ, მათ შორის ღვიძლის ინფილტრაცია ჰემატოპოეტური უჯრედებით (პირველი გამუქებული აბზაცი).

ამიტომ გეუბნები, რომ მედიცინაში კატეგორიულობა არ შეიძლება. შენი ჯინი კი არ მჭირს. მე საკმაოდ დიდი ხანი ვმუშაობ და მინახია ბევრი რამ რომელიც საბოლოოდ გასწავლის, რომ "არასოდეს თქვა 'არასდროს' და არასოდეს თქვა 'ყოველთვის'". ეს არის უპირველესი პრინციპი, რომელიც ყველა ექიმმა უნდა გააცნობიეროს. ეს მარტივი პრინციპი გაძლევს იმის საშუალებას, რომ ნებისმიერი პრეზენტაციის დროს ნამდვილი დიაგნოზის შესაძლებლობა არ გამოგრჩეს მხედველობიდან და დიფერენციალური დიაგნოზი მოიცავდეს რაც შეიძლება მეტ დიაგნოზს. აი ეს არის და ეს.

**************************************************************************************************

QUOTE
70 კგ-ანი ჯანმრთელი მამაკაცი კარგავს 2 ლიტრა სიხთეს ოფლის სახით ფიზიკური მუშაობის პროცესში.ამის შემდეგ იგი სვავს 2 ლიტრა წყალს რათა აანაზღაუროს დანაკარგი.რა ცვლილებებია მოსალოდნელი ?

ა. გაიზრდება უჯრედსგარე სითხის ოსმოლალობა
ბ. გაიზრდება უჯრედსგარე სითხის მოცულობა
გ. გაიზრდება უჯრედშიდა სითხის ოსმოლალობა
დ. გაიზრდება უჯრედშიდა სითხის მოცულობა
ე. გაიზრდება Na-ის კონცენტრაცია პლაზმაში
ამ კითხვაზე მშვენიერი პასუხი გაგცა ლუკამ. მე უბრალოდ მინდა წმინდად ტექნიკური "ილეთი" გაჩვენოთ, რომელიც დააინტერესებს USMLE-ის ჩამბარებლებს და საერთოდ ყველას ვინც ტესტებს აბარებს. ის პასუხი რაც ლუკამ დადო მოითხოვს სპეციფიური ინფორმაციის ცოდნას და, რა თქმა უნდა, კარგია როცა იცი ეს ყველაფერი. როცა არ იცი, მაშინ რა უნდა ქნა? მაშინაც შეიძლება მსგავსი ტიპის კითხვებზე გასცე პასუხი ხუთიანზე (ან ამოსარჩევი პასუხების რიცხვი შეამცირო). სწორედ ამ სტრატეგიას დავდებ მათთვის, ვისაც აინტერესებს ეს ყველაფერი.


ერთი მარტივი ლოგიკის კანონი უნდა დაიმახსოვროთ ამისათვის: მტკიცება და მისი გამომრიცხავი მტკიცება არ შეიძლება ორივე სწორი იყოს. მაგრამ, ერთ-ერთი აუცილებლად სწორი იქნება. როცა ვამბობ, რომ "A არის B" და "A არ არის B", ამ ორი წინადადებიდან ერთი ერთი აუცილებლად სწორია. ანუ A-ს ან გააჩნია თვისება B ან არ გააჩნია-სხვა ვარიანტი არ არსებობს. ეს ყველამ ვიცით და მარტივი წესია. როგორ შეიძლება გამოიყენო ეს წესი ამ კითხვაში? პასუხებში (არა მარტო ამ მაგალითში, არამედ ნებისმიერი ტესტის დროს შეიძლება ეს სტრატეგია გამოიყენო) უნდა ეძებო მსგავსი ურთიერთგამომრიცხავი დებულებები. შეიძლება ურთიერთგამომრიცხავი პასუხები აშკარად არ ჩანდეს და ერთ-ერთი პასუხი იყოს ექვივალენტური ურთიერთგამომრიცხავი პასუხი. ჩვენს პასუხებში გაქვს ასეთი პასუხები: "დ", რომელიც ამბობს "გაიზრდება უჯრედშიდა სითხის მოცულობა" და "გ", რომელიც ამბობს "გაიზრდება უჯრედშიდა სითხის ოსმოლალობა". ერთ-ერთი ეს პასუხი შეიძლება გადაიყვანო ექვივალენტურ პასუხში. მაგალითისათვის ავიღოთ "გ". "გ" იმის ტოლფასია, რომ "შემცირდება უჯრედშიდა სითხის მოცულობა", რადგანაც უჯრედში სითხის მოცულობის გაზრდა იწვევს ოსმოლარობის შემცირებას. ანუ "გ" და "დ" ურთიერთსაწინააღმდეგო (და არა ურთიერთგამომრიცხავ) პასუხებს წარმოადგენენ. ერთ-ერთი პასუხი რომ ყოფილიყო, რომ არ გაიზრდება მოცულობა (ან ოსმოლარობა), მაშინ ნამდვილად ურთიერთგამომრიცხავები იქნებოდნენ და ამ ორიდან ერთ-ერთი იქნებოდა ასპროცენტიანი მართალი. "არ გაზრდა" გულისხმობს ან შემცირებას, ან არ შეცვლას. მაგრამ დანარჩენი პასუხებში არ გაქვს არცერთი პასუხი, რომელიც არ შეცვლაზე საუბრობს. ყველა პასუხი ამტკიცებს, რომ ან გაიზრდება, ან შემცირდება. შესაბამისად "გ" და "დ"-დან ერთ-ერთი აუცილებლად სწორია. სხვა ვარიანტი არ არსებობს. ამრიგად, რომც არ იცოდე მთელი ეს ფიზიოლოგია, მარტო მარტივი ლოგიკის კანონის საფუძველზე არჩევანი დაიყვანე 2 პასუხზე. თუ დამიჯერებთ, USMLE-ს კითხვებში ბევრგან შეგიძლიათ გამოიყენოთ ეს სტრატეგია და მე პირადად გამომიყენებია ყოველთვის, როცა პასუხების აზრზე არ ვარ. ამ კონკტრეტული კითხვის პასუხებში (და მსგავსების დროს) მთლად გვიმართლებს და სწორი პასუხი შეიძლება მაოვირჩიოთ საერთოდ. ამ ტიპის კითხვაში არ არის მოცემული ვარიანტები, რომელიც გეუბნება, რომ რომელიღაც 2 ან 3 პასუხია სწორი. მაგალითად, ბოლოში შეიძლება ყოფილიყო პასუხი, რომელიც ამბობს, რომ "ა" და "გ" არის სწორი-სიტყვაზე ვამბობ. ეს იმას ნიშნავს, რომ მხოლოდ ერთი პასუხია სწორი. ახლა "გ" და "დ" პასუხი ექვივალენტურ პასუხს თუ მოძებნი დარჩენილ პასუხებს შორის, ეს ავტომატურად გამორიცხავს ამ პასუხებს. პასუხი "ა" ამბობს, რომ "გაიზრდება უჯრედსგარე სითხის ოსმოლალობა". სითხის დაკარგვა ან შეძენა ჯერ ხდება უჯრედს გარეთ და უჯრედგარეთა სითხის ცვლილებები იწვევს შემდეგში უჯრედშიდა სითხის ცვლილებებს. ანუ, თუ "ა" ამბობს, რომ უჯრედს გარეთ ოსმოლარობა გაიზრდება, ამას მოყვება წყლის გადანაცვლება უჯრედიდან უჯრედს გარეთ და შესაბამისად უჯრედის შიგნით ოსმოლარობის გაზრდა ხდება. ე.ი. პასუხი "ა" და პასუხი "გ" ფიზიოლოგიურად ექვივალენტურია. ანუ, თუ "ა" არის სწორი, მაშინ "გ"-ც სწორია. მაგრამ ეს შეუძლებელია, რადგანაც არ გაქვს არჩევანი, რომელიც ამბობს რომ "ა" და "გ" სწორია. შესაბამისად რჩება მხოლოდ "დ". მსგავსი სტრატეგია დაგეხმარებათ უსაშველოდ ტესტების ამოხსნაში. ამ შემთხვევაში მინიმალური ინფორმაციის საფუძველზე და მარტივი ლოგიკის კანონზე დაყრდნობით მთელი პასუხი მივიღეთ.

Posted by: LUKA-BRAZI 7 Dec 2008, 16:13
vano_t
ეგ მართლაც ფაქტია მგონი, რომ USMLE-ის ტესტურ გამოცდაზე მშრალი ცოდნის გარდა ალბათ უნარ-ჩვევებსაც გიმოწმებენ (ოღონდ არა ქართული უნარ-ჩვევების მსგავსად biggrin.gif ), ამიტომ 100% ცოდნის გარდა, კიდევ რაღაც ასეთი ხრიკებიც უნდა იცოდე, თორემ ერთ ტესტზე 1 წუთი არც ისე დიდი დროა.... მე ვიტყოდი ცოტაც კია, განსაკუთრებით მათთვის ვინც შედარებით ნელა კითხულობს და პასუხამდე მისასვლელად ბევრი ფიქრი უწევს.... და მართლაც, განსაკუთრებით ასეთი რამე მაშინაა მისაღები, როდესაც სწორი პასუხი არ იცი....
ასე რომ დიდი მადლობა ვანოს biggrin.gif მომავალშიც ველით მსგავს Tips & Hints - ებს biggrin.gif
wink.gif

Posted by: Blind_Torture_Kill 8 Dec 2008, 18:39
10 წლის ბავშვს აღენიშნება მასა მუცლის ღრუში. CT-აჩვენებს გადიდებულ ლიმფურ კვანძებს ჯორჯალში და რეტროპერიტონეუმში. ბიოფსიით მიღებული მასა ნაჩვენებია ქვემოთ.

რა არის და როდის ხდება ?

Posted by: LUKA-BRAZI 8 Dec 2008, 18:56
Blind_Torture_Kill
ლეიკოზი არა ხო? biggrin.gif

Posted by: Blind_Torture_Kill 8 Dec 2008, 19:00
LUKA-BRAZI

smile.gif

იფიქრე ბატონო ლუკა

Posted by: LUKA-BRAZI 8 Dec 2008, 19:02
10 წლის ბავშვს შეიძლება ლიმფოგრანულომატოზი, ან ციტომეგალოვირუსული ინფექცია ქონდეს, რაც ასევე გვაძლევს ლიმფური ჯირკვლების გადიდებას.....

თუმცა ამ შემთხვევაში ყველა ლიმფური კვანძი დიდდება და არა მარტო ჯორჯლის.....

ვააა, რამე ნამიოკი? თუ ჯერ ადრეა? biggrin.gif

Posted by: Blind_Torture_Kill 8 Dec 2008, 19:21
LUKA-BRAZI

ჯერ ადრეა

ძნელი გამოსაცნობი არაა

Posted by: texasuri jleta benzoxerxit 8 Dec 2008, 19:29
QUOTE (Blind_Torture_Kill @ 8 Dec 2008, 18:39 )
10 წლის ბავშვს აღენიშნება მასა მუცლის ღრუში. CT-აჩვენებს გადიდებულ ლიმფურ კვანძებს ჯორჯალში და რეტროპერიტონეუმში. ბიოფსიით მიღებული მასა ნაჩვენებია ქვემოთ.

რა არის და როდის ხდება ?

ია ვსეგდა პლავალ ვ გისტოლოგიი boli.gif


შტერნბერგის გიგანტური უჯრედები?


ალტერ.დიაგნოზი-მეზენტერიალური ჯირკვლების ტუბერკულოზი

Posted by: Blind_Torture_Kill 8 Dec 2008, 19:32
texasuri jleta benzoxerxit

QUOTE
შტერნბერგის გიგანტური უჯრედები?


არა

QUOTE
ალტერ.დიაგნოზი-მეზენტერიალური ჯირკვლების ტუბერკულოზი


არა
smile.gif

Posted by: LUKA-BRAZI 8 Dec 2008, 19:36
texasuri jleta benzoxerxit
QUOTE
მეზენტერიალური ჯირკვლების ტუბერკულოზი

ეგ კი მარა, კაზეოზი ხომ უნდა ჩანდეს მაშინ? რა ვიცი, რა ვიცი.... smile.gif პაწუკა ნამიოკი კლინიკიდან? biggrin.gif

Posted by: Blind_Torture_Kill 8 Dec 2008, 19:40
LUKA-BRAZI

QUOTE
ეგ კი მარა, კაზეოზი ხომ უნდა ჩანდეს მაშინ? რა ვიცი, რა ვიცი....პაწუკა ნამიოკი კლინიკიდან?


უფრო დიდ გადიდებაზე



Posted by: LUKA-BRAZI 8 Dec 2008, 19:57
Blind_Torture_Kill
ჯერ ეს მითხარი, ყვითლები ადიპოციტებია?

Posted by: Blind_Torture_Kill 8 Dec 2008, 20:18
LUKA-BRAZI

არა
ეგენი არც გაინერესებს პრინციპში smile.gif

Posted by: LUKA-BRAZI 8 Dec 2008, 20:40
Blind_Torture_Kill
QUOTE
ეგენი არც გაინერესებს პრინციპში

ჰოოო, არა ისე "პროსტა".... ყვითლებზე იმიტომ გკითხე რომ ის წითლები აზრზე არ ვარ რა არის biggrin.gif

Posted by: basa-ttt 8 Dec 2008, 21:06
QUOTE
10 წლის ბავშვს აღენიშნება მასა მუცლის ღრუში. CT-აჩვენებს გადიდებულ ლიმფურ კვანძებს ჯორჯალში და რეტროპერიტონეუმში. ბიოფსიით მიღებული მასა ნაჩვენებია ქვემოთ.

ეს შეიძლება იყოს ლიმფ ჯირკვლების მეტასტაზური დაზიანება
ან ლიმფოგრანულომატოზი.

Posted by: Guardian 8 Dec 2008, 21:24
Blind_Torture_Kill

პირველი და მეორე სურათი ერთი და იგივე დაავადების არის? spy.gif
პირველი ბერკიტის ლიმფომას ჰგავს (ვარსკვლავებიანი ცა), და მეორე - მწვავე მიელოიდურ ლეიკემიას (ბლასტები).

Posted by: Blind_Torture_Kill 9 Dec 2008, 18:32
Guardian

QUOTE
პირველი ბერკიტის ლიმფომას ჰგავს (ვარსკვლავებიანი ცა)


ხო ეგაა არააფრიკული ფორმა

Burkitt lymphoma cells also have characteristic morphology in air-dried Wright-Giemsa stained smears, ie, the cells have deeply basophilic cytoplasm with lipid-containing vacuoles.

Posted by: LUKA-BRAZI 9 Dec 2008, 18:50
სხვა რაღა დაგვრჩენია smile.gif
Guardian
up.gif

Posted by: Blind_Torture_Kill 9 Dec 2008, 19:17
ახალი ქეისები დადეთ

რა არის და როდის გვხვდება


baby.gif

Posted by: basa-ttt 9 Dec 2008, 19:29
QUOTE
რა არის და როდის გვხვდება

რაქიტის დროს ნამდვილად გვხვდება....
gigi.gif

Posted by: Blind_Torture_Kill 9 Dec 2008, 19:37
QUOTE
რაქიტის დროს ნამდვილად გვხვდება....


კაი გოგო ხარ
up.gif

Posted by: LUKA-BRAZI 9 Dec 2008, 19:55
ხვალ ჩათვლა მაქვს, ასე რომ ვერც ქეისს დავდებ და ვერც განხილვებში მიიღებ მონაწილეობას user.gif ხვალ შემოგიერთდებით smile.gif

Posted by: basa-ttt 9 Dec 2008, 20:11
აბა ამაზე იფიქრეთ - ბავშვს აქვს ასეთი ფორმის გამონაყარი ტანზე -
user posted image
სისუსტე , თავის ტკივილი, სახსრების ტკივილი.

რა შეიძლება იყოს?
დააკვირდით ცენტრს...



Posted by: badu 9 Dec 2008, 20:25
QUOTE (basa-ttt @ 9 Dec 2008, 19:11 )
აბა ამაზე იფიქრეთ - ბავშვს აქვს ასეთი ფორმის გამონაყარი ტანზე -
user posted image
სისუსტე , თავის ტკივილი, სახსრების ტკივილი.

რა შეიძლება იყოს?
დააკვირდით ცენტრს...

lyme disease (Borrelia burgdorferi)?

Posted by: mtvareuli 9 Dec 2008, 21:13
badu
QUOTE
lyme disease (Borrelia burgdorferi)?

yes.gif

ერითემა ქრონიკუმ მიგრანს



* * *


რადგან ეგეთ მარტივებს დებთ

აჰა ესეც კიდევ ერთი მარტივი

რა არის და რამ შეიძლება გამოიწვიოს

Posted by: vano_t 9 Dec 2008, 22:07
QUOTE (mtvareuli @ 9 Dec 2008, 21:13 )
რადგან ეგეთ მარტივებს დებთ

აჰა ესეც კიდევ ერთი მარტივი

რა არის და რამ შეიძლება გამოიწვიოს

დიფუზური ერითროდერმა.

ბევრ დაავადებას ახლავს. შეიძლება იყოს წამლისმიერი, კანის ტ უჯრედოვანი ლიმფომა, დიფუზური დერმატიტი და ა.შ.

Posted by: mtvareuli 9 Dec 2008, 22:25
vano_t
QUOTE
შეიძლება იყოს წამლისმიერი, კანის ტ უჯრედოვანი ლიმფომა, დიფუზური დერმატიტი და ა.შ.

კიდევ?


Posted by: Blind_Torture_Kill 9 Dec 2008, 23:07
mtvareuli

ეს ეხლა ნამდვილია?
სურათს ვგულისხმობ

Posted by: mtvareuli 9 Dec 2008, 23:10
Blind_Torture_Kill

კი, დერმატოლოგიის წიგნიდანაა აღებული rolleyes.gif


Posted by: Blind_Torture_Kill 9 Dec 2008, 23:14
mtvareuli

პლასტელინისგან გამოძერწილს გავს
baby.gif

Posted by: LUKA-BRAZI 9 Dec 2008, 23:17
გუგული დეიდამ (ანუ google.com-მა biggrin.gif ) განაცხადა:
Erythroderma (Generalized Exfoliative Dermatitis) ->
Determining specific etiologies in ED often is not possible; however, it is necessary to attempt since etiology may impact disease course and management options. The list of conditions that can cause ED is extensive and continues to expand. Cutaneous diseases that cause ED and the systemic diseases associated with them include the following:

Atopic dermatitis - Acute and chronic leukemia
Contact dermatitis - Reticulum cell sarcoma
Dermatophytosis - Carcinoma of rectum
Hailey-Hailey disease - Carcinoma of fallopian tubes
Leiner disease - Graft versus host disease
Lichen planus - HIV infection
Lupus erythematosus - Lymphoma (including Hodgkin disease)
Mycosis fungoides - Multiple myeloma
Pemphigoid - Carcinoma of lung
Pemphigus foliaceus - Mycosis fungoides
Pityriasis rubra pilaris - Reiter syndrome
Psoriasis
Sarcoid
Seborrheic dermatitis
Stasis dermatitis

The most common causes of ED are best remembered by the mnemonic device ID-SCALP. The causes and their frequencies are as follows:
Idiopathic - 30%
Drug allergy - 28%
Seborrheic dermatitis - 2%
Contact dermatitis - 3%
Atopic dermatitis - 10%
Lymphoma and leukemia - 14%
Psoriasis - 8%

დანარჩენი სხვა აქ: http://www.emedicine.com/DERM/topic142.htm
smile.gif

Posted by: mtvareuli 9 Dec 2008, 23:18
Blind_Torture_Kill
QUOTE
პლასტელინისგან გამოძერწილს გავს

კი გავს მართლა biggrin.gif

ფსორიაზული ერითროდერმაა მაგ სურათზე, თუმცა სხვა რამემაც შეიძლება გამოიწვიოს დიფუზური ერითროდერმა, ვანო_ტ-მ ჩამოთვალა რამოდენიმე.

Posted by: basa-ttt 9 Dec 2008, 23:27
QUOTE
ფსორიაზული ერითროდერმაა მაგ სურათზე, თუმცა სხვა რამემაც შეიძლება გამოიწვიოს დიფუზური ერითროდერმა, ვანო_ტ-მ ჩამოთვალა რამოდენიმე

ამიტომ კლინიკაც მიაყოლეთ ხოლმე.

Posted by: Blind_Torture_Kill 10 Dec 2008, 02:00
აბა სავარაუდოდ რამ მოკლა ეს პიროვნება

Posted by: badu 10 Dec 2008, 02:10
ფილტვის კიბომ? . ..

Posted by: Blind_Torture_Kill 10 Dec 2008, 02:18
badu

რატო იმენოი ფილტვის კიბომ ?
რამ გავარაუდებინა ეგ ?

Posted by: vano_t 10 Dec 2008, 05:55
Blind_Torture_Kill
QUOTE
აბა სავარაუდოდ რამ მოკლა ეს პიროვნება

მე დავუშვებ რომ პიროვენება უკვე გარდაცვლილია. (თუ გარდაცვლილი არ არის, მაშინ სავარაუდოდ სხვა ნიშნებია, რომლის საფუძველზეც შეიძლება ივარაუდო სხვადასხვა დაავადებები)

თუ გარდაცვლილია ეგ ავადმყოფი, ყურადღებას იპყრობს კანის ფერი. ეგ არ არის ტიპიური მკვდარი ადამიანის ფერი. ადამიანი როცა კვდება, ფერმკრთალობა (და მერე კანის გამუქება) უკვე წუთებშია შესამჩნევი. მაგას ვარდისფერი აქვს, ცოცხალ კანს გავს დაჟე.

2 რამ რაც ვიცი, რომელიც ამას მოგცემს, არის ციანიდებით მოწამვლა და ნახშირჟანგით მოწამვლა. თითების მდგომარეობა მოხრილია და შეიძლება კრუნჩხვაც ქონდა, ასფიქსიის გამო, სიკვდილის წინ (რა თქმა უნდა აქ იგულისხმება, რომ სიკვდილამდე რაიმე მდგომარეობა არ გააჩნდა, რომელიც გაძლევს მასეთ რამეს ცოცხალში-მაგალითად დიუპიტრენის კონტრაქტურა).

Posted by: basa-ttt 10 Dec 2008, 08:10
QUOTE
ნახშირჟანგით მოწამვლა

yes.gif
"ნიკალა" ჰქონდა ეტყობა დამონტაჟებული.
user.gif

Posted by: vano_t 10 Dec 2008, 09:43
basa-ttt
QUOTE
QUOTE
ნახშირჟანგით მოწამვლა

yes.gif
"ნიკალა" ჰქონდა ეტყობა დამონტაჟებული.
user.gif

ალბათ, თუ საქართველოში მოხდა ეგ.

მე მგონი, ბოლო ბოლო, რაღაცაზე შევთანხმდით smile.gif

Posted by: basa-ttt 10 Dec 2008, 09:58
vano_t
QUOTE
მე მგონი, ბოლო ბოლო, რაღაცაზე შევთანხმდით

2kiss.gif


ეს შემთხვევა ნახე?
http://forum.ge/?f=43&showtopic=33683930&st=1695
მაგარ კარგ ხასიათზე ვიყავი 2 დღის წინ -
იმ ბავშვის მშობლის თვალები არ დამავიწყდება.
smile.gif

Posted by: badu 10 Dec 2008, 10:41
QUOTE (Blind_Torture_Kill @ 10 Dec 2008, 01:18 )
badu

რატო იმენოი ფილტვის კიბომ ?
რამ გავარაუდებინა ეგ ?

Digital clubbing (საჩვენებელ თითზე ეტყობა თითქოს). ქართულად არ ვიცი რა ქვია ამ სიმპტომს. ფილტვის კიბო მხოლოდ ერთ-ერთი გამომწვევი მიზეზია. სხვა გულის ან ფილტვის დაავადებასაც შეუძლია ამის გამოწვევა.

Posted by: Blind_Torture_Kill 10 Dec 2008, 11:09
vano_t

QUOTE
2 რამ რაც ვიცი, რომელიც ამას მოგცემს, არის ციანიდებით მოწამვლა და ნახშირჟანგით მოწამვლა


CO-მ მოწამლა
ეგეთ ღია ვარდისფერ ჰიპოსტაზს ეგ იძლევა

ციანიდით მოწამვლა არ მინახია მარა უფრო მოყვითალო-მოწითალო (აგურის) ფერს იძლევა წიგნებს თუ დავუჯერებთ

თითების მოხრა რიგორ მორტისის გამოა

badu

ქლაბინგი არააქ
* * *
რა ჭირდა ამ პიროვნებას ?

user.gif
* * *
http://imageshack.us
http://g.imageshack.us/img201/10002590hr3.jpg/1/

Posted by: LUKA-BRAZI 10 Dec 2008, 14:07
Blind_Torture_Kill
თირკმლის ტუბერკულოზი?

Posted by: basa-ttt 10 Dec 2008, 14:26
QUOTE
რა ჭირდა ამ პიროვნებას ?

თირკმელში - შუა სეგმენტში თეთრი ფერის უბანია.
შემთხვევის კლინიკა დაამატეთ....

Posted by: Cousteau 10 Dec 2008, 16:13
QUOTE (Blind_Torture_Kill @ 10 Dec 2008, 11:09 )

* * *
რა ჭირდა ამ პიროვნებას ?

user.gif 
* * *
http://imageshack.us
http://g.imageshack.us/img201/10002590hr3.jpg/1/

PKD ? eek.gif

პაციენტის ასაკი დაწერე რა, ნეფრობლასტომა ხო არ არის?
PKD მგონი არაა, იმას იმენა ცისტებიაქ

Posted by: Blind_Torture_Kill 10 Dec 2008, 18:55
კიბო და კისტები არაა

QUOTE
პაციენტის ასაკი დაწერე რა


ვთქვათ 47

სურათი რომ აღწერო უფრო გაგიადვილდება

ჰა შედარებისთვის ნორმა

Posted by: basa-ttt 10 Dec 2008, 19:48
Blind_Torture_Kill
შეჭმუხნულ თირკმელს ჰგავს -
ნეფროსკლეროზი?

Posted by: LUKA-BRAZI 10 Dec 2008, 19:53
Blind_Torture_Kill
და თირკმლის ტუბერკულოზი არაა?

Posted by: Blind_Torture_Kill 10 Dec 2008, 20:04
LUKA-BRAZI

არა
შეჭმუხნულიც არის და ზომაში შემცირებულიც smile.gif
და კიდევ ერთი დამახასიათებელი ნიშანია რომლითაც დაავადებას გამოიცნობ

Posted by: LUKA-BRAZI 10 Dec 2008, 20:06
ჰმმმმ.... თირკმლის ამილოიდოზი? მაგრამ მგონი მაგას ზომებში შემცირება კი არა, პირიქით, გაზრდა ახასიათებს....

Posted by: Blind_Torture_Kill 10 Dec 2008, 20:12
QUOTE
ზომებში შემცირება კი არა, პირიქით, გაზრდა ახასიათებს


კი

Posted by: Cousteau 10 Dec 2008, 22:37
no.gif პას. ვერ ვხვდები უბრალოდ რა ჩანს...
გრანულომებია?

Posted by: donvaso 10 Dec 2008, 22:43
..............................................................................


Posted by: vano_t 10 Dec 2008, 22:45
თირკმლის არტერიების განმეორებითი ინფარქტები? ყველაზე დიდი თეთრი არე შეიძლება არის ახალი ინფარქტი, დანარჩენი ძველი და დანაწიბურებული ინფარქტებია.

რა თქმა უნდა მე პათოლოგი არ ვარ და შესაბამისად პასუხიც მოსალოდნელია არ იყოს სწორი.

Posted by: Blind_Torture_Kill 10 Dec 2008, 22:56
vano_t

ინფარქტებია და თუ სწორედ აღწერ სავარაუდო დიაგნოზსაც დასვავ

Posted by: vano_t 10 Dec 2008, 23:07
QUOTE (Blind_Torture_Kill @ 10 Dec 2008, 22:56 )
ინფარქტებია და თუ სწორედ აღწერ სავარაუდო დიაგნოზსაც დასვავ

თუ ინფარქტებია, მაშინ ან ემბოლიებია ან თრომბოზებია. ემბოლიებს არ მგონია რაიმე სპეციფიური ნიშანი ქონდეს (შეიძლება აქვს, მარა მაგას ვერ გეტყვი).

თრომბოზებს ვირჩევ. თრომბოზი მიუთითებს ადგილობრივ პათოლოგიაზე. ასეთი შეიძლება იყოს თირკმლის არტერიების ანევრიზმები ან ფიბრომასკულარული დაავდებები, ან ვასკულიტები.

ისე რავი, ზევით რომ 2 "ღრუა" შეიძლება იყოს ანევრიზმები, ჰა, თუ როგორ? biggrin.gif

Posted by: basa-ttt 10 Dec 2008, 23:36
თირკმლის არტერიის თრომბოზი?
სამკუთხედის ფორმის არ უნდა იყოს დაზიანება თირკმლის ინფარქტის დროს?...
არა რა,
კლინიკური სურათის გარეშე ასეთ კეისებს აზრი ეკარგება...

Posted by: Blind_Torture_Kill 11 Dec 2008, 00:52
vano_t

არა

basa-ttt

კარგი

პაციენტი შემოვიდა მაღალი სიცხით + შემცივნებებით + დიზურიით
აღენიშნებოდა ფერდებში ტკივილი

აი ესაა მთელი კლინიკა

ვიცი ეხლა რა დიაგნოზიც იქნება smile.gif

მარა გაკვეთისას ურადღება მიიქცია კიდევ სხვა რამემ ხოდა ეგ სხვა რამეა ფოტოზე

Posted by: basa-ttt 11 Dec 2008, 01:03
QUOTE
პაციენტი შემოვიდა მაღალი სიცხით + შემცივნებებით + დიზურიით
აღენიშნებოდა ფერდებში ტკივილი

და მოკვდა?
eek.gif
მაგი ნეფრიტის (& პიელონეფრიტის) ნიშნებია
მარა მაგისგან არ კვდებიან...

P.S. - ნეუჟელი ნეკროზი წავიდა?

Posted by: vano_t 11 Dec 2008, 02:56
Blind_Torture_Kill
QUOTE
პაციენტი შემოვიდა მაღალი სიცხით + შემცივნებებით + დიზურიით
აღენიშნებოდა ფერდებში ტკივილი

აი ესაა მთელი კლინიკა

ვიცი ეხლა რა დიაგნოზიც იქნება smile.gif

მარა გაკვეთისას ურადღება მიიქცია კიდევ სხვა რამემ ხოდა ეგ სხვა რამეა ფოტოზე

მაგ თირკმელზე გაკვეთით რა ჩანს მაგას ვერ გეტყვი აწი იმაზე მეტს, რაც გამოვიცანი.

კიდევ ერთს ვსინჯავ და თუ ვერ ამოვალ სახლში წავალ. ერთი სახის ემბოლიური ინფარქტი შეიძლება დაადგინო ალბათ გაკვეთაზე. ეს იქნება სეპტიკური ემბოლიები (დავუშვათ ენდოკარდიტის დროს). ამ დროს შესაძლებელია მრავლობითი აბსცესესები გქონდეს (ამ სურათზე არის თუ არა აბსცესი ძნელი სათქმელია ასე, მარა თირკმელს რომ შეხედავ უშუალოდ, მაშინ ალბათ აბსცესების გარჩევა არ უნდა იყოს ძნელი). ასეთი რამ პიელონეფრიტსაც მოგცემს, როგორც ბასა-ტტტ-მ აღნიშნა უკვე. მაგრამ პიელონეფრიტები ბევრი რამის დროს შეიძლება შეგხვდეს.

basa-ttt
QUOTE
მაგი ნეფრიტის (& პიელონეფრიტის) ნიშნებია
მარა მაგისგან არ კვდებიან...
ნეფრიტში თუ პიელონეფრიტს გულისხმობ, მაშინ მართალია-პიელონეფრიტით მასე მოდიან. მარა თუ გლომერულონეფრიტს გულისხმობ, მაშინ რაღაც არ მგონია. გლომეროლუნეფრიტი არაინფექციური ანთებითი დაავადებაა (უმეტესწილად ავტოიმუნური). ნეფრიტული სინდრომის დროს გაქვს ჰემატურია(წითელი უჯრედებისაგან შემდგარი ცილინდრები), არანეფროზული პროტეინურია, ურემია და ჰიპერტენზია. ეს სულს სხვა რამ არის პიელონეფრიტისაგან განსხვავებით.

Posted by: LUKA-BRAZI 11 Dec 2008, 14:19
vano_t
yes.gif სეპტიკური ენდოკარდიტის დიაგნოზში გეთანხმები.... მაგ დროს მართლაცაა თრომბოემბოლიები და არამარტო თირკმელებში, არამედ ნებისმიერ ორგანოში.... ასე რომ სავსებით რეალურია მართლაც ენდოკარდიტის თრომბოემბოლიურ გართულებებთან გვქონდეს საქმე.....

Posted by: basa-ttt 11 Dec 2008, 18:50
Blind_Torture_Kill
გადაგვარჩინე ბოლო ბოლო
და გვითხარი რაა ეს....
gigi.gif

Posted by: Blind_Torture_Kill 11 Dec 2008, 19:07
QUOTE
გადაგვარჩინე ბოლო ბოლოდა გვითხარი რაა ეს....



რა ნაწილში ჩანს ნეკროზები ?

Posted by: basa-ttt 11 Dec 2008, 19:14
QUOTE
რა ნაწილში ჩანს ნეკროზები ?

მე პირადად ზემო წილებში ვნახე...
ყავისფერი უბნები რომაა ისაა?

Posted by: Blind_Torture_Kill 11 Dec 2008, 19:25
QUOTE
მე პირადად ზემო წილებში ვნახე... ყავისფერი უბნები რომაა ისაა?


რა ნაწილებია ეგენი მერე

ხო პიელონეფრიტმა დაბრიდა ეგ პიროვნება

Posted by: basa-ttt 11 Dec 2008, 19:29
პიელონეფრიტის დროს შეიძლება განვითარდეს
გორგლოვანი ნეკროზი -არ ვიცი სწორედ ვანბობ თუ არა
რუსულად არის
некроз почечных сосочков

და სკლეროზული პროცესები..

Posted by: Blind_Torture_Kill 11 Dec 2008, 19:32
basa-ttt

რა ადგილას გაქ იმ სურათზე ნეკროზი ?

Posted by: basa-ttt 11 Dec 2008, 19:40
ანუ იგივე პაპილარული ნეკროზი -
მაგის თქმა გინდა?

Posted by: Blind_Torture_Kill 11 Dec 2008, 19:43
QUOTE
ანუ იგივე პაპილარული ნეკროზი -მაგის თქმა გინდა


მაგის თქმა კი არ მინდა ეგაა სურათზე smile.gif

Posted by: basa-ttt 11 Dec 2008, 19:58
QUOTE
მაგის თქმა კი არ მინდა ეგაა სურათზე

მერე ზევით ხომ დავწერე-

QUOTE
некроз почечных сосочков

ეგაა...
* * *
Некроз почечных сосочков (синоним: некротический папиллит, папиллярный некроз) — ишемический инфаркт почечного сосочка и прилегающего к нему мозгового вещества почки.

Н.п.с. называют первичным, если ишемия сосочка не связана с воспалением почек, и вторичным, если она развивается в связи с воспалительной инфильтрацией и тромбозом сосудов почечной пирамидки при пиелонефрите. Первичный Н.п.с. развивается вследствие поражения артериол мозгового вещества почки и нарушения реологических свойств крови. Он встречается при сахарном диабете, системных васкулитах, хронической интоксикации анальгетиками, серповидно-клеточной анемии, алкоголизме. Н.п.с. чаще бывает двусторонним. Некротизированный сосочек секвестрируется и кальцифицируется или отторгается, что нередко сопровождается кровотечением и окклюзией мочеточника. Обычно процесс имеет очаговый характер, но иногда, особенно при вторичном Н.п.с., развивается фатальный распространенный медуллярный некроз.

Клинические проявления первичного и вторичного Н.п.с. сходны, но при вторичном они сочетаются с проявлениями пиелонефрита, которые могут преобладать. Первичный Н.п.с. обычно имеет хроническое рецидивирующее течение. Длительное время он может проявляться лишь малой протеинурией, умеренной лейкоцитурией, микрогематурией, но характерно раннее снижение концентрационной способности почек, что приводит к полиурии, гипостенурии, потере натрия с мочой. При отторжении сосочка развиваются почечная колика, макрогематурия, олигурия.

აი ნახე -
პაპილარული ნეკროზი შეიძლება იყოსო პირველადი და მეორადიო -
ანუ პირველადი დაკავშირებულია სისხლძარღვებთან და სისხლის რეოლოგიურ თვისებებთან
მეორადი კი ანთების გამოო
ანუ წამყვანი აქ კლინიკაა.

თუ პიელონეფრიტის გამო განვითარდა პროცესი,
ესე იგი ეს მეორადი ნეკროზია
და მარტო იმ ფოტოთი ამის გარკვევა შეუძლებელია.

Posted by: Blind_Torture_Kill 11 Dec 2008, 20:19
basa-ttt

ოკ

მე არც ვამბობ ზუსტი მიზეზი დამსახელემეთქი მაგის
სავარაუდოდ რას შეიძლება გამოეწვიათქო



Posted by: LUKA-BRAZI 11 Dec 2008, 20:44
Blind_Torture_Kill
მე ვიცი, მე ვიცი! biggrin.gif პაპილარული ნეკროზი ხშირად აღინიშნება შაქრიანი დიაბეტის დროს.... თუ არა? spy.gif მოიცა, მოცა.... არა ჰო, ეგრე უნდა იყოს,,,, დიაბეტს ახასიათებს თუ არ ვცდები პაპილარული ნეკროზები ....
smile.gif

Posted by: Blind_Torture_Kill 11 Dec 2008, 20:55
LUKA-BRAZI

კი

და რატომ მაინც და მაინც დიაბეტი და არა ანალგეტიკებით გამოწვეული პაპილარული ნეკროზი ?

Posted by: LUKA-BRAZI 11 Dec 2008, 21:54
ზუსტად არ ვიცი, ალბათ დიაბეტური მიკრო და მაკრო ანგიოპათიების გამო....

Posted by: Blind_Torture_Kill 11 Dec 2008, 22:02
LUKA-BRAZI

სურათით მინდა მიპასუხო

Posted by: vano_t 12 Dec 2008, 02:12
QUOTE
სურათით მინდა მიპასუხო

http://emedicine.medscape.com/article/379762-overview, ანალგეტიკებით გამოწვეული პაპილარული ნეკროზისათვის უფრო დამახასიათებელია კალციფიკაციები, მაგრამ მარტო ჰემოგლობინოპათიებთან არის შედარებული. სხვა შემთხვევები (მაგალითად დიაბეტის დროს, ციროზის დროს, დეჰიდრატაციის დროს და ა.შ.) არ არის ნახსენები.

http://www.pathologyoutlines.com/kidney.html, დიაბეტის დროსაც იგივე ცვლილებები ხდება.

სხვა წყაროებიც ვნახე. სპეციფიური არაფერია დამახასიათებელი. ეს კალციფიკაციები უფრო ხშირად შეიძლება იყოს სხვა შემთხვევებთან შედარებით. ეგ არის და ეგ.

******************************************************************************************************

ახალი შემთხვევა

55 წლის მექანიკი მოდის 1 კვირის ჩივილებით: კუნთების სპაზმები აქვს (ჩვენ რომ ვეძახით ძარღვის განასკვას ისეთი; ამერიკელები ეძახიან მოკლედ charley horses-ამ ჩივილებით მოკლედ). სიმპტომები დაეწყო 1 კვირის წინ. ადრე კი ქონდა ასეთი სიმპტომები ხანდახან, მაგრამ ეს ერთი კვირაა ძალიან ინტენსიური სპაზმები აქვს და ძალიან ხშირი. ასევე, სპაზმები მოიცავს ზემო და ქვემო კიდურების და ზემო კიდურების სარტყლის კუნთების. ასევე აქვს კუნთების ყრუ ტკივილები.

სიმტპომების დაწყებამდე 1 კვირით ადრე ეპიდურული ინექცია გაუკეთდება წელის ტკივილის გამო. წელის ტკივილები (რადიკულური ტიპის) დაეწყო 1 თვის წინ და მაგნიტური რეზონანსით დაუდგინდა მწვავე დისკის თიაქარი. ეპიდურული ინექცია 2 ჯერ გაუკეთდა და შვება იგრძნო.

გარდა კუნთების ტკივილის და სპაზმებისა, ავადმყოფს აღენიშნება მცირედი თავისტკივილები, ყელის ტკივილი და მშრალი ხველა.

წარსულში გადატანილი აქვს ც ჰეპატიტი (ნემსების გაზიარების გამო).

ასევე, გააჩნია ზოგადი შფოთვა (თუ როგორც არის ქართულად generalized anxiety disorder).

ქირურგიული ისტორია უარყოფითია.

ალერგიები არ აქვს.

სვამს მარტო ეფექსორს (effexor) 2 წელიa. ეფექსორი გენერიული ვენფლაქსინია (სეროტონინის "უკუ-შეწოვის" სელექტიური ინჰიბიტორი).

სვამს მციდრედს მხოლოდ. არ ეწევა. უარყოფს ნარკოტიკული საშუალებების გამოყენებას.

გამოკვლევისას, წნევა, ტემპერატურა, სუნთქვის სიხშირე ნორმალური აქვს. პულსი ცოტათი ტაქიკარდიული (105) და რეგულარული. კარგი ავსების და სიმეტრიული (რადიალური არტერიების პულსები იყო გასინჯული მხოლოდ). ნევროლოგიური გამოკვლვევა ნორმალურია. სახის კუნთების სისუსტე არ აქვს. თვალების მოძრაობა ნორმალური. ზემო და ქვემო კიდურების ძალა და რეფლექსები სიმეტრიულია და ნორმაშია. სიარული ნორმაშია. კუნთების ატროფია, ფასციკულაცია არ არის. დაბეჟილობა არ აღენიშნება. გულსისხლძარღვთა, სასუნთქი სისტემა გამოკვლევით ნორმალური. კანი ნორმალური. პირის ღრუ, ყურები ნორმალური. მუცელი რბილი.

ლაბორატორიებიდან: BUN, Cretinine, Na, K, Cl, HCO3, CBC ნორმალურია. ღვიძლის ენზიემები ცოტათი აწეულია (ძალიან მცირედ).

რას გაუკეთებთ ავამდყოფს. დამატებითი გამოკვლევებიც ჩაუტარდა, მაგრამ მაგას მოთხოვნისდამიხედვით დავდებ. როგორია ზოგადად ამ ავდმყოფის გამოკვლევის და მკურნალობის სტრატეგია?

Posted by: karaoke 12 Dec 2008, 03:26
vano_t
QUOTE
წარსულში გადატანილი აქვს ც ჰეპატიტი

ამდენი რამე თუ მარტო ამ დაავადებასტან გამოვა ასოცირებული თოკი მქონია საძებნი.
ბოდიშით რო შემოგეჭერით ხვალ ვნახავ ერთი პასუხს.

Posted by: Blind_Torture_Kill 12 Dec 2008, 07:30
QUOTE
სვამს მარტო ეფექსორს (effexor) 2 წელიa. ეფექსორი გენერიული ვენფლაქსინია (სეროტონინის "უკუ-შეწოვის" სელექტიური ინჰიბიტორი).


ამ წამალს შევწყვეტ და დავაკვირდები

Posted by: vano_t 12 Dec 2008, 09:09
QUOTE (Blind_Torture_Kill @ 12 Dec 2008, 07:30 )
ამ წამალს შევწყვეტ და დავაკვირდები

შესაძლებელია ეგრეც რომ ქნა, მარა ამ წამალს 2 წელი იღებს და არაფერი გვერდითი მოვლენ არ ქონდა. სამაგიეროდ მწვავედ დაეწყო ეს ყველაფერი.

მეორე პრობლემა (და უფრო მნიშვნელოვანი) ის არის, რომ სხვა რამ რომ იყოს (სერიოზული რამ) და დიაგნოზი არ დასვა დროზე, მაშინ ავადმყოფს შეიძლება სერიოზული პრობლემა შეექმნას.

Posted by: Blind_Torture_Kill 12 Dec 2008, 09:53
vano_t

CT მანახე ამ კაცის

Posted by: basa-ttt 12 Dec 2008, 09:55
კოაგულოგრამაში თუა რამე ცლილებები -
კუნთების სპაზმებს ზოგჯერ იძლევა მომატებული კოაგულაცია -
მაგ - ორსულებში ბოლო თვეებში ეს ჩვეულებრივი მოვლენაა და
ამ დროს შემცირებულია ხოლმე შედედების დრო.



QUOTE
Na, K, Cl, HCO3, CBC ნორმალურია

კალციუმი?
არ გაკეთებულა?
ანუ პარათირეოდული ჯირკვლის გამოკვლევა არ იქნებოდა ურიგო.

Posted by: vano_t 12 Dec 2008, 11:33
Blind_Torture_Kill
QUOTE
CT მანახე ამ კაცის

CT არ გაკეთებულა. რა CT გინდა ისე და რაში დაგეხმარება?

basa-ttt
QUOTE
კალციუმი?
არ გაკეთებულა?
ანუ პარათირეოდული ჯირკვლის გამოკვლევა არ იქნებოდა ურიგო.
კალციუმი ნორმაშია.
მაინც გამოიკლვლევ პარათირეოიდულ ჯირკვალს?

QUOTE
კოაგულოგრამაში თუა რამე ცლილებები -
კუნთების სპაზმებს ზოგჯერ იძლევა მომატებული კოაგულაცია -
მაგ - ორსულებში ბოლო თვეებში ეს ჩვეულებრივი მოვლენაა და
ამ დროს შემცირებულია ხოლმე შედედების დრო.
ამაზე ადრე ვისაუბრეთ. არ არის ასეთი რამ უცხოურ ლიტერატურაში (ყოველ შემთხვევაში აშშ-ს მედიცინაში-სხვაზე თავს ვერ დავდებ) აღწერილი, რომ პროთრომბინის დროის (ან ნაწილობრივი თრომბოპლასტინის დროის) შემცირება კლინიკურად რაიმე ღირებულებას წარმოადგენდეს. ასეა საქართველოს საზღვრებს გარეთ და უნდა შევეგუოთ ამ ფაქტს მიუხედავად იმისა მართალია თუ არა ეს ყველაფერი.

მოკლედ, აქ რომ ტესტს უკვეთავ, თუ დასაბუთებული ჩვენება არ არის, იმ ტესტში დაზღვევა არ გადაიხდის ფულს და ავადმყოფს უწევს ჯიბიდან გადახდა. ეს კიდევ არავის მოწონს, განსაკუთრებით მაშინ, როცა ტესტი არ ითვლება ჩვენებად ადგილობრივი მედიცინის მიხედვით.

ისე ასეთ ავადმყოფს შეიძლება დაჭირდეს კოაგულოგრამა, ოღონდ არა სპაზმების დიაგნოზის დასასმელად, არამედ შესაძლო დიაგნოზის გართულების გამოსარიცხად. ამ მიზეზით დაენიშნა ამ ავადმყოფს კოაგულოგრამა (რაც ნორმაში იყო), ოღონდ კოაგულოგრამა შევუკვეთეთ მას მერე რაც დიაგნოზი დაისვა.

Posted by: LUKA-BRAZI 12 Dec 2008, 14:42
vano_t
როგორი ქეისია იცი? თითქოს საკმარისი მონაცემები გაქვს, მაგრამ მაინც არ გყოფნის.... smile.gif თანაც რაც არ უნდა იყოს, ეს მისი კლასიკური კლინიკა არ არის ალბათ.... ჩემი 10 დღიანი ნევროლოგიით შორს ვერ წავალ, მაგრამ რატომღაც გაფანტული სკლეროზის სპინალური ფორმა ამომიტივტივდა თავში smile.gif ასევე სირინგომიელია.... რატომ? იმიტომ რომ ეს სპაზმები და ტკივილები შესაძლოა ორივეს აზასიათებდეს, დასაწყის სტადიაში მაინც.... მერე პარეზები და პარესთეზიები.... ისე სავსებით შესაძლებელია აქ ნერვ. სისტემა დამნაშავე არც კი იყოს.... ანუ როგორც ვთქვი, სიმპტომები გაქვს მაგრამ ბევრს არაფერს გეუბნება smile.gif ან შეიძლება მე არაფერს მეუბნება user.gif

Posted by: karaoke 12 Dec 2008, 16:45
QUOTE
საუკუნის კატასტროფები: 1. ჩერნობილის აფეთქება 2. ატომური ბომბის ჩამოგდება იაპონიაში 3. განათლების რეფორმა საქართველოში. ვინმე არ მეთანხმება?! smile.gif

ჯერ ყველაფერი წინ გვაქვს.

Posted by: badu 12 Dec 2008, 17:47
ეპიდურალისთვის რა წამალი იხმარეს?
ისე, blood cultures-ზე უარს არ ვიტყოდი

Posted by: LUKA-BRAZI 12 Dec 2008, 18:14
karaoke
QUOTE
ჯერ ყველაფერი წინ გვაქვს

მე უფრო მალე გავხდები ალბათ CERN-ის პრეზიდენტი, ვიდრე ამათ ხელში რამე გვეშველება user.gif
ისე, იმედია მასე იქნება biggrin.gif
badu
QUOTE
ისე, blood cultures-ზე უარს არ ვიტყოდი

yes.gif და სისხლის საერთოც კარგი იქნებოდა biggrin.gif

Posted by: basa-ttt 12 Dec 2008, 19:47
QUOTE
კალციუმი ნორმაშია.
მაინც გამოიკლვლევ პარათირეოიდულ ჯირკვალს?

გააჩნია რისი საშუალება გვაქვს.
აქ საქართველოში ეს ოცნების სფეროდანაა..
მანდ რა შეგვიძლია?
სადღაც მოვკარი თვალი პარათირეოდული ჯირკვლების სიმსივნესაც შეუძლია ასეთი სპაზმები გამოიწვიოს.
რომელი კალციუმი განსაზღვრეთ?
საერთო თუ იონიზირებული?
ძვლებში როგორ აკონტროლებთ კალციუმს - რენტგენით?

ამას ჰგავს - თუმცა მეუბნები,რომ კაცლიუმი ნორმაშიაო
Гипопаратиреоз. При разрушении паращитовидных желез вследствие патологического процесса или после их хирургического удаления возникает гипопаратиреоз – дефицит паратиреоидного гормона. Уровень кальция в крови при этом падает, а содержание фосфора нарастает. Для нормального функционирования тканей, в первую очередь нервной и мышечной, необходим стабильный, нормальный уровень кальция в крови. Его снижение при гипопаратиреозе вызывает приступы повышенной активности нервов и мышц, приводя к тетании – состоянию, характеризующемуся мышечными судорогами в руках и ногах, ощущением покалывания, тревогой и страхом. Основным средством лечения гипопаратиреоза в настоящее время является витамин D, большие дозы которого нормализуют концентрацию кальция в крови.

Изредка встречается псевдогипопаратиреоз – заболевание, обусловленное нечувствительностью костей и почек к действию паратиреоидного гормона. Оно тоже приводит к тетании, казалось бы указывающей на гипопаратиреоз, но все четыре паращитовидные железы в этом случае оказываются нормальными.


QUOTE
გარდა კუნთების ტკივილის და სპაზმებისა, ავადმყოფს აღენიშნება მცირედი თავისტკივილები, ყელის ტკივილი და მშრალი ხველა

ასეთ სიმპტომებზე ჩვენ დავნიშნავდით კუპრუმ მეტალიკუმს-
ანუ სპილენძის პრეპარატს.
პროფესიით მექანიკოსია.
სპილენძთან რამე შეხება ხომ არ ექნებოდა?

Posted by: vano_t 12 Dec 2008, 20:50
LUKA-BRAZI
QUOTE
როგორი ქეისია იცი? თითქოს საკმარისი მონაცემები გაქვს, მაგრამ მაინც არ გყოფნის.... smile.gif თანაც რაც არ უნდა იყოს, ეს მისი კლასიკური კლინიკა არ არის ალბათ.... ჩემი 10 დღიანი ნევროლოგიით შორს ვერ წავალ, მაგრამ რატომღაც გაფანტული სკლეროზის სპინალური ფორმა ამომიტივტივდა თავში smile.gif ასევე სირინგომიელია.... რატომ? იმიტომ რომ ეს სპაზმები და ტკივილები შესაძლოა ორივეს აზასიათებდეს, დასაწყის სტადიაში მაინც.... მერე პარეზები და პარესთეზიები.... ისე სავსებით შესაძლებელია აქ ნერვ. სისტემა დამნაშავე არც კი იყოს.... ანუ როგორც ვთქვი, სიმპტომები გაქვს მაგრამ ბევრს არაფერს გეუბნება smile.gif ან შეიძლება მე არაფერს მეუბნება user.gif

ქრონიკულ დაავადებებზე რატომ იფიქრებ მწვავე პრეზენტაციის დროს? შეძლებით ყველაფერია შესაძლებელი, მარა multiple სკლეროზის დიაგნოზს ქრონიკული სიმპტომები ჭირდება, თანაც სხვადასხვა ნევროლოგიური გამოვლინება. თუ მწვავე დაავადებების დიაგნოზები გამორიცხე და ავადმყოფის სიმპტომები გრძელდება, მერე ალბათ მაგენიც უნდა იფიქრო. ახლა ადრეა ჩემი აზრით.

QUOTE
და სისხლის საერთოც კარგი იქნებოდა
სისხლის საერთო რა არის? ანალიზები ხომ ჩამოვთვალე: CBC (რაშიც შედის ერითროციტები, ლეიკოციტები, თრომბოციტები, ჰემოგლობინი, ჯემატიკრიტი), Creatinine, BUN (შარდოვანა), electrolites (კალციუმი, ნატრიუმი და ა.შ.), მაგნიუმი და ფოსფატებიც ნორმაში იყო.

badu
QUOTE
ეპიდურალისთვის რა წამალი იხმარეს?
ისე, blood cultures-ზე უარს არ ვიტყოდი
რომელიღაც სტეროიდი. ეგ კლინიკას რომელს ხმარობს ზუსტად არ ვიცი.

სისხლი არ დათესილა: სიცხეები არ აქვს ავადმყოფს; ტოქსიურად არ გამოიყურება და ინფექციის ყველაზე ხშირი წყაროები (ფილტვი და სასარდე გზები) ნორმალურია გამოკვლევით და ანამნეზით. ამ ეტაპზე არ გაკეთებულა. დიაგნოზი დაისვა კლინიკაში მოსვლის დღეს და მკურნალობაც ჩაუტარდა. სისხლს დავთესავდით ალბათ მომავალში, თუ დიაგნოზი ვერ დაიმებოდა.

basa-ttt
QUOTE

გააჩნია რისი საშუალება გვაქვს.
აქ საქართველოში ეს ოცნების სფეროდანაა..
მანდ რა შეგვიძლია?
სადღაც მოვკარი თვალი პარათირეოდული ჯირკვლების სიმსივნესაც შეუძლია ასეთი სპაზმები გამოიწვიოს.
რომელი კალციუმი განსაზღვრეთ?
საერთო თუ იონიზირებული?
ძვლებში როგორ აკონტროლებთ კალციუმს - რენტგენით?
კალციუმი თუ ნორმაშია პარათირეოიდული ჰორმონის გამოკვლევა აღარ გჭირდება. ამ ჰორმონის ნაკლებობა/მეტობა კალციუმის მეტაბოლიზმს არღვევს.

მთლიანი კალციუმი გაესინჯა (რაც იყო 9.8 მგ/დლ) და ალბუმინები, რაც იყო (4.0 გ/ლ). ეს ორი ტესტი მოგცემს საკმაოდ კარგ წარმოდგენას იონიზირებული კალციუმის რაოდენობაზე, რაც დამოკიდებულია ალბუმინების შემცველობაზე სისხლში. თუ არ ვცდები, ალბუმინის 1 გრამით დაქვეითება იწვევს კალციუმის 1.8 მგ/დლ-ით შემცირებას. მოკლედ, ამ მონაცემებზე დაყრდნობით, იონიზირებული კალციუმი ნორმაში უნდა იყოს. ისე იონიზირებული კალციუმის გასინჯვა გაცილებით უკეთესია, მაგრამ ყველა ლაბორატორია არ აკეთებს და თუ სასწრაფოდ გჭირდება კალციუმი, არაპირდაპირი მეთოდის გარდა სხვა რამ არ გაგაჩნია.

ძლებში კალციუმის კონტროლში რას გულისხმობ?

მეტს არაფერს შეუკვეთავთ? სისხლის ანალიზი რომელიმე არ შეიძლება რომ დაგვეხმაროს კუნთების ტკივილის და სპაზმის დროს?

Posted by: Cousteau 12 Dec 2008, 22:20
QUOTE (vano_t @ 12 Dec 2008, 20:50 )

მეტს არაფერს შეუკვეთავთ? სისხლის ანალიზი რომელიმე არ შეიძლება რომ დაგვეხმაროს კუნთების ტკივილის და სპაზმის დროს?

CPK მაღალია?
შარდის ანალიზი ნორმაა?
მიოგლობინურია ხო არ აქ? : /

სპაზმები სპონტანურია?

QUOTE
(რადიალური არტერიების პულსები იყო გასინჯული მხოლოდ)

ბარებ ფემორალურისაც თქვი
user.gif

Posted by: basa-ttt 12 Dec 2008, 22:53
QUOTE
მეტს არაფერს შეუკვეთავთ? სისხლის ანალიზი რომელიმე არ შეიძლება რომ დაგვეხმაროს კუნთების ტკივილის და სპაზმის დროს?

სპილენძი სისხლში თუ მისი სამუშაო დაკავშირებულია მეტალებთან

Posted by: vano_t 12 Dec 2008, 23:15
Cousteau
QUOTE
CPK      მაღალია?
შარდის ანალიზი ნორმაა?
მიოგლობინურია ხო არ აქ?

CPK 4000 (ნორმა არის 170-ზე ნაკლები)
შარდში: მკვეთრად დადებითი რეაქცია სისხლზე, ოღონდ RBC (ერითროციტები) არ აღინიშნება მიკროსკოპიულად.

ახლა ქე უნდა დასვათ დიაგნოზი; მერე შესაძლო მიზეზი და რაც მთავარია როგორ მკურნალობ ამ ავადმყოფს; რა გართულებები გაქვს მხედველობაში და რა ლაბორატორიებს უკეთებ ამის დასადგენად?

Posted by: Cousteau 12 Dec 2008, 23:22
QUOTE (vano_t @ 12 Dec 2008, 23:15 )
Cousteau
QUOTE
CPK      მაღალია?
შარდის ანალიზი ნორმაა?
მიოგლობინურია ხო არ აქ?

CPK 4000 (ნორმა არის 170-ზე ნაკლები)
შარდში: მკვეთრად დადებითი რეაქცია სისხლზე, ოღონდ RBC (ერითროციტები) არ აღინიშნება მიკროსკოპიულად.

ახლა ქე უნდა დასვათ დიაგნოზი; მერე შესაძლო მიზეზი და რაც მთავარია როგორ მკურნალობ ამ ავადმყოფს; რა გართულებები გაქვს მხედველობაში და რა ლაბორატორიებს უკეთებ ამის დასადგენად?

რაბდომიოლიზიაქ?
თვითონ დიაგნოზი : / hmm
თირკმლის უკმარისობა ექნება მაგას გართულებად და ეგ უნდა აკონტროლო სისხლის და შარდის ანალიზით?

QUOTE
მკვეთრად დადებითი რეაქცია სისხლზე, ოღონდ RBC (ერითროციტები) არ აღინიშნება მიკროსკოპიულად.

ამაში განმანათლე რა მერე ან რამე ლინკი მომე
user.gif

Posted by: Blind_Torture_Kill 12 Dec 2008, 23:49
ხ.ი.ზ სინდრომი ჭირს როგორც Cousteau იტყვის ხოლმე

იტოკში რამე მიოზიტი დაემართა და გაურთულდა

Posted by: Cousteau 12 Dec 2008, 23:56
ვანო ეს ის დროა როცა ამაზე პატარა ლექცია უნდა წაიკითხო, და ჩვენ უნდა მოვუსმინოთ.

თან არაჰემატურიულ ჰემატურიაზე თუ იტყვი ერთი-ორს კარგი იქნება : )

Posted by: Blind_Torture_Kill 12 Dec 2008, 23:57
Cousteau

სწორედ თქვი მიოგლობინურიის დროს გაქვს სისხლი მარა ერითროციტები არა

Posted by: vano_t 13 Dec 2008, 00:01
Cousteau
QUOTE
რაბდომიოლიზიაქ?
თვითონ დიაგნოზი : / hmm
თირკმლის უკმარისობა ექნება მაგას გართულებად და ეგ უნდა აკონტროლო სისხლის და შარდის ანალიზით?
ნამდვილად, რაბდომიოლიზი აქვს.

თირკმლის უკმარისობა არის მაგის ყველაზე ხშირი გართულება და თირკმლის ფუნქცია უნდა ამოწმო. ამ პაციენტს არ აქვს ეგ თუმცა. უკვე ვთქვი, რომ კრეატინინი და შარდოვანა ნორმალშია.

კიდევ, ასეთ ავადმყოფებს უვითარდებათ ხშირად დისემინირებული სისხლძარღვშიდა შედედება. ამიტომ, თრომბოციტები, PT, aPTT უნდა შეუმოწმდეს.

მკურნალობით, ამათ უნდა ბევრი სითხე და დაკვირვება. ავადმყოფი მივიღეთ საავადმყოფოში. დავიწყეთ NS (ნორმალური ოსმოლარობის ნატრიუმის ქლორიდი) და CPK ჩამოვიდა 400-ზე 2 დღეში. კუნთების სპაზმებიც გაუმჯობესდა და თითქმის აღარ აქვს. კლინიკაში მოდის ორშაბათს, რომ გადავუმოწმოთ CPK.

QUOTE
QUOTE
მკვეთრად დადებითი რეაქცია სისხლზე, ოღონდ RBC (ერითროციტები) არ აღინიშნება მიკროსკოპიულად.

ამაში განმანათლე რა მერე ან რამე ლინკი მომე
user.gif
ე.წ. dipstick არის შარდის სწრაფი ანალიზის ტესტი. ზედ აქვს სხვადასხვა ფერის ოთხკუთხედები და შარდში რომ ჩაყოფ, ფერს იცივლიან ეს ნაწილები. იმის მიხედვით თუ როგორია ფერის ცვალება ამა თუ იმ ნაერთზე, შეგიძლია იმსჯელო შარდში მაგ ნაერთის დაახლოებით რაოდნეობაზე. ამ დიპსტიკებს აქვს როგორც წესი ნაწილები, რომელიც ზომავენ ბილირუბინს, გლუკოზას, ნიტრიტებს, ცილას და სისხლს. ის ნაწილი რაც სისხლს ზომავს, რეაგირებს საერთოდ ჰემის ჯგუფზე. როცა მკვეთრად დადებითია ტესტი ამაზე და ერითროციტი არ არის შარდში, მაშინ რამოდენიმე რამეს ფიქრობ: 1) ცრუ დადებითი ტესტი ათასი მიზეზის გამო; 2) ჰემოლიზი, რაც მოგცემს ხანდახან ჰემოგლობინის გამოყოფას შარდში ერითროციტების გარეშე; 3) ნებისმიერი ცილა, რაც შეიცავს ჰემს მოგცემს დადებით პასუხს. მიოგლობინი შეიცავს ჰემს და ამიტომ გაძლევს დადებით პასუხს ერითროციტების გარეშე.

Posted by: Cousteau 13 Dec 2008, 00:02
QUOTE (Blind_Torture_Kill @ 12 Dec 2008, 23:57 )
Cousteau

სწორედ თქვი მიოგლობინურიის დროს გაქვს სისხლი მარა ერითროციტები არა

რიავი : /

ამას კუნთები ეშლება და რისგან არვიცი, ინფექცია რო ქონდეს სადმე რამე ექნებოდა, სისხლი ნორმააქ, კა ნორმააქ, ელექტრომიოგრაფიას ვკითხავდი მარა რომც დამიწეროს მაგისგან ნაკლებად მომეშვება.

ერთადერთი რაც ვიცი რო ეპილეფსიებს აქვთ მაღალი CPK მაგრამ ნევროლოგია ამას ნორმააქ : /

კსტატი forensic, ადამსში ეწერა რო კინაზის გაზომვა მარტივი მეთოდია სიმულანტი grand-mal-ის გამოვლენის, რავი შეიძლება იცოდე კიდეც...
მოკლედ პასუხს დაველოდები და წავუძინებ...

გარდიანი რატო აღარ შემოდის?


vano_t
ხომარა ეგ რაბდომიოლიზი რატომ აქ? ანუ დიაგნოზი რა არის?
user.gif

Posted by: vano_t 13 Dec 2008, 00:17
QUOTE (Cousteau @ 13 Dec 2008, 00:02 )
[QUOTE=Blind_Torture_Kill,12 Dec 2008, 23:57 ] Cousteau
ამას კუნთები ეშლება და რისგან არვიცი, ინფექცია რო ქონდეს სადმე რამე ექნებოდა, სისხლი ნორმააქ, კა ნორმააქ, ელექტრომიოგრაფიას ვკითხავდი მარა რომც დამიწეროს მაგისგან ნაკლებად მომეშვება.

ერთადერთი რაც ვიცი რო ეპილეფსიებს აქვთ მაღალი CPK მაგრამ ნევროლოგია ამას ნორმააქ : /

კსტატი forensic, ადამსში ეწერა რო კინაზის გაზომვა მარტივი მეთოდია სიმულანტი grand-mal-ის გამოვლენის, რავი შეიძლება იცოდე კიდეც...
მოკლედ პასუხს დაველოდები და წავუძინებ...

გარდიანი რატო აღარ შემოდის?


vano_t
ხომარა ეგ რაბდომიოლიზი რატომ აქ? ანუ დიაგნოზი რა არის?
user.gif

ეპილეფსიის დროს იმიტომ არის აწეული CPK რომ კუნთების "დაშლა" მიდის ისევე, როგორც სხვა პრომლემების დროს. რაბდო სხვადასხვა რამ შეიძლება გამოიწვიოს. ყველაზე ხშირია crush injury. უცებ რომ ფიზიკურ აქტიურობას დაიწყებ მაგანაც; ან პირიქით, აქტიურობით რომ სულ არ აქტიურობ იმან; სხვადასხვა ნარკოტიკებმა და ა.შ.

ამ შემთხვევაში სავარაუდო ვირუსული იყო (ზემო სასუნთქი გზები ინფექციის ნიშნები აქვ). სხვა მიზეზები გამოირიცხა და იმიტომ ვამბობ ამას.

ელექტორმიფოგრაფია არ უნდა ამ შემთხვევაში. ქრონიკული რომ იყოს, მაშინ სხვა საქმეა. მწვავეს დროს არაფერი არ გინდა, გარდა შესაბამისი მკურნალობისა და გამომწვევი მიზეზის დადგენისა.

Posted by: Cousteau 13 Dec 2008, 00:19
ოკეი პონიატნო, ვნახე ვიკიში ეს და გადავიკითხავ.
ისე ეგეთი კეისები თქვენთან რუტინად ითვლება?
აქ რო დაგემართოს ალბათ მაგარ ცუდ გაუგებრობაში მოხვდები : /

Posted by: vano_t 13 Dec 2008, 00:29
QUOTE (Cousteau @ 13 Dec 2008, 00:19 )
ოკეი პონიატნო, ვნახე ვიკიში ეს და გადავიკითხავ.
ისე ეგეთი კეისები თქვენთან რუტინად ითვლება?
აქ რო დაგემართოს ალბათ მაგარ ცუდ გაუგებრობაში მოხვდები : /

რუტინულად არ ითვლება, მაგრამ წელიწადში 2-3 ასეთ ავამყოფს მაინც ნახავ ალბათ. გარდა ამისა, მიოპათიებს ნახავ და CPK ასეთ ავადმყოფებში ყოველთვის უნდა გაკეთდეს. მარტივი ტესტია (საქართველოშიც აკეთებენ მაგას-ტროპონინთან ერთად) და სერიოზული დაავადების დაიგნოზი შეუძლია. შესაბამისად, ავადმყოფსაც ეშველება.

Posted by: basa-ttt 13 Dec 2008, 22:27
გადავხედე რაბდომიოლიზის კლინიკას -
http://www.med74.ru/infoitem1237.html
http://max.1gb.ru/neurology/neuro016a.shtml
http://www.nedug.ru/library/doc.aspx?item=55623

არსად არ ფიგურირებს კუნთის სპაზმები და ე.წ. "გადანასკვა"
კუნთების ტკივილი და სისუსტე კი ბატონო..
მაგრამ არა კრთომები.

თან ახასიათებს სისხლში კალიუმის და ფოსფორის მომატება -
რასაც არ წერდი.
არ დაწერე ასევე შარდში ცვლილებებზე -
ანუ მიოგლობინის არსებობაზე
Повышение уровней калия и фосфора вследствие повреждения мышц
Гипокальциемия в период олигурии служит полезным диагностическим признаком, но не даёт возможности установить диагноз острого некроза скелетных мышц
Высокие гиперкалиемия, гиперфосфатемия и гиперурикемия - весомый аргумент в пользу острого некроза скелетных мышц

არადა შარდის ანალიზი სისხლის საერთოსთან ერთად აუცილებელ გამოკვლევად ითვლება.
ამ დროს გვეუბნები - დამატებით კიდევ რას გამოიკვლევო-
იქმნებოდა შთაბეჭდილება,
რომ ეს სტანდარტული გამოკვლევები უკვე გაკეთებული იყო.



წერდი, რომ
QUOTE
ზემო და ქვემო კიდურების ძალა და რეფლექსები სიმეტრიულია და ნორმაშია.

მაშინ როცა ამ დროს სწორედ კუნთების სისუსტეა დამახასიათებელი

ინტ. ქსელში ხომ არ გეგულება დამატებითი ინფორმაცია?

QUOTE
ამ შემთხვევაში სავარაუდო ვირუსული იყო (ზემო სასუნთქი გზები ინფექციის ნიშნები აქვ). სხვა მიზეზები გამოირიცხა და იმიტომ ვამბობ ამას

მემკვიდრეობითი პათოლოგია?
ეგ როგორ გამორიცხეთ?

Posted by: Cousteau 13 Dec 2008, 22:41
QUOTE (basa-ttt @ 13 Dec 2008, 22:27 )

მემკვიდრეობითი პათოლოგია?
ეგ როგორ გამორიცხეთ?

მემკვიდრეობით smile.gif

Posted by: vano_t 14 Dec 2008, 01:41
basa-ttt
QUOTE
თან ახასიათებს სისხლში კალიუმის და ფოსფორის მომატება -
რასაც  არ წერდი.
არ დაწერე ასევე შარდში ცვლილებებზე -
ანუ მიოგლობინის არსებობაზე
პაციენტების დიდ ნაწილს არ აქვს არც ჰიპერკალემია (შეიძლება ჰიპოკალემიაც ქონდეთ, რამაც თავისთავად შეიძლება გამოიწვიოს რაბდომიოლიზი), არც ჰიპერფოსფატემია და არც მნიშვნელოვანი მიოგლობინურია. თუმცა, ამ ლაბორატორიებზე ყველაფერი ვთქვი მაინც:
QUOTE
ლაბორატორიებიდან: BUN, Cretinine, Na, K, Cl, HCO3, CBC
K არის კალიუმი. ასე ჩაწერილ ლაბორატორიას თუ ვერ გაიგებ, მაშინ მკითხე და გიპასუხებ რა რომელია. ფოსფორის აწევა/ნორმალურობას კლინიკური ღირებულება არ აქვს რაბდომიოლიზის დროს (არ მკურნალობ აწეულიც რომ იყოს). ამ ავდმყოფს ნორმაში ქონდა.
QUOTE
შარდში: მკვეთრად დადებითი რეაქცია სისხლზე, ოღონდ RBC (ერითროციტები) არ აღინიშნება მიკროსკოპიულად.
ეს მიუთითებს მიოგლობინურიაზე. თუმცა, 50 %-მდე ავადმყოფს შეიძლება ეს ტესტი უარყოფითი ქონდეს. ასეთი რამ თავიდანვე არ დავწერე, იმიტომ, რომ ამის მერე აღარაფერია გამოსაცნობი და ზუსტად ის მაინტერესებდა რა ლაბორატორიებს შეუკვეთავდით ამ დროს. კუსტომ იკითხა და პრინციპში უკვე იცოდა დიაგნოზი. ამ შემთხვევის მიზანიც ეგ იყო, რომ კუნთოვანი სისტემის ჩივილის დროს CPK გაზომო და კუნთოვანი პათოლოგიები გვქონდეს მხედველობაში.

QUOTE
QUOTE
ამ შემთხვევაში სავარაუდო ვირუსული იყო (ზემო სასუნთქი გზები ინფექციის ნიშნები აქვ). სხვა მიზეზები გამოირიცხა და იმიტომ ვამბობ ამას

მემკვიდრეობითი პათოლოგია?
ეგ როგორ გამორიცხეთ?
50- წელს გადაცილებული ადამიანია. ის მემკვიდრეობითი დაავადებებ, რაც ქრონიკულ მიოპათიებს იწვევენ, პედიატრიულ ასაკში ხდება დიაგნოზი და უმეტესობს 30-40 წელზე მეტს ვერ ცოცხლობს. მეორე საკითხი ის არის, რომ ავადმყოფის ჰიდრატაციის შემდეგ CPK დაეცა და 2 დღეში თითქმის ნორმაში ჩაჯდა. მემკვიდრეობითი კუნთების დაავადებების დროს CPK ქრონიკულად იქნება აწეული რაღაც დონეზე ან განმეორებითი რაბდომიოლიზები ექნება. ასეთი დაავადების დიაგნოზი ბავშვობაში ხდება.

მოკლედ, როცა კრეატინფოსფოკინაზა ჰიდრატაციის შემდეგ ნორმას უბრუნდება, აღარ არის საჭირო ქრონიკული დაავადებების ძებნა. თუ აწეული დარჩა, მაშინ სხვა საქმეა. ამიტომაც უნდა ავადმყოფს შეუმოწმო ფოსფოკინაზა გაწერიდან რამოდენიმე ხანში. ეს ავადმყოფი დაბრუნდება მომავალ კვირას და თუ კიდევ ექნება აწეული ეს ენზიმი, მაშინ ქრონიკულ დაავადების ძებნა საჭირო იქნება ალბათ.

QUOTE

წერდი, რომ
QUOTE
ზემო და ქვემო კიდურების ძალა და რეფლექსები სიმეტრიულია და ნორმაშია.

მაშინ როცა ამ დროს სწორედ კუნთების სისუსტეა დამახასიათებელი
სისუსტე დამახასიათებელია, მაგრამ არ არის აუცილებელი და ბევრს შეიძლება არ ქონდეს. კუნთების პრობლემა რეფლექსებზე არ ახდენს გავლენას როგორც წესი. ამ დაავადებას, როგორც ყველა სხვა დაავადებას, აქვს მნიშვნელოვანი ვარიაცია პრეზენტაციის დროს.

QUOTE
ინტ. ქსელში ხომ  არ გეგულება დამატებითი ინფორმაცია?
http://emedicine.medscape.com/article/1007814-overview
http://emedicine.medscape.com/article/982711-overview

Posted by: texasuri jleta benzoxerxit 14 Dec 2008, 02:09
QUOTE (Blind_Torture_Kill @ 12 Dec 2008, 07:30 )
QUOTE
სვამს მარტო ეფექსორს (effexor) 2 წელიa. ეფექსორი გენერიული ვენფლაქსინია (სეროტონინის "უკუ-შეწოვის" სელექტიური ინჰიბიტორი).


ამ წამალს შევწყვეტ და დავაკვირდები

და ძალიანაც დააშავებ. ეფექსორი სეროტონინის და ნორადრენალინის უკუშეწოვის ინჰიბიციის უნარის მქონე ანტიდეპრესანტია.და არ მგონია მკურნალობიდან მეორე წლის თავზე მოგცეს მსგავსი გვერდითი ეფექტები boli.gif


ძალიან საინტერესო ქეისი იყო.ეგ არის ტერაპია და მედიცინა,და არა მშრალი ჰისტოლოგიური სურათების დადება და ტავის მტვრევა,თუ რა არის ეს!

Posted by: basa-ttt 14 Dec 2008, 10:55
QUOTE
ლაბორატორიებიდან: BUN, Cretinine, Na, K, Cl, HCO3, CBC

K არის კალიუმი. ასე ჩაწერილ ლაბორატორიას თუ ვერ გაიგებ, მაშინ მკითხე და გიპასუხებ რა რომელია. ფოსფორის აწევა/ნორმალურობას კლინიკური ღირებულება არ აქვს რაბდომიოლიზის დროს (არ მკურნალობ აწეულიც რომ იყოს). ამ ავდმყოფს ნორმაში ქონდა.

რაის ვერ გავიგე -
ძალიან კარგად გავიგე -
და იმიტომაც დავწერე კალიუმზე.
იხ წინა პოსტი.

დანარჩენი გასაგებია.

Posted by: Blind_Torture_Kill 14 Dec 2008, 12:15
QUOTE
55 წლის მექანიკი მოდის 1 კვირის ჩივილებით: კუნთების სპაზმები აქვს (ჩვენ რომ ვეძახით ძარღვის განასკვას ისეთი; ამერიკელები ეძახიან მოკლედ charley horses-ამ ჩივილებით მოკლედ). სიმპტომები დაეწყო 1 კვირის წინ. ადრე კი ქონდა ასეთი სიმპტომები ხანდახან, მაგრამ ეს ერთი კვირაა ძალიან ინტენსიური სპაზმები აქვს და ძალიან ხშირი. ასევე, სპაზმები მოიცავს ზემო და ქვემო კიდურების და ზემო კიდურების სარტყლის კუნთების. ასევე აქვს კუნთების ყრუ ტკივილები.

სიმტპომების დაწყებამდე 1 კვირით ადრე ეპიდურული ინექცია გაუკეთდება წელის ტკივილის გამო. წელის ტკივილები (რადიკულური ტიპის) დაეწყო 1 თვის წინ და მაგნიტური რეზონანსით დაუდგინდა მწვავე დისკის თიაქარი. ეპიდურული ინექცია 2 ჯერ გაუკეთდა და შვება იგრძნო.

გარდა კუნთების ტკივილის და სპაზმებისა, ავადმყოფს აღენიშნება მცირედი თავისტკივილები, ყელის ტკივილი და მშრალი ხველა.

წარსულში გადატანილი აქვს ც ჰეპატიტი (ნემსების გაზიარების გამო).

ასევე, გააჩნია ზოგადი შფოთვა (თუ როგორც არის ქართულად generalized anxiety disorder).


ქირურგიული ისტორია უარყოფითია.

ალერგიები არ აქვს.

სვამს მარტო ეფექსორს (effexor) 2 წელიa. ეფექსორი გენერიული ვენფლაქსინია (სეროტონინის "უკუ-შეწოვის" სელექტიური ინჰიბიტორი).

სვამს მციდრედს მხოლოდ. არ ეწევა. უარყოფს ნარკოტიკული საშუალებების გამოყენებას.

გამოკვლევისას, წნევა, ტემპერატურა, სუნთქვის სიხშირე ნორმალური აქვს. პულსი ცოტათი ტაქიკარდიული (105) და რეგულარული. კარგი ავსების და სიმეტრიული (რადიალური არტერიების პულსები იყო გასინჯული მხოლოდ). ნევროლოგიური გამოკვლვევა ნორმალურია. სახის კუნთების სისუსტე არ აქვს. თვალების მოძრაობა ნორმალური. ზემო და ქვემო კიდურების ძალა და რეფლექსები სიმეტრიულია და ნორმაშია. სიარული ნორმაშია. კუნთების ატროფია, ფასციკულაცია არ არის. დაბეჟილობა არ აღენიშნება. გულსისხლძარღვთა, სასუნთქი სისტემა გამოკვლევით ნორმალური. კანი ნორმალური. პირის ღრუ, ყურები ნორმალური. მუცელი რბილი.

ლაბორატორიებიდან: BUN, Cretinine, Na, K, Cl, HCO3, CBC ნორმალურია. ღვიძლის ენზიემები ცოტათი აწეულია (ძალიან მცირედ).

რას გაუკეთებთ ავამდყოფს. დამატებითი გამოკვლევებიც ჩაუტარდა, მაგრამ მაგას მოთხოვნისდამიხედვით დავდებ. როგორია ზოგადად ამ ავდმყოფის გამოკვლევის და მკურნალობის სტრატეგია?


წამლის შეწყვეტა ვიფიქრე იმიტომ რომ მაგ წამლის გვერდით მოვლენებს გავდა სიმპტომები - შეიძლება გადამეტებული დოზირებით დაიწყო სმა

ხველამ , თავის ტკივილმა ყელის ტკივილმა და ადრე ც ჰეპატიტმა საერთოდ არ მაფიქრებინა მიოზიტი
სხვა დაავადებების ძიება დავიწყე

Posted by: Cousteau 14 Dec 2008, 12:43
სანამ რამე ჩახლართულს დადებენ მანამდე ადვილი:

user posted image

რას შვება ეს ბავშვი?
რის დროს არის ყველაზე ხშირად?
+ ცოტა რო იწვალოთ
ვისი სახელი ქვია?
ის კლასიკური სიმპტომი დაწერეთ რომელიც ამას ხშირად აქვს
smile.gif

Posted by: LUKA-BRAZI 14 Dec 2008, 12:51
Cousteau
თუ არ ვცდები ეს უნდა იყოს კუნთთა პროგრესირებადი დისტროფია, მიოპათია, რომელსაც ახასიათებს კუნთების პსევდოჰიპერტროფია, ცხიმოვანი ქსოვილის გამრავლების გამო კუნთში, სინამდვილეში კი კუნთების დისტროფია და ძალის დაქვეითება. ახასიათებს იატაკიდან ე.წ. "რიტუალური", "კასკადური" ადგომა. თავის თავზე დაყრდნობით, ასევე ახასიათებს "იხვისებრი სიარული", "ბაყაყის მუცელი" და ა.შ. კუნთების ტონუსის დაქვეითების და ატროფიის გამო.... ისი სახელი ქვია, ეს ნამდვილად არ ვიცი smile.gif

Posted by: Cousteau 14 Dec 2008, 12:52
QUOTE (LUKA-BRAZI @ 14 Dec 2008, 12:51 )
ისი სახელი ქვია, ეს ნამდვილად არ ვიცი smile.gif

wikipedia.org wink.gif

QUOTE
თუ არ ვცდები ეს უნდა იყოს კუნთთა პროგრესირებადი დისტროფია, მიოპათია, რომელსაც ახასიათებს კუნთების პსევდოჰიპერტროფია

ამ დისტროფიის სახელს თუ ჩაწერ ამოგიგდებს fig.gif

Posted by: LUKA-BRAZI 14 Dec 2008, 12:55
Cousteau
აააააააააააააააააააა!!!!!!!!!!!!! ჰოოოოოოოოო, დიუშენის ფსევდოჰიპერტროფიული მიოპათია!!!!!!!! ესაა ხო?! smile.gif

Posted by: Cousteau 14 Dec 2008, 12:56
QUOTE (LUKA-BRAZI @ 14 Dec 2008, 12:55 )
Cousteau
აააააააააააააააააააა!!!!!!!!!!!!! ჰოოოოოოოოო, დიუშენის ფსევდოჰიპერტროფიული მიოპათია!!!!!!!! ესაა ხო?! smile.gif

ეს დაავადების სახელი
სიმპტომს სხვა გვარიაქ boli.gif

კაი ხო smile.gif
გოვერის ნიშანია (gower's sign)

აგერ

Posted by: LUKA-BRAZI 14 Dec 2008, 12:58
ლანდაუს, ერბ-როტის, ლანდუზი-ბეჟერინის ვარიანტები, გააჩნია დაზიანებული კუნთების ჯგუფს.... smile.gif
* * *
Cousteau
QUOTE
გოვერის ნიშანია

spy.gif ჰოო? ნაღდად? ნე ზნალ.... smile.gif გოვერსი ნათხემში ექსტეროცეფციის შემტანი გზა არ არის?

Posted by: mtvareuli 14 Dec 2008, 15:17
ამ ბოვშვს რა ჭირს

Posted by: Blind_Torture_Kill 14 Dec 2008, 15:21
mtvareuli

QUOTE
ამ ბოვშვს რა ჭირს


გიგანტიზმი ?

Posted by: LUKA-BRAZI 14 Dec 2008, 15:32
Blind_Torture_Kill
რამე ჭირს რო საერთოდ? ბავშვია და იცინის, ცოტა არაბუნებრივად, თითებზე რაღაც წითელი კვანძებივით თუ ლაქებივით ჩანს და მგონი ინვალიდის სავარძელში უნდა იჯდეს user.gif ამ სურათზე მეტი რა უნდა დაინახო? spy.gif

Posted by: mtvareuli 14 Dec 2008, 15:35
Blind_Torture_Kill

არა biggrin.gif

ისე ცოტა ცუდი სურათი ამოვარჩიე, დაჟე პირიქითაა

თუმცა ორივე სურათზე არის ერთი ის ნიშანი, რითიც შეიძლება მიხვდეთ

Posted by: Blind_Torture_Kill 14 Dec 2008, 15:38
mtvareuli

ხელები მომეჩვენა დიდი boli.gif პირველ სურათში

მეორე სურათში რამე განსაკუთრებული უნდა ჩანდეს ?

Posted by: mtvareuli 14 Dec 2008, 15:42
LUKA-BRAZI
Blind_Torture_Kill

ხე ლე ბი

Posted by: Blind_Torture_Kill 14 Dec 2008, 15:46
mtvareuli


QUOTE
ხე ლე ბი


მარფანი ?

Posted by: mtvareuli 14 Dec 2008, 16:20
Blind_Torture_Kill

არა არა, ხელების მდებარეობას დააკვირდით

პაციენტები არიან გოგონები, რომლებიც 2-3 წლამდე ნორმალურად ვითარდებოდნენ, შემდეგ დაეწყოთ განვითარების პრობლემები
თავის ზრდა ჩამორჩება ნორმალურს, აქვთ მოდუნებული და ჰიპოტონური კიდურები;
აქვთ აუტიზმის მსგავსი სიმპტომები, აპრაქსია, უჭირთ მეტყველება, კოორდინირებული მოძრაობები; გონებრივად ჩამორჩენილები არიან;
შეიძლება ქონდეთ სუნთქვის პრობლემები, სქოლიოზი; სისხლის მიმოქცევის პრობლემები, შეიძლება ხელები და ფეხები ცივი და მოლურჯო ჰქონდეთ; ძილის დარღვევები; საჭმლის მომნელებელი ტრაქტის დარღვევები, შეკრულობა, გერ; ჭარბი სალივაცია.

მეტი რაღა ვთქვა

Posted by: Blind_Torture_Kill 14 Dec 2008, 16:33
Rett syndrome

ოღონდ გუგლიდან
არ ვიცოდი ნაღდად

Posted by: mtvareuli 14 Dec 2008, 16:46
Blind_Torture_Kill
QUOTE
Rett syndrome

ოღონდ გუგლიდან
არ ვიცოდი ნაღდად

ხო ეგაა

არც მე ვიცოდი, ანუ სამედიცინოში არ უსწავლებიათ არსად

მაგრამ ამ ბოლო დროს ორ წიგნში შემხვდა

ბიჰევიორალ საინსში და გენეტიკაში

Posted by: Blind_Torture_Kill 14 Dec 2008, 17:18
mtvareuli

QUOTE
ბიჰევიორალ საინსში და გენეტიკაში


შენც სტეპებს ხომ არ აპირებ ?

Posted by: LUKA-BRAZI 14 Dec 2008, 17:40
mtvareuli
Guardian-თან? smile.gif

Posted by: mtvareuli 14 Dec 2008, 17:46
Blind_Torture_Kill

ჯერ არ ვიცი smile.gif იდეაში მინდა

LUKA-BRAZI
QUOTE
Guardian-თან?

ბევრ რჩევებს ვიღებ, მაგრამ ოფიციალურად არ ვემზადები smile.gif


მაგრამ აქ რა შუაშია ეგ, დადეთ რამე საინტერესო

Posted by: Blind_Torture_Kill 14 Dec 2008, 17:47
65 წლის ქალბატონი გასულ კვირას მკურნალობდა ფართო სპექტრის ანტიბიოტიკებით სინუსურ ინფექციას რამაც შედეგი გამოიღო და გამოჯანმრთელდა. ერთი კვირის შემდეგ მას დაეწყო მუცლის ტკივილი (ბლუმბერგი +) , ტემპერატურა - 39.5c , ფაღარათი - დიდი რაოდენობით მომწვანო ფერის თხიერი განავალით.
პაციენტი გარდაიცვალა.
აუტოპსიაზე მსხვილი ნაწლავის დათვალიერებისას ჩანს



დიაგნოზი

Posted by: LUKA-BRAZI 14 Dec 2008, 17:53
სურათი აკლია ქეისს მგონ, ჰო? smile.gif

Posted by: mtvareuli 14 Dec 2008, 17:53
Blind_Torture_Kill
QUOTE
აუტოპსიაზე მსხვილი ნაწლავის დათვალიერებისას ჩანს

რა ჩანს?

Posted by: Blind_Torture_Kill 14 Dec 2008, 18:06
წეღან ორჯერ დავპოსტე და ერთზე ავაკარი სურათი მარა მგონი ის პოსტი წაიშალა და მეორე დარჩა smile.gif

Posted by: Cousteau 14 Dec 2008, 18:07
QUOTE (Blind_Torture_Kill @ 14 Dec 2008, 17:47 )
65 წლის ქალბატონი გასულ კვირას მკურნალობდა ფართო სპექტრის ანტიბიოტიკებით სინუსურ ინფექციას რამაც შედეგი გამოიღო და გამოჯანმრთელდა. ერთი კვირის შემდეგ მას დაეწყო მუცლის ტკივილი (ბლუმბერგი +) , ტემპერატურა - 39.5c , ფაღარათი - დიდი რაოდენობით მომწვანო ფერის თხიერი განავალით.
პაციენტი გარდაიცვალა.
აუტოპსიაზე მსხვილი ნაწლავის დათვალიერებისას ჩანს



სავარაუდო დიაგნოზი

ფსეუდომემბრანოზული კოლიტისგან მოკვდა? spy.gif

Posted by: Blind_Torture_Kill 14 Dec 2008, 18:10
Cousteau

კი

შენი პაციენტი რომ ყოფილიყო როგორ გადაარჩენდი ?

ცოტა ხანში სერიოზულ ქეისს დავაგდებ
მანამდე თქვენც დადეთ კაი ტვინის სა...ავი რამე

Posted by: Cousteau 14 Dec 2008, 18:12
QUOTE (Blind_Torture_Kill @ 14 Dec 2008, 18:10 )
Cousteau

კი

შენი პაციენტი რომ ყოფილიყო როგორ გადაარჩენდი ?

ყვირილს დავიწყებდი ექიმი დროზე gigi.gif
მართალიგითხრა ეგეთ მწვავე შემთხვევაში მაგას როგორ უნდა უმკურნალო არვიცი, მეტრონიდაზოლი? spy.gif

Posted by: mtvareuli 14 Dec 2008, 18:13
Cousteau
QUOTE
ფსეუდომემბრანოზული კოლიტი

კიარა, ფსევდომემბრანული კოლიტი tongue.gif

Posted by: Blind_Torture_Kill 14 Dec 2008, 18:16
ამ შემთხვევაში მსხვილი ნაწლავის რეზექცია ალბათ

კი მეტრონიდაზოლი მარა ალბათ არ უშველა

Posted by: Cousteau 14 Dec 2008, 18:17
QUOTE (mtvareuli @ 14 Dec 2008, 18:13 )
Cousteau
QUOTE
ფსეუდომემბრანოზული კოლიტი

კიარა, ფსევდომემბრანული კოლიტი tongue.gif

gigi.gif
pseudomembranous რო არის ინგლისურად მაგიტო ვწერ ეგრე user.gif


Blind_Torture_Kill ამას კითხულობ?

cholestyramine, a bile acid resin that can be used to bind C. difficile toxin.

if antibiotics do not control the infection the patient may require a colectomy (removal of the colon) for treatment of the colitis.

ხო და კეფირს მივცემდი კიდე

Anecdotal evidence suggests kefir can help treat pseudomembranous colitis.

Posted by: Blind_Torture_Kill 14 Dec 2008, 18:27
Cousteau

ვიკის biggrin.gif
კი biggrin.gif მარა მანდ ისე ამომწურავად არაახოლმე
შესავალივით წერია


abdominal colectomy with ileostomy has been required for rare cases of fulminant pseudomembranous colitis.
NMS surgery 4th edition


Posted by: Cousteau 14 Dec 2008, 18:30
Blind_Torture_Kill არა კაცო, gigi.gif ის გკითხე ამას მეკითხები მეთქი - ქოლესტერამინზე? წიპა ვიკის კი არ კითხულობ, ამას კითხულობ? დ.ა.შ.

:რაღაცავერგავიგესმაილიკი:

Posted by: Blind_Torture_Kill 14 Dec 2008, 18:35
Cousteau

QUOTE
Blind_Torture_Kill არა კაცო,ის გკითხე ამას მეკითხები მეთქი - ქოლესტერამინზე? წიპა ვიკის კი არ კითხულობ, ამას კითხულობ? დ.ა.შ.

:რაღაცავერგავიგესმაილიკი:


კი მივხვდი smile.gif


Posted by: Cousteau 14 Dec 2008, 18:38
QUOTE (Blind_Torture_Kill @ 14 Dec 2008, 18:35 )
Cousteau

QUOTE
Blind_Torture_Kill არა კაცო,ის გკითხე ამას მეკითხები მეთქი - ქოლესტერამინზე? წიპა ვიკის კი არ კითხულობ, ამას კითხულობ? დ.ა.შ.

:რაღაცავერგავიგესმაილიკი:


კი მივხვდი smile.gif

ისე საერთოდ არ ვიცოდი მაგისგან თუ კვდებიან, მე ისეთი შედარებით light რაღაცა მეგონა... : /


წავალ გასტროს მივასკუმარებ ბოლომდე და მოვალ
(ანატომიას ვსწავლობ gigi.gif ინსტიტუტი დავამთავრე user.gif )
vis.gif

Posted by: LUKA-BRAZI 14 Dec 2008, 18:54
Cousteau
QUOTE
ინსტიტუტი დავამთავრე

რომელი? smile.gif

Posted by: Blind_Torture_Kill 14 Dec 2008, 19:05
Cousteau

არ მოკვდებიან თუ დროული მკურნალობა დაიწყება მეტრონიდაზოლით ან თუ არ უშველა ნაწლავის რეზექცია უნდა გაკეთდეს უეჭველი თორე პიზდეც

Posted by: LULA_QABABI 14 Dec 2008, 19:22
QUOTE (Blind_Torture_Kill @ 14 Dec 2008, 10:05 )
Cousteau

არ მოკვდებიან თუ დროული მკურნალობა დაიწყება მეტრონიდაზოლით ან თუ არ უშველა ნაწლავის რეზექცია უნდა გაკეთდეს უეჭველი თორე პიზდეც

C. difficile უმეტეს შემთხვევაში ფსევდომემრანული კოლიტის გარეშე მიმდინარეობს; კოლიტი უფრო გართულებული ფორმაა; ისევე როგორც ტოქსიური მეგაკოლონი. დიაგნოზი შეიძლება ადვილად დაისვას ტოქსინის A/B ტესტით; სენსიტიურობა სადღაც 70-80% არის (ტესტის კომპანიაზეც არის დამოკიდებული);
მემგონი საქართველოშიც დაიწყეს მაგის კეთება;
თუ კლინიკური ეჭვი არსებობს ამ ინფექციაზე უარყოფოთი ტესტის მიუხედავად დავიწყებდი პერორალურ ვანკომიცინს (საშუალო და მძიმე) ან მეტრონიდაზოლს (მსუბუქი ფორმა);
ბოლოხანებში უფრო მეტად ვხვდებით კლინიკურად მეტრონიდაზოლ-რეზისტენტულ შემთხვევებს; ამიტომ ხშირად პირდაპირ ვანკოს ვნიშნავ, რომელიც საკმაოდ ძვირია;
რეკურენტული შემთხვევებიც ხშირია, როდესაც კურსის დამთავრების შემდეგ დიარეა მეორდება,
ქოლესტირამინი ჩემი პირადი გამოცდილებით დიდ არაფერს არ მატებს; ზოგჯერ პირიქით, აუარესებს ხოლმე ფაღარათს;
პრობიოტიკების ანუ "კარგი ნაწლავური ფლორის" დანიშვნა და სარგებლიანობა დღემდე საკამათოა; ერთი პლიუსი, სხვებისგან განსხვავებით, ისაა რომ თითქმის არ აქვთ გვერდითი ეფექტი;

ეს ინფექცია ერთერთი "ჰოთ ტოპიკია"/პრობლემატურია დღევანდელ დასავლურ მედიცინაში;

Posted by: Cousteau 14 Dec 2008, 19:43
QUOTE
ამიტომ ხშირად პირდაპირ ვანკოს ვნიშნავ, რომელიც საკმაოდ ძვირია;


QUOTE
ქოლესტირამინი ჩემი პირადი გამოცდილებით დიდ არაფერს არ მატებს ; ზოგჯერ პირიქით, აუარესებს ხოლმე ფაღარათს;


სად მუშაობ?




LUKA-BRAZI gigi.gif რომელ ინსტიტუში არ ასწავლიან ანატომიას? gigi.gif

აიეტი დავამთავრე user.gif და ხვალ gen.surg რეზიდენტურას ვიწყებ : /

Posted by: LULA_QABABI 14 Dec 2008, 22:53
QUOTE (Cousteau @ 14 Dec 2008, 10:43 )
QUOTE
ამიტომ ხშირად პირდაპირ ვანკოს ვნიშნავ, რომელიც საკმაოდ ძვირია;


QUOTE
ქოლესტირამინი ჩემი პირადი გამოცდილებით დიდ არაფერს არ მატებს ; ზოგჯერ პირიქით, აუარესებს ხოლმე ფაღარათს;


სად მუშაობ?


მემფისი და ვესტ მემფისი.
http://maps.google.com/maps?ie=utf-8&oe=utf-8&rls=org.mozilla:en-US:official&client=firefox-a&q=memphis&um=1&sa=X&oi=geocode_result&resnum=1&ct=image

Posted by: Cousteau 14 Dec 2008, 23:37
QUOTE (LULA_QABABI @ 14 Dec 2008, 22:53 )
QUOTE (Cousteau @ 14 Dec 2008, 10:43 )
QUOTE
ამიტომ ხშირად პირდაპირ ვანკოს ვნიშნავ, რომელიც საკმაოდ ძვირია;


QUOTE
ქოლესტირამინი ჩემი პირადი გამოცდილებით დიდ არაფერს არ მატებს ; ზოგჯერ პირიქით, აუარესებს ხოლმე ფაღარათს;


სად მუშაობ?


მემფისი და ვესტ მემფისი.
http://maps.google.com/maps?ie=utf-8&oe=utf-8&rls=org.mozilla:en-US:official&client=firefox-a&q=memphis&um=1&sa=X&oi=geocode_result&resnum=1&ct=image

gigi.gif

საღოლ
up.gif

Posted by: LUKA-BRAZI 14 Dec 2008, 23:57
Cousteau
QUOTE
რომელ ინსტიტუში არ ასწავლიან ანატომიას?

აიეტი დავამთავრედა ხვალ gen.surg რეზიდენტურას ვიწყებ : /

და რეზიდენტურას სად იწყებ? US-ში? მე ასე მგონია რომ ყველა აიეტდამთავრებულს სტეპები აქვს უკვე ჩაბარებული... smile.gif

Posted by: Cousteau 15 Dec 2008, 00:07
QUOTE (LUKA-BRAZI @ 14 Dec 2008, 23:57 )
Cousteau
QUOTE
რომელ ინსტიტუში არ ასწავლიან ანატომიას?

აიეტი დავამთავრედა ხვალ gen.surg რეზიდენტურას ვიწყებ : /

და რეზიდენტურას სად იწყებ? US-ში? მე ასე მგონია რომ ყველა აიეტდამთავრებულს სტეპები აქვს უკვე ჩაბარებული... smile.gif

gigi.gif
ტყულია მაგარი
ყველა ჩემი კურსელი (არადა კურსზე ძალიან ძლიერები მყავს) აქ აბარებენ და მერე როგორ იქნება ვნახოთ smile.gif

Posted by: LUKA-BRAZI 15 Dec 2008, 00:08
Cousteau
QUOTE
ტყულია მაგარი ყველა ჩემი კურსელი (არადა კურსზე ძალიან ძლიერები მყავს) აქ აბარებენ და მერე როგორ იქნება ვნახოთ

ვაა, არ ვიცოდი ნაღდად smile.gif აბა წარმატებები wink.gif

Posted by: texasuri jleta benzoxerxit 15 Dec 2008, 00:44
QUOTE (Blind_Torture_Kill @ 14 Dec 2008, 18:16 )
ამ შემთხვევაში მსხვილი ნაწლავის რეზექცია ალბათ

კი მეტრონიდაზოლი მარა ალბათ არ უშველა

Clostridium difficile. ვანკომიცინია არჩევის პრეპარატი! boli.gif


აუ,შემდეგი გვერდი არ მინახავს და ყოფილა პასუხი უკვე... baby.gif

Posted by: Blind_Torture_Kill 15 Dec 2008, 01:42
QUOTE
თუ კლინიკური ეჭვი არსებობს ამ ინფექციაზე უარყოფოთი ტესტის მიუხედავად დავიწყებდი პერორალურ ვანკომიცინს (საშუალო და მძიმე) ან მეტრონიდაზოლს (მსუბუქი ფორმა);


Vancomycin - 200 $ (Vancomycin, 5 g, $245)
Meronidazole - <1 $ (250 mg 100 tablets-9.95$)

ზუსტი ფასები არ ვიცი
მიახლოებულად
* * *
აბა ვინ გამოიცნობს რა არის ეს wink.gif
* * *
http://imageshack.us
http://g.imageshack.us/img408/70328480nt9.jpg/1/

Posted by: vano_t 15 Dec 2008, 12:01
Blind_Torture_Kill
QUOTE
აბა ვინ გამოიცნობს რა არის ეს  wink.gif 
* * *
http://imageshack.us
http://g.imageshack.us/img408/70328480nt9.jpg/1/

giardia lamblia

თუ არა და trichomona vaginalis. უფრო პირველი

Posted by: Blind_Torture_Kill 15 Dec 2008, 12:24
vano_t

QUOTE
giardia lamblia


ოკ მიღებულია

Posted by: LUKA-BRAZI 15 Dec 2008, 14:46
Blind_Torture_Kill
ეს იყო დაპირებული სერიოზული ქეისი? biggrin.gif

Posted by: badu 15 Dec 2008, 21:54
QUOTE (texasuri jleta benzoxerxit @ 14 Dec 2008, 23:44 )
QUOTE (Blind_Torture_Kill @ 14 Dec 2008, 18:16 )
ამ შემთხვევაში მსხვილი ნაწლავის რეზექცია ალბათ

კი მეტრონიდაზოლი მარა ალბათ არ უშველა

Clostridium difficile. ვანკომიცინია არჩევის პრეპარატი! boli.gif


აუ,შემდეგი გვერდი არ მინახავს და ყოფილა პასუხი უკვე... baby.gif

მსუბუქ შემთხვევებს თუ არ ვცდები მკურნალობა შეიძლება სულ არ დაჭირდეს. სხვა შემთხვევაში კი მეტრონიდაზოლია მთავარი პრეპარატი. ვანკომიცინთან სიფრთხილეა საჭირო, რადგან პაციენტს შეიძლება VRE (vancomycin resistrant enterococcus) განუვითარდეს.
აი ვიკიპედიაც მეთანხმება

Three antibiotics are specifically effective against C. difficile in vivo. Metronidazole (500 mg orally three times daily) is the drug of choice, because of lower price and comparable efficacy.[16] Oral vancomycin (125 mg four times daily) is second-line therapy, but is often avoided due to theoretical concerns of converting intestinal flora into vancomycin-resistant organisms. However, it is used in the following cases: severe C. difficile diarrhea (the duration of diarrhea was reported to be reduced to 3 versus 4.6 days with metronidazole); no response to oral metronidazole; the organism is resistant to metronidazole; the patient is allergic to metronidazole; the patient is either pregnant or younger than 10 years of age

Posted by: LULA_QABABI 16 Dec 2008, 07:00
QUOTE (badu @ 15 Dec 2008, 12:54 )
 

QUOTE
მსუბუქ შემთხვევებს თუ არ ვცდები მკურნალობა შეიძლება სულ არ დაჭირდეს

მკურნალობა თითქმის ყოველთვის საჭიროა; პირადი გამოცდილების ჩათვლით; თუმცა, იშვიათად მაპროვოცირებელი ანტიბიოტიკის შეჩერებასთან ერთად ხშირად დიარეაც ქრება; მეორე მხრივ ამ შემთხვევაში ძნელია დიფერენცირება დიარეა ანტიბიოტიკს გვერდითი ეფექტია თუ კდ-ით არის გამოწვეული.

QUOTE
ვანკომიცინთან სიფრთხილეა საჭირო, რადგან პაციენტს შეიძლება VRE (vancomycin resistrant enterococcus) განუვითარდეს.

საუბარია VRE-ით კოლონიზაციაზე და არა დაავადებაზე; თუმცა კოლონიზაცია თავისთავად მნიშვნელოვანი ფაქტორი ხდება თუ ავადმყოფს სჭირდება ფოლის კათეტერი, ცენტრალური ვენური კათეტერი და სხვა ტიპის ინსტრუმენტაცია, რაც ზრდის ჰოსპიტალური ინფექციების, მათ შორის VRE-ს, რისკსაც შესაბამის ორგანო-სისტემებში; მაგრამ კიდევ ერთხელ, კონკრეტულ შემთხვევაში (კდ დიარეა, კოლიტი)) ვანკომიცინის ეფექტურობა/უპირატესობა აშკარად გადასწონის შესაძლო კოლონიზაციის, და მასთან დაკავშირებული გართულებების თეორიულ რისკს. ანუ მოკლედ რო ვთქვათ, ეგ არ დამაბრკოლებდა;

QUOTE
აი ვიკიპედიაც მეთანხმება

ვიკიპედიას ვერ ვუწოდებდი ავტორიტეტულ წყაროს;
პოპულარული მკითხველისთვის
http://www.mayoclinic.com/health/c-difficile/DS00736/DSECTION=treatments-and-drugs
ეს ბოლო და მოძველებული გაიდლაინია, ახალი წელს უნდა გამოვიდეს;
http://www.shea-online.org/Assets/files/position_papers/Cldiff95.PDF

Posted by: Blind_Torture_Kill 16 Dec 2008, 09:44
"Stool transplant" to restore healthy intestinal bacteria by placing donor stool (usually from a relative) in your colon. Although this is rarely done in practice, research has shown stool transplant to be helpful.

რა გაიჩითა biggrin.gif

LUKA-BRAZI

არა ლუკა smile.gif
ეს არაა

Posted by: LUKA-BRAZI 17 Dec 2008, 17:33
Blind_Torture_Kill
QUOTE
"Stool transplant" to restore healthy intestinal bacteria by placing donor stool (usually from a relative) in your colon. Although this is rarely done in practice, research has shown stool transplant to be helpful.

wow.gif რაოო?! ესენი მგონი ვერ არიან biggrin.gif და research-ი როგორ ჩაატარეს? აააა, როგორია ექიმი დისბაქტერიოზის მკურნალობის ამ მეთოდს რომ შესთავაზებს პაციენტს? biggrin.gif

Posted by: Blind_Torture_Kill 18 Dec 2008, 00:38
22-30 mmol/L37 წლის შიდსიან მამაკაცს აქვს
T- 38.3c , შემცივნებები , გამონადენი ცხვირიდან და ხველა რომელიც დაეწყო 3 დღის წინ რის გამოც მიიღო ანტიბიოტიკი მაგრამ მის სახელს ვერ იხსენებს user.gif .
ფილტვების აუსკულტაციით ისმინება კრეპიტაცია
CD უჯრედები < 200
სისხლში გაზების დონე ნორმაა


ბრონქიალური ლავაჟის შედეგი მოცემულია ქვემოთ

პაციენტს დაენიშნა სეპტრინი + ზიდოვუდინი , ლამივუდინი

მე-4 დღეს პაციენტს განუვითარდა აკროციანოზი , ტაქიპნეა + ტაქიკარდია რის გამოც გადაიყვანეს ინტენსიური თერაპიის განყოფილებაში.

უკვე გაზების დონე ესეთია


PaO2 - 94.1 mmHg
PaCO2 - 26.1 mmHg
HCO3 - 16 mmol/L
pH - 7.39


Hct - 44.8 %
hemoglobin - 15.1g/dl




რა ჭირს ?

ჰა როგოთია ლუკა ეს ქეისი ?

Posted by: badu 18 Dec 2008, 03:12
Pneumocystis carinii . . .

Posted by: vano_t 18 Dec 2008, 06:33
badu
QUOTE
Pneumocystis carinii .                                      .                            .

P. carinii კი არის ბრონქიალურ ლავაჯში, მაგრამ ეგ კლინიკურ სურათს ვერ ახსნის. ავადმყოფს აქვს CD4 საკმარისზე დაბლა, რომ ჩამოუყალიბდეს ეგ ყველაფერი, მაგრამ მთელი ეს ყველაფერი ამ კეისში არის ანკესი. P. carinii ბევრ შიდსიანს ამოეთესება მიუხედავად იმისა აქვს თუ არა ამით გამოწვეული პნევმონია ავადმყოფს (თუ ვცდები ამაში, ლულა-ქაბაბი გამისწორებს-მე უბრალოდ ასე მახსოვს). პნევმოცისტისტური პნევმონიისათვის დამახასიათებელია ჰიპოქსია. ჰიპოქსია ერთ-ერთი აუცილებელი რამ არის. კიდევ, როგორც მახსოვს, ლაქტატდეჰიდროგენაზას მომატება აქვს (დიდი ხანი არ მინახია შიდსიანი ავადმყოფები და შეიძლება ვცდებოდე ზოგიერთ რამეში). გულმკერდის რენტგენზე კიდევ ხშირად აქვთ დიფუზური ინფილტრატები (თუმცა ფოკალური ინფილტრატიც შეიძლება ქონდეთ, ან ნორმალური რენტგენი ქონდეთ). გარდა ამისა, ამ ავადმყოფის მკურნალობა ბაქტრიმით (სეპტრინიც მე მგონი იგივე ტრიმეთოპრიმ-სულფამეთოქსაზოლია) თავიდანვე მიმდინარეობდა და მერე ჩამოუყალიბდა ეს ყველაფერი. ეს კიდევ უფრო ამცირებს მაგ დიაგნოზს.

ამ კეისში რაც თვალში საცემია არის ის, რომ ავადმყოფს აქვს ციანოზი, თანაც ჰიპოქსიის გარეშე და თანაც ნორმალური ჰემოგლობინის ფონზე-ეს ერთი. მეორე-ამ ავადმყოფს ტაქიპნეა აქვს მეორადი. ეს იქიდან ჩანს, რომ სისხლის PH არის მცირედ აციდოზური და არა ალკალოზური. ეს კიდევ იმას ნიშნავს, რომ პირველადი მეტაბოლური აციდოზი აქვს რესპირატორული კომპენსაციით. რა ტიპის მეტაბოლური აციდოზი აქვს? ელექტროლიტების გარეშე ამის თქმა შეუძლებებლია. ელექტროლიტები უნდა იყოს მოცემული სრული სურათისათვის, რომ დაინახო გაქვს თუ არა anion gap აციდოზი. თუმცა, როცა ბიკარბონატი ასე დაბალია (ჩვენს შემთხვევაში არის 16, როცა ნორმა 24), ეს იმას გეუბნება, რომ most likely ამ ავადმყოფს აქვს anion gap აციდოზი (შეიძლება baseline ბიკარბონატი იყოს დაბალი, მარა ასე დაბლა არ მგონია რომ ჩავიდეს-მაგალითად, ტუბულარული აციდოზების ზოგიერთ ფორმის დროს პაციენტის ბიკარბონატი შეიძლება დაბალი იყოს).

ახლა ეს ყველაფერი (ჟანგბადის ნორმალური პარციალური წნევა, ნორმალური ჰემოგლობინი და მეტაბოლური აციდოზი) უნდა ჩასვა ერთ სურათში. ეს საერთო რამ შეიძლება გამოწვეული იყოს წამლებით (ბაქტრიმი იწვევს ამას მაგალითად) და ეს საერთო რამ იწვევს ეფექტურ ანემიას, მიუხედავად ჰემოგლობინის ნორმალური დონისა, ჰემოგლობინის ჟანგბადის დისოციაციის მრუდის შეცვლით. ეს საერთო რამ, ეფექტური ანემიის გამო, იწვევს ეფექტურ ჰიპოქსიას და ჰიპოქსიასთან დაკავშირებულ პრობლემებს (მეტაბოლური აციდოზის ჩათვლით-შეუძლია ლაქტმჟავას აციდოზის მოგცეს). ეს საერთო რამ, ჩამოთვლილი მიზეზების გამო, გაძლევს აციდოზსაც, ნორმალური ჰემოგლობინის ფონზე.

როცა პასუხს მოიფიქრებ, 2 რამ აუცილებლად უნდა ახსნა: 1) დაავდებამ უნდა ახსნას გარკვეული სიმპტომების არარსებობა და 2) დაავადებამ უნდა ახსნას გარკვეული სიმპტომების არსებობა.

ახლა უნდა გამოიცნო პასუხი.

Posted by: badu 18 Dec 2008, 09:22
მე მაინც P. carinii მგონია. ჰიპოქსიას რაც შეეხება, ტაქიპნეა ჰიპოქსიის გამო შეიძლება იყოს გამოწვეული რაც ახსნის ნორმალურ ჟანგბადის დონეს და შემცირებულ ნახშიროჟანგის დონეს სისხლში.

აციდოზი რატომ აქვს ამ პაციენტს? PH ნორმალური აქვს. 7.36-7.44 ხომ ნორმალურია?

Posted by: vano_t 18 Dec 2008, 09:51
badu
QUOTE
მე მაინც P. carinii მგონია. ჰიპოქსიას რაც შეეხება, ტაქიპნეა ჰიპოქსიის გამო შეიძლება იყოს გამოწვეული რაც ახსნის ნორმალურ ჟანგბადის დონეს და შემცირებულ ნახშიროჟანგის დონეს სისხლში.
ჰიპოქსია არ აქვს ავამდყოფს. ჟანგბადის პარციალური წნევა ნორმალურია და როგორ ექნება ჰიპოქსია? შენ თუ იმის თქმა გინდა, რომ ჰიპერვენტილაციით ჰიპოქსიის მოხსნა ხდება, მასე არ ხდება პრაქტიკაში. ჰიპერვენტილაცია ცოტას შეამცირებს ჰიპოქსიას, მაგრამ ნორმალურ დონეზე ვერ მიიყვანს ჟანგბადს. მოკლედ, ამ ავადმყოფს არ აქვს ჰიპოქსია.

QUOTE
აციდოზი რატომ აქვს ამ პაციენტს? PH ნორმალური აქვს. 7.36-7.44 ხომ ნორმალურია?
აციდოზათ ითვლება ეგ. ნორმალური PH არის 7.4. რა თქმა უნდა ამას ვარიაცია გააჩნია, მაგრამ მჟავა-ტუტოვანი წონასწორობის გამოთვლის მიზნით, 7.4 ითვლება ნორმად. ყველაფერი მის ქვევით არის აციდოზი და ყველაფერი მის ზევით არის ალკალოზი. დაახლოებით შეგიძლია ისიც გამოთვალო, თუ როგორი უნდა იყოს PH-ის ცვლილება პირველადი მწვავე რესპირატორული ალკალოზის დროს: 0.008X(40-PaCO2)=0.008X14=0.12. თუ პირველადი რესპირატორული ალკალოზი გაქვს და სრული მეტაბოლური კომპენსაცია მოხდა, მაშინ PH-ის ცვლილება გამოითვლება ფორმულით: 0.0017X(40-PaCO2)=0.0017X14=0.02. ანუ, თუ მწვავე რესპირატორული ალკალოზია და ამ ავადმყოს პრობლემის დაწყებამდე PH ქონდა 7.36 (შენს მიერ ნახსენები ქვედა ნორმა), მისი PH მაინც 7.48 გახდებოდა (7.36+0.12). შენი სცენარი რომ იყოს სწორი, ამ ავადმყოფის საწყისი PH (ანუ რესპირატორული ალკალოზის ჩამოყალიბებამდე) უნდა ყოფილიყო ნორმის ქვედა ზღვარი (7.36) და ამასთანავე უნდა გქონოდა რესპირატორული ალკალოზის სრული მეტაბოლური კომპენსაცია. ამ შემთხვევაში PH შეიძლება ყოფილიყო 7.38.

იტოგში, ჩემი აზრით ამ ავადმყოფს არ აქვს P. carinii (არაფერი მეტყველებს ამაზე ავადმყოფის პრეზენტაციაში) და აქვს სავარაუდოდ მეთჰემოგლობინემია, რაზეც ყველაფერი მეტყველებს.. მეთჰემოგლობინემია ბაქტრიმით შეიძლება იყოს გამოწვეული. არ ვიცი ზიდოვუდინის ან ლამივუდინის დროს არის თუ არა აღწერილი, მაგრამ ბაქტრიმს ნამდვილად შეუძლია.

Posted by: Blind_Torture_Kill 18 Dec 2008, 11:14
აი დისკუსიები მესმის smile.gif

vano_t

მედიცინის მამად გაღიარე
P.S - შენი ახსნა ძაან დამევასა
პაციენტს კი აქვს პნევმოცისტური პნევმონია მაგრამ გართულება მკურნალობამ გამოიწვია სეპტრინმა

მეთჰემოგლობინემია განუვითარდა ამ პაციენტს



Posted by: donvaso 18 Dec 2008, 12:41
QUOTE
vano_t

მედიცინის მამად გაღიარე


+100000000000000000000000

არადა საქართველოში ძალიან საჭიროა შენნაირი ექიმები ბატონო ვანო..... yes.gif

Posted by: badu 18 Dec 2008, 17:43
QUOTE (Blind_Torture_Kill @ 18 Dec 2008, 10:14 )
აი დისკუსიები მესმის smile.gif

vano_t

მედიცინის მამად გაღიარე
P.S - შენი ახსნა ძაან დამევასა
პაციენტს კი აქვს პნევმოცისტური პნევმონია მაგრამ გართულება მკურნალობამ გამოიწვია სეპტრინმა

მეთჰემოგლობინემია განუვითარდა ამ პაციენტს

გეთანხმები, ძაან მაგარი კაცია. ამ კაცის პოსტებიდან მართლა ბევრს ვსწავლობ

Posted by: Blind_Torture_Kill 19 Dec 2008, 00:48
აბა რა არის ეს ?

ხვალისთვის კაი ქეისი შემოვინახე
ხოდა თუ გამოიცნობთ ამას იმ შემთხვევაში დავდებ wink.gif

Posted by: vano_t 19 Dec 2008, 00:59
Blind_Torture_Kill
QUOTE
აბა რა არის ეს ?

ხვალისთვის კაი ქეისი შემოვინახე
ხოდა თუ გამოიცნობთ ამას იმ შემთხვევაში დავდებ  wink.gif

ეს კი არის ნაღდად მოზრდილთა PKD. მაგერ ერთი დიდი კისტაც ჩანს სუბკაპფსულარულად.

მეც დავდებ ახლა ახალ კეისებს. fancy ქეისები არ არის, მაგრამ ხშირი პრობლემების სტანდარტული კვლევის გასაცნობად ძალიან კარგია. ორივე ქეისი ამ ბოლო დროს ვნახე. ერთი დღეს და მეორე 2 კვირის წინ და დღეს follow up-ზე იყო კლინიკაში.

მერე მივაყოლებ ბორდის ამოცანას.

donvaso
QUOTE
ბატონო ვანო
კოლეგები ვართ და ვანო დამიძახე თუ ძმა ხარ smile.gif
მეც ვასოს დაგიძახებ, დონის გარეშე biggrin.gif

badu
QUOTE
ამ კაცის პოსტებიდან მართლა ბევრს ვსწავლობ
ძალიან კარგი. ე.ი. ეს ტოპიკი კარგად მუშაობს. მეც შემიძლია ვთქვა, რომ აქ ყველასაგან ვსწავლობ რაღაცას.

Posted by: Blind_Torture_Kill 19 Dec 2008, 01:03
vano_t

QUOTE
ეს კი არის ნაღდად მოზრდილთა PKD. მაგერ ერთი დიდი კისტაც ჩანს სუბკაპფსულარულად.


აი PKD

არაა ეგ smile.gif PKD

Posted by: vano_t 19 Dec 2008, 01:33
მაშინ არ ვიცი.

მოკლედ პირველ ქეისს დავდებ:

82 წლის ქალი მოვიდა კლინიკაში 2 კვირის წინ ჩივილებით. აწუხებს სისუსტე. კლინიკაში მოსვლამდე 2 კვირით ადრე დაეწყო ეს სისუტე. ასევე აწუხებს პირის სიმშრალე და მომატებული ძილიანობა-ავადმყოფი ამბობს, რომ სულ ძილი უნდა. შარდვის სიხშირე არ აქვს. უარყოფს სხვა ჩივილებს: არ აქვს სიცხე, უმადობა, წონაში დაკლება. უარყოფს თავის ტკივილებს, მხედველობის ან სმენის პრობლემებს. არ აქვს მკერდის ტკივილი, სუნთქვის უკმარისობა და ა.შ. მოკლედ, დანარჩენი სისტემები ნორმაშია.

წარსული ისტორია: 1) ჰიპერტენზია, რომელიც კონტროლირდება ასე თუ ისე;
2) რეფლუქსი; 3) გაურკვეველი წარმოშობის ორმხრივი ნეიროპათია ტერფებში; 4) overactive bladder (ჰიპერაქტიური შარდის ბუშტი)

ქირურგია: 1) აპენდექტომია; 2) ჰისტერექტომია; 3) კოლონოსკოპია X1.

არ ეწევა, სვამს და არ იკეთებს.

ალერგია რაიმეზე არ აქვს.

იღებს შემდეგ წამლებს: 1) ატენოლოლი; 2) vesicare, რაც არის სოლიფენაცინი (აცეტილქოლინის რეცეპტორების ანტაგონისტია); 3) ვიტამინები;

წნევა ცოტათი მომატებული (166/77); ტემპერეტურა, გულისცემა ნორმალური. ასევე, გულისცემა რეგულარულია და პულსები კარგი ავსების. ჟანგბადის სატურაცია ნორმალურია. სუნთქვის სიხშირე ნორმალურია. ფიზიკური გამოკვლევისას აღსანიშნავი არაფერია, გარდა საკმაოდ მშრალი პირისა.

ავადმყოფს ჩაუტარდა ლაბორატორიები, რათა გადაუდებელი პრობლემები გამორიცხულიყო (მაგალითად ინფექციები, თირკმლის უკმარისობა, უკონტროლო დიაბეტი და ა.შ.). ლაბორატორიებიდა CBC არის მთლიანად ნორმალური. BUN/Cretinine/Cl/Na/K/HCO3/ALT/AST/ALP (alkaline phosphatase)/გლუკოზა/ბილირუბინი არის ნორმალური. TSH გაუკეთდა და არის ნორმალური. სერიოზული მიზეზების გამორიცხვის შემდეგ, სავარაუდო მიზეზი შეირჩა და შესაბამისი ჩარევა ჩაუტარდა. ამის შემდეგ ავადმყოფი მოდის კლინიკაში დღეს და აღარ აქვს არც სისუსტე, არც ძილიანობა (ჰიპერსომნია) და არც პირის სიმშრალე.

სავარაუდო მიზეზი რა არის? რა თქმა უნდა ქეისი ღია დავტოვე შეკითხვებისათვის. შეკითხვები თქვენზეა დამოკიდებული. ასეთი ქეისები (არა მაინცდამაინც ამ სიმპტომებით) არის ძალიან ტიპიური და ამიტომ კარგია სასწავლად.

Posted by: vano_t 19 Dec 2008, 01:41
Blind_Torture_Kill
QUOTE
აი PKD

არაა ეგ smile.gif PKD

საინტერესო მაკროპრეპარატია. ისე ჩანს თითქოს ამ თირკმელიდან წყალი "აორთქლდა" და შემაერთებელი ქსოვილი კარკასი დარჩა.

კისტებიც ხო ჩანს ამ თირკმელში?
* * *
QUOTE
Blind_Torture_Kill
QUOTE
აი PKD

არაა ეგ smile.gif PKD

საინტერესო მაკროპრეპარატია. ისე ჩანს თითქოს ამ თირკმელიდან წყალი "აორთქლდა" და შემაერთებელი ქსოვილი კარკასი დარჩა.

კისტებიც ხო ჩანს ამ თირკმელში?

შებოლილი და დაკაფჩონებული თირკმელი ხომ არ არის? biggrin.gif კანიბალებში დელიკატესად ითვლება ეგ მე მგონი smile.gif

Posted by: LULA_QABABI 19 Dec 2008, 03:44
QUOTE (vano_t @ 18 Dec 2008, 00:51 )
QUOTE
აციდოზი რატომ აქვს ამ პაციენტს? PH ნორმალური აქვს. 7.36-7.44 ხომ ნორმალურია?
აციდოზათ ითვლება ეგ. ნორმალური PH არის 7.4. რა თქმა უნდა ამას ვარიაცია გააჩნია, მაგრამ მჟავა-ტუტოვანი წონასწორობის გამოთვლის მიზნით, 7.4 ითვლება ნორმად. ყველაფერი მის ქვევით არის აციდოზი და ყველაფერი მის ზევით არის ალკალოზი.

ვანო მაგაზე რა გამახსენდა თუ იცი, ჩვენ რომ დავცხეთ დორმიტორის წინ თუ გახსოვს, ერთდროულად მეტაბოლური ალკალოზი და აციდოზი როგორ შეიძლებოდა ყოფილიყო.
ბოლოს რომ პაკისტანელმა გაგვაშველა lol.gif lol.gif lol.gif lol.gif

ამ ქეისს რაც შეეხება, ცოტა არალოგიკურია (ვანოს ახსნისგან განსხვავებით); რატომ, მაგას ვეცდები მერე დავწერო;

ვანოს მედიცინის მამად აღიარებას ორი ხელით ვუჭერ მხარს; alk.gif

Posted by: vano_t 19 Dec 2008, 04:24
LULA_QABABI
QUOTE
ვანო მაგაზე რა გამახსენდა თუ იცი, ჩვენ რომ დავცხეთ დორმიტორის წინ თუ გახსოვს, ერთდროულად მეტაბოლური ალკალოზი და აციდოზი როგორ შეიძლებოდა ყოფილიყო.
ბოლოს რომ პაკისტანელმა გაგვაშველა  lol.gif  lol.gif lol.gif  lol.gif

lol.gif lol.gif
ეს საიდან გაგახსენდა? მე ბუნდოვნად მახსოვს. ის მახსოვს მაგრა რო ვიხმაურეთ, მარა რატო ვიხმაურეთ არ მახსოვს. მე მგონი მე ვამტკიცებდი არ შეიძლება თქო. არადა შეიძლება.

ახლა რა მაინტერესებს ლადო. ამ შემთხვევაში, PCP-ს დროს კლინიკურად თუ არ გაქვს პნევმონია და ამოითესა PC (პერსონალური კომპიუტერი არა biggrin.gif), მაშინ მაინც იტყვი რომ PCP გაქვს? მოკლედ თუ არ გაქვს პნევმონიის ნიშნები, დაარქმევ ამას PCP-ს? და შეიძლება თუ არა უბრალოდ კოლონიზაცია გქონდეს ინფექციის გარეშე?

Posted by: badu 19 Dec 2008, 08:05
QUOTE (vano_t @ 19 Dec 2008, 00:33 )

სავარაუდო მიზეზი რა არის? რა თქმა უნდა ქეისი ღია დავტოვე შეკითხვებისათვის. შეკითხვები თქვენზეა დამოკიდებული. ასეთი ქეისები (არა მაინცდამაინც ამ სიმპტომებით) არის ძალიან ტიპიური და ამიტომ კარგია სასწავლად.

MCV რამდენი აქვს? ასევე მეინტერესებს სისხლში რკინის, ფერიტინის, TBICის დონე. EPO ნორმალური აქვს?

vesicare-ს რამდენი ხანია რაც იღებს? ორი კვირის წინ ვიზიტის შემდეგ ისევ გააგრძელა ამ წამლის მიღება? აცეტილქოლინის მუსკარინული რეცეფტორის ანტაგონისტს შეუძლია გამოიწვიოს პირის სიმშრალე, დაღლილობა, და მომატებული ძილიანობა. ასე რომ შესაძლებელია ამ წამალმა გამოიწვია ეს რეაქცია.

Posted by: vano_t 19 Dec 2008, 08:24
badu
QUOTE
vesicare-ს რამდენი ხანია რაც იღებს? ორი კვირის წინ ვიზიტის შემდეგ ისევ გააგრძელა ამ წამლის მიღება? აცეტილქოლინის მუსკარინული რეცეფტორის ანტაგონისტს შეუძლია გამოიწვიოს პირის სიმშრალე, დაღლილობა, და მომატებული ძილიანობა. ასე რომ შესაძლებელია ამ წამალმა გამოიწვია ეს რეაქცია.
up.gif
საოფისე ვიზიტების 20-30 % არის იატროგენული პრობლემები. ამიტომ, რა წამალს იღებს, როდის დაიწყო, რაიმე პროცედურა ხომ არ ცაუტარდა ახლახან (მაგალითად კონტრასტული იმიჯინგი) და ა.შ. ყველაზე მნიშვნელოვანია. ეს ავადმყოფი მოვიდა 2 კვირის წინ მაგ სიმპტომებით. მაგრამ მანამდე 3 კვირით უფრო ადრე მოვიდა ხშირი შარდვის სიმპტომებით (overactive bladder-ს გამო-სხვა სიმპტომები არ ქონდა და შარდი ნორმალური იყო). ამიტომ დავუწყე vesicare და შემდეგ დავიბარე follow up ვიზიტზე. followup ვიზიტზე ახალი სიმპტომებით მოდის, რომელიც ჩამოუყალიბდა მკურნალობის დაწყებიდან 1 კვირის შემდეგ. კლინიკაში ძირითადი ლაბორატორიები შევუკვეთე, რომ მწვავე სერიოზული პრობლემა არ ყოფილიყო. ყველაფერი ნორმაში რომ აღმოჩნდა მერე მოვუხსენი vesicare და სიმპტომებიც შეწყდა.

ეს არის ხშირი პრობლემა. წამლის მიერ გამოწვეული გვერდითი მოვლენები ყოველთვის დგას დიფ დიაგნოზში და ამიტომ ნებისმიერ ავადმყოფს უნდა შეეკითხო რა წამალს იღებს და რამდენი ხანი. ასევე სიმპტომები წამლის მიღებამდე დაეწყო თუ მიღების შემდეგ.

QUOTE
MCV რამდენი აქვს? ასევე მეინტერესებს სისხლში რკინის, ფერიტინის, TBICის დონე. EPO ნორმალური აქვს?
MCV ნორმაში ქონდა. მაგრამ, ამას მაინც არ აქვს ღირებულება თუ ავადმყოფი ანემიური არ არის. თუ სხვა რამის გამო კითხულობ (B12 დეფიციტით გამოწვეული ნეიროპათია მაგალითად), მაშინ ეს გამორიცხული იყო ადრე. ნეიროპათიაზე ყველანაირ კვლევა ჩაუტარდა ადრე.

EPO-ს TIBC-ს ჯერ არ შევუკვეთავდი: 1) ავადმყოფს არც ანემია აქვს და არც ერითროციტოზი; 2) რომ ქონდეს პრობლემა რომელიც რაღაცნაირად არის ამ ლაბორატორიებთან დაკავშირებული, სასწრაფო არაფერი იქნება (სასწრაფო პორბლემები სხვა ლაბორატორიებით, პაციენტის სიმპტომებით, ვიტალური ნიშნებით და ფიზიკური გამოკვლევით გამოირიცხა). ამიტომ, როგორც უკვე აღნიშნე შენ, ჯერ წამალს შევუწყვიტავ, როცა იცი რომ ტემპორალური ურთიერთობა არსებობს წამლის დაწყებასა და სიმპტომების შორის (და თანაც წამლის მოქმედების მექანიზმი ასევე ფიზიოლოგიურად ხსნის სიმპტომებსაც), და მერე თუ ამან არ უშველა დავიწყებ კიდევ უფრო მეტ გამოკვლევას.

იტოგში ყოჩაღ up.gif

Posted by: vano_t 20 Dec 2008, 11:29
ოფისში მოდის 26 წლის გოგო სინკოპეს გამო. ავადმყოფს გონება დაეკარგა სამუშაოს დროს. ჯანმრთელობის თვალსაზრისით ჯანმრთელია, ქრონიკული დაავადებები არ გააჩნია და რაიმე წამლებს არ იღებს. ავადმყოფის მამა გარდაიცვალა 41 წლის ასაკში გულის დაავადებით. დედა და 3 ძმა ცოცხალია და ჯანმრთელ მდგომარეობაში არიან.

ფიზიკალური გამოკლვევის დროს ავადმყოფის წნევა არის 110/62, გულისცემის სიხშირე 95. პულსები სიმეტრიულია და კარგი ავსების. აუსკულტაციით აღენიშნება შუასისტოლური შუილი გულის მწვერვალის მოსასმენ ადგილზე. აღენიშნება ასევე S4 გალოპი. ეკგ-ზე გამოხატულია მარცხენა პარკუჭის ჰიპერტროფია. ექოკარდიოგრაფია უჩვენებს საშუალო სიძიმის მარცხენა პარკუჭის კონცენტრულ ჰიპერტროფიას.

რომელი გამოკლვევა მოგვცემს საჭირო და შესაბამის ინფორმაციას (ანუ რომელი გამოკლვევაა "რელევანტური" ამ შემთხვევაში):

1) ენდომიოკარდიუმის ბიოფსია
2) მაგნიტურ რეზონანსური იმიჯინგი
3) ავადმყოფის დედმამიშვილების ექოკარდიოგრაფია
4) ვირუსული ტიტრები
5) 24 საათიანი შარდის შეგროვება კატექოლამინებისა და მისი მეტაბოლიზმის პროდუქტების განსაზღვრისათვის.

Posted by: mika9 20 Dec 2008, 14:50
მე მეორეს ვპიქრობ?????

MღI...............................................................

Posted by: vano_t 20 Dec 2008, 15:26
QUOTE
მე მეორეს ვპიქრობ?????

MღI...............................................................

რატომ?

Posted by: LUKA-BRAZI 20 Dec 2008, 15:26
vano_t
კითხვა ანამნეზიდან: კონკრეტულად გულის რა დაავადებით დაეღუპა დედა? user.gif

Posted by: vano_t 20 Dec 2008, 15:32
QUOTE (LUKA-BRAZI @ 20 Dec 2008, 15:26 )
კითხვა ანამნეზიდან: კონკრეტულად გულის რა დაავადებით დაეღუპა დედა? user.gif

დედა არა, მამა. თანაც ეს ტესტია და არ არის მოცემული ტესტში ეგ ამბავი. თუმცა, თვითონ ქეისიდან ჩანს სავარაუდოდ რა დაავადება უნდა ყოფილიყო.

Posted by: LUKA-BRAZI 20 Dec 2008, 15:36
vano_t
biggrin.gif და საერთოდ რა ჭირს რო? biggrin.gif

Posted by: vano_t 20 Dec 2008, 15:41
QUOTE (LUKA-BRAZI @ 20 Dec 2008, 15:36 )
vano_t
biggrin.gif და საერთოდ რა ჭირს რო? biggrin.gif

ესაო, ისაო, შენი დიდი ხათრი მაქვს, მაგრამ არაო biggrin.gif

აბა გამოიცანი რა ჭირს და რა ჭირდება wink.gif

Posted by: LUKA-BRAZI 20 Dec 2008, 15:43
vano_t
lol.gif
ან აორტული სტენოზი, ან მიტრალური ნაკლოვანება, რახან სისტოლური შუილია და პარკუჭიც ჰიპერტროფირებული.... ჰა, არა? biggrin.gif

Posted by: vano_t 20 Dec 2008, 15:53
QUOTE
ჰა, არა? biggrin.gif

ჰა? არა D

Posted by: LUKA-BRAZI 20 Dec 2008, 16:07
vano_t
რეიზა? biggrin.gif კარგი ბატონო მედიცინის მამავ smile.gif მოგვიტევე უვიცობა მაგრამ მაგ ანამნეზიდან რაიმე სპეციფიურზე უნდა ვიფიქროთ? სეპტიკური ენდოკარდიტი რომ იყოს ექო ვეგეტაციებსაც გამოავლენდა და კლინიკურ სურათში სხვა სიმპტომებიც იქნებოდა.... შუილს კიდე რა იწვევს? ძგიდის დეფექტი? შეუხორცებელი ბოტალი ან ოვალური ხვრელი? მაგრამ პაციენტი 26 წლისაა.... აბა რა დარჩა, მიქსომა სარქველის მახლობლად, რომელიც მის დეფორმაციეს იწვევს? იქნებ გული არაა აქ წამყვანი და ეს მეორადი სიმპტომატიკაა სხვა დაავადების? მარა ჰიპერტროფია შუილთან ერთად? S4-s ქართულად რა შეესაბამება?
რომელიმე ამ კითხვაზე მაინც მიპასუხე და მადლიერი დაგრჩები biggrin.gif

Posted by: zviadcardio 20 Dec 2008, 16:14
vano_t
ექოზე სხვა რა ხდება საინტერესო: LV Outflow ტრაქტი რა მდგომარეობაშია? გრადიენტი როგორია?

Posted by: vano_t 20 Dec 2008, 16:24
QUOTE
რეიზა? biggrin.gif კარგი ბატონო მედიცინის მამავ smile.gif
მედიცინის მამა ჰიპოკრატე იყო, მე რა შუაში ვარ biggrin.gif ეგ მართლა შეეშალა ბლაინდ ტორჩა კილს, მარა აღარ ავტეხე ალიაქოთი smile.gif

QUOTE
S4-s ქართულად რა შეესაბამება?
მე-4 ტონი და გალოპის დროს 4-1 2 ხმა გექნება ტაქიკარდიის ფონზე. S3 გალოპის დროს გაქვს 1 2-3. ამას იმიტომ არ გიხსნი, რომ არ იცი. ვიცი რო იცი, მარა უფრო კარგად რომ გადმოვცე რა იგულისხმება S4 გალოპში იმიტომ ვხლართავ ასე biggrin.gif

QUOTE
მოგვიტევე უვიცობა მაგრამ მაგ ანამნეზიდან რაიმე სპეციფიურზე უნდა ვიფიქროთ? სეპტიკური ენდოკარდიტი რომ იყოს ექო ვეგეტაციებსაც გამოავლენდა და კლინიკურ სურათში სხვა სიმპტომებიც იქნებოდა.... შუილს კიდე რა იწვევს? ძგიდის დეფექტი? შეუხორცებელი ბოტალი ან ოვალური ხვრელი? მაგრამ პაციენტი 26 წლისაა.... აბა რა დარჩა, მიქსომა სარქველის მახლობლად, რომელიც მის დეფორმაციეს იწვევს? იქნებ გული არაა აქ წამყვანი და ეს მეორადი სიმპტომატიკაა სხვა დაავადების? მარა ჰიპერტროფია შუილთან ერთად?
ამდენ მსჯელობას (რაც სწორია) ისიც მიაყოლე, რომ აორტის სტენოზიც და მიტრალური ნაკლოვანებაც გამოჩნდებოდა ექოზე. თანაც, მიტრალური ნაკლოვანების და სტენოზის დროს შუილი ჰოლოსისტოლური იქნება როგორც წესი. სტენოზის დროს შუილება კაროტიდებისაკენ ვრცელდება ორივე მხარეს.

რამ შეიძლება მოგცეს ახალგაზრდა ასაკში მარცხენა პარკუჭის ჰიპერტროფია, როცა მეორადი მიზეზები არ არსებობს (მაგალითად ჰიპერტენზია, ან აორტის სარქვლის სტენოზი, ან მიტრალური ნაკლოვანება)?

Posted by: zviadcardio 20 Dec 2008, 16:30
vano_t
ჩემი შეკითხვა ვერ ნახეთ ალბათ...
QUOTE
ექოზე სხვა რა ხდება საინტერესო: LV Outflow ტრაქტი რა მდგომარეობაშია? გრადიენტი როგორია?

HCM-ს ვფიქრობ, მაგრამ რაკი დიაგნოზისთვის მთლად საკმარისი ინფორმაცია არ იდო, შეკითხვა დავსვი...

Posted by: LUKA-BRAZI 20 Dec 2008, 16:31
vano_t
QUOTE
მედიცინის მამა ჰიპოკრატე იყო, მე რა შუაში ვარ biggrin.gif

როგორც ამბობენ ჰიპოკრატემ მიითვისა ბევრი რამე სხვისიო.... smile.gif რა ვიცი.... ისე მე ზა ტორჩა ქილის ინიციატივაზე! biggrin.gif კითხვაზე კი ვფიქრობ ჯერ biggrin.gif biggrin.gif

Posted by: vano_t 20 Dec 2008, 16:34
zviadcardio
QUOTE
ექოზე სხვა რა ხდება საინტერესო: LV Outflow ტრაქტი რა მდგომარეობაშია? გრადიენტი როგორია?

ეგ არის გამოცდის ტესტი და მასეა მოცემული. დავუშვათ ექოზე ციტა დიასტოლური დისფუნქცია გაქვს და დინება ცოტათი აჩქარებულია LV outflow tract-ის გასწვრივ. შენ უკვე იცი დიაგნოზი, მაგრამ ამ ქეისში იმდენად დიაგნოზზე არ არის აქცენტი გაკეთებული, რამდენადაც შემდეგში რა უნდა გააკეთო იმაზე.

QUOTE
HCM-ს ვფიქრობ, მაგრამ რაკი დიაგნოზისთვის მთლად საკმარისი ინფორმაცია არ იდო, შეკითხვა დავსვი...
კი ჰიპერტროფული კარდიომიოპათიაა. კეისი ტიპიურია მოცემული. მამა ახალგაზრდა ასაკში უცებ გარდაიცვალა გულის დაავდებით. ექოზე ჰიპერტოფიაა მარცხენა პარკუჭის და ავადმყოფს ჰიპერტროფიის მიზეზი არ აქვს. თანაც სინკოპეთი მოდის. ექოზე არ არის მთლიანად ყველაფერი მოცემული, მართალი ხარ.

მარა, როგორც ვთქვი, აქ იმდენად სავარაუდო დიაგნოზზე არ არის საუბრაი, რამდენადაც მერე რა უნდა ვქნათ.

Posted by: LUKA-BRAZI 20 Dec 2008, 16:38
zviadcardio
HCM რაღაა? თსსუ-ს არქაულ წიგნებში ეს აბრევიატურები არ წერია და არ ვიცი ამიტომ user.gif

Posted by: zviadcardio 20 Dec 2008, 16:41
LUKA-BRAZI
HCM - ჰიპერტროფიული კარდიომიოპათია.

Posted by: LUKA-BRAZI 20 Dec 2008, 16:51
თუ დიაგნოზი დაისვა MRI-მ ახალი რა უნდა გვითხრას? ხოლო დედმამისვილების გამოკვლევა გვეტყვის ეს გენეტიკურად პირობადებული HCM-ია თუ არა, რადგანაც ამ უკანასკნელს ინტრამურული სისხლძარღვების დაზიანება ახასიათებს, ხოლო ჰისტოლოგიურად კარგიომიოციტების უსწორმასწორო განლაგება....
* * *
უფრო სწორად ბიოფსია გვეტყვის ამას, მაგრამ რეალურად ეს ტესტი კეთდება პრაქტიკაში?

Posted by: zviadcardio 20 Dec 2008, 16:56
სუბარია იმაზე, თუ რომელი გამოკვლევა დაგვეხმარება დიაგნოზის დაზუსტებაში. (მე ასე გავიგე)

აი შეკითხვა:
რომელი გამოკლვევა მოგვცემს საჭირო და შესაბამის ინფორმაციას (ანუ რომელი გამოკლვევაა "რელევანტური" ამ შემთხვევაში):

1) ენდომიოკარდიუმის ბიოფსია
2) მაგნიტურ რეზონანსური იმიჯინგი
3) ავადმყოფის დედმამიშვილების ექოკარდიოგრაფია
4) ვირუსული ტიტრები
5) 24 საათიანი შარდის შეგროვება კატექოლამინებისა და მისი მეტაბოლიზმის პროდუქტების განსაზღვრისათვის.

პასუხი:
ჩვეულებრივ ექო დიდწილად წყვეტს დიაგნოზს, მის შემდეგ შეიძლება მაგნიტურ რეზონანსის გამოყენება დიაგნოზის დასაზუსტებლად.


აი დოკუმენტიც:
Clinical diagnosis of HCM is usually made with two-dimensional echocardiographic imaging. Cardiac magnetic resonance imaging (CMR) has an expanding role in the noninvasive diagnosis of HCM by virtue of its high-resolution tomographic imaging capability. CMR is complementary to echocardiography by clarifying technically ambiguous LV wall thicknesses or visualizing areas of segmental hypertrophy (specifically, in the anterolateral free wall; or by delineating LV apical pathology, including hypertrophy and aneurysm formation, often not identifiable with echocardiography.

Posted by: vano_t 20 Dec 2008, 17:11
ბიოფსია და MRI აღარ ჭირდება ამ ავდმყოფს. ამ დაავადებას კლინიკურად სვამენ დიაგნოზს (ექოს და ეკგ-ს გამოკვლევების ჩათვლით). MRI შეიძლება გაუკეთდეს, მაგრამ MRI-ს ღირებულება არ არის დიაგნოზის დასმა, არამედ იმ პაციენტების ამორჩევა, რომელთაც ფიბროზი აქვთ ჩამოყალიბებული, ან როცა ექო ტექნიკურად არ არის გამართული (ჩვენს შემთხვევაში არის) მაშინ:
QUOTE
Cardiac MRI is helpful in identifying patients with fibrosis and as an adjunct evаluation of anatomy and outflow tract obstruction in patients with poor echocardiographic windows.

    * Areas of delayed enhancement on MRI correlate with areas of fibrosis and can be found in a subset of patients with hypertrophic cardiomyopathy. These patients may be at increased risk for arrhythmias including nonsustained ventricular tachycardia.
    * Findings of fibrosis may have implications for risk stratification for sudden cardiac death in this group of patients.


HCM-ები ავტოსომურ დომინანტური დაავადებებია. ამიტომ, როცა ერთ პაციენტში დასვი დიაგნოზი, გარკვეული ნათესავების სკრინინგი უნდა მოახდინო ექოთი და ეკგ-თი: ჩვენს შემთხვევაში დედმამიშვილების, რადგანაც თითოეულ მათგანზე ამ დაავადების დამემკვიდრების შანსი არის 50 %.


მანამდე ახალ კეისს დავდებ და წავალ დასაძინებლად smile.gif

64 წლის მამაკაცი მოდის ძლიერი გულმკერდის არის ტკივილით. ეკგ-ზე დაყრდნობით და ენზიმების ანალიზით ხდება ST-ს ელევაციით მიმდინარე მიოკარდიუმის ინფარქტის დიაგნოზი. ავადმყოფ იღებენ კორონარულ განყოფილებაში და მიღებიდან მალე უვითარდება სუნთქვის უკმარისობა, ციანოზი, wheezing (ამას ქართულად მემგონი მსტვენავი ხიხინი ქვია, ასთმის დროს რომ არის) და დიფუზური rales (ამასაც ხიხინს ეძახიან ქართულში მე მგონი, ოღონდ სველ ხიხინს, აი იმას ფილტვების შეშუპების დროს რომ ისმის მაგალითად-თუ ვცდები ტერმინოლოგიაში, შეასწორეთ თუ შეიძლება). ფიზიკური გამოკვლევისას, პაციენტი კითხვებს არ პასუხობს (თითქმის unresponsive არის). პულსი რეგულარულია და 120-ია წუთში. წნევა 90/60. გახანგრძლივებული ციანოზის გამო ავადმყოფს იწყებენ ჟანბადზე (face mask-ით, რომელიც საშუალებას იძლევა მაღალი კონცენტრაციის ჟანგბადის მიწოდებისა). არტერიული გაზები ოთახის ჟანგბადის კონცენტრაციაზე (ანუ სანამ ჟანგბადზე დაიწყებდნენ) არის: PCO2-63 mm Hg (ნორმაა 40), PO2-41 mm Hg (ნორმაა 90-100), PH-7.10 (ნორმაა 7.4) და გამოთვლილი HCO3-20.5 meq/L (ნორმაა 24). მოცემული მონაცემების საფუძველზე რა სახის მჟავა ტუტპვანი პრობლემა აქვს ავადმყოფს?

1) რესპირატორული აციდოზი მეტაბოლური კომპენსაციით
2) რესპირატორული აციდოზი მეტაბოლური კომპენსაციის გარეშე
3) კომბინირებული რესპირატორული აციდოზი და მეტაბოლური ალკალოზი
4) ცომბინირებული რესპირატორული აციდოზი და მეტაბოლური აციდოზი
5) მეტაბოლური აციდოზი ნაწილობრივი რესპირატორული კომპენსაციით

პასუხი უნდა ახსნათ და თუ იმასაც მიამატებთ, რის გამოა მოცემული პრობლემა, ძალიან კარგი იქნება. აბა დროებით smile.gif

Posted by: LUKA-BRAZI 20 Dec 2008, 18:32
vano_t
მართალი გითხრა ამ ქეისის პასუხი დანამდვილებით არ ვიცი.... ის კი არა ერთდროულად ალკალოზი და აციდოზი როგორ უნდა იყოს ეგეც ვერ წარმომიდგენია biggrin.gif ამას რომ აციდოზი აქვს ეს ფაქტია, თან საკმაოდ კარგი აციდოზი smile.gif ამიტომ ვფიქრობ პასუხი უნდა იყოს #4. რადგანაც შენ პასუხები ახსნის გარეშე არ გიყვარს, შევეცდები ავხსნა smile.gif
მაშ ასე:
ამ პაციენტს აქვს ინფარქტი, ალბათ დიდი ზომის, დაემართა გულის უკმარისობა განდევნის ფრაქციის დავარდნის გამო, რის შედეგადაც ფილტვებში შეგუბებაა, შესაძლებელია ბრონქოსპაზმიც, ჯამში ირღვევა სისხლის ოქსიგენეცია და ვითარდება რესპრატორული აციდოზი. მეტაბოლური ფაქტორი არის თუ არა? შესაძლებელია LDH-ის და სხვა უჯრედშიდა ნივთიერებების დიდი რაოდენობით სისხლში გადასვლა ნეკროზის უბნიდან... ეს ისედაც ხდება ინფარქტის დროს, მაგრამ რამდენად შესაძლებელია ეს ასეთი ძლიერი მეტაბოლური აციდოზის ფაქტორი გახდეს? მარა მაინც... ანუ მინდა ვთქვა რომ სისხლში ნახშირორჟანგის და უჯრედული პროდუქტების მატება იწვევს PH-ის გადახრას აციდოზისკენ + სისხლის ბიკარბონატული ბუფერული სისტემის დეფიციტიცაა... ასე რომ მეტაბოლური და რესპირატორული ალკალოზი გვაქვს.... თუ ვცდები, მითხარი სად ვცდები smile.gif


ისე აციდოზ-ალკალოზის ერთდროულ არსებობაზე პატარა ლექციას სიამოვნებით მოვისმენდით smile.gif

Posted by: წრიპა 20 Dec 2008, 19:04

QUOTE (LUKA-BRAZI @ 20 Dec 2008, 18:32 )

ისე აციდოზ-ალკალოზის ერთდროულ არსებობაზე პატარა ლექციას სიამოვნებით მოვისმენდით smile.gif

yes.gif yes.gif yes.gif მეც დიდი სიამოვნებით მოვისმენდი

Posted by: badu 20 Dec 2008, 20:14
ლუკას ვეთანხმები . . .

Posted by: Blind_Torture_Kill 20 Dec 2008, 20:27
QUOTE
4) ცომბინირებული რესპირატორული აციდოზი და მეტაბოლური აციდოზი



Posted by: LUKA-BRAZI 20 Dec 2008, 21:44
vano_t
უი! მექანიკური შეცდომა გამპარვია biggrin.gif :
QUOTE
ასე რომ მეტაბოლური და რესპირატორული ალკალოზი გვაქვს

მინდოდა მეთქვა აციდოზი-თქო smile.gif 1000 excuses! biggrin.gif

Posted by: Blind_Torture_Kill 21 Dec 2008, 01:05
ჩემი თირკმლის ქეისი ჩაკალით ? user.gif

Posted by: vano_t 21 Dec 2008, 09:26
LUKA-BRAZI
QUOTE
ისე აციდოზ-ალკალოზის ერთდროულ არსებობაზე პატარა ლექციას სიამოვნებით მოვისმენდით

კარგი მაშინ. მაშინ ასე ვიზამ. ჯერ მოკლედ დავწერ მჟავა ტუტოვანი წონასწორობის ფიზიოლოგიაზე (იმდენს რაც საჭიროა ქეისებსი ანალიზისათვის) და მერე დავდებ ალგორითმს იმისას თუ როგორ უნდა გაანალიზო ქეისები.


მჟავა-ტუტოვანი წონასწორობის ფიზიოლოგია
ორგანიზმში წყალბადი იონების კონცნეტრაციის ჰომეოსტაზი ნარჩუნდება ძალიან ვიწრო ფარგლებში. PH დაალოებით 7.35-7.45-ის ფარგლებში კონტორლდება. მჟავები (ანუ წყალბადის იონები H+) განუწყვეტლივ წარმოიქმნება და შესაბამისად რა რაოდენობითაც წარმოიქმნება, იმ რაოდენობით უნდა იქნას მოცილებული. სხვანაირად მოხდება წონასწორობის დარღვევა. ყველაზე მნიშვნელოვანი ფორმულა, რომელიც საჭიროა რომ ვიცოდეთ მჟავა-ტუტოვანი წონასწორობის გასაგებად, არის HCO3+H=H2CO3=CO2+H2O (ფორმულა #1). ამ ფორმულაში "=" ნიშანი გამოსახავს წონასწორობას ფაქტიურად. ამ ფორმულას რამოდენიმეჯერ მივმართავ.

როგორ ხდება შემაერთებელი ქსოვილი სითხის (და შესაბამისად სისხლის) მჟავიანობის შეცვლა? მჟავიანობის შეცვლა ხდება ორი მიმართულებით: 1) მჟავიანობით მომატებით (ანუ PH-ის დაკლებით) და 2) მჟავიანობის დაკლებით, ანუ ტუტეობის მომატებით (ანუ, PH-ის მომატებით). თითოეული მიზეზი თავისთავად 2 ნაწილად დაიყოფა. მჟავიანობის მომატება მოხდება თუ A) მჟავას (H+) შეიძენს ინტერსტიციუმი ან B) ტუტეს (HCO3-) დაკარგავს ინტერსტიციუმი. ზუსტად ასე ყალიბდება პათოლოგიაში მჟავიანობის მომატება. ასევეა მჟავიანობის დაქვეითებაც. ისიც ორნაირად ხდება: C) როცა მჯავას (H+) კარგავს ინტერსტიციუმი ან D) როცა ტუტეს (HCO3-) იძენს ინტერსტიციუმი.

ახლა შევხედოთ ფორმულა #1-ს: HCO3+H=H2CO3=CO2+H2O. ამ ფორმულიდან ჩანს, რომ მჟავის შეძენა შეიძლება მოხდეს CO2-ის მომატებით, რაც წონასწორობას გადაწევს მარცხნისაკენ. წონასწორობაზე გამახსენდა biggrin.gif მარტივი პრინციპია ეს და უნდა დაიმახსოვროთ ვისაც არ გახსოვთ: მასათა ქმედების კანონი (ლე-შატელიეს პრინციპი იგივე პონტია) ამბობს, რომ თუ წონასწორულ სისტემას რაღაცნაირად შევცვლით, მაშინ სისტემაში ყველაფერი ისე წავა, რომ "შეცვლა" გამოასწოროს და წონასწორობა დაიბრუნოს. ჩვენს შემთხვევაში, თუ მარჯვენა მხარის ნაერთები მოიმატებს (რაც არის CO2 და H2O), მაშინ პროცესები ისე წავა, რომ ამ ნაერთების მომატება შემცირდეს; ანუ, რეაქცია გადაიხრება მარცხნივ HCO3- და H+ წარმოქმნისაკენ. ე.ი. CO2-ის მომატება არის მჟავიანობის მომატების ანალოგიური. ასე ხდება როცა გაქვს პირველადი რესპირატორული აციდოზი. პირველადი რესპირატორული აციდოზის დროს ხდება ჰიპოვენტილაცია სხვადასხვა მიზეზების გამო. ამას მოყვება CO2-ის მომატება და შესაბამისად მჟავიანობის მომატება. მჟავიანობის მომატება კიდევ მოხდება ორნაირად. მარცხენა მხარეს დააკვირდით. თუ მარცხენა მხარეში მოიმატებს H+, მაშინ ეს გამოიწვევს მჟავიანობის მომატებას და თანაც, ამ შემთხვევაში მოხდება რეაქციის გადანაცვლება მარჯვნისაკენ, რათა მომატებული H+ დაიკლოს (ლე-შატელიეს პრინციპი). ასეთი აციდოზი ხდება როცა ინტერსტიციუმს ემატება რაიმე მჟავა (მაგალითად ლაქტატ-მჟავა, ან ქლორმჟავა). ეს უკვე მეტაბოლური პროცესია და არა რესპირატორული. მეორე შემთხვევა, როცა მეტაბოლური აციდოზი ვითარდება, არის მარჯვენა მხარეს HCO3- იკლებს. ამ შემთხვევაში, ჩვენი რეაქცია გადაიხრება მარცხნისაკენ, ანუ HCO3- და H+ წარმოქმნისაკენ. HCO3-ის დაკარგვა შეიძლება მოხდეს მაგალითად ძლიერი დიარეის დროს. ამ დროს ხდება მსხვილი ნაწლავიდან დიდი რაოდენობით ბიკარბონატის დაკარგვა. ეს კიდევ შეიძლება მოხდეს სხვადასხვა ფისტულების დროს, ან თირკმლიდან მოხდეს ბიკარბონატის დაკარგვა (სხვადასხვა ტუბულარული აციდოზების დროს). ესეც მეტაბოლური პროცესია.

შევხედოთ ისევ ამ ფორმულას: HCO3+H=H2CO3=CO2+H2O. როგორ შეიძლება მოხდეს ტუტიანობის მომატება? 1) რესპირატორული გზით, როცა CO2 ეცემა ჰიპერვენტილაციის გამო (რასაც თავის მხრივ სხვადასხვა მიზეზები აქვს). CO2 დაკლება გამოიწვევს ისეთი პროცესების ჩართვას, რომ CO2 ნორმისკენ წამოვიდეს. ეს კი ნიშნავს ამ რეაქციის მარჯვნივ გადახრას და შესაბამისად მარცხენა მხარის პროდუქტების შემცირებას, მათ შორის H+ისა, რაც ალკალოზის ტოლფასია. 2) ტუტიანობის მომატება შეიძლება მოხდეს მეტაბოლური გზით ორნაირად ისევ. პირველი, როცა მჟავას (ანუ H+) კარგავ. ეს შეიძლება მოხდეს მაგალითად ღებინების დროს. ამ დროს ქდება ქლორმჟავას დაკარგვა. მეორე, როცა ხდება ბიკარბონატის შეძენა ინტერსტიციუმში. ეს შეიძლება მოხდეს მაგალითად თირკმლის მიერ, როცა ბიკარბონატის გენერაცია ხდება და მერე მისი შეწოვა ხდება სისხლში. ან კიდევ ე.წ. კონტრაქციული ალკალოზის დროს (დეჰიდრატაციის ფონზე).

ახლა უკვე საკმარისი ინფორმაცია გვაქვს, რომ შენს კითხვას გავცეთ პასუხი და ამასთანავე მნიშვნელოვანი პრინიციპი ჩამოვაყალიბოთ:

1) რადგანაც რესპირატორული პროცესებით მხოლოდ ერთდაგვარად ხდება მჟავინაობის შეცვლა (ან CO2-ის მომატებით ჰიპოვენტილაციის დროს, ან CO2-ის დაკლებით ჰიპერვენტილაციის დროს), ამიტომ რეპირატორული აციდოზი და ალკალოზი ერთდროულად ვერ იარსებებს!

მეტაბოლური ცვლილებები, როგორც ვნახეთ, შეიძლება მოხდეს სხვადასხვა ორგანოში სხვადასხვა პროცესების წყალობით. შესაბამისად, როცა ეს პროცესები პარალელურად მიმდინარეობს, გექნება კომბინირებული პრობლემა. მაგალითად, დავუშვათ ავადმყოფს აქვს ძლიერი დიარეა, ფაღარათი, უვითარდება შოკი, ამასთანავე უსხამ დიდი რაოდენობით კრისტალოიდებს და თან აქვს თიკრმლის პათოლოგია, რომელიც იწვევს ბიკარბონატის დაკარგვას (პროქსიმალური ტუბულარული აციდოზი). რა გექნება ამ დროს? 1) მეტაბოლური აციდოზი (იმის გამო, რომ კარგავს ბიკარბონატს დიარეის დროს, ასევე თირკმლებით კარგავს ბიკარბონატს); 2) ისევ მეტაბოლური აციდოზი, იმის გამო, რომ დიდი რაოდენობით NaCl-ის გადასხმა იწვევს ქლორმჯავას გენერაციას; 3) #1 და #2 გაძლევს ე.წ. არა-anion gap აციდოზს (სხვანაირად ამას უწოდებენ ჰიპერქლორემიულ აციდოზს). ამათ გარდა შოკის დროს მოხდება ე.წ. anion gap (ამას დავარქვათ ანიონური სხვაობა) აციოდოზის ჩამოყალიბება, იმის გამო რომ მოხდება ლაქტატისა და სხვა მჟავების გენერაცია ანაერობული მეტაბოლიზმის გაძლიერების გამო; 4) მეტაბოლური ალკალოზი, იმის გამო რომ ხდება მჟავას დაკაგვა ღებინების ფონზე. აქ უკვე რამოდენიმე მეტაბოლური პრობლემა გაქვს ერთად.

2) მოკლედ, უნდა გავაცნობიეროთ, რომ მეტაბოლური/რესპირატორული აციდოზი/ალკალოზი არის პროცესი (ან პროცესები) და თუ შესაძლებელია რამოდენიმე პროცესის ერთად მოხდენა, მაშინ შესაძლებელია მეტაბოლური/რესპირატორული ალკალოზი/აციდოზის სხვადასხვა ფორმების ერთად არსებობა! თეორიულად შესაძლებელია 5 პროცესის თანაარსებობა: ერთი რესპირატორული პროცესისა (ან ალკალოზის, ან აციდოზის, მაგრამ არა ორივესი ერთად, რაც უკვე ავღნიშნეთ) და 2 მეტაბოლური პროცესისა (მეტაბოლური აციდოზის და მეტაბოლური ალკალოზის, ამასთანავე არა-ანიონური სხვაობის და ანიონური სხვაობის აციდოზების ერთად არსებობით). ეს უკვე სცემს შენს კითხვას პასუხს.

ახლა გადავედით კიდევ რამოდენიმე პრინციპზე. თითოეული პროცესი (მეტაბოლური აციოდი ან ალკალოზი და რესპირატორული აციდოზი და ალკალოზი) შეიძლება იყოს ან პირველადი, ან მეორადი, როგორც კომპენსატორული პროცესი. ამათ განსხვავებას ალგორითმის დროს დავწერ. ახლა დაიმახსოვრეთ 2 რამ:

3) კომპენსატორული პროცესი არასდროს მოახდენს ზეკომპენსაციას! ეს იმას ნიშნავს, რომ თუ დავუშვათ გაქვს მეტაბოლური აციდოზი (რომელიც PH-ს დაწევს 7.4-ის დაბლა) და ჩაირთვება რესპირატორული კომპენსაცია ალკალოზის სახით, მაშინ ამ პროცესს არ შეუძლია PH ნორმას დაუბურნოს (ან უფრო მეტიც, 7.4-ს გადააჭარბოს). თუ დაადგინე, რომ დავუშვათ გაქვს პირველადი მეტაბოლური აციდოზი და ამასთანავე PH არის 7.4-ის მაღლა (იმის მაგივრად, რომ იყოს დაბლა), მაშინ რესპირატორული ალკალოზიც პირველადი პროცესია და არა კომპენსაცია. მოკლედ, საკომპენსაციო პროცესით ზეკომპენსაცია არასდროს ხდება!

4) ანიონური სხვაობის აციდოზი არასდროს იქნება საკომპენსაციო პროცესი! ეს იმას ნიშნავს, რომ თუ დაადგინე ანიონური სხვაობის აციდოზის არსებობა, ეს არის პირველადი პროცესი. ამას ცოტას განვავრცობ. ანიონური სხვაობა გამოითვლება ასე: Na-(Cl+HCO3) (ან ასე (Na+K)-(Cl+HCO3)). თუ "Na-(Cl+HCO3)" ფორმულას გამოიყენებთ, მაშინ ნორმა ჩათვალეთ 12. თუ "(Na+K)-(Cl+HCO3)" ფორმულას გამოიყენებთ, მაშინ ნორმა ჩათვალეთ 16 (იმის გამო, რომ ნორმალური K-ს რაოდენობა 4 მილიექვივალენტია, ეს გაგიზრდის მაგ ჯამსაც 4-ით). მე ვიყენებ "Na-(Cl+HCO3)" ფორმულას, რაც ძალიან მარტივია. ანიონურ სხვაობას განაპირობებს სისხლში იმ მჟავების არსებობა, რომელიც განსხვავებული ქლორმჟავასაგან (ფოსფორმჟავა, ლაქტატი, ორგანული მჟავები იქნება თუ გოგირდმჟავა და ა.შ.). რაც უფრო მაღალია ამ მჟავების კონცენტრაცია, მით უფრო მაღალია ანიონური სხვაობა. ე.ი. ანიონური სხვაობა არაპირდაპირ მიუთეთებს ქლორმჟავასაგან განსხვავებული მჟავების რაოდენობაზე. ბევრჯერ ამას ძალიან დიდი მნიშვნელობა აქვს. მაგალითად, დიაბეტური კეტოაციდოზის დროს იმდენად იმატებს კეტო-მჟავები, რომ ანიონური სხვაობა 25-30 შეიძლება იყოს. ეს თავისთავად დიაგნოზზეც მეტყველებს და მკურნალობის შედეგიანობასაც ანიონური სხვაობის დახურვით (ანუ ნორმაში ჩამოსვლით) ამოწმებ. ანიონური სხვაობის დათვლა ყოველთვის მნიშვნელოვანია, რამეთუ დიაგოზის დადგენაში გეხმარება. ახლა რაც შეეხება უშუალოდ მე-4 პრინციპს. როცა მეტაბოლური აციდოზი საკმოპენსაციო პროცესია (რეპირატორული ალკალოზის საკომპენსაციო პროცესი), ეს ხდება თირკმელში ბიკარბონატის შეწოვის დაქვეითებით. ანუ, ამ დროს ხდება ტუტის დაკარგვა მომატებული ტუტის საპასუხოდ. არასდროს არ ხდება რესპირატორული ალკალოზის საკომპენსაიცოდ ქლორმჟავისაგან განსხვავებული მჟავის გენერაცია. ამიტომ, თუ გაქვს ანიონური სხვაობის აციდოზი, ეს ყველა ვარიანტში პირველადი პროცესია და არა საკომპენსაციო.

5) რესპირატორული პროცესის კომპენსაცია ხდება ყოველთვის მეტაბოლური პროცესით და მეტაბოლური პროცესის კომპენსაცია ხდება ყოველთვის რესპირატორული პროცესით. თანაც, აციდოზის კომპენსაცია ხდება ალკალოზით და ალკალოზის კომპენსაცია ხდება აციდოზით! ე.ი. თუ გაქვს პირველადი პროცესი რესპირაოტული აციდოზი, მაშინ კომპენსაცია მოხდება მეტაბოლური ალკალოზით. თუ გაქვს პირველადი პროცესი მეტაბოლური ალკალოზი, მაშინ კომპენსაცია ხდება რესპირატორული აციდოზით და ა.შ. თუ გაქვს მაგალითად მეტაბოლური ალკალოზი და აციდოზი ერთად, მაშინ რესპირატორული კომპენსაცია მოხდება იმის მიხედვით თუ რომელი პროცესი გადაწონის. თუ მეტაბოლური ალკალოზი გადაწონის, მაშინ საერთო ჯამში ალკალოზი ჩამოყალიბდება და თუ კომპენსაცია მოხდა, უნდა მოხდეს რესპირატორული აციდოზით.

6) და ბოლო პრინციპი: ნებისმიერი პროცესი (მეტაბოლური აციდოზი/ალკალოზი და რეაპირატორული აციდოზი/ალკალოზი) შეიძლება იყოს მწვავე და ქრონიკული! "მწვავე და ქრონიკული" აქ ცოტა სხვა მნიშვნელობით იხმარება. ქრონიკული რპოცესი შეიძლება რამოდენიმე საათისაც იყოს. საქმე იმაშია, რომ მწვავეს დაარქმევ პროცესს, როცა ჯერ კომპენსაცია არ ჩართულა და ქრონიკულს დაარქმევ პროცესს, რომლის სრული კომპენსაცია მოხდა. გარდა ამისა, "მწვავე" და "ქრონიკული" შეიძლება იხმარებოდეს ტრადიციული გაგებითაც. ანუ, როცა პროცესი ქრინიკულად მწვავედ ან ქრონიკულად მიმდინარეობს. თუ რა მნიშვნელობა დევს ამ სიტყვებში, ეს კონტექსტით უნდა გავარკვიოთ.

ეს არის მთლიანად ის ფიზიოლოგია, რაც მჟავა-ტუტოვანი ქეისების გასაკეთებლად არის საჭირო. ამაზე მეტი ფიზიოლოგია არ არის საჭირო.


ალგორითმი
მჯავა-ტუტოვანი ქეისების დროს გარკვეული რაღაცეები ყოველთვის უნდა დაითვალოს და გაანალიზდეს. გარდა ამისა, ანალიზის დროს გარკვეული ნორმები გაქვს მხედველობაში. თავიდან უნდა მიეჩვიოთ ამ ნორმების "მკაცრი" მნიშვნელობებიდან ანალიზს. ეს მნიშვნელობები ასეთია:

PH-7.4

HCO3-24 meq/L (ანუ მილიექვივალენტი/ლიტრში)

PO2-80-100 mm Hg ეს არის ჟანგბადის ნორმალური პარციალური წნევა. თავისთავად ეს სიდიდე არ გვეხმარება მჟავა-ტუტოვანი წონასწორობის ტიპის დადგენაში, მაგრამ გვეხმარება რომ მიზეზი დავადგინოთ ამა თუ იმ პროცესის არსებობს.

PCO2-40 mm Hg

Na-(Cl+HCO3) ანუ ანიონური სხვაობა, რაც ნორმაში უნდა იყოს 12.

ახლა გადავედით უშუალოდ ალგორითმზე:

1) ჯერ დავადგენთ საერთოდ რა გვაქვს, აციდოზი თუ ალკალოზი. ამისათვის ვუყურებთ PH-ს. თუ PH 7.4-ის მაღლაა, მაშინ გვაქვს ალკალოზი, თუ დაბლაა-მაშინ გვაქვს აციდოზი.
ჩვენს ქეისში PH არის 7.1 და შესაბამისად გვაქს საბოლოოდ აციდოზი.

2) პირველად გავარკვიოთ რამდენია PCO2. თუ PCO2 40-ზე მაღლა დგას, მაშინ გაქვს რესპირატორული აციდოზი. თუ PCO2 40-ზე ნაკლებია, მაშინ გვაქვს რესპირატორული ალკალოზი. რა თქმა უნდა, 40-დან დიდ გადახრაზეა საუბარი. ნორმაში ისე PCO2 35-45 შეიძლება იყოს და როცა ნამდვილად რესპირატორული პროცესი გაქვს PCO2 იქნება 15-20 ან 50 და მაღლა (დაახლოებით ვამბობ, რა თქმა უნდა). ჩვენს ქეისში PCO2 არის 63 და შესაბამისად გვაქვს რესპირატორული აციდოზი.

3) ამის მერე გავარკვიოთ რესპირატორული პროცესი პირველადი პროცესია თუ კომპენსაცია. ამისათვის ვიყენებთ #3 პრინციპს: ზეკომპენსაცია არასდროს ხდება. ჩვენს შემთხევაში სისხლი აციდოზურია და ასევე გვაქვს რესპირატორული აციდოზი. ეს რესპირატორული აციდოზი საკომპენსაციო რომ ყოფილიყო, მაშინ იგი იქნებოდა მეტაბოლური ალკალოზის (რომელიც PH-ს გაზრდიდა) საკომპენსაციოდ და PH-ს 7.4-ის დაბლა ვერ ჩამოიყვანდა, რაც რომ მოხდარიყო იქნებოდა ზეკომპენსაცია. შესაბამისად, გვაქვს პირველადი რესპირატორული აციოდზი. ზოგადად, როცა PH გაქვს აციდოზური და PCO2-ც მაღალია (ანუ რესპირატორული აციდოზი გაქვს), მაშინ რესპირატორული აციდოზი იქნება პირველადი პროცესი აუცილებლად. როცა PH ალკალოზური გაქვს და PCO2-ც დაბალია (ანუ გაქვს რესპირატორული ალკალოზი), მაშინ პირველადი რესპირატორული ალკალოზი გაქვს. როცა PH აციდოზურია და გაქვს რესპირატორული ალკალოზი, მაშინ, რა თქმა უნდა, რესპირატორული ალკალოზი აციდოზის (მეტაბოლურის) საკომპენსაციოდ ხდება. ასეთ შემთხვევაში 2 კურდღელს იჭერ ერთად და იცი, რომ გაქვს მეტაბოლური აციდოზი და საკომპენსაციო რესპირატორული ალკალოზი. იგივე ითქმის, როცა PH ალკალოზურია და გაქვს რესპირატორული აციდოზი. ამ შემთხვევაში პირველადი პროცესი არის მეტაბოლური ალკალოზი და ხდება მისი კომპენსაცია რესპირატორული აციდოზით. ბოლო ორ შემთხვევაში არ ირღვევა ზეკომპენსაციის "არ შეუძლებლობის" პრინციპი.

4) გამოვთვალოთ ანიონური სხვაობა. თუ ანიონური სხვაობა მომატებულია, მაშინ ყველა ვარიანტში გვაქვს პირველადი აციდოზი მომატებული ანიონური სხვაობით და ეს პროცესი, როგორც ვთქვით არასდროს არის საკომპენსაციო. ჩვენს შემთხვევაში ელექტროლიტები არ არის მოცემული, მაგრამ შეიძლება ივარაუდო, რომ გექნება პირველადი აციდოზი მომატებული ანიონური სხვაობით. მაგალითად, ამ კეისში ავადმყოფს აქვს ჰიპოქსია (PO2 არის 40) და ამასთანავე კარდიოგენული შოკი (დაბალი წნევა ტაქიკარდიით). ეს ორივე პროცესი გამოიწვევს ქსოვილურ ჰიპოქსიას და დიდი რაოდენობით არაქლოროვანი მჟავების წარმოქმნას. ჩვენს შემთხვევაში ასპროცენტიანი ვერ ვიტყვით გვაქვს თუ არა პირველადი აციდოზი მომატებული ანიონური სხვაობით (იმიტომ, რომ ანიონურ სხვაობას ვერ გამოვთვლით ელექტროლიტების გარეშე), მაგრამ ასპროცენტიანი შეიძლება იმის თქმა, გვაქვს თუ არა პირველადი მეტაბოლური აციდოზი vs. მეტაბოლური ალკალოზი, როგორც პირველადი რესპირატორული აციდოზის კომპენსაცია.

ეს პირველი 4 პუნქტი ყველანაირი მჟავა-ტუტოვანი კეისების ანალიზისას უნდა გამოვიყენოთ. დანარჩენი საფეხურები დამოკიდებულია ამ 4 საფეხურზე რას ვიპოვით.

5) თუ გვაქვს პირველადი რესპირატორული აციდოზი (როგორიც არის ჩვენი შემთხვევა), მაშინ უნდა ნახო გაქვს თუ არა მეტაბოლური კომპენსაცია მეტაბოლური ალკალოზის სახით. აქ 3 ვარიანტია: A) ან გაქვს მეტაბოლური კომპენსაცია, რომელიც შეიძლება იყოს სრული ან არასრული; B) არ გაქვს მეტაბოლური კომპენსაცია ჯერ დაწყებული; C) არათუ არ გაქვს მეტაბოლური კომპენსაცია, არამედ პირველადი მეტაბოლური აციდოზიც ერთვის თან ამ ყველაფერს. როგორ გავარკვიოთ ეს? პირველად შევხედოთ იმას, გვაქვს თუ არა C. მოდი ავიღოთ ჩვენი პანაცეა ფორმულა ისევ: HCO3+H=H2CO3=CO2+H2O. ჩვენს შემთხვევაში მარჯვენა მხარეს ხდება CO2-ის მომატება, რამაც ლე-შატელიეს პრინციპის თანახმად სისტემაში უნდა აღძრას ისეთი ცვლილება, რომ CO2 დააგდოს დაბლა: ეს მოხდება იმით, რომ რეაქციის წონასწორობა გადაინაცვლებს მარცხნივ HCO3-ის და H-ის მომატებისაკენ. ეს იმას ნიშნავს, რომ, თუ მარტო რესპირატორული აციდოზი გაქვს, ბიკრაბონატმა უნდა აიწიოს 24-ის ზევით. ეს ყოველგვარი კომპენსაციი გარეშე. თუ კომპენსაცია მოხდა, მაშინ #5 პრინციპის თანახმად რესპირატორული აციდოზის კომპენსაცია უნდა მოხდეს მეტაბოლური ალკალოზით. ეს ფიზიოლოგიურად ხდება HCO3-ის (რაც ტუტეა) შეწოვის გაძლიერებით და შესაბამისად HCO3 კიდევ უფრო უნდა გაიზარდოს (იმის ზევით, რაც რესპირატორული აციდოზით გაიზარდა). ჩვენს შემთხვევაში კი, არათუ გაზრდილია ბიკარბონატი, არამედ ნორმის დაბლაა! ეს იმიტომ, რომ მარცხენა მხარეს მოხდა ან HCO3-ის მოცილება, ან H-ის გაზრდა (რომელსაც შემდგომ ლე-შატელიეს პრინციპის თანახმად მიყავხარ H-ის, და შესაბამისად, ბიკარბონატის შემცირებისაკენ). ეს ორივე პროცესი ტოლფასია პირველადი მეტაბოლური აციდოზის. ანუ, ჩვენს შემთხვევაში გვაქვს პირველადი მეტაბოლური აციდოზი და პირველადი რესპირატორული აციდოზი.

6) მე-5 პუნქტში ავღნიშნეთ, რომ პირველადი რესპირატორული აციდოზის დროს შეიძლება გვქონდეს 3 შემტხვევა და განვიხილეთ ბოლო ©. არ გაგვიხილავს A და B. იმისათვის, რომ დავადგინოთ კომპენსაცია არსებობს თუ არა (და თუ არსებობს სრულია თუ არა), ამისათვის უნდა გამოვიყენოთ გარკვეული ფორმულები.

როცა რესპირატორული აციდოზი მწვავეა და ჯერ კომპენსაცია არ არის დაწყებული, მაშინ PCO2-ის ყოველი 10-ით გაზრდა ნორმის ზემოთ (ანუ 40-დან), იწვევს შემდეგ ცვლილებებს:
PH მატულობს შემდეგნაირად: pH = 0.008 X (40 - PaCO2)
HCO3 იწევს 1 მილიექვივალენტით მაღლა.
ჩვენ რომ გვქონოდა მარტო პირველადი რესპირატორული აციდოზი ყოველგვარი კომპენსაციის გარეშე, მაშინ ბიკარბონატი აიწევდა 2.3-ით (თუ 10-ზე 1-ით ადის მაღლა, მაშინ 23=53-40-ზე 2.3-ით ავიდოდა მაღლა) და შესაბამისად დაახლოებით იქნებოდა 24+2.3=26.3. PH კი დაეცემოდა ნორმიდან 0.184-ით (0.008X(40-63)=-0.184) დაბლა და შესაბამისად უნდა ყოფილიყო 7.4-0.184=7.216. აქედანაც შეიძლება იმის დადგენა, რომ პირველადი მეტაბოლური აციდოზიც გვაქვს. კომპენსაცია რომ ყოფილიყო, მაშინ PH 7.216-დან უნდა წანაცვლებულიყო მაღლა. მაგრამ პირიქით მოხდა, დაეცა. პირველი მეთოდი, მაღლა რაც ვახსენე, სწრაფია და გამოთვლას არ მოიცავს.

როცა რესპირატორული აციდოზი სრულად არის კომპენსირებული, მაშინ PCO2-ის ყოველი 10-ით გაზრდას ახლავს:
HCO3-ის 3.5-ით გაზრდა
PH-ის დაქვეითება შემდეგი ფორმულით: pH = 0.003 X (40 - PaCO2)
ისევ, ჩვენ რომ გვქონოდა სრული კომპენსაცია, მაშინ ბიკარბონატი უნდა აწულიყო 2.3X3.5=8-ით და შესაბამისად უნდა ყოფილიყო 24+8=32. ასევე, PH უნდა დავარდნილიყო 0.069-ით (0.003X(40-63)=-0.069) და გამხდარიყო 7.32 (7.4-0.069). თუ PH და HCO3 არის სრულ კომპენსაციის და არაკომპენსირებულ სიდიდეებს შორის, მაშინ გაქვს ნაწილობირვი კომპენსაცია.

7) თუ გაქვს პირველადი რესპირატორული ალკალოზი, მაშინ მსჯელობა იგივეა, რაც პირველადი რესპირატორული აციდოზის დროს იყო, ოღონდ საპირისპირო მიმართულებით და ამიტომ მოკლედ ვიტყვი ამაზე. მარტო რესპირატორული ალკალოზის დროს, PCO2-ის დაცემას უნდა ახლდეს თან (შეხედეთ მაგიურ #1 ფორმულას და მაგიურ ლე-შატელიეს პრინციპს). ამიტომ, თუ ვნახულობთ, რომ ბიკარბონატი 24-ის მაღლაა, მაშინ ვიცით, რომ პირველადი მეტაბოლური პროცესის გამო უნდა იყვეს ეს გამოწვეული. პირველადი მეტაბოლური პროცესი ამას მოგცემს ან ბიკარბონატის გაზრდით ინტერსტიციუმში ან H-ის მოცილებით ინტერსტიციუმიდან (რაც მაგიურ ფორმულაში წონასწორობას გადახრის მარცხნივ, ანუ HCO3-ის მომატებისაკენ). ეს ორივე მეტაბოლური პროცესი ტოლფასია პირველადი მეტაბოლური ალკალოზისა. მოკლედ, როცა პირველადი რესპირატორული პროცესი გაქვს და HCO3 სავარაუდო მიმართულებით არ იცვლება, მაშინ პირველადი მეტაბოლური პროცესიც გაქვს!

თუ პირველადი რესპირატორული ალკალოზი გაქვს და HCO3 სავარუდო მიმართულებით წავიდა, მაშინ უნდა გაარკვიო გაქვს თუ არა კომპენსაცია და თუ გაქვს როგორია კომპენსაცია. აქაც ვიყენებთ ფორმულებს.

თუ გაქვს მწვავე რესპირატორული ალკალოზი კომპენსაციის გარეშე, მაშინ PCO2-ის ყოველი 10-ით დაკლებას ახლავს HCO3-ის 2 ერთეულით დაქვეითება. ხოლო PH იცვლება შემდეგი ფორმულის მიხედვით: pH = 0.008 X (40 – PaCO2).

თუ გაქვს მრესპირატორული ალკალოზი სრული მეტაბოლური კომპენსაციით, მაშინ PCO2-ის ყოველი 10-ით დაკლებას ახლავს HCO3-ის 5 ერთეულით დაქვეითება. ხოლო PH იცვლება შემდეგი ფორმულის მიხედვით: pH = 0.0017 X (40 – PaCO2).

8) პირველადი მეტაბოლური აციდოზის დროს ასეა საქმე: აციდოზი გამოწვეულია HCO3-ის დაქვეითებით (ან მისი მეტაბოლური დაკარგვით ან H-ის მეტაბოლური შეძენით). ამ შემთხვევაში შემთხვევაში რეაქცია გადაინაცვლებს მარცხნივ, რომ გაიზარდოს შემცირებული HCO3. ეს კი იმას ნიშნავს, რომ CO2 უნდა დაიკლოს. ამ დროს, ასევე, რესპირატორული კომპენსაცია წესით ეგრევე იწყება და ჰიპერვენტილაცია კიდევ უფრო დაბლა აგდებს CO2-ს. PCO2-ის მოსალოდნელი ცვლილება გამოითვლება ფორმულით: PCO2 = 1.5 X(HCO3) + 8±2. ანუ, თუ დავუშვათ მეტაბოლური აციდოზის დროს HCO3 დაეცა 15-მდე, მაშინ სრული რესპირატორული კომპენსაცია (ანუ რესპირატორული ალკალოზი) გამოიწვევს PCO2-ის დაცემას 28-32-ის ფარგლებში. შესაბამისად, თუ PCO2 40-სა და გამოთვლის სიდიდეს შორის არის, მაშინ გაქვს დაწყებული კომპენსაცია, რომელიც არასრულია. თუ PCO2 40-ზე ძალიან ზევით არის, მაშინ პირველად მეტაბოლურ აციდოზთან ერთად გვაქვს პირველადი რესპირატორული აციდოზიც (ამაზე უკვე ვისაუბრეთ და უკვე ნასაუბრევი მხრიდანაც შეგიძლიათ ამას შეხედოთ). თუ PCO2 გამოთვლილ სიდიდეზე დაბლაა (მაგალითად 15-ია), მაშინ პირველად მეტაბოლურ აციდოზთან ერთად გვაქვს პირველადი რესპირატორული ალკალოზიც, რადგანაც ზეკომპენსირება არ არის შესაძლებელი.

9) და ბოლოს, თუ გაქვს პირველადი მეტაბოლური ალკალოზი, ეს გამოწვეული იქნება HCO3-ის გაზრდით (ან ბიკარბონატის შეძენის გამო ან ან H-ის დაკარგვის გამო). ორივე შემთხვევაში რეაქცია გადაინაცვლებს მარჯვენა მხარეს CO2-ის წარმოქმნისაკენ და შესაბამისად CO2 გაიზრდება. აქაც, რესპირატორული კომპენსაცია საკმაოდ სწრაფად ხდება. რესპირატორული კომპენსაცია ნიშნავს რესპირატორულ აციდოზს (#5 პრინციპი), ანუ CO2-ის კიდევ უფრო მაღლა აწევას. სრული კომპენსაციისას HCO3-ის ყოველი 1 ერთეულით გაზრდას მოყვება PCO2-ის 0.5-0.7-თ გაზრდა. დავუშვათ გვაქვს მეტაბოლური ალკალოზი და HCO3 გვაქვს 32. სრული რესპირატორული კომპენსაცია რომ გქონოდა, მაშინ PCO2 უნდა აწეულიყო მაქსიმუმ 0.7X(32-24)=5.6-ით და შესაბამისად გამხდარიყო 45.6. თუ PCO2 40-სა და 45.6-ს შორის არის, მაშინ იცი, რომ კომპენსაცია არის დაწყებული, მაგრამ არასრულია. თუ 45.6-ზე მაღალია (მაგალითად 55-ია), მაშინ ვიცით რომ ზეკომპენსაცია ვერ მოხდება და შესაბამისად რესპირატორული აციდოზი პირველადი პროცესიც არის (და არა კომპენსაცია). თუ PCO2 40-ს დაბლაა, მაშინ არათუ რესპირატორული კომპენსაცია არ მომხდარა, არამედ პირველადი რესპირატორული ალკალოზიც გაქვს თან.

ეს არის სულ smile.gif ამის მერე, როცა დაადგენთ ყველაფერს, საჭიროა იმ პროცესების დადგენა, რამაც გამოიწვია ეს ყველაფერი.

ჩვენი ქიეისის შემთხვევაში მივიღეთ რესპირატორული აციდოზი და მეტაბოლური აციდოზი ერთოდროულად. მეტაბოლური აციდოზის ახსნა ადვილია: ჰიპოქსია (ფილტვების შეშუპების გამო) და შოკი იწვევს მეტაბოლურ აციდოზს. მაგრამ რესპირატორული აციდოზის მიზეზი თვალშისაცემი არ არის. თვითონ ამ კეისის ავტორს უწერია, რომ ფილტვების შეშუპების დროს ფიტლვის მექანიკა ისე იცვლება, რომ რესპირატორულ აციდოზს გაძლევსო. ეს ძაან ზერელე ახსნაა ჩემი აზრით. მე ამას მივაწერდი იმას, რომ 1) ინფარქტის დროს ხშირად ვხმარობთ ნარკოტიკებს, რამაც სასუნთქი გზების დეპრესია შეიძლება გამოიწვიოს, რასაც მოყვება ჰიპოვენტილაცია და შესაბამისად რესპირატორული აციდოზი; 2) ამ ავადმყოფს თუ აქვს wheezing, ეს ნიშნავს, რომ აქვს ბრონქოსპაზმი და ეს თავის მხრივ მოითხოვს გაძლიერებულ მუშაობას საუნთქი კუნთების მხრიდან. ეს უკნასკნელი, თუ ბრონქოსპაზმი არ მოხსენი, მოგცემს სასუნთქი კუნთების დაღლას და შესაბამისად სუნთქვის (ანუ წუთვენტილაციის) დაქვეითებას და რესპირატორულ აციდოზს.

QUOTE
ამ პაციენტს აქვს ინფარქტი, ალბათ დიდი ზომის, დაემართა გულის უკმარისობა განდევნის ფრაქციის დავარდნის გამო, რის შედეგადაც ფილტვებში შეგუბებაა, შესაძლებელია ბრონქოსპაზმიც, ჯამში ირღვევა სისხლის ოქსიგენეცია და ვითარდება რესპრატორული აციდოზი. მეტაბოლური ფაქტორი არის თუ არა? შესაძლებელია LDH-ის და სხვა უჯრედშიდა ნივთიერებების დიდი რაოდენობით სისხლში გადასვლა ნეკროზის უბნიდან... ეს ისედაც ხდება ინფარქტის დროს, მაგრამ რამდენად შესაძლებელია ეს ასეთი ძლიერი მეტაბოლური აციდოზის ფაქტორი გახდეს? მარა მაინც... ანუ მინდა ვთქვა რომ სისხლში ნახშირორჟანგის და უჯრედული პროდუქტების მატება იწვევს PH-ის გადახრას აციდოზისკენ + სისხლის ბიკარბონატული ბუფერული სისტემის დეფიციტიცაა... ასე რომ მეტაბოლური და რესპირატორული ალკალოზი გვაქვს.... თუ ვცდები, მითხარი სად ვცდები
ინფარქტის დროს კი გადადის მჟავები (და არა LDH, რადგანაც LDH არის ენზიმი და აციდოზთან არ აის კავშირში-ისე კი იწევს მაღლა) ნეკროზული არიდან სისხლში, მაგრამ გულის კუნთის ზომა პატარაა და არ შეუძლია მნიშვნელოვანი ცვლილება მოახდინოს მთელს სისხლზე. თუ მსგავსი პროცესი მოხდა დიდ არეში (მაგალითად რაბდომიოლიზის, ან ქვედა კიდურის განგრენის დროს), მაშინ შესაძლებელია მსგავსი ახსნა. მეორეს მხრივ, როგორც ვთქვი, მარტო ბრონქოსპაზმი ვერ ახსნის რესპირატორულ აციდოზს (თუ საბოლოოდ კუნთები არ დაიღალა). ამაზე ისიც მეტყველებს, რომ ასთმის დროს (როცა ძლიერი ბრინქოსპაზმიც შეიძლება იყოს) დასაწყისში პაციენტები ჰიპერვენტილაციასაც ახერხებენ და ხშირად რესპირატორული ალკალოზი აქვთ. აი როცა სამკურნალო ჩარევა გვიანდება და სასუნთქი კუნთების დაღლა ხდება, მაშინ უკვე იწყება ჰიპოვენტილაცია და რესპირატორული აციდოზი.

თუ რაიმეს ვერ გაიგებთ, კითხვა შეგიძლია დასვათ biggrin.gif კიდევ ერთი რამ: ამხელა ნაწერს თავიდან ვერ წავიკითხავ მე და თუ რაიმე წინააღმდეგობა ნახეთ, შეიძლება მექანიკური შეცდომის ბრალი იყოს. ამიტომ ძალიან კარგი იქნება თუ რაიმეს აღმოაჩენთ და მიუთითებთ.

მერე რამოდენიმე ქეისს დავდებ მჟავა-ტუტოვან წონასწორობაზე ნელ-ნელა და უფრო დაამუღამეთ ამ ყველაფერს.

Posted by: vano_t 21 Dec 2008, 09:33
Blind_Torture_Kill
QUOTE
ჩემი თირკმლის ქეისი ჩაკალით ? user.gif

არა, დროებით ჩავწიხლეთ biggrin.gif

მე არაფერ შუაში ვარ. მე ვერ გამოვიცანი და თავი დავანებე. ისე ძალიან საინტერესო პრეპარატი ჩანს, მარა შენ არ გაგიცია პასუხი ჩემს ვარაუდზე და პათოლოგი არ ვარ რომ ეგრევე დავსვა დიაგნოზი. სხვებმა შეხედეს თუ არა, ეგ მე არ ვიცი.

ცოტა ნამიოკი მაინც დადე, რომ საინტერესო გახდეს.

Posted by: vano_t 21 Dec 2008, 11:16
ერთ შემტხვევას დავდებ კიდევ. საინტერესო შემთხვევაა.

32 წლის მამაკაცი მოყავთ სასწრაფო დახმარების პუნქტში (ER-ში მოკლედ). ავადმყოფი უჩივის მცირედ მუცლის ტკივილს და ღებინებას. სიმპტომები დაეწყო 4 საათის წინ. წარსული ისტორია არ არის აღსანიშნავი. პაციენტი ეწევა ნახევარ პაჩკა სიგარეტს დღეში. ასევ აღსანიშნავია, რომ პაციენტი ბოლო 6 წლის მანძილეზე თითქმის ყოველდღე სვამს ალკოჰოლს (ძირითადად hard liquor-ს). პაციენტმა ბოლოს დალია 7 საათის წინ კუსტარულად დამზადებული სასმელი.

ფიზიკური გამოკვლევისას პაციენტი ნასვამ მდგომარეობაშია. აქვს ტაქიკარდია და ტაქიპნეა. მხედველობის ველები დაქვეითებულია. ფუნდოსკოპიით რეტინა ნორმალურია და აღენიშნება ოპტიკური დისკის შეშუპება ორმხრივად. მუცელი რბილია გამოკვლევით.

ანალიზები: ლიპაზა და ამილაზა ნორმალურია. გლუკოზა -120 (ნორმაა და ეს მოლებში არ არის გამოსახული, არამედ მგ/დლ-ში); შარდოვანა 28; კრეატინინი 1.2; CBC ნორმაშია; ღვიძლის ენზიმები, ბილირუბინი და ალბუმინი ნორმაშია; ნატრიუმი 134 (ნორმაა 135-145); კალიუმი 4; ქლორიდი 98; ბიკარბონატი 14; ეთილ-ალკოჰოლის კონცენტრაცია მცირედ მომატებულია მხოლოდ; გაზომილი ოსმოლალობა არის 320; არტერიული სისხლის გაზები უჩვენებს შემდეგს:
PH -- 7.26
PCO2 30 mm Hg
PO2 90 mm Hg
გამოთვლილი HCO3 -- 13 meq/L

მუცლის ღრუს კომპიუტერული ტომოგრაფია ნორმაშია; შარდის ანალიზი ნორმაშია;

რა არის სავარაუდო დიაგნოზი? რა ტესტებს ჩაუტარებთ ავადმყოფს დიაგნოზის დასადგენათ და რა მკურნალობა უნდა დაენიშნოს?

Posted by: Cousteau 21 Dec 2008, 12:55
QUOTE (vano_t @ 21 Dec 2008, 11:16 )
წინ კუსტარულად დამზადებული სასმელი.


მეთანოლის დონეს სისხლში
ალბათ ''ხის სპირტი'' დალია

უბრალოდ

1. არვიცი მუცელი რატომ ტკივა (იწვევს ეეს?)
2. აციდოზი ამისგანააქ? (იდეაში ღემინებამ მგონი წესით ალკალოზი უნდა მოგცეს მჟავის დაკარგვისაგან დ.ა.შ???)
3.ანტიდოტია ეთანოლი (მაგრამ ამის მენეჯმენტი როგორ უნდა მთლიანად არვიცი ალბათ ლავაჟი და მსგავსი რამეები)

აციდოზზე ვნახე ეგ იწვევს

Posted by: vano_t 21 Dec 2008, 13:21
Cousteau
QUOTE
მეთანოლის დონეს სისხლში
ალბათ ''ხის სპირტი'' დალია

მართალი ხარ, ოღონდ პასუხს თუ დაასაბუთებ მთლად კაი იქნება smile.gif
რა მიუთითებს მეთანოლით მოწამვლას (მაგალითად, რას მიუთითებს გაზომილი ოსმოლალობა რომ 320-ია)? გაზების ანალიზი რას ამბობს? smile.gif

QUOTE
1. არვიცი მუცელი რატომ ტკივა (იწვევს ეეს?)
კი, იწვევს. მუცლის ტკივილი შეიძლება კეთილთვისებიანი იყოს ამ დროს (როგორც ამ შემთხვევაში), ან შეიძლება პანკრეატიტის გამოც გქონდეს. თუმცა ნორმალური ამილაზა და ლიპაზა პანკრეატიტის შანს მკვეთრად ამცირებს.

QUOTE
2. აციდოზი ამისგანააქ? (იდეაში ღემინებამ მგონი წესით ალკალოზი უნდა მოგცეს მჟავის დაკარგვისაგან დ.ა.შ???)
კი, აციდოზი მაგისგან აქვს. ეთილენგლიკოლიც (ანტიფრიზის ძირითადი სითე) და მეთანოლიც საბოლოოდ გარდაიქმნებიან მჟავებში, რომელიც იწვევენ აციდოზს. ეთილენგლიკოლი გარდაიქმნება გლიკოლატში და ოქსალატში. მეთანოლი-ჭიანჭველა მჟავაში (ფუმარატში). ორივეს შეუძლია ძლიერი anion gap მეტაბოლური აციდოზის გამოწვევა.

QUOTE
3.ანტიდოტია ეთანოლი (მაგრამ ამის მენეჯმენტი როგორ უნდა მთლიანად არვიცი ალბათ ლავაჟი და მსგავსი რამეები)
ნამდვილად, ანტიდოტი ეთანოლია ან ფომეპიზოლი (რომელიც ძაან ძვირი წამალია). ორივეს მოქმედების პრინციპი ერთიდაიგივეა. ორივე ალკოჰოლ დეჰიდროგენაზის სუბსტრატია და კონკურენციას უწევენ მეთანოლს (ასევე ეთილენგლიკოლს), რაც მეთანოლის მეტაბოლიზმს ბლოკავს. თვითონ მეთანოლი არ არის მომწამვლელი, არამედ მისი მეტაბოლიზმის პროდუქტები (ფორმალდეჰიდი და ფუმარატი).

მენეჯმენტი ზოგადად არის უზარმაზარი რაოდენობით სითხეები, ნატრიუმის ბიკარბონატი (თუ PH დაბალია ძალიან-7.2-ზე ნაკლები), თუ აციდოზი სერიოზულია და მისი გართულებები ჩნდება, მაშინ დიალიზიც შეიძლება დაჭირდეს, ინტუბაცია დაჭირდება განსაკუთრებით მაშინ, როცა ავადმყოფი უგონოდ არის (ასპირაციის ტავიდან ასაცილებლად) ან რესპირატორული გართულებები უყალიბდება. ავადმყოფი უნდა იყოს ICU-ში და ლაბორატორიები ხშირად უნდა მოწმდებოდეს.

Posted by: Cousteau 21 Dec 2008, 13:30
QUOTE (vano_t @ 21 Dec 2008, 13:21 )
Cousteau
QUOTE
მეთანოლის დონეს სისხლში
ალბათ ''ხის სპირტი'' დალია

მართალი ხარ, ოღონდ პასუხს თუ დაასაბუთებ მთლად კაი იქნება smile.gif
რა მიუთითებს მეთანოლით მოწამვლას (მაგალითად, რას მიუთითებს გაზომილი ოსმოლალობა რომ 320-ია)? გაზების ანალიზი რას ამბობს? smile.gif

მართალიგითხრა ოსმოლარობაზე არ მიფიქრია საერთოდ. ეგ იმიტომ ვიფიქრე რო

1. ''კუსტარული დალიაო'' (ქეისებში მაგას ტყუილად არ წერენ ხოლმე smile.gif )
2. ეთილ-ალკოჰოლის კონცენტრაცია მცირედ მომატებულია მხოლოდ (ეთანოლის იტოქსიკაცია რო ყოფილიყო მაღალი ექნებოდა + ბევრი თუ დალია და არ აქვს ესეიგი ეგ არ დაულევია)
3. ღებინება უცებ დაეწყო (ამისათვის საჭიროა ან ნეიროგენული ან მექანიკური მიზეზი, მექანიკური არ აქვს)
4. ანალიზებით ღვიძლი/პანკრეასი მიზეზი უფრო რო არ არის
5. მხელდველობის პრობლემააქ
6. მუცელი რბილია ესეიგი უფრო რო არაფრის პერფორაცია არ ექნება + CT აქ გაკეთებული
7.pH აციდოზია როცა ღებინებისაგან თუ არ ვცდები პირიქით ალკალოზი უნდა ქონდეს იმიტორო მჟავას დაკარგავს კუჭიდან

რაც ოსმოლარობას ეხება საერთოდ არ მიფიქრია მაგაზე (ეხლაც არ მინდა სიცხე მაქ
gigi.gif ) ერთი-ორი წტყვით თუ დაწერ : )

Posted by: Blind_Torture_Kill 21 Dec 2008, 13:39
QUOTE
შარდოვანა 28



QUOTE
აღენიშნება ოპტიკური დისკის შეშუპება ორმხრივად


აი ესენი რატომაა ?

Posted by: vano_t 21 Dec 2008, 14:07
Cousteau
QUOTE
1. ''კუსტარული დალიაო'' (ქეისებში მაგას ტყუილად არ წერენ ხოლმე smile.gif )
ეგ ერთი ნამიოკია. მეორე ის არის, რომ მხედველობის პრობლემა დაეწყო და გამოკლვევისას რეტინა ნორმალურია, მაგრამ ოპტიკური დისკი შეშუპებულია. მე-3 ნამიოკი ის არის, რომ გაქვს anion gap აციდოზი. ანუიონური სხვაობა არის 134-(98+14)=22. ეს საკმაოდ მომატებული ანიონური სხვაობაა. ნორმალური ანიონური სხვაობა არის 12. ყველაზე მთავარი (რისთვისაც ეს კეისი დავდე biggrin.gif) არის ის, რომ ოსმოლალური სხვაობა (osmolal gap) არის ძალიან დიდი, 35. ოსმოლალური სხვაობა გამოითვლება ასე: OG=გაზომილი ოსმოლალობა-გამოთვლილი ოსმოლალობა. გამოთვლილი ოსმოლალობა=2X[Na]+[glucose]/18+BUN/2.8. ოსმოლალურ სხვაობას განაპირობებს ოსმოსურად აქტიური მოლეკულების არსებობა, რომელიც გამოთვლაში არ მონაწილეობს. ნორმაში სისხლის ოსმოლალობას ქმნის ნატრიუმი და მასთან დაკავშირებული ანიონები (ამიტომაც ამრავლებ ნატრიუმის კონცენტრაციას 2-ზე, რომ ნატრიუმიც და მასთან დაკავშირებული იონებიც სრული დისოციაციის გამო მონაწილეობენ ოსმოლალობის შექმნაში მთლიანად), გლუკოზა და BUN (ანუ შარდოვანა). გამოთვლილი ოსმოლალობის ფორმულაში გლუკოზის კონცეტრაციას (რომელიც გამოსახულია მგ/დლ) ყოფ 18-ზე რომ მიიღო გლუკოზის მოლარული (და შესაბამისად ოსმოლარული) კონცენტრაცია. საქართველოში, მე მგონი, გლუკოზას მილიმოლებში ზომავენ. ამიტომ, ასეთ შემთხვევაში აღარ დაგჭირდება ეს გაყოფა. პირდაპირ მიუმატებ ამ რიცხვს მაგ ფორმულაში. BUN-საც იმიტომ ყოფ 28-ზე, რომ მისი ოსმოლალობა მიიღო (მილიოსმოლებში გამოსახული). მოკლედ, ჩვენს შემთხვევაში გამოთვლილი ოსმოლალობა იქნება 134X2+120/18+28.2.8=285. გაზომილ ოსმოლალობას-გამოთვლილი=320-185=35. ნორმალში ეს სიდიდე არ უნდა აღემატებოდეს 10-ს, იმიტომ რომ გამოთვლილ ოსმოლალობაში არ შევიდა კალიუმი და მასთან დაკავშირებული ანიონები (რაც დაახლოებით 8 მილიოსმოლს გაძლევს) და პროტეინები და ლიპიდები, რაც მცირე ოსმოლალობას უმატებს. როცა ასე მომატებული ოსმოლალული სხვაობა გაქვს, ამას ბევრი რამ არ გაძლევს. ამას ძირითადად გაძლევს ეთილის სპირტი, მეთილის სპირტი და ეთილენგლიკოლი. ამიტომ უნდა გამოთვალო ეს სიდიდე და დიფ დიაგნოზს ძალიან შეავიწროვებ თუ ძაან მომატებულია ეგ სიდიდე. ჩვენს შემთხვევაში ავადმყოფს ქონდა მხოლოდ მცირედ მომატებული ეთანოლი და ძალიან მაღალი ოსმოლალური სხვაობა, რაც ვერ აიხსნება მარტო ეთილის სპირტის მოხმარებით. ესეც მნიშვნელოვანი ნამიოკია.

რაც შეეხება მჟავა-ტუტოვან წონასწორობის მთლიან ანალიზს, ეგ სიცხე რომ გაგივა შენთვის მომინდია biggrin.gif უნდა ივარჯიშო ამეებში და დაამუღამებ მერე კარგად.

Blind_Torture_Kill
QUOTE
QUOTE
შარდოვანა 28
აი ესენი რატომაა ?

შარდოვანა 28 ნორმაა. თუმცა, შარდოვანა რომ იყოს მომატებული იზოლირებულად ამ შემთხვევაში, ეგ შეიძლება იყოს დეჰიდრატაციის ბრალი (ღებინების ფონზე). მძიმე შემთხვევებში კარდიოგენული შოკიც შეიძლება გქონდეს, რასაც მოყვება წნევის ვარდნა და თირკმლების მწვავე უკმარისობა. ოღონდ ამ დროს კრეატინინიც მაღალი იქნება.

ოპტიკური ნერვის დაზიანების მექანიზმი არ იციან, მაგრამ ისე ძალიან ხშირია მხედველობის პრობლემა (სიბრმავის ჩათვლით) მეთანოლით მოწამვლის დროს.

Posted by: PETRA_ 23 Dec 2008, 17:42
აბა ეს რა არის
სხვათაშორის ქართული შემთხვევაა

Posted by: Solveig 23 Dec 2008, 17:44
PETRA_
მიკროცეფალიას გავს.

Posted by: PETRA_ 23 Dec 2008, 17:48
მიკროცეფალია თავისთავად არის




ანენცეფალია


სრული უტვინობა yes.gif

Posted by: basa-ttt 23 Dec 2008, 17:49
QUOTE (PETRA_ @ 23 Dec 2008, 17:42 )
აბა ეს რა არის
სხვათაშორის ქართული შემთხვევაა

თავის ტვინი ჰემისფეროები საერთოდ არ აქვს მე მგონი- -
ასეთი ბავშვები მხოლოდ რამდენიმე საათი ცოცხლობენ...

Posted by: Solveig 23 Dec 2008, 17:52
საწყალი ბავშვი sad.gif და მისი მშობლებიც.

Posted by: PETRA_ 23 Dec 2008, 18:02
basa-ttt


კი
ორი საათი იცოცხლა ზუსტად

Posted by: *tamtam* 24 Dec 2008, 20:27
PETRA_

რა საშინელებაა sad.gif

მაქვს ერთი კითხვა: ორსულობამდე მქონდა ძალიან ხვეული თმა და მშობიარობის მერე აღარ მეხვევა sad.gif
შეიძლება ძალიან "უბადრუკი" კითხვაა ამ თემისთვის, მაგრამ შესაბამისი თემა ვერ ვიპოვნე.. თან მგონი ძალიან მარტივი პასუხი უნდა ქონდეს და ესეც ერთგვარი გამოძიებაა smile.gif)
ეშველება რამე ანუ დაუბრუნდება ძველ იერს?? (ისე მართლა ძალიან მაწუხებს ეს საკითხი და თუ იცით აუცილებლად დაწერეთ რა sad.gif )

Posted by: Cousteau 24 Dec 2008, 21:37
გასამხნევებლათ...

user posted image


''სწორი'' ცხოველი დაწერეთ wink.gif

Posted by: Blind_Torture_Kill 24 Dec 2008, 22:27
Cousteau

ეს რაღაა biggrin.gif

სისხლჩაქცევაა და რითია გამოწვეული მაგას კითხულობ ?

Posted by: LUKA-BRAZI 24 Dec 2008, 22:32
Cousteau
QUOTE
''სწორი'' ცხოველი დაწერეთ

ადამიანი biggrin.gif

Posted by: Guardian 24 Dec 2008, 22:54
Cousteau
QUOTE
''სწორი'' ცხოველი დაწერეთ

Lupus vulgaris - ჩვეულებრივი (ტუბერკულოზური) მგლურა.

Posted by: Cousteau 25 Dec 2008, 07:59
no.gif
მოკლედ არასწორედ დავსვი კითხვა.

ეგ არის რაღაცის sign-ი, ამ რაღაცას კიდევ ერთი sign-ი აქვს რომელიც რაღაც ცხოველის სახელთან არის ასოცირებული gigi.gif


user posted image

QUOTE
სისხლჩაქცევაა

Ja Ja yes.gif

Posted by: vano_t 25 Dec 2008, 09:16
QUOTE (Cousteau @ 25 Dec 2008, 07:59 )
no.gif
მოკლედ არასწორედ დავსვი კითხვა.

ეგ არის რაღაცის sign-ი, ამ რაღაცას კიდევ ერთი sign-ი აქვს რომელიც რაღაც ცხოველის სახელთან არის ასოცირებული gigi.gif


user posted image

QUOTE
სისხლჩაქცევაა

Ja Ja yes.gif

ქალას ფუძის მოტეხილობის დროს იცის ეგ Battle sign. შენი ცხოველი რაკუნია biggrin.gif (raccoon eyes-ბიტლების სიმღერა გამახსენდა, რაკი რაკუუნ smile.gif

თავიდან კი ვიფიქრე ეგ, მარა მერე გარდიანმა ლუპუსიო. კი მაქვს ნანახი დისკოიდური ლუპუსი, ფერი წააგავს, მარა მაგ დროს გამონაყარი ცოტა სხვანაირია და flaking აქვთ.

Posted by: Cousteau 25 Dec 2008, 09:21
QUOTE (vano_t @ 25 Dec 2008, 09:16 )
ქალას ფუძის მოტეხილობის დროს იცის ეგ Battle sign. შენი ცხოველი რაკუნია biggrin.gif (raccoon eyes-ბიტლების სიმღერა გამახსენდა, რაკი რაკუუნ smile.gif

correct yes.gif
ქალას ფუძის მოტეხილობაა, battle sign-ით


ესეც სწორი ცხოველი smile.gif
user posted image

ეგეც ენოტის თვალები (raccoon eyes a.k.a. panda eyes)
user posted image

Posted by: Blind_Torture_Kill 25 Dec 2008, 09:57
Cousteau

გავნათლდი კიდე smile.gif
პრაქტიკაში გამომადგება

Posted by: vano_t 25 Dec 2008, 10:12
2 ქეისს დავდებ. ორივე დღეს ვნახე. ერთი ხშირია და მეორე ნაკლებად ხშირი. გამოკვლევა არ არის დამთავრებული არცერთზე ჯერ. მარა, ერთში თითქმის ნათელია დიაგნოზი. მე არ ვიტყვი ლაბორატორიებს. თქვენ მკითხეთ რისი გაკეთება გინდათ და იმის მიხედვით მოგაწოდებთ ინფორმაციას.

1) 22 წლის გოგო მოდის კანზე გამონაყარის ჩივილებით. 2 დღის წინ დაეწყო რაღაც ქავილივით და რომ დააკვირდა სისხლჩაქცევა ნახა. მერე დაახლოებით კიდევ 3 ადგილას გაუჩნდა იგივე.

სხვანაირად ჯანმრთელია. 4 წლის წინ გადიატანა მონონუკლეოზი და იმის მერე ქრონიკული დაღლილობა აქვს.

წამლებიდან იღებს მარტო BCP (birth control pills), მენორაგიების გამო.

გამოკითხვით კიდევ დგინდება, რომ ბოლო ერთი წელი შემაჩნია ცხვირიდან გახანგრძლივებული სიხსლდენა. (შენიშვნა: აქ ძაან ცივი და მშრალი ამინდებია და ძალიან ხშირია ამის გამო ცხვირიდან სისხლდენები, თუმცა ამ გოგოს ზაფხულშიც ქონდა პრობლემები).

მუქ განავალს ან აშკარა სისხლდენას უარყოფს. ასევე უარყოფს ჰემატურიას.

ეს გოგო ნაშვილებია და გენეტიკური დედ-მამის და ნათესავების ისტორია არ იცის.

ბოლო რამოდენიმე წელი არც კბილის ამოღება გაუკეთდა და არც რაიმე ქირურგიული ჩარევა. ასევე უარყოფს რაიმე ტრამვას, რომელსაც შეიძლება გამოეწვია სისხლჩაქცევები. ასპირინს და არასტეროიდულ ანთების საწინააღმდეგო პრეპარატებს არ იღებს.

გამოკვლევით: წნევა, ტემპერატურა, პულსი და სუნთქვის სიხშირე ნორმაშია. ნევროლოგიური გამოკვლევა ნორმაშია. ყური, თვალები (სკლერა, კონიუნქტივა), პირის ლორწოვანი და ცხვირის ლორწოვანი ნორმაშია. კანზე რაიმე გამონაყარი არ აქვს გარდა სისხლჩაქცევებისა მარჯვენა და მარცხენა ბარძაყზე. სისხლჩაქცევები დაახლოებით 10-15 სმ-ია დიამეტრში. პეტექიები და ჰემართროზები არ აღენიშნება.

დანარჩენი თქვენზეა. რამოდენიმე ლაბორატორია შევუკვეთე და პასუხი ზოგზე მოვიდა. ზოგი არ არის ჯერ მოსული. რა იდეები გაქვთ და როგორ გამოიკვლევთ საერთოდ ამ ავადმყოფს?

მე-2 ქიეისში სასწავლებელი ის არის, რომ კლინიკურ სურათზე დაყრდნობა მარტო არ შეიძლება (მიუხედავად იმისა, რომ კლინიკური სურათი მკვეთრად მიუთითებს რომელიღაც პათოლოგიაზე და სხვა პათოლოგიაზე ნაკლება).

2) 80 წლის ქალი მოდის მუცლის ტკივილით. ტკივილი დაეწყო წუხელ. ტკივილი ეპიგასტრიუმის არეშია. ტკივილი თანდათან გაძლიერდა. ბოლოს ძალიან რომ გაძლიერდა გადაწყვიტა კლინიკაში მოსვლა. ტკივილი გადაეცემა ზურგისკენ და რეტროსტერნალურად. 2-ჯერ აღებინა. ნაღებში სისხლი არ იყო. ნორმალური ნაწლავების მოქმედება ქონდა ამ დილით.

აქვს არტერიული ჰიპერტენზია და ჰიპოთირეოზი.

იღებს ნიფედპინს და სინთროიდს (ლევოთიროქსინია).

სხვა სისტემების მიმოხილვა ნორმალურია, გარდა მცირედი ზოგადი სისუსტისა.

მოკლედ, გამოკვლევის დროს პულსი იყო 80, წნევა 120/80, ტემპარატურა 37,2.

მუცელი რბილი აქვს. ნაწლავების ხმები ნორმალურია, ოღონდ ეპიგასტრიუმის არეში აქვს მტკივნეულობა პალპაციით. პალპაციის დროს ცდილობს რომ დაიცვას მუცელი (ხელები ჩემს ხელებთან აქვს ახლო და თუ ცოტა ღრმად ვაკეთებ პალაპციას, ხელებში მწვდება smile.gif). მარჯვენა ზედა კვადრატის პალპაცია (ღრმაც და ზედაპირულიც) არ იწვევს მტკივნეულობას. მარცხენა ზედ კვადრატის დაბლა აქვს კიდევ მცირედი ტკივილები პალპაციისას.

სხვანაირად ყველაფერი ნორმაშია, გარდა სიფერმკრთალისა.

ლაბორატორიაში: ლეიკოციტები აქვს 12 900 და მნიშვნელოვანი მარცხნივ გადახრით (band-ს არის 35 პროცენტი).

დანარჩენი ლაბორატორიები მოთხოვნისამებრ smile.gif

Posted by: basa-ttt 25 Dec 2008, 12:15
პირველ შემთხვევაში თრომბოციტებია საინტერესო

მეორეში კუჭი + ნ/ბ (ენდოსკოპია + ექოსკოპია)

Posted by: Blind_Torture_Kill 25 Dec 2008, 12:22
bleedin time , CBC , და თუ თრომბოციტები ნორმაა მაშინ ristocetin cofactor assay

მეორეში ეკგ + ამილაზა ლიპაზა და მერე დალშე კველაფერი + upper abdominal series

Posted by: basa-ttt 25 Dec 2008, 12:40
QUOTE
მეორეში ეკგ

პალპაციით სტკივა ეპიგასტრიუმის არე -
ნაკლებ სავარაუდოა გულის პათოლოგია...
თუმცა....

Posted by: vano_t 25 Dec 2008, 12:42
basa-ttt
QUOTE
პირველ შემთხვევაში თრომბოციტებია საინტერესო

მეორეში კუჭი+ნ/ბ (ენდოსკოპია + ექოსკოპია)

თრომბოციტები ნორმაშია. ისე თრომბოციტების პრობლემები უფრო პეტექიებს გაძლევს, მარა მაინც უნდა შეუკვეთო.

მეორე შემთხვევაში პანკრესი გამოგრჩა. რომელ ენდოსკოპიაზე საუბრობ? ენდოსკოპიამდე სისხლის ანალიზები უნდა. თუ მოითხოვ, მაშინ დავწერ smile.gif

Blind_Torture_Kill
QUOTE
bleedin time , CBC , და თუ თრომბოციტები ნორმაა მაშინ ristocetin cofactor assay
up.gif უი, bleeding time არ შემიმოწმებია biggrin.gif ეგ კი უნდა თავიდან. კაია რო გამახსენე. ristocetin cofactor assay ჯერ არ ვუკვეთავ. როცა ჩვეულებრივ კოაგულოგრამა მთლიანად მოვა, მერე. ისე ვონ-ვილებრანდი დიფერენციალშია-ერთერთი მთავარია. შეიძლება კიდევ ჰემოფილია B იყოს (შედარებით იოლი ფორმა ჰემოფილიის-christmas disease). საბოლოოდ მაინც ჰემატოლოგთან მოუწევს გაგაზავნა, მაგრამ დიაგნოზი მე მირჩევნია დავსვა.

PT ნორმალური აქვს. PTT ჯერ არ მოსულა.

მეორეში, ეკგ ნორამალური სინუსური რითმია. მოკლედ ნორმაა. cardiac enzymes ნორმაა. ამილაზა ნორმაა (ლიპაზას ჩვენთან ვერ ვამოწმებთ). სხვა რა ლაბორატორიები გჭირდება? abdominal xrays არ შემიკვეთია გარკვეული მიზეზების გამო (ამ შემთხვევაში სხვა ლაბორატორიებმა მიუთითა პრობლემაზე).

Posted by: basa-ttt 25 Dec 2008, 12:44
QUOTE
მეორე შემთხვევაში პანკრესი გამოგრჩა. რომელ ენდოსკოპიაზე საუბრობ?

ექოსკოპია რომ დავწერე-
მანდ პანკრეასიც იგულისხმება - ანუ ამ ორგანოს პათოლოგია უნდა გამოჩნდეს ამ გამოკვლევით...
ენდოსკოპიური გამოკვლევა გასტროსკოპიას გულისხმობს და 12-გოჯას გამოკვლევასაც.

QUOTE
თრომბოციტების პრობლემები უფრო პეტექიებს გაძლევს

არა რას ამბობ -
თრომბოციტოპენიის დროს შეიძლება პეტექიაც იყოს და ჩქცევებიც..


თუ თრომბოციტები ნორმაშია მაშინ სრული კოაგულოგრამა როგორც უკვე თქვეს...

Posted by: vano_t 25 Dec 2008, 13:17
QUOTE
ექოსკოპია  რომ დავწერე-
მანდ პანკრეასიც იგულისხმება  - ანუ ამ ორგანოს  პათოლოგია უნდა გამოჩნდეს  ამ გამოკვლევით...
ენდოსკოპიური გამოკვლევა გასტროსკოპიას გულისხმობს და 12-გოჯას გამოკვლევასაც.
ექოსკოპია არ არის კარგი ტესტი პანკრეასის გამოკვლევისთვის (მითუმეტეს როცა პანკრეატიტზე ფიქრობ). რამოდენიმე ნაკლი აქვს: 1) ნაწლავის გაზი ხშირად ვერ მოგცემს მისი დათვალიერების საშუალებას; 2) სპეციალისტი ჭირდება წასაკითხავად; 3) ამილაზა და ლიპაზა ძალიან მგრძნობიარე და სპეციფიური ტესტებია (ამილაზა არ არის სპეციფიური, მაგრამ ლიპაზა არის) და არავითარი სპეციალისტი არ ჭირდება; დიაგნოზსაც სწრაფად იღებ;

გასტროსკოპია შეიძლება აღმოჩნდეს სავსებით ზედმეტი ტესტი. ამიტომ, ჯერ ლაბორატორიული გამოკვლევა ჭირდება, რაც უფრო მარტივია და სავარაუდო დიაგნოზს მოგცემს, რომელიც თავიდან აგარიდებს (ან გეტყვის, რომ საჭიროა ენდოსკოპია).

კიდევ რა ტესტებს შეუკვეთავდი?

QUOTE
QUOTE
თრომბოციტების პრობლემები უფრო პეტექიებს გაძლევს

არა რას ამბობ -
თრომბოციტოპენიის დროს შეიძლება პეტექიაც იყოს და ჩქცევებიც..
მე ხომ არ მითქვამს, რომ ჰემორაგიებს არ გაძლევს თქო. მე ვთქვი უფრო პეტექიებს გაძლევს მეთქი. რა თქმა უნდა ჰემორაგიებსაც გაძლევს. და რა თქმა უნდა, ყველა ვარიანტში ჭირდება თრომბოციტებზეც გამოკვლევა და სისხლდენის დროის დადგენაც. ნორმალური რაოდენობის თრომბოციტები და გახანგრძლივებული სისხლდენა მიუთითებს თრომბოციტების თვისებრივ პრობლემაზე, რაც შეიძლება გქონდეს ვონ-ვილებრანდის დროს-ერთერთი ხშირი დაავადებაა (ბლაინდ ტორჩა კილი ამ მიმართულებით უმიზნებს).

Posted by: PETRA_ 25 Dec 2008, 17:24
vano_t

პირველ შემთხვევაში სისხლის სრული ანალიზის სურათი მაინტერესებს

ანუ სისხლის საერთო თავისი ერითროციტების ,თრომბოციტების როცხვით და ფორმითურთ





მეორე შემთხვევაში უფრო ღრმა ანამნეზი თუ შეიძლება


მაგალითად ამ ტკივილს უკავშირებს თუ არა საკვების მიღებას ,

Posted by: Blind_Torture_Kill 25 Dec 2008, 20:07
QUOTE
სხვა რა ლაბორატორიები გჭირდება?


მეორეში გემოგლობინი როგორია ?


Posted by: vano_t 26 Dec 2008, 02:34
PETRA_
QUOTE
პირველ შემთხვევაში სისხლის სრული ანალიზის სურათი მაინტერესებს
ჰემოგლობინი ნორმაა, ერითორციტები ნორმაა. ამ შემთხვევაში უკვე ბლაინდ_ტორჩა_კილმა ფაქტიურად ახსენა რა უნდა გაკეთდეს. ლაბორატორიებს ველოდები. ახალი ლაბორატორია რომ გამოჩნდება, მერე დვდებ.

QUOTE
მაგალითად ამ ტკივილს უკავშირებს თუ არა საკვების მიღებას ,
უცებ დაეწყო ტკივილები. საკვებთან კავშირში არ არის. მწვავე ტკივილია და ადრე არ ქონია ასეთი.

Blind_Torture_Kill
QUOTE
მეორეში გემოგლობინი როგორია ?

ჰემოგლობინი ნორმაა.

მოდი დავამატებ ლაბორატორიებს:
ALT 450 (ნორმაა 50-მდე)
AST 750 (ნორმაა 70-მდე)
მთლიანლი ბილირუბინი 2.9 (ნორმაა 1-მდე)
ტუტე ფოსფატაზა ცოტათია აწეული

ხოდა სავარაუდო დიაგნოზები; შემდეგი გამოკვლევა და რა თქმა უნდა მკურნალობა როგორია? (მკურნალობა საბოლოო დიაგნოზის დასმამდე უნდა დაიწყო ყველა ვარიანტში. ამიტომ ემპირიული მკურნალობა მაინტერესებს ჯერ და, მერე დიაგნოზის მერე, საბოლოო მკურნალობა).

კიდევ ისა, შენი თირკმელის სურათი რა პათოლოგიას აჩვენებს? (ან რაღაც ნამიოკი მომეცი, მაინტერესებს რა არის).

Posted by: Blind_Torture_Kill 26 Dec 2008, 10:01
QUOTE
მოდი დავამატებ ლაბორატორიებს: ALT 450 (ნორმაა 50-მდე) AST 750 (ნორმაა 70-მდე) მთლიანლი ბილირუბინი 2.9 (ნორმაა 1-მდე) ტუტე ფოსფატაზა ცოტათია აწეული

ხოდა სავარაუდო დიაგნოზები; შემდეგი გამოკვლევა და რა თქმა უნდა მკურნალობა როგორია? (მკურნალობა საბოლოო დიაგნოზის დასმამდე უნდა დაიწყო ყველა ვარიანტში. ამიტომ ემპირიული მკურნალობა მაინტერესებს ჯერ და, მერე დიაგნოზის მერე, საბოლოო მკურნალობა).


ალკოჰოლური ჰეპატიტია

QUOTE
კიდევ ისა, შენი თირკმელის სურათი რა პათოლოგიას აჩვენებს? (ან რაღაც ნამიოკი მომეცი, მაინტერესებს რა არის).


კისტები გაქვს მაგ თირკმელში (რა ნაწილში) მარა PKD არაა smile.gif



Posted by: vano_t 26 Dec 2008, 10:41
Blind_Torture_Kill
QUOTE
ალკოჰოლური ჰეპატიტია
არა, ალკოჰოლური ჰეპატიტი არ დაიწყება ესე მწვავედ მუცლის ტკივილით. არც ბილირუბინის აწევა იცის. ეგ ქრინუკული ჰეპატიტია ფაქტიურად. ამხელა ამინოტრასფერაზების აწევა ფაქტიურად გამორიცხავს დიაგნოზს.

რამ შეიძლება მწვავედ მოგცეს ბილირუბინის აწევა და ამინოტრანსფერაზების აწევა მუცლის მწვავე ტკივილთან ერთად?

QUOTE
კისტები გაქვს მაგ თირკმელში (რა ნაწილში) მარა PKD არაა smile.gif
ვსერჩე და ბოლოს მივიღე medullary cystic kidney და nephronophthisis.
* * *
user posted image
ეს არის? ანუ MCKD?

Posted by: Blind_Torture_Kill 26 Dec 2008, 11:26
QUOTE
ვსერჩე და ბოლოს მივიღე medullary cystic kidney და nephronophthisis.


კიდე რა დაავადებაა

მაშინ ა ჰეპ

ისე ასტ-ს აწევა ალტ-ზე 2 ჯერ მეტად ალკოჰოლურ ჰეპ ახასიათებს და მწვავედ არ შეიძლება გამოვლინდეს ?

Posted by: vano_t 26 Dec 2008, 11:34
QUOTE (Blind_Torture_Kill @ 26 Dec 2008, 11:26 )
QUOTE
ვსერჩე და ბოლოს მივიღე medullary cystic kidney და nephronophthisis.


კიდე რა დაავადებაა

მაშინ ა ჰეპ

არ არის ა ჰეპატიტი.

შენი პრეპარატი რა არის?

Posted by: Blind_Torture_Kill 26 Dec 2008, 11:39
QUOTE
შენი პრეპარატი რა არის?


medullary sponge kidney

მოკლედ მუცლის CT მინდა smile.gif ვეღარ ვფიქრობ
ნაღვლის ბუშტი როგორია


Posted by: PETRA_ 26 Dec 2008, 12:44
პირველ შემთხვევაში გამორიცხული არ არის ჰემორაგიული ვასკულიტი იყოს (შონლაინ ჰენოხი)


სიმეტრიული სისხლჩAქცევა ბარძაყებზე, ქავილი , ასევე ინფექციური მონონუკლეოზი შეიძლება ყოფილიყო ამის გამშვები მექანიზმი


აარააააა? smile.gif

Posted by: basa-ttt 26 Dec 2008, 20:43
QUOTE
პირველ შემთხვევაში გამორიცხული არ არის ჰემორაგიული ვასკულიტი იყოს (შონლაინ ჰენოხი)

ამას უფრო პეტექიები ახასიათებს-
აღწერით კი იმენნო ჩაქცევებს აღწერენ , დიამეტრით - ანუ ლაქებივით...

Posted by: vano_t 26 Dec 2008, 21:47
PETRA_
QUOTE
პირველ შემთხვევაში გამორიცხული არ არის ჰემორაგიული ვასკულიტი იყოს (შონლაინ ჰენოხი)
არა, ჰენოხ-შონლაინს აქვს პუპურები, რომლის პალპაციაც არის შესაძლებელი. პურპურა განსასხვავებელია ჰემორაგიისაგან და ექკიმოზებისაგან.

ვირუსებთან კი არის კავშირში, მაგრამ ეგ დაავადება იწყება ვირუსული დაავადებიდან დღეებში და კვირეებში. ამ ავადმყოფს მონონუკლეოზი ქოდნა წლების წინ.

ამ ავადმყოფს აქვს ალბათ რომელიღაც ფაქტორის ან შეძენილი (სავარაუდოდ ვონ ვილებრანდის დაავადება) ან გენეტიკური დეფიციტი (მაგალითად ჰემოფილია ბ).

basa-ttt
QUOTE
ამას უფრო პეტექიები ახასიათებს-
აღწერით კი იმენნო ჩაქცევებს აღწერენ ,  დიამეტრით - ანუ ლაქებივით...
პატარა გამონაყარია, მართალია. მაგრამ პეტექია არ არის ეგ, პუპურაა. დაავადების სახელიც მიუთითებს მაგაზე (ჰენოხ-შონლეინის პურპურა). პურპურასა და პეტექიას შორის განსხვავებაა როგოც მაკროსკოპულად, ისევე მიკროსკოპულად.

Blind_Torture_Kill
QUOTE
მოკლედ მუცლის CT მინდა smile.gif ვეღარ ვფიქრობ
ნაღვლის ბუშტი როგორია
ავადმყოფი იმ დღესვე გავუშვა სხვა საავადმყოფოში. იქ ჩაუტარდა ულტრაბგერაც და კომპიუტერული ტომოგრაფიაც. ნაღვლის ბუშტის ზუსტი აღწეა არ მაქვს. იმ ექიმს ვესაუბრე, ვინაც მიიღო ავადმყოფი და ასეთი ინფორმაცია მომცა: ნაღვლის საერთო სადინარი გაფართოებულია და შესაძლებელია ქვის არსებობა დისტალურად, რომელიც ასევე ახშობს პანკრეასის სადინარს. ლიპაზებმა აწევა დაიწყესო.

ავადმყოფს ჩაუტარდება ERCP ალბათ მალე. ამით დიაგნოზსაც დასვამ და ქვასაც ამოუღებ.

მოკლედ, პრინციპში შეუძლია ბევრ რამეს გამოიწვიოს მსგავსი რამ, მაგრამ მე რაც შემთხვევები მინახია (და აშშ-ში ყველაზე ხშირია ესენი) არის ნაღვლის საერთო სადინარში ქვაა გაჩხერილი. ხშირად პანკრეასთან ახლოსაც შეიძლება იყოს. ერთდროულად ავადმყოფს შეიძლება ქონდეს მწვავე ქოლეცისტიტიც და პანკრეატიტიც. ან ქოლანგიტი შეიძლება ქონდეს. პრინციპში მუცლის ტკივილი, ტრანსამინაზების აწევა და ბილირუბინემია შეიძლება ბევრი რამის (რაც ინტრა ან ექსტრაჰეპატური ნაღვლის სადინარების დახშობას იწვევს) გამო იყოს. ალბათ სხვადასხვა ქვეყნებში სხვადასხვა რამ იქნება ხშირი მიზეზი. აფრიკაში შეიძლება რაღაც ენდემური პარაზიტი იყოს, ჩრდილოა ამერიკაში სხვა და ა.შ.

მეც ცოტა ისე დავდე ეს კეისი, გამოიცანი მე რას ვფიქრობ თქოს პონტში. მარა, იმისთვის დავდე, რომ მეთქვა ქოლეცისტიტის დროს არ არის აუცილებელი მარჯვენა ნეკნების ქვეშ გქონდეს ტკივილი. ავადმყოფს შეიძლება მარცხენა მხარეს ქონდეს ტკივილი საერთოდ.

Posted by: LUKA-BRAZI 26 Dec 2008, 21:53
vano_t
ძაან ჩამოვრჩი ამ ქეისს smile.gif ამიტომ შემდეგი ქეისიდან მივიღებ მონაწილეობას smile.gif

Posted by: Blind_Torture_Kill 26 Dec 2008, 22:32
QUOTE
ავადმყოფი იმ დღესვე გავუშვა სხვა საავადმყოფოში. იქ ჩაუტარდა ულტრაბგერაც და კომპიუტერული ტომოგრაფიაც. ნაღვლის ბუშტის ზუსტი აღწეა არ მაქვს. იმ ექიმს ვესაუბრე, ვინაც მიიღო ავადმყოფი და ასეთი ინფორმაცია მომცა: ნაღვლის საერთო სადინარი გაფართოებულია და შესაძლებელია ქვის არსებობა დისტალურად, რომელიც ასევე ახშობს პანკრეასის სადინარს. ლიპაზებმა აწევა დაიწყესო.


ტუტე ფოსფატაზა არ უნდა მომატებულიყო ამ შემთხვევაში ? და ბილირუბინიც არ ავარდებოდა ?

QUOTE
მეც ცოტა ისე დავდე ეს კეისი, გამოიცანი მე რას ვფიქრობ თქოს პონტში. მარა, იმისთვის დავდე, რომ მეთქვა ქოლეცისტიტის დროს არ არის აუცილებელი მარჯვენა ნეკნების ქვეშ გქონდეს ტკივილი. ავადმყოფს შეიძლება მარცხენა მხარეს ქონდეს ტკივილი საერთოდ.


smile.gif

ჩემი თირკმელზე ქეისი ეგეთი გამოვიდა
biggrin.gif

აბა კიდე ერთი ეგეთი ქეისი smile.gif
რა არის CT-ზე ?

Posted by: basa-ttt 26 Dec 2008, 22:46
QUOTE
პატარა გამონაყარია, მართალია. მაგრამ პეტექია არ არის ეგ, პუპურაა. დაავადების სახელიც მიუთითებს მაგაზე (ჰენოხ-შონლეინის პურპურა).

პირიქით -
პურპურას ამბობენ თრომბოციტოპენიაზე
და პეტექია კი შონლაინ ჰენოხისაა.
პეტეხიები წვრილი სისხლჩაქცევებია.
წერტილივით,
პურპურა კი ლაქებია.

Posted by: Cousteau 26 Dec 2008, 23:22
QUOTE (Blind_Torture_Kill @ 26 Dec 2008, 22:32 )

აბა კიდე ერთი ეგეთი ქეისი smile.gif
რა არის CT-ზე ?

ტყვიააქ? spy.gif

Posted by: vano_t 26 Dec 2008, 23:22
Blind_Torture_Kill
QUOTE
ტუტე ფოსფატაზა არ უნდა მომატებულიყო ამ შემთხვევაში ? და ბილირუბინიც არ ავარდებოდა ?
ამ ავადმყოფს ტუტე ფოსფატაზა მომატებული ქონდა ცოტათი. ბილირუბინიც 3 იყო (ნორმა დაახლოებით 1-ია).

QUOTE
აბა კიდე ერთი ეგეთი ქეისი smile.gif
რა არის CT-ზე ?
ქალას მოტეხილობას გავს. ისე ჩანს თითქოს ძვლის ნაწილებია ტემპორალურ ნაწილში და ფრონტალურად ეპიდურული ჰაერია.

basa-ttt
QUOTE
პირიქით -
პურპურას ამბობენ თრომბოციტოპენიაზე
და პეტექია კი შონლაინ ჰენოხისაა.
პეტეხიები წვრილი სისხლჩაქცევებია.
წერტილივით,
პურპურა კი ლაქებია.

http://emedicine.medscape.com/article/1083588-overview
QUOTE
The presenting history varies with each patient. The hallmark of the disease is the characteristic palpable purpura, which is seen in almost 100% of patients.


http://en.wikipedia.org/wiki/Purpura

http://en.wikipedia.org/wiki/Petechia
* * *
Cousteau
QUOTE

ტყვიააქ?                                                            spy.gif

ეგეც შესაძლებელია ისე, რაღაც უშნო ფორმის საფანტი ან რამე smile.gif

Posted by: Blind_Torture_Kill 27 Dec 2008, 00:03
Cousteau

კი

vano_t

QUOTE
ქალას მოტეხილობას გავს. ისე ჩანს თითქოს ძვლის ნაწილებია ტემპორალურ ნაწილში და ფრონტალურად ეპიდურული ჰაერია.


yes.gif

Posted by: Cousteau 27 Dec 2008, 00:17
აგერ გასართობათ მსგავსი რამე

Nail Gun Accident

user posted image

user posted image

გადარჩა
yes.gif


და ერთი კიდევ

Discovery-ზე იყო ამ ტიპზე, ეს ნეილ განი მოხვდა (პისტოლეტივით რო ისვრის ლურსმნებს), ERში მიიყვანეს რენტგენი რატომღაც არ გადაუღეს (!) და გაუშვეს სახლში და ერთი კვირა ''თავის ტკივილი აწუხებდა'' მერე მივიდა ისევ, გადაუღეს რენტგენი და

user posted image

ესეც გადარჩა, გამოუძრეს უბრალოდ



Posted by: Guardian 27 Dec 2008, 00:21
http://imageshack.us

Posted by: Blind_Torture_Kill 27 Dec 2008, 00:21
Nail Gun Accident biggrin.gif

აქსიდენთ კი არა ალბათ ნაწამები იყო ეგრე smile.gif

Guardian

პატარა შესავალი smile.gif

Posted by: basa-ttt 27 Dec 2008, 00:21
QUOTE
Henoch-Schönlein Purpura



Под пурпурой понимают мелкопятнистые капиллярные кровоизлияния в кожу, под кожу или в слизистые оболочки. Единичные кровоизлияния могут быть точечными (петехии), реже полосовидными (вибекс), мелко- (экхимозы) или крупнопятнистыми (кровоподтёки). Обычно наблюдается в виде множественных петехий и экхимозов диаметром до 1 см
В общем случае причиной возникновения пурпуры является формирование склонности к кровотечениям (геморрагического диатеза) вследствие таких патологических процессов как:

Патология тромбоцитарного звена гемостаза (повреждения или дефицит тромбоцитов):
первичная тромбоцитопеническая пурпура;
вторичная тромбоцитопеническая пурпура;
токсико-аллергические нарушения гемокоагуляции (например, лекарственные или инфекционные);
застой крови;
патология со стороны коагуляционного звена гемостаза:
синдром диссеминированного внутрисосудистого свёртывания.
патология сосудистой стенки (васкулит, например, геморрагический васкулит - пурпура Шенлейна-Геноха).


http://www.artrocenter.ru/article92.htm
Болезнь Шенлейна—Геноха (геморрагический васкулит) характеризуется преимущественным поражением микроциркуляторного русла и проявляется повышенной кровоточивостью сосудов кожи, слизистых, синовиальных и серозных оболочек, почек, желудочно-кишечного тракта, реже — других органов и систем. Различают несколько форм геморрагического васкулита: кожную (простую), кожно-суставную, почечную, абдоминальную и смешанную. Обязательный признак всех форм заболевания — геморрагические высыпания петехиального типа на коже

მოკლედ ორივე სწორია.


Posted by: Cousteau 27 Dec 2008, 00:27
QUOTE (Blind_Torture_Kill @ 27 Dec 2008, 00:21 )
Nail Gun Accident biggrin.gif

აქსიდენთ კი არა ალბათ ნაწამები იყო ეგრე smile.gif


user posted image


ამას მენჯის ძვლის პროტეზიაქ თან

user posted image

და one of the favorites

user posted image
თავის ძრავით ჩანს user posted image

Posted by: Blind_Torture_Kill 27 Dec 2008, 00:30
QUOTE
თავის ძრავით ჩანს



lol.gif lol.gif lol.gif lol.gif lol.gif lol.gif lol.gif lol.gif lol.gif

Posted by: Cousteau 27 Dec 2008, 00:31
. .

Posted by: vano_t 27 Dec 2008, 00:47
QUOTE (Cousteau @ 27 Dec 2008, 00:27 )
თავის ძრავით ჩანს user posted image

lol.gif
რამდენი ცხენის ძალიანია?

Posted by: LUKA-BRAZI 27 Dec 2008, 01:11
vano_t
Blind_Torture_Kill
Cousteau
თქვე ცუდებო lol.gif lol.gif


Posted by: Guardian 27 Dec 2008, 01:33
რა განსხვავებაა წინა ორ სურათსა და ამას შორის?

http://imageshack.us

Posted by: LULA_QABABI 27 Dec 2008, 05:25
QUOTE (Guardian @ 26 Dec 2008, 16:33 )


http://imageshack.us

QUOTE
რა განსხვავებაა წინა ორ სურათსა და ამას შორის?


კრიპტოკოკი india ink და GMS

Posted by: Blind_Torture_Kill 27 Dec 2008, 10:29
აბა ეს რაღაა ?


baby.gif

user.gif

Posted by: Cousteau 27 Dec 2008, 10:34
QUOTE (Blind_Torture_Kill @ 27 Dec 2008, 10:29 )
აბა ეს რაღაა ?


baby.gif

user.gif

სტრიქტურაა საყლაპავის ?

Posted by: Guardian 27 Dec 2008, 10:34
LULA_QABABI
QUOTE
კრიპტოკოკი india ink და GMS

გამომწვევიც სწორია და შეღებვის მეთოდებიც, მაგრამ მარტო ეგ არ არის განსხვავება. wink.gif

Blind_Torture_Kill
QUOTE
აბა ეს რაღაა ?

Schatzki ring.

Posted by: Blind_Torture_Kill 27 Dec 2008, 10:36
Guardian


კი

up.gif

შენ ქებას აღარ მოვყვები smile.gif

Posted by: Cousteau 27 Dec 2008, 10:38
დანარჩენი საყლაპავი ნორმალურია? spy.gif

Posted by: Blind_Torture_Kill 27 Dec 2008, 10:41
Cousteau


QUOTE
დანარჩენი საყლაპავი ნორმალურია?


ალბათ

Posted by: vano_t 27 Dec 2008, 10:43
Guardian
QUOTE
გამომწვევიც სწორია და შეღებვის მეთოდებიც, მაგრამ მარტო ეგ არ არის განსხვავება. wink.gif
პირველი სურათი 50-ე გვერდზეა და მეორე სურათი 51-ზე? user.gif

Blind_Torture_Kill
QUOTE
შენ ქებას აღარ მოვყვები

მე მოვყვები biggrin.gif
Guardian მართლა მაგარი ხარ მედიცინაში შენ!

Posted by: Guardian 27 Dec 2008, 10:50
vano_t
QUOTE
პირველი სურათი 50-ე გვერდზეა და მეორე სურათი 51-ზე?

http://forum.ge/?showtopic=33902549&view=findpost&p=12037514
http://forum.ge/?showtopic=33902549&view=findpost&p=12038404

აბა, ამაზე რას იტყვით?

http://imageshack.us

Posted by: Guardian 27 Dec 2008, 10:54
http://imageshack.us

Posted by: vano_t 27 Dec 2008, 11:04
უცებ ქეისი. მე უფრო პრაქტიკულ ქეისებს ვდებ და არ გამლახოთ.

78 წლის ქალი მოვიდა კლინიკაში. უჩივის თავბრუსხვევას და მარცხენა მხრის ტკივილს. რამოდენიმე დღის წინ დაეცა და humeral head-ის (მხრის ძვლის თავი ქვია ხო?) მოტხდა. დიდ ქალაქში ER-ში იყო ერთ-ერთ სავადმყოფოში და sling ჩამოაცვეს (მოკლედ ასე მკურნალობენ თავიდან ამ მოტეხილობას-მხრის ძვალი სიმძიმის ძალით ეშვება და შეხორცებაც ხდება და ტკივილებსაც აქვეითებს). იქ დაუნიშნეს ჰიდროკოდონი სამკურნალოდ (ნარკოტიკული ანალგეტიკია). მერე მოვიდა ჩვენთან სოფელში smile.gif ზემოაღნიშნული ჩივილებით.

მოკლედ ღებინებაც აქვს. წნევა დაქვეითებულია 84/45. სიცხე არ აქვს და ლაბორატორიები სერიოზულ მიზეზს არ მიუთითებენ. მივიღე საავადმყოფოში და გადავუსხი ფიზიოლოგიური ხსნარის ბოლუსი (1 ლიტრი 2 საათის განმავლობაში) და მერე maintenance ხსნარები მივეცი. მოკლედ გადასხმას უპასუხა და წნევა დარეგულირდა. ავადმყოფი მოკვიტინდა. მაგრამ განაგრძობს მხრის ტკივილს. ჰიდროკოდონი არ არის საკმარისი. 3 დღის წინ დავუმატე fentanil patch (ფენტანილის მისაკრავი ფორმაა). ტკივილები გაუმჯობესდა. მერე სახლში უნდა გავწერო და ჯერ არ უნდა წასვლა. იტოგში დავტოვეთ საავადმყოფოში.

დღეს დილით ექთანი მეუბნება, წუხელ ძაან mean იყო და სულ გვიხურებდა და გაწერეო (არადა არ არის მასეთი ქალი).

ამ საღამოს დაეწყო ღებინებები. წნევა დაუვარდა ისევ 74/54 და მირეკავს ეთქდანი. ახლა თქვენ ხართ სოფლის საავადმყოფოში (კაი სოფლის ოღონდ smile.gif) და რას გააკეთებთ?

Posted by: vano_t 27 Dec 2008, 11:14
Guardian
QUOTE
QUOTE
პირველი სურათი 50-ე გვერდზეა და მეორე სურათი 51-ზე?

http://forum.ge/?showtopic=33902549&view=findpost&p=12037514
http://forum.ge/?showtopic=33902549&view=findpost&p=12038404
biggrin.gif
მე განსხვავებას მივუთითე. შენ თქვი, რომ კიდევ სხვა სხვაობა არისო მაგათ შორის. ხოდა ერთ-ერთი სხვაობა ეგ არის მეთქი (სხვადასხვა გვერდებზე არიან ეგ სურათები).

QUOTE
აბა, ამაზე რას იტყვით?
ეგ ტვინია (აი გიტარა! ცხვირში მოგუდული-ოსური აქცენტით). ისიც დავადგინე, რომ გვამის ტვინია და თანაც არ არის ნორმალური smile.gif

დიაგნოზზე ვიფიქრებ (მარა არა მგონია რო გამოვიცნო-პათოლოგიაში არ ვარგივარ).

შენ პათოლოგი ხარ?

Posted by: Guardian 27 Dec 2008, 11:17
vano_t
QUOTE
მე განსხვავებას მივუთითე. შენ თქვი, რომ კიდევ სხვა სხვაობა არისო მაგათ შორის. ხოდა ერთ-ერთი სხვაობა ეგ არის მეთქი (სხვადასხვა გვერდებზე არიან ეგ სურათები).

მაგას გარდა. biggrin.gif

QUOTE
ეგ ტვინია

წავშალე ეგ სურათი და სხვა დავდე - იქნებ, ეხლა უფრო გაგიადვილდეს. smile.gif

QUOTE
შენ პათოლოგი ხარ?

კი. yes.gif

შენს ქეისში პაციენტს, მგონი, დელირიუმი ჰქონდა.
მაგ ტკივილგამაყუჩებლების ნაწილს მოვუხსნიდი.
ღებინებები რატომ ჰქონდა და წნევა რატომ დაუვარდა?
რავი, აბა.
ცხიმოვანი ემბოლია ხომ არა აქვს?

Posted by: basa-ttt 27 Dec 2008, 11:28
QUOTE
78 წლის ქალი მოვიდა კლინიკაში. უჩივის თავბრუსხვევას და მარცხენა მხრის ტკივილს. რამოდენიმე დღის წინ დაეცა და humeral head-ის (მხრის ძვლის თავი ქვია ხო?) მოტხდა. დიდ ქალაქში ER-ში იყო ერთ-ერთ სავადმყოფოში და sling ჩამოაცვეს (მოკლედ ასე მკურნალობენ თავიდან ამ მოტეხილობას-მხრის ძვალი სიმძიმის ძალით ეშვება და შეხორცებაც ხდება და ტკივილებსაც აქვეითებს). იქ დაუნიშნეს ჰიდროკოდონი სამკურნალოდ (ნარკოტიკული ანალგეტიკია). მერე მოვიდა ჩვენთან სოფელში  ზემოაღნიშნული ჩივილებით.

მოკლედ ღებინებაც აქვს. წნევა დაქვეითებულია 84/45. სიცხე არ აქვს და ლაბორატორიები სერიოზულ მიზეზს არ მიუთითებენ. მივიღე საავადმყოფოში და გადავუსხი ფიზიოლოგიური ხსნარის ბოლუსი (1 ლიტრი 2 საათის განმავლობაში) და მერე maintenance ხსნარები მივეცი. მოკლედ გადასხმას უპასუხა და წნევა დარეგულირდა. ავადმყოფი მოკვიტინდა. მაგრამ განაგრძობს მხრის ტკივილს. ჰიდროკოდონი არ არის საკმარისი. 3 დღის წინ დავუმატე fentanil patch (ფენტანილის მისაკრავი ფორმაა). ტკივილები გაუმჯობესდა. მერე სახლში უნდა გავწერო და ჯერ არ უნდა წასვლა. იტოგში დავტოვეთ საავადმყოფოში.

დღეს დილით ექთანი მეუბნება, წუხელ ძაან mean იყო და სულ გვიხურებდა და გაწერეო (არადა არ არის მასეთი ქალი).

ამ საღამოს დაეწყო ღებინებები. წნევა დაუვარდა ისევ 74/54 და მირეკავს ეთქდანი. ახლა თქვენ ხართ სოფლის საავადმყოფოში (კაი სოფლის ოღონდ ) და რას გააკეთებთ?

78 წლია ქალია -
ეკგ გადააკონტროლე.?

ღებინებია + თაბრუსხვევა + დაბალი წნევა -
ჰემოგლობი - ჰემატოკრიტიც ნახეთ...
სანამ წამლებს დაუნიშნავდით, ეს ჩივილები მანამდეც იყო?

Posted by: vano_t 27 Dec 2008, 11:54
Guardian
QUOTE
მაგას გარდა. biggrin.gif
ახლა მე გარტყმაზე ვმუშაობ. მეორე სურათზე (გმვ შენაღებზე), სხვა უჯრედებიც ჩანს. მაგ საღებავით რასაც ღებავენ ისინი ვნახე (კოინფექცია ხო არ არის თქო). კოკციდიოდოზის სურათი ვნახე. სადღაც წააგავს (ჩემგან უკეთეს პასუხს ნუ მოითხოვ). შეიძლება პნევმოცისტაც იყოს, მარა დავძებნე სად შეიძლება ამათი ერთდროული ინფექცია (რომელ ქსოვილში მოკლედ) და სწრაფი სერჩით ვერაფერი გავარკვიე. აგერ კოკციდიას სურათები:
http://library.med.utah.edu/WebPath/HISTHTML/STAINS/STAIN024.html
დაწვრილებით აღარ წამიკითხია კოკციდიას ანატომია, მარა ის წერტილებიანი რაღაცეები ორივე სურათზეა (ენდოსპორები ხომ არ არის?)

QUOTE
შენს ქეისში პაციენტს, მგონი, დელირიუმი ჰქონდა.
მაგ ტკივილგამაყუჩებლების ნაწილს მოვუხსნიდი.
ფენტანილმა იცის (სხვა ნარკოტიკებმაც, მარა ფენტანილმა განსაკუთრებით) დელირიუმი (მარა მთლად დელირიუმი შეიძლება არ არის-მენტალ სტატუს ჩენჯი ნამდვილად არის). ღებინებაც იცის ამან. ასევე წნევის დაქვეითება (ნებისმიერმა ნარკოტიკმა შეიძლება გამოიწვიოს).

basa-ttt
QUOTE
78 წლია ქალია -
ეკგ გადააკონტროლე.?

არა, მაგრამ ალბათ შეიძლება რომ გაუკეთდეს. გამოცდილების მიხედვით, სწრაფ სკრინინგს ვაკეთებ თავიდან. კარდიოგენული შოკის შანსი ყოველთვის არის, მარა როცა უცებ უვითარდება ეს ყველაფერი და ახალდაწყებული წამლის შემდეგ არის (სხვა სიმპტომების არარსებობასთან ერთად და მოკლე გამოკვლევასთან ერთად), უფრო სავარაუდოა წამალი, განსაკუთრებით ნარკოტიკი. მისაკრავი ფენტანილი ავაცალე და სითხე გადავუსხი. წნევა 120/70 ამოვიდა. რა თქმა უნდა პოტენციურად ყველაფერია შესაძლებელი, მაგრამ არსებობს ხშირი რაღაცეები, რომელიც გიყალიბებს ინტუიციას და იმის მიხედვით მოქმედებ. ასეთი რამ ბევრჯერ მინახია და როცა სერიოზული კატასტროფაა (კარდიოგენული შოკი, სიფსისი, ჰემორაგიული შოკი), მაშინ სხვა რამეებია ხშირად. მოკლედ, ავადმყოფი მოკვიტინდა. რომ არ მოკვიტინებულიყო, აუცილებლად გავუკეთებდი ეკგ-საც და სისხლის ანალიზსაც.
* * *
P.S. კრიპტოკოკოზის ფილტვის ფორმაც ყოფილა. ასე რომ ეგეც შესაძლებელია მე მგონი. მეორე სურათზე ზოგიერთი რაღაც პნეუმოცისტასავით გამოიყურება.

Posted by: Blind_Torture_Kill 27 Dec 2008, 15:32
Guardian

QUOTE
აბა, ამაზე რას იტყვით?


coccidioidomycosis

Posted by: Guardian 27 Dec 2008, 17:45
Blind_Torture_Kill
QUOTE
coccidioidomycosis

დიახთ. up.gif

vano_t

არა, ვანო, არ არის შენი ვერსიები სწორი.
თვითონ სურათებს დააკვირდი.
ცოტას გიკარნახებ - მესამე სურათზე კრიპტოკოკული ინფექცია ძალიან შორს წასული შიდსის ფონზეა განვითარებული.

Posted by: donvaso 28 Dec 2008, 00:24
აბა, აქ რას ხედავთ?????????????????????????????????

Posted by: Cousteau 28 Dec 2008, 01:12
QUOTE (donvaso @ 28 Dec 2008, 00:24 )
აბა, აქ რას ხედავთ?????????????????????????????????

მე ძუძუებს
user posted image

honeycombing აქვს თუ? spy.gif
რაქვია მაგას ქართულად user.gif

Posted by: vano_t 28 Dec 2008, 01:27
donvaso
QUOTE
აბა, აქ რას ხედავთ?????????????????????????????????

ფლუოროგრაფიის ანალიზ ცოტა ძნელია ჩემთვის (არ მინახია ამ ქვეყანაში მაგას რომ აკეთებდნენ), მარა, კუსტოს არი იყოს, ძუძუების გარდა ვერაფერს ვერ ვხედავ biggrin.gif ძუძუები, მე მგონი, ნორმაა თუ რა თქმა უნდა ეგ არ არის კაცის სურათი smile.gif

გულის ზომა და კონტურები ნორმაა. აორტის რკალი ნორმაა. ფილტვის კარი ნორმაა. კოსტოფრენული კუთხეები ნორმაა (ანუ პლევრის გამონადენი არ არის). დაიფრაგმა არ არის გამბრყელებული. ფილტვში რაიმე ინფილტრატი ან კვანძი/მასა არ ჩანს. ჰიპერინფლაციაც არ არის (9 ნეკნი ჩანს ფილტვის არეში). ნეკნების მოტეხილობას ვერ ვხედავ. ლავიწი ნორმაშია.

ჩვეულებრივი რენტგენი არ გაქვს მაგ ავადმყოფის?

Posted by: LULA_QABABI 28 Dec 2008, 10:36
QUOTE (Guardian @ 27 Dec 2008, 08:45 )

არა, ვანო, არ არის შენი ვერსიები სწორი.
თვითონ სურათებს დააკვირდი.

ცოტას გიკარნახებ - მესამე სურათზე კრიპტოკოკული ინფექცია ძალიან შორს წასული შიდსის ფონზეა განვითარებული.

იმუნოდეფიციტის დროს ვითარდება. შიდსის შემთხვევაში

არ ვიცი "ძალიან შორს წასული შიდსი" -ს განსაზღვრება მაგრამ ზოგადად კრიპტოკოკული ინფექცია როგორც წესი CD4<50-ის დროს ვითარდება. გამონაკლისი ამ სურათზე აღბეჭდილი კრიპტოც არ იქნება

Posted by: donvaso 28 Dec 2008, 10:37
Cousteau
QUOTE
honeycombing

აზრზე არ ვარ ეს რა არის...... sad.gif

vano_t
ჩვეულებრივი არ მაქვს.... sad.gif


მინიშნება: სისტემური პათოლოგიაა...... wink.gif

Posted by: basa-ttt 28 Dec 2008, 11:34
QUOTE
აბა, აქ რას ხედავთ?????????????????????????????????

მეჩვენება, რომ ფილტვის ქსოვილი ზედმეტად ფიბროზირებულია -
განსაკუთრებით კართან -
ზემო წილებში "ფიჭისებრი" შენებაც თითქოს ჩანს -
ანუ ფილტვის ქსოვილი ერთგვაროვანი არაა -
თუმცა თავს ვერ დავდებ.

და აორტის რკალი ცოტა მეტადაა გამოწეული.

კლინიკა რაა?

user posted image

Posted by: irakli222 28 Dec 2008, 11:36
მილიარული ტუბერკულიოზი?
(თითქმის მთელი ფილტვი დაფარულია წერტილოვანი ჩრდილებით)

Posted by: Cousteau 28 Dec 2008, 11:46
honeyocmbing - "ფიჭისებრი შენება''

Posted by: PETRA_ 28 Dec 2008, 15:23
ფილტვის გამჭვირვალობაა მომატებული და კასრისებურიც მეჩვენება

დიაფრაგმაც დაწეულია


emphysema pulmonary ია |?

Posted by: PETRA_ 28 Dec 2008, 21:52
აბა ეს რა დაავადებაა smile.gif

ავადმყოფის კონფიდენციალობა დაცულია

Posted by: Blind_Torture_Kill 30 Dec 2008, 01:09
იმედი მაქვს მოდერი თავს არ მომადრუჟბავს user.gif

64 წლის ქალბატონი გარდაიცვალა ბრონქოპნევმონიით. გაკვეთისას ამოღებული ფილტვის სურათი მოცემულია ქვემოთ. რა ჭირდა ?

Posted by: basa-ttt 30 Dec 2008, 01:17
QUOTE
გაკვეთისას ამოღებული ფილტვის სურათი მოცემულია ქვემოთ. რა ჭირდა ?

პნევმოთორაქსი?

Posted by: Blind_Torture_Kill 30 Dec 2008, 01:18
არა რა პნევმოთორაქსი ამ სურათზე ეგ სად გამოჩნდებოდა smile.gif

Posted by: basa-ttt 30 Dec 2008, 01:52
QUOTE
არა რა პნევმოთორაქსი ამ სურათზე ეგ სად გამოჩნდებოდა

ფილტვს ფორმა აქვს დაკარგული -
ჰოდა ვიფიქრე -
ჩაიჩუტა თქო...
კავერნები ხომ არაა შავი წერტილები...

Posted by: Blind_Torture_Kill 30 Dec 2008, 01:58
განსაკუთრებული სხვა არაფერი ჩანს ?

Posted by: PETRA_ 30 Dec 2008, 14:59
გაკირულია ფილტვი სულ /

კრუპოზული პნევმონია

ისე ბრონქოპნევმონიიტ თუ გარდაიცვალა ბრონქოპნევმონია ჭირდა smile.gif

Posted by: LUKA-BRAZI 30 Dec 2008, 21:35
Blind_Torture_Kill
შავი ისარით რა არის მითითებული? ატელექტაზირებული ფილტვია?

Posted by: Blind_Torture_Kill 30 Dec 2008, 22:06
LUKA-BRAZI

ამ სურათს რომ დახედავ პირველ რიგში რა გეცემა თვალში
ხოდა ეგაა მონიშნული ისრით smile.gif

Posted by: LUKA-BRAZI 30 Dec 2008, 22:09
Blind_Torture_Kill
რაღაც ჩაღრმავებასავითაა.... ალბათ ქრონიკული ფილტვ-გულის უკმარისობა ქონდა, ამან პარკუჭის დილატაცია გამოიწვია, რამაც მასეთი კვალი დატოვა ფილტვზე..... რა ვიცი აბა.... smile.gif

Posted by: Blind_Torture_Kill 30 Dec 2008, 22:25
LUKA-BRAZI

QUOTE
რაღაც ჩაღრმავებასავითაა.... ალბათ ქრონიკული ფილტვ-გულის უკმარისობა ქონდა, ამან პარკუჭის დილატაცია გამოიწვია, რამაც მასეთი კვალი დატოვა ფილტვზე..... რა ვიცი აბა....


იფიქრე ...

Posted by: LUKA-BRAZI 30 Dec 2008, 22:31
Blind_Torture_Kill
ეს "იფიქრე" რას ნიშნავს, წინა ვარაუდი არასწორია თუ პირიქით ახლოს ვარ ? smile.gif

Posted by: Blind_Torture_Kill 30 Dec 2008, 22:34
LUKA-BRAZI

არა არაა სწორი smile.gif

Posted by: donvaso 30 Dec 2008, 22:39
ატელექტაზია??????????????????????????

Posted by: Blind_Torture_Kill 30 Dec 2008, 22:44
კაი კითხვას სხვანაირად დავსვავ
რა მიკროორგანიზმით გამოწვეულმა ბრონქოპნევმონიამ იმსხვერპლა ეს ქალი smile.gif

Posted by: LUKA-BRAZI 30 Dec 2008, 22:45
Blind_Torture_Kill
ფანტაზია მეტზე აღარ გამწვდა biggrin.gif მასეთი რამ არასდროს მინახავს, ასე რომ..... ისე ის შავი წერტილები რა არის? აშკარად ნორმალურ ფილტვს ეგეთი "დირკები" არ უნდა ქონდეს smile.gif ბასა_ტტტ-ს თქმისა არ იყოს, მართლაც კავერნები ხომ არაა (რომლების პლერის ღრუში გაიხსნენ) ? ისრითაც ეგენია მითითებული? ან ეს ყვითელი შეფერილობა რაღაა, ან აპიკალურ ნაწილებზე ის ყავისფერი? მოკლედ ან ნამიოკი, ან ალტერნატიული სურათი biggrin.gif
* * *
Blind_Torture_Kill
QUOTE
რა მიკროორგანიზმით გამოწვეულმა ბრონქოპნევმონიამ იმსხვერპლა ეს ქალი

პიოგენურმა მიკრობებმა? spy.gif

Posted by: donvaso 30 Dec 2008, 22:52
Chlamydophila ( Chlamydia ) pneumonia ?

Posted by: Blind_Torture_Kill 30 Dec 2008, 22:53
LUKA-BRAZI

აი აგიმუშავდა გონება smile.gif
შავი წერტილები აბსცესები იყო პლევრის ღრუში გახსნილი
მიკროს სლაიდიც დავდო ?

donvaso

user.gif no.gif

Posted by: basa-ttt 30 Dec 2008, 23:18
QUOTE
შავი წერტილები აბსცესები იყო პლევრის ღრუში გახსნილი

მერე მაი პნევმოთორაქსი არაა - მე რომ დავწერე?

Posted by: LUKA-BRAZI 30 Dec 2008, 23:27
Blind_Torture_Kill
QUOTE
აი აგიმუშავდა გონება

lol.gif ხო კარბურატორი თუ ჰიპოკამპი მიჭედავს ხოლმე და რო გაიტანს მერე მიდიიიი biggrin.gif
მოკლედ ამათგან ერთერთი იქნება რა:
The most common anaerobes are Peptostreptococcus, Bacteroides, Fusobacterium species, and microaerophilic streptococcus.

Other organisms that may infrequently cause lung abscess include Staphylococcus aureus, Streptococcus pyogenes, Streptococcus pneumoniae (rarely), Klebsiella pneumoniae, Haemophilus influenzae, Actinomyces species, Nocardia species, and gram-negative bacilli.
ან აქედან რომელიმე:
Nonbacterial pathogens may also cause lung abscesses. These microorganisms include parasites (eg, Paragonimus, Entamoeba), fungi (eg, Aspergillus, Cryptococcus, Histoplasma, Blastomyces, Coccidioides), and Mycobacterium.

Posted by: Blind_Torture_Kill 30 Dec 2008, 23:29
basa-ttt

ემპიემას მოგცემს აბსცესი თუ ჩაიღვრება პლევრის ღრუში
მარა მე მაგი არ მიკითხავს

გამომწვევი მიკროორგანიზმი მაინტერესებს

LUKA-BRAZI

ყველა ჩამოთვალე თუ გამოგრჩა რომელიმე ?

Posted by: LUKA-BRAZI 30 Dec 2008, 23:36
Blind_Torture_Kill
რადგანაც მეკითხები ე.ი. გამომრჩა biggrin.gif ლურჯ-მწვანე ჩირქმბადი ჩხირი? spy.gif

Posted by: Blind_Torture_Kill 30 Dec 2008, 23:47
LUKA-BRAZI

მოკლედ მაკროც და მიკროც დევს
დანარჩენი თქვენ იცით

Posted by: LUKA-BRAZI 30 Dec 2008, 23:50
Blind_Torture_Kill
ლოლ, მიკროპრეპარატის სურათს ეხლა დავაკვირდი, ერთი ეს მითხარი, ის წვრილი ძაფებივით, მიცელიუმია?

Posted by: vano_t 31 Dec 2008, 01:34
QUOTE (Blind_Torture_Kill @ 30 Dec 2008, 23:47 )
LUKA-BRAZI

მოკლედ მაკროც და მიკროც დევს
დანარჩენი თქვენ იცით

მრავლობითი აბსცესი ბევრ რამეს შეუძლია გამოიწვიოს, მარა მიკროსურათზე ნოკარდიაა მე მგონი.გრამ დადებითი ბაქტერიაა თავისი ფსევდოჰიფებითურთ.

Posted by: Blind_Torture_Kill 31 Dec 2008, 01:35
vano_t

მიკროს სურათი ნოკარდიისთვის დამახასიათებელია მარა მაკროზე რას ხედავ ?

Posted by: vano_t 31 Dec 2008, 01:58
სინუსური სავალები (sinus tracts)

Posted by: Blind_Torture_Kill 31 Dec 2008, 02:01
vano_t
QUOTE
სინუსური სავალები (sinus tracts)



კარგი
კიდევ რა ჩანს ?

Posted by: LUKA-BRAZI 31 Dec 2008, 13:03
კი მაგრამ მუსიე დავით, ნოკარდია მეც ვახსენე წინა პოსტში..... თუ არ მეთვლება იმიტომ რომ copy-paste-ია ? biggrin.gif

Posted by: Blind_Torture_Kill 31 Dec 2008, 13:36
LUKA-BRAZI

ნოკარდიას წააგავს მარა არაა

smile.gif


user.gif

შენ ჩამონათვალში ერთი სწორია smile.gif

Posted by: Guardian 31 Dec 2008, 13:40
Blind_Torture_Kill
QUOTE
ნოკარდიას წააგავს მარა არაა

აქტინომიკოზი.
QUOTE
მაკროზე რას ხედავ ?

sulfur granules.

Posted by: Blind_Torture_Kill 31 Dec 2008, 13:59
Guardian

QUOTE
sulfur granules.

საღოლ
up.gif


Posted by: Guardian 31 Dec 2008, 14:02
------------------------------------------------------------

Posted by: Blind_Torture_Kill 31 Dec 2008, 14:03
Guardian

მარტო ეს სურათია ?

Posted by: LUKA-BRAZI 31 Dec 2008, 14:05
Guardian
ჰიპერტენზიული რეტინოპათია?

Posted by: PETRA_ 31 Dec 2008, 22:07
Guardian


ალუბლის კურკის ფენომენი
ბადურის არტერიის ტრომბოზი

Posted by: LUKA-BRAZI 31 Dec 2008, 22:36
PETRA_
ჰო, ჰო, ნამდვილად ეგაც შეიძლება იყოს..... მოკლედ ბადურის არტერიის პრობლემაა მანდ, ან შევიწროება ჰიპერტენზიის დროს ან თრომბოზი..... პასუხს ალბათ დააგვიანდება..... სავარაუდოდ გარდიანი უკვე საახალწლო სუფრას უზის biggrin.gif

Posted by: mtvareuli 1 Jan 2009, 00:33
Blind_Torture_Kill
QUOTE
იმედი მაქვს მოდერი თავს არ მომადრუჟბავს user.gif

კი გეკუთვნის თავის მოდრუჟბვა, მაგრამ ამ ახალ წელს კეთილი ვიქნები biggrin.gif

Posted by: Guardian 1 Jan 2009, 14:22
PETRA_
QUOTE
ალუბლის კურკის ფენომენი

ყოჩაღ, სიმპტომი სწორია, ინგლისურად მაგას cherry red spot ჰქვია.
QUOTE
ბადურის არტერიის ტრომბოზი

მაგრამ ამ შემთხვევაში ეგ დაავადება არ არის - მიაქციე ყურადღება, რომ cherry red spot-ს აქ თან ახლავს ბადურის სხვა მიდამოების ინტენსიური გაუმჭვირვალობა.

LUKA-BRAZI
QUOTE
ჰო, ჰო, ნამდვილად ეგაც შეიძლება იყოს..... მოკლედ ბადურის არტერიის პრობლემაა მანდ, ან შევიწროება ჰიპერტენზიის დროს ან თრომბოზი...

ეგ ორი დაავადება ერთმანეთისგან სრულიად განსხვავებულ სურათს გაძლევს, ასე რომ, არავითარი "ან -, ან -".
შეუძლებელია, რომ ბადურის არტერიის თრომბოზი ჰიპერტენზიულ რეტინოპათიაში აგერიოს -

Posted by: LUKA-BRAZI 1 Jan 2009, 16:20
Guardian
პირველ რიგში გილოცავ ახალ წელს, მრავალს დაესწარი smile.gif
პრინციპში მართალი ხარ, მარა სანამ მაგ დონემდე მიიყვანს ჰიპერტენზია ბადურას, მანამდე ასე არ გამოიყურება? ..... ჰოდა ეს სურათი რომ ვნახე მაგიტომაც შემეშალა:

Posted by: Blind_Torture_Kill 1 Jan 2009, 17:37
Guardian

QUOTE
მაგრამ ამ შემთხვევაში ეგ დაავადება არ არის - მიაქციე ყურადღება, რომ cherry red spot-ს აქ თან ახლავს ბადურის სხვა მიდამოების ინტენსიური გაუმჭვირვალობა.


ეს ხომ არა vitelliform dystrophy ?

Posted by: Guardian 1 Jan 2009, 17:43
LUKA-BRAZI
QUOTE
გილოცავ ახალ წელს, მრავალს დაესწარი

მეც გილოცავთ ყველას, ვინც ამ განყოფილებაში პოსტავთ. smile.gif

Blind_Torture_Kill
QUOTE
vitelliform dystrophy

არა.

Posted by: LUKA-BRAZI 1 Jan 2009, 17:53
Guardian
მოიცა, შენი მიმაგრებული სურათი ახალი ქეისია? spy.gif ეგ ხო ჰიპერტენზიული რეტინოპათიაა? მე მეგონა ეგ სურათი მე დამიდე cherry red spot-სა და hipertensive retinophaty-ს ერთმანეთისგან განსასხვავებლად.... თუ შენ, Blind_Torture_Kill, პირველ სურათზე ამბობ? სულ ავირიე lol.gif

Posted by: Guardian 1 Jan 2009, 17:55
LUKA-BRAZI
QUOTE
მოიცა, შენი მიმაგრებული სურათი ახალი ქეისია?

არა.
QUOTE
ეგ ხო ჰიპერტენზიული რეტინოპათიაა?

კი.

Posted by: Blind_Torture_Kill 1 Jan 2009, 21:51
Guardian

ხო მაშინ აზზე არა ვარ რა შეიძლება იყოს

Posted by: Guardian 2 Jan 2009, 01:11
მოკლედ, თეი-საქსის დაავადება იყო ეგ.

Posted by: Guardian 2 Jan 2009, 01:21
ეს რა არის?
------------------------------

Posted by: LUKA-BRAZI 2 Jan 2009, 11:03
Guardian
21-ე ქრომოსომის ტრისომია? (იგივე დაუნის სინდრომი)

Posted by: Guardian 2 Jan 2009, 13:36
LUKA-BRAZI
QUOTE
21-ე ქრომოსომის ტრისომია? (იგივე დაუნის სინდრომი)

რატომ? biggrin.gif

Posted by: LUKA-BRAZI 2 Jan 2009, 15:14
Guardian
QUOTE
რატომ?

რა "რატომ"? spy.gif შენს ამ ერთსიტყვიან კითხვებს ვერ ვხვდები ხოლმე lol.gif რატომ და უნაგირა ცხვირი აქვს, ფართო "ბრტყელი" სახე და თვალების ჭრილიც ისეთ "პონტში"..... სხვა სტიგმებიც ააქვს დაუნის სინდრომს, მაგრამ მაგ სურათზე არ ჩანს....
ჰიდროცეფალიასაც ვფიქრობდი თავიდან, მარა no.gif IMHO

Posted by: Guardian 2 Jan 2009, 15:33
LUKA-BRAZI

არ არის ეგ დაუნის სინდრომი.
აი, დაუნის სინდრომი -

http://imageshack.us

Posted by: donvaso 2 Jan 2009, 15:36
Guardian
ეგ ის ხომ არ არის, ნაადრევად რომ ბერდებიან -არ მახსოვს მისი სახელი.... sad.gif

ისეთი "ჭკვიანი" თვალებით იყურება...... baby.gif

Posted by: LUKA-BRAZI 2 Jan 2009, 15:39
Guardian
ჰოოო..... აბა რა არის? ინტერესნო.... ე.ი. ჰიდროცეფალიაც არ არის? "ნამიოკი" biggrin.gif

Posted by: Guardian 2 Jan 2009, 15:43
donvaso
QUOTE
ეგ ის ხომ არ არის, ნაადრევად რომ ბერდებიან -არ მახსოვს მისი სახელი..

პროგერია?
გარტყმაზე მუშაობთ? biggrin.gif
აი, პროგერია -

Posted by: donvaso 2 Jan 2009, 15:46
QUOTE
გარტყმაზე მუშაობთ?


yes.gif biggrin.gif biggrin.gif

Posted by: Guardian 2 Jan 2009, 16:03
LUKA-BRAZI
QUOTE
ე.ი. ჰიდროცეფალიაც არ არის?

ვითომ დიდი თავი აქვს ეხლა მაგას? spy.gif

QUOTE
"ნამიოკი"

ნამიოკი არ იქნება - შემიძლია კიდევ ორი სურათი დავდო -

http://imageshack.us

Posted by: LUKA-BRAZI 2 Jan 2009, 16:09
Guardian
აშკარაა რაღაც სინდრომია, ვიღაცის გვარიც ქვია ალბათ biggrin.gif , შეიძლება გენეტიკურიც იყოს, რაღაცის დეფიციტი და ა.შ. და ა.შ. ხომ ხედავ არ ვიცით! gigi.gif ან კიდევ რამე სხვა ტიპის სურათი დადე, მნიშნელობით ეს სურათები ხომ იდენტურია?

Posted by: Guardian 2 Jan 2009, 16:16
LUKA-BRAZI
QUOTE
აშკარაა რაღაც სინდრომია, ვიღაცის გვარიც ქვია ალბათ

ნამდვილად. biggrin.gif
QUOTE
ხომ ხედავ არ ვიცით!

სად არიან ჩვენი პედიატრები?
QUOTE
ან კიდევ რამე სხვა ტიპის სურათი დადე, მნიშნელობით ეს სურათები ხომ იდენტურია?

მესამე სურათზე რაღაც ისეთი ჩანს, რაც პირველ ორ სურათზე არ არის.

Posted by: donvaso 2 Jan 2009, 16:19
LUKA-BRAZI
ტუჩები, წარბები, წამწამები და ცხვირი სამივე სურათზე ერთნაირი აქვთ ამ საწყლებს.....
უფრო გენეტიკური პატოლოგიია ალბათ .....

Posted by: LUKA-BRAZI 2 Jan 2009, 16:22
donvaso
ალბათ მასეა, საცოდავი ბავშვები user.gif

Guardian
QUOTE
მესამე სურათზე რაღაც ისეთი ჩანს, რაც პირველ ორ სურათზე არ არის

კი ბატონო, მარჯვენა ზედა კიდურის განვითარების ანომალია, მარა ეს სურათების აღწერას სკოლის მოსწავლესავით აღარ შევეშვა მასწ? biggrin.gif ნამიოკი, ნამიოკი რააა, თუნდაც სულ პატარა lol.gif

Posted by: roo 2 Jan 2009, 16:33
user posted image

ყურზე რაჭირს? spy.gif

სიფილისია?

Posted by: LUKA-BRAZI 2 Jan 2009, 16:36
Guardian
QUOTE
The great tragedy of science - the slaying of a beautiful hypothesis by an ugly fact.

რთული სათქმელია.... მარტო ექსპერიმენტები და Evidence-Based Medicine სულ ყველაფერს ვერ ხსნის.... არადა უმაგათოდ რამდენი უაზრო ჰიპოთეზა თუ თეორია იარსებებდა? მაგრამ აი რა თქვა ალბერტ აინშტაინმა: "Imagination is more important than knowledge..." რომ არა მაგისი Imagination-ი და წარმოსახვა, არ იქნებოდა ფარდობითობის თეორია smile.gif მოკლედ მართლაც რთული სათქმელია


Posted by: Guardian 2 Jan 2009, 16:38
roo
QUOTE
სიფილისია?

სიფილისი ვინმეს გვარია? spy.gif

LUKA-BRAZI

ნახე, რა ლამაზი ბავშვია, არადა, მაგასაც ეგ სინდრომი სჭირს -

Posted by: LUKA-BRAZI 2 Jan 2009, 16:41
Guardian
QUOTE
სიფილისი ვინმეს გვარია?

lol.gif lol.gif
QUOTE
ნახე, რა ლამაზი ბავშვია, არადა, მაგასაც ეგ სინდრომი სჭირს

ოკ მარა მერე რა რო? დავდგე და ისევ სურათი აღვწერო? biggrin.gif

roo

ეგ თანდაყოლილი სიფილისი რომ იყოს წინა სურათში იმ გოგოს ეგეთი ლამაზი კბილები არ ექნებოდა, არამედ ჰატჩინსონის კბილები

Posted by: roo 2 Jan 2009, 16:41
QUOTE (Guardian @ 2 Jan 2009, 16:38 )
roo
QUOTE
სიფილისია?

სიფილისი ვინმეს გვარია? spy.gif


ჰატჩისონია გვარი

Hutchinson's triad, a set of symptoms consisting of deafness (მეგონა რო ყურზე რაღაც უკეთია)
Hutchinson's teeth (centrally notched, widely-spaced peg-shaped upper central incisors),
and interstitial keratitis

+ ყველა სურათზე imho straddle nose აქვთ მაგიტომაც დავწერე
+ frontal bossing
poorly developed maxillae

Posted by: Guardian 2 Jan 2009, 16:50
LUKA-BRAZI
QUOTE
ეგ თანდაყოლილი სიფილისი რომ იყოს წინა სურათში იმ გოგოს ეგეთი ლამაზი კბილები არ ექნებოდა, არამედ ჰატჩინსონის კბილები

მართალი ხარ.

roo
QUOTE
ჰატჩისონია გვარი

ჰატჩისონი არა, ჰატჩინსონი.
ისე, ჰატჩისონიც იყო, ძალიან მაგარი ინტერნისტი -
http://forum.ge/?showtopic=33728987&view=findpost&p=5331637

Posted by: roo 2 Jan 2009, 16:52
QUOTE
LUKA-BRAZI
QUOTE
ეგ თანდაყოლილი სიფილისი რომ იყოს წინა სურათში იმ გოგოს ეგეთი ლამაზი კბილები არ ექნებოდა, არამედ ჰატჩინსონის კბილები

მართალი ხარ.

roo
QUOTE
ჰატჩისონია გვარი

ჰატჩისონი არა, ჰატჩინსონი.
ისე, ჰატჩისონიც იყო, ძალიან მაგარი ინტერნისტი -
http://forum.ge/?showtopic=33728987&view=findpost&p=5331637


ოკ

Posted by: LUKA-BRAZI 2 Jan 2009, 17:01
Guardian
QUOTE
"უუნარობისაგან,რომ ყველაფერი დავტოვოთ ისე, როგორც არის; ზედმეტი თავგამოდებისგან ახლისადმი და უპატივცემულობისგან ძველისადმი; ცოდნის სიბრძნეზე, მეცნიერების ხელოვნებაზე და ნიჭიერების საღ აზრზე წინ დაყენებისგან; პაციენტებთან, როგორც შემთხვევებთან მოპყრობისგან; და იმისგან, რომ დაავადების განკურნება უფრო სერიოზულად მიგვაჩნდეს იყოს, ვიდრე მისი გაძლება, ღმერთო, დაგვიფარე."

up.gif ვააა, მაგარი რამე უთქვამს ჰატჩინსონს! მე ვეთანხმები! თუმცა მის ნათქამში "არ ავნოს" პრინციპი უფრო მყარად რომ იყოს წარმოჩენილი, უფრო კარგი იქნებოდა yes.gif
QUOTE
ე.ი. ამის კეთება არც გვიზიდავს, ხოლო ადამიანი, ჩვეულებრივ, იმ საქმეს, რომელიც არ იზიდავს, ცუდად აკეთებს.

ამასაც ვეთანხმები smile.gif

Posted by: donvaso 2 Jan 2009, 17:07
ახლა გადმოვიღე გენეტიკის წიგნი და ეს სურათები ყველაზე მეტად შეესაბამება ბეჭდისებური მე-9 ქრომოსომის სინდრომს....

Posted by: Guardian 2 Jan 2009, 17:13
LUKA-BRAZI
QUOTE
აი რა თქვა ალბერტ აინშტაინმა: "Imagination is more important than knowledge..."

არ არის საჭირო ერთმანეთთან დაპირისპირება - ჰიპოთეზა მუტაციებს ჰგავს, ხოლო ექპერიმენტი - ბუნებრივ გადარჩევას - ხვდები, ალბათ, რასაც ვგულისხმობ. biggrin.gif

donvaso
QUOTE
ეს სურათები ყველაზე მეტად შეესაბამება ბეჭდისებური მე-9 ქრომოსომის სინდრომს....

არა.

Posted by: LUKA-BRAZI 2 Jan 2009, 17:17
Guardian
მე კი მივხვდი, მახვილგონივრული ნათქვამია btw, მაგრამ ალბერტს შეიძლება სხვა პასუხი გაეცა აქ რომ ყოფილიყო biggrin.gif
სწორი პასუხი რა ასოზე იწყება ის მაინც თქვი გარდიან lol.gif

Posted by: BEKO 2 Jan 2009, 18:04
Patau's syndrome ან Edward's syndrome....
უკაცრაცად ექიმებს რომ გეჭრებით, მაგრამ მეგობარს ჰყავს დაუნის სინდრომით ბავშვი და რაღაცების წაკითხვა მომიწია ნეტში. ოღონდ რომელია, ნამდვილად არ ვიცი... ეგ თქვენ გაარჩიეთ, ექიმებმა.

Posted by: PETRA_ 2 Jan 2009, 19:30
Apert Syndrome\?

თავის ქალის და კიდურების მიხედვით

Posted by: Guardian 2 Jan 2009, 19:32
BEKO
QUOTE
Patau's syndrome ან Edward's syndrome....

არა.
QUOTE
Apert Syndrome

არა.

არადა, საკმაოდ ხშირი დაავადებაა.
კაი, ცოტას გიკარნახებთ - ეგ დაავადება სტრუქტურული (და არა - ფერმენტის) გენის მუტაციითაა გამოწვეული.
ეს რამეს გეუბნებათ?

Posted by: SOULFUL 2 Jan 2009, 20:30
Guardian

დე ლანგეს სინდრომი

Posted by: Guardian 2 Jan 2009, 20:46
SOULFUL
QUOTE
დე ლანგეს სინდრომი

აბსოლუტურად მართალია. up.gif

http://en.wikipedia.org/wiki/Cornelia_de_Lange_Syndrome

Posted by: SOULFUL 2 Jan 2009, 22:54
Guardian

user posted image

ესეც დე ლანგეს სინდრომია, იაშვილში იწვა , საკუთარი მასალაა , სახე არ ჩანს და იმედია ეთიკურია აქ ამ ფოტოს დადება , რეალში ნანახი უფრო შთამბეჭდავია

Posted by: Blind_Torture_Kill 4 Jan 2009, 22:33
ძლივს ვნახე ეს სურათი smile.gif
რა არის ?

Posted by: LUKA-BRAZI 5 Jan 2009, 10:45
Blind_Torture_Kill
მართალი გითხრა რადიოლოგიის კურაციაზე სულ თითზე ჩამოსათვლელი რენტგენოგრამები გვანახეს, CT კი სულ 2 თუ 3 სურათი, ასე რომ აქ ვერ ვიპროფესორებ biggrin.gif ეს მრგვალი თეთრი რაღაც მეჩვენება საეჭვოდ და რა არის? ჰიპოფიზის ადენომა რომ იყოს მაგ განივკვეთში არ უნდა ჩანდეს წესით... ეგ რაღაც ატლანტ-კეფის საზღვართან ან "დაჟე" ცოტა ქვევითაც კია მგონი biggrin.gif IMHO

Posted by: Cousteau 6 Jan 2009, 15:29
user posted image

სოფელში, 14 წლის ბავშვს, რამოდენიმე დღე ახველებდა/ქონდა სიცხე და გამოაყარა, თავს გრძნობდა კარგად, იფიქრეს რო ჩუტყვავილაა (ბავშვის მეგობარი ქონდა დიაგნოსტირებული რომელთანაც კონტაქტი ქონდა 3 კვირის განმავლობაში), მშობლებს ექიმისთვის არ მიუმართავთ ვინაიდან თავს გრძნობდა ნორმალურად, სპეციფიური მკურნალობა არ ჩატარებია, მხოლოდ აბანავებდნენ და აძლევდნენ წამალს სიცხის დაწევისთვის. რამოდენიმე დღეში ბავშვს დაეწყო ძლიერი ღებინება, თავის ტკივილი, ''იყო მოთენთილი, და ღამე ცუდად ეძინა, ''
აჩვენეს ექიმს რომელმაც თქვა რომ ღვიძლიაქ ოდნავ გადიდებული და ''რეფლექსები გაცხოველებული'', ურჩია რო წაეყვანათ ქალაქში, მეორე დღეს ბავშვის მდგომარეობა გაუარესდა, განუვითარდა სტუპორი (უგონოდ იყო), გულყრები და ქალაქში ჩასვლამდე გარდაიცვალა.

სხვა არაფერი მონაცემები არ არის, გაკვეთა არ გაკეთებულა.
ამ ანამნეზიდან შეიძლება სიკდილის მიზეზის (ნუ ძაააალიან სავარაუდოდ) დადგენა

wtf?


PS რო გაეკვეთათ ამას ნახავდნენ (ღვიძლია)

user posted image

PPS პრეპარატი დაgoogleულია ისერო ვერ გეტყვით ზუსტად ეგაა თუ არა (სახელით მოვძებნე)


Posted by: Guardian 6 Jan 2009, 15:37
Cousteau

რეის სინდრომი.

Posted by: Cousteau 6 Jan 2009, 15:44
QUOTE (Guardian @ 6 Jan 2009, 15:37 )
Cousteau

რეის სინდრომი.

გადადექი!

yes.gif



http://en.wikipedia.org/wiki/Reyes_syndrome

შემდეგში რებუსებით დავწერ

user posted image

+

user posted image

=

user posted image


Posted by: basa-ttt 6 Jan 2009, 19:41
QUOTE
რეის სინდრომი.

სიცხის დამწევად რომ არ გამოეყენებინა ასპირინი -
ეს ბავშვი არ მოკვდებოდა და ჩვეულებრივ გადაიტანდა ჩუტყვავილას.
ვინმეს აქვს სტატისტიკა -
წამლებით გამოწვეული სიკვდალიანობის და გართულებების შესახებ?

Posted by: mtvareuli 6 Jan 2009, 23:09
Cousteau

ჯერ წინა ქეისისთვის გეპასუხა და მერე გეკითხა


basa-ttt

აქ არ არის მაგის გასარჩევი ადგილი, ცალკე თემა გახსენი თუ გინდა

Posted by: Cousteau 6 Jan 2009, 23:10
QUOTE
იცხის დამწევად რომ არ გამოეყენებინა ასპირინი

ან პარაცეტამოლი (აცეტამინოფენი)

QUOTE (basa-ttt @ 6 Jan 2009, 19:41 )

წამლებით   გამოწვეული სიკვდალიანობის  და გართულებების შესახებ?

არამგონია საქართველოში არსებობდეს ზუსტი სტატისტიკა.



meta: Guardian,LUKA-BRAZI

http://www.humanmetrics.com/cgi-win/JTypes2.asp

ეს ტესტია (რომელზეც იმ წიგნში წერია Myers-Briggs) ნამდვილი ტესტი ფასიანია.

ბოლოში ტიპის აბრევიატურას წერს (მაგ INFJ) იმ წიგნში (ultimate guide) წერია რომელ ტიპზე რომელი sub-medical პროფესია 'მიდის'

QUOTE
ჯერ წინა ქეისისთვის გეპასუხა და მერე გეკითხა


ეეხ სადამაქ იმაზე პასუხის ტრ... ტვინი user.gif



Posted by: LUKA-BRAZI 6 Jan 2009, 23:22
Cousteau
Thanks in advance biggrin.gif აუცილებლად გავაკეთებ yes.gif

Posted by: vano_t 7 Jan 2009, 07:40
LUKA-BRAZI
QUOTE
მართალი გითხრა რადიოლოგიის კურაციაზე სულ თითზე ჩამოსათვლელი რენტგენოგრამები გვანახეს, CT კი სულ 2 თუ 3 სურათი, ასე რომ აქ ვერ ვიპროფესორებ biggrin.gif ეს მრგვალი თეთრი რაღაც მეჩვენება საეჭვოდ და რა არის? ჰიპოფიზის ადენომა რომ იყოს მაგ განივკვეთში არ უნდა ჩანდეს წესით... ეგ რაღაც ატლანტ-კეფის საზღვართან ან "დაჟე" ცოტა ქვევითაც კია მგონი biggrin.gif IMHO

CT არ არის ეგ ლუკა, ეგ MRI არის.

Blind_Torture_Kill
პროფესორობიც ვერც მე ვიპროფესორებ. მარა მაგ დონეზე მრავალი სხვადასხვა სიმსივნე შეიძლება წარმოიქმნას ისე. pituitary adenoma ერთ-ერთია მათ შორის. კრანიოფარინგიომა შეიძლება იყოს მაგ დონეზე. სურათები ვნახე ადენომების და კრანიოფარინგიომების. ადენომებზე მასეთი ინტენსიური სიგნალი არ მინახია. კრანიოფარიონგიომას აქვს მასეთი სიგნალი. მაგ დონეზე შეიძლება გაჩნდეს კიდევ ბევრი სხვა სიმსივნე (ჰამარტომა, მენინგიომა, გერმინალურ უჯრედოვანი სიმსივნეები, მეტასტაზური სიმსივნეები). რადიოლოგიურად ალბათ რაღაც განმანსხავებელი ნიშნები იქნება.

კიდევ შეიძლება ბაზილარული არტერიის ანევრიზმა იყოს, მარა რანაირ სიგნალს იძლევა ეგ, არ ვიცი. ან შეიძლება ნორმალური ზომის სისხლძარღვის სანათურია და სულ სხვა რამეა სურათზე biggrin.gif

იტოგში, რადიოლოგის დახმარებაა საჭირო.

Posted by: basa-ttt 7 Jan 2009, 08:28
ხაალხ -
ეგ არის ცხვირის ნიჟარების დონეზე გაკეთებული ჭრილი -
და საერთოდ მაგი თავის ტვინშია?
თითქოს სინუსების თუ ცხვირის ღრუს გაგრძელებაზეა...
და ცხავის ძვალი არ ჩანს -
თუ არადა, მაშინ თურქული კეხის არეა ნამდვილად...

ცხვირის ღრუს შიგთავსიც არაერთგვაროვანია - ..
უცხო სხეული ხომ არ იქნება - ზედმეტად სწორი ფორმისაა ის თეთრი ჩრდილი...
რა ასაკისაა ავადმყოფი?
ცოტა კლინიკაც დაამატეთ...
ჰოდა კიდევ -
მარჯვენა სინუსის ღრუ დაჩრდილულია -
ჰაიმორიტიც ხომ არაა ამასთან ერთად?



QUOTE
იცხის დამწევად რომ არ გამოეყენებინა ასპირინი


ან პარაცეტამოლი (აცეტამინოფენი)

კი, yes.gif
ამ შემთხვევაში ბავშვი არ მოკვდებოდა და გადარჩებოდა -
თუმცა პარაცეტამოლსაც აქვს თავისი მინუსები...
იცით თქვენ მე რასაც გამოვიყენებდი ამ შემთხვევაში...
wink.gif

Posted by: Cousteau 7 Jan 2009, 12:21
LUKA-BRAZI
Guardian

ველკომწ



QUOTE (basa-ttt @ 7 Jan 2009, 08:28 )

იცით  თქვენ მე რასაც გამოვიყენებდი ამ შემთხვევაში...
wink.gif

yes.gif

რეის სინდრომს გალაპაგოსის მაჩვის კლანჭი უხდება ძაან gigi.gif
2kiss.gif

შობას გილოცავთ


Posted by: basa-ttt 7 Jan 2009, 12:33
QUOTE
რეის სინდრომს გალაპაგოსის მაჩვის კლანჭი უხდება ძაან

რეის სინდრომზე არ ვამბობ -
მაგას მართლა არაფერი ეშველება-
ჩუტყვავილას მკურნალობას და სიცხის დაწევას ვგულისხმობ...

გილოცავთ შობას ყველას კიდოვ ერთხელ....
2kiss.gif

Posted by: Led_Zeppelin 7 Jan 2009, 12:41
ჩუტყვავილა. მენინგო ენცეფალიტი, თვის ტვინის შეშუპება, კომა, კლონური გულყრები ,სუნთქვის გაშერება, სიკვდილი. სურთზე ღვიძლის ცხიმოვანი დისტროფია..........................................ჩემის აზრით.

Posted by: Freedoctor 7 Jan 2009, 15:09
ემერაიზე რომ სურათია.. პაციენტს ბიტემპორალური ჰემიანოპია ხომ არ აქვს?.. ანუ ეგ რაღაც ქიაზმას ხომ არ აწვება?...

Posted by: zviadcardio 7 Jan 2009, 22:09
basa-ttt
QUOTE
რეის სინდრომზე არ ვამბობ - მაგას მართლა არაფერი ეშველება-

თუ სწორედ მახსოვს მაგ დროს ნ-აცეტილცისტეინის ვენის ფორმა გამოიყენება. Orphan Drug-ია.
თემას ოდნავ გადავუხვიე, მაგრამ...

Posted by: vano_t 7 Jan 2009, 22:32
zviadcardio
QUOTE
QUOTE
რეის სინდრომზე არ ვამბობ - მაგას მართლა არაფერი ეშველება-

თუ სწორედ მახსოვს მაგ დროს ნ-აცეტილცისტეინის ვენის ფორმა გამოიყენება. Orphan Drug-ია.
თემას ოდნავ გადავუხვიე, მაგრამ...

აცეტილცისტეინს იყენებენ აცეტამინოფენით მოწამვლისას (overdose).

რეის სინდრომი შეიძლება ასპირინის მცირე დოზითაც მოხდეს. კორტიკოსტეროიდები და დიურეტიკები ჭირდებათ ამ დროს.

Freedoctor
QUOTE
ემერაიზე რომ სურათია.. პაციენტს ბიტემპორალური ჰემიანოპია ხომ არ აქვს?.. ანუ ეგ რაღაც ქიაზმას ხომ არ აწვება?...
ჰემიანოპია კლინიკური დიაგნოზია. მაგას MRi-ით ვერ დასვამ. ეგ რომ დასვა, ავადმყოფის მხედველობის ველები უნდა შეამოწმო.

Posted by: theaeka 7 Jan 2009, 22:33
იმიტომ რომ შეშუპებულ ლორწოვანში იფარება (ეფლობა) რეცეპტორებიი.

Posted by: Freedoctor 7 Jan 2009, 22:50
QUOTE (vano_t @ 7 Jan 2009, 22:32 )
zviadcardio
QUOTE
QUOTE
რეის სინდრომზე არ ვამბობ - მაგას მართლა არაფერი ეშველება-

თუ სწორედ მახსოვს მაგ დროს ნ-აცეტილცისტეინის ვენის ფორმა გამოიყენება. Orphan Drug-ია.
თემას ოდნავ გადავუხვიე, მაგრამ...

აცეტილცისტეინს იყენებენ აცეტამინოფენით მოწამვლისას (overdose).

რეის სინდრომი შეიძლება ასპირინის მცირე დოზითაც მოხდეს. კორტიკოსტეროიდები და დიურეტიკები ჭირდებათ ამ დროს.

Freedoctor
QUOTE
ემერაიზე რომ სურათია.. პაციენტს ბიტემპორალური ჰემიანოპია ხომ არ აქვს?.. ანუ ეგ რაღაც ქიაზმას ხომ არ აწვება?...
ჰემიანოპია კლინიკური დიაგნოზია. მაგას MRi-ით ვერ დასვამ. ეგ რომ დასვა, ავადმყოფის მხედველობის ველები უნდა შეამოწმო.

vax shen ra magari rame tqvi exla.. me magas vekitxebi marto tsingliani gamosaxuleba rom chans rame daayole metqi.. klinika an rame vax.. amixsna exla bitemporaluri hemianopia.. smile.gif

Posted by: Guardian 7 Jan 2009, 22:56
Freedoctor
QUOTE
vax shen ra magari rame tqvi exla.. me magas vekitxebi marto tsingliani gamosaxuleba rom chans rame daayole metqi.. klinika an rame vax.. amixsna exla bitemporaluri hemianopia..

ჯერ ერთი, არ გირჩევ ვანოსთან მაგ ტონით საუბარს.
და მეორე - ლათინური ასოებით ნუ წერ - ძნელი წასაკითხია.

Posted by: Freedoctor 7 Jan 2009, 23:12
natserit tons rogor archev.. me megobrulad mivtsere da ratom aRiqvi ase cudad ver vxvdebi.. smile.gif

latinurad imitom vtser, rom sxvanairad ar itsereba.. da ra vqna.. smile.gif

Posted by: Cousteau 7 Jan 2009, 23:13
was ist das?

user posted image


Posted by: basa-ttt 7 Jan 2009, 23:18
QUOTE
was ist das?

აპენდიქს???
ნამეტანი დიდია....


Posted by: Cousteau 7 Jan 2009, 23:19
QUOTE (basa-ttt @ 7 Jan 2009, 23:18 )
QUOTE
was ist das?

აპენდიქს???
ნამეტანი დიდია....

no.gif
თუმცა არა მაგიტომ რო დიდი არის

Posted by: donvaso 7 Jan 2009, 23:21
Cousteau
დივერტიკული?
rolleyes.gif

Posted by: Cousteau 7 Jan 2009, 23:24
QUOTE (donvaso @ 7 Jan 2009, 23:21 )
Cousteau
დივერტიკული?
rolleyes.gif

ნუ მთლად იმას არ ამბობ რაც უნდა თქვა

Posted by: Guardian 7 Jan 2009, 23:36
Freedoctor
QUOTE
natserit tons rogor archev..

დარწმუნებული ვარ, მშვენივრად მიხვდი, რასაც ვგულისხმობდი.
QUOTE
me megobrulad mivtsere da ratom aRiqvi ase cudad ver vxvdebi..

კარგი, გავიარეთ.
QUOTE
latinurad imitom vtser, rom sxvanairad ar itsereba.. da ra vqna..

"გამოხმაურების" ფანჯრის ზემოთ ჩაკლიკე "ქართული კლავიატურა"-ს წინ.
თუ მაინც არაფერი ეშველა, იმავე ფანჯრის გვერდზე, მარცხნივ არის "abg --> აბგ" და ტექსტს რომ აკრეფ, მერე მაგაზე დაკლიკე.

Cousteau
QUOTE
ნუ მთლად იმას არ ამბობ რაც უნდა თქვა

biggrin.gif

Posted by: Cousteau 7 Jan 2009, 23:46
QUOTE (Guardian @ 7 Jan 2009, 23:36 )


Cousteau
QUOTE
ნუ მთლად იმას არ ამბობ რაც უნდა თქვა

biggrin.gif

gigi.gif user.gif
შენ რო გეკითხა, დივერტიკულს ფაისაღი არ ჩათვლიდი პასუხად gigi.gif


hint:
ხშირად აკეთებენ ოპერაციას სადაც ეძებენ ამას (სიმპტომების გამო)

user posted image

მაგრამ ხვდებათ ეგ

hint 2: დივერტიკულია ოღონდ special დივერტიკული

Posted by: SOULFUL 8 Jan 2009, 00:33
Cousteau


მეკელის დივერტიკული

Posted by: Cousteau 8 Jan 2009, 02:39
QUOTE (SOULFUL @ 8 Jan 2009, 00:33 )
Cousteau


მეკელის დივერტიკული

smile.gif
yes.gif
Один за всех, и все за одного !!!

Posted by: Freedoctor 11 Jan 2009, 15:53
აბა თუ გამოიცნობთ.. smile.gif

http://imageshack.us
http://g.imageshack.us/img80/98532f1cb2.jpg/1/


Posted by: LUKA-BRAZI 11 Jan 2009, 16:18
Freedoctor
ინტერესნო biggrin.gif რაღაც სახის ტრამვაა მგონი baby.gif ფიზიკური, ქიმიური და ა.შ.

Posted by: Freedoctor 11 Jan 2009, 16:34
LUKA-BRAZI


არა... no.gif

მაშინ დავამატებ რომ ყურზეც აქვს მსგავსი ლაქა.. smile.gif

Posted by: SOULFUL 11 Jan 2009, 18:41
Freedoctor

მელანოზი rolleyes.gif

Posted by: Freedoctor 11 Jan 2009, 19:23
QUOTE (SOULFUL @ 11 Jan 2009, 18:41 )
Freedoctor

მელანოზი rolleyes.gif

კაი მაშინ ეხლა ისეთს დავამატებ, რომ უნდა მიხვდეთ.. smile.gif

შარდიც იგივე ფერისაა... yes.gif

Posted by: SOULFUL 11 Jan 2009, 19:47
Freedoctor
QUOTE
შარდიც იგივე ფერისაა...


მაშინ ალკაპტონურიაა

Posted by: mtvareuli 11 Jan 2009, 19:51
Freedoctor

ალკაპტონურია
* * *
SOULFUL

უი, გითქვია biggrin.gif
* * *
Led_Zeppelin

spy.gif

აქ არ არის მაგის ადგილი

Posted by: LUKA-BRAZI 11 Jan 2009, 19:56
SOULFUL
up.gif ალკაპტონურია
* * *
Led_Zeppelin
QUOTE
7329 წელი

მართლაც მასეა yes.gif

Posted by: Freedoctor 11 Jan 2009, 20:47
ჰო... ალკაპტონურია... You're Damned Correct... biggrin.gif

Posted by: Guardian 11 Jan 2009, 21:40
----------------------------------------------------------------

Posted by: LUKA-BRAZI 11 Jan 2009, 23:54
Guardian
QUOTE
It is wrong for a man to say that he is certain of the objective truth of any proposition unless he can produce evidence which logically justifies that certainty.

And if he does so, I mean if that man produces evidence which logically justifies that certainty, one can require the evidence, which will logically justify the certainty of that evidences, that was produced before.
biggrin.gif
That's the answer for your signature! biggrin.gif what about the case? oh dear, the picture didn't open in my browser givi.gif sorry for that! lol.gif any kind of complaints? address them to Mr. Bill Gates, Microsoft Corporation biggrin.gif

Posted by: Guardian 12 Jan 2009, 00:14
LUKA-BRAZI
QUOTE
And if he does so, I mean if that man produces evidence which logically justifies that certainty, one can require the evidence, which will logically justify the certainty of that evidences, that was produced before.

http://forum.ge/?f=84&showtopic=33811792&st=45
biggrin.gif

Posted by: LUKA-BRAZI 12 Jan 2009, 00:45
Guardian
spy.gif
I didn't quite guess what you mean by givvin' me that link Mr. Guardian! Oh, btw, does that quote belong to Mr. Thomas Huxley?
და რატომ ათეიზმი?! ათეიზმისგან ისევე შორს ვარ როგორც ანდრომედას გალაქტიკა დედამიწიდან და კიდე უფრო შორსაც, რაღა ეგ ლინკი დადე? smile.gif
and still.... if you like quotations, I'll write here one for you wink.gif
Here it is:
Every day I remind myself that my inner and outer life are based on the labors of other men, living and dead, and that I must exert myself in order to give in the same measure as I have received and am still receiving.

Albert Einstein



ჰოდა მოდერი რომ არ გაგვინაწყენდეს, რა ოფთოფიქებია აქო.... smile.gif შენ ქეისზე, ის წითლები შემთხვევით ჰემოსიდერინის მარცვლები ხომ არ არის?

Posted by: Guardian 12 Jan 2009, 01:25
LUKA-BRAZI
QUOTE
does that quote belong to Mr. Thomas Huxley?

yes.
QUOTE
და რატომ ათეიზმი?

ათეიზმი არა, აგნოსტიციზმი.
QUOTE
ის წითლები შემთხვევით ჰემოსიდერინის მარცვლები ხომ არ არის?

არა.

Posted by: Blind_Torture_Kill 12 Jan 2009, 08:43
LUKA-BRAZI
QUOTE
ეს მრგვალი თეთრი რაღაც მეჩვენება საეჭვოდ და რა არის?


smile.gif

vano_t
QUOTE
სურათები ვნახე ადენომების და კრანიოფარინგიომების. ადენომებზე მასეთი ინტენსიური სიგნალი არ მინახია


yes.gif
ინტენსიური სიგნალი იმიტომაა რომ სითხეა

basa-ttt
QUOTE
თუ არადა, მაშინ თურქული კეხის არეა ნამდვილად...


კი


მოკლედ ამ პიროვნებას ჰიპოფიზი აღარ აქვს და მისი ადგილი სითხეს უკავია

მე მაპატიეთ ცოტა კლინიკა უნდა დამედო
* * *
Guardian

სხვა სურათს არ გვანახებ ?

Posted by: LUKA-BRAZI 12 Jan 2009, 13:48
Guardian
QUOTE
აგნოსტიციზმი.

მაგისგანაც კატასტროფულად შორს ვარ... მე მართლმადიდებელი ქრისტიანი ვარ და მწამს ღმერთის არსებობა, არავითარი ათეიზმი და აგნოსტიციზმი! და ნაციონალისტობა-მიშისტობა კიდე biggrin.gif და დადე რამე 'ადამიანური' სურათი მაგ ქეისის, თორე ხო იცი, დიდების გვირგვინი ისევ შენ დაგრჩება smile.gif
Blind_Torture_Kill
დღეს ვაპირებდი შენთვის PM-ის მოწერას, ხო მშვიდობაა რო არ ჩანხარ? თუ ბაკურიანში იყავი საგუნდაოდ? givi.gif

Posted by: Freedoctor 12 Jan 2009, 14:16
Guardian

ლიპოფუსცინოზი?



ან წამლისმიერი.. :unsure:

Posted by: Blind_Torture_Kill 12 Jan 2009, 18:27
LUKA-BRAZI

smile.gif

მაზერი დამეწვა და გამოთიშული ვიყავი თან მეცადინეობა დავიწყე და ხშირად ვეღარ გავიჩითები
მარა ქეისებს დავყრი smile.gif

ამ სურათს კიდე ვერ გავუგე ვერაფერი ხოდა
იქნებ გვეშველოს და გარდიანმა გვაღირსოს რამე სხვა სურათიც smile.gif


Posted by: mtvareuli 12 Jan 2009, 21:10
LUKA-BRAZI
Guardian

რელიგიური შეხედულებები სხვაგან გაარჩიეთ გეთაყვა(თ) 2kiss.gif

Posted by: Guardian 12 Jan 2009, 23:20
Freedoctor
QUOTE
ლიპოფუსცინოზი? ან წამლისმიერი

არა.

Blind_Torture_Kill
QUOTE
იქნებ გვეშველოს და გარდიანმა გვაღირსოს რამე სხვა სურათიც

აჰა -

Posted by: Cousteau 13 Jan 2009, 00:12
http://forum.ge/?f=43&showtopic=33921799

ამას შეხედეთ რა

Posted by: Blind_Torture_Kill 13 Jan 2009, 02:28
ცოტა რამე მინიშნებები თუ კიდე შეიძლება დადე

Posted by: LUKA-BRAZI 13 Jan 2009, 14:40
Blind_Torture_Kill
QUOTE
ცოტა რამე მინიშნებები თუ კიდე შეიძლება დადე

yes.gif
ჰო რა Guardian, ცოტა მიგვანიშნე რამეზე smile.gif


Posted by: Cousteau 13 Jan 2009, 16:45
Guardian

მიკროზე ამილოიდებია?

Posted by: Guardian 13 Jan 2009, 19:55
Cousteau
QUOTE
მიკროზე ამილოიდებია?

არა.

Blind_Torture_Kill
QUOTE
ცოტა რამე მინიშნებები თუ კიდე შეიძლება დადე

აღარ შეიძლება.

მიკროს სურათი აღწერეთ - რას ხედავთ?

Posted by: LUKA-BRAZI 13 Jan 2009, 22:52
Cousteau
QUOTE
ამას შეხედეთ რა

რადიოლოგიაში..... user.gif

Posted by: Guardian 14 Jan 2009, 23:31
ეეეჰ...
ტაფა მაინც არ გაგირეცხავთ როდესმე? biggrin.gif

Posted by: LUKA-BRAZI 14 Jan 2009, 23:39
Guardian
QUOTE
ტაფა მაინც არ გაგირეცხავთ როდესმე?

no.gif სამზარეულო მე არ მეხება სახლში smile.gif
და btw მაგ ქეისის ამოსახსნელად ტაფის გარეცხვა აუცილებელია? spy.gif
givi.gif
* * *
მოდი მივყვეთ ლოგიკურად... ტაფა ყველასათვის ნაცნობი ნივთია და იმედია Mr. Guardian-ს რაიმე განსაკუთრებული ტაფა არ აქვს მხედველობაში biggrin.gif რა არის ტაფაში?... თან გასარეცხ ტაფაში?.... ლოგიკური პასუხი: ის რაც იმ ტაფაში იქნა მომზადებული... და მაინც?... გამომდინარე ტაფის ფიზიკო-ქიმიური თვისებებიდან, იმისათვის რომ ტაფამ კერძი არ მიიწვას, საჭიროა ცხიმი !!! ე.ი. გარდიანის ქეისი ცხიმს უკავშირდება...
ეს გავარკვიეთ....
რა იქნება შემდეგი ნაბიჯი?
მიკროზე აშკარად ბაზოფილური გარემოა yes.gif ის წითლები ცხიმის ჩანართებია ალბათ... next... მარა რა ქსოვილიდან არის პრეპარატი დამზადებული? მიკროდან ვერ ვხვდები.... რატომ დადო CT სურათი? რადიოლოგიაში მოვიკოჭლებ, მაგრამ პირველ კტ-ზე ერთერთი ჰემისფერო out of order არის biggrin.gif ძვლის სურათი რატომ წყდება მაგ სურათზე? არტეფაქტია?
ხომ ხედავ რამდენი კითხვებია გარდიან.... არ შეიძლებოდა ტაფის ნაცვლად რაიმე რაციონალური მაგალითი მოგეყვანა? biggrin.gif
ერთ გაფიქრებით ესეც კი ვიფიქრე, რომ მიკროზე ნერვული ქსოვილის პრეპარატია, ის კი კაპილარის სანათური.... მაგრამ არა, ეს არც ნერვულ ქსოვილს გავს და არც ენდოთელიოციტები ჩანს....
აბა რაღაააა ვაა?!
ბოლო-ბოლო მართლა ტაფის ნარეცხი ხო არაა?!
vik.gif

Posted by: vano_t 15 Jan 2009, 01:54
ლუკა
QUOTE
მიკროდან ვერ ვხვდები.... რატომ დადო CT სურათი? რადიოლოგიაში მოვიკოჭლებ, მაგრამ პირველ კტ-ზე ერთერთი ჰემისფერო out of order არის biggrin.gif
biggrin.gif
ჩემი შვილი იტყოდა "გაუფუჭებულია თავი" biggrin.gif
ასეთ CT-ზე პათოლოგიის უბნების გამოცნობა ძალინა მარტივია (თვითონ დაზიანებულ უბანს ვგულისხმობ და არა დიაგნოზის დადგენას დაზიანების მიხედვით). ის თეთრი უბნები, რაც რუხ და თეთრი ნივთიერებების უბნებში ჩანს (ერთგან თვითონ დურაც არის ძალიან თეთრი, ესეც პათოლოგიურია), არის ყველა არანორმალური. მასეთ სიგნალს ბევრი რამ არ გაძლევს. მასეთ სიგნალს რომ ნახულობ, ან ჰემორაგიაზე ფიქრობ (ჰემორაგია ასე გაფანტული არ არის-უფრო ერთ უბანს მოიცავს), ან კალციუმის ჩალაგებაზე. კალციაუმი ბევრმა რამემ შეიძლება მოგცეს (მაგალითად ქრონიკულმა ანთებებმა). მარა, კალციუმის გარდა სხვა მეტალმაც შეიძლება მოგცეს მასეთი სიგნალი (ალბათ ალუმინმაც-ისე ვცდილობ გუგლ-იმიჯში ვნახო მასეთი კტ, მარა ჯერ ვერ ვნახე). გარდა ამისა, ალუმინმა შეიძლება მოგცეს ალცჰეიმერის მსგავსი ნეიროფიბრილარული გროვა (Neurofibrillary tangles). გარდიანის პირველ სურათზე რომ ლურჯი ოვალური არეები კიდევ უფრო ინტენსიური ლურჯი ფერის "ბირთვებით" წააგავს მასეთ რამეს. თავს ვერ დავდებ. უბრალოდ (ტაფა რომ ახსნეა biggrin.gif) მერე ვნახე ნეიროფიბრილარული გროვების სურათები მცირე გადიდებაზე ალცჰეიმერის დროს და წააგავს.
user posted image

ჩემი სავარაუდო დიაგნოზია "ალუმინით მოწამლვა" biggrin.gif

Guardian
ალუმინით მოწამლვაა?

Posted by: Guardian 15 Jan 2009, 02:07
vano_t
QUOTE
ალუმინით მოწამლვაა?

არა. biggrin.gif

LUKA-BRAZI
QUOTE
ერთ გაფიქრებით ესეც კი ვიფიქრე, რომ მიკროზე ნერვული ქსოვილის პრეპარატია, ის კი კაპილარის სანათური.... მაგრამ არა, ეს არც ნერვულ ქსოვილს გავს და არც ენდოთელიოციტები ჩანს....

არა კი არა, კი.
მაშ, რა არის დიაგნოზი?

Posted by: Cousteau 15 Jan 2009, 02:36
QUOTE
ალცჰეიმერის დროს

ალცჰეიმერის დროს არის პერივასკულარულად ამილოიდის ჩალაგება და არაო გარდიანმა და რიავიცი : /

Posted by: vano_t 15 Jan 2009, 02:49
Guardian
კაი მაშინ biggrin.gif

მე მაგ პათოლოგიით მაინც ვერ გამოვიცნობ, ამიტომ ვეძებ ტვინის ინტენსიური კალციფიკაციის მიზეზებს. რამოდენიმე მიზეზი ვნახე და ყველა დიაგნოზს ჩამოვთვლი biggrin.gif
http://cat.inist.fr/?aModele=afficheN&cpsidt=3089223

ჰიპოპარათირეოიდიზმი

კალმანის სინდრომი
Cockayne's, Kearns-Sayre and Down's syndromes

tuberous sclerosis

carbonic anhydrase II deficiency

სისტემური სკლეროზი

აქედან ერთ-ერთი თუ სწორი არ არის, მაშინ გამაგდე ამ განყოფილებიდან biggrin.gif


Posted by: Guardian 15 Jan 2009, 02:55
vano_t
QUOTE
აქედან ერთ-ერთი თუ სწორი არ არის, მაშინ გამაგდე ამ განყოფილებიდან

არცერთი არაა სწორი, მაგრამ არ გაგაგდებ. biggrin.gif

btw, ტაფა ტყუილად არ მიხსენებია. tongue.gif

Posted by: LUKA-BRAZI 15 Jan 2009, 14:43
Guardian
ათეროსკლეროზი?! just simply atherosclerosis და ამდენი ამაზე ვიჯახირეთ? spy.gif

Posted by: Guardian 15 Jan 2009, 14:58
LUKA-BRAZI
QUOTE
ათეროსკლეროზი?! just simply atherosclerosis და ამდენი ამაზე ვიჯახირეთ?

არა.
არადა, კარგად მიდიოდი. smile.gif

Posted by: LUKA-BRAZI 15 Jan 2009, 16:19
Guardian
QUOTE
btw, ტაფა ტყუილად არ მიხსენებია.

ჰმმ, ტაფა ისევ საქმეშია smile.gif კარგი, ჯერ ჩამოვწეროთ რა ვიცით: ვიცით რომ მიკრო პრეპარატზე გამოსახულია ნერვული ქსოვილი სისხლძარღვით, რომლის კედლებთან გარეთ და შიგნით ცხიმის მარცვლებია... კითხვა მდგომარეობს იმაში, რა როცესია გამოსახული სურათზე. ე.ი. რახან ვანოს ვარიანტები გამოირიცხა, აქედან დასკვნა: Ca აქ მთავარ პათოფიზიოლოგიურ და პათომორფოლოგიურ როლს არ თამაშობს, ე.ი. დამნაშავე ცხიმია და CT-ზეც არა კალციფიცირება არამედ სკლეროზული უბნებია გამოსახული....
კეთილი და პატიოსანი biggrin.gif
რა გვაძლევს სკლეროზს? უამრავი ფაქტორი, მაგრამ ამ ფაქტორებიდან რა უკავშირდება ცხიმს? ერთი ვარიანტია ათეროსკლეროზი, რაც სავსებით გვაძლევს ქსოვილოვან იშემიას შემდგომი სკლეროზული უბნების განვითარებით, რაც თავისმხრივ კტ-ზე მოგვცემს მკვეთრ გამოსახულებას... მაგრამ გარდიანმა არაო!
ok, კიდევ რა პათოლოგიური პროცესების მექანიზმშია ცხიმოვანი ცვლის მოშლა წამყვანი? ცხიმოვანი დისტროფია? ცხიმოვანი ბალონური დისტროფია? შაქრიანი დიაბეტი, რომელსაც თან ახლავს ცხიმოვანი ცვლის მოშლა? მეზენქიმური ცხიმოვანი დაგროვება-ლიპიდოზები??? რა, რა, რააა???

Posted by: Guardian 15 Jan 2009, 16:24
LUKA-BRAZI
QUOTE
არამედ სკლეროზული უბნებია გამოსახული....

და, იქნებ, სულაც არ არის სკლეროზული? biggrin.gif

Posted by: LUKA-BRAZI 15 Jan 2009, 16:35
Guardian
კაცო შენ ბოლო-ბოლო გინდა თუ არა რო ეგ ქეისი გამოვიცნოთ? gigi.gif
თუ ჩაფიქრებული გაქვს სანამ 10 გვერდს არ გავავსებთ მარჩიელობაში, არაფერს მიგვანიშნებ?
From now tips are required for your cases Mr. Huxley! biggrin.gif რამე კიდე მიგვანიშნე, კიდე რომელი ჭურჭლის გარეცხვა მიგვიყვანს დიაგნოზამდე? biggrin.gif

Posted by: basa-ttt 15 Jan 2009, 17:15
QUOTE
ეეეჰ...
ტაფა მაინც არ გაგირეცხავთ როდესმე?

მარცხენა მხარეს ძვლის მთლიანობაა დარღვეული?
მოტეხილობაა თუ ტაფა ჩაარტყეს თავში?
ის ჭრელი უბნები ჰემორაგიებია?


user posted image


user posted image
* * *
QUOTE
ok, კიდევ რა პათოლოგიური პროცესების მექანიზმშია ცხიმოვანი ცვლის მოშლა წამყვანი? ცხიმოვანი დისტროფია? ცხიმოვანი ბალონური დისტროფია? შაქრიანი დიაბეტი, რომელსაც თან ახლავს ცხიმოვანი ცვლის მოშლა? მეზენქიმური ცხიმოვანი დაგროვება-ლიპიდოზები??? რა, რა, რააა???

დამწვრობა?
მე მაინც ტრავმა მგონია.

Posted by: LUKA-BRAZI 15 Jan 2009, 18:21
basa-ttt
ძვლის ამბავი მეც მაინტერესებს, მაგრამ ტრამვა რომ იყოს მიკროპრეპარატი რა შუაში იქნება?

Posted by: basa-ttt 15 Jan 2009, 18:27
QUOTE
ძვლის ამბავი მეც მაინტერესებს, მაგრამ ტრამვა რომ იყოს მიკროპრეპარატი რა შუაში იქნება?

ნუუ -
თუ ცხელი ტაფა ჩაარტყეს თავში -
ალბათ ცხიმის წვეთები მოხვდა ტვინის ქსოვილში..
lol.gif
შეიძლება დამწვრობაც იყოს...
...

Posted by: LUKA-BRAZI 15 Jan 2009, 18:38
basa-ttt
QUOTE
თუ ცხელი ტაფა ჩაარტყეს თავში -

biggrin.gif
QUOTE
შეიძლება დამწვრობაც იყოს...

რეიზა? დამწვრობას კოაგულაციური ნეკროზი ახასიათებს თუ არ ვცდები (თუ მაღალი ტემპერატურით მიღებულ დამწვრობას გულისხმობ)....

Posted by: Guardian 15 Jan 2009, 18:47
LUKA-BRAZI
QUOTE
ძვლის ამბავი მეც მაინტერესებს

ძვალი მანდ არაფერ შუაშია - უბრალოდ, ტრეპანაციაა გაკეთებული, მგონი.
მომწყინდა უკვე - თუკი მანამდე ვერავინ გამოიცნო, სწორ პასუხს ხვალ დილას გეტყვით. smile.gif

Posted by: LUKA-BRAZI 15 Jan 2009, 19:26
Guardian
შე ცუდო biggrin.gif სულ პაწუკა მინიშნება არ შეიძლება? ისევ შენ უნდა დაიდგა არა დიდების გვირგვინი ხომ? gigi.gif

არაუშავს, სამაგიეროდ მე 118 გვერდი მაქვს დავალება მოცემული ხვალისთვის ერთერთ საგანში (არ დავკონკრეტდები).... კიდე იტყვის ვინმე სამედიცინოზე სწავლა არააო? მერე რა რომ იქ რაციონალური აზრი სანთლით საძებარია? მთავარია: სწავლა, სწავლა და სწავლა.
უბრალოდ ერთხელ Abraham Lincoln-მა თქვა:
“If I had eight hours to chop down a tree, I'd spend six hours sharpening my ax”
კარგი სიტყვებია. კარგი იქნებოდა მასალა კურაციის ხანგრძლივობისთვის (6 დღე) შეეფარდებინათ.... მაგრამ ეს არ გააკეთეს, ამიტომ შემდეგი 8 საათი ძალიან უაზროდ უნდა გავატარო და მოვალეობა მოვიხადო.... ისინიც მოვალეობას იხდიან.... კი ჩვენთან ევროპული სწავლის დონეა yes.gif არა კაცო, მართლა მასეა, სარკასტულად კი არ ვამბობ smile.gif

Posted by: megusa 15 Jan 2009, 19:46
არც არაფერი ისეთიsmile.gif))))))))))))))))))))))))))))))))))))))))))))))))))))))))))))))))))))))))))))))))))))))))))))))))))))))))))))))))))))))))))

Posted by: basa-ttt 15 Jan 2009, 20:04
QUOTE
Guardian
შე ცუდო



LUKA-BRAZI
ვდრუზოთ ერთი ცხიმიანი ტაფა....
lol.gif vik.gif

Posted by: Guardian 16 Jan 2009, 10:19
ცხიმოვანი ემბოლია.
-------------------------

Posted by: vano_t 16 Jan 2009, 10:23
ტაფასთან მაგიტომ არის კავშირში? biggrin.gif ცხიმი და ტაფა.

გარდიან, ისე ინტერესის გულისათვის, კტ-ზე რა არის სპეციფიური ცხიმოვანი ემბოლიისათვის?

და მიკროპრეპარატის მიხედვით რას ეძებ მაგ დროს?

Posted by: Guardian 16 Jan 2009, 10:30
vano_t
QUOTE
ტაფასთან მაგიტომ არის კავშირში?

უბრალო ტაფასთან - არა, გასარეცხ ტაფასთან. biggrin.gif
QUOTE
კტ-ზე რა არის სპეციფიური ცხიმოვანი ემბოლიისათვის?

არც არაფერი.
ვგონებ, ტვინის ნივთიერებაში ეგეთი თეთრი ლაქები სხვა დაავადებებმაც შეიძლება მოგცეს.
სამაგიეროდ, მიკროს სურათია სპეციფიური - ცხიმიან ტაფას რომ წყალს დაასხამ და ცხიმის ბუშტუკები რომ დაიწყებენ წყალში ტივტივს, იმას ჰგავს ძალიან. biggrin.gif
ეგ პრეპარატი, რა თქმა უნდა, ჰემატოქსილინ-ეოზინით არ არის შეღებილი, თორემ ცხიმი არ გამოჩნდებოდა.
მაგ შეღებვას Oil red O stain ჰქვია.

Posted by: vano_t 16 Jan 2009, 10:40
Guardian
QUOTE
უბრალო ტაფასთან - არა, გასარეცხ ტაფასთან. biggrin.gif
ამას არ დავაკვირდი-მე მეგონა ტაფა გარეცხილი იყო smile.gif აჩმა ხაჩაპური მაინც გეხსენებინა, ან გრძელი ძვლების მოტეხილობა და მისი გართულებები biggrin.gif

QUOTE
არც არაფერი.
ვგონებ, ეგეთი თეთრი ლაქები სხვა დაავადებებმაც შეიძლება მოგცეს.
თეთრ ლაქებს საერთოდ გაძლევს ან სისხლი ან კალციუმი (და ალბათ სხვა მეტალებიც). ცხიმოვანი ემბოლიის დროს შესაძლებელია ჰემორაგია ისე.

QUOTE
სამაგიეროდ, მიკროს სურათია სპეციფიური - ცხიმიან ტაფას რომ წყალს დაასხამ და ცხიმის ბუშტუკები რომ დაიწყებენ წყალში ტივტივს, იმას ჰგავს ძალიან. biggrin.gif
ეს კარგი ინფორმაციაა.

Posted by: Blind_Torture_Kill 17 Jan 2009, 01:27
რა ჭირს ამ ბოვშვს ?

წავედი დავწექი

Posted by: Guardian 17 Jan 2009, 01:56
Blind_Torture_Kill
QUOTE
რა ჭირს ამ ბოვშვს ?

გულის მანკი.
ყველაზე სავარაუდოდ - ფალოს ტეტრადა.

Posted by: Blind_Torture_Kill 17 Jan 2009, 11:41
Guardian
QUOTE
გულის მანკი. ყველაზე სავარაუდოდ - ფალოს ტეტრადა.



up.gif

ეგრე რატოა ჩაკუზული ?

Posted by: SOULFUL 17 Jan 2009, 13:59
Blind_Torture_Kill
QUOTE
ეგრე რატოა ჩაკუზული ?


ქოშინის შესამსუბუქებლად user.gif

Posted by: Blind_Torture_Kill 17 Jan 2009, 19:02
SOULFUL
QUOTE
ქოშინის შესამსუბუქებლად


კი
ისე მექანიზმით რომ დაგეწერა უფრო კაი იქნებოდა

Posted by: vano_t 18 Jan 2009, 03:17
Blind_Torture_Kill
QUOTE
QUOTE
ქოშინის შესამსუბუქებლად


კი
ისე მექანიზმით რომ დაგეწერა უფრო კაი იქნებოდა

ჩაცუცქვით ხდება მხოლოდ preload-ის შემცირება (ქვემო ღურ ვენაში ნაკლები სისხლი შემოდის, იმის გამო, რომ ქვემო კიდურების ვენებში ხდება მისი შეგუბება). ფალოს ტეტრადა: 1) ფილტვის არტერიის სტენოზი და 2) ამასთან დაკავშირებული მარჯვენა პარკუჭის ჰიპერტროფია, 3) VSD, 4) overriding აორტა. ქოშინის მექანიზმი? right-to-left შუნტი (იწვევს ჰიპოქსიას არტერიული და ვენური სისხლის შერევის გამო) ორ ადგილას: VSD და overriding აორტა.

აქედან გამომდინარე, ლოგიკურად, მხოლოდ შუნტის ხარისხის შემცირებამ უნდა გამოიწვიოს ქოშინის შემსუბუქება. შესაბამისად, preload-ის შემცირებამ უნდა გამოიწვიოს შუნტის ხარისხის შემცირება (შუნტის ხარისხში ვგულისხმობ არტერიული და ვენური სისხლის პროცენტულ თანაფარდობას მარცხენა პარკუჭისათვის) შუნტის ერთ-ერთ (ან ორივე) ადგილას. ექსპერიმენტალურად ამის დადგენა არ არის ძნელი. უნდა გასინოჯო ჟანგბადით სისხლის სატურაცია ან არტერიული გაზები დამდგარ მდომარეობაში და ჩაცუცქვის შემდეგ. არტერიული გაზის გასინჯვა გაცილებით უკეთესია, იმიტომ რომ ჰემოგლობინის ჟანგბადით სატურაციის მრუდი ისეთია, რომ ერთიდაიგივე სატურაცია შეიძლება მოგცეს სხვადასხვა PO2-მა.

თუ არტერიული O2-ის ცვლილება მნიშვნელოვნად არ ხდება, მაშინ ეს მექანიზმი სინამდვილეში ვერ ახსნის ქოშინის შემცირების მიზეზს და სხვა მიზეზი უნდა ეძებო. სხვა მიზეზი თვითონ გულის მანკის ანატომიიდან არ ჩანს.

პრინციპში, VSD-იდან მარცენა პარკუჭშიც მცირდება preload (იგივე მიზეზის გამო: ჩაცუცქვა -> მარჯვენა პარკუჭის preload-ის შემცირება -> მარცხენა პარკუჭის პრელოუდის შემცირება) და ამას შეიძლება მოყვეს მარცხენა პარკუწში ბოლო დიასტოლური წნევის შემცირება, რამაც თავის მხრივ შეიძლება გამოიწვიოს ფილტვების წრეში კაპილარული და ვენულური წნევის დაქვეითება. ეს მექანიზმი მოგეხმარება ქოშინის შემცირებაში, თუ ფალოს ტეტრადას თან ახლავს ფილტვების შეშუპება ბოლო დიასტოლური წნევის გაზრდი გამო (მარა, მე არ ვიცი თუ ესეც შეიძლება იყოს სინამდვილეში ქოშინის მიზეზი).

Posted by: Blind_Torture_Kill 18 Jan 2009, 04:22
vano_t

შენ რაც ჩამოთვალე საინტერესო მექანიზმია მარა
მე რაც ვიცი ესეა რომ ჩაკუზვა ზრდის პოსტდატვირთვას რის გამოც გული უფრო მძლავრად იკუმშება და მარცხენა პარკუჭში წნევა საგრძნობლად იმატებს რაც მოგვცემს სისხლის მარცხნიდან მარჯვნივ გადასროლას და არა პირიქით
ვენური სისხლი არ აირევა არტერიულში და ჟანგბადის სატურაციაც ნორმა შენარჩუნდება + გაიზრდება ფილტვში სისხლის ასვლა რაც ასე თუ ისე შეზღუდულია სტენოზით
* * *
..........................................................................
აბა ეს რა არის ?

* * *
http://imageshack.us
http://g.imageshack.us/img132/75594497ca6.jpg/1/

Posted by: donvaso 18 Jan 2009, 12:45
Blind_Torture_Kill
თიაქარი?????

Posted by: Blind_Torture_Kill 18 Jan 2009, 13:51
donvaso

არა
იმენა პიზდეცი აქ ამ პიროვნებას

Posted by: basa-ttt 18 Jan 2009, 14:07
Blind_Torture_Kill
სათესლე ჯირკვლებია?
ქსოვილის ნეკროზი და სისხლჩაქცევებია ?

Posted by: LUKA-BRAZI 18 Jan 2009, 14:37
Blind_Torture_Kill
yes.gif
სათესლე ჯირკვლის ტრამვაა ჰემატომით.

Posted by: vano_t 18 Jan 2009, 14:39
QUOTE (Blind_Torture_Kill @ 18 Jan 2009, 04:22 )
შენ რაც ჩამოთვალე საინტერესო მექანიზმია მარა
მე რაც ვიცი ესეა რომ ჩაკუზვა ზრდის პოსტდატვირთვას რის გამოც გული უფრო მძლავრად იკუმშება და მარცხენა პარკუჭში წნევა საგრძნობლად იმატებს რაც მოგვცემს სისხლის მარცხნიდან მარჯვნივ გადასროლას და არა პირიქით
ვენური სისხლი არ აირევა არტერიულში და ჟანგბადის სატურაციაც ნორმა შენარჩუნდება + გაიზრდება ფილტვში სისხლის ასვლა რაც ასე თუ ისე შეზღუდულია სტენოზით
* * *
..........................................................................
აბა ეს რა არის ?

* * *
http://imageshack.us
http://g.imageshack.us/img132/75594497ca6.jpg/1/

ჩაკუზვამ პოსტლოუდი როგორ უნდა გაზარდოს? პოსტლოადი არის არტერიული წნევის ფუნქცია (შეიძლება პოსტლოადი აორტის სარქვლის სტენოზითაც გაიზარდოს, მარა სტენოზი უცვლელი იქნება). არტერიული წნევა როგორ იცვლება ჩაცუცქვით? თუ რამე, უნდა შემცირდეს, იმიტომ რომ არტერიული წნევა პრელოუდზეა დამოკიდებული და რაც უფრო დაბალია პრელოუდი, მით უფრო მდაბალია წნევა. (შეიძლება ვცდები, მაგრამ ვერ ვხედავ კავშირს ჩაცუცქვასა და წნევის გაზრდა შორის). ზვიადკარდიო რას იტყვის ნეტა? ისე შუნტის შებრუნება კი მოხსნის ქოშინს, მარა, როგორც ვთქვი, მე ვერ ვხვდები ჩაცუცქვით როგორ უნდა გაიზარდოს პოსტლოადი.

შენ სურათზე არის ფურნიერის განგრენა, კაცის სასქესო ორგანოების მანეკროტიზებელი ფასციიტი.

Posted by: Blind_Torture_Kill 22 Jan 2009, 02:18
QUOTE
ჩაკუზვამ პოსტლოუდი როგორ უნდა გაზარდოს? პოსტლოადი არის არტერიული წნევის ფუნქცია (შეიძლება პოსტლოადი აორტის სარქვლის სტენოზითაც გაიზარდოს, მარა სტენოზი უცვლელი იქნება). არტერიული წნევა როგორ იცვლება ჩაცუცქვით? თუ რამე, უნდა შემცირდეს, იმიტომ რომ არტერიული წნევა პრელოუდზეა დამოკიდებული და რაც უფრო დაბალია პრელოუდი, მით უფრო მდაბალია წნევა. (შეიძლება ვცდები, მაგრამ ვერ ვხედავ კავშირს ჩაცუცქვასა და წნევის გაზრდა შორის). ზვიადკარდიო რას იტყვის ნეტა? ისე შუნტის შებრუნება კი მოხსნის ქოშინს, მარა, როგორც ვთქვი, მე ვერ ვხვდები ჩაცუცქვით როგორ უნდა გაიზარდოს პოსტლოადი.


ცდა გააკეთე და ნახავ


სურათზე განგრენაა მართლაც სასქესო ორგანოსი
(ესეთი რამე პირველად ვნახე)

* * *
http://imageshack.us
http://g.imageshack.us/img217/74835645xl3.jpg/1/

Posted by: basa-ttt 22 Jan 2009, 18:48
Blind_Torture_Kill
ბოლო ფოტო რენტგენია?
ის ბრჭყვიალა წერტილებია ყურადმისაქცევი -
თუ უბრალოდ ფირის დეფექტია?

ხალხო ცოტა კლინიკაც დადეთ ხოლმე -
თუ გინდათ, რომ თემამ პრაქტიკული ღირებულება შეიძინოს..

Posted by: Cousteau 22 Jan 2009, 18:49
QUOTE (Blind_Torture_Kill @ 22 Jan 2009, 02:18 )

* * *
http://imageshack.us
http://g.imageshack.us/img217/74835645xl3.jpg/1/

საფანტიაქ მოხვედრილი ?

Posted by: LUKA-BRAZI 22 Jan 2009, 21:33
Cousteau
კუსტოს გაუმარჯოს smile.gif მგონი მართლაც საფანტი აქვს მოხვედრილი yes.gif აბა Blind_Torture_Kill-ის პროფესიიდან გამომდინარე, პირველი მაგას იფიქრებ smile.gif სუიციდის მცდელობა იყო? საფეთქელთან ააქვს კონცენტრირებული საფანტი (თუ საფანტია).... დაინტუბირებულიც არის ხო? smile.gif

Posted by: Blind_Torture_Kill 23 Jan 2009, 00:18
QUOTE
ბოლო ფოტო რენტგენია? ის ბრჭყვიალა წერტილებია ყურადმისაქცევი - თუ უბრალოდ ფირის დეფექტია?

ხალხო ცოტა კლინიკაც დადეთ ხოლმე -თუ გინდათ, რომ თემამ პრაქტიკული ღირებულება შეიძინოს..


CT-ა
სურათით იყო გამოსაცნობი უბრალოდ smile.gif



QUOTE
საფანტიაქ მოხვედრილი ?

კი დაინტუბირებულია



QUOTE
აბა Blind_Torture_Kill-ის პროფესიიდან გამომდინარე, პირველი მაგას იფიქრებ სუიციდის მცდელობა იყო? საფეთქელთან ააქვს კონცენტრირებული საფანტი (თუ საფანტია).... დაინტუბირებულიც არის ხო?


biggrin.gif
სუიციდი არა ლუკა ჰომიციდი
კი დაინტუბირებულია

Posted by: Thandrus 23 Jan 2009, 05:48
საღოლ თქვენ... ერთ წამში როგორ ხვდებით...

Posted by: mtvareuli 3 Feb 2009, 20:58
Blind_Torture_Kill
LUKA-BRAZI
Cousteau
vano_t
Guardian

რა უნდა ამ ტემას მესამე გვერდზე? vik.gif

Posted by: შოთა.03. 3 Feb 2009, 21:13
აბა ეს რაარის
biggrin.gif .....................................

Posted by: mtvareuli 3 Feb 2009, 21:29
შოთა.03.

პროლაფსია მგონი vis.gif


* * *

ჯერ ჩემ სურათზე გეპასუხა.... დავუშვათ ქუჩაში დაინახე ადამიანი ეგეთი ფერით... რას იფიქრებ რა ჭირსო

Posted by: Thandrus 3 Feb 2009, 22:26
mtvareuli

ადდისონის დაავადება ხომ არა?
baby.gif

Posted by: mtvareuli 3 Feb 2009, 22:36
Thandrus
QUOTE
ადდისონის დაავადება

yes.gif

Posted by: basa-ttt 3 Feb 2009, 23:32
შოთა.03.
სწორი ნაწლავის გამოვარდა

ან უფრო ბუასილი

Posted by: Blind_Torture_Kill 4 Feb 2009, 01:14
რაც არ ვარ არაფერი შეცვლილა დიდად
აბა ეს რა არის მარტივი ქეისით დავიწყებ

Posted by: Thandrus 4 Feb 2009, 01:53
Blind_Torture_Kill

რავი, ჰიდრონეფროზი?

Posted by: LULA_QABABI 4 Feb 2009, 06:02
QUOTE (Blind_Torture_Kill @ 3 Feb 2009, 16:14 )
რაც არ ვარ არაფერი შეცვლილა დიდად
აბა ეს რა არის მარტივი ქეისით დავიწყებ

ადრენალების ნეკროზს ვხედამ; waterhouse-friderichsen syndrome ?

Posted by: Blind_Torture_Kill 4 Feb 2009, 14:46
LULA_QABABI

QUOTE
ადრენალების ნეკროზს ვხედამ; waterhouse-friderichsen syndrome ?


ჰო ეგაა

Posted by: Thandrus 4 Feb 2009, 21:03
Blind_Torture_Kill
LULA_QABABI

კი ბატონო... სისხლჩაქცევა არის... ვოტერჰაუს-ფრიდრიხსენი... მაგრამ თირკმელები რატომღა გასივდა ასე? baby.gif

Posted by: შოთა.03. 5 Feb 2009, 11:35
basa-ttt
QUOTE
ან უფრო ბუასილი


Posted by: basa-ttt 5 Feb 2009, 18:32
QUOTE
კი ბატონო... სისხლჩაქცევა არის... ვოტერჰაუს-ფრიდრიხსენი... მაგრამ თირკმელები რატომღა გასივდა ასე?

აბა რა იქნება -
თირკმლის მწვ უკმ და მორსი....

Posted by: Blind_Torture_Kill 7 Feb 2009, 16:12
წინა ქეისში არი თირკმელზედა ჯირკვლების ჰემორაგიული ნეკროზი და უკმარისობა

თირკმელი ჯანმრთელია





2.აბა რა იყო სიკვდილის მიზეზი user.gif

Posted by: nini_blini 7 Feb 2009, 16:27
ორივე თირკმელი ამოგიჭრიათ და მოკვდებოდა აბა რა იქნებოდა biggrin.gif

Posted by: Blind_Torture_Kill 7 Feb 2009, 16:50
nini_blini

ბოროტმა ძია-ექიმმა ჯერ უნდა ამოაჭრას ორგანოები და მერე ისე უნდა დასვას დიაგნოზი
yes.gif

Posted by: nini_blini 7 Feb 2009, 17:04
და დასვით ეგ დიაგნოზი ბოლო ბოლო???

Posted by: Thandrus 7 Feb 2009, 17:09
მენინგოკოკური მენინგიტი... ვოტერჰაუს-ფრიდრიხსენი მაგას ახლავს.

Posted by: nini_blini 7 Feb 2009, 17:12
ვაა, მე მეგონა ეგ მარტო თავის ტვინს ნერვებს აზიანებდა..

Posted by: Blind_Torture_Kill 8 Feb 2009, 17:33
ძველი ხალხი სადაა

აღარავინ აღარ ჩანხართ sad.gif


Posted by: LUKA-BRAZI 8 Feb 2009, 17:48
Blind_Torture_Kill
QUOTE
ძველი ხალხი სადაა

აღარავინ აღარ ჩანხართ sad.gif

აბა ბიჯო! მართლა სად დავიკარგეთ? smile.gif სად არიან vano_t, Guardian, Cousteau და სხვები? იცი რა გამახსენდა? აღარ მახსოვს როდის იყო, თითქმის ღამე რომ გავათენეთ ქეისების გარჩევაში.... კაი იყო smile.gif მომენატრა user.gif შენი ავათარიდან გამომდინარე, ალბათ რაც დრო გადის უფრო მუღამში შედიხარ არა? smile.gif
QUOTE
ბოროტმა ძია-ექიმმა
biggrin.gif
მე ვერ ვიცლი, რა ვქნა.... ისეთი კურაციები მიეწყო რომ..... ისე მოდი დავთქვათ და რომელიმე დღეს შევიკრიბოთ ფორუმზე online.... რამე ჯიგრული ქეისი მოიტანე და დავცხოთ smile.gif

Posted by: Blind_Torture_Kill 8 Feb 2009, 18:38
LUKA-BRAZI

QUOTE
მე ვერ ვიცლი, რა ვქნა.... ისეთი კურაციები მიეწყო რომ..... ისე მოდი დავთქვათ და რომელიმე დღეს შევიკრიბოთ ფორუმზე online.... რამე ჯიგრული ქეისი მოიტანე და დავცხოთ


მოსულა


მანამდე დევს მაღლა სურათი smile.gif

QUOTE
აბა ბიჯო! მართლა სად დავიკარგეთ?  სად არიან vano_t, Guardian, Cousteau და სხვები? იცი რა გამახსენდა? აღარ მახსოვს როდის იყო, თითქმის ღამე რომ გავათენეთ ქეისების გარჩევაში.... კაი იყო  მომენატრა  შენი ავათარიდან გამომდინარე, ალბათ რაც დრო გადის უფრო მუღამში შედიხარ არა? 


მართლა კაი დრო იყო

Posted by: LUKA-BRAZI 8 Feb 2009, 19:36
Blind_Torture_Kill
ეგ ქეისი რომ გამოიცნო, ჯერ ორგანო უნდა გამოიცნო biggrin.gif
ვა, რა აფორიზმივით გამომივიდა biggrin.gif
მგონი ეგ გულია და დაავდება კი ჰიპერტროფიული კარდიომიოპათია, მაგრამ თუ ეგ გულია მაშინ მასეთი ფერის გული არსად არ მინახავს smile.gif მარა შენ ისეთ სურათებს აძრობ ხოლმე რომ smile.gif

Posted by: Blind_Torture_Kill 9 Feb 2009, 12:07
QUOTE
ეგ ქეისი რომ გამოიცნო, ჯერ ორგანო უნდა გამოიცნო 
ვა, რა აფორიზმივით გამომივიდა 
მგონი ეგ გულია და დაავდება კი ჰიპერტროფიული კარდიომიოპათია, მაგრამ თუ ეგ გულია მაშინ მასეთი ფერის გული არსად არ მინახავს  მარა შენ ისეთ სურათებს აძრობ ხოლმე რომ 


biggrin.gif
გულია
ფერი რატო არ მოგწონს ?
მწვერვალთანაა გადაჭრილი და ჰიპერტროფიულ კარდიომიოპათიას გავს მარა ეგ არაა smile.gif
რამე განსაკუთრებული თვალში არ გეცა ?

Posted by: LUKA-BRAZI 9 Feb 2009, 16:02
Blind_Torture_Kill
QUOTE
მწვერვალთანაა გადაჭრილი და ჰიპერტროფიულ კარდიომიოპათიას გავს მარა ეგ არაა smile.gif რამე განსაკუთრებული თვალში არ გეცა ?

რა გითხრა აბა? ჯერ ერთი ის მუქი მერი არ მომწონს წითლად რომ ავღნიშნე.... რა უნდა იყოს ეგ? ინფარქტს კი ახასიათბს ქსოვილის ფერის შეცვლა იმიტომ რომ ინფარქტის ზონაში ქსოვილი მკვდარია, მარა ენდოკარგიუმიც მთლიანად მასე ჰომოგენურად როა შეფერილი? და მაიც ვერ მივხვდი რა ჭრილშია განაკვეთი გაკეთებული, ფრონტალურ თუ საგიტალურ ჭრილში, იმიტომ რომ ის რაღაცა რაც მწვანედ მოვნიშნე თუ წინაგულია, მაშინ სად არის მეორე წინაგული? და თუ როგორც შენ ამბობ მწვერვალთანაა გადაჭრილი, მაშინ მწვანედ აღნიშნული ნაწილი ყოვლად გუგებარი რაღაც გამოდის იმიტომ რომ სარქველი ეგ ვერ იქნება... biggrin.gif მოკლედ ლოლზ! ვერ მივხვდი biggrin.gif ჩამინამიოკე smile.gif

QUOTE
Doc I Have Diarrhea

ეს უკვე ახალ ავათარზე აღარ მიდის biggrin.gif

Posted by: basa-ttt 9 Feb 2009, 17:11
user posted image

გულია - და მისი
ენდოკარდიუმის და წინაგულების არე ფერშეცვლილია -
სავარაუდოდ ნეკროზია -
თუ არ ვცდები მწვერვალის და მარცხენა პარკუჭის

ის რასაც ლუკა ამბობს -
მრგვალი ფორმის წარმონაქმნი -
ნუ მოჟეტ მიქსომაა -
ინფარქტი ნამდვილად არის -
შეიძლება მიქსომაც?
კლინიკას რატომ არ დებთ?

Posted by: @@@@ 9 Feb 2009, 18:03
თუ შეგიზლიათ მითხრათ, რას ნიშნავს "ძარღვის გაკვანძა". ამიხსენით სამედიცნი ენაზე რა იწვევს ამ აუტანელ ტკივილს და როგორ ქრება ესე უკვალოდ. მე მემართება ხოლმე ხIშრად დაცოლილ მდგომარეობაში და თან ერთიდაიგივე ფეხზედა ნეტა რისი ბარლი უნდა იყოს? rolleyes.gif
წინასწარ დიდი მადლობა პასუხისათვის

Posted by: LUKA-BRAZI 9 Feb 2009, 19:03
@@@@
ეგეთი რამე სისხლძარღვოვან პრობლემებს ახლავს თან... მაგალითად ვენების ვარიკოზულ გაგანიერებას.... შეიძლება ამის სიმპტომი იყოს... ოღონდ ზუსტად არ ვიცი biggrin.gif
basa-ttt
QUOTE
ნუ მოჟეტ მიქსომაა

მიქსომა მართლაც საფიქრებელია, მარა მიქსომას მასეთი ღრუ უნდა ქონდეს? ახასიათებს ეგეთი რამე? თუ განაკვეთზე ჩანს მასე? spy.gif

Posted by: basa-ttt 9 Feb 2009, 19:48
QUOTE
თუ შეგიზლიათ მითხრათ, რას ნიშნავს "ძარღვის გაკვანძა". ამიხსენით სამედიცნი ენაზე რა იწვევს ამ აუტანელ ტკივილს და როგორ ქრება ესე უკვალოდ. მე მემართება ხოლმე ხIშრად დაცოლილ მდგომარეობაში და თან ერთიდაიგივე ფეხზედა ნეტა რისი ბარლი უნდა იყოს?

კუნთის დაჭიმვაა - ხშირად ახლავს თან სისხლის შედედების მომატებას
როცა ვარიკოზული პრობლემებია,
ზოგჯერ კი მინერალური ცვლის (ხშირად კალციუმის, კალიუმის, მაგნიუმის) მოშლის გამოა.
http://www.neuro.net.ru/bibliot/b004/n29.html
Причин для появления судорог несколько. В частности, перенапряжения (например, долгое стояние, непривычная деятельность и т. п. ), расстройства артериального и венозного кровоснабжения, раздражения корней нервов вследствие поражений позвонковых дисков и полинейропатий, а также, что очень вероятно, нарушение электролитного баланса (содержание калия, кальция, магния и т. д. ). Необходимо провести обследование сердечно- сосудистой системы, ортопедическое и неврологическое обследование на предмет исключения сжатия корней нервов и нарушений нервных функций, а также сдать анализы крови на электролиты. Для постановки более точного диагноза необходимо обследоваться.



QUOTE
მიქსომა მართლაც საფიქრებელია, მარა მიქსომას მასეთი ღრუ უნდა ქონდეს? ახასიათებს ეგეთი რამე? თუ განაკვეთზე ჩანს მასე?

პათ ანატომი არ ვარ - ამიტომ ვითქხოვ ხოლმე კლინიკის დადებას.
ვერ გეტყვი ზუსტად როგორ გამოიყურება მიქსომა ჭრილში -
აშკარად ზედმეტი წარმონაქმნია - და არ ჰგავს გულის კუნთს -
შეიძლება თრომბებიც კი იყოს.

Posted by: Blind_Torture_Kill 10 Feb 2009, 15:41
QUOTE
პათ ანატომი არ ვარ - ამიტომ ვითქხოვ ხოლმე კლინიკის დადებას.
ვერ გეტყვი ზუსტად როგორ გამოიყურება მიქსომა ჭრილში -
აშკარად ზედმეტი წარმონაქმნია - და არ ჰგავს გულის კუნთს -
შეიძლება თრომბებიც კი იყოს.


კრგი

27 წლის მამაკაცი. შიგადაშიგ აღნიშნავდა გულის ფრიალს მაგრამ ეკგ არ გადაუღია (საჭიროდ არ ჩათვალა გადაღლას დააბრალა ვთქვათ). სხვა მხრივ არანაირი ჩივილები არ ჰქონია. ერთ დღესაც გულის ფრიალის მონაკვეთი გაურთულდა სუნთქვის უკმარისობით და საავადმყოფოში მიყვანამდე გარდაიცვალა.
არ ეწეოდა, იშვიათად სვავდა, არ აწუხებდა წნევები და დიაბეტი, აქტიური ცხოვრებით ცხოვრობდა-იყო ერთი სიტყვით სრულიად ჯანმრთელი.

აქედან თუ დასვავ დიაგნოზს smile.gif საღოლ

ლუკა გულის განივი განაკვეთია (ორად რომ გაჭრა გული - ზედა მხარე თავისი წინაგულებით და ცოტა პარკუჭებით და ქვედა ნაწილი პარკუჭებით)
რაც შენ მოხაზე მაგისთვის ყურადღებაც არ მიმიქცევია smile.gif ეხლა შევამჩნიე ეგ მარჯვენა პარკუჭის ნაწილია და რამე პათოგნომურ ნიშანს არ წარმოადგენს smile.gif



Posted by: LUKA-BRAZI 10 Feb 2009, 16:22
Blind_Torture_Kill
QUOTE
27 წლის მამაკაცი. შიგადაშიგ აღნიშნავდა გულის ფრიალს მაგრამ ეკგ არ გადაუღია (საჭიროდ არ ჩათვალა გადაღლას დააბრალა ვთქვათ). სხვა მხრივ არანაირი ჩივილები არ ჰქონია. ერთ დღესაც გულის ფრიალის მონაკვეთი გაურთულდა სუნთქვის უკმარისობით და საავადმყოფოში მიყვანამდე გარდაიცვალა. არ ეწეოდა, იშვიათად სვავდა, არ აწუხებდა წნევები და დიაბეტი, აქტიური ცხოვრებით ცხოვრობდა-იყო ერთი სიტყვით სრულიად ჯანმრთელი.

აქედან თუ დასვავ დიაგნოზს smile.gif საღოლ

ეგ კლინიკა ფილტვის არტერიის თრომბოემბოლიას ახასიათებს. მაგ კაცს ქონდა მოციმციმე არითმია, ამიტომაც აღნიშნავდა გულის ფრიალს. მოციმციმე არითმიის შედეგად განვითარდა თრომბი წინაგულში, იქიდან კი პატარა ნაწილი მოწყდა შემდეგ და მოხვდა ფილტვის არტერიაში. მარა პრეპარატზე რომ მარცხენა პარკუჭია განსაკუთრებული? იქნებ გული სისტოლის მომენტში გაჩერდა? გულის ფრიალის გართულება ჰაერის უკმარისობით შეიძლება ინფარქტიც იყოს, მაგრამ რახან ტკივილის სინდრომს არ ახსენებ, ეს ვარიანტი გამოვრიცხოთ?

Posted by: Blind_Torture_Kill 10 Feb 2009, 16:36
LUKA-BRAZI

მოციმციმე არითმიის დროს განვითარებული თრომბი დიდ წირკულაციაში წავა
ხოლო ფილტვის არტერიის ემბოლიზაცია შეიძლება მოხდეს ღრმა ვენებში წარმოქმნილი თრომბისგან (ქვედა კიდურები ან მენჯი)

ასე რომ ყოფილიყო ფილტვის სურათს დავდებდი ან ეკგ-ს

smile.gif

ამ შემთხვევაში კლინიკა არაფერს არ გაძლევს smile.gif
ბასას მოთხოვნის გამო დავწერე

ეხლა გავედი და რამე უფრო სერიოზულ ქეისს მოვიფიქრებ smile.gif

Posted by: LUKA-BRAZI 10 Feb 2009, 16:41
Blind_Torture_Kill
QUOTE
მოციმციმე არითმიის დროს განვითარებული თრომბი დიდ წირკულაციაში წავა

spy.gif რეიზა? მარჯვენა წინაგულიდან ფილტვში არ წავა თრომბი?

Posted by: Blind_Torture_Kill 10 Feb 2009, 16:53
QUOTE
რეიზა? მარჯვენა წინაგულიდან ფილტვში არ წავა თრომბი


კი
მარა მარჯვენაში განვითარებული თრომბი არ გამიგია

Posted by: basa-ttt 10 Feb 2009, 18:50
წამიშალა დაწერილი -
cry.gif
კლინიკურად გვაქვს
ახალგაზრდა მამაკაცში უეცარი სიკვდილის მიზეზი
რასაც ახლავდა ფრიალი -
ანუ არითმია +-ფირბილაცია

ამის მიზეზები მრავალია
http://med-lib.ru/speclit/card/71.php
აქ ჩამოთვლილთგან ყველაზე ახლოს მეჩვენება გულის სიმსივნე -
ვინაიდან ფოტოზე პარკუჭის ღრუს მოცულობა აშკარად შემცირებულია
და ამის მიზეზი ან ჰიპერტროფიული კარდიომიოპათია უნდა იყოს -
ან სიმსივნე.
ორივეს შეუძლია გამოიწვიოს ფიბრილაცია და სიკვდილი.


Posted by: vano_t 10 Feb 2009, 20:42
ჰიპერტროფიული კარდიომიოპათიაა: მარცხენა პარკუჭია ჰიპერტროფირებული და ძგიდეც ჩათრეულია (სხვანაირად კიდევ ასიმეტრიული სეპტალური ჰიპერტროფიაც ქვია ამ დაავდებას, ან იდიოპათური ჰიპერტროფული სუბაორტული სტენოზი). ეს დაავადება არის sudden cardiac death-ის ყველაზე ხშირი მიზეზი. სიკვდილის მიზეზი ამ დაავადების შემთხვევაში არის ვენტრიკულური ტაქიკარდია/ფიბრილაციები.

Posted by: LUKA-BRAZI 10 Feb 2009, 22:08
vano_t
ვანოს გაუმარჯოს, როგორ ხარ ? smile.gif ჰიპერტროფიული კარდიომიოპათია მეც ვთქვი მაგრამ ბ-ნ დავითმა ცივი წყალი გადაგვასხა biggrin.gif გულის ფრიალს რაც შეეხება, ბოლოს რაც დაემართა ის ფიბრილაცია იყო ალბათ, მაგრამ ფიბრილაცია როგორც ჩივილი არ გამიგია (პარკუჭოვანი, თორემ წინაგულოვანი კი ბატონო biggrin.gif )

Posted by: Blind_Torture_Kill 10 Feb 2009, 22:28
QUOTE
ჰიპერტროფიული კარდიომიოპათიაა: მარცხენა პარკუჭია ჰიპერტროფირებული და ძგიდეც ჩათრეულია (სხვანაირად კიდევ ასიმეტრიული სეპტალური ჰიპერტროფიაც ქვია ამ დაავდებას, ან იდიოპათური ჰიპერტროფული სუბაორტული სტენოზი). ეს დაავადება არის sudden cardiac death-ის ყველაზე ხშირი მიზეზი. სიკვდილის მიზეზი ამ დაავადების შემთხვევაში არის ვენტრიკულური ტაქიკარდია/ფიბრილაციები.


კი უეცარი სიკვდილი ახასიათებს მაგასაც მარა ამ შემთხვევაში ეგ არაა
განაკვეთია ცუდ ადგილას გაკეთებული

დვდო ჰისტოლოგიური პრეპარატიც ?

Posted by: basa-ttt 10 Feb 2009, 22:54
QUOTE
ჰიპერტროფიული კარდიომიოპათიაა

არაო....
გვითრეს უკვე.
აშკარად პარკუჭის ღრუ შემცირებულია -
ამიტომ ვიფიქრე სიმსივნეზე.

Blind_Torture_Kill
დაამატე რამე -

Posted by: LUKA-BRAZI 11 Feb 2009, 00:14
Blind_Torture_Kill
ლიპოიდური დისტროფიაა მიკროპრეპარატზ? baby.gif ანუ გულის გაცხიმოვნება?

Posted by: Blind_Torture_Kill 11 Feb 2009, 02:12
QUOTE
ლიპოიდური დისტროფიაა მიკროპრეპარატზ?ანუ გულის გაცხიმოვნება


რაღაც ეგეთი ჭირს მარა გენეტიკურად განსაზღვრული (ARVC)

ამ გულში მიოკარდი არის ჩანაცვლებული ცხიმოვანი ქსოვილით (თუ დააკვირდები მარჯვენა პარკუჭს მთელი კედელი ცხიმითაა შემდგარი) რაც ხშირად არითმიის მიზეზი ხდება, შეკუმშვის უნარის დაქვეითება გულის უკმარისობას იწვევს და სიმპტომებიც ამის შესაბამისია


აბა ამას რა ჭირს ?

Posted by: basa-ttt 11 Feb 2009, 02:29
QUOTE
ამ გულში მიოკარდი არის ჩანაცვლებული ცხიმოვანი ქსოვილით (თუ დააკვირდები მარჯვენა პარკუჭს მთელი კედელი ცხიმითაა შემდგარი) რაც ხშირად არითმიის მიზეზი ხდება, შეკუმშვის უნარის დაქვეითება გულის უკმარისობას იწვევს და სიმპტომებიც ამის შესაბამისია

ამას ჰქვია არითმოგენული დისპლაზია
დავწერე -
მაგრამ მერე წავშალე
გამაბრაზეს...
mad.gif

Posted by: Blind_Torture_Kill 11 Feb 2009, 02:32
basa-ttt

ვინ გაგაბრაზა
ქართულად თუ ეგრე ქვია ეგაა მაშინ
smile.gif

Posted by: basa-ttt 11 Feb 2009, 02:37
Blind_Torture_Kill
აღარ მკითხო მაგი...
lol.gif

მაგიც ვიფიქრე მანამდე არითმოგენული დისპლაზია ხომ არ არის თქო -
მაგრამ მომეჩვენა, რომ ჰიპერტროფია მარცხენა პარკუჭში იყო -
ეგ კი მარჯვენა პარკუჭში ხდება ხოლმე.
თან შენც და ვანომაც მარცხენა პარკუჭი ახსენეთ და რჩებოდა სიმივნე დიფ დიაგნოზში

Posted by: Blind_Torture_Kill 11 Feb 2009, 02:44
QUOTE
თან შენც და ვანომაც მარცხენა პარკუჭი ახსენეთ და რჩებოდა სიმივნე დიფ დიაგნოზში


მე მარცხენა პარკუჭი არ მიხსენებია გითხარით უბრალოდ ჰიპერტროფია არ არისთქო smile.gif

Posted by: Cousteau 11 Feb 2009, 15:51
user posted image

ნეხუია სებე eek.gif ეს რაარი? spy.gif

ვიტყოდი ნაზოგასტრალურს უდგავდნენ და კრუბრიფორმი გახვრიტეს თქო მარა გარეთ აქვს გამოსული... რა არის საერთოდ?
სიფათი მიიკერა? gigi.gif კათეტერია ჩარჩენილი? აზრზე არ ვარ

Posted by: Blind_Torture_Kill 11 Feb 2009, 22:26
QUOTE
იტყოდი ნაზოგასტრალურს უდგავდნენ


ქალა ფუძის მოტეხილობა ქონდა ხოდა რო შეუყვანეს ნაზოგასტრული ზონდი ავიდა თავში smile.gif

Posted by: Cousteau 11 Feb 2009, 23:18
QUOTE (Blind_Torture_Kill @ 11 Feb 2009, 22:26 )
QUOTE
იტყოდი ნაზოგასტრალურს უდგავდნენ


ქალა ფუძის მოტეხილობა ქონდა ხოდა რო შეუყვანეს ნაზოგასტრული ზონდი ავიდა თავში smile.gif

არ შეიძლება ჩადგმა მაგ დროს (ყოველშემთხვევაში ''მასაჩუსეტში'' ეგრე წერია)

ქართული შემთხვევაა?

Posted by: Blind_Torture_Kill 12 Feb 2009, 00:01
QUOTE
არ შეიძლება ჩადგმა მაგ დროს (ყოველშემთხვევაში ''მასაჩუსეტში'' ეგრე წერია)

ქართული შემთხვევაა?


არა
წიგნიდან ამოვიღე smile.gif


Posted by: basa-ttt 12 Feb 2009, 13:10
QUOTE
ქალა ფუძის მოტეხილობა ქონდა ხოდა რო შეუყვანეს ნაზოგასტრული ზონდი ავიდა თავში

და თვალბუდეები ეგეთი უცნაური ფორმის რათაა?

Posted by: the_lizard_king 12 Feb 2009, 13:27
Blind_Torture_Kill
მაგარი ავატარი გაქვს yes.gif

Posted by: Cousteau 12 Feb 2009, 22:31
მგონი ადვილია:

55 წლის ქალი , წარსულში დიაგნოსტირებული შიზოფრენიფორმული დაავადებით (schizophreniform disorder), მოდის თქვენთან შემდეგი ჩივილებით: მხედველობის გაუარესება, თავის ტკივილი, ჭამის დროს ტკივილი (ძირითადად ყბებში), ძლიერი დისკომფორტი, ტკივილი და ''წვა'' თავის კანზე როცა ივარცხნის თმებს.

გასინჯვით: ყველაფერი ნორმ გარდა:
t-39.3
T/A 150/90
სისხლის ანალიზში მკვეთდად მომატებული ე.დ.ს (ESR) ვთქვათ 35 (mm/hr)

კიდევ რაღაც აქვს პალპაციით ისეთი რომელიც ''ძალიან ნამიოკია'' ისერო მაგას არ დავწერ.

and the diagnosis is...

ხო და ბარემ ცოტა რო გაწვალდეთ და დაგუგლოთ (თუ რათქმაუნდა არ გახსოვთ) რა მკურნალობას და რა 1 დიაგნოსტიკურ ტესტს დაუნიშნავთ ამ პაციენტს

Posted by: Thandrus 12 Feb 2009, 23:37
Cousteau

Temporal Arteritis? wink.gif

Posted by: basa-ttt 12 Feb 2009, 23:57
QUOTE
ჭამის დროს ტკივილი (ძირითადად ყბებში

ეს ტეტანუსმა იცის.

QUOTE
მხედველობის გაუარესება, თავის ტკივილი, ჭამის დროს ტკივილი (ძირითადად ყბებში), ძლიერი დისკომფორტი, ტკივილი და ''წვა'' თავის კანზე როცა ივარცხნის თმებს.
t-39.3
T/A 150/90
სისხლის ანალიზში მკვეთდად მომატებული ე.დ.ს (ESR) ვთქვათ 35 (mm/hr)

ეს კი მენინგიტმა

Posted by: Cousteau 13 Feb 2009, 00:00
QUOTE (basa-ttt @ 12 Feb 2009, 23:57 )
QUOTE
ჭამის დროს ტკივილი (ძირითადად ყბებში

ეს ტეტანუსმა იცის.


ტეტანუსმა ''იცის'' ტრიზმი (ანუ ყბების ძლიერი შეკუმშვა)

no.gif
არც ტეტანუსი
არც მენინგიტი

QUOTE
Temporal Arteritis?

ეს ვერ დავინახე smile.gif
yes.gif temporal arteritis (ტემპორალური არტერიტი) a.k.a. Giant - Cell Arteritis

პალპაციით: გადიდებული და შეხებაზე მტკივნეული არტერიები თავის ქალაზე

QUOTE
ხო და ბარემ ცოტა რო გაწვალდეთ და დაგუგლოთ (თუ რათქმაუნდა არ გახსოვთ) რა მკურნალობას და რა 1 დიაგნოსტიკურ ტესტს დაუნიშნავთ ამ პაციენტს

Posted by: basa-ttt 13 Feb 2009, 00:10
QUOTE
ტეტანუსმა ''იცის'' ტრიზმი (ანუ ყბების ძლიერი შეკუმშვა)

ტკივილიც -
განსაკუთრებით თავიდან,როცა ტრიზმი ჯერ არაა მკვეთრად გამოხატული

Thandrus
უკვე გასცა პასუხი -
წინა პოსტში

Posted by: vano_t 13 Feb 2009, 05:18
Cousteau
QUOTE
სისხლის ანალიზში მკვეთდად მომატებული ე.დ.ს (ESR) ვთქვათ 35 (mm/hr)
ერთ მეგობრულ შენიშვნას დავდებ, იმედია არ გეწყინება კუსტო.

რა თქმა უნდა, ტემპორალური არტერიტის დროს ედს მკვეთრად მომატებულია პაციენტების უმრავლესობაში (თუმცა მცირე ნაწილს შეიძლება აღენიშნებოდეს მცირე მომატება, ან საერთოდ ნორმა ქონდეს). ოღონდ, 55 წლის ქალში 35 მმ/სთ-ში შეიძლება ნორმად ჩაითვალოს. ნორმალური ედს იზრდება ასაკის მიხედვით და ქალებში კიდევ ნორმის ზემო ზღვარი 5-ით მაღლაა. ზოგადად შეგიძლიო ასეთი ფორმულა გამოიყენო: (ედს-ის ნორმის ზედა ზღვარი)=(ასაკი)/2 (+5 თუ ქალია პაციენტი). ჩვენს შემთხვევაში 55 წლის ქალისათვის ნორმალური ედს-ის ზედა ზღვარი იქნება 55/2+5=27.5+5=32.5. ისე, ტემპორალური არტერიტის დროს ედს ძაან მაღალია. მე რამოდენიმეჯერ მაქვს ეგ ნანახი და ედს სულ 100-ზე მეტი მახსოვს.

LUKA-BRAZI
QUOTE
ვანოს გაუმარჯოს, როგორ ხარ ?
ლუკასაც გაუმარჯოს. თავად როგორ გიკითხო? მე კარგად ვარ ისე smile.gif

Posted by: Cousteau 13 Feb 2009, 09:37
QUOTE (vano_t @ 13 Feb 2009, 05:18 )


QUOTE
იმედია არ გეწყინება კუსტო.

: ) კაი რა biggrin.gif

QUOTE
(ედს-ის ნორმის ზედა ზღვარი)=(ასაკი)/2 (+5 თუ ქალია პაციენტი)

yes.gif ვნახე ეგ ვიკიზე, არ ვიცოდი

Posted by: Blind_Torture_Kill 13 Feb 2009, 12:00
კარგია
გააქტიურდით ცოტა თორე სულ ღაფას ეს თემა smile.gif
ბევრი და ბევრზე ბევრი ქეისები

QUOTE
და თვალბუდეები ეგეთი უცნაური ფორმის რათაა?


სად ჩანს თვალბუდეები

Posted by: Blind_Torture_Kill 14 Feb 2009, 22:20
აბა ეს რა არის ?
............................................................................................

და სავარაუდოდ რისგან მოკვდა პაციენტი


სავარაუდოდ

Posted by: donvaso 14 Feb 2009, 22:30
Blind_Torture_Kill
ფილტვის სიმსივნე(არ გავს მარა მაინც იყოს სხვამ რომ არ თქვას... biggrin.gif )???

Posted by: Blind_Torture_Kill 14 Feb 2009, 22:36
donvaso

არა ეგ არაა

რა ხდება რადიოლოგიაში დავალაგო რენტგენოგრამები ?

Posted by: donvaso 14 Feb 2009, 22:38
Blind_Torture_Kill
დაალაგე, ოღონდ ჯერ მარტივები რა..... biggrin.gif biggrin.gif biggrin.gif
ხო, პაციენტი სუნთქვის უკმარისობისგან გარდაიცვლებოდა სავარაუდოდ....

Posted by: Blind_Torture_Kill 14 Feb 2009, 22:43
donvaso

biggrin.gif

კაი ეგრე ვიზავ

QUOTE
ხო, პაციენტი სუნთქვის უკმარისობისგან გარდაიცვლებოდა სავარაუდოდ....

ეგ სიკვდილის მიზეზად ვერ ჩაიწერება smile.gif

Posted by: Cousteau 14 Feb 2009, 23:17
hmm ემფიზემაა?
შეიძლება სისულელეს ვამბობ, ისე 'დაგატკით' ვიძახი

Posted by: basa-ttt 14 Feb 2009, 23:24
Blind_Torture_Kill
რა ქსოვილიც არ უნდა იყოს - ისეთი ფერი აქვს -
ნეკროზი უნდა იყოს -
თითქოს სამკუთხა ფორმისაა -
ფ.ა. თრომბოემბოლია?

Posted by: LUKA-BRAZI 14 Feb 2009, 23:30
შემთხვევით თიმუსი ხომ არაა დამნაშავე პაციენტის სიკვდილში?

Posted by: Blind_Torture_Kill 14 Feb 2009, 23:42
LUKA-BRAZI

არა ლუკა

კოსტამ გამოიცნო რაც არის smile.gif გაარტყა
აბა როგორი ემფიზემაა ეგ მითხარით

Posted by: Cousteau 14 Feb 2009, 23:55
QUOTE (Blind_Torture_Kill @ 14 Feb 2009, 23:42 )
გაარტყა

როგორ გეკადრებათ smile.gif
ლოღიკურად


პანაცინალურია ალბათ, და რაღან ფილტვის კიბო არ ჭირს მაშინ ალბათ MI-სგან მოკვდა ან რამე ისეთისგან რაც სიგარეტთან არის დაგავშირებული (რომელიმე სიმსივნე, COPD etc)


QUOTE
კოსტამ

როგორ გეკადრებათ 2
კუსტომ yes.gif
Jacques-Yves Cousteau a.k.a Captain Cousteau - French naval officer, explorer, ecologist, filmmaker, innovator, scientist, photographer, author and researcher who studied the sea and all forms of life in water. He co-developed the aqua-lung, pioneered marine conservation and was a member of the Académie française

user posted image

Posted by: vano_t 15 Feb 2009, 00:49
ბულოზური ემფიზემა-ბულებია (ბუშტუკები) ფილტვის ზედაპირზე.

სიკვდილის მიზეზი შეიძლება პნევმოთორაქსი ყოფილიყო (ბულა გასკდა)-სიკვდილის მიზეზი ნებისმიერი რამ შეიძლება ყოფილიყო, მარა ბულოზური ემფიზემის დროს სპონტანური პეუმოთორაქსი შეიძლება ჩამოყალიბდეს.

Posted by: Cousteau 15 Feb 2009, 10:26
QUOTE (vano_t @ 15 Feb 2009, 00:49 )
ბულოზური ემფიზემა-ბულებია (ბუშტუკები) ფილტვის ზედაპირზე.

სიკვდილის მიზეზი შეიძლება პნევმოთორაქსი ყოფილიყო (ბულა გასკდა)-სიკვდილის მიზეზი ნებისმიერი რამ შეიძლება ყოფილიყო, მარა ბულოზური ემფიზემის დროს სპონტანური პეუმოთორაქსი შეიძლება ჩამოყალიბდეს.

eek.gif

user.gif პირველად მესმის უნამუსო ვიყო

Posted by: Blind_Torture_Kill 15 Feb 2009, 10:50
Cousteau

იზვინი smile.gif

vano_t
QUOTE
ბულოზური ემფიზემა-ბულებია (ბუშტუკები) ფილტვის ზედაპირზე.

სიკვდილის მიზეზი შეიძლება პნევმოთორაქსი ყოფილიყო (ბულა გასკდა)-სიკვდილის მიზეზი ნებისმიერი რამ შეიძლება ყოფილიყო, მარა ბულოზური ემფიზემის დროს სპონტანური პეუმოთორაქსი შეიძლება ჩამოყალიბდეს.


ეგაა

ამ 2 თვის ბავშვს განუვითარდა ჰიპოვოლემიური შოკი და გარდაიცვალა.

რა ჭირდა ? სავარაუდოდ
* * *
http://imageshack.us
http://g.imageshack.us/img145/89145268oe7.jpg/1/

Posted by: vano_t 15 Feb 2009, 11:49
პირველი სურათით ვერ მივხვდებოდი (ალბათ მაგ სურათზე ჰიპერპლაზიური თირკმელზედა ჯირკვალია), მაგრამ ქვედა სურათის (ambiguous genital-ალბათ მეგაკლიტორია, ვირილიზაცია) და კლინიკის მიხედვით (მარილების დაკარგვა/ღებინება/დეჰიდრატაცია) უნდა იყოს congenital adrenal hyperplasia (ყველაზე ხშირი ფორმა 21-ჰიდროქსილაზის ძლიერი დეფიციტი).

Posted by: Blind_Torture_Kill 15 Feb 2009, 12:23
vano_t
QUOTE
პირველი სურათით ვერ მივხვდებოდი (ალბათ მაგ სურათზე ჰიპერპლაზიური თირკმელზედა ჯირკვალია), მაგრამ ქვედა სურათის (ambiguous genital-ალბათ მეგაკლიტორია, ვირილიზაცია) და კლინიკის მიხედვით (მარილების დაკარგვა/ღებინება/დეჰიდრატაცია) უნდა იყოს congenital adrenal hyperplasia (ყველაზე ხშირი ფორმა 21-ჰიდროქსილაზის ძლიერი დეფიციტი).


yes.gif


Posted by: Cousteau 16 Feb 2009, 00:41
user posted image


20 წლის ბიჭი

Posted by: mtvareuli 16 Feb 2009, 00:57
Cousteau

რა ორგანოა biggrin.gif




Posted by: Cousteau 16 Feb 2009, 01:12
QUOTE (mtvareuli @ 16 Feb 2009, 00:57 )
Cousteau

რა ორგანოა biggrin.gif

ისეთ ნაგავ ხასიათზე ვარ რო სიგმოიდური ნაწლავის (თუ როგორ ქვია ქართულად sigmoid colon-ს ) გარდა რას დავდებდი

Posted by: mtvareuli 16 Feb 2009, 01:15
Cousteau

ოჯახური ადენომატოზური პოლიპოზი

Posted by: Cousteau 16 Feb 2009, 01:18
QUOTE (mtvareuli @ 16 Feb 2009, 01:15 )
Cousteau

ოჯახური ადენომატოზური პოლიპოზი

smile.gif

yes


Goodnight

Posted by: LUKA-BRAZI 16 Feb 2009, 15:28
Cousteau
QUOTE
ისეთ ნაგავ ხასიათზე ვარ რო სიგმოიდური ნაწლავის (თუ როგორ ქვია ქართულად sigmoid colon-ს ) გარდა რას დავდებდი

რატომ? ისევ ვერ დალაგდა შენი საქმეები? არ ხარ კმაყოფილი ორთოპედიით? user.gif

Posted by: Cousteau 16 Feb 2009, 16:34
QUOTE (LUKA-BRAZI @ 16 Feb 2009, 15:28 )
Cousteau
QUOTE
ისეთ ნაგავ ხასიათზე ვარ რო სიგმოიდური ნაწლავის (თუ როგორ ქვია ქართულად sigmoid colon-ს ) გარდა რას დავდებდი

რატომ? ისევ ვერ დალაგდა შენი საქმეები? არ ხარ კმაყოფილი ორთოპედიით? user.gif

არა კაცო მაგიტომ არა

არა საქმეებიც არ დალაგებულა მარა მაინც : )

Posted by: Blind_Torture_Kill 17 Feb 2009, 00:00
აბა რისგან მოკვდა ეს პიროვნება ?
.................................................................

Posted by: Thandrus 17 Feb 2009, 00:42
Blind_Torture_Kill

ეს რა არის, ის მაინც გვითხარი... ჰისტოლოგია ქსეროქსებიდან მაქვს ნასწავლი sad.gif biggrin.gif

Posted by: donvaso 17 Feb 2009, 00:47
Thandrus
მე უცებ ზღვის სურათი მეგონა...... biggrin.gif biggrin.gif

Posted by: vano_t 17 Feb 2009, 02:29
Blind_Torture_Kill
QUOTE
აბა რისგან მოკვდა ეს პიროვნება ?
.................................................................

ჩემი მწირი ფანტაზიით, ეგ უნდა იყოს aortic dissection-ით მკვდარი. (მე მგონი სურათზე აორტის შუა გარსის ჭრილია Cystic medial necrosis-ით).

donvaso
QUOTE
მე უცებ ზღვის სურათი მეგონა......
biggrin.gif თავიდან მეც ზღვა მეგონა და ვიფიქრე დაიხრჩო თქო ავადმყოფი.

Posted by: Blind_Torture_Kill 17 Feb 2009, 10:54
მუღამი იმას აქვს შენით რომ გამოიცნობ ჰისტოლოგიურ პრეპარატსაც და პათოლოგიასაც smile.gif



QUOTE
ჩემი მწირი ფანტაზიით, ეგ უნდა იყოს aortic dissection-ით მკვდარი. (მე მგონი სურათზე აორტის შუა გარსის ჭრილია Cystic medial necrosis-ით).


არა ეგ არაა

Posted by: Blind_Torture_Kill 18 Feb 2009, 23:03
რაღაც მოიკოჭლებთ მოზგებო
გინდათ პასუხი თუ კიდევ იდოს პაწა ხანი ?

Posted by: Blind_Torture_Kill 20 Feb 2009, 17:57
გულის ამილოიდოზია სლაიდზე (congo red stain)

Posted by: Thandrus 20 Feb 2009, 18:12
Blind_Torture_Kill

... კი გამიელვა ერთხელ მაგ აზრმა... მაგრამ რაღაც ვერ გავბედე მეთქვა biggrin.gif

Posted by: Blind_Torture_Kill 20 Feb 2009, 20:02
Thandrus

უნდა გაბედო მერე

Posted by: mtvareuli 20 Feb 2009, 22:51
რა ქვია და რაზე შეიძლება მიანიშნებდეს
user posted image

Posted by: Blind_Torture_Kill 20 Feb 2009, 23:24
მარტო მაგ თითზე აქვს თუ სხვებზეც ?
* * *
თუ სხვებზეც აქვს მაშინ ბაქტერიული ენდოკარდიტი (splinter hemorrhage)
თუ მარტო მაგაზე მაშინ მელანომა ან ინტენსიური პიგმენტაცია

Posted by: Thandrus 22 Feb 2009, 23:26
რამე ახალი, გეთაყვა...

თორემ გადავიდა ეს თემა B-კლასში...

Posted by: LUKA-BRAZI 23 Feb 2009, 18:27
Blind_Torture_Kill
QUOTE
ექიმებო დაგენძრათ მოვდივარ !!!

biggrin.gif
ვაიმეე რეიზაა? /ნანული, ტოისწ მ. ანდღულაძე/
და რაღა ექიმებო? ბარემ დამნაშავეებს დაემუქრე, რომლებმაც შენიღბეს თავიანთი დანაშაული smile.gif

Posted by: Blind_Torture_Kill 23 Feb 2009, 19:00
60 წლის მამაკაცს აწუხებს წელის ტკივილი წინ გადახრისას. რენტგენოგრამამ აჩვენა L2-L3 მალების კომპრესიულ მოტეხილობა. ლაბ: სისხლში-ნორმოციტული ანემია, ჰიპერკალცემია, შარდში ცილა

რა შეიძლება იყოს ?
* * *
LUKA-BRAZI
QUOTE
ვაიმეე რეიზაა? /ნანული, ტოისწ მ. ანდღულაძე/
და რაღა ექიმებო? ბარემ დამნაშავეებს დაემუქრე, რომლებმაც შენიღბეს თავიანთი დანაშაული


lol.gif lol.gif
ექიმების იმ ნაწილს ეხება ვინც ძაან სლეა მედიცინაში
ეგეთი ექიმების დედის ალერსს მოვ...ნავ მომავალში

Posted by: mtvareuli 23 Feb 2009, 19:25
Blind_Torture_Kill

კომპრესიული მოტეხილობაზე და ჰიპერკალცემიაზე ჰიპერპარათიროიდიზმი ვიფიქრე თავიდან

მარა რო დავფიქრდი ანემია და შარდში ცილა რა შუაში იყო, მერე ვიფიქრე მრავლობითი მიელომა yes.gif



პ.ს. ეგეთი სიტყვები არ გინდა

Posted by: Blind_Torture_Kill 23 Feb 2009, 19:35
mtvareuli
QUOTE
პ.ს. ეგეთი სიტყვები არ გინდა


აღარ ვიზავ smile.gif

QUOTE
მალების კომპრესიული მოტეხილობა და კჰიპერკალცემია ჰიპერპარათიროიდიზმისთვისაა დამახასიათებელი


კი არის მარა ეგ არაა ეხლა

Posted by: mtvareuli 23 Feb 2009, 19:36
Blind_Torture_Kill
QUOTE
კი არის მარა ეგ არაა ეხლა

ხო, ჩავასწორე

Posted by: Blind_Torture_Kill 23 Feb 2009, 19:39
mtvareuli

კაი გოგო ხარ
ეგაა

ჰისტოპათოლოგიური სლაიდები რატო არ გიყვართ ?

Posted by: mtvareuli 23 Feb 2009, 19:41
Blind_Torture_Kill
QUOTE
კაი გოგო ხარ ეგაა

jump.gif

QUOTE
ჰისტოპათოლოგიური სლაიდები რატო არ გიყვართ ?

გვიყვარს, დადე yes.gif

Posted by: Blind_Torture_Kill 23 Feb 2009, 20:36
34 წლის მამაკაცი კრუნჩხვებით, ტახიპნეით, ღებინებით მოიყვანა სასწრაფომ. წნევა 90/40 ზე. საავადმყოფოში პიროვნებას განუვითარდა სტუპორი. მიუხედავად მთელი რიგი მანიპულაციებისა პაციენტი გარდაიცვალა.
გაკვეთისას ნახული იქნა დილატირებული გული.



თირკმლის პრეპარატი მოცემულია ქვემოთ

რა არის და რისგან მოკვდა ეს პიროვნება ?

Posted by: LUKA-BRAZI 23 Feb 2009, 21:21
Blind_Torture_Kill
QUOTE
ჰისტოპათოლოგიური სლაიდები რატო არ გიყვართ ?

ვისაც ჰისტოლოგია და პათანატომია ქსეროქსებიდან გვაქვს ნასწავლი არ გვიყვარს biggrin.gif თირკმელებში რაღაც დეპოზიტებია, რაც ფლუოროსცენციით ანათებს და რა დეპოზიტებია? გვითხარი და გეტყვით დიაგნოზს biggrin.gif

Posted by: Blind_Torture_Kill 23 Feb 2009, 21:43
LUKA-BRAZI

QUOTE
ვისაც ჰისტოლოგია და პათანატომია ქსეროქსებიდან გვაქვს ნასწავლი არ გვიყვარსთირკმელებში რაღაც დეპოზიტებია, რაც ფლუოროსცენციით ანათებს და რა დეპოზიტებია? გვითხარი და გეტყვით დიაგნოზს


ეჰჰ
biggrin.gif შენ რეალში რატო არ გიცნობ

Posted by: vano_t 23 Feb 2009, 21:56
Blind_Torture_Kill
QUOTE
60 წლის მამაკაცს აწუხებს წელის ტკივილი წინ გადახრისას. რენტგენოგრამამ აჩვენა L2-L3 მალების კომპრესიულ მოტეხილობა. ლაბ: სისხლში-ნორმოციტული ანემია, ჰიპერკალცემია, შარდში ცილა

რა შეიძლება იყოს ?
ეგ არის არასპეციფიური წარმოდგენა და არ მიუთითებს მარტო მრავლობით მიელომაზე. თუ გინდა სინამდვილეში იცოდე როგორ ხდება ასეთი ავადმყოფის გამოკვლევა, მაშინ ასეთ შემთხვევაში დიაგნოზში შედის არამარტო მრავლობითი მიელომა, არამედ ნებისმიერი სიმსივნე რომელიც ძვალში მეტასტაზებს იძლევა! პროსტატის კიბო ასევე ერთ-ერთი პირველია. როცა გაქვს პათოლოგიური მოტეხილობა (მიუხედავად იმისა კალციუმი მაღალია თუ არა), უნდა იფიქრო როგორც მრავლობით მიელომაზე, ასევე სიმსივნეებზე, რომლებიც მეტასტაზებს იძლევიან ძვალში (ან პირველად ძვლის სიმსინეებზე, რაც უფრო იშვიათია).

შარდში ცილა ძალიან არასპეციფიურია. რომ გეთქვა UPEP (urine protein electophoresis) ან SPEP (serum protein electophoresis) მონოკლონურ პიკებს გაძლევსო, მაშინ სხვა საქმეა.

QUOTE
34 წლის მამაკაცი კრუნჩხვებით, ტახიპნეით, ღებინებით მოიყვანა სასწრაფომ. წნევა 90/40 ზე. საავადმყოფოში პიროვნებას განუვითარდა სტუპორი. მიუხედავად მთელი რიგი მანიპულაციებისა პაციენტი გარდაიცვალა.
გაკვეთისას ნახული იქნა დილატირებული გული.



თირკმლის პრეპარატი მოცემულია ქვემოთ

რა არის და რისგან მოკვდა ეს პიროვნება ?
რაღაც მწვავე ინფექცია (ან ვირუსული ან ბაქტრიული), რომელიც გაძლევეს მიოკარდიტს (ან გულის უკმარისობას სხვანაირად) და გლომერულონეფრიტს იმუნური მექანიზმებით. ან შეიძლება ამილოიდოზი აქვს (თუ ინტრავენური მომხამერებელია) და რაღაც მწვავე ინფექციით იღუპება.

მოკლედ, შემთხვევა ისევ არასპეციფიურია. შეიძლება სლაიდი დიაგნოზის დასმის საშუალებას იძლევა, ეგ არ ვიცი. სლაიდზე ისე გლომერულებს და მილაკებს შორის ჩანს პატარა წერტილები-არა მგონია ბირთვები იყოს. შეიძლება ბაქტერიაა. თუ ბაქტერიაა, კოკებს უფრო გავს წყვილებში.

Posted by: LUKA-BRAZI 23 Feb 2009, 22:23
Blind_Torture_Kill
QUOTE
ეჰჰ biggrin.gif შენ რეალში რატო არ გიცნობ

იმ დღეს არ დაგვდე პატივი, ბ-ნ დავით, საიდუმლოდ რომ შევიკრიბეთ ღუდუშაურში მედიცინის მამასთან მე და კუსტო და ამ ქვეყნის ბედ-იღბალზე რომ ვიბჭეთ მეცნიერულად biggrin.gif ისე კი გავაწყდა თვალები ჭიშკრისკენ ყურბაში biggrin.gif სადაა დათო, სადაა დათო.... არაუშავს, მორიგი პლენარული სხდომა თუ იქნა გაგაგებინებ biggrin.gif

ვანოს ვთანხმები რომ შეიძლება ის იმუნური დეპოზიტები იყოს, მაგრამ ბაქტერიებში არ ვეთანხმები smile.gif ასე უტიფრად კოკები თირკმელში ვერ წარმომიდგენია smile.gif დილატირებული გული, ალბათ რაღაც დროის განმავლობაში ჩამოყალიბდა ჰო? უეცრად გულის დილატაცია, ალბათ უფრო მასიური ტრანსმურული ინფარქტის მერე შეიძლება განვითარდეს.....

Posted by: Thandrus 23 Feb 2009, 22:31
LUKA-BRAZI

ასე უეცრად ინფარქტიც, დეპოზიტებიც, კრუნჩხვებიც და ასე შემდეგ? boli.gif


Posted by: Blind_Torture_Kill 23 Feb 2009, 22:35
QUOTE (vano_t @ 23 Feb 2009, 21:56 )
Blind_Torture_Kill
QUOTE
60 წლის მამაკაცს აწუხებს წელის ტკივილი წინ გადახრისას. რენტგენოგრამამ აჩვენა L2-L3 მალების კომპრესიულ მოტეხილობა. ლაბ: სისხლში-ნორმოციტული ანემია, ჰიპერკალცემია, შარდში ცილა

რა შეიძლება იყოს ?
ეგ არის არასპეციფიური წარმოდგენა და არ მიუთითებს მარტო მრავლობით მიელომაზე. თუ გინდა სინამდვილეში იცოდე როგორ ხდება ასეთი ავადმყოფის გამოკვლევა, მაშინ ასეთ შემთხვევაში დიაგნოზში შედის არამარტო მრავლობითი მიელომა, არამედ ნებისმიერი სიმსივნე რომელიც ძვალში მეტასტაზებს იძლევა! პროსტატის კიბო ასევე ერთ-ერთი პირველია. როცა გაქვს პათოლოგიური მოტეხილობა (მიუხედავად იმისა კალციუმი მაღალია თუ არა), უნდა იფიქრო როგორც მრავლობით მიელომაზე, ასევე სიმსივნეებზე, რომლებიც მეტასტაზებს იძლევიან ძვალში (ან პირველად ძვლის სიმსინეებზე, რაც უფრო იშვიათია).

შარდში ცილა ძალიან არასპეციფიურია. რომ გეთქვა UPEP (urine protein electophoresis) ან SPEP (serum protein electophoresis) მონოკლონურ პიკებს გაძლევსო, მაშინ სხვა საქმეა.

QUOTE
34 წლის მამაკაცი კრუნჩხვებით, ტახიპნეით, ღებინებით მოიყვანა სასწრაფომ. წნევა 90/40 ზე. საავადმყოფოში პიროვნებას განუვითარდა სტუპორი. მიუხედავად მთელი რიგი მანიპულაციებისა პაციენტი გარდაიცვალა.
გაკვეთისას ნახული იქნა დილატირებული გული.



თირკმლის პრეპარატი მოცემულია ქვემოთ

რა არის და რისგან მოკვდა ეს პიროვნება ?
რაღაც მწვავე ინფექცია (ან ვირუსული ან ბაქტრიული), რომელიც გაძლევეს მიოკარდიტს (ან გულის უკმარისობას სხვანაირად) და გლომერულონეფრიტს იმუნური მექანიზმებით. ან შეიძლება ამილოიდოზი აქვს (თუ ინტრავენური მომხამერებელია) და რაღაც მწვავე ინფექციით იღუპება.

მოკლედ, შემთხვევა ისევ არასპეციფიურია. შეიძლება სლაიდი დიაგნოზის დასმის საშუალებას იძლევა, ეგ არ ვიცი. სლაიდზე ისე გლომერულებს და მილაკებს შორის ჩანს პატარა წერტილები-არა მგონია ბირთვები იყოს. შეიძლება ბაქტერიაა. თუ ბაქტერიაა, კოკებს უფრო გავს წყვილებში.

თუ დააკვირდი ისტორიას ძაან მწირი რაოდენობით ვიძლევი იმიტომ რომ თერაპიული ავადმყოფები მე არ მეხება
პირველ ქეისში კითხვაც ესე მქონდა დასმული-რა შეიძლება იყოს მეთქი smile.gif
ხოდა ერთ-ერთი რაზეც შეიძლებოდა ეჭვი მიეტანათ თქვეს
ასეთი პატარა ინფოთი მეც მაგას ვიფიქრებდი პირველს

მეორეზე სლაიდი ძირითადი კომპონენტია. ამას თუ გამოიცნობ მერე სიმპტომებსაც მიუმატებ სიკვდილის მიზეზამდე მიხვალ smile.gif

ინფარქტი არ მიხსენებია მე
პაციენტს შეიძლება საავადმყოფოში მიყვანამდე ქონდა რამე პრობლემა მარა ისე არ შეუწუხებია რომ ექიმთან მისულიყო.
კრუნჩხვები, ტახიპნეა, ღებინება განუვითარდა ბოლო დღეს სასწრაფოშიც მაგიტომ გააქანეს smile.gif

Posted by: LUKA-BRAZI 23 Feb 2009, 22:39
Thandrus
QUOTE
ასე უეცრად ინფარქტიც, დეპოზიტებიც, კრუნჩხვებიც და ასე შემდეგ?

არა მე სხვა გაგებით ვთქვი. კლინიკიდან ინფარქტს ვერ იფიქრებ (ნუ ღებინება შეიძლება ექვივალენტი იყოს) ამიტო მწვავედ დილატირებული გული რაას უნდა ნიშნავდეს მეთქი.... ეს ვიგულისხმე.

Posted by: Thandrus 23 Feb 2009, 23:18
რავიცი... დიაბეტური კომით მოკვდა... givi.gif

სხვა უკეთესი არაფერი მომივიდა თავში....

Posted by: vano_t 23 Feb 2009, 23:29
Blind_Torture_Kill
QUOTE
თუ დააკვირდი ისტორიას ძაან მწირი რაოდენობით ვიძლევი იმიტომ რომ თერაპიული ავადმყოფები მე არ მეხება
პირველ ქეისში კითხვაც ესე მქონდა დასმული-რა შეიძლება იყოს მეთქი smile.gif
ხოდა ერთ-ერთი რაზეც შეიძლებოდა ეჭვი მიეტანათ თქვეს
ასეთი პატარა ინფოთი მეც მაგას ვიფიქრებდი პირველს
კი, ერთ-ერთი მიზეზი შეიძლება იყოს მრავლობითი მიელომა-მე ავღნიშნე ეგ. მაგრამ, რადგანაც შენ თერაპიაში ისე ვერ ხარ და ეს თერაპიას მოიცავს, მეც გითხარი რა შეიძლება იყოს სხვა მიზეზები (არნაკლებ სავარაუდო) და რაზე უნდა იფიქრო თქო ასეთ შემთხვევაში. თუ დამიჯერებთ დამიჯერებთ, თუ არადა არ დამიჯერებთ, გაატარე ნათქვამი.

LUKA-BRAZI
QUOTE
არა მე სხვა გაგებით ვთქვი. კლინიკიდან ინფარქტს ვერ იფიქრებ (ნუ ღებინება შეიძლება ექვივალენტი იყოს) ამიტო მწვავედ დილატირებული გული რაას უნდა ნიშნავდეს მეთქი.... ეს ვიგულისხმე.
ამას მივუთითე უკვე. მწვავე მიოკარდიტი არის ერთ-ერთი მიზეზი ამისა. გარდა ამისა, მწვავე უკმარისობა სარქველებისა არის მეორე მიზეზი (მაგალითად, სოკოვანი ენდოკარდიტების დროს შეიძლება მოხდეს მიტრალური სარქვლის სერიოზული პერფორაცია და ამას მოყვეს მწვავე გულის უკმარისობა. ინფარქტი შენ უკვე აღნიშნე, მარა არაო, ეგ არ არისო.

ახალგაზრდა ადამიანში ასეთი სიმპტომებით სიკვდილი მიზეზი ბევრი რამ არ იქნება. ერთია მწვავე ვენტრიკულური არითმიები და ფიბრილაციები მიოკარდიტის გამო. მეორეა, მწვავე ფილტვების შესუპება, ჰიპოქსია და სიკვდილი (ან მიოკარდიტის ან სარქველების უკმარისობის გამო). მესამეა სიფსისი multiple oრgan failure-ით. კიდევ შეიძლება თირკმლის უკმარიოსბას სერიოზული ჰიპერკალემია მოყვეს და ამან გამოიწვიოს პარკუჭების არითმია/ფიბრილაცია. ახალგაზრდა ასაკში ინტოქსიკაციაც უნდა მიიღო მხედველობაში (ინტრავენური კაიფი, სუიციდის მცდელობა და ა.შ.)

Posted by: Thandrus 24 Feb 2009, 00:05
vano_t

ისე, ინტოქსიკაცია შესაძლებელია... მაგალითად ლითიუმი... (თუ ისევ სისულელეს ვამბობ? gigi.gif)

დილატაცია, კრუნჩხვები... რავი...

Posted by: Blind_Torture_Kill 24 Feb 2009, 00:12
QUOTE
ახალგაზრდა ადამიანში ასეთი სიმპტომებით სიკვდილი მიზეზი ბევრი რამ არ იქნება. ერთია მწვავე ვენტრიკულური არითმიები და ფიბრილაციები მიოკარდიტის გამო. მეორეა, მწვავე ფილტვების შესუპება, ჰიპოქსია და სიკვდილი (ან მიოკარდიტის ან სარქველების უკმარისობის გამო). მესამეა სიფსისი multiple oრgan failure-ით. კიდევ შეიძლება თირკმლის უკმარიოსბას სერიოზული ჰიპერკალემია მოყვეს და ამან გამოიწვიოს პარკუჭების არითმია/ფიბრილაცია. ახალგაზრდა ასაკში ინტოქსიკაციაც უნდა მიიღო მხედველობაში (ინტრავენური კაიფი, სუიციდის მცდელობა და ა.შ.)


QUOTE
ისე, ინტოქსიკაცია შესაძლებელია... მაგალითად ლითიუმი... (თუ ისევ სისულელეს ვამბობ? )

დილატაცია, კრუნჩხვები... რავი...


გოუ ონ

Posted by: donvaso 24 Feb 2009, 00:17
http://www.myvideo.ge/?video_id=400856 ეს რა არის??????

სორი ოფისთვის....... smile.gif biggrin.gif
მარა, ეს თქვენთვის რომ არ მეჩვენებინა არ შემეძლო..... biggrin.gif biggrin.gif

Posted by: Blind_Torture_Kill 24 Feb 2009, 00:31
donvaso

ძაან უმაგრესი იყო კვერნული იმენა lol.gif lol.gif lol.gif

კაი გავედი ხალხო
ზეგ პასუხი დამახვედრეთ smile.gif

Posted by: Tornike Alashvili 24 Feb 2009, 02:12
QUOTE (Blind_Torture_Kill @ 23 Feb 2009, 20:36 )
34 წლის მამაკაცი კრუნჩხვებით, ტახიპნეით, ღებინებით მოიყვანა სასწრაფომ. წნევა 90/40 ზე. საავადმყოფოში პიროვნებას განუვითარდა სტუპორი. მიუხედავად მთელი რიგი მანიპულაციებისა პაციენტი გარდაიცვალა.
გაკვეთისას ნახული იქნა დილატირებული გული.



თირკმლის პრეპარატი მოცემულია ქვემოთ

რა არის და რისგან მოკვდა ეს პიროვნება ?

რა არის პრეპარატზე გამოსახული არ ვიცი მაგრამ პაციენტი მწვავე ურემიით გარდაიცვალა (რასაც ფილტვების შეშუპებაც მოჰყვება ხშირად)და ამიტომ ან ნეფროსკლეროზი იქნება პრეპარატზე ან ინტესტიციური ნეფრიტის რაიმე ძალზედ აგრესიული ფორმა war.gif

Posted by: Thandrus 24 Feb 2009, 02:42
Tornike Alashvili

გულის სიმპტომები აიხსნება კი მაგით?

Posted by: Tornike Alashvili 25 Feb 2009, 02:25
ფილტვის შეშუპებისას მცირე წრეში ძლიერი ჰიპერტენზია ვითარდება რასაც გულის დილატირება და უკმარისობა მოყვება

Posted by: CVS 25 Feb 2009, 03:19
Tornike Alashvili
QUOTE
ფილტვის შეშუპებისას მცირე წრეში ძლიერი ჰიპერტენზია ვითარდება რასაც გულის დილატირება და უკმარისობა მოყვება

და არ შეიძლება შეშუპება, გულის უკმარისობამ გამოიწვიოს?

Posted by: basa-ttt 25 Feb 2009, 09:26
QUOTE
და არ შეიძლება შეშუპება, გულის უკმარისობამ გამოიწვიოს?

34 წლის მამაკაცში?
ყველაფერი შეიძლება მოხდეს...

Posted by: Blind_Torture_Kill 25 Feb 2009, 09:28
QUOTE
რა არის პრეპარატზე გამოსახული არ ვიცი მაგრამ პაციენტი მწვავე ურემიით გარდაიცვალა (რასაც ფილტვების შეშუპებაც მოჰყვება ხშირად)და ამიტომ ან ნეფროსკლეროზი იქნება პრეპარატზე ან ინტესტიციური ნეფრიტის რაიმე ძალზედ აგრესიული ფორმა


no.gif

QUOTE
34 წლის მამაკაცში? ყველაფერი შეიძლება მოხდეს...


რომ მოხდა მაგიტომ მოხვდა სასექციო მაგიდაზე

Posted by: basa-ttt 25 Feb 2009, 09:33
Blind_Torture_Kill
პრეპარატები აქ იციან მხოლოდ ჰისტოლოგებმა და სტუდენტებმა (რომელთაც ახლახან ისწავლეს ჰისტოლოგია gigi.gif )
-
დაამატე კიდევ რამე ამ შემთხვევას.

Posted by: Blind_Torture_Kill 25 Feb 2009, 09:47
basa-ttt

ამას რაც არ უნდა ვამატო კლინიკა თუ პრეპარატს ვერ გამოიცნობ ვერ მიხვალ სიკვდილის მიზეზამდე

დაახლოებით უნდა აღწერო რა არის პრეპარატზე გამოსახული

Posted by: basa-ttt 25 Feb 2009, 09:52
QUOTE
დაახლოებით უნდა აღწერო რა არის პრეპარატზე გამოსახული

ჩანართებია რაცხა -
ამილოიდოზი მოჟეტ...

Posted by: Blind_Torture_Kill 25 Feb 2009, 09:54
QUOTE
ჩანართებია რაცხა -ამილოიდოზი მოჟეტ...

no.gif

რა ნაწილშია ჩანართები ?

Posted by: LUKA-BRAZI 25 Feb 2009, 15:49
Blind_Torture_Kill
იმუნური ჩანართები არის დათო?

Posted by: Blind_Torture_Kill 25 Feb 2009, 17:43
LUKA-BRAZI
QUOTE
იმუნური ჩანართები არის დათო?


no.gif

Posted by: Thandrus 26 Feb 2009, 02:02
Blind_Torture_Kill

Lead poisoning...

Posted by: LUKA-BRAZI 26 Feb 2009, 22:45
Blind_Torture_Kill
QUOTE
რა ნაწილშია ჩანართები ?

ბოუმანის კაფსულაში?

ამ თემის 10 ყველაზე აქტიური იუზერი smile.gif

Posted by: vano_t 27 Feb 2009, 01:45
Tornike Alashvili
QUOTE
ფილტვის შეშუპებისას მცირე წრეში ძლიერი ჰიპერტენზია ვითარდება რასაც გულის დილატირება და უკმარისობა მოყვება

no.gif ფილტვის წრეში ჰიპერტენზია გაძლევს მხოლოდ მარჯვენა გულის ჰიპერტროფია/დილატაციას და არა მარცხენა გულის ჰიპერტროფია/დილატაციას. გულის უკმარისობაში წამყვანია მარცხენა გულის მიოკარდიუმის უკმარისობა (ან დიასტოლური დისფუნქცია, ან მიტრალური სარქველის სერიოზული უკმარისობა-მოკლედ, ნებისმიერი რამ რაც მარცხენა გულში მოგცემს დიასტოლური წნევის გაზრდას ან რეგურგიტაციას წინაგულებში და შემდეგ ფიტლვის წრეში საბოლოოდ ფილტრაციული წნევის გაზრდას ფილტვების შეშუპებას).

ფილტვის შეშუპება, რა თქმა უნდა, სხვა მექანიზმებითაც შეიძლება ჩამოყალიბდეს. მაგალითად, ფილტვების სისხლძარღვების უშუალო დაზიანებით და კაპილარების გამტარობის გაზრდით. მარა მარჯვენა გული მაინც არფერ შუაშია. თანაც, ფილტვის შეშუპება უკვე თუ გაქვს, გაქვს შეშუპების მიზეზი უკვე, და გულის უკმარისობა აღარ არის მისი მიზეზი.

Posted by: vano_t 27 Feb 2009, 02:17
Thandrus
QUOTE
Lead poisoning...

ტყვიით მოწამვლაზე გადავხედე. არ იძლევა თირკმლებში ტყვიის დაგროვებას-ეს უფრო ქრონიკულ მოწამლვაზე ეწერა. მწვავე მოწამლვის პათოლოგიაზე ვერ ვნახე.

მარა, ვერცხლისწყლის სხვადასხვა ფორმებით მწვავე მოწამლვა მოგცემს თირკმელში ვერცხლისყწლის აკუმულაციას. ამ პრეპარატზე თირკმლის მილაკებშია რაც არის.

მოკლედ, ასეც და ისეც, პრეპარატი თუ არ გამოვიცანით ზუსტად, პონტია რაიმე თქვა. ტეხავს biggrin.gif

Posted by: basa-ttt 28 Feb 2009, 19:33
QUOTE
Blind_Torture_Kill

ამოშაქრე!
- ხავსი მოეკიდა ამ თემას.
lol.gif

Posted by: Blind_Torture_Kill 2 Mar 2009, 00:51
ხო
იცი რა მომწონს აქ დისკუსიებში ჭეშმარიტება იბადება

smile.gif
ვერცხლისწყალი არაა

ამ პრეპარატზე პოლარიზებული სინათლის მოქმედებით ჩანს რაგაც ჩანართები ხოდა რეებია აბა

Posted by: vano_t 2 Mar 2009, 03:00
Blind_Torture_Kill
QUOTE
ამ პრეპარატზე პოლარიზებული სინათლის მოქმედებით ჩანს რაგაც ჩანართები ხოდა რეებია აბა

ეთილენ გლიკოლით მოწამვლაა მაშინ და კალციუმის ოქსალატის კრისტალები უნდა იყოს (მაგათ აქვთ ძაან კაშკაშა birefringence).

Posted by: Thandrus 2 Mar 2009, 03:19
vano_t

ახლა ვიფიქრე მაგაზე... მაგრამ გულს და ჰიპოტენზიას ახსნის?
* * *
მაგრამ ალბათ ეგ იქნება... სხვა სიმპტომები ჯდება...

ვიკიში ეწერა, რომ თურმე ეთილენგლიკოლის მოწამვლის მეორე სტადიას Congestive heart failure ახასიათებს... ესეიგი დილატაცია... მაგრამ ეს ჰიპოტენზია... mad.gif

Posted by: vano_t 2 Mar 2009, 03:48
QUOTE (Thandrus @ 2 Mar 2009, 03:19 )
vano_t

ახლა ვიფიქრე მაგაზე... მაგრამ გულს და ჰიპოტენზიას ახსნის?
* * *
მაგრამ ალბათ ეგ იქნება... სხვა სიმპტომები ჯდება...

ვიკიში ეწერა, რომ თურმე ეთილენგლიკოლის მოწამვლის მეორე სტადიას Congestive heart failure ახასიათებს... ესეიგი დილატაცია... მაგრამ ეს ჰიპოტენზია... mad.gif

ჰიპოტენზია ნებისმიერი მოწამვლის დროს შეიძლება გქონდეს. თანაც, როცა ღებინება გაქვს ჰიპოტენზია შეიძლება ჩამოყალიბდეს ადვილად ინტრავასკულარული სითხის დაკარგვის გამო. საერთოდ ყველაზე ხშირი მიზეზია ეს ჰიპოტენზიის. გარდა ამისა, CHF-იც მოგცემს ჰიპოტენზიას. ჰიპოტენზია არ არის ძნელი ასახსენლი მოწამვლების დროს.

Posted by: Thandrus 2 Mar 2009, 03:53
vano_t

ნუ მაშინ ესეიგი ეგაა biggrin.gif (ენამ არ მიყივლოს lol.gif )

Posted by: Blind_Torture_Kill 2 Mar 2009, 10:30
vano_t

საღოლ
კიდევ ერთხელ ვაღიარებ რომ მედიცინის მამა ხარ
up.gif

Posted by: Thandrus 2 Mar 2009, 14:43
მოკლედ ჰაუსის ბატალიებს არ ჩამოუვარდებოდა რა ეს ქეისი lol.gif მართლა ფილმს გადაიღებდა კაცი... ახლა მოდი ცოტათი განვიტვირთოთ... დავბრუნდეთ საშუალო სკოლაში... ისე ეს სურათიც გექნებათ ალბათ ბევრს ნანახი. smile.gif

რა სჭირს ამ ხალხს? (ნუ მალნუტრიციის და მაგდაგვართა გარდა)

Posted by: LUKA-BRAZI 2 Mar 2009, 16:38
Thandrus
მაგ საწყლებს კეთრი ჭირთ user.gif

და ისე კი სურათის საიდენტიფიკაციო ყველანაირი ინფო უნდა წაშალო ხოლმე, თორემ ეგრე ადვილი გამოსაცნობია smile.gif მაგალითად სურათის მარცხენა ქვედა კუთხეში მაგ სურათის fingerprint-ია smile.gif
http://images.google.com/imgres?imgurl=http://upload.wikimedia.org/wikipedia/commons/thumb/6/61/Bundesarchiv_Bild_105-DOA0199,_Deutsch-Ostafrika,_Leprakranke.jpg/300px-Bundesarchiv_Bild_105-DOA0199,_Deutsch-Ostafrika,_Leprakranke.jpg&imgrefurl=http://en.wikipedia.org/wiki/Leprosy&usg=__xmklHMllLJlwyxCZO51vuJ0CECM=&h=229&w=300&sz=21&hl=en&start=1&um=1&tbnid=csm_wF5MsV8jGM:&tbnh=89&tbnw=116&prev=/images%3Fq%3Dbundesarchiv%2Bbild%2B105-DOA%2B0199%26um%3D1%26hl%3Den%26client%3Dfirefox-a%26rls%3Dorg.mozilla:en-US:official%26sa%3DX

Posted by: Blind_Torture_Kill 2 Mar 2009, 16:46
აბა ნექსთ
ამ სურათში მე უფრო კვაშიორკორი ან მარაზმი მეგონა

Posted by: Thandrus 2 Mar 2009, 21:06
LUKA-BRAZI

უპს... აწი მივაქცევ ხოლმე ყურადღებას wink.gif

Posted by: LUKA-BRAZI 2 Mar 2009, 21:32
Blind_Torture_Kill
უი, "მეგონას" ნაცვლად "მგონია" წავიკითხე biggrin.gif პარდონ! smile.gif მარცხნიდან პირველივე მჯდომარეს, ტუ დააკვირდი ცხვირისა და პირის მიდამოში დეფექტი აქვს. ეს ან კეთრის ნიშანია ან ტროპიკული ლიშმანიოზის. ამ შემთხვევაში კეთრი აღმოჩნდა. smile.gif

Posted by: Blind_Torture_Kill 3 Mar 2009, 23:39
რა არის აბა და როდის გვხვდება ?

Posted by: LUKA-BRAZI 3 Mar 2009, 23:54
Blind_Torture_Kill
კრონის დაავადებაა?

Posted by: Blind_Torture_Kill 4 Mar 2009, 01:30
LUKA-BRAZI
კი
(string sign)

ეს რაღაა ?

Posted by: Thandrus 4 Mar 2009, 03:45
Blind_Torture_Kill

Spongiform Degeneration?


Posted by: vano_t 4 Mar 2009, 08:43
Blind_Torture_Kill
QUOTE
ეს რაღაა ?

ტვინია biggrin.gif (აი გიტარა! კოპირაით "ოსი კონსერვატორიის აბიუტურიენტი")

P.S. მარა აშკარად პათოლოგიური ტვინია. მრავლობითი კისტებია ტვინში (განსაკუთრებით თალამუსში). ან კისტებია ან მრავლობითი აბსცესები.

კლინიკა მიაყოლე ცოტა.

Posted by: Blind_Torture_Kill 4 Mar 2009, 13:28
არა აქ კლინიკა
ტვინია ლპობის პროცესში ფიქსირებული biggrin.gif

Posted by: Irakli.B 4 Mar 2009, 16:01
QUOTE (Blind_Torture_Kill @ 4 Mar 2009, 12:28 )
არა აქ კლინიკა
ტვინია ლპობის პროცესში ფიქსირებული biggrin.gif

მოიცა ეს ტვინი ცოცხალ ადამიანს ულპებოდა? თუ გაკვეთის მერე დაალპეს? baby.gif

Posted by: Blind_Torture_Kill 4 Mar 2009, 16:26
QUOTE
მოიცა ეს ტვინი ცოცხალ ადამიანს ულპებოდა? თუ გაკვეთის მერე დაალპეს?


არა გვამში

Posted by: LUKA-BRAZI 4 Mar 2009, 18:49
ეგ კი არა და Guardian სად არის? რამდენი ხანია აქ არ შემოუხედავს. ექიმობა ხომ არ გადაიფიქრა? smile.gif ჰეი, Guardian სად ხარ?

Posted by: vano_t 5 Mar 2009, 04:42
Blind_Torture_Kill
QUOTE
არა აქ კლინიკა
ტვინია ლპობის პროცესში ფიქსირებული biggrin.gif

ხოდა, ქე ვთქვი აბსცესია მეთქი biggrin.gif

Posted by: LULA_QABABI 5 Mar 2009, 08:23
QUOTE (Blind_Torture_Kill @ 4 Mar 2009, 04:28 )
არა აქ კლინიკა
ტვინია ლპობის პროცესში ფიქსირებული biggrin.gif

ეს მე სამედიცინო ფორუმი მეგონა, ნეკროცერებროფილიის ყოფილა;
ისა და, ავამდყოფის სიმპტომები რა იყო, ცივი კიდურები და ასისტოლია;



Posted by: Blind_Torture_Kill 5 Mar 2009, 16:15
QUOTE
ნეკროცერებროფილიის ყოფილა


რაში გამოიხატა ნეკროცერებროფილია ?

QUOTE
ისა და, ავამდყოფის სიმპტომები რა იყო, ცივი კიდურები და ასისტოლია

+ კიდე რეფლექსები არ ქონდა

boli.gif

Posted by: Cousteau 5 Mar 2009, 16:18
QUOTE (Blind_Torture_Kill @ 5 Mar 2009, 16:15 )

QUOTE
ისა და, ავამდყოფის სიმპტომები რა იყო, ცივი კიდურები და ასისტოლია

+ კიდე რეფლექსები არ ქონდა

boli.gif

შელესილი ენა ქონდა კიდევ yes.gif

:ქართულიმედიცინისდედაცმ*****ნსმაილიკი:

Blind_Torture_Kill
უახლოეს მომავალში ტესტბუკებზე/ვიდეოებზე და მსგავს კაი რაღაცეებზე შეგაწუხებ

:ქართულიმედიცინისდედაცმ*****ნსმაილიკი2:

Posted by: LUKA-BRAZI 5 Mar 2009, 16:32
Cousteau
აუ კუსტო ნამდვილად თანაგიგრძნობ! რა ხდება შენს თავს რომ ვერა და ვერ მოგვარდა ეგ პრობლემები? რეზიდენტურაზე ხარ მასე აკრეფილი? იქნებ გვითხრა ჩვენც, ა? სანამ თავს რამე სისულელეში გავყოფთ user.gif

Posted by: Blind_Torture_Kill 5 Mar 2009, 18:19
Cousteau
QUOTE
:ქართულიმედიცინისდედაცმ*****ნსმაილიკი:


lol.gif lol.gif lol.gif lol.gif
ეს სმაილიკიც უნდა ჩაისვას სმაილიკების გრაფაში

როცა გაგისწორდება მაშინ შემაწუხე


------------------------------------------------
პაციენტი შეშუპებული იყო (ანასარკა)
ალბუმინი < 2g/dl


რა არის სურათზე და რა სჭირდა ამ პიროვნებას ?

Posted by: LUKA-BRAZI 5 Mar 2009, 22:41
Blind_Torture_Kill
QUOTE
"Whether buried, burnt, snatched, dissected or decomposed, some people have been more useful dead than alive."

r u sure about that? spy.gif
smile.gif
მაგ სურათს ქვემოთ ან გვერდზე სახაზავის მაგვარი რამე არ ქონდა? ეგ იმიტომ რომ ზომებმა შეიძლება რაღაცაზე მიგვანიშნოს, იმიტომ რომ ის ორგანო თუ ქსოვილი, რისი პრეპარატიც არის წარმოდგენილი, ძალიან შეცვლილია და ერთი შეხედვით ძნელია მაგისი გამოცნობა.

ე.ი. ანასარკა? ანასარკას იწვევს ჰიპოალბუმინემია, რაც ან ღვიძლში მისი სინთეზის დარღვევითაა განპირობებული, ან კიდევ ალბუმინის პერმანენტული დაკარგვით. პირველ შემთხვევაში ეს შეიძლება იყოს ღვიძლის რამის ნებისმიერი პათოლოგია (სიმსივნეებიდან დაწყებული(?) ვირუსული ჰეპატიტებით დამთავრებული); მეორე შემთხვევაში კი ესაა ნეფროზული სინდრომი, ნეფროზული რანგის პროტეინურია, რაც სხვადასხვა ეტიოპათოგენეზის ნოზოლოგიებითაა განპირობებული.

რა უნდა იყოს პრეპარატზე? ნამდვილად ვერ ვხვდები ჯერ. ეგ რაღაცა ეტიოლოგიური ფაქტორი ვერ იქნება. შეიძლება ეგ იყოს ნებისმიერი ქსოვილი, რომელიც ანასარკის გამოა მასე შეცვლილი. ის ღარები და ხვეულები ერთი შეხედვით კუჭს გაგახსენებს, მაგრამ "რაცხა" არამგონია smile.gif მოკლედ ეგეთი ორგანო (თუ კუჭის არ არის, უფრო სწორაედ მისი ნაწილი) მე არ მინახავს smile.gif ეგ ორგანო არც არის ალბათ, რამე ქსოვილია, იმიტომ რომ პატარა უნდა იყოს ზომით. მასეთი პატარა ორგანოები კი ბევრი არ გავაქვს: საკვერცხე, წინამდებარე ჯირკვალი, ფარისებრი etc.

დათო ეგ მასალა შემთხვევით Area 51-ის არქივიდან ხომ არ არის? biggrin.gif მაშინ გასაგები იქნება რატომაც არ მეცნობა biggrin.gif
smile.gif

Posted by: Guardian 5 Mar 2009, 22:51
LUKA-BRAZI
QUOTE
ეგ კი არა და Guardian სად არის? რამდენი ხანია აქ არ შემოუხედავს. ექიმობა ხომ არ გადაიფიქრა? smile.gif ჰეი, Guardian სად ხარ?

მოვედი. biggrin.gif

Blind_Torture_Kill
QUOTE
პაციენტი შეშუპებული იყო (ანასარკა) ალბუმინი < 2g/dl
რა არის სურათზე და რა სჭირდა ამ პიროვნებას ?

ფსევდომემბრანოზული კოლიტი.

Posted by: LUKA-BRAZI 5 Mar 2009, 23:06
Guardian
QUOTE
მოვედი.

კაი გამარჯობა შენი wink.gif
QUOTE
ფსევდომემბრანოზული კოლიტი.

up.gif შე ცუდო! შე ცუდო! biggrin.gif დათო შენც ცუდი ხარ! lol.gif ფსევდომემბრანული კოლიტის დროს შეიძლება იყოს ანასარკა, მაგრამ რაღა ანასარკა დაწერე. დაგეწერა რამე აქედან:
* Diarrhea
* Abdominal cramps and pain
* Fever, which may be higher than 101 F (38.3 C)
* Urge to have a bowel movement (fecal urgency)
* Pus or mucus in your stool
* Nausea
* Dehydration
smile.gif

ისე კრონის დაავადების მერე, ფსევდომემბრანული კოლიტის ქეისის დადება ფრიად ლოღიკური რამ იყო biggrin.gif დამაბნეველი ტაქტიკაა smile.gif მორიგი ქეისი პოლიპოზზე იქნება? biggrin.gif

Posted by: Blind_Torture_Kill 6 Mar 2009, 00:37
Guardian
QUOTE
ფსევდომემბრანოზული კოლიტი.


no.gif
გაგრძელდა მსჯელობა

Posted by: Guardian 6 Mar 2009, 00:59
Blind_Torture_Kill
QUOTE
გაგრძელდა მსჯელობა

ჰო, აშკარად კუჭს უფრო ჰგავს ეგ.
Zollinger-Ellison-ის სინდრომი ხომ არ არის?

Posted by: Blind_Torture_Kill 6 Mar 2009, 01:43
QUOTE
Zollinger-Ellison-ის სინდრომი ხომ არ არის?


no.gif

Posted by: vano_t 6 Mar 2009, 06:35
Blind_Torture_Kill
QUOTE
პაციენტი შეშუპებული იყო (ანასარკა)
ალბუმინი < 2g/dl


რა არის სურათზე და რა სჭირდა ამ პიროვნებას ?

აბა მე ვსინჯავ მსჯელობას (როგორც მივედე სავარაუდო დიაგნოზთან და იმედია მართალია smile.gif). დავიწყებ ჰიპოალბუმინემიით, რომელიც საკმაოდ არის გამოხატული. ჰიპოალბუმინემიის ყველა მიზეზს (ციროზი, ნეფროზული სინდრომი, protein losing enteropathy, CHF) შეუძლია შეშუპების გამოწვევა გასტროინტესტინალურ ტრაქტში. მაგრამ, შეშუპებამ წესით უნდა მოგცეს ნაოჭების გადასწორება. ამ პრეპარატზე კიდევ ჰიპერტროფიული ნაოჭები გაქვს და კუჭს გავს. ამიტომ, ჩემი აზრით, კუჭის დაავადება უნდა იყოს პირველადი, რომელიც პროტეინების დაკარგვით მიმდინარეობს. ასეთ დაავადებას, რომელიც პროტეინების დაკარგვით მიმდინარეობს და კუჭის ჰიპერტროფიული ნაოჭები ახასიათებს, წარმოადგენს მენეტრიერის დაავადება (ჰიპერტროფიული ჰიპერსეკრეციული გასტროპათია). ხოდა მენეტრიერი უნდა იყოს ჩემი აზრით.

Posted by: Blind_Torture_Kill 6 Mar 2009, 10:49
vano_t

QUOTE
აბა მე ვსინჯავ მსჯელობას (როგორც მივედე სავარაუდო დიაგნოზთან და იმედია მართალია ). დავიწყებ ჰიპოალბუმინემიით, რომელიც საკმაოდ არის გამოხატული. ჰიპოალბუმინემიის ყველა მიზეზს (ციროზი, ნეფროზული სინდრომი, protein losing enteropathy, CHF) შეუძლია შეშუპების გამოწვევა გასტროინტესტინალურ ტრაქტში. მაგრამ, შეშუპებამ წესით უნდა მოგცეს ნაოჭების გადასწორება. ამ პრეპარატზე კიდევ ჰიპერტროფიული ნაოჭები გაქვს და კუჭს გავს. ამიტომ, ჩემი აზრით, კუჭის დაავადება უნდა იყოს პირველადი, რომელიც პროტეინების დაკარგვით მიმდინარეობს. ასეთ დაავადებას, რომელიც პროტეინების დაკარგვით მიმდინარეობს და კუჭის ჰიპერტროფიული ნაოჭები ახასიათებს, წარმოადგენს მენეტრიერის დაავადება (ჰიპერტროფიული ჰიპერსეკრეციული გასტროპათია). ხოდა მენეტრიერი უნდა იყოს ჩემი აზრით.


up.gif

Posted by: Guardian 6 Mar 2009, 11:53
დამადეტრენირეთ და დამჯაბნეთ, ხო? user.gif
კაი, ბატონო... biggrin.gif
აბა, ეს რა არის?

http://img17.imageshack.us/my.php?image=63187501.jpg

Posted by: LUKA-BRAZI 6 Mar 2009, 14:28
vano_t
ე.ი. შენ Guardian-ზე უფრო ცუდი ყოფილხარ lol.gif
ისე:
QUOTE
ის ღარები და ხვეულები ერთი შეხედვით კუჭს გაგახსენებს © LUKA-BRAZI

biggrin.gif
Guardian
Cabot's ring bodies
მეგალობლასტური ანემია.

Posted by: Blind_Torture_Kill 6 Mar 2009, 16:04
Guardian

QUOTE
აბა, ეს რა არის?

babesiosis

Posted by: Cousteau 6 Mar 2009, 16:41
QUOTE (vano_t @ 6 Mar 2009, 06:35 )
Blind_Torture_Kill
QUOTE
პაციენტი შეშუპებული იყო (ანასარკა)
ალბუმინი < 2g/dl


რა არის სურათზე და რა სჭირდა ამ პიროვნებას ?

აბა მე ვსინჯავ მსჯელობას (როგორც მივედე სავარაუდო დიაგნოზთან და იმედია მართალია smile.gif). დავიწყებ ჰიპოალბუმინემიით, რომელიც საკმაოდ არის გამოხატული. ჰიპოალბუმინემიის ყველა მიზეზს (ციროზი, ნეფროზული სინდრომი, protein losing enteropathy, CHF) შეუძლია შეშუპების გამოწვევა გასტროინტესტინალურ ტრაქტში. მაგრამ, შეშუპებამ წესით უნდა მოგცეს ნაოჭების გადასწორება. ამ პრეპარატზე კიდევ ჰიპერტროფიული ნაოჭები გაქვს და კუჭს გავს. ამიტომ, ჩემი აზრით, კუჭის დაავადება უნდა იყოს პირველადი, რომელიც პროტეინების დაკარგვით მიმდინარეობს. ასეთ დაავადებას, რომელიც პროტეინების დაკარგვით მიმდინარეობს და კუჭის ჰიპერტროფიული ნაოჭები ახასიათებს, წარმოადგენს მენეტრიერის დაავადება (ჰიპერტროფიული ჰიპერსეკრეციული გასტროპათია). ხოდა მენეტრიერი უნდა იყოს ჩემი აზრით.

ვანო შემეხე !
gigi.gif

Posted by: LUKA-BRAZI 6 Mar 2009, 17:31
Cousteau
QUOTE
ვანო შემეხე ! gigi.gif

lol.gif

Posted by: vano_t 6 Mar 2009, 21:40
Cousteau
QUOTE
ვანო შემეხე                                                                              !
gigi.gif

lol.gif

აი შენ, მარა, მე მგონი, მამაჯემალი ან მამაენუქი უფრო გამოგადგება მაგ ამბავში biggrin.gif
user posted image

Blind_Torture_Kill
QUOTE
ბებეზიოზი
მეც ეგ მგონია. პლაზმოდიუმები ვნახე და სხვანაირები არიან.

Posted by: Guardian 6 Mar 2009, 23:06
Blind_Torture_Kill
QUOTE
babesiosis

კი. მალტური ჯვრები. up.gif

Posted by: mtvareuli 7 Mar 2009, 00:38
Guardian

უი მართლა... სისხლზე გამახსენდა biggrin.gif იმ ჰემატოლოგიის რედაქტირებული ვარიანტი შენახული გაქვს? user.gif

Posted by: LUKA-BRAZI 7 Mar 2009, 12:18
Guardian
ეგ კი მარა ესენი Cabot's ring-ს არ გავს?

Posted by: Blind_Torture_Kill 8 Mar 2009, 00:54
აბა თუ შეგიძლიათ გამოიცნოთ რა ჭირს ნახატის ავტორს smile.gif

მარცხენაზე ნიმუშია მარჯვენაზე მისი ნახატი baby.gif

Posted by: Tornike Alashvili 8 Mar 2009, 01:06
ჰომონიმური ჰემიანოპსიაა
სივრცული პროცესი აწვება მარჯვენა მხედველობით ტრაქტს
და ამოვარდნილია მარცხენა მხედველობის არე

Posted by: Blind_Torture_Kill 8 Mar 2009, 01:48
QUOTE
ჰომონიმური ჰემიანოპსიაა სივრცული პროცესი აწვება მარჯვენა მხედველობით ტრაქტს და ამოვარდნილია მარცხენა მხედველობის არე


არა

მარა შეიძლება იყოს ეგეც smile.gif

Posted by: Tornike Alashvili 8 Mar 2009, 02:49
თუ გემიანოპსიაზეა საუბარი 4 ან 5 ვარიანტია ამ სქემიდან

user posted image

http://medarticle.moslek.ru/articles/40277.htm

Posted by: vano_t 8 Mar 2009, 03:34
Blind_Torture_Kill
QUOTE
აბა თუ შეგიძლიათ გამოიცნოთ რა ჭირს ნახატის ავტორს smile.gif

მარცხენაზე ნიმუშია მარჯვენაზე მისი ნახატი  baby.gif

ტიპიური ჰემინიგლექტი (ან ჰემისპაციუალური ნიგლექტი)-სივრცის ნახევარი მხარის "უარყოფა". მხედველობის დაზიანება არაფერ შუაშია ასეთ შემთხვევებში. მხედველობის ცენტრი რომ იყოს დაზიანებული (ან რანაირადაც არ უნდა იყოს მხედველობა დაზიანებული) ავადმყოფი თავს მოატრიალებს და მაინც დახატავს მთლიანად სურათს.

ასეთი ავადმყოფები მარცხენა სივრცეს "უარყოფენ". მარჯვენა ჰემისფეროს დაზიანება (თუმცა, სხვაგანაც შეიძლება იყოს დაზიანება და მსიქიურიც შეიძლება იყოს ეგ) იძლევა მაგას, რაც იწვევს მარცხენა მხრივ ჰემინიგლექტს. მარჯვენამხირივი ჰემინიგლექტი იშვიათია. ასეთ ავადმყოფებს უფრო ხშირად პარიეტალური წილი აქვთ დაზიანებული და არა კეფის (სადაც მხედველობის ცენტრია) წილი. მე მყავს ინსულტიანი ავამდყოფი ჰემინიგლექტით ნანახი. ასე სურათი დახატე თქო არ მითქვია, მარა, მარცხენა მხარეს რომ დავუდგებოდი, არ იყურებოდა იქით საერთო. ყველაფერი ესმოდა ისე და რომც ეტყოდი გამოიხედე აქეთო, არ იხედებოდა.

LUKA-BRAZI
QUOTE
ეგ კი მარა ესენი Cabot's ring-ს არ გავს?
არა, ლუკა ეგენი არიან ბაბეზიის განვითარების რგოლის სტადიები. ბირთვიც ჩანს კარგად. ყველაზე მთავარი მაგ სურათზე არის ის, რომ ინტრაერითროციტული პარაზიტი აშკარად ჩანს.

http://www.icp.ucl.ac.be/~opperd/parasites/babe_life.html არის სქემატურად მოცემული ბაბეზიის სტადიები. აი ტროფოზოიტი რომ შეიჭრება უჯრედში და მის მერე რომ სტადია ჩანს (ring stage-ც წერია), ეგ არის ის, რასაც შენ უთითებ.

Posted by: Blind_Torture_Kill 8 Mar 2009, 14:42
vano_t

ჰო ეგაა
მე არასდროს არ მინახავს ეგეთი ხალხი
საერთოდ პაციენტები დაავადებებით თეორიულად გვყავს "ნანახი"

Posted by: Thandrus 9 Mar 2009, 08:19
გადავხედე ბოლო გვერდებს, კარგი ქეისები იყო... smile.gif

Blind_Torture_Kill

ისე ტვინის სურათი რომ იყო, იმ ქეისში რა იყო დიაგნოზი??? ვერ გამოვიტანე პოსტებიდან დასკვნა gigi.gif

მენეტრიერის ამომხსნელს კიდევ, რესპექტი smile.gif

Posted by: Tornike Alashvili 9 Mar 2009, 14:47
QUOTE
ჰემისპაციუალური ნიგლექტი


საინტერესოა -არც გამიგონია აქამდე
რა თქმა უნდა ჰემიანოფსიის დროს მაინც ხედავს პაციენტი მთლიანობას -მაგდენი კი ვიცოდი მაგრამ მაინც ვწერდი -"ნა ვსიაკი სლუჩაი"
ვნახო ერთი მკურნალობა თუ არსებობს ჰემინიგლექტის

Posted by: Thandrus 9 Mar 2009, 17:26
Tornike Alashvili

მკურნალობა რა... თუ ქვა ჩაარტყეს მარჯვენა ნახევარსფეროში, შელოცვები თუ უშველის... givi.gif

Posted by: Tornike Alashvili 9 Mar 2009, 18:51
თუ ტრავმული დაზიანებაა -პრობლემა არ არის -არნიკა ,ჰიპერიკუმი და ბოლოს და ბოლოს ნატრიუმ სულფურიკუმი
მაგრამ მინდა მოვიძიო რაიმე სპეციფიური თუ არის ,მაგრამ ჯერ ვერაფერი ვიპოვნე

vano_t ინტერნეტში თუ არის მასალები ჰ/ნიგლექტზე?

Posted by: vano_t 9 Mar 2009, 20:15
Tornike Alashvili
QUOTE
vano_t  ინტერნეტში თუ არის მასალები ჰ/ნიგლექტზე?

ინგლისურად:
http://www.scholarpedia.org/article/Hemineglect
http://en.wikipedia.org/wiki/Hemispatial_neglect

რუსულად:
http://medbiol.ru/medbiol/har/00155bd8.htm ამაზე მეტი რუსულად ვერ ვიპოვე. ინგლისურად მასალები ბევრია. რუსულში ეს სიტყვა (ჰემიაგნოზია, რაც იგივე ჰემინიგლექტია) შეგიძლია ნახო მარჯვენამხრივი პარიეტალური წილის დაზიანების სიმპტომებში სხვადასხვა დაავადებების დროს.

http://www.google.com/search?hl=en&q=%D0%93%D0%B5%D0%BC%D0%B8%D0%B0%D0%B3%D0%BD%D0%BE%D0%B7%D0%B8%D1%8F+%D0%BF%D1%80%D0%BE%D1%81%D1%82%D1%80%D0%B0%D0%BD%D1%81%D1%82%D0%B2%D0%B5%D0%BD%D0%BD%D0%B0%D1%8F&btnG=Google+Search&aq=f&oq= საძებნი გვერდი ყველა იმ ვებგვერდებით, რომელიც შეიცავს სიტყვებს Гемиагнозия пространственная. აქაც არის მიმობნეული რაღაცეები ჰემიაგნოზიაზე.

Posted by: Tornike Alashvili 10 Mar 2009, 01:02
vano_t
მადლობა , აქაც ვნახავ და სახლში -ცოლი ნევროპათოლოგი მყავს და ახლახან უყიდე ახალი წიგნი ნეიროფსიქოლოგიაზე ამერიკელი ავტორების
იქაც დეტალურად არის განხილური აგნოზიების ყველა ფორმები

Posted by: Blind_Torture_Kill 10 Mar 2009, 03:49
Thandrus

ლპობის პროცესშია ტვინი
მაგას ეძახიან swiss cheese brain

Posted by: mtvareuli 10 Mar 2009, 15:08
....................................................................................................................

Posted by: mtvareuli 10 Mar 2009, 15:11
იგივე
....................................................................................................................

Posted by: Blind_Torture_Kill 10 Mar 2009, 18:31
ვოლფ პარკინსონ ვაითის სინდრომი
...........................................................................

Posted by: LUKA-BRAZI 12 Mar 2009, 13:41
vano_t
QUOTE
ამ ვებგვერდზე არის სქემატურად მოცემული ბაბეზიის სტადიები. აი ტროფოზოიტი რომ შეიჭრება უჯრედში და მის მერე რომ სტადია ჩანს (ring stage-ც წერია), ეგ არის ის, რასაც შენ უთითებ.

ვნახე ვანო და გასაგებია უკვე ყველაფერი, დიდი მადლობა smile.gif
* * *
Blind_Torture_Kill
QUOTE
ვოლფ პარკინსონ ვაითის სინდრომი

დათოს ვეთანხმები yes.gif
WPW syndrome.

Posted by: mtvareuli 12 Mar 2009, 19:46
Blind_Torture_Kill
LUKA-BRAZI

დიახთ tan.gif

Posted by: Blind_Torture_Kill 13 Mar 2009, 00:10
მოდი იცი როგორ ვქნათ ყოველ კვირას ერთმა დადოს რამე სერიოზული ქეისი და კვირის მანძილზე განვიხილოთ

Posted by: llaasshhaa 13 Mar 2009, 04:30
ფუუუ ეს რა ფოტოები ვნახე ცუდად ვარ ........ sad.gif((((((

Posted by: LUKA-BRAZI 13 Mar 2009, 15:05
llaasshhaa
QUOTE
ფუუუ ეს რა ფოტოები ვნახე ცუდად ვარ ........ sad.gif((((((

biggrin.gif ეგ რა არის....
Blind_Torture_Kill
QUOTE
მოდი იცი როგორ ვქნათ ყოველ კვირას ერთმა დადოს რამე სერიოზული ქეისი და კვირის მანძილზე განვიხილოთ

я за! ვინ იყოს პირველი? smile.gif ალბათ იდეის ავტორი smile.gif

Posted by: Cousteau 13 Mar 2009, 17:26
Σσς - Sigma

Posted by: Guardian 14 Mar 2009, 11:30
-------------------------------------------------------------------------------

Posted by: Zumbizu 14 Mar 2009, 12:49
Guardian


რავა ხარ კოლეგა ?


დისექციაა აორტის

Posted by: Guardian 14 Mar 2009, 13:27
Zumbizu
QUOTE
რავა ხარ კოლეგა ?

არა მიშავს - თავად?

QUOTE
დისექციაა აორტის

დიახ. smile.gif

ესა? -

Posted by: Zumbizu 14 Mar 2009, 16:06
Guardian


QUOTE
არა მიშავს - თავად?

ნიჩო

QUOTE
ესა? -

ეს არ ვიცი smile.gif

Posted by: Blind_Torture_Kill 14 Mar 2009, 17:46
Guardian

smile.gif
ამ სურათიდან პასუხს ვერ გაგცემთ
დიდ წვივშია რაღაც მარა ხ.ი.ზ. რა არის

Posted by: Guardian 14 Mar 2009, 23:01
პაციენტს ძლიერი ტკივილები ჰქონდა, განსაკუთრებით - ღამ-ღამობით.

Posted by: LUKA-BRAZI 14 Mar 2009, 23:07
Guardian
ოსტეომიელიტი მაშინ smile.gif სურათზე კი ალბათ სეკვესტრის მოცილების შემდეგ შექმნილი ღრუა ნაცვენები (ანუ ჩირქი სადაც არის ხოლმე)....

Posted by: roo 14 Mar 2009, 23:09
QUOTE (Guardian @ 14 Mar 2009, 23:01 )
პაციენტს ძლიერი ტკივილები ჰქონდა, განსაკუთრებით - ღამ-ღამობით.

იშემიური?
დღისით სიარული უჭირდა ?

Posted by: Cousteau 14 Mar 2009, 23:15
: აქ სისულელე ეწერა და აღარ წერია:
gigi.gif
* * *

Osteoid osteoma

მადლობა გუგლს

Posted by: Guardian 14 Mar 2009, 23:24
Cousteau
QUOTE
Osteoid osteoma

Mnemonic: OO (osteoid osteoma) makes you to scream OO (pain) at 00 00 (night time) with an O shape (round).

QUOTE
მადლობა გუგლს

biggrin.gif

Posted by: Cousteau 14 Mar 2009, 23:26
QUOTE (Guardian @ 14 Mar 2009, 23:24 )
Cousteau
QUOTE
Osteoid osteoma

Mnemonic: OO (osteoid osteoma) makes you to scream OO (pain) at 00 00 (night time) with an O shape (round).

მიყვარს ეგეთი რაღაცეები

yes.gif

ერთხელ წაიკითხავ და იმახსოვრებ (სანამ არ დაგავიწყდება)

Posted by: Blind_Torture_Kill 14 Mar 2009, 23:26
დამასწარით არა დაწერა ბლიად
baby.gif

mnemonics forever

Posted by: Guardian 15 Mar 2009, 04:28
რა არის ეს?

http://www.link.ge/file/225495/H.wav.html

Posted by: irakli222 15 Mar 2009, 09:38
Guardian
QUOTE
რა არის ეს?

http://www.link.ge/file/225495/H.wav.html



გალოპის რითმი?

Posted by: Guardian 15 Mar 2009, 12:46
irakli222
QUOTE
გალოპის რითმი?

კი.
ესა? -
http://www.link.ge/file/225558/G.mp3.html

Tornike Alashvili
QUOTE
Guardian-ს გაუმარჯოს !როგორახარ?

არა მიშავს.
თავად?
QUOTE
როგორ არის მტკიცებულებითი მედიცინა?

ეეჰ, აბა როგორ იქნება სააკაშვილის ხელში. biggrin.gif

Posted by: Blind_Torture_Kill 15 Mar 2009, 13:03
Guardian

ვა რა კვერნობებია ეს აუდიო ფაილები
VSD მგონია

Posted by: Guardian 15 Mar 2009, 13:08
Blind_Torture_Kill
QUOTE
VSD მგონია

არა. smile.gif

Posted by: Blind_Torture_Kill 15 Mar 2009, 13:10
კაი მაშინ სიტყვიერად გადმოეცი რა არის მაგ აუდიო ფაილში biggrin.gif
და გიპასუხებ

Posted by: LUKA-BRAZI 15 Mar 2009, 13:43
Guardian
მე ვფიქრობ ეგ ხმა II ტონს ახლავს და არა სისტოლურს.... შეიძლება ეგ იყოს მიტრალური სტენოზი....

Posted by: Guardian 15 Mar 2009, 13:50
Blind_Torture_Kill
QUOTE
კაი მაშინ სიტყვიერად გადმოეცი რა არის მაგ აუდიო ფაილში biggrin.gif და გიპასუხებ

არ გამოვა მაი ამბავი. biggrin.gif

LUKA-BRAZI
QUOTE
შეიძლება ეგ იყოს მიტრალური სტენოზი....

არა.

Posted by: LUKA-BRAZI 15 Mar 2009, 14:03
Guardian
კარგი ეგ გასაგებია, მარა what about this statement?
QUOTE
მე ვფიქრობ ეგ ხმა II ტონს ახლავს და არა სისტოლურს

right or wrong? ამას შეუძლია რამეზე მიგვანიშნოს.....
და საერთოდაც ასეთი უხეში ხმა აქვს პარკუჭთაშორისი ძგიდის დეფექტს, მაგრამ შენ უარი ტკიცე დათოს ამ მოსაზრებას და რაღა დარჩა ასეთი მაშინ?
იქნებ ეგ არის პერიკარდიუმის ხახუნის ხმიანობა? ისეთივე უხეშია ეგ....

Posted by: Guardian 15 Mar 2009, 15:32
LUKA-BRAZI
QUOTE
right or wrong?

right.

TETISA

უმჯობესი იქნება პირად ჯანმრთელობასთან დაკავშირებული კითხვები აქ დასვა -
http://forum.ge/?f=43&showtopic=33759979&st=0

Posted by: Tornike Alashvili 15 Mar 2009, 19:12
Guardian
Синдром «кошачьего мурлыканья»

Posted by: LUKA-BRAZI 15 Mar 2009, 22:01
Tornike Alashvili
QUOTE
Синдром «кошачьего мурлыканья»

ეგ ქართულად დიასტოლური კატის კრუტუნია ჰო? ეგ ხომ მიტრალური სტენოზის დროსაა.... ეს უკანასკნელი კი გარდიანმა უარყო....

Posted by: basa-ttt 15 Mar 2009, 22:26
QUOTE
ეგ ქართულად დიასტოლური კატის კრუტუნია ჰო? ეგ ხომ მიტრალური სტენოზის დროსაა.... ეს უკანასკნელი კი გარდიანმა უარყო....

სიმპტომი და არა სინდრომი -
ეგ არაა აუცილებელი მიტრალური სტენოზის დროს იყოს-
უხეშ დეფექტზე მიუთითებს და ვიბრაციასთანაა დაკავშირებული.
უხეში შუილია.
შეიძლება ბოტალის სადინრის შეუხორც იყოს(??)
თუმცა კი სისტოლაშიც უნდა ისმოდეს ასეთ დროს.

თუ აორტის სარქველების უკმარისობაა -
მაშინ ძალზე უხეში უკმარისობა უნდა იყოს ,რომ ასეთი უხეში შუილი მოგვცეს.
პაციენტის ასაკი გვჭირდება.

Posted by: Guardian 15 Mar 2009, 22:51
basa-ttt
QUOTE
თუ აორტის სარქველების უკმარისობაა

თუ - არა, აორტის სარქვლის უკმარისობაა.
სხვა დროს ერთდროულად რამდენიმე პასუხი აღარ მიიღება.

ეს რა არის? -

http://www.link.ge/file/225838/O.mp3.html



Posted by: basa-ttt 15 Mar 2009, 23:08
QUOTE
სხვა დროს ერთდროულად რამდენიმე პასუხი აღარ მიიღება.

კაი ერთი...
ასაკიც უნდა მიაწერო -
და ცოტა ჩივილებიც
თორემ შენივე შექმნილი თემის სათურს არ შეესაბამება...

Posted by: LUKA-BRAZI 15 Mar 2009, 23:18
Guardian
კომბინირებული მანკია ალბათ, სტენოზი+უკმარისობა ერთიდაიგივე სარქველზე, ან შერწყმული მანკი, ერთზე უკმარისობა, მეორეზე სტენოზი....

Posted by: Guardian 15 Mar 2009, 23:23
basa-ttt
QUOTE
კაი ერთი...

მე ჩემი გითხარი და აწი რომ აღარ ჩაგითვლი, მერე დაიწყე "კაი ერთი"-ს ძახილი.

QUOTE
ასაკიც უნდა მიაწერო - და ცოტა ჩივილებიც თორემ შენივე შექმნილი თემის სათურს არ შეესაბამება...

რა მივაწერო და რა - არა, და ჩემი თემის სათაურს რა შეესაბამება და რა - არა, მაგას როგორმე თავად გადავწყვიტავ.

LUKA-BRAZI
QUOTE
კომბინირებული მანკია ალბათ, სტენოზი+უკმარისობა ერთიდაიგივე სარქველზე, ან შერწყმული მანკი, ერთზე უკმარისობა, მეორეზე სტენოზი....

არა.

Posted by: Blind_Torture_Kill 15 Mar 2009, 23:29
იმენა ამ გულის გარდა არაფერი არაა
თან სულ აუსკულტაცია
სადამდე უნდა გვტანჯო Guardian ?

AVM

Posted by: Guardian 16 Mar 2009, 00:07
Blind_Torture_Kill
QUOTE
სადამდე უნდა გვტანჯო Guardian ?

რავი, აბა - სანამ არ მომწყინდება. biggrin.gif

QUOTE
AVM

არა.

Posted by: basa-ttt 16 Mar 2009, 00:13
QUOTE
მე ჩემი გითხარი და აწი რომ აღარ ჩაგითვლი, მერე დაიწყე "კაი ერთი"-ს ძახილი.

აი დარდი...
gigi.gif

QUOTE
ეს რა არის? -

http://www.link.ge/file/225838/O.mp3.html

ჩემი აზრით, II ტონზეა აქცენტი
შუილი არ მესმის.


QUOTE
იმენა ამ გულის გარდა არაფერი არაა
თან სულ აუსკულტაცია

ეტყობა ცხოვრებაში -
პერიოდი აქვს გულიანი
givi.gif

Posted by: mtvareuli 16 Mar 2009, 22:41
წრიპა
QUOTE
ვიცი, რომ სხვა თემაში უნდა დამეფოსტა,მაგრამ.....

Thandrus
QUOTE
ოფტოპიკივით გამომდის, მაგრამ

კხმ.... rolleyes.gif

ეხლა გადავიტან შესაბამის თემებში თქვენ პოსტებს და მეორედ ასეთი წინასწარგასძრახული ოFტოპიკები არ ვნახო biggrin.gif

Posted by: Guardian 17 Mar 2009, 00:29
მოკლედ, გავიდა 24 საათი და ვერავინ გამოიცნო, ასე რომ, ისღა დამრჩენია, სწორი პასუხი გითხრათ.

სწორი პასუხია: გულის ნორმალური ტონები. biggrin.gif

ეს რა არის?

http://www.link.ge/file/225496/1.mpg.html


Posted by: basa-ttt 17 Mar 2009, 02:15
QUOTE
რა მივაწერო და რა - არა, და ჩემი თემის სათაურს რა შეესაბამება და რა - არა, მაგას როგორმე თავად გადავწყვიტავ

ჰოო?
გუშინ დამეზარა დაწერა -
და გულის ტომებით კლინიკურ დიაგნოზს ვინ სვამს?

თუ კლინიკურ დიაგნოზი გნებავთ -
მაშინ გულის ტონების აუსკულტაცია არაა ის მეთოდი , რომელიც იძლევა საშუალებას გაარჩიო
აორტალური უკმარისობისას მოსმენილი დიასტოლური შუილი მიტრალურ სტენოზით გამოწვეული შუილისაგან. მაშინ დამატებით უნდა დაწერო კიდევ სხვა მონაცემები ან მინიმუმ ის მაინც უნდა დაწერო რომელ წერტილშია ეს მოსმენილ
არაა გულის აუსკულტაცია სპეციფიური მეთოდი და აქ უკვე გარტყმაზეა.

QUOTE
სწორი პასუხია: გულის ნორმალური ტონები

არა -
II ტონი საკმაოდ გამოხატულია.
ეს შეიძლება ნორმა იყოს - შეიძლება პათოლოგია.

QUOTE
ეს რა არის?

http://www.link.ge/file/225496/1.mpg.html

არითმია გვაქვს -
და დამატებითი ტონია, რომელიც სხვადასხვა ფაზაში ჩნდება
ეს შეიძლება მოციმიმე ართმიის დროს იყოს
ან ექსტრსსტოლების გამო იყოს.
მარტო აუსკულტაციით კლინიკური დიაგნოზი
ვერ იქნება.

Posted by: Guardian 17 Mar 2009, 02:26
basa-ttt
QUOTE
გულის ტონების აუსკულტაცია არაა ის მეთოდი , რომელიც იძლევა საშუალებას გაარჩიო აორტალური უკმარისობისას მოსმენილი დიასტოლური შუილი მიტრალურ სტენოზით გამოწვეული შუილისაგან. მაშინ დამატებით უნდა დაწერო კიდევ სხვა მონაცემები ან მინიმუმ ის მაინც უნდა დაწერო რომელ წერტილშია ეს მოსმენილ არაა გულის აუსკულტაცია სპეციფიური მეთოდი და აქ უკვე გარტყმაზეა.

გაარტყი მერე. biggrin.gif
აირჩიე ყველაზე სავარაუდო დიაგნოზი და თქვი.
თუ ვერ გაარტყამ - მერე სხვა თქვი, და ა.შ.
ორი წელია ამ თემაში პოსტავ და კიდევ ვერ ისწავლე თამაშის წესები? biggrin.gif
QUOTE
II ტონი საკმაოდ გამოხატულია.

მეორე ტონი ნორმაშია.
QUOTE
ეს შეიძლება მოციმიმე ართმიის დროს იყოს ან ექსტრსსტოლების გამო იყოს.

არა.
QUOTE
მარტო აუსკულტაციით კლინიკური დიაგნოზი ვერ იქნება.

მანდ ეკგ-ც მიჰყვება.

Posted by: basa-ttt 17 Mar 2009, 02:56
QUOTE
მეორე ტონი ნორმაშია

პირველს ჭარბობს ტემბრით.
პირიქით უნდა იყოს -
შეიძლება ეგ ნორმის ვარიანტიც იყოს.

QUOTE
მანდ ეკგ-ც მიჰყვება.

სად? ცალკე ამოიღე მაშინ
ვნახე -
QRS კომპლექსს მიჰყვება ტონები
და დამატებითი ტონი მათ შუაშია.

Posted by: Guardian 19 Mar 2009, 22:41
მოკლედ, გულის აუსკულტაციაში მოვიკოჭლებთ. rolleyes.gif
ეხლაც თუ ვერ დასვით დიაგნოზი მერე პირდაპირ პასუხს დავწერ -

Posted by: Blind_Torture_Kill 19 Mar 2009, 23:24
QUOTE
მოკლედ, გულის აუსკულტაციაში მოვიკოჭლებთ. ეხლაც თუ ვერ დასვით დიაგნოზი მერე პირდაპირ პასუხს დავწერ -


Intra-aortic balloon pump

Posted by: Guardian 20 Mar 2009, 00:25
Blind_Torture_Kill
QUOTE
Intra-aortic balloon pump

არა.

Posted by: Blind_Torture_Kill 20 Mar 2009, 00:43
Guardian
მაშინ რამე მიამატე თორე ესე გაუგებარია

Posted by: Guardian 20 Mar 2009, 00:47
Blind_Torture_Kill
QUOTE
მაშინ რამე მიამატე თორე ესე გაუგებარია

აუსკულტაცია გაქვს, ეკგ გაქვს, ექო გაქვს - არ არის საკმარისი? biggrin.gif

Posted by: Blind_Torture_Kill 20 Mar 2009, 01:07
ეკგ სად მაქვს ?
ნორმალური ეკგ ააკარი და გეტყვი

Posted by: Guardian 20 Mar 2009, 01:13
Blind_Torture_Kill
QUOTE
ეკგ სად მაქვს ? ნორმალური ეკგ ააკარი და გეტყვი

ნორმალურია ძალიან - აუსკულტაციის ვიდეოზეა. smile.gif

Posted by: Blind_Torture_Kill 20 Mar 2009, 01:20
ვერ ვარჩევ იქ ვერაფერს
სურათი დადე მარტო

Posted by: Guardian 20 Mar 2009, 01:21
Blind_Torture_Kill
QUOTE
ვერ ვარჩევ იქ ვერაფერს სურათი დადე მარტო

ეკგ საერთოდ არ გჭირდება.

Posted by: Blind_Torture_Kill 20 Mar 2009, 01:25
მაშინ სურათი შეცვალე

მიქსომაა ?

Posted by: Guardian 20 Mar 2009, 01:29
Blind_Torture_Kill
QUOTE
მაშინ სურათი შეცვალე
მიქსომაა ?

სურათს არ შევცვლი, იმიტომ, რომ მარცხენა წინაგულის მიქსომაა biggrin.gif -

http://img18.imageshack.us/my.php?image=12395679.jpg

ეს რა არის? -

http://www.link.ge/file/227480/W.mp3.html

Posted by: Blind_Torture_Kill 20 Mar 2009, 01:33
QUOTE
ეს რა არის? -


აუ კიდე აუსკულტაციაა ?

არა არ შემიძლია

Posted by: basa-ttt 20 Mar 2009, 02:09
აი საინტერესოა -
მიქსომის დიაგნოზი გულის აუსკულტაციით
როგორ უნდა დაგვესვა?
spy.gif



Posted by: Guardian 21 Mar 2009, 01:32
basa-ttt
QUOTE
მიქსომის დიაგნოზი გულის აუსკულტაციით როგორ უნდა დაგვესვა?

აუსკულტაციაში გაწაფული რომ იყო, ნამდვილად მიიტანდი მაგაზე ეჭვს.

Posted by: basa-ttt 21 Mar 2009, 01:39
QUOTE
აუსკულტაციაში გაწაფული რომ იყო, ნამდვილად მიიტანდი მაგაზე ეჭვს.

lol.gif

Posted by: Guardian 21 Mar 2009, 01:48
basa-ttt
QUOTE
lol.gif

biggrin.gif
არადა, ბოლო ქეისი მაგარი ადვილი იყო -
Patent Ductus Arteriosus - machinery-like murmur.

Posted by: zviadcardio 21 Mar 2009, 11:51
მოგესალმებით ყველას!
რა ხდება ამ ეკგ-ზე და რა შეიცვალა მის მეორე ნახევარში?


Posted by: basa-ttt 21 Mar 2009, 12:08
QUOTE
არადა, ბოლო ქეისი მაგარი ადვილი იყო

ბოლო არ მინახავს
ანუ არ მომისმენია .

zviadcardio
რაღაც ვერ გავიგე -
I , II, III განხრები სადაა?
გულმკერდის განხრები ყველა არაა

მეორე ნახევარში ელექტროდები შეიცვალა?

Posted by: LUKA-BRAZI 21 Mar 2009, 13:19
zviadcardio
ჰმ, საინტერესო ეკგ არის smile.gif პირველ ნახევარში PQ ინტერვალია მოკლე, ისე თითქოს WPW სინდრომი იყოს + QRS-ია დეფორმირებული (გავს ჰისის კონის რომელიმე ფეხის ბლოკს, თუ იშემიური დაზიანებით ინდუცირებული დეფორმირება არ არის QRS-ის), მეორე ნახევარში კი პირიქით, PQ ინტერვალია გრძელი, როგორც I ხარისხის AV ბლოკის დროს. ისე კი ვილსონებში T კბილებია ინვერსირებული მეორე ნახევარში.

Posted by: zviadcardio 21 Mar 2009, 14:19
basa-ttt
QUOTE
რაღაც ვერ გავიგე -
I , II, III განხრები სადაა?
გულმკერდის განხრები ყველა არაა


ეგენი აუცილებელი არაა გამოცნობისთვის.
მაგის გარეშეც მიხვდებით რაცაა.

QUOTE
მეორე ნახევარში ელექტროდები შეიცვალა?

არა

LUKA-BRAZI
QUOTE
ჰმ, საინტერესო ეკგ არის  პირველ ნახევარში PQ ინტერვალია მოკლე, ისე თითქოს WPW სინდრომი იყოს

აქ თბილა...

QUOTE
ავს ჰისის კონის რომელიმე ფეხის ბლოკს, თუ იშემიური დაზიანებით ინდუცირებული დეფორმირება არ არის QRS-ის

აქ ცივა...


QUOTE
მეორე ნახევარში კი პირიქით, PQ ინტერვალია გრძელი, როგორც I ხარისხის AV ბლოკის დროს. ისე კი ვილსონებში T კბილებია ინვერსირებული მეორე ნახევარში.

კი, ეს მართალია, მაგრამ რატომ?
ანუ ეკგ-ს აღწერას არ გთხოვთ...
რა ჭირს პაციენტს, და რა ხდება ეკგ-ს მეორე ნახევარში?


Posted by: LUKA-BRAZI 21 Mar 2009, 16:03
zviadcardio
QUOTE
აქ თბილა...

bis.gif
QUOTE
აქ ცივა...

user.gif
QUOTE
კი, ეს მართალია, მაგრამ რატომ? ანუ ეკგ-ს აღწერას არ გთხოვთ... რა ჭირს პაციენტს, და რა ხდება ეკგ-ს მეორე ნახევარში?

კორდარონი გაუკეთს? მგონი კორდარონი ახანგრძლივებს PQ ინტერვალს... მაგრამ დადებითი T კბილებს ასე ჩავარდნას რა იწვევს? თუ ამ ნიშანს ამ შემთხვევაში გადამწყვეტი მნიშვნელობა არ აქვს? ჯერ ეს ვარიანტი იყოს, თუ ისევ მაინც ეცივება, მერე კიდე მაქვს ერთი biggrin.gif

Posted by: Cor-toni 21 Mar 2009, 16:13
zviadcardio

WPW+ჩემი ამჟამინდელი თემა smile.gif


Posted by: basa-ttt 21 Mar 2009, 16:49
ყურადღება მიაქციეთ V1 V2 და V3 განხრებს -

პირველ ნახევარში აწეულია ST სეგმენტი -
მეორე ნახევარში კი ასეთი ეკგ ამ განხრებში არ უნდა იყოს.
R -კბილი ამოდის ზემოთ -
არც არ უნდა იყოს ნორმაში კგ - ზე

თუ მეხსიერება არ მღალატობს -
ეს ბლოკადის ნიშანია (?) -
ჰისის კონის მარჯვენა ფეხის სრული ბლოკადა?
ანუ როცა მოხსნეს WPW სინდრომი - გამოჩნდა ბლოკადა?
(???)
ეს ზეპირად რაც გავიხსენე
- მოვიძიებ...

Posted by: zviadcardio 21 Mar 2009, 19:02
LUKA-BRAZI
Cor-toni
basa-ttt
პირველ ნახევარში WPW სინდრომია, ეს მართალია.
რა ხდება მეორე ნახევარში და რატომ?
არა, არგაუკეთებიათ კორდარონი. მანდ პირველ ნახევარში სინუსური რიტმია პრეეგზიტაციით (დელტა ტალღით და PR-ის შემოკლებით). სინუსური რიტმის დროს კორდარონი არ გვჭირდება.

QUOTE
პირველ ნახევარში აწეულია ST სეგმენტი

ამას ნუ მიაქცევთ ყურადღებას. შეკითხვა სხვა მიმართულებითაა დასმული.



QUOTE
+ჩემი ამჟამინდელი თემა

არანაირი + ...
უფრო სწორად...
მოდი შეგაფასებთ: WPW-ს გამოცნობაში პლიუსი, შემდეგ მინუსი


ანუ შეკითხვა ასეთია:
რა ხდება ეკგ-ს მეორე ნახევარში და რატომ?

Posted by: Cor-toni 21 Mar 2009, 19:25
zviadcardio

WPW აბლაციის შემდეგ?

Posted by: LUKA-BRAZI 21 Mar 2009, 19:38
zviadcardio
შესაძლოა მართლაც აბლაცია გაკთდა, როგორც Cor-toni-მ თქვა, wpw-ს დროს აბლაცია ჩვეულებრივი რამეა, მაგრამ ეგ კი არ ვიცი PQ უნდა გახანგრძლივდეს მაგ მანიპულაციის მერე ? ან იქნებ მართლაც
QUOTE
როცა მოხსნეს WPW სინდრომი - გამოჩნდა ბლოკადა?
© basa-ttt.


Posted by: basa-ttt 21 Mar 2009, 20:15
აი ესაა საინტერესო -

Приблизительно в 10-20% случаев дополнительный проводящий путь расположен в непосредственной близости к естественным электрическим соединителям сердца. Аблация сопряжена с незначительным риском повреждения нормальной электрической системы сердца, что приводит к очень медленному сердцебиению. Ситуацию можно поправить с помощью искусственного ритмоводителя (см. раздел о ритмоводителях). Вероятность риска обычно составляет не более одного процента, но иногда она равняется 3-5%, и тогда врач может принять решение использовать другую форму энергетических импульсов – не прижигать, а заморозить дополнительный проводящий путь (криоаблация).

ანუ საუბარია იმაზე, რომ აბლაციით შეიძლება დაზიანდეს
გულის ნორმალური გამტარებელი სისტემა
ანუ არაა გამორიცხული ბლოკადა სწორედ აბლაციამ გამოიწვია.

Posted by: zviadcardio 21 Mar 2009, 21:34
Cor-toni
QUOTE
WPW აბლაციის შემდეგ?

ყ ო ჩ ა ღ !


LUKA-BRAZI
QUOTE
აგრამ ეგ კი არ ვიცი PQ უნდა გახანგრძლივდეს მაგ მანიპულაციის მერე ?

კი, WPW სინდრომის დროს არსებული დამატებითი გზა ზემოდან ქვემოთ იმპულსებს სწრაფად ატარებს. შესაბამისად PQ ინტერვალი მოკლეა.
აბლაციის შემდეგ იწვება დამატებითი გზა, შესაბამისად იმპულსები მხოლოდ ავ კვანძით ტარდება და PQ ინტერვალიც რამდენადმე ხანგრძლივდება.


QUOTE
საუბარია იმაზე, რომ აბლაციით შეიძლება დაზიანდეს
გულის ნორმალური გამტარებელი სისტემა

ცხადია შეიძლება.
ჩვენს შემთხვევაში ბლოკადა არ ვითარდება (QRS ნორმალურია). უბრალოდ პრეეგზიტაცია ქრება.
რაც შეეხება ინვერტირებულ (უარყოფით) T კბილებს აბლაციის შემდეგ, ამას არითმოლოგები ხატოვნად "დამატებითი გამტარი გზის მეხსიერებას" უწოდებენ.


Cor-toni
LUKA-BRAZI
basa-ttt
არ იყო მარტივი ეკგ, უნდა ითქვას, რომ კარგად იმუშავეთ

Posted by: basa-ttt 21 Mar 2009, 21:38
QUOTE
ჩვენს შემთხვევაში ბლოკადა არ ვითარდება (QRS ნორმალურია

ამის დადგენა ამ ეკგ - ით შეუძლებელია.
მილივოლტი არ ჩანს (უჯრედიანი ფონი უნდა იყოს).
ამიტომ ის რომ QRS ნორმალურია თუ განიერია -
ვერ ვიტყვით

QUOTE
რაც შეეხება ინვერტირებულ (უარყოფით) T კბილებს აბლაციის შემდეგ, ამას არითმოლოგები ხატოვნად "დამატებითი გამტარი გზის მეხსიერებას" უწოდებენ

და როგორ ხსნიან მის წარმოქნას?

Posted by: Cor-toni 21 Mar 2009, 21:58
basa-ttt

QUOTE
ამის დადგენა ამ ეკგ - ით შეუძლებელია.
მილივოლტი არ ჩანს (უჯრედიანი ფონი უნდა იყოს).
ამიტომ ის რომ QRS ნორმალურია თუ განიერია -
ვერ ვიტყვით

no.gif აშკარად ჩანს რომ ვიწრო, ნორმალური კომპლექსია.

Posted by: zviadcardio 21 Mar 2009, 23:25
basa-ttt
QUOTE
და როგორ ხსნიან მის წარმოქნას?

დაწვრილებით ეგ საკითხი გარკვეული არ არის. ითვლება რომ ამის მიზეზი რეპოლარიზაციის ცვლილებაა.
Normalization of the pre-excited QRS following ablation is accompanied by repolarization changes...

QUOTE
ამის დადგენა ამ ეკგ - ით შეუძლებელია.
მილივოლტი არ ჩანს

მართალია ჯობდა უჯრიანი ეკგ, მაგრამ მასეთს EP LAB-ში ვერ იშოვი.
ერთი რამ უდავოა, ეკგ-ს მეორე ნახევარში QRS მნიშვნელოვნად ვიწროა.


Posted by: LUKA-BRAZI 22 Mar 2009, 12:50
zviadcardio
მეც მაქვს ერთი შეკითხვა და ბარემ გკითხავ smile.gif
PQ ინტერვალი რომ გახანგრძლივდება wpw-ს აბლაციის მერე ეგ ცხადაი, მაგრამ მგონი ნორმაზე მეტად გახანგრძლივდა არა შენს ეკგ-ზე? უჯრები რო ეხატოს კარგი იქნებოდა, მაგრამ მგონი 0.2 წმ-ზე მეტი უნდა იყოს.
კიდევ ერთი კითხვა: PQ ინტერვალის გახანგრძლივება შდის უეცარი სიკვდილის რისკფაქტორებში არა?

Posted by: zviadcardio 22 Mar 2009, 13:30
LUKA-BRAZI
QUOTE
მაგრამ მგონი ნორმაზე მეტად გახანგრძლივდა არა შენს ეკგ-ზე?

კი, ცოტა ხანგრძლივია. ამის მიზეზი შეიძლება 2 რამ იყოს:
1. ნორმალური გამტარი გზის თავისებურება
2. დამატებითი გამტარი გზის აბლაციისას ნორმ. გამტარი გზის დაზიანება. აქვე უნდა ითქვას რომ დამატებითი გზის აბლაცისას ეს იშვიათად ხდება (დამოკიდებულია დამატებითი გამტარი გზის ლოკალიზაციაზე). აი ატრიოვენტრიკულური კვანძოვანი რეციპროკული ტაქიკარდიის დროს, განსაკუთრებით სწრაფი გზის აბლაციისას (ამას იშვიათად მიმართავენ) ავ ბლოკადის რისკი მნიშვნელოვნად მაღალია.


QUOTE
კიდევ ერთი კითხვა: PQ ინტერვალის გახანგრძლივება შდის უეცარი სიკვდილის რისკფაქტორებში არა?

არა, იზოლირებულად PR(Q) ინტერვალის გახანგრძლივება Sudden Death-ის პრედიქტორი, ან რისკ ფაქტორი არ არის.
აი, გრძელი, ან მოკლე QT კი არის.
ისევე როგორც, WPW სინდრომის მქონე პაციენტს აქვს მცირდ მომატებული უეცარი სიკვდლის რისკი.


Posted by: basa-ttt 22 Mar 2009, 13:32
QUOTE
2. დამატებითი გამტარი გზის აბლაციისას ნორმ. გამტარი გზის დაზიანება

და ეს შეუქცევადია?

Posted by: zviadcardio 22 Mar 2009, 13:45
basa-ttt
გააჩნია დაზიანებას, არის შექცევადი, რომელიც მალე ალაგდება და შეუქცევადი.
მაგალითად ყველაზე გავრცელებული რეგულარული SVT-ის - AVNRT-ს ნელი გზის აბლაციისას, პაციენტების 0,5%-თან შესაძლოა მიიღო შეუქცევადი სრული ავ ბლოკადა. ეს მდგომარეობა კი ცხადია პერმანენტული პეისმეიკერის იმპლანტაციას მოითხოვს.

Posted by: mtvareuli 24 Mar 2009, 21:59
განაგრძეთ ბატონებო, განაგრძეთ, აქედან მაინც გავიგო რამე, თორე იქ ST ელევაციების გარდა არაფერს მასწავლიან biggrin.gif


user.gif

Posted by: Guardian 25 Mar 2009, 00:38
You have recently read a randomized controlled trial assessing
the efficacy of patient self-management of anticoagulation to
reduce bleeding risks. In 1 group, patients were trained to adjust
their own warfarin dosage according to the international normalized
ratio (INR) results obtained using a home INR device.
This group was compared with a clinic-based group of patients
who visited their clinic every 4 weeks for INR checks. A physician
then adjusted the dosage and arranged for the next INR
check. At 1 year, the self-management group had a 2% risk of
any (major or minor) bleeding events. The clinic-based group
had a 7% risk of any bleeding events. On the basis of this information,
how many patients need to be enrolled in a system of
self-management of anticoagulants for a year to eliminate 1
adverse event of bleeding?

Posted by: LUKA-BRAZI 25 Mar 2009, 00:49
Guardian
სენიორ გარდიან, კვლევის აზრი და results გასაგებია, მაგრამ კითხვას ვერ მივხვდი :
QUOTE
On the basis of this information, how many patients need to be enrolled in a system of self-management of anticoagulants for a year to eliminate 1 adverse event of bleeding?

რა ზუსტად აინტერესებთ პაციენტების რაოდენობა? ანუ რამე კვლევის მეთოდიკის ცოდნასთან არის ეს დაკავშირებული?

P.S. ისე არ მეგონა სელფ-მენეჯმენტს ასეთი მირწევები თუ ექნებოდა. აბა ეგ კვლევა აქ ჩაეტარებინათ...... biggrin.gif კითხვას აშკარად სხვანაირად დასვამდნენ biggrin.gif

Posted by: Guardian 25 Mar 2009, 00:53
LUKA-BRAZI
QUOTE
რა ზუსტად აინტერესებთ პაციენტების რაოდენობა?

კი.
QUOTE
ანუ რამე კვლევის მეთოდიკის ცოდნასთან არის ეს დაკავშირებული?

არა.

Posted by: basa-ttt 25 Mar 2009, 11:54
Guardian
ტექსტი ქართულად დადეთ.
ფორუმი ქართულენოვანია.

Posted by: Guardian 25 Mar 2009, 12:08
basa-ttt
QUOTE
ტექსტი ქართულად დადეთ.

ვინც ინგლისური არ იცის, Evidence-Based Medicine-ში არაფერი ესაქმება.
QUOTE
ფორუმი ქართულენოვანია.

ამ ფორუმზე ინგლისურენოვანი ტექსტების დადება აკრძალული არ არის.

Posted by: basa-ttt 25 Mar 2009, 12:09
QUOTE
ვინც ინგლისური არ იცის, Evidence-Based Medicine-ში არაფერი ესაქმება.

Guardian
საქართველოში ცხოვრობ
და კეთილი ინებე
და ქართული ქუდი დაიხურე.
თუ არადა მიბრძანდი აშშუში საცხოვრებლად.

Posted by: the_lizard_king 25 Mar 2009, 12:22
Guardian
მასონო yes.gif

Posted by: Guardian 25 Mar 2009, 12:27
basa-ttt
QUOTE
საქართველოში ცხოვრობ და კეთილი ინებე და ქართული ქუდი დაიხურე.

თორემ? smile.gif
QUOTE
თუ არადა მიბრძანდი აშშუში საცხოვრებლად.

თავად კი დიდი რუსეთი გელოდება. biggrin.gif

Posted by: DICOM 25 Mar 2009, 12:34
basa-ttt

QUOTE
Guardian
საქართველოში ცხოვრობ
და კეთილი ინებე
და ქართული ქუდი დაიხურე.
თუ არადა მიბრძანდი აშშუში საცხოვრებლად.


ეს არასწორი მიდგომაა. დღეს წარმოუდგენელია ექიმმა ინგლისური არ იცოდეს და პროფესიონალად ჩამოყალიბდეს. EBM ზე აბსოლუტურად ვეთანხმები გარდიანს. EBM-ის იდეოლოგია და წარმართვის პრინციპები ძირეულად ინგლისურ ენაზეა დამყარებული. იგივე ეხება სამედიცინო ინფორმაციის აგრეგაციის და ძიების სისტემებს - ინგლისური თუ არ იცი ფონს ვერ გახვალ.

Posted by: basa-ttt 25 Mar 2009, 12:44
QUOTE
ეს არასწორი მიდგომაა. დღეს წარმოუდგენელია ექიმმა ინგლისური არ იცოდეს და პროფესიონალად ჩამოყალიბდეს. EBM ზე აბსოლუტურად ვეთანხმები გარდიანს. EBM-ის იდეოლოგია და წარმართვის პრინციპები ძირეულად ინგლისურ ენაზეა დამყარებული. იგივე ეხება სამედიცინო ინფორმაციის აგრეგაციის და ძიების სისტემებს - ინგლისური თუ არ იცი ფონს ვერ გახვალ.

როცა კლინიკურ შემთხვევას დებთ ფორუმზე -
უნდა დაიდოს ქართულად.
ვინ რა იცის -
ეს სხვა საკითხია.


QUOTE
თუ არადა მიბრძანდი აშშუში საცხოვრებლად.


თავად კი დიდი რუსეთი გელოდება

მე რუსულად არ გესაუბრებით

Posted by: Solveig 25 Mar 2009, 12:45
QUOTE
თუ არადა მიბრძანდი აშშუში საცხოვრებლად.


ნეტავ შენ იყო კონსული smile.gif))

DICOM
QUOTE
დღეს წარმოუდგენელია ექიმმა ინგლისური არ იცოდეს და პროფესიონალად ჩამოყალიბდეს.

არამარტო ექიმმა.

Posted by: mtvareuli 25 Mar 2009, 13:30
basa-ttt
QUOTE
ტექსტი ქართულად დადეთ. ფორუმი ქართულენოვანია.

არაქართულენოვანი ტექსტები აქ არავის აუკრძალია. წყაროებს და არგუმენტებს თუ დებთ სხვა ენაზე, აქ რატომ არ შეიძლება დაიდოს? სხვა ენაზე მგონი არავინ გესაუბრებათ, მხოლოდ შემთხვევაა დადებული.

Posted by: zviadcardio 25 Mar 2009, 13:31
Guardian
QUOTE
You have recently read a randomized controlled trial assessing
the efficacy of patient self-management of anticoagulation to
reduce bleeding risks. In 1 group, patients were trained to adjust
their own warfarin dosage according to the international normalized
ratio (INR) results obtained using a home INR device.
This group was compared with a clinic-based group of patients
who visited their clinic every 4 weeks for INR checks. A physician
then adjusted the dosage and arranged for the next INR
check. At 1 year, the self-management group had a 2% risk of
any (major or minor) bleeding events. The clinic-based group
had a 7% risk of any bleeding events. On the basis of this information,
how many patients need to be enrolled in a system of
self-management of anticoagulants for a year to eliminate 1
adverse event of bleeding?


ჩემი პასუხია: 20 პაციენტი

P.S. ჩემი თხოვნაა ყველასთან, ნუ ვიკამათებთ ყველაფერზე ერთმანეთთან. შენ ის რატომ თქვი, როგორ თქვი და ა.შ...
არ მინდა ეს თემა ბევრ სხვა თემას დაემსგავსოს...
ცოტა მეტი რამ ვაპატიოთ ერთმანეთს და ვისაუბროდ მხოლლოდ სამედიცინო საკითხებზე.
გმადლობთ.

Posted by: LUKA-BRAZI 25 Mar 2009, 15:00
zviadcardio
Guardian-i ენაწყლიანობით არ გამოირჩევა და შენ მაინც მითხარი რას ნიშნავს შენი პასუხი (ანუ როგორ გამოიანგარიშე 20 პაციენტი) და როგორ უნდა გაიგო რამდენი პაციენტი უნდა ჩართო კვლევაში. რამე წესია რანდომიზირებულ კვლევებში ამდაგვარი, თუ..... საერთოდ რაზე დასვა კითხვა გარდიანმა ვერც მივხვდი.... ლოლ user.gif

Posted by: zviadcardio 25 Mar 2009, 17:13
LUKA-BRAZI
შეკითხვა იყო ასე დასმული:
რამდენი პაციენტი უნდა ჩავრთოთ ამ ჯგუფში რომ სისხლდენის1 შემთხვევა ავიცილოთ თავიდანო (მე ვფიქრობ რომ შეკითხვა დასმულია კლინიკურ (clinic-based ) ჯგუფთან შესადარებლად).
ერთ ჯგუფში 7% იყო, მეორეში 2%. სხვაობა მათ შორის 5%.
ანუ ყოველ 100 კაციდან 5-ით ნაკლებს ემართება სისხლდენა მეორე ჯგუფში.
ანუ სისხლდენის 5 შემთხვევის შემცირებისთვის საჭიროა 100 პაციენტი
1 შემთხვევის შემცირებისთვის საჭირო იქნება 20 პაციენტი.
(მათემატიკური პროპორციაა: თუ 5 შემთხვევის შემცირებისთვის საჭიროა 100, 1-სთვის საჭირო იქნება X; X=100 გავამრავლოთ 1 / 5 = 20).
ანუ პასუხია 20.
P.S. არ ვიცი რამდენად გასაგებად ვწერ...

Posted by: LUKA-BRAZI 25 Mar 2009, 18:21
zviadcardio
ააა თურმე რა ყოფილა biggrin.gif მე მეგონა randomized controlled trial-ის რამე პრინციპი იყო და ა.შ. lol.gif
დიდი მადლობა smile.gif

Posted by: zviadcardio 25 Mar 2009, 18:24
LUKA-BRAZI
ეს ჩემი აზრია, ლოგიკურად ასე უნდა იყოს.

QUOTE
დიდი მადლობა

არაფერს მეგობარო.

Posted by: Guardian 25 Mar 2009, 22:09
zviadcardio
QUOTE
რამდენი პაციენტი უნდა ჩავრთოთ ამ ჯგუფში რომ სისხლდენის1 შემთხვევა ავიცილოთ თავიდანო (მე ვფიქრობ რომ შეკითხვა დასმულია კლინიკურ (clinic-based ) ჯგუფთან შესადარებლად). ერთ ჯგუფში 7% იყო, მეორეში 2%. სხვაობა მათ შორის 5%. ანუ ყოველ 100 კაციდან 5-ით ნაკლებს ემართება სისხლდენა მეორე ჯგუფში. ანუ სისხლდენის 5 შემთხვევის შემცირებისთვის საჭიროა 100 პაციენტი 1 შემთხვევის შემცირებისთვის საჭირო იქნება 20 პაციენტი. (მათემატიკური პროპორციაა: თუ 5 შემთხვევის შემცირებისთვის საჭიროა 100, 1-სთვის საჭირო იქნება X; X=100 გავამრავლოთ 1 / 5 = 20). ანუ პასუხია 20.

yes.gif

You are asked to determine the number needed to treat (NNT) in
a randomized controlled trial showing that the self-management
anticoagulant therapy group had a bleeding event rate of 2%; that
is, the experimental event rate (EER) = 2%. The group receiving
usual care group had an event rate of 7%; that is, the control event
rate (CER) = 7%. NNT = 1/ARR, where ARR is the absolute risk
reduction. ARR = (CER−EER). In this case, 7%−2% = 5%.
Therefore, NNT = 1/.05 = 20.

Posted by: Blind_Torture_Kill 26 Mar 2009, 17:26
Guardian
ამაზე უფრო ტვინის სა...ვი ვერაფერი ნახე smile.gif

Posted by: zviadcardio 27 Mar 2009, 13:04
დიდი ხანია არაფერი დაგვიდია.
აი ECG. რა რიტმია ამ კარდიოგრამაზე წითელ ხაზამდე და მის შემდეგ?
რომელი პრეპარატების გამოყენება არ შეიძლება ამ დროს?

Posted by: LUKA-BRAZI 27 Mar 2009, 14:43
zviadcardio
წითელ ხაზამდე იდიოვენტრიკულური რითმია ვითომ? spy.gif მერე კი წვრილტალღოვანი ფიბრილაციაა smile.gif

Posted by: basa-ttt 27 Mar 2009, 17:06
QUOTE
წითელ ხაზამდე იდიოვენტრიკულური რითმია

ჰო
პარკუჭოვანი რითმია
და მერე ფიბრილაცია.
დეფიბრილატორი და მისი ჯანი.

Posted by: zviadcardio 27 Mar 2009, 17:16
LUKA-BRAZI
QUOTE
წითელ ხაზამდე იდიოვენტრიკულური რითმია ვითომ?

QUOTE
პარკუჭოვანი რითმია

არა.
ხაზის შემდეგ მართლები ხართ.
ხო დეფიბრილატორი კი ბატონო მაგრამ, სანამ მაგამდე მივა საქმე (ანუ წითელ ხაზამდე) რომელი პრეპარატების გამოყენება არ შეიძლება და რატომ?

დავწეროთ რა ხდება წითელ ხაზამდე.



Posted by: Vagusa 27 Mar 2009, 22:45
წითელ ხაზამდე მოციმციმე არითმიაა, რომელიც ტარდება დამატებითი გზით, წითელი ხაზის შემდეგ ტორსადე დე პოინტესი. ასეთ პაციენტთან მომავალში (თუ რაღა თქმა უნდა ამ არითმიას გადაურჩა) არ უნდა გაკეთდეს ატფ-ი.

Posted by: Solveig 27 Mar 2009, 23:40
რა პათოლოგიურ პროცესზე მიუთითებს ღვიძლში ინტენსიურად გამოხატული და გაფართოებული სისხლძარღვებისს ქსელი?

Posted by: zviadcardio 28 Mar 2009, 00:16
Vაგუსა
QUOTE
წითელ ხაზამდე მოციმციმე არითმიაა, რომელიც ტარდება დამატებითი გზით,

დიახ ეს წინაგულთა ფიბრილაციაა პრეეგზიტაციით.
რაც შეეხება მეორე ნახევარს ეს არ არის კლასიკური ტორსადესი (ცოტათი გავს), პარკუჭთა ფიბრილაცია.
შეკითხვას პრინციპულად სწორედ გაეცა პასუხი.
მიხარია.
"ყოჩაღ კლიმ!"

P.S. არა მხოლოდ ატფ-ს (თუმცა, ამ დროს ატფ-ს გამოყენება ისედაც არ არის სწორე გადაწყვეტილება, ადამიანს მოციმციმე არითმია აქვს, ატფ კი არც სიხშირის და არც რიტმის კონტროლისთვის არ გამოგადგებათ), არც ერთი ავ მაბლოკირებელი აგენტის (ბეტა ბლოკატორი, არადიჰიდროპირიდინული კალციუმის ანტაგონისტი, დიგოქსინი) გამოყენება არ შეიძლება. ავ კვანძის ბლოკირება ხელს შეუწყობს დამატებით გზაში იმპულსების გატარების სიხშირის მატებას და ხელს შეუწყობს პარკუჭთა ფიბრილაციის განვითარებას.

Posted by: Blind_Torture_Kill 28 Mar 2009, 09:11
QUOTE
რა პათოლოგიურ პროცესზე მიუთითებს ღვიძლში ინტენსიურად გამოხატული და გაფართოებული სისხლძარღვებისს ქსელი?


ინფო მეტი არ მოყვება ?

(პოსტ-ჰეპატური ობსტრუქცია)

Posted by: Solveig 28 Mar 2009, 16:32
Blind_Torture_Kill
QUOTE
ინფო მეტი არ მოყვება ?


დამატებითი ინფორმაციის სახით შემიძლია ვთქვა, რომ არის კანქვეშა სიმსივნე (მელანომა), რომლის უჯრედებშიც გაძლიერებულია ერთ-ერთი ზრდის ფაქტორი (გრანულოციტ-მაკროფაგების კოლონიამასტიმულირებელი ფაქტორი). ამის პირდაპირი შედეგია სპლენომეგალია და მასში დიდი რაოდენობით მოუმწიფებელი მიელოიდური უჯრედების დაგროვება (ეს ლოგიკური და ახსნადია).

ჰო, მკითხველს რომ არ გაუკვირდეს-ადამიანზე არ არის ლაპარაკი, თაგვზეა. ჩემი ინტერესი იმან გამოიწვია, რომ რამდენიმე თაგვი უეცრად მოკვდა (წინა დღით არაფერი ემჩნეოდათ და სიმსივნეც არ იყო დიდი ზომის). სამწუხაროდ, იმავე დღეს ვერ ვნახე და მერე უკვე გაკვეთას აზრი აღარ ჰქონდა sad.gif

ცოცხლად დარჩენილი თაგვების გაკვეთის დროს კი ვნახე ღვიძლში გაფართოებული სისხლძარღვების ქსელი. ჰეპატომეგალია განსაკუთრებით გამოხატული არ ყოფილა.

Posted by: LUKA-BRAZI 28 Mar 2009, 20:40
Solveig
კვლევას ატარებთ რამეს?
QUOTE
კანქვეშა სიმსივნე (მელანომა)

QUOTE
ამის პირდაპირი შედეგია სპლენომეგალია და მასში დიდი რაოდენობით მოუმწიფებელი მიელოიდური უჯრედების დაგროვება (ეს ლოგიკური და ახსნადია).

და ღვიძლში არ არის მელანომის დროს მეტასტაზები? მე ვგულისხმობ როგორც წესი, თუა მაგ დროს მეტასტაზები და თუ კი მაშინ რა დროში ვითარდება? არამგონია 1-2 დღეში ეგ თაგვები მეტასტაზებს მოეკლა, მასე სწრაფად არ ხდება ეგ პროცესი, მიუხედავად იმისა რომ მელანომა ავთვისებიანი და სწრაფადპროგრესირებადი სიმსივნეა.

მისმინე სოლვეიგ, მაგ თაგვებს რამე წამალს აძლევდით? რამე ექსპერიმენტულს, ან ახალ კომბინაციას, ან სიმსივნე რომ დამართეთ რამე side effect-ის მსგავსი ხომ არ აქვს არჩეულ მეთოდიკას, ტექნიკურად როგორ ჩატარდა, რამე სხაც ხომ არ გამოიწვიეთ პროცედურის დროს etc.

თორე ისე კი ძნელია ფორუმის ერთ პოსტზე დაყრდნობით ექსპერიმენტზე რამე თქვა smile.gif დიდად მეც ვერ ვერკვევი სამწუხაროდ user.gif

ისე რაც ამ თაგვებს მედიცინის განვითარებაში მიუძღვით წვლილი.... ნამდვილად სიმბოლურ რამეს იმსახურებენ ეგ თაგვები, ძეგლს, მონუმენტს, მემორიალს..... ა.შ. მე აუცილებლად დავუდგამდი, იმსახურებენ ნამდვილად yes.gif , ისევე როგორც ძაღლები პავლოვის ექსპერიმენტში smile.gif

Posted by: Solveig 28 Mar 2009, 21:45
LUKA-BRAZI

მაგარი სურათია smile.gif)) მაგრამ ეგ თაგვები (თეთრები) ძალიან ჩურჩუტები არიან. მე შავები მყავს და ისეთი სხარტები და მკვირცხლები არიან..ოჰოჰოჰ biggrin.gif

კვლევას ვატარებ, კი. ღვიძლში მეტასტაზი არ მინახავს, მარტო ერთ თაგვში ვნახე-ფილტვში, მაგრამ ის თაგვი მე მოვკალი და გაკვეთის დროს ვნახე. თაგვები მოკვდნენ მე-5 თუ მე-6 დღეს, როცა სიმსივნე მაინცდამაინც დიდი არ იყო. არანაირი წამალი არ მიმიცია. ეგ არ არის ჩემი სფერო smile.gif ტექნიკურად კი ძალიან მარტივი ჩასატარებელია-მარცხენა უკანა კიდურში, კანქვეშ შეგყავს გარკვეული რაოდენობით სიმსივნური უჯრედები (უჯრედული ხაზია).

ეგ ექსპერიმენტი იმისთვის წამოვიწყე, რომ დიდი რაოდენობით მოუმწიფებელი მიელოიდური უჯრედები მჭირდებოდა, მივიღე კიდეც, მაგრამ როგორც უკვე გამოჩნდა, აშკარად გაუთვალისწინებელი მოვლენებიც მოხდა. სხვა სიმსივნური უჯრედებიც (მათ შორის, ღვიძლის კარცინომის) გამომიყენებია, მაგრამ მაგგვარი ცვლილები ღვიძლში არ მიმიღია..ის ციტოკინი, რომლის ექსპრესიაც იყო გაძლირებული სიმსივნეში, პირდაპირ არ მოქმედებს ანგიოგენეზზე..მოკლედ ეტყობა, უნდა დაველოდო, როდის შეიწუხებს ჩემი უფროსი თავს და წაიღებს ღვიძლს ჰისტოლოგიაზე...მანამდე კია ჯერ ფიქსირებული...biggrin.gif



Posted by: LUKA-BRAZI 29 Mar 2009, 12:25
Solveig
QUOTE
მაგრამ ის თაგვი მე მოვკალი

შე "ცუდო", რას ერჩოდი? biggrin.gif
QUOTE
მივიღე კიდეც, მაგრამ როგორც უკვე გამოჩნდა, აშკარად გაუთვალისწინებელი მოვლენებიც მოხდა.

ჰმმ, რა იცი, იქნებ რამე ახალიც კი აღმოაჩინე smile.gif ეგრე აღმოაჩინა ალექსანდრე ფლემინგმაც პენიცილინი smile.gif
QUOTE
მოკლედ ეტყობა, უნდა დაველოდო, როდის შეიწუხებს ჩემი უფროსი თავს და წაიღებს ღვიძლს ჰისტოლოგიაზე

კვლევა საქართვლოში ტარდება? აბა ზარმაციუფროსბი სხვაგან სად იქნებიან biggrin.gif ეს ხუმრობით...... კოლაბორაცია არ გაქვთ სხვა ლაბორატორიებთან? შეიძლება მათ იცოდნენ და აღწერილიც ქონდეთ ეგეთი შემთხვევები. ისე კი რა თქმა უნდა ჯერ ჰისტოლოგიის პასუხს უნდა დაელოდო yes.gif
ისე რა ციტოკინი იყო ექსპრესირებული თუ საიდუმლო არ არის? smile.gif

აი შავი თაგუნაც smile.gif

Posted by: Solveig 29 Mar 2009, 17:37
LUKA-BRAZI
QUOTE
კვლევა საქართვლოში ტარდება?

სამწუხაროდ, არა..გერმანიაში ვარ.
ეგ უჯრედული ხაზი ვინ გააკეთა, არ ვიცი, 8 წლის წინანდელი კი იყო...გრანულოციტ-მაკროფაგების კოლონიამასტიმულირებელი ფაქტორის (GM-CSF) სინთეზის უნარი იყო გაძლიერებული. საერთოდ, სიმსივნის დროს ისედაც დიდი რაოდენობით გროვდება მოუმწიფებელი მიელოიდური უჯრედები (რაც სიმსივნის საწინააღმდეგო იმუნური პასუხის სუპრესიის ერთ-ერთი გზაა) და ეს ციტოკინი სიმსივნის თანაობისას კიდევ უფრო უწყობს ამას ხელს..ამიტომაც გამოვიყენე.

QUOTE
რა იცი, იქნებ რამე ახალიც კი აღმოაჩინე


არ ვიცი, მაგაზე თავს ვერ გადავდებ..იდეაში, ჯერ ჰისტოლოგია უნდა გაკეთდეს. მერე-ვნახოთ..

ზარმაც უფროსებს რაც შეეხება-აქ არც ისე კარგადაა საქმე, როგორც შეიძლება მანდ ეგონოს ადამიანს .

გავაოფტოპიკეთ..წაშლიან ამას ალბათ smile.gif

Posted by: LUKA-BRAZI 29 Mar 2009, 18:11
Solveig
ჰოო, რა ვიცი აბა, მეტი რა გითხრა smile.gif შეიძლება Guardian-საც კითხო. გარდიანი და მიკროსკოპი მეგობრობენ და შეიძლება იცოდეს რამე. Anyway good luck wink.gif

Posted by: basa-ttt 2 Apr 2009, 20:55
რა არის ეს?
vik.gif
რატომ არ დებთ ახალ შემთხვევებს?
war.gif
რას ჰგავს თქვენი საქციელი?
ის "გულიანი" სად წავიდა?
gigi.gif

Posted by: Guardian 4 Apr 2009, 11:49
basa-ttt
QUOTE
რატომ არ დებთ ახალ შემთხვევებს?

აჰა -

http://img12.imageshack.us/my.php?image=16977439.jpg

http://img12.imageshack.us/my.php?image=66509195.jpg

Posted by: Guardian 4 Apr 2009, 23:05
ვერა?
კაი, რითი განსხვავდება წინა სურათები ამისგან? -

Posted by: basa-ttt 5 Apr 2009, 03:14
eek.gif

Guardian
ეს ხომ რამე ნორმალურს არ დადებს რა....
არ გაგახარებს...
lol.gif

Posted by: Blind_Torture_Kill 7 Apr 2009, 21:24
Guardian
ბიჭო ესე მშრალად რა გავიგოთ დაამატე რამე
საერთოდ რა არის ვერ ვხვდები baby.gif

Posted by: Cousteau 7 Apr 2009, 22:41
QUOTE (Guardian @ 4 Apr 2009, 11:49 )
basa-ttt
QUOTE
რატომ არ დებთ ახალ შემთხვევებს?

აჰა -

http://img12.imageshack.us/my.php?image=16977439.jpg

http://img12.imageshack.us/my.php?image=66509195.jpg

კაი პირველ სისულელეს მე ვიტყვი.

ვითომ კუნთების განივი ჭრილია და ვითომ ბოჭკოები არის შემცირებული და ის თეთრი არის ცხიმი და ჰე-ჰეიიი! ცხიმით ჩანაცვლება კუნთების ქსოვილის გვხვდება? Thats right, დუშენის დისტროფია
?
არა?
ნწ?
ნუ კაი ხო user.gif

P.S.
შემდეგ ჯერზე თემა ჯერ ქსოვილების გამოცდნობით და სლაიდების აღწერით დავიწყოთ

Posted by: mtvareuli 7 Apr 2009, 23:17
Cousteau
QUOTE
შემდეგ ჯერზე თემა ჯერ ქსოვილების გამოცდნობით და სლაიდების აღწერით დავიწყოთ

ბიძია გარდიანმა პათანატომიის გაკვეთილები ჩაგვიტაროს? biggrin.gif

Posted by: Guardian 8 Apr 2009, 12:59
პირველ ორ სურათზე იყო ძუძუს წილაკოვანი კიბო in situ, მესამეზე - ძუძუს წილაკოვანი ინვაზიური კიბო.

mtvareuli
QUOTE
ბიძია გარდიანმა პათანატომიის გაკვეთილები ჩაგვიტაროს?

უკვე ბიძია გავხდი?
ეჰ... user.gif

biggrin.gif 2kiss.gif

ეს რა არის? -

http://img27.imageshack.us/my.php?image=93568608.jpg



Posted by: irakli222 8 Apr 2009, 15:21
Guardian
გამარჯობა!
რამოდენიმე ჩემს კოლეგას ვაჩვენე შენს მიერ დადებული სურათები.
ყველანი ჰისტოლოგიაში სუსტები ვართ.
ერთობლივი აზრი------ინვაზიური ეპითელიური კიბო. რა ორგანოსი ვერ ვაზუსტებთ,
(ძუძუ ალბათ არა)

Posted by: Blind_Torture_Kill 8 Apr 2009, 19:38
Guardian

კანია
ცოტა დიდი გადიდება რა თუ შეიძლება

actinic keratosis

Posted by: mtvareuli 9 Apr 2009, 04:03
ოFF ტოპიკ:

Guardian
QUOTE
უკვე ბიძია გავხდი?

ჩემთვის ბიძია გახდი, ტანკე ჯანმედისთვის მოდერობის კანდიდატად გასახელებს, რა დრო მოვიდა biggrin.gif

Posted by: Guardian 9 Apr 2009, 11:50
mtvareuli
QUOTE
ჩემთვის ბიძია გახდი, ტანკე ჯანმედისთვის მოდერობის კანდიდატად გასახელებს, რა დრო მოვიდა

ეუჰ, მართლა?
O, tempora, O, mores. biggrin.gif
არ გამოვა მაი ამბავი, არ მინდა მოდერობა. no.gif
შენ ცოტა ხანს კიდევ იყავი, რა, და გამოჩნდება ვინმე შესაფერისი, იმედია.
vano-t-ს კანდიდატურას შემოგთავაზებდით, მაგრამ არ ვიცი, თავად უნდა თუ არა, თან ამ ბოლო დროს დაიკარგა სადღაც. user.gif

irakli222
QUOTE
ერთობლივი აზრი------ინვაზიური ეპითელიური კიბო. რა ორგანოსი ვერ ვაზუსტებთ, (ძუძუ ალბათ არა)

დავდე უკვე სწორი პასუხი ზემოთ.

Blind_Torture_Kill
QUOTE
actinic keratosis

არა.
QUOTE
ცოტა დიდი გადიდება რა თუ შეიძლება

აჰა -

Posted by: LUKA-BRAZI 9 Apr 2009, 14:05
Blind_Torture_Kill
მომენატრა ჯიგრული, სუფთა კლინიკური ქეისი biggrin.gif ანამნეზით და ობიექტური მონაცემებით. დათო დადე რა რამე მასეთი smile.gif
Cousteau
კუსტო როგორ ხარ მეგობარო? smile.gif რასა იქმნ, დალაგდა რეზიდენტურის ამბები? გასწავლიან რამეს, თუ ისევ თვითგანათლებაზე არიან რეზიდენტები? user.gif

Posted by: mtvareuli 9 Apr 2009, 18:48
LUKA-BRAZI

მე რო წავიჩეთავე ცოტა, იმას კი არ ნიშნავს, რომ უნდა მომბაძოთ biggrin.gif


Guardian

ამ ნოთ შუარ, მაგრამ შავი აკანტოზი მგონია.

Posted by: Blind_Torture_Kill 9 Apr 2009, 22:48
LUKA-BRAZI
აი ეს გამოიცნონ და მერე დავდებ

Guardian
ბრტყელუჯრედოვანი კიბოა მაშინ

Posted by: Thandrus 10 Apr 2009, 07:11
Guardian

Basal Cell Carcinoma?

Posted by: Cousteau 10 Apr 2009, 12:11
QUOTE (LUKA-BRAZI @ 9 Apr 2009, 14:05 )

Cousteau
კუსტო როგორ ხარ მეგობარო? smile.gif რასა იქმნ, დალაგდა რეზიდენტურის ამბები? გასწავლიან რამეს, თუ ისევ თვითგანათლებაზე არიან რეზიდენტები? user.gif

იქით კვირიდან ვანებებ თავს ალბათ
თვითაგანათლებაც ვეღარ მიქაჩავს.

QUOTE
ე რო წავიჩეთავე ცოტა, იმას კი არ ნიშნავს, რომ უნდა მომბაძოთ biggrin.gif


როგორ გეკადრებათ no.gif

Posted by: Guardian 10 Apr 2009, 12:44
mtvareuli
QUOTE
შავი აკანტოზი მგონია.

არა.
Blind_Torture_Kill
QUOTE
ბრტყელუჯრედოვანი კიბოა მაშინ

არა.
Thandrus
QUOTE
Basal Cell Carcinoma?

არა.

Posted by: LUKA-BRAZI 10 Apr 2009, 14:04
mtvareuli
QUOTE
მე რო წავიჩეთავე ცოტა, იმას კი არ ნიშნავს, რომ უნდა მომბაძოთ

1000 excuses smile.gif

Posted by: Blind_Torture_Kill 10 Apr 2009, 14:35
Guardian
დადე სწორი პასუხი
ვერ გამოვიცანით

Posted by: Guardian 10 Apr 2009, 19:50
Blind_Torture_Kill
QUOTE
დადე სწორი პასუხი ვერ გამოვიცანით

ცოტას გიკარნახებთ ექიმი ჰაუსის სტილში biggrin.gif -
რა არის ის, რაც სინამდვილეში ისე არ გამოიყურება, როგორც თქვენ გგონიათ, რომ უნდა გამოიყურებოდეს? wink.gif

Posted by: Blind_Torture_Kill 10 Apr 2009, 20:59
QUOTE
ცოტას გიკარნახებთ ექიმი ჰაუსის სტილში  -
რა არის ის, რაც სინამდვილეში ისე არ გამოიყურება, როგორც თქვენ გგონიათ, რომ უნდა გამოიყურებოდეს?


ამას არ ჯობდა პირდაპირ პასუხი

Posted by: LUKA-BRAZI 10 Apr 2009, 22:38
Guardian
QUOTE
რა არის ის, რაც სინამდვილეში ისე არ გამოიყურება, როგორც თქვენ გგონიათ, რომ უნდა გამოიყურებოდეს?

მასეთი რამ სიფილისზე გვითხრეს, განსაკუთრებით II (თუ არ მეშლება) სტადიაში, როცა სიფილისის კლინიკური გამოვლინება ძალიან გავს ბევრი სხვა დაავადების კლინიკას; თუმცა შენი ქეისის პასუხი არამგონია სიფილისი იყოს. ჰაუსს უფრო კარგი მინიშნებებიც აქვს ხოლმე by the way smile.gif

Posted by: Thandrus 11 Apr 2009, 00:01
Guardian

ანუ სურათზე კანი შეიძლება რომ არ იყოს? wink.gif
* * *
კანის T-cell Lyphoma ხომ არ არის?

Posted by: Guardian 11 Apr 2009, 13:35
სწორი პასუხია - მელანომა, ზედაპირულად გავრცელებადი ფორმა.

H&E-თ შეღებილ მიკროსკოპულ სურათზე მელანომის უჯრედები შავი სულაც არაა. biggrin.gif

Posted by: Thandrus 11 Apr 2009, 22:48
Guardian

ჰმ... მეგონა იყო ეს ვარიანტი... ახლა ვნახე არ ყოფილა biggrin.gif

Next, Please!

Posted by: Blind_Torture_Kill 12 Apr 2009, 19:02
სავარაუდოდ სიკვდილის მიზეზი რა შეიძლება ყოფილიყო ?

Posted by: LUKA-BRAZI 12 Apr 2009, 21:17
Blind_Torture_Kill
ჯორჯლის არტერიის თრომბოზი? baby.gif

Posted by: Blind_Torture_Kill 14 Apr 2009, 09:50
LUKA-BRAZI
no.gif

შოკით მოკვდა და შოკი რამ გამოიწვია ?

Posted by: irakli222 14 Apr 2009, 10:44
გასტრო-დუოდენული სისხლდენა?
ჩაღვრილი სისხლი ხომ არ აძლევს ნაწლავებს მუქ ფერს?

Posted by: LUKA-BRAZI 14 Apr 2009, 13:50
Blind_Torture_Kill
მეც ნაწლავების მუქმა ფერმა მაფიქრებინა ჯორჯლის არტერიის თრომბოზი. შეიძლება ნაწლავების ჩაჭედვამ, ან ერთმანეთზე გადაგრეხვამაც გამოიწვიოს ნეკროზი.
მაგრამ შოკი??
ის ყვითელი რღაცეები, ვიფიქრე რომ ცხიმოვანი ქსოვილია და თუ აბსცესებია (რაშიც ეჭვი მეპარება) მაშინ შესაძლოა სიკვდილის მიზეზი სეპტიკური შოკი იყოს. ცოტა სხვა რამეც მიგვანიშნე რა smile.gif ისე ამის მერე უფრო კლინიკური სურთით დახუნძლული, მაგრამ ძნლი ქეისის დადე თუ არ დაგეზაროს smile.gif

Posted by: Blind_Torture_Kill 14 Apr 2009, 16:14
LUKA-BRAZI

smile.gif
ეს ქეისი არ უნდა გაგიჭირდეთ პროსტა სურათით იხელმძღვანელეთ
QUOTE
ჩაღვრილი სისხლი ხომ არ აძლევს ნაწლავებს მუქ ფერს

შოკის გამოა და + მაგისგანაც

QUOTE
ის ყვითელი რღაცეები, ვიფიქრე რომ ცხიმოვანი ქსოვილია და თუ აბსცესებია (რაშიც ეჭვი მეპარება) მაშინ შესაძლოა სიკვდილის მიზეზი სეპტიკური შოკი იყოს

არაა smile.gif

QUOTE
ისე ამის მერე უფრო კლინიკური სურთით დახუნძლული, მაგრამ ძნლი ქეისის დადე თუ არ დაგეზაროს

კი დავდებ
ყველამ მიიღეთ ქეისების დადებაში მონაწილეობა

Posted by: Mrs_Zum 14 Apr 2009, 18:38
ვითომ ტრამვული და ჰემორაგიული? spy.gif
ანუ ტრამვა შეიძლება გამხდარიყო სისხლდენის მიზეზი და ამით დაღუპულიყო....

Posted by: Thandrus 14 Apr 2009, 22:29
დიზენტერიამ და მაგის გამო დიარეამ ხომ არ გამოიწვია ეს შოკი?

Posted by: Blind_Torture_Kill 14 Apr 2009, 23:26
Thandrus
QUOTE
დიზენტერიამ და მაგის გამო დიარეამ ხომ არ გამოიწვია ეს შოკი?


არა

სურათზე რამე განსაკუთრებულს ხედავთ ?

Posted by: Tornike Alashvili 15 Apr 2009, 00:34
ეჰ როგორ არ მიყვარს ეს სახაშე მასალა
მაგრამ ცხადია სქელი ნაცლავი არ გადადის მარჯვნიდან მარცხნივ
თითქოს გაწყვეტილია და თავევი დამალული აქვთ ღრმად
ვიღაცამ დაჯრა ალბათ დანით მაგრამ სისხლში და განავალში იქნებოდა ყველაფერი ავსებული
დაუშვათ -გარეცხეს
Blind_Torture_Kill მოჰკიდე ხელი სქელ ნაქლავს და მიჰყევი -მთლიანია თუ არა?

Posted by: irakli222 15 Apr 2009, 08:20
თუ იმაზე შევთანხმდით , რომ ნაწლავში სისხლია ჩაღვრილი შესაძლო ვარიანტი ასეთია:
პორტული ჰიპერტენზია----საყლაპავის ვენების ვარიკოზული გაგანიერება-----სისხლდენა
ამ ვენებიდან----ჰემორაგიული შოკი----სიკვდილი.


QUOTE
სურათზე რამე განსაკუთრებულს ხედავთ ?


ნაწლავის ჯორჯალში ზედმეტად არის დაგროვილი ცხიმი, რაიმე კავშირშია ეს პორტულ
ჰიპერტენზიასთან თუ არა ვერ ვიტყვი.
ჰიპერტენზია გამოწველია ღვიძლის ციროზით თუ სხვა დაავადებით ამასაც ვერ ვაკონკრეტებ.
* * *
სამსახურში რომ მივედი ჩემს უფროსს ვაჩვენე ეს სურათი

მისი აზრია პანკრეონეკროზი და მის ფონზე განვითარებული შოკი

Posted by: Blind_Torture_Kill 15 Apr 2009, 21:57
QUOTE
მისი აზრია პანკრეონეკროზი და მის ფონზე განვითარებული შოკი


ცხიმის ნეკროზია
განვითარებული მწვავე ჰემორაგიული პანკრეატიტის დროს
სურათზე ჩანს მონელებული ჯორჯლის ცხიმის ნარჩენები

Posted by: LUKA-BRAZI 16 Apr 2009, 14:58
Blind_Torture_Kill
მონელებული ცხიმი თუ იყო ეგ ვერც კი წარმოვიდგენდი smile.gif
irakli222
QUOTE
სამსახურში რომ მივედი ჩემს უფროსს ვაჩვენე ეს სურათი
მისი აზრია პანკრეონეკროზი და მის ფონზე განვითარებული შოკი

up.gif შენს უფროსს biggrin.gif

კარგით ეხლა მე დავდებ სუფთა კლინიკურ ქეისს:

საავადმყოფოში მოვიდა 50 წლის მამაკაცი, პერიოდულად აღენიშნება ტკივილი მუცელში, სისხლიანი განავალი, რასაც ბოლო პერიოდში დაერთო ასთმური შეტევები. ანამნეზში - ორჯერ გადატანილი მიკარდიუმის ინფარქტი. ფილტვების აუსკულტაციით მოისმინება მშრალი მსტვინავი ხიხინი. მუცელი პალპაციით მგრძნობიარეა, ვლინდება პერიტონეუმის გაღიზიანების სიმპტომები. პაციენტს ასევე არენიშნება ჰიპერტენზიის ნიშნები. სისხლში HB - 100 გ/ლ, ლეიკოციტოზი, ეოზინოფილები - 30%.
დასვით დიაგნოზი (მკურნალობასაც თუ მოაყოლებთ კარგი ხალხი იქნებით biggrin.gif)

რახან სურატების გარეშე ვერ ძლებთ, დავდებ ერთ სურათს smile.gif
პრეპარატზე თირკმლის ქსოვილია.

Posted by: Blind_Torture_Kill 16 Apr 2009, 22:39
LUKA-BRAZI

ვასკულიტს გავს
სურათზე ვერაფერს ვერ ვარჩევ და სხვას თუ დადებ კარგი იქნება

Posted by: LULA_QABABI 16 Apr 2009, 23:34
QUOTE (LUKA-BRAZI @ 16 Apr 2009, 05:58 )
Blind_Torture_Kill
მონელებული ცხიმი თუ იყო ეგ ვერც კი წარმოვიდგენდი smile.gif
irakli222
QUOTE
სამსახურში რომ მივედი ჩემს უფროსს ვაჩვენე ეს სურათი
მისი აზრია პანკრეონეკროზი და მის ფონზე განვითარებული შოკი

up.gif შენს უფროსს biggrin.gif

კარგით ეხლა მე დავდებ სუფთა კლინიკურ ქეისს:

საავადმყოფოში მოვიდა 50 წლის მამაკაცი, პერიოდულად აღენიშნება ტკივილი მუცელში, სისხლიანი განავალი, რასაც ბოლო პერიოდში დაერთო ასთმური შეტევები. ანამნეზში - ორჯერ გადატანილი მიკარდიუმის ინფარქტი. ფილტვების აუსკულტაციით მოისმინება მშრალი მსტვინავი ხიხინი. მუცელი პალპაციით მგრძნობიარეა, ვლინდება პერიტონეუმის გაღიზიანების სიმპტომები. პაციენტს ასევე არენიშნება ჰიპერტენზიის ნიშნები. სისხლში HB - 100 გ/ლ, ლეიკოციტოზი, ეოზინოფილები - 30%.
დასვით დიაგნოზი (მკურნალობასაც თუ მოაყოლებთ კარგი ხალხი იქნებით biggrin.gif)

რახან სურატების გარეშე ვერ ძლებთ, დავდებ ერთ სურათს smile.gif
პრეპარატზე თირკმლის ქსოვილია.

Churg-Srauss გავს;

მკურნალობა სისტემური გლუკოკორტიკოიდებით

Posted by: LUKA-BRAZI 17 Apr 2009, 00:13
Blind_Torture_Kill
QUOTE
ვასკულიტს გავს

yes.gif და რომელი ვასკულიტი მაინც? მაგის პათოგნომური სურათი რომ დავდო, ეგრევე მიხვდებით, ამიტომ ამჯერად კლინიკით გამოიცანი smile.gif
LULA_QABABI
ლადოს გაუმარჯოს, როგორ ხარ? smile.gif
QUOTE
Churg-Srauss გავს

yes.gif გავს, მაგრამ მაინც ცოტა სხვაა smile.gif ისე ერთი პერიოდი Churg-Srauss syndrome მის ერთერთ ტიპად ითვლებოდა, იმიტომ რომ ძალიან გავს.
QUOTE
მკურნალობა სისტემური გლუკოკორტიკოიდებით

das ist gut smile.gif

მოკლედ ორივე ახლოს ხართ და ერთი ნაბიჯი გიკლიათ smile.gif მიგანიშნებთ - ყურადღება მიაქციეთ გადატანილ ინფარქტებს. პაციენტს ასევე შესაძლოა ქონოდა Stroke, Kidney failure და Intestinal necrosis and perforation.....


P.S. რა უბედურებაა ასე "სრაზუ" ქეისის გახსნა? biggrin.gif გეცლიათ ცოტა smile.gif

Posted by: Blind_Torture_Kill 17 Apr 2009, 00:26
polyarteritis nodosa მაშინ
მახსოვს ადრეც რაღაც ამდაგვარი ქეისი იყო ხოდა იქაც მივანიშნეთ რომ პილტვების ჩათრევა ძაან იშვიათია
ასთმური სურათი ჩურგ სტრაუს უფრო აქვს

Posted by: LUKA-BRAZI 17 Apr 2009, 00:45
Blind_Torture_Kill
QUOTE
polyarteritis nodosa მაშინ

უსინდისოდ მართალი ხარ! lol.gif
up.gif
QUOTE
მახსოვს ადრეც რაღაც ამდაგვარი ქეისი იყო ხოდა იქაც მივანიშნეთ რომ პილტვების ჩათრევა ძაან იშვიათია ასთმური სურათი ჩურგ სტრაუს უფრო აქვს

მაგიტომაც იყო LULA_QABABI ძალიან ახლოს პასუხთან და მაგიტომაც ითვლებოდა Churg-Srauss-ი კვანძოვანის ერთერთ ტიპად. კვანძოვანის დროსაც ხდება ფილტვის ჩათრევა და ასთმური მოვლენებიც არის დამახასიათებელი.

ფუ რა smile.gif აი მოვიფიქრებ ქეისს ისევ პრიონებზე და მერე ვნახავ მაგასაც 2 პოსტში თუ გამოიცნობ biggrin.gif

სად არის ძველი ხალხი? რატომ აღარ არის ეს თემა მწვავე?! cry.gif სადაა vano_t ?! ვანო დადე ქეისი, ლადო დადე ქეისი, დათო დადე ქეისი, Guardian შენ ან კლინიკური დადე რამე ან მოეშვი ჩვენს ტანჯვას პათოლოგიით gigi.gif ვანოო სად ხარ!!!! Cousteau-მ ქეისი ბოლოს როდის დადო? გახსოვთ ვინმეს? ხო ეკუთვნის ვორნი 120% არააქტიურობისათვის? biggrin.gif

Posted by: LULA_QABABI 17 Apr 2009, 07:07
QUOTE (LUKA-BRAZI @ 16 Apr 2009, 15:45 )
Blind_Torture_Kill
QUOTE
polyarteritis nodosa მაშინ

უსინდისოდ მართალი ხარ! lol.gif
up.gif
QUOTE
მახსოვს ადრეც რაღაც ამდაგვარი ქეისი იყო ხოდა იქაც მივანიშნეთ რომ პილტვების ჩათრევა ძაან იშვიათია ასთმური სურათი ჩურგ სტრაუს უფრო აქვს

მაგიტომაც იყო LULA_QABABI ძალიან ახლოს პასუხთან და მაგიტომაც ითვლებოდა Churg-Srauss-ი კვანძოვანის ერთერთ ტიპად. კვანძოვანის დროსაც ხდება ფილტვის ჩათრევა და ასთმური მოვლენებიც არის დამახასიათებელი.

ფუ რა smile.gif აი მოვიფიქრებ ქეისს ისევ პრიონებზე და მერე ვნახავ მაგასაც 2 პოსტში თუ გამოიცნობ biggrin.gif

გაუმარჯოს ლუკას,

რაზეც ვერ დავდებ თავს პათანატომიური სურათის ინტერპრეტაციაა;
ჩარგ სტრაუსი 2-3 ჯერ მაქვს ნანახი; პან - ერთხელ; ორივე კარგა ხნის წინ;

მახსოვდა რომ პერიფერული ეოზინოფილია და ასთმა ჩარგ სტრაუსს ახასიათებს და არა პან-ს; მიოკარდის ინფარქტის დეტალები არ მახსოვდა;
გადავხედე review მასალებს (harrison და uptodate-ს) ;
პერიფერული ეოზინოფილია და ასთმა პნ-ს მართლაც არ ახასიათებს; მიოკარდის ინფარქტიც კი მეტნაკლებად ორივეს ახასიათებს, მაგრამ აქაც "უპირატესობა" ჩარგ-სტრაუსისკენ არის (39% სიკვდილის მიზეზი, ჰარისონის მიხედვით); uptodate-ს ვენდობით საკმაოდ იშვიათია პან-ის დროს;

ასე რომ რაღაც შეუსაბამობაა ქეისის ტექსტურ და ვიზუალურ (რომელიც იმედია მართლაც არის პნ ) ნაწილში; ჩემი პასუხი იგივე რჩება, ჩარგ-სტრაუსი - givi.gif
ისე თუ არ დაგვზარდები ორიგინალის ლინკიც დადო

აქვე დავდებ ჰარისონიდან-ს ამონარიდებს; და ცხრილს პან-ის კლინიკურ გამოვლინებებზე, სადაც შეამჩნევთ რომ სასუნქი სისტემა არ შედის ჩამონათვალში;

Churg-Strauss Syndrome

Definition
Churg-Strauss syndrome, also referred to as allergic angiitis and granulomatosis, was described in 1951 by Churg and Strauss and is characterized by asthma, peripheral and tissue eosinophilia, extravascular granuloma formation, and vasculitis of multiple organ systems.

Pathology and Pathogenesis
The necrotizing vasculitis of Churg-Strauss syndrome involves small and medium-sized muscular arteries, capillaries, veins, and venules. A characteristic histopathologic feature of Churg-Strauss syndrome is granulomatous reactions that may be present in the tissues or even within the walls of the vessels themselves. These are usually associated with infiltration of the tissues with eosinophils. This process can occur in any organ in the body; lung involvement is predominant, with skin, cardiovascular system, kidney, peripheral nervous system, and gastrointestinal tract also commonly involved. Although the precise pathogenesis of this disease is uncertain, its strong association with asthma and its clinicopathologic manifestations, including eosinophilia, granuloma, and vasculitis, point to aberrant immunologic phenomena.

Clinical and Laboratory Manifestations
Patients with Churg-Strauss syndrome often exhibit nonspecific manifestations such as fever, malaise, anorexia, and weight loss, which are characteristic of a multisystem disease. The pulmonary findings in Churg-Strauss syndrome clearly dominate the clinical picture with severe asthmatic attacks and the presence of pulmonary infiltrates. Mononeuritis multiplex is the second most common manifestation and occurs in up to 72% of patients. Allergic rhinitis and sinusitis develop in up to 61% of patients and are often observed early in the course of disease. Clinically recognizable heart disease occurs in ~14% of patients and is an important cause of mortality. Skin lesions occur in ~51% of patients and include purpura in addition to cutaneous and subcutaneous nodules. The renal disease in Churg-Strauss syndrome is less common and generally less severe than that of Wegener's granulomatosis and microscopic polyangiitis.
The characteristic laboratory finding in virtually all patients with Churg-Strauss syndrome is a striking eosinophilia, which reaches levels >1000 cells/ L in >80% of patients. Evidence of inflammation as evidenced by elevated ESR, fibrinogen, or 2-globulins can be found in 81% of patients. The other laboratory findings reflect the organ systems involved. Approximately 48% of patients with Churg-Strauss syndrome have circulating ANCA that is usually antimyeloperoxidase.

Diagnosis
Although the diagnosis of Churg-Strauss syndrome is optimally made by biopsy in a patient with the characteristic clinical manifestations (see above), histologic confirmation can be challenging as the pathognomonic features often do not occur simultaneously. In order to be diagnosed with Churg-Strauss syndrome, a patient should have evidence of asthma, peripheral blood eosinophilia, and clinical features consistent with vasculitis.
=========================================================================



Polyarteritis Nodosa

Definition

PAN, also referred to as classic PAN, was described in 1866 by Kussmaul and Maier. It is a multisystem, necrotizing vasculitis of small and medium-sized muscular arteries in which involvement of the renal and visceral arteries is characteristic. PAN does not involve pulmonary arteries, although bronchial vessels may be involved; granulomas, significant eosinophilia, and an allergic diathesis are not observed.

Pathology and Pathogenesis

The vascular lesion in PAN is a necrotizing inflammation of small and medium-sized muscular arteries. The lesions are segmental and tend to involve bifurcations and branchings of arteries. They may spread circumferentially to involve adjacent veins. However, involvement of venules is not seen in PAN and, if present, suggests microscopic polyangiitis (see below). In the acute stages of disease, polymorphonuclear neutrophils infiltrate all layers of the vessel wall and perivascular areas, which results in intimal proliferation and degeneration of the vessel wall. Mononuclear cells infiltrate the area as the lesions progress to the subacute and chronic stages. Fibrinoid necrosis of the vessels ensues with compromise of the lumen, thrombosis, infarction of the tissues supplied by the involved vessel, and, in some cases, hemorrhage. As the lesions heal, there is collagen deposition, which may lead to further occlusion of the vessel lumen. Aneurysmal dilatations up to 1 cm in size along the involved arteries are characteristic of PAN. Granulomas and substantial eosinophilia with eosinophilic tissue infiltrations are not characteristically found and suggest Churg-Strauss syndrome (see above).

Multiple organ systems are involved, and the clinicopathologic findings reflect the degree and location of vessel involvement and the resulting ischemic changes. As mentioned above, pulmonary arteries are not involved in PAN, and bronchial artery involvement is uncommon. The pathology in the kidney in classic PAN is that of arteritis without glomerulonephritis. In patients with significant hypertension, typical pathologic features of glomerulosclerosis may be seen. In addition, pathologic sequelae of hypertension may be found elsewhere in the body.

The presence of hepatitis B antigenemia in ~10–30% of patients with systemic vasculitis, particularly of the PAN type, together with the isolation of circulating immune complexes composed of hepatitis B antigen and immunoglobulin, and the demonstration by immunofluorescence of hepatitis B antigen, IgM, and complement in the blood vessel walls, strongly suggest the role of immunologic phenomena in the pathogenesis of this disease. Hairy cell leukemia can be associated with PAN; the pathogenic mechanisms of this association are unclear.

Clinical and Laboratory Manifestations

Nonspecific signs and symptoms are the hallmarks of PAN. Fever, weight loss, and malaise are present in over one-half of cases. Patients usually present with vague symptoms such as weakness, malaise, headache, abdominal pain, and myalgias that can rapidly progress to a fulminant illness. Specific complaints related to the vascular involvement within a particular organ system may also dominate the presenting clinical picture as well as the entire course of the illness (Table 319-5). In PAN, renal involvement most commonly manifests as hypertension, renal insufficiency, or hemorrhage due to microaneurysms.

There are no diagnostic serologic tests for PAN. In >75% of patients, the leukocyte count is elevated with a predominance of neutrophils. Eosinophilia is seen only rarely and, when present at high levels, suggests the diagnosis of Churg-Strauss syndrome. The anemia of chronic disease may be seen, and an elevated ESR is almost always present. Other common laboratory findings reflect the particular organ involved. Hypergammaglobulinemia may be present, and up to 30% of patients have a positive test for hepatitis B surface antigen. Antibodies against myeloperoxidase or proteinase-3 (ANCA) are rarely found in patients with PAN.

Diagnosis

The diagnosis of PAN is based on the demonstration of characteristic findings of vasculitis on biopsy material of involved organs. In the absence of easily accessible tissue for biopsy, the angiographic demonstration of involved vessels, particularly in the form of aneurysms of small and medium-sized arteries in the renal, hepatic, and visceral vasculature, is sufficient to make the diagnosis. Aneurysms of vessels are not pathognomonic of PAN; furthermore, aneurysms need not always be present, and angiographic findings may be limited to stenotic segments and obliteration of vessels. Biopsy of symptomatic organs such as nodular skin lesions, painful testes, and nerve/muscle provides the highest diagnostic yields.
============================================================
მე ჩემი საკუთარი ქეისები (ინფექციური) დავდო პერიოდულად;

Posted by: LUKA-BRAZI 17 Apr 2009, 11:55
LULA_QABABI
ნუ შენ შანსს არ მიტოვებ biggrin.gif ვერ შეგეწინააღმდეგები, იმიტომ რომ რაც შენ დაწერე ჰარისონიდან იყო, ხოლო რაც მე, ტატიშვილის შინაგანი სნეულებანი user.gif რას იზამ, ჩამოვრჩებით აქ თანამედროვე მედიცინას user.gif
QUOTE
ისე თუ არ დაგვზარდები ორიგინალის ლინკიც დადო

არ დაგზარდებოდი ლადო, მაგრამ წიგნიდან გადმოვწერე, ტატიშვილის ტესტებიდან, უბრალოდ იქ სავარაუდო პასუხებიც იყო მითითებული და ისენი აღარ დავწერე. სავარაუდო პასუხები კი იყო:
* რევმატოიდული ართრიტი
*სისტემური სკლეროდერმაი
*სისტემური წიტელი მგლურა
*კვანძოვანი პერიარტერიიტი
*დერმატომიოზიტი

სწორ პაუხებში კი ეწერა რომ სწორი პასუხი იყო კვანძოვანი პერიარტერიიტი და მართლაც, ჩამოთვლილთაგან ყველაზე მეტად ეგ შეესაბამება მოცემულ კლინიკას. თუმცა ჰარისონში ნათლად წერია ყველაფერი. ასე რომ აღარ ვიცი, სწორი პასუხი ჩაგ-სტრაუსის სინდრომია თუ PAN. ჩემი წყაროს მიხედვით PAN, მაგრამ ჰარისონის მიხედვით ჩაგ-სტრაუსის. რახან ჰარისონი ჯობია ტატიშვილს (მარტივი ლოგიკაა smile.gif), ე.ი. ჩაგ-სტრაუსის სინდრომი ყოფილა სწორი პასუხი.
smile.gif
* * *
LULA_QABABI
QUOTE
მე ჩემი საკუთარი ქეისები (ინფექციური) დავდო პერიოდულად;

დადე, დადაე, კარგს იზამ yes.gif ინფექციურზე ბევრი ქეისი არასდროს დადებულა, ამიტომ შენ დადე ხოლმე smile.gif

Posted by: Romina 17 Apr 2009, 18:51
თქვენი ნებართვით ერთ კლინიკურ შემთხვევას დავდებ. ძალიან მარტივია.

პაციენტი X, ქალი 49 წლის. დაავადება დაიწყო კუნთოვანი სისუსტით, რომელიც პროგრესირებდა, 3-4 დღეში ამ სიმპტომს დაემატა გამონაყარი. როცა კლინიკაში მოვიდა ჰიპერემიული კანის ფონზე აღინიშნებოდა პოლიმორფული გამონაყარი, მთელ სხეულზე. გარდა ამისა გამოხატული იყო პერიორბიტული შეშუპება, მოლურჯო-მოიისფრო ელფერით.
საეჭვო დიაგნოზი შეხედვისთანავე დაისვა და მოგვიანებით ლაბორატორიულადაც დადასტურდა. თქვენი აზრით რა დაავადებაა?

Posted by: LUKA-BRAZI 17 Apr 2009, 19:03
Romina
დერმატომიოზიტი............

Posted by: Romina 17 Apr 2009, 19:06
LUKA-BRAZI
კი smile.gif
სანამ ჩვენამდე მოაღწევდა ალერგოლოგთან მკურნალობდა 1 თვე, მერე იმან კარდიოლოგთან გაუშვა და კარდიოლოგი მიხვდა საბოლოოდ :-)

Posted by: LUKA-BRAZI 17 Apr 2009, 19:08
Romina
smile.gif
მე კი იმით მივხვდი, რომ შენ რევმატოლოგი ხარ და ქეისსაც რევმატოლოგიურს დადებდი smile.gif წიგნში პატარა გადახედვა და სწორი პასუხიც მზად იყო smile.gif

P.S. 1000-ე საიუბილეო პოსტი!

Posted by: Romina 17 Apr 2009, 19:16
LUKA-BRAZI

მარტივია ძალიან. იმ იშვიათ დაავადებებს ეკუთვნის, რომელთა დიაგნოზი შეხედვითაც შეიძლება დასვა (თუ კლასიკური შემთხვევაა რა თქმა უნდა). სტუდენტობის დროსაც რაღაცნაირად გამახსოვრდება "დერმატომიოზიტური სათვალე" :-)

Posted by: LUKA-BRAZI 17 Apr 2009, 19:24
Romina
QUOTE
მარტივია ძალიან. იმ იშვიათ დაავადებებს ეკუთვნის, რომელთა დიაგნოზი შეხედვითაც შეიძლება დასვა (თუ კლასიკური შემთხვევაა რა თქმა უნდა). სტუდენტობის დროსაც რაღაცნაირად გამახსოვრდება "დერმატომიოზიტური სათვალე" :-)

yes.gif
"დერმატომიოზიტური სათვალე", "მეანის ხელები", "დოლის ჩხირისებრი თითები", "მეწაღის გულმკერდი", "სარდონიკული ღიმილი", "ჰიპოკრატეს სახე", "ბერლინის ლაჟვარდი", "კატის კრუტუნის სინდრომი" და ა.შ.
biggrin.gif

Posted by: Romina 17 Apr 2009, 19:34
LUKA-BRAZI
biggrin.gif biggrin.gif
"გულის კუზი"; "ქისის სიმპტომი" "დაკბილული ბორბლის ფენომენი".........მეტი არ მახსენდება, თითქმის ყველა ჩამოთვალე :-)

Posted by: LUKA-BRAZI 17 Apr 2009, 19:46
შევაგროვოთ ყველაფერი, განმარტებებიც დავურთოთ და გამოვცეთ წიგნი. ფასი 20 ლარი, ავტოგრაფით 25 smile.gif
უი ეხლა მოვა მთვარეული და გაგვიბრაზდება თემის გაოფფტოპიკების გამო.
smile.gif

Posted by: Romina 17 Apr 2009, 19:59
LUKA-BRAZI
მართალიც იქნება. ამიტო გავჩუმდეთ
biggrin.gif

Posted by: Blind_Torture_Kill 18 Apr 2009, 01:41
42 წლის ქალბატონს რომელსაც აღენიშნებოდა პროტეინურია 5 წლის განმავლობაში განუვითარდა თირკმლის უკმარისობა. ასევე აწუხებდა ჰიპერტენზია და ბოლო 10 წლის განმავლობაში გადაიტანა მრავლობითი სოკოვანი და ბაქტერიული ინფექციები. ქონდა 2 გაურთულებელი ორსულობა. ორივეზე გაკეთდა საკეისრო. ნაყოფის წონა ერთზე იყო - 4.05 მეორეზე 4.5კგ. თირკმლის ბიოფსიის სურათი მოცემულია ქვემოთ

დიაგნოზი ?



P.S. ლუკა აი შენ ქეისი როგორსაც ითხოვდი

Posted by: Thandrus 18 Apr 2009, 06:32
Blind_Torture_Kill

დიაბეტი ჰქონდა? ახსნიდა ინფექციებს, თირკმლის უკმარისობასა და და ნაყოფის ასეთ დიდ წონას.

Posted by: LUKA-BRAZI 18 Apr 2009, 10:54
Blind_Torture_Kill
QUOTE
ლუკა აი შენ ქეისი როგორსაც ითხოვდი

დანქე smile.gif
Thandrus-ს ვეთანხმები. დიაბეტი... დაიბეტური ნეფროპათია, იმუნიტეტის მოშლა, სოკოვანი (რაც პრინციპში ოპორტუნისტულ ინფექციად შეიძლება მივიჩნიოთ იმუნური სტატუსის დაქვეითების დროს ჰო?) და ბაქტერიული ინფექციები. ერთ-ერთი ასეთი ინფექციის დროს, შესაძლოა განუვითარდა ბაქტერიული გლომერულონეფრიტი, რამაც დაუტოვა პროტეინურია. ჰიპერტენზიაც ამით აიხსნება, ასევე მშობიარობასთან დაკავშირებული მოვლენები.

Posted by: Blind_Torture_Kill 18 Apr 2009, 17:29
Thandrus
ეგაა

LUKA-BRAZI
აბა ეხლა ეს სურათი ამიღწერეთ smile.gif
და ამ ცვლილებიდან რომელი რის გამოა

Posted by: LUKA-BRAZI 18 Apr 2009, 18:40
Blind_Torture_Kill
ვერ ვიტან ჰისტოლოგიურ სურათებს, იმიტომ რომ ჰისტოლოგია და პათანატომია ქსეროქსებიდან გვასწავლეს user.gif
არ ვიცი ეგენი რომელ პათოლოგიურ სურათებს შეესაბამება, მაგრამ მარცხენა და მარჯვენა ბოლო ერთმანეთს გავს, მარჯვენა ზედა ორი კი სხვა მექანიზმით განვითარებული..... რეავიცი, რეავიცი. ეგ ალბათ ინფილტრატებია გაჭედილი ბოუმანის კაფსულაში და დანარჩენში ნახევარმთვარისებრი გლომერულოპათიაა?

Posted by: Blind_Torture_Kill 18 Apr 2009, 20:45
აბა დადეთ რამე კარგი და საინტერესო

Posted by: mtvareuli 18 Apr 2009, 20:47
Blind_Torture_Kill

საკმაოდ პატარა სურათია და ძნელი გასარჩევია სტრუქტურები, მაგრამ მარცხენა და მარჯვენა ქვედა ისრები დიაბეტურ კვანძოვან გლომერულოსკლეროზზე უნდა მიუთითებდნენ. მარჯვენა შუა-დიაბეტური არტერიოსკლეროზია.

Posted by: Blind_Torture_Kill 18 Apr 2009, 22:58
mtvareuli

ჰო ეგაა
რამე კარგ მამალ ქეისს რომ გადავაწყდები დავდებ

Posted by: mtvareuli 18 Apr 2009, 23:00
Blind_Torture_Kill
QUOTE
რამე კარგ მამალ ქეისს რომ გადავაწყდები დავდებ

მანამდე მე დავდებ ძალიან ძალიან მარტივს biggrin.gif


Posted by: LUKA-BRAZI 18 Apr 2009, 23:05
mtvareuli
წინა კედლის მწვავე ინფარქტი...... ნუ თუ ჯერ ინფარქტი არ არის ჩამოყალიბებული, იაშემია წინა კედელზეა

Posted by: mtvareuli 18 Apr 2009, 23:10
LUKA-BRAZI

no.gif


--------------------

Posted by: LUKA-BRAZI 18 Apr 2009, 23:15
mtvareuli
როგორ ST ელევაციებია მანდ და....... ?
მაშინ სხვა სიტყვებით იგივე მოვლენა - მწვავე კორონარული სინდრომი?
smile.gif
ნუ ჰო ექსტრებიც არის მგონი წინაგულოვანი

Posted by: mtvareuli 18 Apr 2009, 23:19
LUKA-BRAZI

ST ელევაციები კი არის, მაგრამ

წინა კედლის ინფარტის დროს რომელ განხრებში უნდა იყოს ეგ ელევაციები?

და ელევაციების გარდა რა უნდა იყოს?


Posted by: DrJohnes 18 Apr 2009, 23:25
mtvareuli

ვახ, შემოვიხედე და დავინტრიღდი...
ჰიპოკალიემიამ არ იცის თ კბილების ელევაცია?, თუ მეშლება?...
* * *
ჰოდა კიდე C განხრები, რეებია, ვილსონებია? თუ ასეა, მაშინ მარცხნისკენაა უფრო გამოხატული...

Posted by: mtvareuli 18 Apr 2009, 23:34
DrJohnes
QUOTE
ჰიპოკალიემიამ არ იცის თ კბილების ელევაცია?

არა ჰიპოკალემიის დროს პირიქით, T კბილები პატარაა ან შეიძლება სულ არ იყოს გამოხატული და თან ST სეგმენტის ელევაცია კიარა, ოდნავი დეპრესიაა ამ დროს.

T კბილები ჰიპერკალემიის დროსაა დიდი, მაგრამ ამ კარდიოგრამაზე T-ს კიარა, ST სეგმენტის ელევაციებია. ჰიპერკალემიის დროს კი ST სეგმენტი შემცირებულია ან სულ არაა გამოხატული.. და სხვა რაღაცეებიც ახასიათებს კიდე, მაგ QRS კომპლექსის გაფართოება.

მოკლედ აქ არც ჰიპერკალემიაა, არც ჰიპოკალემია

არც მიოკარდიუმის მწვავე ინფარქტი (არც მწვავე კორონარული სინდრომი)


QUOTE
ჰოდა კიდე C განხრები, რეებია, ვილსონებია?

ეგ მართლა არ ვიცი C-ები რატო წერია V-ების მაგივრად user.gif

Posted by: DrJohnes 18 Apr 2009, 23:40
mtvareuli

თუ იშემიური არაა მაშინ მოწამვლა იქნება, არა?.. ups.gif
* * *
და თუ Iშემიაა, მაშინ ძგიდის იშემია

Posted by: mtvareuli 18 Apr 2009, 23:48
DrJohnes
QUOTE
და თუ Iშემიაა, მაშინ ძგიდის იშემია

რატო?

Posted by: DrJohnes 18 Apr 2009, 23:55
mtvareuli

იმიტომ რომ ელევაცია უფრო მეტად გამოხატულია C2-C6-ში და II განხრებში
მასრა ვცდები ალბათ, ჰო user.gif
ამას ჰქვია: вот почему я стал психиатром gigi.gif

Posted by: mtvareuli 18 Apr 2009, 23:59
DrJohnes
QUOTE
ელევაცია უფრო მეტად გამოხატულია C2-C6-ში და II განხრებში

ხო, მაგრამ რეციპროკულ განხრებში უფრო ნაკლები ელევაცია კიარა, დეპრესია უნდა იყოს ინფარქტის შემთხვევაში

აქ aVR-ის გარდა ყველა განხრაშია ელევაცია
რომელი დაავადებისთვისაა ეს დამახასიათებელი?


QUOTE
ამას ჰქვია: вот почему я стал психиатром

biggrin.gif 2kiss.gif

Posted by: DrJohnes 19 Apr 2009, 00:26
mtvareuli
QUOTE
რომელი დაავადებისთვისაა ეს დამახასიათებელი?


ტაკს, მე აზზე არ ვარ რისი პათოგნომური ნიშანია ეს, მაგრამ ფაქტია, რომ თუ AVR-ში არაა ელევაცია ესე იგი პროცესი მარჯვნივ არა და მარცხნივაა და რახან არც იშემიაა და არც მოწამვლა და რაღაც უცნაურია ესე იგი იდეოპათიურია (ან აუტოიმუნურია) © House M.D. biggrin.gif იმის გამო რომ აუტოიმუნური მეეჭვება, ესე იგი იდეოპათიურია და ალბათ კარდიომიოპათიასთან გვაქვს საქმე.

მაინც არა, ჰო? user.gif

Posted by: zviadcardio 19 Apr 2009, 00:53
პერიკარდიტი - მთვარე ექიმო...
^^^^^^^^^^^^^^^^^^^^^^^^^^^^

Posted by: mtvareuli 19 Apr 2009, 01:10
zviadcardio

თქვენ კი გამოიცნობდით user.gif

QUOTE
QRS-ში ჩაფლული...

biggrin.gif

Posted by: DICOM 19 Apr 2009, 01:15
mtvareuli

კაი რა ამას ხომ მეც გამოვიცნობ რაა biggrin.gif ეგი არის ინფაქტიიიი biggrin.gif

Posted by: DrJohnes 19 Apr 2009, 02:01
ეგრეც ვიცოდი, რომ ვცდებოდი, პერიკარდიტის კარფიოგრაფიულ ნიშნებში ნამდვილად ვერ ვერკვევი. სამაგიეროდ ერკვევა ფორუმის მთავარი კარდიოლოგი zviadcardio smile.gif

ახლა მე შემოგთავაზებთ ქეისს:
მამაკაცი, 83 წლის, მოყვანილ იქნა გადაუდებელი დახმარების ცენტრში პრეკომატოზურ მდგომარეობაში (somnulatio), რომელიც განვითარდა უეცრად ამავე დილით. ასევე აღენიშნებოდა გულისრევა, პირებინებით და პრაქტიკულად არ ჰქონდა დიურეზი. ანამნეზში აღენიშნება სარკოიდოზი ფილტვების (ემფიზემა, ბრონქოექტაზები) და თირკმლების (პროტეინურია) დაზიანებით. გულ-სისხლძარღვთა სისტემის მხრივ ჰისის კონის მარცხენა ფეხის ბლოკადა და დროთა განმავლობაში ჩამოყალიბებული მარჯვენამხრივი გულის უკმარისობა. კუჭ-ნაწლავის მხრივ აქვს დიდი მოცულობის ჩაჭედილი საზარდულის თიაქარი (რენტგენოლოგიურად დადგინდა რომ თიაქრის შიგთავს წარმოადგენს სიგმოიდური ნაწლავი). აქვს ქრონიუკული, მაგრამ ნაწილობრივი გაუვალლობა. ასევე ბოლო დროს უჩიოდა მწვავე ტკივილს ეპიგასტრიუმის არეში.
გადაუდებელი დახმარების ცენტრის ექიმთა ძალისხმევის და მიუხედავად პაციენტი გარდაიცვალა დაახლოებით 12 საათის განმავლობაში
კითხვა: სიკვდილის სავარაუდო მიზეზი

Posted by: Thandrus 19 Apr 2009, 02:54
ჩემი აზრით, სისხლიდან ცილების კარგვამ გამოიწვია სისხლძარღვებში სისხლის მოცულობის შემცირება და წნევის ვარდნა, რასაც ჰისისკონაბლოკირებულმა გულმა ვერ გაუწია სათანადო კომპენსაცია. სისხლის ცუდი მიმოქცევა ახსნიდა პრეკომატოზურ მდგომარეობას. სიკვდილის სავარაუდო მიზეზი, ჩემი აზრით, არის გულის დეკომპენსაცია. თიაქარს და სარკოიდოზს არამგონია პირდაპირ მოეკლა smile.gif

Posted by: LUKA-BRAZI 19 Apr 2009, 10:57
გილოცავთ ყველას აღდგომის ბრწყინვალე დღესასწაულს! მრავალს დაესწარით, რაც მთავარია ჯანმრთელნი და შემდეგ ყველაფერ კარგს გისურვებთ smile.gif


Thandrus, DrJohnes
QUOTE
ჩემი აზრით, სისხლიდან ცილების კარგვამ გამოიწვია სისხლძარღვებში სისხლის მოცულობის შემცირება და წნევის ვარდნა

და სისხლიდან ცილების კარგვა პროტეინურიამ გამოიწვია? პროტეინურიას შეუძლია მასეთი რამე ქნას? პრინციპში 83 წლის მოხუცში, თან მასეთი ანამნეზით არაფერია გამორიცხული. უეცრად განვითარებული პრეკომატოზური მდგომარეობა, ამ შემთხვვაში მართლაც გულის ბრალი უნდა იყოს, სხვა რა გამოიწვევდა მაგას? "რაცხა" მწვავე ტკივილი ეპიგასტრიუმის არეში არ მომწონს მე და არც გულისრევა და პირღებინება smile.gif ვიფიქრებდი მიოკარდიუმის ინფარქტის ატიპიური გამოვლინება ხომ არ არის მეთქი. სარკოიდოზი გულსაც აზიანებს, მაგრამ მანდ ფილტვ-გულის უკმარისობა უფრო იყო აშკარად, ვიდრე გულ-ფილტვის.

zviadcardio
QUOTE
პერიკარდიტი

up.gif up.gif up.gif ეხლა დავაკვირდი, თითქმის ყველა განხრაშია ელევაციები.

DrJohnes

OFFTOPIC:
ზურაბ ბერიასთან ხომ არ მუშაობთ? smile.gif


Posted by: Blind_Torture_Kill 19 Apr 2009, 14:21
QUOTE
პირებინებით და პრაქტიკულად არ ჰქონდა დიურეზი


მწვავე ტუბულარული ნეკროზი განუვითარდა

რა გამოკვლევები და რა მკურნალობა ჩატარდა

Posted by: Thandrus 19 Apr 2009, 14:40
LUKA-BRAZI

QUOTE
სისხლიდან ცილების კარგვა პროტეინურიამ გამოიწვია?


პირიქით, ცილების კარგვამ პროტეინურია smile.gif

Posted by: DrJohnes 20 Apr 2009, 01:04
Blind_Torture_Kill

QUOTE
რა გამოკვლევები და რა მკურნალობა ჩატარდა


გაუკეთდა გადასხმები ჰიპოვოლემიის საწინააგმდეგოდ, და ჩაედგა კათეტერი, რის მიუხედავადაც დიურეზი არ გაუმჯობესებულა.

LUKA-BRAZI
სწორედ ესაა საინტერესო, რამ გამოიწვია ეპიგასტრიული ტკივილი და რა კავშირში იყო სიკვდილთან. ატიპიური სტენოკარდია მართლაც კარგი აზრია

Posted by: LUKA-BRAZI 20 Apr 2009, 23:26
DrJohnes
QUOTE
სწორედ ესაა საინტერესო, რამ გამოიწვია ეპიგასტრიული ტკივილი და რა კავშირში იყო სიკვდილთან. ატიპიური სტენოკარდია მართლაც კარგი აზრია

smile.gif
არის მეორე ვარიანტიც smile.gif გულის პრობლემების გამო, ან სხვა პრობლემების გამო, შესაძლოა მას დანიშნული ქონდა რაიმე სახის ანტიკოაგულანტი ან ანტიაგრეგანტი. ვთქვათ ასპირინი, ჰეპარინი, კარდიომაგნილი (?), ასპირინს და შესაძლოა მის გარეშეც, დროტა განმავლობაში პაციენტს განუვითარდა კუჭის წყლული, რომელმაც შესაძლოა განიცადა პენეტრაცია მეზობელ ქსოვილებში, რის გამოც პაციენტს აღენიშნებოდა ტკივილი ეპიგასტრიუმში, ბოლოს კი მოხდა კუჭის პერფორაცია, განვითარდა სისხლდენა -> კოლაფსი -> ჰიპოტენზია -> ანურია -> კომატოზური მდგომარეობა..... ანუ კლინიკა მეტნაკლებად შეესატყვისება შენს ქეისში მოყვანილ კლინიკურ სურათს....

Posted by: LUKA-BRAZI 22 Apr 2009, 21:37
DrJohnes
სად წახვედი? smile.gif ბლო-ბოლო რა დიაგნოზი დაესვა იმ ადამიანს, ან თუ დაიდო პათანატომიური დასკვნა, ან გამოვიცანით თუ არა etc. რომ დაგვაინტრიგე და აგვაფორიაქე, შემოიხედე კაცო განყოფილებაში biggrin.gif

Posted by: Mrs_Zum 23 Apr 2009, 01:20
მმ სარკოიდოზი , ამის გამო გლუკოკორტიკოსტეროიდებზე იკებოდა ალბათ , ასე რო კუჭის ან თორმეტგოჯას პერფორაცია რომ ყოფილიყო გასაკვირი არ არის, პერიტონიტი ვიტომ, შეიძლება ასაკისა და ჩაჭედილი თიაქრის ფონზე კლასიკური სურათი არ გამოიხატა...


Posted by: lgogokhia 24 Apr 2009, 01:47
ოკ, მე მაქვს ერთი კეისი, რომელზეც მაინტერესებს ხალხის აზრი...

35 წლის ჯანმრთელი პაციენტი, მდედრობითი. 2 კვრის წინ ქონდა მაკულოპაპულარული გამონაყარი (როგორც თვითონ აღწერენ) მენჯის არეში, ლატერალურად, რომელიც განსაკუთრებული შემაწუხებელი ქავილით გამოირჩეოდა.. ჰიდროკორტიზონის ხმარების შედეგად გაუარა გამონაყარმა და ქავილმაც, მაგრამ ამ პერიოდში აღენიშნა საზარდულის ლიმფური კვანძის გადიდება და მტკივნეულობა, რომელმაც 2-3 დღეში გაიარა... იგივე სამსახურში 38 წლის გოგოს აღენიშნა იდენტური სიმპტომები 2 კვირის შემდეგ, ოღონდ ამას უკვე მკლავის გარეთა ზედაპირზე და გაუდიდდა იღლიის ლიმფური კვანძები. პაციენტები არ წარმოადგენენ რისკის ქვეშ მყოფ პირებს რომელიმე დაავადების საწინააღმდეგოდ. დაოჯახებულები, ერთი პარტნიორით. მხოლოდ სამსახურში ხვდებიან ერთმანეთს, სხვა სოციალური კონტაკტი არ აქვთ. ხმარობენ იგივე ტუალეტს, რომელსაც კომპანიის სხვა თანამშრომლები. არცერთი არ არის განსაკუთრებით ალერგიული და უახლოესი 1 თვის მანძილზე არ გაუკეთებიათ არანაირი yard-work სახლში ... ორივე იხენებს, რომ გამონაყრიდან რამდენიმე დღის წინ მიართვეს ვარდები...

მოკლედ, გამოთქვით აზრი და მკითხეთ თუ რამე კონკრეტული შეკითხვა გაქვთ... არაფერი გამოკვლევა არ ჩაუტარებიათ, ამიტომ არანაირი ლაბ.რესულტატები არ მაქვს..

Posted by: Thandrus 24 Apr 2009, 02:05
lgogokhia

სპოროტრიქოზი? (ვარდებით ეგ გადადის...)

Posted by: LUKA-BRAZI 24 Apr 2009, 02:31
lgogokhia
კლინიკა სოკოვან დაავადებებს შეესაბამება, ხოლო Thandrus-ის ვარაუდს ვეთანხმები, ვარდებით გადადის მართლაც.

Sporotrichosis is a disease caused by the infection of the fungus Sporothrix schenckii.[1] This fungal disease usually affects the skin, although other rare forms can affect the lungs, joints, bones, and even the brain. Because roses can spread the disease, it is one of a few diseases referred to as rose-thorn or rose-gardeners' disease.[2] - source Wikipedia

Sporotrichosis is an infection of the skin caused by a fungus, Sporothrix schenckii. This infection-causing fungus is related more closely to the mold on stale bread or the yeast used to brew beer than to bacteria that usually cause infections. The mold is found on rose thorns, hay, sphagnum moss, twigs, and soil. Therefore, the infection is more common among gardeners who work with roses, moss, hay, and soil. - source eMedicine

smile.gif

Posted by: lgogokhia 24 Apr 2009, 02:46
Thandrus
არა smile.gif ვიცოდი რომ მაგას იტყოდით პირველად... ზუსტად მაგიტომ ავღნიშნე ეგ, მეც მასე მეგონა რომ rose thorn disease შეიძლება ყოფილიყო, მაგრამ კლინიკა არ ემთხვევა smile.gif ანუ არ მოვყვები ეხლა კლინიკას, მაგრამ არ ემთხვევა... smile.gif თან 3-4 დღეში ალაგდა ყველაფერი...

სხვა რამე იდეა??? ბანალურ სოკოვან ინფექციად ჩავთვლიდი რომ არა ლიმფური კვანძების გადიდება, რომელიც ამ დროს ნაკლებად ხდება... ანუ როგორც მახსოვს ეგრეა...

Posted by: Thandrus 24 Apr 2009, 03:15
lgogokhia

ჰმ... მაშინ საინტერეოსოა... მუნი ხომ არა? შემაწუხებელი ქავილი, თან ერთი ტუალეტი... მაგრამ მაშინ ლიმფური კვანძების გადიდებაა უცნაური.

სამ-ოთხ დღეში თავისით მორჩა თუ რამე წამლების შემდეგ?

Posted by: lgogokhia 24 Apr 2009, 03:53
Thandrus LUKA-BRAZI
არა მუნის არც ლოკალიზაციაა და არც გამონაყრის ტიპი, თან როგორც თქვი ლიმფური კვანძების გადიდება მაგ დროს ნაკლებად ხდება... თან მუნი ამ პოპულაციაში და გარემოში საკმაოდ უცნაური იქნება... თუმცა გამორიცხული არაფერია occurrence-ის თვალსაზრისით... მარტო ჰიდროკორტიზონის მალამო წაისვეს და მორჩნენ... მაგრამ რაღაც მეეჭვება რომ ასე უბრალოდ ჩაევლო... უბრალოდ წინათგრძნობა მაქვს, თორემ არანაირი საფუძველი...


Posted by: LUKA-BRAZI 24 Apr 2009, 14:41
lgogokhia
მოკლედ ეგენი ექიმთან არც მისულან, რახან აღარაფერი აწუხებთ და არც გამოკვლევები აქვთ ჩატარებული, შესაბამისად დიაგნოზიც არ აქვთ დასმული ხომ? ჰოდა მაშინ ძნელია რამის გამოცნობა. ჰიდროკორტიზონმა რომ უშველა, ეგ სიმპტომური მკურნალობა იყო და არა ეტიოლოგიური. შეამცირა ანთბის და ალერგიის მედიატორები ლოკალურად და ქავილიც გაქრა. მუნი ნამდვილად არ იქნებოდა ჩემი აზრით, მუნს ჰიდროკორტიზონი ვერ მოერევა. მეტი არაფერი აწუხებთ მაგ ხალხს? რა დროა გასული გამონაყრიდან? არაფერი ხელჩასაჭიდი არ არის smile.gif ლიმფური კვანძების ადგილობრივი გადიდება ისევ ლოკალური პროცსზე მიუთითებს, რომელსაც სავარაუდოდ გაუმკლავდა იმუნიტეტი რახან გადიდებამაც გაიარა და მის მერე არაფერს არ შეუწუხებია პაციენტი (ნუ გავლილ დროსაც გააჩნია).
smile.gif

Posted by: Thandrus 24 Apr 2009, 15:50
რამე ისეთი ხომ არ იყო, მაგალითად, რამე ინფექციურ ალერგენზე მისცა რეაქცია, წაისვეს ჰიდროკორტიზონი, ალერგიამ გაუარათ, სამაგიეროდ ცოტატი იმუნიტეტი შესუსტდა და ამ ინფექციამ ლიმფურ კვანძებში შეაღწია მაგრამ სპონტანურად ალაგდა.

Posted by: lgogokhia 24 Apr 2009, 20:22
LUKA-BRAZI
2 კვირაა გასული... ექიმთან არ მისულან მაგრამ მე ჩავაგდე პანიკაში, გავეკაიფე STD უნიტაზზე დასაჯდომიდან აგეკიდათ რაღაცათქო biggrin.gif
Thandrus
smile.gif შეიძლება... მაგრამ ძალიან ბანალური ახსნა იქნება smile.gif აი სოფლის უბნის ექიმის დიაგნოზივით გამომივა smile.gif

Posted by: donvaso 24 Apr 2009, 20:53
სტუდენტებო!
რას ხედავთ??????????

Posted by: LUKA-BRAZI 24 Apr 2009, 23:24
donvaso
ძალიან ზერელედ შევხედე, ვცქარობ და.... smile.gif მგონი კავერნები უნდა იყოს, ანუ ტუბრკულოზია და + კავერნები.

Posted by: donvaso 25 Apr 2009, 00:26
LUKA-BRAZI
yes.gif
...........................

Posted by: Thandrus 25 Apr 2009, 00:38
donvaso

კი ეგ ალბათ ტუბერკულოზია (კავერნები) მაგრამ ფილტვების დანარჩენი ნაწილიც არ მომწონს... ერთი ძალიან განათებულია, მეორე პირიქით... ესეც ტუბერკულოზისგანაა, თუ რამე ანთებაა ზედდართული?

Posted by: LUKA-BRAZI 25 Apr 2009, 00:54
Thandrus
ისე შენ რატომ არ დებ ქეისებს? შენი პასუხებიდან გამომდინარე რატომღაც მგონია რომ კარგ ქეისს დადებ smile.gif

Blind_Torture_Kill
ქრისტე აღსდგა! გილოცავ აღდგომას დათო, მრავალს დაესწარი. რატომ არ ჩანხარ?
smile.gif

Posted by: Thandrus 25 Apr 2009, 01:29
LUKA-BRAZI

რავიცი აბა... არც საიტები ვიცი სადაც ქეისები წერია, არც რამე ისეთი სპეციალური წიგნი არ მაქვს... მარტო ეს საბაზისო საგნების სახელმძღვანელოები ჯერ.., (საწყალიბიჭისსმაილიgigi.gif) კარგი, ვცდი რამის მონახვას smile.gif

Posted by: Thandrus 25 Apr 2009, 01:45
აი ქეისისმაგვარი რამ:

რა სჭირს ამ კაცს?

Posted by: Romina 25 Apr 2009, 10:05
ტულარემიის ბუბონებს გავს. თუმცა შავი ჭირიც შეიძლება იყოს (თუ ამერიკელი-ინდიელია) smile.gif smile.gif

Posted by: donvaso 25 Apr 2009, 11:05
Thandrus
ყველაფერი რეზისტენტული ტუბერკულიოზისგანაა, 29 წლის გოგონას სურათია, დინამიკაში გაუარესება იყო , რაც არ მოგწონს ეგ მისგან განვითარებული ციროზული ცვლილებებია.....
სურათი მობილით არის გადაღებული, შემდეგში ვეცდები უკეთესად გადავიღო... smile.gif

Posted by: LUKA-BRAZI 25 Apr 2009, 17:01
Thandrus
ნამდვილად არ ვიცი biggrin.gif შეიძლება მართლაც ბუბონი იყოს, როგორც Romina-მ თქვა, ოღონდ რისი ბუბონი არ ვიცი smile.gif ინფექციური დაავადებები 10 დღიან კურაციად მქონდა და ბევრი ვერაფერი გვასწავლეს user.gif

Posted by: Thandrus 26 Apr 2009, 03:21
Romina
LUKA-BRAZI

ბუბონია, შავი ჭირის smile.gif

Posted by: Romina 26 Apr 2009, 10:12
ბოდიში, სურათი არ აიტვირთა. სხვა დროს ვცდი.

Posted by: LUKA-BRAZI 26 Apr 2009, 14:25
Thandrus
QUOTE
ბუბონია, შავი ჭირის

smile.gif
Romina-მ გამოიცნო smile.gif

Posted by: mtvareuli 26 Apr 2009, 16:58
ჰეი პიპლ http://forum.ge/?f=43&showtopic=33959886

Posted by: Romina 26 Apr 2009, 19:38
აბა, ამ დაავადებას თუ გამოიცნობთsmile.gif

Posted by: Romina 26 Apr 2009, 19:43
ეს კი არის ამ დაავადების გამომწვევის კულტურის ნაზარდი - ძალიან დამახასიათებელია, შემწვარ კვერცხს ადარებენsmile.gif.

Posted by: LUKA-BRAZI 26 Apr 2009, 19:53
Romina
ეგეთი კულტურა მიკოპლაზმას არ ახასიათებს?
Mycoplasmas cannot be detected by the naked eye or even by typical light microscopy. Therefore, mycoplasma contamination must be detected by alternative testing methods. The image provided here is of a mycoplasma "colony" after being propagated on special agar. The colony has a typical "fried egg" appearance. - source www.unclineberger.org/tcf/contaminant_GI.asp
დაავადების სახელს კი ვერ ვიხსენებ smile.gif

Posted by: Romina 26 Apr 2009, 20:01
LUKA-BRAZI
QUOTE
ეგეთი კულტურა მიკოპლაზმას არ ახასიათებს?

მიკოპლაზმა არ არის. ერთ მინიშნებას დავწერ კიდე. ამ დაავადების დროს კანის დაზიანებები უმტკივნეულოა. smile.gif

Posted by: LUKA-BRAZI 26 Apr 2009, 20:04
Romina
კიდე რას გავს შემწვარ კვერცხს? biggrin.gif მაგრამ ამ პოსტიდან გამომდინარე http://forum.ge/?showtopic=33902549&view=findpost&p=13561535 , არამგონია ადვილად გამოვიცნო user.gif მაინც დავფიქრდები.....
დიდი ინფექციონისტი, LULA_QABABI გვჭირდება smile.gif

Posted by: irakli222 26 Apr 2009, 20:09
ჯილეხი?
(მას ახასიათებს კანის უმტკივნელო დაზიანება)


ჯილეხის კულტურის ფოტო მოვძებნე.
ჰგავს?

Posted by: LUKA-BRAZI 26 Apr 2009, 20:12
irakli222
ჯილეხის გამომწვევის კოლონიები შემწვარ კვერცხს გავს? smile.gif

Posted by: LUKA-BRAZI 26 Apr 2009, 20:18
აი ციმბირის წყლულის (ჯილეხის, ანტრაქსის) გამომწვევები აგარზე:

რაღაც არ გვანან შემწვარ კვერცხს biggrin.gif

Posted by: Romina 26 Apr 2009, 20:19
irakli222
კი, ეგ არის bis.gif
LUKA-BRAZI
კი გავს, ზემოთ რომ სურათი დავდე ნამდვილად ჯილეხის ნაზარდია, თან საქართველოში გაზრდილი smile.gif

Posted by: agnia 26 Apr 2009, 20:20
კეთრი?

მას ახასიათებს უმტკივნეულო დაზიანებები

დაუწერიათ უკვე სწორი პასუხი smile.gif

Posted by: LUKA-BRAZI 26 Apr 2009, 20:22
Romina
QUOTE
კი გავს, ზემოთ რომ სურათი დავდე ნამდვილად ჯილეხის ნაზარდია, თან საქართველოში გაზრდილი

დასკვნა: ქართული და ამრიკული შემწვარი კვერცხები ერთმანეთისგან რადიკალურად განსხვავდებიან biggrin.gif

P.S. mtvareuli რატომაა IMG ტეგები გამორთული? eek.gif

Posted by: Romina 26 Apr 2009, 20:23
LUKA-BRAZI
ხოოოო smile.gif აქ ნამდვილად არ გავს smile.gif ეტყობა გარკვეულ ნიადაგზე, გარკვეული გადიდებით ჩანს მასე.

Posted by: Solveig 26 Apr 2009, 20:38
LUKA-BRAZI
QUOTE
რაღაც არ გვანან შემწვარ კვერცხს

რას ლაპარაკობ ახლა შენ biggrin.gif


გავს, როგორ არა-..ახლა, ერბოკვერცხი კი არ იქნება. მსგავსებაში იგულისხმება უფრო მუქი ფერის, თან ამოზნექილი შუაგული და პერიფერიებზე-უსწორმასწორო ფორმა.

ისე, ხელებზე დაზიანების და უმტკივნეულობის მიხედვით უფრო უნდა მიმხვდარიყავი დაავადებას (არ გეწყინოს) smile.gif


Posted by: irakli222 26 Apr 2009, 20:40
LUKA-BRAZI
QUOTE
ჯილეხის გამომწვევის კოლონიები შემწვარ კვერცხს გავს?


გავს თუ არა ვერ შეგედავები.

გამოცნობისთვის მნიშვნელოვანია ის რომ ეს წყლულები უმტკივნეულოა.

Posted by: Guardian 26 Apr 2009, 20:55
რა არის ის, რის გამოც ეს ხალხი ნიღაბს ატარებს?
P.S. სურათი დღეს არის გადაღებული მეხიკოში.

Posted by: Thandrus 26 Apr 2009, 21:09
Guardian

Swine Flu "of pandemic potential"...


* * *
Romina
LUKA-BRAZI
irakli222

ვაჰ, რა კარგი ქეისი გავაცდინე,,, sad.gif სახლიდანაც ვეღარ გავა კაცი კვირაობით რა... lol.gif

ისე, ამ სურათებს საიდან შოულობთ ხოლმე, მეც მითხარით რააა... baby.gif არის რამე ნორმალური რესურსი?

Posted by: Solveig 26 Apr 2009, 21:13
Guardian
QUOTE
სურათი დღეს არის გადაღებული მეხიკოში.

ღორის გრიპი.

Posted by: LUKA-BRAZI 26 Apr 2009, 21:15
Solveig
irakli222
კარგით ეხლა, ეჭვქვეშ კი არ ვაყენებ ქეისის სისწორეს და პასუხს!!!! აშკარად ვიხუმრე შემწვარ კვერცხზე, მაინცდამაინც ათი სმაილიკი უნდა მიმეხატა? lol.gif ისიც ვიცი მედიცინაში შედარებები რას ნიშნავს. biggrin.gif


Guardian
QUOTE
რა არის ის, რის გამოც ეს ხალხი ნიღაბს ატარებს?

ორიგინალური ქეისია biggrin.gif

ისე მარტო მეხიკოში კი არა ნიუ-იორკშიც მოუწევთ ნიღბების ტარება, რამდენიმე სტუდენტს დაუდასტურდა lab. test Swine Flu-ზე. - CNN
* * *
Thandrus
QUOTE
ისე, ამ სურათებს საიდან შოულობთ ხოლმე, მეც მითხარით რააა... baby.gif არის რამე ნორმალური რესურსი? 

რა გითხრა აბა.... როგორც ასეთი ყველაფერი ერთგან რომ იყოს თავმოყრილი არ ვიცი..... გუგლი და მისი ჯანი smile.gif ისე რატომღაც რასაც ვეძებ უმეტესად 2-4 გვერდებზე ვპოულობ ხოლმე..... ხშირად რატომრაც მასეა, პირველი გვერდის სურათები არასდროს არ მომწონს biggrin.gif

Posted by: Guardian 26 Apr 2009, 21:31
Thandrus
QUOTE
Swine Flu

ნეირამინიდაზისა და ჰიალურონიდაზის ტიპის მითითებაა საჭირო.

Posted by: LUKA-BRAZI 26 Apr 2009, 21:33
Guardian
H1N1, H1N2, H3N1, H3N2, და H2N3

მარტის შემდეგ US-ში ფიქსირდება H1N1 ქვეტიპი.

Posted by: Thandrus 26 Apr 2009, 21:52
LUKA-BRAZI

QUOTE
H1N1


Guardian

აჰა biggrin.gif

ისე, Spanish Flu რომ იყო, საუკუნის დასაწყისში, ისიც H1N1 იყო, არა? მგონი ქათმის გრიპი სამოთხე იქნება ამასთან შედარებით...

Posted by: LUKA-BRAZI 26 Apr 2009, 22:34
Thandrus
QUOTE
ისე, Spanish Flu რომ იყო, საუკუნის დასაწყისში, ისიც H1N1 იყო, არა?

კი მასე იყო. მაგრამ იმედია პანდემია არ იქნება..... ჯანდაცვის მსოფლიო ორგანიზაციის 24 აპრილის განცხადება განცხადება თუ report-ი: http://www.who.int/csr/don/2009_04_24/en/index.html ცოტა ცუდი ციფრებია სტატისტიკაში მოყვენილი user.gif ესეც ნაწყვეტი სტატიიდან, თუ რა მგრძნობელობა აქვთ წამლების მიმართ ვირუსებს:
The Swine Influenza A/H1N1 viruses characterized in this outbreak have not been previously detected in pigs or humans. The viruses so far characterized have been sensitive to oseltamivir, but resistant to both amantadine and rimantadine.

აშშ-ს დაავადებათა მართვისა და პრევენციის ცენტრის თქმით, 2005 წლისდან მოყოლებული აშშ-ში დაფიქსირდა 20-მდე შემთხვევა: http://www.cdc.gov/swineflu/ და http://www.cdc.gov/swineflu/investigation.htm

არ ვიცი, მიუხედავად ყველაფრისა რამდენად იძლევა ეს ციფრები იმის ვარაუდის დაშვებას, რომ ეპიდემია ან უფრო უარესი პანდმია განვითარდეს.

Posted by: Romina 26 Apr 2009, 22:40
Thandrus
QUOTE
ისე, Spanish Flu რომ იყო, საუკუნის დასაწყისში, ისიც H1N1 იყო, არა?

კი, ეგ იყო ცნობილი "ისპანკა"
smile.gif
ღმერთმა დაგვიფაროს "შიფტისგან", ე.წ. ფრინველის გრიპის და სეზონური გრიპის ვირუსებმა თვისებები თუ გაცვალა, რამის პროგნოზირება ძნელი იქნება smile.gif

Posted by: Thandrus 6 May 2009, 15:31
მოკლედ, ქეისი არაა კლასიკური გაგებით, მაგრამ თქვენი აზრი მაინტერესებს...

ბავშვი წევს თირკმელების ქრონიკული უკმარისობით და დიალიზზეა, რის გამოც ჰეპარინზე ჰყავთ. ჰოდა, ამ ბავშვმა გაიტეხა თავი და სისხლდენას ვერ უჩერებენ (სისხლი გადაუსხეს უკვე ორჯერ). ჰოდა მგონი ქირურგებსაც ეშინიათ რამე მანიპულაციების ჩატარება...

თქვენი აზრით, როგორ უნდა წავიდეს თერაპია?


Posted by: LUKA-BRAZI 6 May 2009, 16:47
Thandrus
რა სახის დაზიანებაზეა საუბარი? მარტო სკალპი თუა დაზიანებული და პატარა კალიბრის სისხლძარღვები, მაშინ ელექტრო დანით კოაგულაცია გაუკეთონ სისხლძარღვებს, რა პრობლემაა და დაადონ ნაკერები. თვის კანის მიკროცირკულაცია უხვია და თან სისხლძარღვები რომ გადაიკვეთება, პირი პირთან აღარ მიაქვთ, ამიტომ ჰემოსტაზია საჭირო, რომელიც თუ ვერ ხერხდება ამ შემთხვევაში, უნდა სისხლძარღვზე იმოქმედონ ჩემი აზრით და არა სისხლზე. ამინოკაპრონის მჟავა, დიცინოლი, ვიკასოლი (? თუ არ მეშლება) ცადეს? თუ დიალიზის გამო უკუნაჩვენებია?

Posted by: Thandrus 6 May 2009, 17:06
LUKA-BRAZI

QUOTE
მარტო სკალპი თუა დაზიანებული და პატარა კალიბრის სისხლძარღვები


ზუსტად არ ვიცი, დღეს მომიყვნენ გაკვრით. ხვალ რომ მივალ, დავადგენ უფრო კონკრეტულად...

QUOTE
ამინოკაპრონის მჟავა, დიცინოლი, ვიკასოლი (? თუ არ მეშლება) ცადეს? თუ დიალიზის გამო უკუნაჩვენებია?


შემადედებლები არამგონია რომ ღირდეს, დიალიზის გამო.

QUOTE
ამ შემთხვევაში, უნდა სისხლძარღვზე იმოქმედონ ჩემი აზრით და არა სისხლზე


ალბათ იდეალური ეგ იქნებოდა, მაგრამ ქირურგებსაც ეშინიათ მგონი (ვაითუ ოპერაციის დროს სისხდენა მოხდეს...) მოკლედ კონსილიუმი ყოფილა დღეს შეკრებილი... მაინტერესებს ძალიან მეც, ხვალ უნდა ვკითხო, ასი პროცენტით.

Posted by: LUKA-BRAZI 6 May 2009, 18:16
Thandrus
კარგი. როცა გაიგებ დაპოსტე smile.gif მე მაინც ვფიქრობ რომ ელექტრო დანით კოაგულაცია შესაძლოა გაკეთდეს პაციენტთან....

წმინდა გიორგიმ დაიფაროს პატარა.

ყველას გილოცავთ გიორგობის ბრწყინვალე დღესასწაულს. მისი მადლი გვფარავდეს!

Posted by: Thandrus 2 Jun 2009, 01:21
საღოლ ჩვენ. მეხუთე გვერდზე რომაა ეს თემა...

აბა, ერთი ქეისი: უფრო იმათთვის ვინც რადიოლოგიაში კარგად ერკვევა. ავადმყოფი ქალი უჩივის ბოლო 30 წლის განმავლობაში კუნთების გაბნეულ ტკივილს (კუნთების სისუსტის გარეშე). იგი აშშ-ში ემიგრაციამდე ინდოეთში ცხოვრობდა. სამწუხაროდ ასეთი მწირი ისტორიაა. ახლა სურათები:

http://images.beef.ge

http://images.beef.ge

http://images.beef.ge

http://images.beef.ge

http://images.beef.ge

http://images.beef.ge





ახლა გავჩუმდები და თქვენ განიხილეთ... biggrin.gif

Posted by: mtvareuli 2 Jun 2009, 16:13
რატო ჩააგდეთ ეს თემა? mo.gif

Posted by: Thandrus 3 Jun 2009, 01:36
mtvareuli

იმედია აღარ ჩავარდება ასე უფსკრულში... მოვიმარაგებ რაღაც ქეისებს smile.gif

ჰა, აბა იდეები biggrin.gif

Posted by: Blind_Torture_Kill 4 Jun 2009, 09:49
QUOTE
იგი აშშ-ში ემიგრაციამდე ინდოეთში ცხოვრობდა


ჩაკალციფიცირებული პარაზიტები

Posted by: Thandrus 4 Jun 2009, 21:55
Blind_Torture_Kill

აჰა yes.gif და რომელი პარაზიტი, კონკრეტულად, შეგიძლია თქვა? wink.gif

Posted by: Solveig 5 Jun 2009, 16:58
Thandrus
ტრიქინელა ხომ არა?

Posted by: anarxisti 5 Jun 2009, 20:06
Thandrus

ცისტიცერკოზი



--------------------------------

Posted by: Thandrus 5 Jun 2009, 23:06
Solveig
anarxisti

არა smile.gif

Posted by: LUKA-BRAZI 6 Jun 2009, 11:01
Thandrus
Guinea worm ? ინტერნეტში ეს შემხვდა biggrin.gif
ის ბავშვი როგორაა, წინა გვერდზე რომ წერდი?

Posted by: mtvareuli 6 Jun 2009, 22:09
მაგ პარაზიტებს რომ გაარკვევთ, მერე ისიც მითხარით ჩემ ბავშვს რა ჭირს user.gif

დედის მონაყოლი: 3 წლის ბიჭია, ორი კვირის წინ ჰქონდა მაღალი სიცხე, ყელის ტკივილი, ტუჩები, ენა და მთლიანად პირის ღრუ-მუქი წითელი. გამოიძახა უბნის პედიატრი, რომელმაც ჩახედა ყელში, დაუსვა ჩირქოვანი ანგინის დიაგნოზი და დაუნიშნა რაღაც ანტიბიოტიკი (სახელი არ ახსოვს), რომელიც უნდა ესვა 10 დღე. 3 დღეში სიცხე დაეწია, მდგომარეობა გაუმჯობესდა (თუმცა ბავშვი ოდნავ მოთენთილი რჩებოდა და ტუჩების ფერი ისევ წითელი) და შეუწყვიტეს ანტიბიოტიკის მიცემა.
10 დღის მერე სიმპტომები კვლავ გაუმეორდა, სიცხე 40 C-მდე ავიდა, ტუჩებისა და ენის მუქი წითელი ფერის გარდა დედამ შენიშნა ასევე ხელისა და ფეხის გულების შესივება, სიწითლე და კანის აფცქვნა. გაახსენდა რომ ორი კვირის წინაც ეფცქვნებოდა კანი, თუმცა ავიტამინოზს მიაწერა და ყურადღება არ მიუქცევია.

user posted image

რა დაავადებაზე იფიქრებთ და რა გამოკვლევებს ჩაატარებთ? smile.gif

Posted by: Thandrus 7 Jun 2009, 00:38
LUKA-BRAZI

QUOTE
Guinea worm ? ინტერნეტში ეს შემხვდა


საღოლ, ეგენია, დრაკუნკულიკოები biggrin.gif up.gif

mtvareuli

QUOTE
რა დაავადებაზე იფიქრებთ და რა გამოკვლევებს ჩაატარებთ?


ამ სურათის და სიმპტომების მიხედვით ქუნთრუშა მგონია... გამოკვლევები არ ვიცი, მგონი ამის დიაგნოზი მხოლოდ კლინიკური სურათითაც ისმევა. თუ ტესტებზე მიდგა საქმე, ედს, C-რეაქტიული ცილა და ანტისტრეპტოლიზური ანტისხეული უნდა გამოჩნდეს + კულტურა ყელის ნაცხში, თუმცა არამგონია ამდენი რამე საჭირო იყოს...

Posted by: vano_t 7 Jun 2009, 09:59
QUOTE (mtvareuli @ 6 Jun 2009, 22:09 )
მაგ პარაზიტებს რომ გაარკვევთ, მერე ისიც მითხარით ჩემ ბავშვს რა ჭირს user.gif

დედის მონაყოლი: 3 წლის ბიჭია, ორი კვირის წინ ჰქონდა მაღალი სიცხე, ყელის ტკივილი, ტუჩები, ენა და მთლიანად პირის ღრუ-მუქი წითელი. გამოიძახა უბნის პედიატრი, რომელმაც ჩახედა ყელში, დაუსვა ჩირქოვანი ანგინის დიაგნოზი და დაუნიშნა რაღაც ანტიბიოტიკი (სახელი არ ახსოვს), რომელიც უნდა ესვა 10 დღე. 3 დღეში სიცხე დაეწია, მდგომარეობა გაუმჯობესდა (თუმცა ბავშვი ოდნავ მოთენთილი რჩებოდა და ტუჩების ფერი ისევ წითელი) და შეუწყვიტეს ანტიბიოტიკის მიცემა.
10 დღის მერე სიმპტომები კვლავ გაუმეორდა, სიცხე 40 C-მდე ავიდა, ტუჩებისა და ენის მუქი წითელი ფერის გარდა დედამ შენიშნა ასევე ხელისა და ფეხის გულების შესივება, სიწითლე და კანის აფცქვნა. გაახსენდა რომ ორი კვირის წინაც ეფცქვნებოდა კანი, თუმცა ავიტამინოზს მიაწერა და ყურადღება არ მიუქცევია.

user posted image

რა დაავადებაზე იფიქრებთ და რა გამოკვლევებს ჩაატარებთ? smile.gif

კავასაკის დაავადება შეიძლება იყოს. ოღონდ, კავასაკი სხვა დაავადებების გამორიცხვით ისმება და ამასთანავე სპეციალური კრიტერიუმები ახასიათებს. ამ შემთხვევაში 2 კრიტერიუმი გვაქვს 5-დან. დიაგნოზისათვის (გარდა იმისა, რომ სხვა დაავადებები გამორიცხო) საჭიროა კიდევ 2 კრიტერიუმი. რადგანაც ბავშვის თვალები არ არის მოცემული, ვერ ვიტყვით თუ ორმხრივი კონიუნქტივიტია თუ არა. ასევე არ არის ნახსენები ცერვიკალური ლიმფადენოპათია და პოლიმორფული გამონაყარი კანზე.

კავასაკის დიფერენციალური დიაგნოზები უნდა გამოირიცხოს. მასე შეიძლება ენტეროვირუსული (კოქსაკივირუსით გამოწვეული) hand foot mouth disease-იც წავიდეს-იშვიათად ეს დაავადება რეკურენტულია. შეიძლება სტივენ-ჯონსონიც იყოს. ნაკლებად სავარაუდოა, მაგრამ შესაძლებელია. კიდევ სხვა ვირუსული ეგზანთემები.

ლაბორატორიის ამბავში, თანდრუსის ლაბორატორიებს დავამატოთ CBC, AST/ALT, UA, CPK, BMP, ვირუსული სეროლოგიები. ყველა ლაბორატორია გარკვეული მოსაზრებით შეიძლება გააკეთო (სხვა დავადებები რომ გამორიცხო და კავასაკის გართულებებს რომ დააკვირდე).

P.S. ინფორმაცია ინტერნეტში შევაგროვე. ძებნის წესი: 1) hand foot mouth disease დაიგუგლა; 2) ამ დაავადების დიფენრეციალური დიაგნოზები იქნა ნაპოვნი; 3) დადგინდა თვითონ ამ დაავადების რეკურენტულობა; 4) დიფერნეციალური დაიგნოზები და მათი work up გადაიხედა და მოკლე დასკვნა დაიდო smile.gif მოკლედ, მე არაფერი წვლილი არ მიმიძღვის აქ (იმ შემთხვევაში თუ სწორი მიმართულებით წავედი).

Posted by: LUKA-BRAZI 7 Jun 2009, 16:19
ჟოლოსფერი ენა ქუნთრუშას არ ახასიათებს?

Posted by: mtvareuli 7 Jun 2009, 17:16
Thandrus
LUKA-BRAZI

ქუნთრუშას კი ახასიათებს ეგეთი ენა, მაგრამ როგორც vano_t-მ თქვა, შეიძლება იყოს კავასაკი. ტუჩები და ენა ამ დროსაც მუქი წითელია, გარდა ამისა ხელისა და ფეხისგულების შეშუპება, სიწითლე და კანის აფცქვნა ახასიათებს. გამომრჩა მეთქვა, რომ კისრის ლიმფური კვანძებიც გადიდებული ქონდა.
თვალებზე კონიუქტივიტი და გამონაყარი არ ქონია, მაგრამ ეს ტემპერატურის დაგდება და მომატება ანტიბიოტიკებთან შეიძლება სულაც არ იყოს კავშირში და პერიოდულ სახეს ატარებდეს.
ეს ბავშვი რეალურად ვნახე რამოდენიმე დღის წინ და კავასაკზე ჰქონდათ ეჭვი. დასადასტურებლად საჭირო იყო სხვა დაავადებების (პირველ რიგში სტრეპტოკოკული აგრესიის) გამორიცხვა. სამწუხაროდ მე იქ აღარ მივდივარ და არ ვიცი რას იზამენ, დაადასტურებენ თუ არა, მაგრამ ალბათ უფრო კი.
იმიტომ დავსვი კითხვა ასე
QUOTE
რა დაავადებაზე იფიქრებთ
და არა "ზუსტად რა სჭირსთქო".

აქ იმიტომ დავდე ეს შემთხვევა, რომ ჯერ ერთი, ჩემთვის დასამახსოვრებელი იყო და თან ძალიან ბევრმა ექიმმა შეიძლება ყურადღება არ მიაქციოს სხვა სიმპტომებს და არ ჩაატაროს დამატებითი გამოკვლევები, ჩვეულებრივი ანგინის ან ქუნთრუშას დიაგნოზი დასვას და დრო დაკარგოს. არ არის ძალიან გავრცელებული დაავადება, მაგრამ ეგეთებიც ხდება ხოლმე : )

Posted by: LULA_QABABI 7 Jun 2009, 23:44
ვანო შენ ჩუ ჯერჯერობით;
მოკლედ ეს რენტგენი შემხვდა ონლაინ; გამახსენდა ერთერთი ჩემი ავადმყოფი ზუსტად იგივე დაავადებით, წარსულითა და რადიოლოგიური მონაცემებით;

Posted by: კელა 8 Jun 2009, 00:48
QUOTE (mtvareuli @ 6 Jun 2009, 22:09 )
მაგ პარაზიტებს რომ გაარკვევთ, მერე ისიც მითხარით ჩემ ბავშვს რა ჭირს user.gif

დედის მონაყოლი: 3 წლის ბიჭია, ორი კვირის წინ ჰქონდა მაღალი სიცხე, ყელის ტკივილი, ტუჩები, ენა და მთლიანად პირის ღრუ-მუქი წითელი. გამოიძახა უბნის პედიატრი, რომელმაც ჩახედა ყელში, დაუსვა ჩირქოვანი ანგინის დიაგნოზი და დაუნიშნა რაღაც ანტიბიოტიკი (სახელი არ ახსოვს), რომელიც უნდა ესვა 10 დღე. 3 დღეში სიცხე დაეწია, მდგომარეობა გაუმჯობესდა (თუმცა ბავშვი ოდნავ მოთენთილი რჩებოდა და ტუჩების ფერი ისევ წითელი) და შეუწყვიტეს ანტიბიოტიკის მიცემა.
10 დღის მერე სიმპტომები კვლავ გაუმეორდა, სიცხე 40 C-მდე ავიდა, ტუჩებისა და ენის მუქი წითელი ფერის გარდა დედამ შენიშნა ასევე ხელისა და ფეხის გულების შესივება, სიწითლე და კანის აფცქვნა. გაახსენდა რომ ორი კვირის წინაც ეფცქვნებოდა კანი, თუმცა ავიტამინოზს მიაწერა და ყურადღება არ მიუქცევია.

user posted image

რა დაავადებაზე იფიქრებთ და რა გამოკვლევებს ჩაატარებთ? smile.gif

ენა ჟოლოსებრს გავს...

Posted by: Blind_Torture_Kill 8 Jun 2009, 01:19
LULA_QABABI
კლინიკა ან მაკრო და მიკრო მიამაგრე
მარტო რენტგენი არაფერია


Posted by: LULA_QABABI 8 Jun 2009, 04:13
QUOTE (Blind_Torture_Kill @ 7 Jun 2009, 16:19 )
LULA_QABABI
კლინიკა ან მაკრო და მიკრო მიამაგრე
მარტო რენტგენი არაფერია

შემთხვევა რადიოლოგიური კუთხით უფრო საინტერესოა, ვიდრე კლინიკურით; გეტყვი რომ პაციენტი 85 წლის ქალბატონია;
შეგიძლიათ ჩამოთვალოთ დიფერენციალური დიაგნოზი, თუ რას გაგონებთ, ან შეიძლება იყოს ამ რენტგენზე და კტ-ზე;

Posted by: LUKA-BRAZI 8 Jun 2009, 09:13
LULA_QABABI
სალამი ლადო smile.gif
მართლა მაგარი რენტგენია, არ მინახავს არასდროს მასეთი smile.gif
თავიდან ტუბერკულოზის კავერნები ვიფიქრე და ამ თემაზე გავგუგლე და აღმოვაჩინე რომ ეგ კტ და რენტგენი ახასიათებს ტუბერკულოზის პლომბაჟით მკურნალობას.

http://en.wikipedia.org/wiki/Plombage

Posted by: LULA_QABABI 8 Jun 2009, 18:59
QUOTE (LUKA-BRAZI @ 8 Jun 2009, 00:13 )
LULA_QABABI
სალამი ლადო smile.gif
მართლა მაგარი რენტგენია, არ მინახავს არასდროს მასეთი smile.gif
თავიდან ტუბერკულოზის კავერნები ვიფიქრე და ამ თემაზე გავგუგლე და აღმოვაჩინე რომ ეგ კტ და რენტგენი ახასიათებს ტუბერკულოზის პლომბაჟით მკურნალობას.

http://en.wikipedia.org/wiki/Plombage

"გად ბლეს სერიოჟა ბრინ"'

ისე რა ჩაწერე სერჩში;

ეს არის 85 წლის იაპონელი ქალის რენტგენი, პლომბაჟი გაკეთებული აქვს 57 წლის წინ; სურათზე ხედავთ ამ ბურთებს; პლომბაჟზე მოკლე ინფო ვიკიპედიაზეც არის;
ჩემს ავადმყოფს, როგორც თვითონ მითხრა, პინგპონგის ბურთები ქონდა ჩალაგებული;

Posted by: LUKA-BRAZI 8 Jun 2009, 20:28
LULA_QABABI
QUOTE
ისე რა ჩაწერე სერჩში;

lol.gif
აღარ მახსოვს gigi.gif

Posted by: LUKA-BRAZI 12 Jun 2009, 23:48
ჰა, აბა ყურადღება, მოვიდა ცხელი კლინიკური ქეისი biggrin.gif
მაგრამ მგონი აღარავინ ხართ აქ user.gif
მოკლედ პაციენტი, ასაკს არ აქვს მნიშვნელობა, უჩივის:
ნელა პროგრესირებად, ყრუ ხასაითის თავის ტკივილს, ამბობს რომ აქვს ამენორეა და წონაშიც მოიმატა. აღენიშნება სიცივისადმი ტოლრანტობის დაქვეითება, ადვილად დაღლილობა.
პაციენტი უჩივის მეხედველობის პრობლემებს.
სისხლის ანალიზით გამოვლინდა ელექტროლიტური ცვლის დარღვევები.
ყურადღებას იპყრობს პაციენტის ემოციური ლაბილობა, ჩივილბი მადის დაკარგვაზე და ჰიპოტენზიაზე.

მოკლედ ამ დაავადების ბევრი სიმპტომი ჩამოვთვალე.

თუ გაჭირდა მინიშნებებსაც გავაკეთებ.

აბა კლინიცისტები აქეთ (present and upcoming biggrin.gif)

კანFეტი ქეისია biggrin.gif

Posted by: Cousteau 13 Jun 2009, 00:00
QUOTE (LUKA-BRAZI @ 12 Jun 2009, 23:48 )
ჰა, აბა ყურადღება, მოვიდა ცხელი კლინიკური ქეისი biggrin.gif
მაგრამ მგონი აღარავინ ხართ აქ user.gif
მოკლედ პაციენტი, ასაკს არ აქვს მნიშვნელობა, უჩივის:
ნელა პროგრესირებად, ყრუ ხასაითის თავის ტკივილს, ამბობს რომ აქვს ამენორეა და წონაშიც მოიმატა. აღენიშნება სიცივისადმი ტოლრანტობის დაქვეითება, ადვილად დაღლილობა.
პაციენტი უჩივის მეხედველობის პრობლემებს.
სისხლის ანალიზით გამოვლინდა ელექტროლიტური ცვლის დარღვევები.
ყურადღებას იპყრობს პაციენტის ემოციური ლაბილობა, ჩივილბი მადის დაკარგვაზე და ჰიპოტენზიაზე.

მოკლედ ამ დაავადების ბევრი სიმპტომი ჩამოვთვალე.

თუ გაჭირდა მინიშნებებსაც გავაკეთებ.

აბა კლინიცისტები აქეთ (present and upcoming biggrin.gif)

კანFეტი ქეისია biggrin.gif

ჰიპოფიზთან მწყრალად ხო არ არის ?

Posted by: Thandrus 13 Jun 2009, 01:29
LUKA-BRAZI

კრანიოფარინგიომა მგონია. ახსნის ჰიპოფიზისეულ სიმპტომებს, პლუს მხედველობის პრობლემებს და ყრუ ტკივილსაც.

Posted by: LUKA-BRAZI 13 Jun 2009, 01:33
Cousteau
QUOTE
ჰიპოფიზთან მწყრალად ხო არ არის ?

biggrin.gif yes.gif
Thandrus
QUOTE
კრანიოფარინგიომა მგონია. ახსნის ჰიპოფიზისეულ სიმპტომებს, პლუს მხედველობის პრობლემებს და ყრუ ტკივილსაც.

მგლები ხართ, მგლები! smile.gif

up.gif

ბევრი სიმპტომი მივაწრე. სხვა დროს ეგრე აღარ ვიზამ, თორემ დაპოსტილი არ არის, რომ ეგრევე იცნობთ ხოლმე smile.gif

ქეისი დაიდოს! war.gif

Posted by: Thandrus 13 Jun 2009, 01:56
LUKA-BRAZI

არის ქეისი! chest.gif

64 წლის "მემარჯვენე" ქალს ერთი კვირის განმავლობაში პროგრესულად ჰქონდა მხედველობასთან და კითხვასთან დაკავშირებული პრობლემები. მას 3 წლის მსხვილი ნაწლავის კიბოს ისტორია ჰქონდა, ღვიძლში მეტასტაზებით. შემოწმება უმეტესწილად ნორმალური იყო ერთი რამის გარდა: პაციენტი ვერ კითხულობდა დაწერილ სიტყვებს. ჰქონდა short-term verbal memory-ს პრობლემები და მარჯვენა ჰომონიმური ჰემიანოპია. მას წერისას არავითარი პრობლემა არ ჰქონია, მაგრამ რამდენიმე წუთის შემდეგ, თავის ნაწერს ვეღარ კითხულობდა. მას აგრეთვე აღენიშნებოდა მცირედი დისნომია: მან გაიხსენა watch, ring, finger, elbow, lips-ის სახელი მაგრამ ვერ გაიხსენა რა ერქვა knuckle, nail, veins და hand-ს. Color naming-ი არ შემოწმებულა.

კითხვა: 1. რა ჰქვია სინდრომს, რომელიც ამ პაციენტს აქვს, და სადაა იგი ლოკალიზირებული (ახსენით პათოლოგიური მექანიზმიც)? 2. რა შეიძლება გამხდარიყო ამ პაციენტში ამ სინდრომის მიზეზი?

Posted by: LUKA-BRAZI 13 Jun 2009, 02:06
Thandrus
QUOTE
რა ჰქვია სინდრომს, რომელიც ამ პაციენტს აქვს

აზრზე არ ვარ smile.gif ტატიშვილში ალბათ არც წერია biggrin.gif
QUOTE
სადაა იგი ლოკალიზირებული (ახსენით პათოლოგიური მექანიზმიც)?

კეფის წილებში, მარცხნივ, ვითომ?
QUOTE
რა შეიძლება გამხდარიყო ამ პაციენტში ამ სინდრომის მიზეზი?

სიმსივნის მეტასტაზი?
მოკლედ ეგ სინდრომი მგონი არ ვიცი, ისე ვწერ რაღაცეებს :|

QUOTE
13 Jun 2009, 01:56 

რატომ არ გძინავს? biggrin.gif

Posted by: Thandrus 13 Jun 2009, 02:52
LUKA-BRAZI

QUOTE
კეფის წილებში, მარცხნივ, ვითომ?


ნაწილობრივ მართალი ხარ, მაგრამ კიდევ უფრო ზუსტი ლოკალიზაციაა საჭირო smile.gif

QUOTE
სიმსივნის მეტასტაზი?


კი, ამაში მართალი ხარ smile.gif

QUOTE
რატომ არ გძინავს?


კარდიოლოგიაში გამოცდა მაქვს ხვალ, რა დამაძინებს?! biggrin.gif

Posted by: LUKA-BRAZI 13 Jun 2009, 10:20
Thandrus
ვა მცოდნია, უყურე შენ! biggrin.gif
QUOTE
ნაწილობრივ მართალი ხარ, მაგრამ კიდევ უფრო ზუსტი ლოკალიზაციაა საჭირო

ფრინველის დეზის მიდამოები მაშინ, ან ოთხგორაკის ბორცვები მაგრამ ოთხგორაკი კეფის წილებიდან მოშორებითაა, ე.ი. ესეც გამოირიცხა, ქიაზმა არ არის დამნაშავე.....
QUOTE
კარდიოლოგიაში გამოცდა მაქვს ხვალ, რა დამაძინებს?!

წარმატებები, წარმატებები!!!
მაგრამ, მოიცა, როგორ კარდიოლოგიაში? კარდიოლოგია თერაპიაში არ შედის? ცალკე რატომ ტარდება?
უი, თსსუ-ში არ სწავლობ შენ?
რამდენი კითხვა დავსვი smile.gif

Posted by: Thandrus 14 Jun 2009, 02:36
LUKA-BRAZI

QUOTE
უი, თსსუ-ში არ სწავლობ შენ?


არა, აიეტში ვსწავლობ smile.gif იქ კარდიოლოგია ცალკეა გამოყოფილი.

QUOTE
ფრინველის დეზის მიდამოები მაშინ


wow.gif ქართულ ანატომიურ ტერმინოლოგიაში ცოტათი მოვიკოჭლებ... ეს ინგლისურად როგორ იქნება?

მოკლედ, საჭიროა სინდრომის სახელი, ზუსტი ლოკალიზაცია და მექანიზმი!

Posted by: LUKA-BRAZI 14 Jun 2009, 11:53
Thandrus
ჰო რა ვიცი, მხედველობის ქერქული ანალიზატორის მიდამოს ეგრე ქვია.......
სინდრომს რა ქვია ნამდვილად არ ვიცი, შესაბამისად მექანიზმიც არ ვიცი. ნუ ალბათ სიმსივნური მეტასტაზის გამო.....
რა ქენი, ჩააბარე კარდიოლოგია?

Posted by: Thandrus 16 Jun 2009, 21:27
LUKA-BRAZI

QUOTE
რა ქენი, ჩააბარე კარდიოლოგია?


კი, მგონი კარგად ჩავაბარე smile.gif

აბა, აბა, იდეები smile.gif

Posted by: LUKA-BRAZI 16 Jun 2009, 21:54
Thandrus
QUOTE
კი, მგონი კარგად ჩავაბარე

კარგია! წარმატებები!
smile.gif

Posted by: Thandrus 19 Jun 2009, 00:34
... დავწერო პასუხი? user.gif

ვერ ხსნის ვერავინ თუ არ ახსენდებათ საერთოდ ეს თემა? sad.gif

Posted by: Thandrus 30 Jun 2009, 15:40
კარგი, იყოს პასუხი:

ამ სინდრომს უბრალოდ ჰქვია ალექსია აგრაფიის გარეშე (Alexia Without Agraphia). იგი გამოწვეულია დომინანტური (უმრავლეს შემთხვევაში მარცხენა) ოქციპიტალური კორტექსის ინფარქტით, რომელიც კორპუს კალოზუმის უკანა ნაწილზეც გადადის. მარცხენამხრივი ოქციპიტალური კორტექსის დაზიანების გამო მხედველობის არის მარჯვენა ნაწილიდან წამოსული ინფორმაცია (წერითი ინფორმაციის ჩათვლით) ვერ აღიქმება (ანუ გვაქვს მარჯვენა ჰომონიმური ჰემიანოპია), კოპუს კალოზუმის უკანა ნაწილის დაზიანების გამო კი, მხედველობის არის მარცხენა მხარიდან მარჯვენა ოქციპიტალურ კორტექსში შესული ინფორმაცია დომინანტურ მხარეს, ვერნიკეს არეში ვეღარ გადაეცემა... ასე რომ, პაციენტი ლაპარაკს იგებს, ხედვაც ნაწილობრივ შენარჩუნებული აქვს, წერა შეუძლია, წაკითხვა (უფრო სწორად წაკითხულის გააზრება) კი არა smile.gif


Posted by: Thandrus 4 Jul 2009, 00:28
ვინმემ რამე დადეთ რა...

ნუ მოვკლავთ ამ თემას...

Posted by: LUKA-BRAZI 7 Jul 2009, 15:25
Thandrus
აჰა ქეისი.... თანაც შენ რომ დადე მაგის მსგავსი biggrin.gif
პაციენტი შემოვიდა კლინიკაში ჩივილებით: - - - (კლინიკას არ აღვწერ, თორემ ეგრევე მიხვდები smile.gif)
ჩაუტარდა გამოკვლევები: - - - . ამ გამოკვლევებიდან ყურადღებას იპყრობს: სისხლში აღენიშნა ლეიკოპენია (1), მუცლის მიმოხილვით რენტგენოგრამაზე კი ჰაერის არსებობა ასწვრივ და დასწვრივ კოლინჯში, ხოლო განივ კოლინჯში ჰაერი არ დაფიქსირდა (2). Hint: სისხლში და შარდში მაღალია ამილაზის დონე.

ჰოდა ეხლა დაასახელე ამ ორი სინდრომის სახელი biggrin.gif

მეტა: mtvareuli
vano_t
Blind_Torture_Kill
LULA_QABABI
Cousteau

smile.gif

Posted by: Blind_Torture_Kill 7 Jul 2009, 20:26
LUKA-BRAZI

ვა გამოცოცხლდა თემა

მწირი ინფორმაციაა ან მე არ ვიცი პასუხი smile.gif

Posted by: LUKA-BRAZI 7 Jul 2009, 21:20
Blind_Torture_Kill
უი, უი შემეშალა, ლეიკოპენია კი არა, ლიმფოპენია უნდა დამეწერა! ასევე, გამოსაცნობია არა სინდრომის, არამედ სიმპტომის დასახელება. 1000 excuses! biggrin.gif მომიტევეთ უზუსტობები, 2 დღეში 5 ქირურგიულ საგანში მაქვს კომბინირებული გამოცდა და სულ გამომაშტერა წინასაგამოცდო მეცადინეობამ smile.gif
Hint-ი ისეთი დავდე რომ დაავადებას სრაზუ გამოიცნობ! არ დაუკვირდი ეტყობა.... smile.gif თუ დაავადებას გამოიცნობ, სიმპტომებსაც მიხვდები. ისე ეს ქეისი უფრო Thandrus დავუდე, რევანში მაქვს ასაღები biggrin.gif
sa.gif

biggrin.gif

Posted by: Thandrus 7 Jul 2009, 22:01
LUKA-BRAZI

ჰმ... ძალიან ცოტა ხელჩასაჭიდია user.gif

ნუ, რახან ამილაზაა მომატებული, (და თანაც "Hint"-ია biggrin.gif) ალბათ პანკრეატიტზე ლაპარაკობ... რომელიც ალბათ შემდეგ მსხვილი ნაწლავის ინფარქტით გართულდა... (ნუ ეგ ამილაზა რომ თქვი, მაგიტომ biggrin.gif)

სიმპტომს რა ქვია, String sign?

ცოტა მიგვანიშნე biggrin.gif

Posted by: LUKA-BRAZI 7 Jul 2009, 22:32
Thandrus
არააა biggrin.gif
QUOTE
პანკრეატიტზე

დაავადება გამოცნობილია up.gif , ეხლა დაგრჩა სიმპტომების დასახელება smile.gif
QUOTE
ცოტა მიგვანიშნე

Nicht, უნდა გაწრუწუნო givi.gif
QUOTE
ჰმ... ძალიან ცოტა ხელჩასაჭიდია 

მეტი აღარც გჭირდება smile.gif
QUOTE
მსხვილი ნაწლავის ინფარქტით გართულდა

no.gif
არა კაცო, კითხვა მარტივია: პერიტონიტის დროს ტკივილს + დეფანსი ხო ბლუმბერგის სიმპტომი? ჰოდა რა ქვია პანკრეატიტის დროს ჩემს მიერ აღწერილ სიმპტომებს?

აბა რა გეგონა Alexia Without Agraphia რო მამტვრევინე თავი, კინაღამ ჰემისფეროები გადამიხურდა biggrin.gif
* * *
გამოცდებს რომ მოვრჩები მერე გავახუროთ "ქეისობანა" smile.gif
* * *
QUOTE
სიმპტომს რა ქვია, String sign?

სთრინგ საინ უფრო კრონის და არასპეციფიური კოლოტის დროს......

Posted by: Blind_Torture_Kill 7 Jul 2009, 22:53
მე დავდებ ხვალ რამეს თუ მოვიცალე

ამილაზა შეიძლება სანერწყვე ჯირკვლის პათოლოგიამაც გაზარდოს

Posted by: LUKA-BRAZI 7 Jul 2009, 22:56
Blind_Torture_Kill
ოჰ შე ექსპერტო, არ შეგვარჩინო არაფერი! biggrin.gif ეგ კი მარა კოლინჯი ხომ არ იცვლება მაგ დროს? თუ იცვლება?

Posted by: Blind_Torture_Kill 7 Jul 2009, 23:37
LUKA-BRAZI
aba.gif biggrin.gif no.gif

არა
მარა ლიმფოპენიამ მაფიქრებინა რაღაც ვირუსული ნოზოლოგია
რამაც სანერწყვე ჯირკვლებს და პანკრეას user.gif

Posted by: LUKA-BRAZI 7 Jul 2009, 23:54
Blind_Torture_Kill
QUOTE
მარა ლიმფოპენიამ მაფიქრებინა რაღაც ვირუსული ნოზოლოგია
რამაც სანერწყვე ჯირკვლებს და პანკრეას 

ეგ ვარინატიც შეიძლებოდა. ნუ ასეა თუ ისე დაავადება მწვავე პანკრეატიტია. ეხლა კი დასახელდეს სიმპტომთა სახელები, გრამატიკული შეცდომების გარეშე smile.gif ნუ ეს უფრო ისეთი, სახუმარო ქეისია, Thandrus-ს გავეხუმრესავით. უბრალოდ ამ სიმპტომების სახლები უნდა იცოდე და ეგაა რა. ისე კაი ქეისი გამოცხვებოდა ამაზე, ცოტა სერიოზულად რომ დამედო. კოლინჯის ამბები პანკრეატიტის დროს, რაღაც უცხოდ გეჩვენათ.
smile.gif

Posted by: Thandrus 7 Jul 2009, 23:55
ახლა ვკითხე მეგობარს...

ლიმფოციტოპენიას გერპორტის სინდრომი ჰქვია, რენტგენზე მოცემულ ნიშანს სტიუარტის ნიშანი (პანკრეატიტის მეორადი ნიშანიაო, "სეგმენტური მეტეორიზმის" გამო ვითარდებაო).

არ ვიცი, ჰარისონი გადავქექე რასაც ქვია, და ეგენი ვერ ვიპოვე... საერთოდ ლიმფოციტოპენია არ ეწერა იქ (ლეიკოციტოზი, კიბატონო biggrin.gif). მეგობარმაც, ქართული წიგნიდან მითხრა. ასე რომ... Global Confusion biggrin.gif

Posted by: LUKA-BRAZI 8 Jul 2009, 00:01
Thandrus
ყოჩაღ შენს მეგობარს! biggrin.gif

ეგრეც ვიცოდი რომ ტატიშვილი ჯობდა ჰარისონს lol.gif

QUOTE
Global Confusion

შენ იცინე და ეგრე SDN ფორუმზე დიდი გაუგებრობა იყო: რომელი მუტაგენური ცილის შემცველობაზე იყო დამოკიდებული სფეროციტოზის განვითარება, ანკირინის (ჰარისონი) თუ სპექტრინის (კაპლანები). ასე რომ, დროა მთელი მსოფლიო გადავიდეს ტატიშვილის "შინაგან სნეულებებზე" ! მაშ, მაშ ! biggrin.gif

Posted by: anarxisti 8 Jul 2009, 00:20
აბა, რადიოლოგიის ლომებო!
35 წლის ქალი, გადატანილი აქვს საშვილოსნოს ექსტირპაცია ვაგინალური მიდგომით. ოპერაციის შემდგომ დაეწყო ძლიერი ტკივილები მუცლის ქვედა ნაწილში. გაუკეთდა CT, რომელიც მოტანილია ქვევით
რა პათოლოგიასთან გვაქვს საქმე?

Posted by: Thandrus 8 Jul 2009, 00:24
LUKA-BRAZI

QUOTE
ეგრეც ვიცოდი რომ ტატიშვილი ჯობდა ჰარისონს


მეც ვგრძნობდი ამას გულის სიღრმეში lol.gif

QUOTE
შენ იცინე და ეგრე SDN ფორუმზე დიდი გაუგებრობა იყო: რომელი მუტაგენური ცილის შემცველობაზე იყო დამოკიდებული სფეროციტოზის განვითარება, ანკირინის (ჰარისონი) თუ სპექტრინის (კაპლანები).


ბოლოს რაზე შეთახმდნენ biggrin.gif სფეროციტოზის გამოწვევა, ანკირინის, სპექტრინის და რამდენიმე სხვა ცილის მუტაციებსაც შეუძლიათ. ციტოსკელეტონის ცილები ისეა ერთმანეთთან ფიზიკურად და ფუნქციურად დაკავშირებული, რომ მათგან რომელიმეს მუტაცია გამოიწვევს ერითროციტის ფორმის ცვლილებას. ჰარისონს აქვს კარგი ცხრილი ამ მუტაციების.

Posted by: LUKA-BRAZI 8 Jul 2009, 00:34
Thandrus
უფროსწორად კითხვა იყო, მთავარი დამნაშავე რომელი ცილააო biggrin.gif შედეგი არ ვიცი. ტატიშვილში მგონი სპექტრინი წერია, წავალ დავპოსტავ და მორჩება "ბაზარი" biggrin.gif

anarxisti
მე არ ვარ რადიოლოგიის ლომი, მაგრამ ისე ჰაერზე გაფიქრებით: დუგლასის ფოსოში შეიძლება რამეა, სისიხლი, ჩირქი etc. ან საკვერცხეების ამბავია (საკვერცხის აპოპლექსია) ან რამე იატროგენული დაზიანება... ვერსიებიც ამას ქვია smile.gif

Posted by: anarxisti 8 Jul 2009, 00:54
QUOTE (LUKA-BRAZI @ 8 Jul 2009, 00:34 )

anarxisti
მე არ ვარ რადიოლოგიის ლომი, მაგრამ ისე ჰაერზე გაფიქრებით: დუგლასის ფოსოში შეიძლება რამეა, სისიხლი, ჩირქი etc. ან საკვერცხეების ამბავია (საკვერცხის აპოპლექსია) ან რამე იატროგენული დაზიანება... ვერსიებიც ამას ქვია smile.gif

ცხელაgigi.gif

სწორედ საკვერცხის პათოლოგიის დიაგნოზი დაისვა და მკურნალობაც დაენიშნა, მაგრამ უშედეგოდ.

Posted by: LUKA-BRAZI 8 Jul 2009, 01:12
anarxisti
smile.gif
ნუ მაშინ იქნებ ჰემორაგიული კისტა იყო? ან სისხლდენა საკვერცხიდან, პოსტოპერაციული.
QUOTE
მაგრამ უშედეგოდ

სამწუხაროა, მაგრამ ეს ყველაფერი მოკლე დროში განვითარდა თუ შორეული შედეგია? ანუ შეიძლება რამე ავთვისებიანი იყო და მაგიტომაც დამთავრდა უშედეგოდ.....

Posted by: anarxisti 8 Jul 2009, 01:21
ტკივილები დაიწყო ეგრევე, ოპერაციის შემდგომ. სპონტანური ტკივილები და ტკივილები სქესობრივი აქტის დროს. ასევე ხელით გასინჯვის დროს. კლინიკა მნიშვნელოვანია, ცხადია, მაგრამ CT ზე არის პასუხი, ანუ ტიპიური სურათია, ანამნეზთან ერთად.
ხო, "მკურნალობა" დაენიშნა CT მდე, უბრალო ვაგინალური ექოსკოპიის შემდეგ, რომელმაც მართლაც აჩვენა მარჯვენა დანამატის არეში საკვერცხის რაღაც წარმონაქმნის არსებობა, რომელიც საკვერცხის კისტად ჩაითვალა.

Posted by: Blind_Torture_Kill 8 Jul 2009, 19:33
anarxisti
დერმოიდული კისტა ხო არაა ?

Posted by: anarxisti 8 Jul 2009, 19:46
QUOTE (Blind_Torture_Kill @ 8 Jul 2009, 19:33 )
anarxisti
დერმოიდული კისტა ხო არაა ?

არა

აგერ ლაპაროსკოპიული სურათი

Posted by: LUKA-BRAZI 8 Jul 2009, 21:28
anarxisti
ლუთეინური კისტა მაშინ?

smile.gif

Posted by: anarxisti 8 Jul 2009, 22:38
QUOTE (LUKA-BRAZI @ 8 Jul 2009, 21:28 )
anarxisti
ლუთეინური კისტა მაშინ?

smile.gif

არა...
კარგი, აღარ გაგაწვალებთ...მეგონა რომ ლაპაროსკოპიის სურათი დაგეხმარებოდათ.. მაგრამ ეტყობა რომ არ იცნობთ ლაპაროსკოპიულ სემეიოლოგიას..

მოკლედ, ტექსტილომაა, უცხო სხეული ტექსტილის, ანუ პირველი ოპერაციის დროს დარჩენილი ტამპონი, რომელმაც გაიკეთა კაფსულა...შეეხორცა საშოს, მარჯვენა საკვერცხესა და სწორ ნაწლავს, რაც იწვევდა ტკივილებს და ცხადია ვერავითარი მკურნალობა, გარდა ქირურგიული მოშორებისა, ვერ განკურნავდა პაციენტს.
მართალია რთული იყო, მაგრამ მისი სრული მოშორება ლაპაროსკოპიულად მოხერხდა..

CT ის რაც შეეხება, ნათლად ჩანს მოცულობითი წარმონაქმნი, მკვეთრი კონტრასტული უბნებით, (თეთრი უბნები) რაც შეესაბამება ტიპიურ რენტგენოკონტრასტულ ქირურგიულ ტამპონს.
PS. ევროპაში და სხვა "განვითარებულ" ქვეყნებში მხოლოდ ასეთი ტამპონების გამოყენებაა ნებადართული, რომ საჭიროების შემთხვევაში მოხდეს ადვილი დიაგნოსტიკა და დიფერენცირება.

Posted by: LUKA-BRAZI 8 Jul 2009, 23:02
anarxisti
QUOTE
ტექსტილომაა

ვაახ, საინტერსეო შემთხვევა იყო, რაღაც გაგებით smile.gif კიდევ დადე smile.gif


Posted by: mika9 8 Jul 2009, 23:10
დევლინში ხატია კარგად მაგ სპექტრინები და არკირინები რა როგორ ზის და სად ...............ცჰეცკ იტ

Posted by: Blind_Torture_Kill 8 Jul 2009, 23:16
QUOTE
მოკლედ, ტექსტილომაა, უცხო სხეული ტექსტილის, ანუ პირველი ოპერაციის დროს დარჩენილი ტამპონი, რომელმაც გაიკეთა კაფსულა...შეეხორცა საშოს, მარჯვენა საკვერცხესა და სწორ ნაწლავს, რაც იწვევდა ტკივილებს და ცხადია ვერავითარი მკურნალობა, გარდა ქირურგიული მოშორებისა, ვერ განკურნავდა პაციენტს.


ამას ლაპარასკოპიულად როგორ გამოვიცნობდით ?

Posted by: anarxisti 8 Jul 2009, 23:31
QUOTE (LUKA-BRAZI @ 8 Jul 2009, 23:02 )
anarxisti
QUOTE
ტექსტილომაა

ვაახ, საინტერსეო შემთხვევა იყო, რაღაც გაგებით smile.gif კიდევ დადე smile.gif


კი "საინტერესოა", უფრო სწორედ ჭკუის საწავლებელი.. ახლა ეს პაციენტი უჩივის თავის ქირურგს, რომელმაც საკმარისი ყურადღება არ მიაქცია პაციენტის ჩივილებს და ლამის 1,5 წელი "უმკურნალა" , მაშინ როდესაც სასწრაფო ოპერაცია იყო საჭირო... ხდება ასეთებიც ევროპაში..

სხვა "საინტერესო" შემთხვევებსაც დავდებ, მაგის მეტი რა მაქვს gigi.gif
Blind_Torture_Kill
QUOTE
ამას ლაპარასკოპიულად როგორ გამოვიცნობდით ?


კი ძნელია, მაგრამ ჯერ ერთი მარჯვენა დანამატი არ სჩანს და ასევე აშკარაა რაღაც მოცულობითი პროცესის არსებობა ამ ადგილას
ამასთან, როგორც ნებისმიერი დიაგნოზის დროს, ყველა ნიშნის სინთეზია მთავარი..

ანამნეზი (გადატანილი ოპერაცია ანუ უცხო სხეულის არსებობა, შესაძლებელი),
რაც მთავარია რადიოლოგიური ნიშნები რენტგენოკონტრასტული ტამპონის.

Posted by: Blind_Torture_Kill 9 Jul 2009, 00:12
axalSobili vaJi woniT 3600g. daibada garTulebuli mSobiarobis Sedegad orsulobis 39-kviras. sigrZe – 51sm.
dabadebis pirveli wuTis Semdeg apgari – 7, me-5 wuTs – 8, 10 wuTis Semdeg ki 9.
fizikuri gamokvleviT anomaliebi ar aReniSneba.
pirvelive dRes bavSvma daiwyo dedis rZis miReba magram ar moxda mekoniumis gamoyofa. meore dRes ganuviTarda Rebineba. (amoRebul masaSi aRiniSneboda naRveli). kuWis moqmedebis gamosawvevad gaukeTda oyna. ganavalSi aRiniSneboda mosqelo mekoniumi.
mesame dRes igive ganmeorda.
rentgenogramam gamoavlina airis da siTxis doneebi mucelSi marcxena mxares. swor nawlavSi airi ar dafiqsirda.
SemdgomSi kuWis moqmedebis gamosawvevad saWiro gaxda swori nawlavis stimulacia an oynebi.
ori kviris Semdeg zogadi mdgomareoba damZimda. T- 39.6c, ganuviTarda xvela, sunTqvis ukmarisoba, daintubirda da gadayvanil iqna meqanikur ventilaciaze magram zogadi mdgomareobis simZimis gamo gadarCena ver moxerxda.



savaraudo diagnozi ?
აუ ბლიად ეს როგორ დაწერა sad.gif
აუტოფსიის სურათებიც გნებავთ ?

Posted by: LUKA-BRAZI 9 Jul 2009, 00:18
ახალშობილი ვაჟი წონით 3600გ. დაიბადა გართულებული მშობიარობის შედეგად ორსულობის 39-კვირას. სიგრძე – 51სმ.
დაბადების პირველი წუთის შემდეგ აპგარი – 7, მე-5 წუთს – 8, 10 წუთის შემდეგ კი 9.
ფიზიკური გამოკვლევით ანომალიები არ აღენიშნება.
პირველივე დღეს ბავშვმა დაიწყო დედის რძის მიღება მაგრამ არ მოხდა მეკონიუმის გამოყოფა. მეორე დღეს განუვითარდა ღებინება. (ამოღებულ მასაში აღინიშნებოდა ნაღველი). კუჭის მოქმედების გამოსაწვევად გაუკეთდა ოყნა. განავალში აღინიშნებოდა მოსქელო მეკონიუმი.
მესამე დღეს იგივე განმეორდა.
რენტგენოგრამამ გამოავლინა აირის და სითხის დონეები მუცელში მარცხენა მხარეს. სწორ ნაწლავში აირი არ დაფიქსირდა.
შემდგომში კუჭის მოქმედების გამოსაწვევად საჭირო გახდა სწორი ნაწლავის სტიმულაცია ან ოყნები.
ორი კვირის შემდეგ ზოგადი მდგომარეობა დამძიმდა. თ- 39.6ც, განუვითარდა ხველა, სუნთქვის უკმარისობა, დაინტუბირდა და გადაყვანილ იქნა მექანიკურ ვენტილაციაზე მაგრამ ზოგადი მდგომარეობის სიმძიმის გამო გადარჩენა ვერ მოხერხდა.



სავარაუდო დიაგნოზი ?
აუ ბლიად ეს როგორ დაწერა
აუტოფსიის სურათებიც გნებავთ ?

**************
ესე წავა? givi.gif

სავარაუდო დიაგნოზი არის 12-გოჯა ნაწლავის გაუვალობა. მაგას ახასიათებს ნაღვლოვანი მასებით ღებინება, გაუვალობა და ჰაერის 2 დონე რენტგენოგრამაზე..... IMHO smile.gif

Posted by: anarxisti 9 Jul 2009, 00:20
QUOTE (Blind_Torture_Kill @ 9 Jul 2009, 00:12 )
axalSobili vaJi woniT 3600g. daibada garTulebuli mSobiarobis Sedegad orsulobis 39-kviras. sigrZe – 51sm.
dabadebis pirveli wuTis Semdeg apgari – 7, me-5 wuTs – 8, 10 wuTis Semdeg ki 9.
fizikuri gamokvleviT anomaliebi ar aReniSneba.
pirvelive dRes bavSvma daiwyo dedis rZis miReba magram ar moxda mekoniumis gamoyofa. meore dRes ganuviTarda Rebineba. (amoRebul masaSi aRiniSneboda naRveli). kuWis moqmedebis gamosawvevad gaukeTda oyna. ganavalSi aRiniSneboda mosqelo mekoniumi.
mesame dRes igive ganmeorda.
rentgenogramam gamoavlina airis da siTxis doneebi mucelSi marcxena mxares. swor nawlavSi airi ar dafiqsirda.
SemdgomSi kuWis moqmedebis gamosawvevad saWiro gaxda swori nawlavis stimulacia an oynebi.
ori kviris Semdeg zogadi mdgomareoba damZimda. T- 39.6c, ganuviTarda xvela, sunTqvis ukmarisoba, daintubirda da gadayvanil iqna meqanikur ventilaciaze magram zogadi mdgomareobis simZimis gamo gadarCena ver moxerxda.



savaraudo diagnozi ?
აუ ბლიად ეს როგორ დაწერა sad.gif
აუტოფსიის სურათებიც გნებავთ ?


დაავადების სახელი "ჰ" თი იწყება

gigi.gif

Posted by: LUKA-BRAZI 9 Jul 2009, 01:07
anarxisti
QUOTE
დაავადების სახელი "ჰ" თი იწყება

ჰირშპრუნგს გულისხმობ? smile.gif ისე მართალია, ჰირშპრუნგი უფრო ჯდება ვიდრე 12 გოჯას გაუვალობა. დათო ალბათ ჰირშპრუნგია პასუხი. ვეთანხმები anarxisti-ს (ნუ რა თქმა უნდა თუ "ჰ"-ში ჰირშპრუნგს გულისხმობს biggrin.gif)

Posted by: Thandrus 9 Jul 2009, 01:59
მე მუკოვისციდოზი მგონია, ანუ Cystic Fibrosis. მასაც შეუძლია გაუვალობის გამოწვევა. მოსქელო განავალი და სხვა კლინიკური ნიშნებიც ამ დაავადებაზე მაფიქრებინებს.

Posted by: Blind_Torture_Kill 9 Jul 2009, 12:05
Thandrus
QUOTE
ე მუკოვისციდოზი მგონია, ანუ Cystic Fibrosis


ეგაა

Posted by: LUKA-BRAZI 10 Jul 2009, 11:25
Thandrus
Blind_Torture_Kill
და მაინც რამ მიგანიშნათ რომ ეგაა და არა ჰირშპრუნგი? ანუ დიფერენციული დიაგნოზი როგორ გაატარეთ..... smile.gif
* * *
აა, მოსქელო მეკონიუმი კუჭნაწლავის მხრივ და სუნთქვის უკმარისობა სასუნთქი სისტემის მხრივ ჰო? smile.gif

Posted by: Blind_Torture_Kill 10 Jul 2009, 19:29
QUOTE
აა, მოსქელო მეკონიუმი კუჭნაწლავის მხრივ და სუნთქვის უკმარისობა სასუნთქი სისტემის მხრივ ჰო?


ხოდა კიდე ფილტვის სურათები უნდა დამედო smile.gif

ჰა აბა შობეთ ახალი ქეისები

Posted by: LUKA-BRAZI 10 Jul 2009, 22:19
Blind_Torture_Kill
აუ მე მგონი ნევროზი, დეპრესია და ასპერგერი ერთდროულად მჭირს რა!!!!!!!!!!!!!!!!!!!!!!!!!!!!!!!

ჰა დავდებ ამას, ისე უბრალოდ, კლინიკური ქეისის დაწერის თავი არ მაქვს, თან შენზე უკეთეს ქეისებს ვერ ვდებ ხოლმე + ამას გამოცნობის პროცესი უფრო მიყვარს smile.gif

მოკლე ანამნეზი: კაცი, 44 წლის, 2 შვილის მამა. ცხოვრობს ჩვეულებრივად, სხვებისგან დიდად არ გამოირჩევა. მუშაობს (as a civil servant ანუ საჯარო მოხელე, ჰო?). ერთ დღესაც იგრძნო სისუსტე მარცხენა ფეხში და მიმართა ექიმს. გადაუღეს კტ ექიმებმა და "პირი დარჩათ ღია". მერე IQ შეუმოწმეს - 75 ქულა.

ეს შემთხვევა დაიბეჭდა The Lancet-ში.

ნუ რავი, დიაგნოზი დასვით მეთქი ვერ გეტყვით, იმიტომ რომ დიაგნოზი ბავშვობიდან დაესვა და ანამნეზშიც თვითონ აღნიშნა. ეს უფრო curious case არის. მოკლედ რატო ვდებ არ ვიცი, მარა ვდებ biggrin.gif მაინც თქვით აბა დიაგნოზი. მერე სტატიის ლინკს დავდებ და გადაიკითხეთ. smile.gif

user posted image

სურათის მარცხნივ ზემოთ და ქვემოთ ამ კაცის ტომოგრამაა, მარჯვნივ კი ნორმა (შედარებისთვის ანუ smile.gif)

Posted by: Thandrus 10 Jul 2009, 22:37
LUKA-BRAZI

ჰიდროცეფალიაა? ლატერალური პარკუჭებია უზომოდ გადიდებული. ჩემი აზრით, ერთადერთი შანსი ამ კაცს 44 წლამდე მიეღწია ისაა, რომ ეს ჰიდროცეფალუსი ქრონიკულად განვითარებოდა. ამ დროს თავის ტვინს შეუძლია რაღაცნაირად შეგუება (თუმცა ამ ზომამდე თუ შეეძლო, ვერ წარმოვიდგენდი).

Posted by: LUKA-BRAZI 10 Jul 2009, 23:31
Thandrus
QUOTE
ჰიდროცეფალიაა?

yes.gif
ამ კაცს ბავშვობაშივე დაესვა ეს დიაგნოზი და შუნტიც ქონდა ჩადგმული დეკომპრესიის მიზნით. 14 წლამდე ატარებდა ამ შუნტს, მერე ამოუღეს. ალბათ ამან უშველა. ესეც სტატია NewScientist-ში: http://www.newscientist.com/article/dn12301 ლანცეტში მგონი მემბერ სტატუსი გჭირდება სტატიის სრულად ნახვისთვის: http://www.thelancet.com/journals/lancet/article/PIIS0140-6736(07)61127-1/fulltext

smile.gif

Posted by: Blind_Torture_Kill 10 Jul 2009, 23:39
LUKA-BRAZI
ესეთი თავით სიცოცხლე no.gif
ესეთი კომპრესიული ატროფიის დროს აი ქიუ 75 კიდე კაი შედეგია

Posted by: LUKA-BRAZI 10 Jul 2009, 23:45
Blind_Torture_Kill
შენ ხო თსსუ-ში სწავლობდი? თუ კი, მაშინ ბელა მამულაშვილს ექნება მოყოლილი ადამიანზე, რომელსაც სასიცოცხლო ცენტრები (სუნთქვა, გულის მუშაობა....) ზურგის ტვინში ქონდა დუბლირებული smile.gif

Posted by: Blind_Torture_Kill 11 Jul 2009, 00:02
QUOTE
შენ ხო თსსუ-ში სწავლობდი? თუ კი, მაშინ ბელა მამულაშვილს ექნება მოყოლილი ადამიანზე, რომელსაც სასიცოცხლო ცენტრები (სუნთქვა, გულის მუშაობა....) ზურგის ტვინში ქონდა დუბლირებული


არა არ გამიგია ეგეთი

Posted by: LUKA-BRAZI 11 Jul 2009, 00:13
Blind_Torture_Kill
არ ესწრებოდი მაგის ლექციებს? ისეთ რაღაცეებს ყვებოდა, რომ.... biggrin.gif

Posted by: Blind_Torture_Kill 11 Jul 2009, 00:19
აბა თუ გამოიცნობთ ამ სურათზე რა არის

http://img53.imageshack.us/i/45229818.gif/

Posted by: Cousteau 11 Jul 2009, 00:34
უკაცრავად ეგრევე რო ვდებ (ანუ წინას რო არ აქვს პასუხი გაცემული)

user posted image

რა დაავადებაა?

აგერ მინიშნებაც:
user posted image

Posted by: mika9 11 Jul 2009, 10:39
Blind_Torture_Kill

inclusion body myositis an normaluri kuntebi smile.gif
* * *
Cousteau
სოკოა მაგრამ იმ გროსზე უფრო პრორიაზს გავს

Posted by: basa-ttt 11 Jul 2009, 11:29
ვიზუალურად :
კანის სოკოვანი დაავადება,
ფსორიაზი.
იქტიოზი..... (თუმცა მიკროსკოპიით არ ჰგავს)
ფსორიაზს ხშირად ახლავს სოკოვანი დაზიანებაც

მიკროსკოპია თქვენ უფრო იცით - და მიკრსკოპიით მართლაც ფსორიაზს წააგავს.

Posted by: Cousteau 11 Jul 2009, 12:04
QUOTE
სოკოა


რა დაავადებაა?
ამ დაავადებას რამოდენიმე სხვადასხვა სოკო იწვევს (ერთ ერთი ისაა რომელიც სურათზე დავდე)

user posted image

Posted by: basa-ttt 11 Jul 2009, 12:53
QUOTE
რა დაავადებაა?

ცენტრში მუქი შეფერვაა -
შავი ფერის შეგერვას ასპერგილოზი იძლევა
სხვადასხვა სოკო-ში არ ვიცი რას გულისხმობ.

ასე ერთი შეხედვით შეიძლება ლიქენიც კი იყოს

QUOTE
ერთ ერთი ისაა რომელიც სურათზე დავდე

უი - ეს არ მინახავს.

Posted by: Thandrus 11 Jul 2009, 14:43
Cousteau

ქრომობლასტომიკოზია მგონი. რაღაც ეგეთი მაქვს ნანახი სადღაც, ოდესღაც biggrin.gif

Posted by: Cousteau 11 Jul 2009, 14:48
QUOTE (Thandrus @ 11 Jul 2009, 14:43 )
Cousteau

ქრომობლასტომიკოზია მგონი. რაღაც ეგეთი მაქვს ნანახი სადღაც, ოდესღაც biggrin.gif

იეს, ქრო-მო-ბლა-სტო-მი-კოზია

მიკროზე Cladosporium

Posted by: Blind_Torture_Kill 13 Jul 2009, 17:47
mika9
QUOTE
inclusion body myositis an normaluri kuntebi
no.gif
კი კუნთია მარა საიდან არის პრეპარატი აღებული და რა პათოლოგია ჩანს ?


Cousteau
ესეთი არ მქონდა საერთოდ გაგებული


ისე ეს ქეისები ავკინძოთ და ერთ წიგნად გამოვუშვათ არ გინდათ ?

Posted by: LUKA-BRAZI 13 Jul 2009, 18:09
Blind_Torture_Kill
QUOTE
ისე ეს ქეისები ავკინძოთ და ერთ წიგნად გამოვუშვათ არ გინდათ ?

ვინ წაიკითხავს.... =\

Posted by: irakli222 13 Jul 2009, 18:27
QUOTE
ვინ წაიკითხავს.... =\


მე სიამოვნებით ვკითხულობ ამ თემას
(ყოველთვის ვნახულობ რამე ახალი ხომ არ არის)

ცალკე წიგნად დადებას აზრი ალბათ არ აქვს
მსურველი ისედაც ნახავს ინტერნეტში

Posted by: person 13 Jul 2009, 22:48
QUOTE
მე სიამოვნებით ვკითხულობ ამ თემას

+1

Posted by: E-36 14 Jul 2009, 00:10
QUOTE (person @ 13 Jul 2009, 22:48 )
QUOTE
მე სიამოვნებით ვკითხულობ ამ თემას

+1

+1 yes.gif
...............................................................

Posted by: Thandrus 14 Jul 2009, 03:10
Blind_Torture_Kill

QUOTE
ისე ეს ქეისები ავკინძოთ და ერთ წიგნად გამოვუშვათ არ გინდათ ?


საინტერესო იდეაა smile.gif შემოსული ფული ქველმოქმედებას მოვახმაროთ yes.gif
* * *
Blind_Torture_Kill

კარდიომიოპათია?

Posted by: Blind_Torture_Kill 14 Jul 2009, 10:17
Thandrus
QUOTE
კარდიომიოპათია?


გულია smile.gif მარა არა კარდიომიოპათია



Posted by: Thandrus 14 Jul 2009, 10:19
Blind_Torture_Kill

QUOTE
გულიამარა არა კარდიომიოპათია


biggrin.gif

მაშინ, დეგენერაციული / ინფილტრაციული ცვლილებები? (ისევ ამილოიდოზი ხომ არაა biggrin.gif)

თუ არადა, რას მივაქციოთ ყურადღება, ის მაინც გვითხარი smile.gif

Posted by: LUKA-BRAZI 16 Jul 2009, 20:07
100 გვერდი! ვინ გახსნის თემის მესამე ნაწილს? smile.gif Guardian მოი აქა! givi.gif თუ გარდიანი არ მოვა, მაშინ and the Oscar goes to.... biggrin.gif

Powered by Invision Power Board (http://www.invisionboard.com)
© Invision Power Services (http://www.invisionpower.com)